You are on page 1of 860

MATHEMATICAL METHODS

for Scientists and Engineers

Donald A. McQuarrie
UNIVERSITY OF CALIFORNIA, DAVIS

University Science Books


Sausalito, California
University Science Books
www.uscibook.s.com
Orde.r Informal.ion:
Phone(703)661-1572
Fa.x (703) 661- 150 I

Fin:t Printing

l1lu.,trator: Mt,rvin Hanson


De,igner: Rohen lshi
fl.fanuscript Editor: Jo/111 Murrh.ek
Compositor: Windfall Software. 11.1i11g Z:JtX
Printer & Binder: £d1rards BrothaJ, Inc.
Product.ion Mnnager. Christine• Taylor

This book is printed on acid-free paper.

Cop) right fl 2003 by University Science Book..;

Reproduction or lmnsl::u..ion or any part of this work beyond that pcrmincd by Sect.ion I 07
or 108 of 1hc 1976 United Stutes Copyright Acl without 1.he penni"i(ln of 1he copyright
owner is unlawful. Requests for permission or funher infonna1ion ,hould be add.re~scd to
Lhe Pennisi.ions Department. Universi1y Science Books.

ISBN I -891 389-24-6 ( c lolh cover J. Nonh A mL'Tican Edi I ion


ISBN 1-R9 I )89-29- 7 (soft CO\lcr). avai lnble oulsidc of North America only

Library of Congress Ca1aloging-in-Publication Data

McQuarric. Donald A. (Donnld All~n)


Mmhematical methods for scientists and engineers/ by Donald A. McQuarric.
p. cm.
Includes bibliographical references and index.
ISBN 1-891389-24-6 (cloth). ISBN 1-891389-29-7 (pbk .).
I. Ma1hema1ks. I. TILie.

QAJ7.3 .M36 2003


510---dc21 2003043448

Printed in the United S1n1es or America


10 9 8 7 6 5 4 3 2 I
Contents

Preface xi

CH PTER 1 Functions of a Single Variable


I l E1mctiaos
J ') I irniis 9
1.3 Continuity 15
1.4 Differentiation 19
1.5 Differenti als 2 7
16 Meao Yahre Ibi--wems 31
1.7 lnlegration 3 7
1.8 Improper Integrals 47
1.9 niform Converg n of Integrals 55
Re erences 6 I

(HAPTFR 2 Infinite Series 63


2.1 lnfinile Sequence 64
Co nvergen e and Di verg nee of Infinite Series 66
Tests f , c 70
6
83

101
2.9
Reieceoces

CHAPTER 3 Functions Defined As Integrals 1J 5


3 1 Ibe Gamma Eirocriao 11 5
32 The Bera Euoc1io □ I 22

al I 3

144
.7 rnoulli Polynomial 149

V
VI Cuntenl'i

9
4.1
4.2
4.3 Eul ompl x Number. 16
4.4 Tri s 176
4.5 The logarithm 81
4 .6 Powers of Complex Numbers 184
Refeceoces I AA

fHAPJFK 5 Vectors J 91
5 J Yecracs io Iwo Diroeosioos 19 1
5 J Yecroc Ftioctioos io Iwo Oiroeosioos J92
Vecrocs io Ibcee Diroeosioos 205
· io Three Diroeosiaos 2J 2
in Space 2.. 1

CHAPTER 6 Functions of Several Variables 23 ,


6I fuoc1iaos 232
6.2 Limits and Continuity 239
63 Partial Derivatives 244
6.4 Chain Rul e for Partial Differentjation 253
6 5 Diffcccorials ,wd 1hr Ialal Diffcrcotial ' 6D
6 6 The Diceciiaoa l DeciYalive and tbe Gradient 267
6.7 Taylor's Formula in Several Variables 273
68 Maxima aod Minima 229
6.9 The Method oi Lagrange Multipliers 285
6.1 0 Multiple Integrals 291
References 299

Cl:li'e:IEK z ~ctor Calculus 30 1


Zl Vec1or Eield ]OJ
7.2 Line lnt grals 11
7.3 Surface Integrals 322
7.4 33 1
Z5
Referenc s 347

rni'mH 8 Curvilinear Coordinates 349


8J Plaoe Palac Caacdioates 349
8 2 355
8.3
8.4
85 (t,cvilioeac Coocdloaies 328
8.6 Som r Coordinale Syst 87
References 395

C p r 1ht
Contents VII

CHAPTER 9 Linear Algebra and Vector Spaces 397


9 I Derecroioaors 392
9.2 Ga u ia n Elimination 409
9-3 Matrices 418
9.4 Rank of a Malrix 430
9.5 Vector Spaces 436
9.6 Inner Product Spaces 444
9.7 Complex Inner Prucllict Spa(,es 449
References 453

CHAPTER 10 Matrices and Eigenvalue Problems 455


10.1 Orthogonal and Unitary Transformations 455
10.2 Eigenvalues and Eigenvectors 462
10.3 Some Applied Eigenvalue Problems 47 I
10.4 Change of Ba is 481
10.5 DiagonalizaIion of Matrices 491
10.6 Quadratic Forms 499
References 51 J

CHAPTUl 11 Ordinary Differential Equations 515


11.1 Differential Equations of First Order and First Degree 516
11.2 linear first-Order Differential Equations 524
11.3 Homogeneous Linear Differential Equations with Constant Coefficicnls 530
11.4 Nonhomogen ou Lin e•. r Differential EquaIions wi1h Const.·rnt Coefficienls .541
11 ..5 Some Other Types of Higher-Order Differential Equations 556
11.6 Systems o( Linear Differential Equations 563
11.7 Two Jnvaluable Resources for Solutions 10 Differential Equations '170
Ref renc.es 5 7 2

CHAPTER 12 Series Solutions of Differential Equations 575


J 2 J Ibe Power Series Metbad 526
12.2 Ordinary Point nd Singular Points of Differential Equation 58 I
12.3 Series Solutions ear an Ordinary Point: Legend re's Equation 588
I 2.4 Solution N ar R gu lar Singular Point 593
12.5

Referen 623

CHAPTER 1J ualitative Methods for Nonlinear Differential E


I3 J Ibe Phase Plane 626
13 2 Critical Paints io tbe Phase Plane 634
13.3 Stability of Critical Points 642
13 4 Naolioeac Qscillarars 65 I
13.5 Population Dynamic 657
Reference._ 665
viii Conr nts

CHAPTER 14 Orthogonal Polynomials and Sturm-Liouville Problems 667


14. 1 Leg ndr Polynomials 668
t 4.2 Orthogona l Polynomial 678
t 4 .3 Sturm-Liouville Theory 687
14.4 Eigen(unction Expansions 696
14 .5 Green's Functions 702
Re(erenc s 710

CHAPTER 15 Fourier Series 713


15 . I ouri r S ri as Eigenfu nction E pan ions 714
15.2 Sine and Cosine Series 724
15.
I Equation 71 9
References 744

CHAPTER 16 Partial Differential Equations 747


16.1 Some Examples of Partial D ifferenliil I Equations 7 48
16.2 Laplace's EquaIion 755
16.3 The One--Dirnensional Wave Equation 768
16.4 The Two-Dimension I Wave Equation 779
16.5 Th H eat Equation 786
16. 6 The SchrBdinger Equation 796
A. A Particle in a Box 796
B. A Rigid Rotator 798
C. The Electron in a Hydrogen Atom 803
16.7 Th Cla ifiration of Parti. I Diff r ntial Equation 807
References 8 J 2

CHAPTER 17 Integral Transforms 81 5


17 .1 The Lap lac Transform 816
17 .2 The Inversion o( Laplace Transforms 824
·17.3 Laplac Tr, n forms and Ordinary Diff r nti al Equatio n 83 2
Lapla e Tra nsforms and Partial Diif ren ti al Equation 839
Fouri r Tran form 84 5
17.6 Fourier Transforms and Partial D ifferential Equations 856
17.7 Th Inv r ion Fom1ula for Lapl e Tran f rm 865
References 867

CHAPTER 18 Functions of a Com lex Variable: Theor 869


18.1
18.2 : The Cauchy-Ri emann Equat ion 875
18.3 ration : Cauchy's Theorem 862
18.4 Cauchy's Integral Formula 894
18. 90 1
ern 911
References 9 J 9

Id
Cont ~nts IX

CHAPTER 19 Functions of a Complex Variable: Applications 92 '1


922

Summat ion of S ric 938


L ation of Zero · 94 5
oniormal Mapping 954
onformal Mapping and Boundary Value Problems 970
Conformal Mapping and Flu id Flow 977
References 903

CHAPJEB 20 Calculus of Variations 9A'l


~0. 1
20.2 995
20 3 Yaciatiaoal Pcoblems witb Caostcainrs JQQJ
20.4 Variational Formulation of Eigenvalue Problems 1006
20.5 Multidimen ional Variational Problems 1015
Referenc 1021

CHAPTER 21 Probability Theory and Stochastic Processes 1023


I 1 Discrete Random Variables J024
2 1.2 Continuous Random Vari able 1037
71 3 Cbacac1ecistic f11nctians 1044
?J 4 105 2
21.5 S10 ha tic Pro e · 'amples I 063
A. Poi sson Process I 063
B. Th Shor Effect 1067
Refcceoces JQZ3

CHt\PIER 22 Mathematical Statistics I QZS


22 J Estirnatiao of PiJciJooelecs JQ7')
2. Thre K y DL!ribution. U d in tatisri _al li I I 085
A Ibe Nocmal Dislcibutioo mas
B. Chi-Squar Oislribution 1087
C. Student I-Distribution I 089
22.3 Confidence Interval!- 1091
A. Co11fiden ,e Intervals for the Mean o( a Nom1JI Dis1rrbution Whose Variance is Known 1093
B. C nfid •n t ~ I ntervil Is for lhe M an of a Normal Di stribution wilh U nkno wn V.:1ri,1nce l 095
Yaciaoce of a No · ihu1 ioo JQ9Z

Answers to Selected Problems 11 3


llluslralion Credits 1154
lodex 1155
Biographies and Historical Notes

Isaac Newton aod GauCcied I ieboitz xiv


Brook Taylor and Colin Maclaurin 62
I eaobacd E11lec I J 4
William Hamilton J 58
T

George Stoke and George re n 300


Carl Jacobi 348
Ca rl Friedrich Gauss 3%
Arthur Cayley and James Joseph Sylvester 454
Jacob Bernoulli S14
The Weierstrass Function 573
Adrien-Marie Legendre and Wilhelm Bessel 574
Henri Poincare 624
Charles-Franc;ois Sturm and Joseph Liouville 666
Transfin il Num 711
Jo

Why o Women Mathematicians? 8 13


Pi rr - imon lapla 814
Augustin Louis Ca u hy and Bernhard Riem;inn 868
Harry Nyqui st and Nikolai Joukowski 920
lohann Bernoulli 984
Simeon De□ i"' Pnj<t<;nn aod Andrei A Markov !022
Kaci Pearson aod Ronald A Eisbec J OZ4

ial
Preface

I have wrillen this book for s1ude11\$ who have had one and a half or lwo years
of calculus and liule else. The most important prerequisite is that students realize
lhe need to use mathematics in their studies or work. [ should say at the outset
tha1 this is not a mathematics book in 1he sense that I do not prove many t.heorems
and may have occa.--ional lapse~ of the degree of rigor that would satisfy a pure
111athema1ician. However, I have endeavored to present 1heorcms that scicn1is1s and
enginec~ rnight use in !Lhcir work in a manner 1hat is both accurate and intelligible.
(] don '1 know who 6rs1 said it. but Lhere is a saying 1ha1. "Pure mathematicians
don'L trust applied 1na1hematjcians. and applied mathematicians don't understand
pure matmcmaticians.") TheFc are entire books 1ha1 cover each of the topics that
we djscuss ,in a single chapter or even less, so we can give only an introduction 10
each. 11have tried to make my tlieatment of each topic self-contained. but I would
consider i,1 a great success ,j,f you became intcres1cd enough in any topic that you
sought oul further study by going 10 a more detailed treatment-not because my
treatmenl is opaque, but because you wan1 to know more. The references at the
end of each chapter should gel you started in this direction.
The first chapter is a review of calculus. which some readers may find easy or
superfluous. while others may find ii 10 be helpful. l1s purpose is to bring everyone
up to speed and 10 provide practice for those whose math is rusty. Although lhe
1rea1ment is elementary, I introduce the t-8 notation for the definition of limits and
continuity and then discuss I.he idea of uniform continuity and uniform conver-
gence of integrals. Even though your interest may not lie in mathematical rigor.
you should be aware of when interchanging Ii mi ting operations is pennissible. For
example. when can we write

dF=
-
dx
1 o
00
aJ(x.l)I
---Cl
ox

if F(x) = ft j(x, t) dt? Chapter 2, io which we discuss series. is aJso a review of


material that is treated in all calculus courses. The use and manipulaLion of series
play such an imponant role throughout applied malhematics thal it is imponant to
appreciate the concept of convergence and when cenain opcmtions such as lerm-
by-term differentiation and 1erm-by-1erm integration are valid. In ChapLcr 3. we
in1roduce a number of non-elementary functions. such as the gamma function.
the error function. and the Dirac dell.a func1ion. that are defined by certain inte-
gral expressions. Then. in Chapter 4. we discuss complex numbers, the complex
plane. and, very brielly. the propenies of functions of complex variables. We intro-
duce vectors in Chapter 5 and illustrate the power of vector noLation by applying xi

gh
XII Pr(•l,u ('

ii to a number of problems in analytic geometry that arc fairly easy using vec-
1or no1a1ion but would be 1edious without it. Functions of more than one variable
arc discussed in Chapter 6. This material leads inio Chapter 7. where we discuss
vector caJculus, which is indispensable lhroughout 1he sciences and engineering.
After discussing various coordinate sys1ems in Chap1er 8. we go on to linear a.Jge-
bra and vcc1or spaces in Chapter 9 and then mairiccs and eigenvalue problems in
Chapter 10. The ncx1 four chapters cons1irute a scgmen1 on differential equations,
including nonlinear differential equal ions and pha,c :--pace in Chapter 13 and spe-
cial functions and Sturm-Liouville Lheory in Chapter 14. The next rwo cha.piers
treat Fourier .s~rie~ and Lheir application to ~olving panial differential equations
by 1he method of separation of variables. We continue our study of panial differ-
emiaJ equations in Chapter 17. where we disi:uss integral transfonns. particularly
L1placc transforms and Fourier 1ransforms. The need to invcn Laplace transforms
leads naturally lo functions of complex variables and integration in the complex
plane. which we discuss in Chapter 18. Complex variable theory is one of lhc most
profound and beautiful subjecL.:; in applied mathematics. and aJI scien~ and engi-
neering studcnls should have some familiarily with this subject~ even if their work
doesn't oflcn require it. In Chapter 19. we show how complex variable theory can
be uSed 10 evaluate real integrals. to sum series into closed forms. to solve boundary
value problems. and to solve fluid-flow problems. The final 1wo chapters disc'll,'iS
probability theory and mathematical s1atis1ics. In particulaF. we discu ·s confidence
intervals. goodness-of-fit tests, and regmssion and correlat..ion in 1hc last chapter.
No ant: can learn this material (or anylhing else· in the sci,en.ces or i1n engi-
neering. for that maner) without doing lots of problems. For 1h,is rca soA,. I have
provided at leas! 15 10 20 problems al the end of each section. These problems
sometimes M:rve 10 fill in gops or lo extend the material presented in 1hc section,
but 1hcy arc most often used to illuslrnte applicutions of the ma1crial. In all. there
arc almost '.\(X}() problems in I.he book. and I have pro\•ided answers to many of
them at 1he back of the book.
A number of powerful commercial compuler packages are available nowa-
days that am be used to solve many of the problems in this book. These programs
not only provide numerical answers, but they can also perform algebraic manipu-
lations. and for that reason they arc called computer algebra systems (CAS). Some
of the prominent CAS are Matlab, Maple. Mathematica. and MathCad. I happen
10 know and use Ma1hema1.ica, and I have presented examples of one-line Mathe-
matica commands throughout the book 1hat can be used Lo solve given problems.
These commands are jus1 meant 10 provide examples of the u1ili1y of any CAS.
and there are a number of problems that ask you to ··use any CAS to solve ... "
These programs are so available and user-friendly 1ha1 you might wonder al times
"Why do I need 10 learn all the slUff in this book when I could use a CAS to solve
my problem?" I 1hink that anyone with experience would agree when I say thal
these programs arc a wonderfully useful supplement to 1hc material in this book,
bu1 arc no substjtute for it. During the writing of this book, I found count..lcs.s ex-
amples where a thoughtless use of a CAS would lead you astray. Funhermore.
many problems are such that you need lo apply mathematical knowledge to get
XIII

I.hem into a fonn that the CAS can handle. In spite of 1he friendliness of CAS. as
in most things, you have to know whal you're doing first in order to use them with
confidence.
A singular feature of the book is the inclusion of biogn1phies al the bc§rinn.ing
of e.ach ch.:iptcr. Many of the mathematician thru we refer LO werre rai heE colorful
characters, and I personally find it enjoyable reading about them. I wish 10 thank my
publisher for encouraging me to include them and my wife. Carole. for researching
the material and for wriLing every one of them . Each one could ea.~ily have been
several pages long. and it was difficult to cut them down 10 one page. We both
wish to acknowledge a terri lie website at the University of St. Andrews in Scotland
(www-his1ory.mcs.l:-t-and.ac.uk. and yes. the www- is correct) that lists hundreds
of biographies of famous ma1hema1ician$, as well as other mathematical subjects.
You read in many prefaces that "this book could nor have been wriuen and
produced wi1houL Lhe help of many people." and it is definitely true . I um particu-
larly grateful 10 my reviewers. Dennis DeTurck of the University of Pennsylvania.
Scon Feller of Wnba,;;h College. David Wunsch of the University of Mac;sachuselts
al Lowell. Mervin Hanson of Humboldt State University, and Heal.her Cox of the
California Institute oiTechnology, who s·logged through first drafts of all the chap-
ters and who made many great suggestions. I am also grateful 10 my son, Allan,
of the Johns Hopkins University Applied Physics Laboratory. who contributed a
great deal to Chapters 2 I and 22. I also wish to thank Christine Taylor and her staff
at Wilsted & Taylor Publishing Services for coordinating the ent.irc projecl, espe-
cially Caroline Roberts and Melody L1cina for correcting all my spelling errors. I
thank as well Bob lshi for designing his usual beautiful-looking and inviting book.
Jane Ellis for dealing with many of lhc production dct.ails and for procuring aJI
the photographs and likenesses for the biographies. Mervin Hanson for rendering
over 700 figures in Ma1hemalica and keeping Lhem all $lraight in spire of countless
alterat·ions, John Murdzek for very helpful copyediting. Paul Anagnostopoulos for
composing 1hc entire book. and my publisher Bnice Armbrus1er and his wife and
associate, Kathy, for being the best publishers around and good friends in addition.
FinaJly, I wish 10 thank my wife, Carole. for preparing the manuscript in TEX, for
reading many of the chapters, and for being my best critic, in general.
There are bound 10 be both typographical and conceptual errors in a book of
this breadth and length, and I would appreciate your letling me know about them so
that I.hey Ci.ln be corrected in subsequent printings. I also would welcome general
comments, questions, and suggestions either at mquarric@mcn.org or through
the University Science Books website. www.uscibooks .com. where any ancillary
material or notices will be posted.

Donald A. McQuarrie
CHAPTER 1
Functions of a Single Variable

This fir.:l chapter is a review of a number of topics from your beginning calculus
course. It assumes lhat you haven·t forgouen how 10 differentiate and that you
have access to a rab le of integrals st:1ch as the CRC Stondard Mathemn1ical Tables
1

and Fonnu/as. Jn add iLion. ~bcre ore a number of computer programs such as
Ma1hcmatica. Ma1lab, Maple. and MalhCad thai have sophis1icated numerical and
symbolic capabilities. We'll refer to these programs as compwer algebra !>Y,~tems
(C AS) and illustrate I.heir use a number of I imes throughout the book. Any student
of applied malhemalics should become comfonable wil.h one of lhese programs,
Some mathcmatica 1 methods book-:s slali'I off with more advafilced topics, t s-
sum ing I.hat you already know or are al leas1 familiar with the material in this
chapter. Some of you will find this chapter 10 be fairly easy. wh.ile others will have
to work through some of the problems to get back up to speed. Every one of you
can become quite proficient :11 mathematics al the level pre~ented 1hroughou1 the
book. however. by doing lots of problems and I.hereby gaining experience and con-
fidence. The chapter starts wilh a definition of the idea of a func1ion of a single
variable and then goes on to disc.:u~s ltmiting processes and limits. We will intro-
duce the €---<l notation, a conci.,;e and precise notation that is worth lhe effort to
learn. The logical topic after studying limits is thar of continuity, which is defined
through a limiting process. Then we go on Lo define derivatives of functions of a
~ingle variable and lhen we take up integrals. again defined hy u limit·ing process.
We spend some amount of time on what are called improper integrals. integrals
whose limits are infinite or whose integrands are unbounded (blow up) somewhere
in the range of iniegration. Thi: last sectjon deaJs wlt.h the notion of the uniform
convergence of integrals. which may be new to many of you. h's in this sec1ion
thai we'll learn about Lhe propenics of a function F(x) if it is de-lined by

·;)0

F(x) =
1.
f(:c. t)dt

Notice lhat we are intq;rating overt. so that the resulting integral is a function of x.
We might ask under whal conditions is F(x) a continuous function of x: or when
can we di ffrrcntiate with re.spec1 1.0 x under the integral sign Lo write t.he derivative
of F(x) as 1

gh al
2 Chapter I / F1111ttions oi a Single Variable

F'(x) = rxi df(x. r) dt


lo ax
Most of the examples and problems in this chap1er are what you might call
"just math problems". The material in this chapter and 1hc next fow chapters is
background for later chapters. where many physically moriva1ed problems arc
discussed.

1.1 Functions

Recall from calculus that a function is a rule tha1 r~lat~s one number. x, 10
another, y. We express this relationship by writing y = f (x). where f represents
lhe func1ion. The set of values of x for which f (x) is defined is called the domain
of the function and the set of all values of y produced from a)l the x is called
1hc rongt of J. If only one value of y is produced from each vaJue of x, then the
function is sa.id to be single-valued. Tf more lhan one value of y is produced from a
value of x, then J is said 10 be. m11ltiple-val11ed. We will show latenhat a multiple-
valued function can be viewed as a collect.ion of single-valued functions, called
branches, and so we will assume that all our functjons are single-valued. Some
authors even require a function to be single-valued, bur we'll adopt Lhc somewhat
more liberal definition given above.
Let's look at some examples. Consider Lhe relaLion y = x::, or y = f(x) = x 2 ,
for values of x given by -2 ~ x .:::: 2. In 1his case, f is single-valued because each
value of x leads to only one value of y. Note 1hat the domain of J is the intervaJ
( -2. 21 and the range of J is JO. 4]. The notation [ -2. 21 corresponds 10 lhe closed
inren,ol. -2 ~ x :5 2. We call the interval closed because it includes its endpoints.
-2 and 2. The no1a1ion for rbe corresponding open interval. -2 < x < 2. is
(-2. 2). If the domain of f(x) = x 2 had been the open interval (-2, 2). thea
the range of f would have been [ 0. 4) . Now consider y 2 ::: .x, where O < x ~ I.
(We denote the inrervaJ for x by (0.1 l) Solving for y. we obtain y =
±.Jx,
showing that !here arc two values of y for each value of x. We can view this
relationship as corresponding to two single-valued functions, y = J 1(x) = Jx and
=
y = h(x) -Ji. Note lhat J1 and h have the same domains. bul completely
different ranges. (0.11 and [- t, 0).
St.riclly speaking. a funcLion is denoted by .f and the vaJue obtained when J
is applied to x is denoted by y = f (x). However, it is common practice 10 call
j(x) a function. and a ''function of x". in panicular. We even write y == y(x) to
indicate 1ha1 1he value y resuhs when the rule for sending x into y is applied to x.
This notation is very common and very convenient. rn any case, :r is called the
i11depe11den1 variable and y is called 1he depe'1de111 variable.
There are two broad cla~!-.eS of functjons. algebraicfimctions and transce11de,i-
ralflmc1io11s. An algebraic function. y(x). is a solution to Lhe polynomial equation
1.1 Func1ions 3

where Lhe p j (x )s arc polynomials in x. If y(x) can be expressed as lhe rntio of two
polynomiaJs, 1hen it is a ra1io11al algebraicj,mction; otherwise ii is an irrmional
algebraic function. For example. y = (x 2 + 2)/(x - I) is a rational aJgebraic
=
funclion and y (x 2 + 2)/ Jx-=-1 is an irrational algebraic function.
Functions that are no1 algebraic f unciions are called 1ranscenden1al Fu net.ions.
Exponential func1ions. logarithmic functions. trigonome1ric func1ions (Figures 1.1
through 1.3), and hyperbolic functions (Figures 1.4 through I .6) are examples of
transcendental functions. Recall that the hyperbolic functions are
e-r - e-x
sinh.x = ----
2
2 2
cschx = ----
e·t - e-.r
sechx = ----
e-r + e-:r
(I)

~ - e ,% r +e-.x
1anhx = ----
e-r + e-.x
cothx= - - --
;::r - e-.r

X X

Figure 1.1 Figure 1.2 Figure 1.3


The t.rigonomenic fu11..:1iom, ,in .r and The trigonoml't.ric fon1..·1i11ll', co, .r and The trigonometric function., Ian x and
csc x = I/ sin x (color) plol led .,~ai nsl x. sc.c .r = I/ cos .r ( color) plot 1cd again ,;.t x. col x = I/ um .r (color) ploHed n~ain.sL
The a.-.ymplOtc.<; of c,c x ilIC ~hown a..,;; the The asymp101es of ~ .x arc !:hown :.i-.. the asymp101e.s of tan .x arc ~how n a.~
,r. The:
colored da.,;hcd line.~ and 1hc _, axis. colorcd du.shed line .. the dashed lines fil x = -,r /2 and ;r /2.
The asympll!ll"' nf cot .t are shown as 1he
colored da~hcd lines and lh<:- y ax.is.

Figure 1.4 Figure 1.S Figure T.6


The hyperbolic functions !-inh x 11.nd The hyperbolic functions cos_h x and The hypi:rbolic func1io11s 1anh :c and
c~ch .r = I/ sinh .r (color) ploued sceh x = I/ cosh x (color) ploued e<>Lh x =
1/ Hinh .r (color) ploucd
again.s1 .r . agains1 .r . ag·ain.-.1 .r.
4 Ch,1plL•r 1 / flm • tio n~ f a Single Vilriable

The exponenLiaJ func1ion ex and lhe logarithmic function In x bear a speciaJ


relation to efich other. If y = y(x) = ~-r.
then x = x (y) =
In y. The rwo functions.
y(x) = ex and .x(y) = In y are inverses of each other; the function er sends x into
y = ex and the funcLion x = In y sends y back into x. Using y = e-t and x = In y,
we sec that elny = y and thal x = In ex. The graphs of _v = e-r and x = In y are
shown in Figure 1.7. N01e !hat one graph can be obtained from 1heo1her by simply
interchanging the x and y axes. or what amounts to the same thing. by flipping
X
either curve about lhe line _v = x.
Nore that 1hc correspondence berwcen x and y in Figure 1.7 is one-to-one,
in 1ha1 each value or x corresponds 10 exactly one value or y and each value of y
corresponds to exactly one value of x. We express such a correspondence by

Figure 1.7
The cxponcnli.al ,..s. and 1hc logari1hmic
1

func1ion In x (color) plottetl againsr JC


Note that 1hc 1wo func1ion arc symme1ric where 1- 1 is the inversefu11c1irm of J. For example, if y = f(x) = 2x - 3, 1hen
about rhc line y = x. x = J- 1(y) = (3 + y )/2. [Figure 1.8 shows the rela1ion between y = .f(x) and
x = J- 1(.x) .l The function y = .f(x) = x 2 doe~ nol have a un.ique inverse because
y x = ± .jy. The inverse funclion in this case is double-valued. and its two branches
are .jy and -.jy. We can choose 1.0 work wilh either one of these branches so
long as we keep the restric1ion x ~ 0 or x .:5 0 in mind. Problem 2 has you show
=
lhat Lhe graphs of y(x) x 2 and y(x} = Ji arc symmetric about the line y = x .
A class of impo11ant inverse functions are the inverse 1rigonome1ric functions.
Consider x = sin y. We assign un angle _v in radians and calculate a unique
value of x. But as Figure I. 9 shows, si.n y is periodic in _v with a period 2rr;
in other words. sin y repeals itself every 2Jr units along the y axis. or in an
X
equation. sin y = sin(y + 2;r11) where n = 0. ± I. ±2 •.... Consequently. the
inverse function . which we write as y = sin - I x. is hardly unique, in lhe sense
lhat many values of y (actually. an infinite number in this case) correspond 10 !he
same value or x. as shown in Figure I. I0. For example, if x = v'2/2. then y = ;r / 4,
Figure 1.8
rr /4 + 211. rr /4 + 4rr. or generally rr /4 + 27T11 . where n = 0, ± I. ±2 ... .. Thus.
The relation between rhe function
y = 2x - 3 lllld its inverse .r =
(3 + y)/2 1he inverse function y === sin- 1 x has an infinite number of branches. one for each
[plotted as O + x)/2)1. Noh! that the two value of 11. As Figure I. I O shows. the value of y will be detcrmi ncd uniquely
functions are symmemic abou1 1he line
from the value of x if we res1rict y 10 1he values -rr /2 .::S y .::S rr /2. Thus. we can
=
y .r. ju.-,1 m, in Figure J .7.
write the inverse of x = sin y a" y = sin- 1 x with 1he restriction -rr /2 5 y .::: rr /2.
1
We caJI 1his function I.he principal branch of sin- x and we caJI the value of y the
1
pri11cipal value of sin- x (Figure 1.11 ). The other inverse trigonomelric functions
X
have similar res1ric1ions, and their principal branches are

y:::::.sin- 1 .r -rr/2 ~ \.' > rr/2


1
y
y =coic x 0 .::s y .::s ;r (2)
v=lan- 1 x -n/2 ~ y ~ JT/2

(See Figures 1.12 and 1.13.) We shall see in Section 4 tJrnl the apparemly arbitrary
Figure 1.9 domains for 1he inverse trigonometric functions in Equation 2 result in simple
The function x =
i;in y is a periodic
fonnulas for 1heir derivatives..
function of y with a period 21t .
1.1 Fu11( riori~ 5

y y

t _\'
....
J

-- .\'

--Ti
-- - I X
2

Figure l .11 Figure 1.12


X
The pnm:ipal branch of _1 = ,in - 1 .x . The pri nc:ipul brunch of -'" - l'.11,
1
x.

.,
~21r - - -

-- X

Figure 1.10
The functinn sin- 1 .x G. o muhipk -vulm·d
function of x. Note that ~in x and ~in - 1 x
can be ob1 :i ined from fin :lno1hcr by Figure 1.13
inten:han.,ing , he x and . ax t·s . The The principal branch of y a: 1a.n- 1 .r.
principal branch i. rhe solid line-.

Example 1:
Show Lhlt

if J; < I.

SOLUTION: Let a= sin- 1 x. so thut .t" = sin a . Figure 1.14 illusrratcs


X
that sin a ...., .\/I. Then

X
tan a= ~
v l -x 2
Figure 1.14
and so ll1e rig_hr rrfangk used in Example I.
et= sin ·
1
x = Ian-• ----==x==
I- .\- 2

rn
6 Chapter I / Func1ions oi a Single Variable

Another common notation for 1he inverse trigonometric func1ions is arcsin(x),


arccos(x), and arctan(x). If they are restricted to their principal values, then they
are often denoted by .A.resin (x), Arccos (x). and Arctan (x).
Using the definitions of the hyperbolic functions given by Equa1ions I. we can
=
derive explicit expressions for the inverses. Consider y sinh x = (c - e-x)/2.
If we lei:::= e-X. then we have y = (z: - l/z)/2. We can rearrange this expression
=
into z2 - 2yz - I = 0. and solving for z tr gives

Taking the logarithm gives

We reject the negative sign above because the argument of the logarithm must be
positive. lnlerchanging y and x for conformity gives us

y = sinh- 1
x= In (x + J;2+i) all x

Example 2:
Derive an explici1 cxprc,~ion for cosh- 1 x.

SOLUTION: Let ;: =
e-' in Equation I for COl\h x and rearnmgc 10 obtain
z 2 - 2y;:+I= 0. Solving for z e.x gives=

Choosing the+ sign as the principal value and 1aking logarithms gives

Interchanging y and x, we obLain

Note that the restriction x ~ I in the expression for cosh- 1 x is related 10 1he
fact that cosh u ~ I for any \'alue of u. (See Figure 1, 15 .)
X

Expressions for other inverse hyperbolic functions can be found in many mathe-
Figure 1.15 matical Lables. such a~ the CRC Standard Mathema.rical Tables.
The func1lons sinh x (color) and rnsh .r Figure 1.15 shows graphs of sinh x .md cosh x. Note Lha1 cash x is symmeuic
ploued against x. Note that cosh x i,,
symmec.ric and that sinh x is antisymmetric
about the y axis and that sinh x changes sign when it is reflected through lhe
about they llXis. y axis. Analytjcally, these propert.ies are expressed by cosh(-x) = cosh(x) and

al
1. 1 Fune rions 7

sinh(-x) =-
sinh(x). Generally. a func1ion wi1h the propeny that J(-x) f(x) =
is called an ever1 fi111e1ion of x :llld one with the property that f (-x) = - f (x) is
caJled an nddf1111c1io11 oh, Nor all functions are even or odd. but any function can
be wrillen as a sum of an even function and an odd function by writ.ing

We shall see later tha1 a r~cognition of 1he pari1y of a function can be very useful
when integrating.

Example 3:
Prove that sinh x is an odd function of .r .

SOLUTION:

e·r - t' - r
- - - =- sinhx
2

1 .1 Problems

1. Dctcnnine the maximum domains of the real-valued functions


I
(a) _v=/16-x 2 (b) r=--
• x2 6

(c) y = lnx (d) y = - -


x -1
2. Show 1ha1 the graphs of y(.r) =x 2 and _v = .Jx are symmetric about the line _r =x.
3. Plot the function., (a) y = l.rl, (b) y::: -Ix!. and (c) y = I - l.r l for -3.::: .r ~ 3.

4. The Hcaviside s1ep function. H (x ). is defined by H (x) = I O x ' O. Plot Lhe funcLion y(x)
I x > 0
=
H(x) - H(x - I) .

5. Plot the func1ion defined by f(x) =2- 6(x - l)H(x - I)+ 4(x + 4)/-l(x - 3). where H(x) is del'ined in
Problem 4.

6. Plot rhe function defined by

f(r) =r - 2(1 - 2)H(I - 2) + 2(t - 4)H(r - 4) - 2(r - 6)H(t - 6) + ·· ·


7. Plot y(x) = x /1.rl for -to< x < 10.

8. Plo1y(.r)=x-lxlfor-lO< x < 10.


8 Chap1cr 1 / Fun lion . of a in le Variable

9. Consider Lhe points (-c. 0) and (c. 0). Derive an equa1ion for 1he set of all points (.x. y) such that the sum of
I.be distances from (x, y) to ( -c, 0) and 10 (c. 0) is il constant = 2n. Do you recognize the equal.ion? What is
t1 called?
10. How would 1he resulting equation in Problem 9 change if Ihe cenler were changed 10 (2. -l) instead of al
(0. O)?
11. Consider a venical line Lal x = -p and the point (p. 0). Now derive an equation for 1he set of all points
equi&,tanl from 1hc line L anc.J the point (p. 0). Do you recognize this curve?
12. Derive the relations ,in- 1 x + cos- 1 x = rr /2 and 1an- 1 .r + co1- 1 x = 1r /2. Him: U~ Figure 1.16 and
a + f3 = rr /2 for the tirsl part.

Figure 1.16
The gei)meny for Problem I'.L

13. Show that cot- 1 x = tan- 1( 1/x) = rr /2 - cot- 1( 1/x). Him: Use Figure 1.17 and a+ fJ = rr /2 for the first
part.

Figurel.17
X
Tht· geometry for Problem 13.

14. Showthattanh- 1 x= ~
2
In (
1
I-
+x)X

I5. Which of 1hc following funclion!- i!- periodic? Whal are their periods?

(b) I cos xl (c)


X

16. Classify each of the following functions as even, odd, or neither.

(a) lanh x (b) 1:r sin x (c) (d) cos x + sin x


17. Show that the equation tanh x =e- 2.x is L-quivalent to 2r = sinh-l I.
18. Prove the triangle inequaliry. la+ bi ~ la! lbl. Him: Add the two inequalities -lol ~a~ fol and
-lbl.::: b.:'.:: ;bi.
19. De!erminc J(/(x)) if f(x) = x 2 + I.
1.2 Limits 9

20. Find g(J(y)) if g(x) = 1/(1 + .x) and j(y) = \Jy 2.


21. Show tha1 y = f(x) = (x + 1)/(x - I) is its own inverse; in other words. show that x = (y + 1)/(_y - I).

22. Find the values of x for which

(a) .r 2 +2 < 3.x (b) -2 < 3x + I < 2


x-1
(X;;

23. Plot the func1ion f(x) = L H(x - ,,).


n -0

00

24. Plol the function f(.r) = L( 1) 11 H(x - 11).


n=◊

1.2 Limits

(n this section. we are going to discuss lhc idea of a limiL. but first we need 10
define a neighborhood. The set of all poin1s x such 1ha1 a - 8 < x < a + ,5 where
o> 0 is called a 8-11eighborlwod of the poinl a. The interval a - i5 < x < a + 8
can be written as Ix - a I < 8 (Problem l ). If the point a is excluded, then we
have the set of all poincs x such 1hat O < Ix - al < 8. which is called a deleted
8-11eiRhborlwod of a. Now lei J(x) be defined and single-valued for all x in a
dele1cd 8-neighborhood of a. We say 1ha1 f(x) has 1hc limit/ a.s x approuchcs a
if for any positive number f. however small. we can find some positive number 8
such that

I J(x) - / I < E if O < I .r - a I < li

and write

,(
lim.,, .{(x)=I or f (x)-+ I as x ~ a (I)

In other words. we can make f(x) as close a.,; we wish to I so long as we choose
x sufficiently close to a. but not equol 10 a itself. ln fac1, f (x) need not even be
defined at a. The value of 8 may depend upon the value of€. so we will oflen
write 8(t:).
Let's use 1.his E-8 notation to show rhat lim (3x + 2) = 5. Having chosen
.r----1
an E, however small. we must find a 8 such that 1(3x + 2) - 51 < E whenever
0 < Ix - II < 8. First note 1ha1 1(3x + 2) - 51 = 31x - 11. Now if we take 8 to be
E/3. 1hen 1(3x + 2) - 51 = 31.t - II< E ifO < Ix - II < lL
10 Chapter 1 / Functions o/ a Single Variable

Example 1:
Use rhe €-8 notation to show that Jim x sin( 1/x) = 0.
x-o
SOLUTION: The Iimit I is equal to 0, and so we wish to show that

. I I
X
I
Sin; - 01< (
if O < jx - 01 < o. The value of I sin( l/x)I .'.: .: I for all x =;= 0, and s.o

where the Ja.-.t inc::qualiry follows from O < Ix - OJ < S. Now if we just let
8 =€,we get

Figure T.18 and so lim x sin( I/ x) =0 a.,;; was to be proved (see Figure 1.18). Nott! that
=
The function f (x) x sin( 1/.r) plo11cd f (.t)
.f ()
= x sin( 1/.r) is not even defined at x = 0.
against x for small vnlues of x.

It is easy to prove that limits have the following prope11ics:

Jim laf(x)
:r:-o
+ ,Bg(x)I = a lim j(x)
r-u
+ f, :r:-a
lim J?(X)

f (x) lim /(_-.;) (2)


Iim - - == -·'_._a_ _
:r:-u g(x) lim g(x)
x - •u
♦ .f
(a> 0)
I .\-,. (}

sr ------ where a. {J. and rare any real numbers.


Although the €~ no1a1ion provides a fom,al definition of a limit. we shall
4
.
usually determine 1·1m1L-. Jx+T6
. more d'1rectly. Le·c ' s Ioo k· at 1·1m - - - -- - . 1711s
·
X r-•O X
limit is of 1hc form 0/0. so we certainly cannot let x = 0. Figure 1.19 shows
figure 1.19
j(x) = (Jx""+7'6- 4)/x plotted against x, and it seems to go to a finite limit
The func1ion f(x) = (.._!x+T6 - 4 )/x
ploncd agains.t .t for small values of .r. ( 1/8) a.,;; X ~ 0.
To see that this is indeed so, muhiply and divide by Jx""+7'6 + 4 to obtain

. ,Jx + 16 - 4 Jx+T6 + 4
hm - -- - - · - - - - -
. . .....,.0 X Jx"+T6+4
I
.
= ltm ------
X .
= hm ----- =I
.r-•O x( ~ + 4) .r-0 Jx""+7'6 + 4 8

where we realize that x + 16- 16 as x - 0.


1.2 Limih 11

f
Example 2:

•r O x
1
lnvcsligalc lim .!_ ( -- -
x+3 3
!) . X

SOLUTION: The runction /(x) is plotted in Figure 1.20.


-
91
-'-~
1
= = lim ~
lim
x ox
_!_ ( - - - ~)
x +3 3
lim
.r-0 3x(x
-x
+ 3) .r-•O 3(.
- l
+ 3)
=-
9 I
Figure 1.20
as Figure I .20 ~uggests. =
The function f(x) ll/(x + 3)- 1/3]/x
ploued agains1 x for small value. of x.

We ofrcn wish to find the limit of a function /(x) as x oo. What we mean ~
in 1his case is 1har lht!re exisrs a number N such tha1 IJ(x) - /I < € whenever
x > N. Clearly, funcrions like 1/x. l/x1, ere. have lhc lirni1 0 as x - oo. How
aboul something like

2
+ 6x - 1
f(x) = 2r
-.,----
3.r~ - 2t +7
We rewrite J (x) as

and use lhe facr that 1/x and l/x 2 go to zero to obtain lim f(x)
,r
= 2/3. A more
demanding example is given in 1he following Example.

Example 3:
Show that lim (x - ~ )
~--:,o
= 0. where a is a constanl.
SOLUTION: Muhiply :ind divide by x + Jx 2 + a to gel

lim - a = 0
.c- x + Jx~ +a

Example 4:
Find .J Jim • x(.r - / x 1 - a). where" is a conslanL
12 Chapter 1 / Func1i1u,~ of ~1 Si111:l1· Variable

f SOLUTION: Multiply and divide by x + \, r 2 - a as in the previous


Example to get
4

2
lim
A-oo x + Jx2
ax
- a
= Jim
' ~'Xl
x +x
R
ax
1. - ,
x~
= lim nx
x-.oc 2r
=~
2

2 X Figure 1.21 shows the graph of x (.r - Jx 2 - 4) plotted agai nsu·. Notice
that it approaches 2 as x gets large.
Figure 1.21
The function f(x) =.r(.f - Jx 1 - 4)
plo11ed :ig:-iin,a .\'. The graph (color) begins
Ul lhc point (2. 4) and the asymptote is A limil lha1 we shall use fairly of1en is
shown as a gray dashed line.
. sin x
I1m - -
.r-,0 X
=I (3)

This limit has a nice geometric proof. which is developed in Problem 2. Notice
once again that sin x / x is not defi ncd al x = 0. nor need it be. You may remember
1hat limits of the type 0/0 (like 1he one above). 00/00. and some others like these
arc called inderenninate Jom1.1· and can be readily evaluated by rHopital's rule.
We shall derive l'H6pital's rule in Section 6.
In many applications we arc interested in the limit ofa function al some point a
when x approaches a from one side only. If f (x) h.is Lhc limit/ when x approaches
a through positive values of x - a (in other words. from the right). we write

lim /(x) = /, (4)


.c-tJ+

We will often use an equivalent no1a1ion. which reads

H lim j(x)
.c-ri+
= .,-o·
lim ((a+€) (5)
(0, I)+---- ---
where E > 0. This limit is called the right-hand limit of j(x) al n. and is sometimes
denoted by f (a+). Of course. we can have lcfl~hand limits also. in which case

lim J(n - 1:) = /_ = f(a-) (6)


.\" E-0

A good example of a function tha1 has different right-hand and lcfl-hand limits
Figure 1.22 is the Heavi.side step function (Figure 1.22). defined by
l11e Hcavi, ilk- st~p fumaion. H (x) = 0
when x - 0 and f/(x) = I when x > 0.
H(x)= { ~ X
X>O
< 0
(7)

In !his case, Ii m H ( -E)


t!:-,0
= 0 and t-•O
Ii m H (E) = l. As you might have guessed
already. a function will have different righl-hand and left-hand limits at a point
where it is discontinuous.
The ncxl Example illustrates different right-hand anJ left-hand limit~ and
infinite limits. If lf(x)I > N. where N is a positive number. however large, as
€ ~ 0. 1hen we say I.hat 1/(x)I --+ oo as€--+ 0.
1.2 Li111i1~ 13

Example 5: 'I -r
Investigate the behavior of j(x) = (x + 2)/(x - I) near I.he point x = I.
SOLUTION: It is clear that lim ((x) does nor exist. But let ·s look at the
.r-1·
two one-sided limi1:;. X

. x +2
I1m - - = 1.1m I-
----
E + 2 = -oo
,-r-x I ,--0l-t:-1
. x +2 \' I +( + 2 Figure 1.23
I1m - - = 1m - - - - = oo The function f(x) = (x + 2)/(x - I)
I i~(\ 1 + E - I
.r- I+ X -
plotted agai11s1 x near che point x = I.
Figure 1.23 :-hows j(.r). The a~~ rnptotc of /(x) i~ indicated by the
-.,crtical da._,hcd Ii nc.

You should be aware thal a number or the commercially available computer


programs. such as Mathematica. Maple. Mat.lab. and MathCad. can evaluate limit'-.
For example. the one-line command in Mathematica

Limit [ ( ( 3+x r2 -9 ) / x, X➔ 0 ]

gives 6. These programs not only carry out numerical calculations bur can per-
fonn algebraic manipulation~ as well. Consequently. they are often referred to as
Computer Algebr.i Syiaems (CAS). We shall refer to them collectively as CAS. To
encourage you 10 learn how to use one of Lhese CAS. Problems 15 through 20 ask
you 10 use any one of them to evaluate some limits.

1.2 Problems
t. Show 1ha1 a - ,5 < x < a + 8 can be wriucn as Ix - o I < 8.
. sin x
.i..
2 . Use FiIgure I. 24 to prove mat 11m - - :=: I.
.r--0 X

T
sin x

Figure 1.24
Geometry :is$ociated with lhc proof thul unit
lim (sin x)/x
,-o
= I. circle
14 .h a pll!r I / func tioM~ or a ingle V,ai"iabl •

1- c.o (
3. Use the result of Problem 2 10 show 1ha1 ' - 0 us x ...... 0. Notice that this ratio is of the limiting form
X
0/0.

4. Find 1he following limits:


. sin 3x sin x
(a) 11 m - - (b) lim .,
.r-•O X .t-0 X 1/-
. .,
. sm· x . lan lr
(c) lim , (d) 11 m - -
.r-•fl x- x-•O Jx
S. Find the following limits:
sin 2x (b) I + cos ;rx
(a) - - as x - O - - - - as x -,.
sin x tan 2 ,r.r
x2 - 9 I - cos x
( c) - - as x - -3 ( d) .. as x - 0
X +J X"'
6. Find the limit of 1J (x + h) - f(x) 1/ II ash ➔ 0 for I.he following tu net.ions:

(a) J (x) (b) J(..--, = 1/x


(c) J (x) = sin x (d) f (x) = cos x
7. Find the limits of 1he following functions a ' x-,.

(a) Jx+a - Jx + b (b) ( Jx+I - Jx)j x + ~

8 . F .md lh e 1·mllt
. of -/x+2 - .fi. a.-. x ---+ 0•
X

9. Dctem1ine the one- ided limits of j(x) = x/ lxl .


. I
10 . Evaluate lim
.r-o+ I + e- 1/ .

11. Given 1hat 11 11 = 12 + u,,. 1 has a limi1. find its value.


12. Suppose 1ha1 j(x ) _:;; g(x) ;S li(x) for all values of x in some dektl·d neighbohoud of a. If lim /(xJ
r~u
=
lim h(x) = L, prove Lha1 lim g(x) = L. Some author,; call 1his resull lhe sq1u-1'::_1· law.
x--u a

13. Prove Lhar lim j(x)g(x) = [ lim f(x>] [ lim g(x>] .


.r---a x-•LJ .r-•a

14. The function y = 1/( I - x)2 is unbounded as x -> I. Calculnre the value of t5 such 1/1a1 _r > I0 6 if Ix - 11 < 8,

The n<!xl 6 problems involve using any CAS ro C'Va/110/e limils. Use a11_v CAS to fmd rhe followi11g limits:

x- 2 2x 2 + xe-.r
15. lim ( ·-
x ---I ) l6. lim , 17. lim
.t-•I Jx - I .I _. 2 .\' - - 1 Y +2 .i:-oo 3.r 2 - 2 r + I

18. lim
, 2 -·4
. ., 19. Jim
X - ... 20. lirn
<x;\ - 1)1.rl
x -.2 · ( x - 2)~ X 2- (x - 2)- .r-0 ' X
1.J Conlinuily 15

1.3 Continuity

A function f (x) is continuous al a if the following three conditions hold:

1. J(x) is defined at a
2. lim J(x) exis1s
x-u
3. lim f(x)
x-~u
=f (a}

We can summarize these 1hree conditions by wri1ing

lim f (x)
.t-•IJ
=f ( lim
_f-, (I
x) = /(a)
If /(x) does not $atisfy these conditions. then /(x) is said to be disconlinuous
at a. For example. the function 1/x is discon1jnuous al x = 0 because j(x) is not
defined al .r::;:: 0. nor does l.im f(x) exist The function /(x)
.r->O. .
= (x 2 - 1)/(.,· - I) is
discontinuous al x = I because J (I) is not deli ncd. but using the fac1 thal x 2 - I =
(x + l)(x - I). we sec !hat lim f(x)
x-. J .
= x-•
lim (x + l) = 2. so tJ1at condition 2 holds.
I
In this case we say that x =
I is a removable discontinuity.
A more fom1al definition of con1inui1y is that J (x) is continuous al x = o
if ii is possible to find a 8. which may depend upon both E and a, such that
lf(x) - f(o)] < E. however small. if Ix - al < 8. Using this 8-€ definition. ii is
f
easy 10 .,;how using Equal.ion 2.2 Lha1 if f(x) and g(.r) are conlinuous au= a. so
are f(x) + g(x). f (.x)g(x). and f(x)/g(x) pro\'idcd g(a) t= 0.
ln1ui1ively, a discontinui1y is a jump in the graph of the function. For example.
the Heaviside step func1ion (Figure 1.22) has a jump di.';con1inuiry al .r 0. and =
1he func1ion defined by
-I X

- I ::: x ::: I bul x f= 0


figure 1.25
x=O =
The di1>C.ontlnuou~ f"unc1ion f (x ) x 2 + I
=
for - I ::. ., ::.. I hut .r -#; 0 ::ind f U:) 0
is discontinuous al x = 0 (Figure 1.25). for ,1 .-:- 0 plotted ag.ain.,t .r.
Another type of discontinuity is displayed by the runction 1/(1 - x) 2 al x =I
(Figure 1.26). We say that 1/( I - .r) 2 has an infinite discontinuity at .x I. = •J
Just as we have right-hand and lef1-hand limi1s, we have continuity from Lhe
right and conunui1y from the lefL For example. we say that J(x) is continuous
from 1he right at a if lim, J(x) = f(a), or more simply. if /(a+)= j(a). A
,r-,.a,
function is continuous at x = a if it is continuous both from the righl and from 1he
left at x = a.
X
We say 1ha1 a function is continuous in an interval if i1 is continuous at all
poin1s in the imerval. If the interval is <.:loscd. then ir /(x) is continuous. it must Figure 1.26
be conlinuous at the end poinrs o and b; in other words. it must be continuous from T11c function /1.r) = 1/ ( l - x)~ plotted
the left at b und continuous from the righl al a . ug.iinsl x ne11r lhe point x - I.
16 hapt r 1 / Fun lions oi .1 ingl Variabl

Example 1:
Is f(.x) = lxl - x continuous for -oo < x < oo'!

SOLUTION: /(x) = 2x when x:::; 0 and f(:c) = 0 whl!n :c ~ 0. Thus


f
J (x) is rnntinuou.s over the entire x axis (Figure 1.27).

-I X hample 2:
Is /(x) = 1/x conIinuous in 1hc. inten•al l 0. I ]? In (0. I \'?
Figure 1.27
The function /(x) = lxl - x ploue,d SOLUTION: Because x =0 is included in the interval [ 0. I]. /(x) is
;1gain,1 x. nor continuous in that interval because f (x) i~ not dcti.ned ai .t 0. The =
onl y possib le 1roublesome regi on in the half-open intcr.•al is near x 0. To =
investi gate 1hi~ region. let r > 0 be as smaJI as you wish. TI1en. we need lo
find a 8 = ~ (€. r ) such that

Solving rnis inequaJity for,5 gives 8 <er- / ( l -€r)~er 2• so /(x) is


continuous on the half-open interval (0, 1 ]. Nore tha1 iS depend.,; upon both
F a.nd r in this ca.'-e.
f
2
There arc a number of properties of conti.nuous functions 1ha1 we use fre-
quenlly (and intuitively). One propeny i~ given by 1he extreme value theorem:

If f (.r) i.~ cor1ri11uous in rh,, clmcd i,,ren·al I a, b L 1l,e11 ir ha." o maximwrt


2 X v(l/ue ond a mi11inwm vo/11e in 1ht· imerval.

Figure 1.28 Nole 1hat the inlcrval must be closed for the cx1reme value theorem 10 apply. For
=
The function / .r ) x 3 - 3:r 2 + 4 definerl example. consider f(x) = x 3 - 3x 2 + 4 defined on the half-open [nterval [ I. 2)
on the half-open interval ( I. 2).
(Figure 1.28). This function attains a maximum value at x = I, but attains no
minimum value since x = 2, the only possibility for yielding a minimum value
f of j(x) according 10 Figure 1.28, is nol included in the interval. No matter how
=
close we get 10 x 2. there are smaller values of f(x) as ,r geL~ closer to 2. The

l 2 X
necessity of the conti.nuity of Lhc function can be seen by considering the function
(Figure 1.29)

I< x .:S 2

x=I
figure 1.29
The function /(.r) =
1/(.r - I) when This function is not continuous on the closed i nterva1 I I, 2 ] because the Ii m f (x)
I < x S 1 und /(x) = 0 when x = I .l •I!
pinned ag:iin~I .x. docs nor exist. It attains minimum values at x = I and at x = 2. bul alluins no
1.3 Continuity 17

maximum value. No maner how closely we approach x = I from the right. lherc
arc larger values of j(x) as x get$ closer 10 I.
Another useful property of continuous functions is g-[ven by the i11ten1tedia1e
value theorem:

~( .f (x) is continuous in the closed i1,rer1,al I a. b I. then f (x) m.rnmes


t!l'ery intermediate value between f (a) and f (b). So ,f 1/ is o 11w11her rncl,
that f (a) ~ '1 ::":: f (b ). then there is al least one poim c iTL (a. b I such tlwr
f(c) = ,,.
A consequence of this theorem says that if f (a) and J(b) have opposite signs,
1.hen f (x) wi II be zero for at least one value of x in I a. b I.
To see that the function in lhe intermediate value theorem mu.st be continuou_.,,
note lhal the Heaviside srep function attains TIO intermediate values between y = 0
and y = I (Figure 1.22).

Ex.ample 3:
Show thal /(x) = x'.! + x - I has at least one zero I a point at which /(x) = OJ
in the interval [ 0. I J.
X

SOLUTION: f(x) is con1inuous in [ 0. I J and /( I) = I and _f (0) = -1.


Therefore. j(x) rnusl cross the x axis DI least once in I 0. l I (Figure 1.30). -I

Figure 1.30
The function f (x) = x 2 _ - I ploucd
aguinst x in rhc in1erval I 0, I ].
The Heaviside s1ep funclion is ;in example of a fyncti on lhar, is not continuous
over lhe whole interval, {- . oo). but is corui,nuous over the two subinlcrvals
(-oc. 0) and (0. oc). Suppose that a funcLion f (x) is nor continuous over an entire
interval [a. b J. bu1 the imerval can be divided inlo a finite number of subintervals
where f (x) is con Ii nuous over each subi nlcrval and finite al the end poi n1s of each
subinterval. Then f (x) is said 10 be sectionally comi11uous or pieall'ise crmtimwus
over [a. b ]. The Heaviside step func1ion is piecewise continuous over the x axis. •J
Another example of a piecewise continuous function is
-a a
< -a
X
X
-a< x < a
x>a

which is the potential used for a quantum-mechanical particle in a finite well


(figure 1.31 ). Figure 1.31
The. func1ion Jrx) = 0 for x < - a.
We shall discuss one last brief topic before leaving this )\eel.ion. Go back
f {X) = - Yo for - a x < a. and
10 Example 2 and notice 1ha1 the value of £ depended upon bo1h t: and r. Lei f (x)= Oforx o.
f (x) be conlinuous in an interval. If it is possible ro find a value of 8 such lha1
IJ(x) - j(a)I < E whenever Ix - al< 8 for each point of the interval and where
8 is independent of a. then f (x) is said to be 1111ijormly con1i1111ous in 1he interval.
18 Ch.1pt<•r I / Functions or ,1 Single Variable

Example 2 shows that j(x) = 1/x is not unifom1ly continuous in the half-open
interval (0,J ). (It is nor even continuous in 1he closed interval [ 0. I I.) Clearly if a
func1ion is unifonnJy continuous, then ii is con1inuous.

1.3 Problems
I. Use Lhe €-S nolnlion lo prove 1hnl j(x) = x 2 is continuous at x = 2.
2. ls J (.x) = (x 4 + x 3 -
+ 2r - 1)/(.l" - I) con1inuous al x = I?
3.r

3. Prove thar the equation 2x 5 + 2x + 1=0 has al least one solution between I and -1.
4. Prove thar cos :c = x has a solution between O and ,r /2.

S. Graph 1he function J (x) = Ix - 11/(x - I). Find

(a) lim /(x) (b) lim j(.rl (c) lim f(x)


.,-1- .r-1+ .~-1

6. At what points is .f (x) = x csc x discontinuous?


7. Prove tha1 f(x) = x 2 is uniformly continuous in (0.1).
8. Show 1ha1 sin .x and emu· am conrinuous functions for all values of x . Hill/: Use the relation sin ex · ,in ft
a+ fJ _.
= .,- CO'S -2
-
a - f',
)-1.0 -
2
-.

9. Discuss the difference in behaviorof 1hc two func1ions f(x) = 1/(2 -x) and g(x) =
1/(2 -x) 2 at I.heir point,;
of discontinuity.
10. The function f (x) = (xJ - l)/(x 2 - I) is no1 defined al x = I. Is 1hc point x = I a removable discontinuity?
Whal vulue mus1 j(x) be assigned al x = 1 in order 10 make it con1inuou.,; there?
11. Show thal the equation .r3-r = 2 ha.<; at Jeasl one solution in the intervaJ O < x < I.

-J;)'
12. Find a and /j such that f(x) given below is con11nuous for O < x < 2;r.

sin .x 0< .r < rr /2

13. Show tha1 if g(x) is conrinous at x


J(x)=
I a sin x
(, -
+ f3 TT /2

Jrr/2 <
< X < 3JT /2

X < 2JT

= a and if .f is continuous at ,:(n). then f(~(x)) i$ continuous al x = a.


14. Prove tha1 if f(x) and g(x) arc continuous at x =a.then so is f(x)g(xj. Hinr : Use Equation 2.2.
15. Prove 1hat if f (x) and g(x) are continuous at x = a. then so is f (x)/ g(x). provided g(a) ~ 0. Hil/1: Use
Equation 2.2.

qh al
1.4 Oilh·r1·111i,11i,111 19

1.4 Differentiation

Recall lha1 the derivative of a function j(x) at a point a is rhe slope of the straight
line tangent 10 f(x) at a. The derivative of /(x) at ::i. point a is defined by 1he
limiLing process

'( . .f(a + h)-.f(a)


{ {I)= 11111 (Il
. J,-0 J,

Anoihcr commonly u~cd no1.i1ion is

dr . 6r . y(a + 6.r) - y(a)


.d:<
. :. . = .'ulim - · =
O 6.r
11m
r o x
(2)
X

Figure 1.32 illustrates Lhis limit.ing process. In 1he beginning of your calculus Figure 1.32
course. you learned how to differentiate a number of function~ using the above An illw.tratiM of the limiting prol· s in
rhc dclini.Lion of rhe d'e rivul.iH· of y(x ).
formula.,. For example. if y(.r) = ..r:! + 2. then As 6 _r - 0. l_d.\ · x) y x)IJ r
llpproach · rhc 1ongcnt line DI the point
(.I' •.1•)_

= lim (2x
·-•O
-r- L\..r) = 2x
A liule more challenging is J(x) = cos x:
d cos .r . cos(x + 6x) - cos x
- - - = hm
d.r ~-•-·n 6x
. cos x cos 6.r - sin x sin 6.r - cos .r
= hm
il._,-.. o 6.x

or

d cos .r
--- = ).1rn cosx
(co,6.r-1) . .
- - - - - SIil X Jim
sin6x
dx .r-\l {l_x X O 6X

Problem 2.3 .shows that tcos 6x - I)/ 6x -, 0 as 6x - 0. and so

d rns x .. . sin 6 .\'


- - - =- SIil X 11111 - - -
dx ~.r--o ox
But we saw in the previou~ section that this limit is equal to one. so we have

d cos X .
- - - = - smx
d.x

Typically. in a calculus course. you us.c Equa1ion I or 2 lo differentiate a


\'aricry of funcrions and 1hen use these results and some general rule, such as
(11 v)' = "v' + 11 v and (11 /v )' = (tJ11
1 1
- 11 p')/v:i 10 differen1ia1c just about any1hing.
Ma1hcma1ical rnbles have several pages of derivative formulas and most of1he CAS
c::in differcn1ia1c symbolically.
20 Chap! ' I ' Fu n Lions. of a ingle Vsi ri.1ble

Example 1:
Differentiate y = f (x) = x 2e-.r cos x.

SOLUTION: Use _v' = (u11w) = 11 11w + 11v'u; + 11t•w':


1 1

= Xe-xf 2 COS X - X(C'OS X + sin X))

II is often convenient 10 think of differeniaLion as an operation on a func-


1ion J(x). An operator is a symbol thal tells you to do somel.hing 10 whale\ler
function follows the symbol. For example . we can consider df /dx to be 1he d /dx
operator acting upon f (x). If we denote the differen1iation opcra1or by b = d jdx.
then we c.an write

, df
Df(x) =-
dx

You le.am in calculus that if c 1 and c2 are cons1an1s. 1hen

(3)

An operator with this property is suid robe a linear operator. Diffcren1iation is a


linear orcration.
Jusl as we define left-hand and right-hand limits. we can define left-hand and
right-hand derivatives by

f'(a) = lim J(a + h) - f(a)


1i -~ o l;

J'(h) = lim f (b + /r) - /(b)


h--o - h

We can say lhat a function has a deriva1ive al some poin1 .r c if its right-hand =
and left-hand derivativeg are equal at x = c. A function that has a derivative at all
points in an intervaJ is said to be differenriablc in rhar interval. If lhe interval is
dosed. [ a. b J. Lben nol only must J'(x) exist for all x in Lhe open interval (a. b).
bur the end point deriva1ives must exist also.

Example 2:
Suppose lhat / (:r l = (sinh x)/x in thi: interval (0. I ) and that .f(x) = (IX +b
in the interval [ I. 2 ). Find the rwu ~onstanrs a and h such that f (x) and
f'(x) arc continuous at .r = I. Plo1 1he resulting funclion.
1.4 O,ii,·11 ·ntia1ion 21

SOLUTION: Continuity of j(.x) at x = I gives sinh I= u +band


conrinuity of f'(x) at x = l gives cosh I - .s_i11h I , a. Solving for a and b
gives a == cosh I - sinh I = 1/ e and h = 2 inh I - cosh I = (e 2 - J)/2e .
The rc.:sulting function

y
O<x::::l
X
J(x) = { X e2 - 3
-i' + -2,.-
is plolled in Figure 1.33.

2 X

You also learn 10 diffcrcnt.iale composite func~ions in a cakulus da~s. A Figure 1.33
composite function is a function of a function. If y = J (11) and u = g(x). then The fonclion dclermined in Ex.ample 2
plotted :i~ainsl x .
y = J(g(x)) is a compm.ile func1iun of x. Recall that lhe derivative of y with
respect to x is given by the chain rule.

dy dy du
-=--
dx d11 dx

For example, if y(x) ==sin(.\'.!+ 2), then we have y =f (11) = sin II and 11 = x 2 + 2.
so

-dy = (cos 11)2.r = 2.r cos(x - + 2) 1

dx

Example 3:
1
Find dy/dx if y(x) :....: t. - (x ui>• ~ \~here o is a con,1an1.

SOLUTION: We use the chain rule wi1h y = £'_,. and = (.r 2 + a 2) I/~.
11

We don't have to stop al tirs1 derivatives, do we? We can cake sequential


dcriva1ivcs 10 fonn second derivatives . third derivatives, and so on. For example.
if y(x) = x=\,.t. then
22 Ch,1p!!•r 1 / Funt lions of J Si11::IL' V.:iri.1hl...-

where we have used the opera1or notation for a derivative by writjng D.i: for
• l • • ,
d /dx. Note thnt 1hc notation D; means Dx applied seque111iolly, so that D;f (x) ==
Dx[ D_,(J (x)) J.
Frequently a func1ion y(x) is defined implicirlr through a rela1ion such as
J(x. y) = 0, and ii may not be possible or conn:nil:nr to solve expliciLly for y(x).
For L'.\ ample. the implicit func1ion of y and x might be

1 I
f(x. y) = -
X3
- ---:;
y~
+ xy - I= 0

We can differentiate this equation with respect 10 x using the chain rule to obtain

and solve for dy /dx:

dv 3- _\A)' y3
-=------
dx x 4 x yJ + 2

Usually the result will contain bothy and x explicitly. bu1 lhis is no problem. U we
wanted to know.the derivative al x = I. we would sub~1i1u1c x into /(x. y) = 0 =\
to find y (in this case y =
I) and I.hen evaluate d ) /dx al the point ( I. I ) to get
dy/dx = 2/3.

Example 4:
Find the value of dy/dx at the point (0. I) if y(,\) = e-'· sin xy.
SOLUTION; Differenliating implicitly ~l\'l!S

y' = y' <' ·, liin .I")' ·•- .ry' e·'· cos. x y + ye-'· cos x_,.
At (0. I). y' = t: .

We said in Secrion I that the apparenrly arbit:raJ)' choices for the domains of
I.he inverse trigonometric functions were made so tha1 their derivatives have simple
formulas. Let's sec why this is ~o. Consider _v ,in- 1 x. where -Tf/2 s y rr/2 = :s
(figure I. 11 ). To find lhc derivative of y = sin- 1 .r, we wri1e ~in y = x and then.
differentiate implici1ly wilh respect 10 x 10 obtain y' cosy = I. or

I I
\' - - - ·- ±---;:::==== - ± - - -
- -- cosy - Ji -sin2 J - J1-x2

Now cosy ::::: 0 when -:;r /2 S y · 11 /2. and so we use the positive sign for y' over
the en1ire r-..inge of x and write
I ..:I Diff r nriation 23
d . -I
-~m x=--- IX I
dx

Example 5:
Show that

-"
dx
t'(i)S - 1 .\' = - ----===
I - xl
1.1· I< I

SOLUTION: We ,I art wilh y = cos- 1 x. where O::; y ~ 1T (Figure 1.12).


Oifforen1ia1ing x =1.:ns y impl1ci1ly with rcspt."<.:11n x give~ -y' sin y = I. or

·
\' ' = - -- I = ±----===
sin y I _ :c 2

Now sin y ~ 0 when O::: y i rr and so we use ihe n1;ga1ivc sign for y' over
the entire range o , .r and wliilt'
I
d - 1
- CO'- X = I .r I < I
dx

.r

One of the principal applicn1.ion" of derivative~ is to find lhc local maximum


and minimum values (extrema) of a function in an interval. Point" al which f (x) =
0 arc called cri1iml points of j(x). Although the condi1ion f'(c) 0 is used 10 = Figure 1.34
locale cx1rcm:1. it docs no1 guarnnree that f (x) have a loc.11 cx1remum there. The The func1ion Jtx) =x plo11.:d again~\ x.
simplest illw-arnrion of this is I (.r) = .r>: =
/'(0) 0. bul J (x) is not an exrremum
al x = 0 (sec Figure 1.34 ). y
Let .f(x) be con1inuous on the open interval (o . h) and let f'(x) exisl and be
continuous in (a. }J) . If J'(.r) > 0 in (a. c) and J'(.r) < 0 in (c. h) . then f(x) is
concnvc downward al c . A function _f (x) is concave downward at .r = c if the g raph
of J (.r) in rhc ncighborhood of c lies below 1hc tangent line at x = c (Figure ~,35a).
On the other hand, if f'(x) < 0 in (a. c ) and j'(.,· ) > 0 in (c. I,) _ then f(x) is
concave upward al c. A function f(.r) is concave upward at x = c if the graph of (a) X
/(x) in the ncighborhood of c lies above the tangent \line al .r c (Figure 1.35b). =
The type of concavi1y is rclul('. c.l 10 1hc sign of the- :sc•cond d(•ri vativc. and !-O we y
have the second derivati,·c test l(I cJeLem1ine if a critico:1! point is a local exrrcmum
or nor: If J'(c) = 0 and f' 'td exists, then

1. if J''(c) < 0. then .f(.r) has a local maximum at :r =c


=c
2. if f"(c) > 0. then
J. ir f (c) = 0. 1hen
11
/(.t) has a local minimum at
110
x
condusion <.:an be drJwn wi1hout further analysis
--------
(b) --
x

A point .r = c is called an i11Jlenio11 point if j(x) is concave upward on one Figure 1.35
( a) 11:ie !'.mph of ;1 cf•n~~,v~· d11w11\,~.1rd
side of c and concave downward on tht: other ~ide. Consequently. f"(x) = 0 ar func t,ion. (b) T he graph of a concave
an infkcrion poinr. It is not necessary thal J ' (.r) = 0 at an innectiun point. For up wa rd /unction.
24

example, the graph of f(x) = +


(x - 1) 3 2(x - I)+ I has an inflec1ion poinl at
x = I If"( I)= OJ even though f'( I)= 2 there (figure 1.36). lncidenlly, i1 seems
1ha1 mnny students 1hink 1ha1 1..be condition J"(c) = 0 implies an inflection point
al x = c. but this is no1 so. All 1hree func1ions . .f(x) = xJ. g(x) = x\ li(x) = -.rJ
have tiN and second derivatives 1hat are equal to zero at x = 0. yet f (x) has a
X
relative minimum. g(x) has an inflection poinL and h(x) has a relative maximum
= =
a1 x 0. (See Figure 1.37.) If j"(x) 0 al a critical point. you must investigate
hig,hcr order dcriva1ives 10 de1ermine i1s narure (see Scc1ion 6.8).
figure 1.36 As ano1her tw b 110 consider, lei's look a1 f (x) = .Jt2.13 defined on the interval
The funclfon /(x)
1
= (-1. I J. In this case. _f'(x} =
2/(3x vn). which diverge. to oo as x ~ 0 through
(x - 1} + 2(.r - I) I plo11cd
positive valut·s and 10 - oo ;.is x-. 0 through negative values. Thus. f'(x) is nol
ngui1r.,1.r.
defined at x = 0. yet f (x) ha~ a minimum value there (see Figure 1.38). Th.is
illLLstrates the fact that J'(x) mus1 exis1 and be continuous throughout 1hc interval
in order to use the above criteria.
We have assumed 1ha1 f (x) is defined over an open interval. If f (x) is defined
over o closed interval. you mus1 also check Lhc end point.s of lhe interval. Consider
X the function /(x) = 2x 3x'2 - l 2x - 5defined overlhc closed interval I -3. 3 J.
It has two critical points, one at x = I wi lh f'' (I) = ,18 > 0 andi one at x = -2 with
j"'(-2) = -18 < 0. Thus. there is a local mirumum at .r = J [wi,Lh f( \)1= - I 2J
(a)
I

and a local maximum al x =


-2 [with /(-2) =
151. Although we find a local
y maximum at x = -2, it is not an ohsolllfe max.imum because j(:r:) = 40 al its
endpoint x == 3 (see Figure 1.39). The message here is Lhat if /(x) i.s defined over
a closed interval. then you mus1 cxc1mi11c the bchavior of /(.r;) no1 only at i1s cri1ica\
poin1s. but al its end points as well.
X

(b)

y J f

(c) -I I X
X

Figure 1.37 Figure 1.38 Figure 1 .39


The functions (al, j(.r) = .r..1. (b) The funcrion / •.x) - • 21 defined on lhc TI1c func1ion / (x) 2 • = ' - 12.r - 5
g(.rl = .r1. and (c:) /i(x) =
-x 4 plotted cl~d inten·al 1-1. I I plotted ::ig.;iins-t x. defined on the clo d interval [-3. ) I
ogninst x. ploue.d ag::iin 1 .1· .
IA Difil'rl'nli.ilion 25

Example 6:
Find the local exrrema and 1he inflection poin1s of /(x) = x.!(I - x).! over
the entire .r axi.s.

SOLUTION: The cquatiOt\

/'(x) = 2x( I - x) 2 - 2l" 2 (1 - .T) = 2r(l - x)( I - 2.r) =0

shows that 1here arc critical poinr.s at .r = 0. x = 1/2. and x = I. The second
dcriva.1.ivc is

J"(x) = l2x 2 - 12.x +2 f

The fact that f''(O) > 0 and j"'(I) > 0 and 1ha1 J"(l/2) < U tells us that
the critical points x = 0 1md x = I are local minima and that x = 1/2 is
a local maximum. The inflection points are given by /" (x) = 0. or at
x = (3 ± ./3)/6 (f,igurc 1.40).

-0.5 0.5 1.0 X

Figure 1.40
Note from Figure 1.40 Lhat j(x) = x 2( I - x) 2 is symmelric about the vertical line TI1e function f(:q "" x" t I .l J~ ploncd
=
at .x = 1/2. To see that this i~ so analytically, lei~ x - 1/2, ~o 1ha1 the function a~;iin,;1 x.
now reads j(t.) = (f ~) ! - ~)
2
(
2
, which is an even func1ion in(.

1.4 Problems
l. Differentiate
sin x
(b) (c) x 2 lan 2x
r
2. Differentiate

(b) Jx'- - 3x + I; (c) rr'

3. Differentiate

(b) ln(scc x + tan xj; (cl x"in .r

4. The t.angen1 line to a curve ul some point (a. b) has the slope 111 = (dy/d.r>x=-u = J'(aJ. Show that rhe slope
of the 1inc perpendicular lo rhc curve at (fl. b) i.s equal IO - I/ m.

5. Docs f (x) = lxl have a derivative at x = O'?


6. Prove that J(x) = x 3 is difforcn1iable in the dosed interval ( 0, I J.
7. The grJph corresponding to 2.r 2 - 2ry + .\ 2 = 4 is an ellipse whmc major axis makes an angle wirh respect
to the x arxJs. Plot the func:tir1n. Show thai thi: slopes oft.he Lang,ml lines 10 this curve at 1..he two points where
it cros,.cs 1hc x axis arc I.he -.amc. In other word<;. show th~u die 't angent lines al (±h. 0) are parallel. Do the
same for the tangent 1lines al (0 , ±2). whe.re the eUipsc croises they axis.
26 hapr,·1 I / 1-unrlions oi a Singl Variabl

8. Find rhc local cxrrema and 1hc inl1cc1ion puint~ of /(x l = :-x-1 - .:l.r .1 - ::!4x 1 + -ll:h - ::w over the entire .r axi~.
9. Determine the minimum value of .f(x) = I , x 1 /.'.

10. Prove rhal .f(,) = (.r - 1J 2(x - 2) 2 is symmetric about rhc ve.nical line ln,:atcd at.,·= J/2 .
11. Prove that if y = /(.r) and .t -' g(_\"l. then

dy
J.r = Ji
d\'

Him: Differentiate f(~(y)) as a cornposirc funcrion.

I 2• Show that -d 11 i = ,.
I'll -
,Ju
- + (In 11)11 ,,di'
- . /-li111:
Le·1 \' = 11 ,. an d·1·r· . I
ur ll'n:ntralc n \'.
dx dx dx ·
. . d t
13. Usc the rcs u It of the- prev 1ou s problem to h nd - .r .
tlx
14. Show 1hn1 a =
fJ in Figure 1.41. This propeny i-., 1.:nown as the rt;ffl-ctiw1 11w1>a/\' o( ,1 {>llfabo/(J and is the
basis for a par.1bolic lc11-.; incoming light is focused .it the point F, rhe focu~ of the par:1hola . Thl' equation of
1hc parabola i, \-~ =-l p.r. where p i, the disrancc OF in Figure 1.41.

Figure 1.41
lllu$\rntion nf 1hc re Acct ion property of ;.i
p;ir;1hol.1. (St'l' Pr.ihll'm 1-1.)

15. No1ice 1.hat J'(a) is defined by Equation I. Normally we evalua1e j'(a) by linding j'(x) ancl then le11ing
x =a.bur 1his procedure is 1101 qui1e rhe same as u.sin£ Equaiion I. Consider 1he funclion

2 sin( 1/x) .\" ¥- ()


J(x} ={x
0 X =0
The dcrviarive of J(x) al x == 0 is. by defi11itio11.

j'(O> = lim f(O + ~x > - f(O)


r- o 6.r

Show lhat /'tO) = 0 in thi ~ ca:,,.e. Now detcmli1ne J'(.~·) and th,m let .r -,. 0 and ~how lhat rhe limit of
J'<x) a., .r - 0 doe,s. not exi,sl bcl:nu,.;e lim c,'<hl: 1/x) due-. not c.11:i!>I. TI,crcforc. we ,i;.c 1hat in this c;-L,e.
0
/'(0) :/= lim /'(x). l'he problem here i 1ha1 j'(.r) is n.o1 coruimuous at .r
\ _fii
= 0 . The functiuns 1.h:11 we dea,1wi1h
in physica'l problems almost always have continuous deri\'oli\'es. but it's ~uud 10 keep Equarion I i 11 ntilildi.
27
16. Thl.'. hl•ight of a body shot vertically upward is gin:n as u fonc.:tion of1ime by h(t) = 401 - 2
J2t . How high will
ii :!O'!
17. Show 1hnl 1hc rccranglc or largest pm,siblc arra for a givcll perimeter is a square.
18. Which poirnt.s on ·the culiVe .ty::! = I arc clo~l-110 the origin?

19. Two pan ides arc moving in a plane au::ording 10 lhc parametric equations (31. 4r·' - 61 + I) :ind
(31 + I. 4t 5 - 81 - :!). How dose do rhcy come lu each other'!

20. Find the lar,0csl possible area of :m irsoscclcs I.Fianglc inscribed in a circle of radius r.

21. 1h: hlackbo<l1y r.1Jin1ion la.w i!.s gJ\'c.n hy

81th"
P ( i.. ') - - - s- -__,- ,,-,.-.,-. -
1-. - er' ·"B - I

where r)d). is I1hc eneri:?y be,twcen >.and),+ dA.. A i,s the wnvcleng1Jh of the radiation., I, i~ rhc Planck
p().,
conslanl. k 13 is the Bolmnann
co nstant, c is Lhe speed of light. umJ T is t he kelvin ,femperaturc . l 1he Wien
di:-plm:emcm law say.. tha! ).. 111,,(T = c.:nnstant \vhcrc Anw:1. is the \'a) uc of A al which />(A. f) is a m.Lximum.
Deri\'e Ihe Wit:n 1.f i,placcmcnt law from 1 hc bl.1c:kbody raJiaLion law. Shuw 1ha1 "cuns1an1'" = /rc/4.9651.. 0 .
22. Prove that if .f(.r) has a dtniva1ivc :11 x =a.then ii nwst he co111inuous tliicre.
=
23. It j.., Ca.'-Y 10 prow (Problem 2'2) that ,jf f (x) has a derivative al x 11 . then ii 111u_c;1 be continuous at x ==a.The
conwr~c is not nccc.•s-~ari ly tme, however. For example. sl:w w that, the func,t ion

.
1

I
I
X SIii - .\' ::p 0
_\' ( X ) = X

() X = ()
is continuous al x ,- 0 hu1 has no dcriva11vc there. Actuallly. nrn1hcma1,ic:ians have cons1 ructcd tunc,1ion:- 1ha1 1

.ire c:onlinuou, at all point:- in un in1c.rval but diffcrt:n1-i.1ble ,H no roinl '- . (Sec pa.gic: 573.) Fonuna1cly. such
patholuf!.it"al fuJK'.tions do not norm:dly arise in physical applications .
24. Dcriv' t.hc fomwln for the derivative of the product of two functions .

25. Veliiify 1l1e second dcrivat.i1,·c conditicins for a lo{.;al cx1rcmum.

Tht· ,w.\1 4 problnn\· /wire yoI1 1/Sl' any C1\S 10 differe111ia1e )1mc1in11s sym/J()/ically.

26. Diffe'rieliltia1e f(x) = t' ·•x:!' ~in 3 (3x~ + 2).


27. Evaluate J'{ 1I) from 1he Jilrcv iou, problem.
28. Find the ~cc:nnd derivative of f(.n frorn Problem 26.
29. Evaluate J''( I) for 1he prl!v,ious problem.

1 .5 Diffe rential's
Calculus book!'- emphasize 1hat the cxpre..j n dy/d.r is a single quan1i1y and 1101
the r.itio of d.r over d:r. Yet we later u~e c,xprcssions such as. dy = y'(x)dx with
abandon. The use of dy =
y'(x)dx is ju~tified. however. if we think of ii as the
28 ( l1,1pll'r I / Furu lions 111 ,1 !>inglc Vari,1hl1·

differential of y. We also use differemials 10 estimate errors in measuremen1s. ns


we sha.l I see.
The derivative of y = y (x) is given by
. Li\'
-
I
v (x) = ,lx-►
11m --
0 Lix
( I)

where Liy = y(x + Lix) - y(x). For small values of Lix then. we expect y'(x) lo
beclo$e to t::.y/t::.x.or

(small Lix) (2)

We can rewrite this as

y(x + Lix) ~ y(x) + y'(x)Lix (3)

Let's use 1his expression 10 cs1ima1e sin(0.10). In Lhis case. we Lake


y(x) = sin x. x = 0. Lix = 0.10. and y'(x) = cos x in Equat.ion 3 to get

sin(O. I 0) ~ sin O + (cos 0)(0 . 10) = 0.10


The actunl value is 0.09983 10 four places.

Example 1:
Suppose that we eslimate the volume of a sphere by measuring its
circumference and find it Lo be I 62 cm with an uncertainty of 0.20 cm.
Eslimnie Lhe uncertainty in the volume of the sphere.

SOLUTION: The volume is 4rr r·'/3 and the circumference is C = 2,r r .


Thus.

4nr 3
V=--= -
3 c
3 6rr 2

Using Equation 3. we write

uncenamty . . V = t::. V = (dV)


in dC l\ C

So that

l\ V = (-
c2 ) l\C = (1622,rcm)~
., (0 .~0 cm)= 266 cnr1
2rr 2 ~

or about a 0.4% unccrlll.inty in the sense thnl l\ \I/ V = (266 cmJ/71800 cmJ)
=o.oo:n.
1.5 Oii r ntials 29

The increments 6y and 6x and the tangent line atx are shown in Figure 1.42.
The quantity 6y = y(x + .6x) - y(x) is the change in y that results if we continue \! Line
along the curve and d_v is 1hc change in y that results if we continue along the
1angi::111 line from x ro x + 6x. This change in y is the differr:111ial of y nnd is _., , + ay
given by
dy = y'(x)6x (4) y + dy

Fiiurc 1.42 suggests that 1he smalkrthc value of 6x, the closer .6y and dy become.
To ~ee 1hat this is so, let .6x = x - x 0 and write Equation I as

X X + .!l.r X

Equa1ion 5 impJ,ies I.hat 1he dilfference between 6y and dy in Figure Jl.42 not m1ly
1

Figure 1.42
goes 10 zero as x - x0, but goes ,i,o zero faster 1han 6x = (x - x0 ) !!Des to zero An illu~1r;11i11n of the diffcrc mcc between
I for example. as (6x) 2 '1 because nx appears in the denominator. To sec what we dy and 6..v.
mean by this. <.Jefine <! by

(6)

lf we substitute Equation 6 into Equation 5. we see that

. ~r - v'(x)L'.x . E 6.r .
I 1m
x-x 0
· ·
L\x
:::::: 111n - -
x-• xn f'::.x
= ~.r-0
ilm t: = 0 (7)

Thus we see that € ➔ 0 as .6.r ➔ 0. Now multiply Equa1ion 6 by 6x and use


Equation 4 to write

6y =dy +~ 6x (8)

Thus we see that ny - dy even faster than nx lmorc like ( 6x) 2 ] as 6..r ➔ 0.

Example 2:
. . . dv 'f
Denve an expression tor 1: = -6 ·r - -=- 1 v = x · + x and show that E ➔ t
1 )

tu dx ·
ai. 6x ➔ 0.

SOLUTION:

b.y = y(.r + b.x) - y(x) = (.r + 6.r) 3 + (.r + 6x) - .r 3 - x

= (3.x 2 + 1)6x + ld6x) 2 + (6x)J


6V
-· = 3.x- + I+ 3xL\x + (6x)-
l 1

6x
L\v d 'I-' .,
1: = -·
6.x
- -=- = Jx b.x + (6x )-
dx
30 Cl , p1t~r 1 / Funt !iom nf a in >IL• V,HiJ hl •

Because fly= dy +€ 6x and f -> (J as ~x - 0. Equal ion 2 can he written


as

(9)

which shows that

dy = y'(x) dx ( I 0)

is an excel lent approximation in 1hc sense that the correct ion term, € dx goes to
zero fas1cr 1han flx or dx go to zero.
Lcibnltz. co-founder of calculus wi lh Newton. introduced <li ffl'rl'ntial notation
and considered 1hc .,lope of 1hc t.angcn1 line 10 be the ,dlio of 1hc infinitesimal
increments dy and dx, as shown in Figure 1.42. Rcali,c. hO\\ c, er. that dy and dx
are 11011hc limits of 6y and 6x as 6.r - 0. since tl1L·se limits ::ire nee~ ·uily zero.
whcrl'as dy and dx. as we have in1roducc.d 1hcm ahovc, arc no1 nl'ccssarily zero.
Ocrasionally you'll see wrinen thai fly~ y'(x)6x becomes dy = _r'(x)dx when
6x become$ .. intinitesimally small". where 1he quotation marks emphasize th:11
the 1crm "infinitesimally small .. is vague. The early devclopmen1 of calculus was
frc.1ugh1 with examples where L·l!nain quantities were ":mrnll" at one stage, 1.cro
al anothc:r, and then ··small" .igain a1 a la1er stage in thL' same discus,ion. These
vagaries were eventually ironed out by the introduction of limits as. we use them
today.
The introduction of diffcreniials allows us to express derivatives in what is
called differcnliill notation

d(i.11') = udr

and so on.

Example 3:
Use differen1ial nota1ion to lind dy/dx for xy - 2x 2 - 1/y~ = 6.
SOL UT JON: We have

'!.tl\·
:rdy + ydx - 4xdx + - \ I-· =0
or
dr 4x - ,. 4x\· - \'.i

d:r = --------t"
x+,
= xyJ +2

I .fi Mc.:in Value Thcor •n1-s 31

1.5 Problem
1. Estima1e Yi2o from ✓125 = 5.
2. The side$ of a cube are ob~l!rw.d 10 be 8.00 ± 0.02 cm. Esrimnte r.he error in i1s \iOlume.

. an cxprc~s,on
. tor
. th . .
3. Dcnve E = -·
6. \" - -'- 11 \" = x-' + x·, + 6.x - 3 S how
.. 1ha1 f --,.
O ns 6.x ➔
0.
6.x dx ·
D..\' dr ..
4. Derive an expression fore - · - -'- it y = 1/(x +a).Show thnl E - 0 as 6.x....,,. 0.
6.x tl.r
5. Find 6.y and dy for y = y(.r) = x 2 - x al x = I 0.0 and 6.x = 0.10. Sketch a figure similar 10 Figure 1.42.
6. Find 6.y and dy for y = y(x) = x 11 1 at .r = 4.00 and 6.x = 0.35. Calculate the absolute and relative errors in
replacing ~y by dy.
7. fatimatc the change in cos lJ if 0 is changed from 25.00° to 25.20°.
8. Find dy if y =
xl / 2
(al (b)
4 4.rJ
~
(c) tan~ x (d)
]xl 1
9. Find dy if y =
(a) (.r2 _ 2)1/J (b) Sin Jx
,,\.·(1, l
(c)

IO. Doe.s y = (I + 2 In .r)/(x - In x) satisfy the relation (.ry - y - 2)dx + (x '.! - x In x)d_\' = 0?
11. Use differential/. to find dy/dx for
(a) (h) x 3 + y 2 = 6xy (c)

1.6 Mean Value Theorems

There is a 1hcorcm concerning diffcrcn11ablc functions Lhat is used often in bolh


pure and applied ma1hcma1ics. The theorem i.".- culled 1hc mean voluc theorem for 'f
derviaJive.,·. Before we di~cuss this theorem. we shall discuss a somewhat simpler
theorem called Rolle'.,; 1heore111. Rolle's lheorem says Lha1

If f (x) is cm1timw11s on tl,e clo.w,·J inren·nl l u. h I and tli/(ervnliable on the


=
open inrcn•al (a. b ). a11d if f (a) = f (b) 0. rlie11 there must J,,, at least 011e
point~ in (a. b) such thlll /' (~ ) 0. =
(Figure 1.43) In other words, if the graph of a continuous f-unction j(x) intersecL-. Figure 1.43
1he x axis al x = a and x = h. and if 1he funcrion is differentiable between a and An ill~lrJ1ion or lfollc"~ 1hcorcrn.
32 ih.1p1 ·r I ,I Fu111 1io ns of ;1 Sing.le VJri.1bl ~

b, 1hen there is at least one point between a and b such 1hat the derivative of /(x)
is zero. (The proof of Rolle's theorem is sketched in Problem 13.)
Now on to the mean value 1heorem for derivatives.

If f (x) ;_~ co111inuo11s in 11,e closed i11ten-al I n. b I 1111d differentiable in the


open interval (a. b). tlie11 there is a pui111 l; in (a, b) such rlwr

f(b) - f (a) = f'cO o<~<h (I)


b-n .

No1c that we can express Equation I in 1.hc fonn

f(x) = f(a) + f'(~) (x - a) (2)

Problem 14 skc1ches the proof of this lheorem.


Equation I has a nice physical interpretation. Suppose that /(b) - /(a)
repn:,enls the distance between two points and 1hat b - a represc nl.S 1he lime it
takes to 1..ravcl from" to b. Then fj(b) - j(a)l/(b- a) represents 1he average
speed for that trip. The mean value Lheorern (Equa1ion I) says I.hat if you average
I 00 km • hr- 1. say, then al some point during your trip, your ins1an1aneous speed
must be 100 km• hr- 1•
An immediate consequence of the mean value theorem is that if f (x) is
continuous in I a. h I and diffcrcnliablc in (a. b). 1hcn j(x) is an increasing
function in [a. h )ir j'(x) > 0 for all x in (a.b).To prove 1his, let x be in I x 1, x 2 I
with x~ > x 1 und use Equation 2 with a replaced wi1h x 1 and .x wilh x 2 to get

where .r 1 :::: t =:S x 2 • Since x 1 > x 1 a.ad J'(s) > 0. we sec that j(x2 ) > /(x 1).
Con\'er,cly. if J'(.xl < 0. then f(x} is a decreasing function in fa. b 1-

Example 1:
Show thal f(x) = x 3 + 3x - I has exai:.:t..ly one (real) zero in the interval
I -I, I].
-I X
SOLUTION: Since f(x) is equal lo -5 at x = - I and +J al x =I.there
is al lca:-1 one rero in I -1, I ]. Because

J'(x) = 3x 2 + 3 > 0

Figure 1.44 for all x. f(x) is an incrca.~ing function on lhc cn1irc x axis J.rtd so cannot
The functiDn /(x) = x 3 + Jx - I ploitcd have more than one 1.cro (Figure 1.44 ).
n :•uin.st x.

al
1.6 Mean Value Theurt·ms 33

There is an extension of r.he me.an value theorem 1ha1 we can use 10 derive
the formula for a Taylor series with a remainder. This higher-order mean vallle
theorem says Lha1

If f (\) and its first II derivo1ives are contin11011.f in la. b I and if J'n+ 1) (x)
exist,· 011 (a. h). 1he11 there is at letw one point~ in I a. b I s11ch rliar

The proof of this Lheorcm is outlined in Problem 17. The mean value theorem is
just EquaLion 3 with 11 = 0.
If we let b = x in Equation 3, we get

I (''(f..l) '.l
J(x) =/(a)+ J (a)(x - a)+ ~ ( x - at

+ ... + / "(a) (x - ll)" + /'1 '(t) (x - a)'1+1 (4)


n! (11+! )!

where a < ~ < x. Equal.ion 4 is the formula of a Taylor series with remainder,
where the remainder is

(5)

We shall use this fonnula in later chapters.

hample 2:
Use Equation 4 10 calculate the value of In I.200 lo four-place accuracy.

SOLUTION: Let /(x) =In.rand"= I in Equation 4:

. I ~ I ,
In x = (x - I J - - t r - I)- + - (x - I)·
2 3

where I < ~ < 1.200. \Ve wani 1ht magnitude of 1he remainder term 10
be .::: 0.00005. This term will be greate... 1 if we choose ~ :;:: 1.200 nnd so
o- =
I Rnl:;:: ((l.20)" 1/ (n + I) will equal 6.40 x I 5 if n 4; 1.077 x I 5 if o-
11= o- =
5: :rnd 1.83 x I 6 if 11 6. Thus we will be assured four-place 2ccuracy
= =
if we choose 11 5. TI1is ~ives In 1.200 0.18233 compared to 1he tabu lmed
value 0.18232.
34 Lh,1ptt•r 1 l·u11( tio ns oi a. in le ariabl e

You probably remember I' Hopitars rule from your eak:ulus course. You used
it to evaluate the limits of indeterminate forms such a, 0/0, 0 • oo. and oo/
l'HopitaJ's rule says that if _{(x) and g(.r) both approach LL'rO or both approach
±oc. then

• J (x) . j'(.t)
I1m - - = 11m - - (6)
g,(x) g ' (x)

provided the right-hond limit cxisL,. r Hopital's rule is included in this section
jf
because its proof follows immediately from a version of the mean value theorem
(Problems 15 and 16).
For example. lirn sin x /x j,_ or I.he inde1cm1inate form 0/0. 1' Hopitars rule
.r-,O
tells us 1ha1
X
. sin x . cos x
Im, - -
x-0 X
= 1101 -- =1
Figure 1.45 t" )

The bch,w ior of 1hc func1ion


f(:c) = (~in .r) /. as .r -, 0. which agrees with our geometric proor in Seel ion 2. Figure I .45 shows the behavior
of sin x /.x .is x -> 0. How about Jim x In x? This limit oci:urs fairly often in the
l >0 I
physical chemistry of electrolyte solutions. :-.-uch as aqueous sotu1ions of sodium
chloride. In Lhis case. we have the indelemtinale form -0 - oo, so let's look at

. In x . 1/.r . O
I,m
, - .o
-
1/x
= ,· -11m
-0
--
- 1/x ..,
= - ., 1,11111
o-,
x = 1

Figure 1.46 shows the behavior of x In x as x -,. 0.

Example 3:
Detennine lim xe- ·
Figure 1.46
T11c beh:iviorof the fun 1ion .f ( r) =x In x so Lu TI o N: 11,i cxpres:;ion is of the indc1e.m1ina1 · frwm oo • 0. ll
!JS.I'_.. 0. becomes an / ,. ._ form by writim! it a~

r I
l,im .:...
e·t
= ., lim - =0
-'"".\.. e-r

The result of Example 3 i!i- a special ea~ of the limit of x"e-x as x - oo.
where 11 is any integer. We can ea.-:ily find this generc1I limit using mathematical
induction. We know from Example 3 thatx" e-· Oa-.x -> whenn = I. When
using ml.lthematical induction, we assume thal if a slatement is true for some value
of 11 L then it must be I rue for n + \ also. So

. r" ! I • (11 + 1).x" yll


11111
.r--:x,
--
e·\
= lim - - - -
,r-.'X' e-r
= (11 + 1) ,-x
lim :_ = 0
e·'
(7)
1. 6 ,\\,',Ill V,l lu, · I heor ffi$ 35
But we know that lim xne-.t 0 as x---.. oo for 11 =
I. so it must he 1rue for 11 2, = =
11 = 3. and ~u on. This result. which ts wor1h remembering. says 1ha1 e- .f __,, 0

fa.,ler than any power of x as x - oo.


Ano1her limi1 wonh remembering is

. lnx
I 1m - = 0 (8)
I •"- ,rU

for any et > 0. This limit says 1ha1 In-~ - oo more slowly than :my positive power
of x. no ma11er how small (Problem 7): or cquivalcnt.ly. that xc, In x - 0 a."- .x - 0
for any a > 0 (Problem 8).
Other indetenninatc forms such as o0 • cx:/1• and 1-x can often be handled by
tak.ing the logarithm and manipulating the result into the standard indeterminate
fonns 0/0 or 00/ 00. For exumple. consider ,!!_~~ x,(. Let y = .\..i·. and then look al

t'
lirn In y
0
= X
lim x In x
O
= 0: and so .I
lim y =Lor final l y lim x 1·
- •0 .I 1J •
= I.

Example 4:
Dc1cnnine lim
,1 - .....
:Jr. where p > 0. 2
SOLUTION: We'll let y = 11 p . lake logarithms. and I real II as a continuous
variable.

. . I
hm ln y = hm - In p =U
I/ '\.. I, • '\.., JI n

So lim r = lim ::fp = 1. Figure 1.47 shows ::/2 plouc-d against n. figure 1.47
,,i •:\.. ,, x,,
Problem 12 has you show th.:11 ;j,i - I as 11 - oc. The fun tinn /(x) = ,f j_ plo11ed agains1
11.The 3!-)°m,ptote i~ shown ~ a da~hcd
line.

Whal if you apply I' H6pital's rule and you still get an indctcnninate fonn?
Simply apply ii succc--.sivcly Ulillil you no lomgcr obtnin an indetenninatc form.
for example.

. 2 \'.' - 9x~+ 12x - 5 . 6x 2 - I 8.x + 12


lim , = tm, --,- -- --
.r-1 2x 3 - \8x-- + 30x - 14 .i:-1 6x - - 36x + 30
12x - 18
=lim - - - - =
.1 - I 12.r - 36 4

(See Problem 11. however.)

1.6 Problems
I. Use Equaiion 4 to calculme sin{;r /4) to four~place aC(;uracy. Hi111: Realize thal I sin x I _ I and that I cos x I ~ I.
2. Use Equation 4 to calculate the value of e to five-place accuracy. Hi,,,: Use (he fact that e ::c 3.
36 Chapter 1 / Fune-lion of a ingl£> \ l;iri .:ibl ·

3. Argue that j(x) = x 3 + px + q has one real roo1 if p > 0.


4. Use l"Hopital"s rule 10 dctennine 1hc following limits:
sin 2 x . I - cos x
(a) l.im - - (b) hm
.c-0 X .1-,0 X

(c) lim
ex
-
I
- .l
·
(d) lim
I + cos 2x
.r-,,0 x .r rr ~ I - sin 2r
S. Use l'Hupital's rule 10 dctem,inc 1hc following limits:
. e' - I . I - cos X
{a) 1lffi - - (b) I1m .,
.r-•0 X x-•0 .r-

(c) lim ~ (d) lim ( J :r: 2 + 2..r - .r)


x-1 I- x x
6. Use l'H6piial's rule 10 de1em1inc lhc following limits:
·1rn I - c~s2 .r
(8) lim (c.sc x - col x) (b) 1 .
.1~0 x-•O x-
. ln sin x (d) Jim ln ( I + .r)
(c) l1m ---
.r-.o+ In tan .t .r-0 X

. lnx
7. Show tha1 for every a> 0. I1m
x-oo x«
- = 0.
8. Show that for every a > 0, xa In x -. 0 a.-. x ➔ 0.
1
9. Determine lim x /.r_
.i:-oo

. . J i + x:?
lo . Dctcnnine 11m - - - .
,t -<• X

11. At the end of the section. we found the limit of (2r1 - 9x 2 + l2x - 5)/(2x3 - 18.r 2 + 30x - 14) as x ➔
and got 1/4 as an answer. What's wrong with the following?

ltm
.
1
. 2:c - 9x~
.,
+ 12.x - 5 .
= lim 6x 2 - I 8x + 12
-.,-----
x-• I 2.x 3 - 1Rx -- + 30x - 14 x - 1 6x -- - 36.\ T 30

= Jim llr - IS= lim ~=I


.r-- I I 2x - 36 (-• 1 12

12. Show that :/ii - I as 11

13. We'll prove Rolle's lheorem in this problem. Use the extreme value theorem to argue that f (x) must have a
maximum and a minimum value on [a. b l. Now argue thot if f(x) has any posi1ivc values at ull. then there is
al least one point ( in (11, b) where f(x) is a maximum, so lhal f'(~) == 0. Similarly, argue 1hat if J(x) has a
negative value. 1hen there is at least one point 11 in (o, b) where f(x) is. a minimum, so that J'('Y/) = 0. Finally.
show thar if f ( r) ha"' no positive or negative value, in (o, h ). !hen _I \X) i:-.. idcnticnl\y zero on \ a, b I. and so
J'(x) = 0 for all x un (a. bl.

14. Figure 1.48 illustrates the mean value theorem for derivatives. To prove this 1hcorcm. we consider the f unc1ion
F(x), which is the difference between / (x) and the straight line connecting the points (a. J (a)) and (t,, J (h)).
as shown in Figure 1.48. Show that the equal ion for F (x) is

J(b) - f (a )
F(x) = f(x) - j(a) - . (x - a)
b- a
I .7 J111tp,1tion 37

Figure 1.48
t\n aid tn rhc proof of Lhc mean value
lhl'cm..·m Clf derivar.ives.
l1 X b X

Note thu1 F(o) = F(b) = U. as implied by Figure 1.48 and that F(x) sati'< fi •-. 1hc criteri a of Rollc"s
theorem. Using the fact that rhcre is al least one poin1 c in (a. b) at which F' (x) = 0. show that
f(b) - f(u) =
f'(c)(b - t1). Note thut we obtain Rollc"s theorem ir /(/,) = f(a) = 0.
15. The .;;1Jndard pmof of 1· H6pita1 ·,._ rule i." ba..'-Cd on what i~ cal lcl.l the exu,,ulnl 111e011 l'(J/11e tlworem 1.ifderiva1i1•es.
It :-.ll)''i- that if {(x) and g(x) are continuous in [ a. b I and differentiable in (n. b).then there is at least one
poin1 c in (a. b) for which

j(b) j(o ) /'kl


~(b) - g(a)
= ~'(c)
where we as ume that g(a) f. g(b) and /'(x) and g'(x) are not simultaneously zero. Nore lhat this theorem
reduces 10 the mean vuluc theorem if g(x) =
.r. Prove the extended mean value theorem by sraning with

• , f (b) - f (a)
f (x) = Jtx) - f(h) - - - - - [g(x) - g(h)I
g(b) - g (a)

16. Use the result of Problem 15 10 p-rovc l"Hopi1al's rule for the form 0/0. Assume 1hat /(x) and g(x) arc
differentiable. that g (x) ;c O in (a. b). a.nd that lim J (x) = lim g (.r) = 0.
X (I .f a
17. We $hall outline the proof of Equmion 3, the higher-order mean value 1heorem, in this problem. Let a cons1.an1
k be defined by

Now apply Rolle's theorem to

J< 11 l(.x)
F(x) = f(x) - f(b ) + j'(x)(b - x) + -J" ·-r) (b - xl 2 + ·· · + --(b - xt + k(b - x)"+'
2! 11!

1. 7 Integration

The idea of an integral wa.:; originally developed 10 calculale the area bounded by
given curves. but nowadays an integral is defined by a l.imiting process. Consider
the situarion in Figure 1.49. The interval l a, b J is subdivided into n sub-intervals.
38 ( h,1p1er I / Fu:11 lions u( a Single Vari,1hlt!

Figure 1.49 • .. X j b
a j- 1 X
TI1c construction a~sociarcd wi1h a
Riemann sum. (;

(a, x 1). (x 1• x 2 ). ... , (xn-l· b) and the point. {j, is located anywhere wi1hin rhe
j lh interval for j = I. 2. . .. , We now form the sum
11.

II

Sn= L J(~j){xj - xj_ 1) = L fl< 1 )tlxj


)==-I }=I

where x 0 = a. x 11 = b. and tlxj = xj


Geometncally this sum repres~nts
- -"j-l ·
lhe sum of the areas of the rectangles in Figure 1.49 and is called a Riemn1111 ~Lim.
If we subdivide I a. h) into more and more subintervals wi1h smaller and sma.ller
widths, we cvcnrually reach a limit. called 1.hc Rienu11111 i111egrol off (x), denoted
by

(I)

where (LI is Ille width of the largest subinterval. Of course, we are assumjng here
lhar I.he above limjt exists.In the e-S no1a1ion. we have

whenever IL) < 8 (2)

This limit exists if f (x) is continuous (or even piecewise continuous) in I a, b /.


Geome1rically. 1hc integral of J (x) from " 10 b represents 1he area between
f(x) and the x axis and the vertical lines at x = a and x = b if j(x) is positive
everywhere between o and b. 01.herwi~c. it represents the net area. with areas above
lhe x axis treated as positive and arc~ below the x axis ~ated as negative.
We cenainly don'1 use Equations I or 2 to evaluate integrals. As you know. and
we·11 show below. in1egra1ion and diffrren1ia1ion are inver.-e operations, so we'll
I. 7 lnle~r.11ion 39

evaluate integrnls by work..ing backwards from tables of differen1iation formula..,;. In


f:.tct. the grca1 achievement of Newton and LcibnilZ in 1hcirdcvelopment ofcalculu:-
in the late I 600's was to appreciate the inverse rc!arionship bcrwccn differential ion
and integration .
Prior to that lime. the fomwlas of the slopes of rnany curves had been derived
and 1he areas bounded by mi.tn)' curve~ had been t.ktennincd. usually hy ingenious.
if not even quirky. mclhoili. but th~ inverse relation between finding the slope of
the tangent lines to curves and area..-; hounded by these curves was not recognized.
A good example of 1he. dc1erminarion of areas hounded by given curves is due
to Archimedes. who determined the area under the parat>ola. r = x 2 • al111os1 two
thousand years before 1he fonnal development of calculus. We know 1his area 10
he ' ,.

bu1 let ·s spend a page or two discussing 1his ··prc -c::ilculus·· calculation. Let's start
with lhe simple example. y = x. We don't even need calculus to detcm1inc the
(1 b X
area under this curve from II to b bcc;,.iusc ii is the Jiffcrenc.:c between the areas
of the two triangle~ in figure 1.50. Of course. the integ-ral of y = x from £1 to Figure 1.50
J:
b gives I x 2 /2 = (b 2 - a 2)/2. whi~h is the s.amc thing . As a preparation for The in1ewJI f~ .,·,Ix is gi\'e11 by 1he
An.:himedcs·s calculation for _r = x~. let's dc1cm1inc the integral off (x) = x using shaded area.
a limiting rroc.:css. For convenience only. let\; take 1hc 11 ~ubintcrvab in Equation I
to be equal and given by D-x = (b - t1)/11 and take ~j 10 he the extreme right-hand
value wirhin each interval. Then Sn becomes

IJ /l

Sn= L ~ill..,· = 1:)1 + jll.x)6x


J=I j=I

n 11

= a L ll.x + L j(ll.x)'2.
J=I 1=1

The above surnrn~lion over j is the sum of the first II integers. which is equal
to 11 (11 + I )/2 (Problem 12). This rc:-ult was well known in Archimedes· time. anll
so

But ~x = (h - 0)/11. so
40
and
1' b"!. - a 2
1 u
xdx = ,,Jim '-
__ .._ S,, = - -
-

For the case of the area under the curve y = x 2• we have


TI II n
S,, = 2)a + j D.. .r} 2 /:}..x = na 26.x + 2a(.6x) 2 L j + (6.x) 3 L J2
j;::;I j=I J=I

The sum of the squares of I.he first II integers is equal to 11 (n + I) (2,, + I) /6. again
well-known 10 the ancient Greek ma1hema1i~i;1ns (see Problem I 3). Therefore,
~
_ 2 (/ _ a(b - a)~11(n
'\
+ I) (h - a)- 11(11 + \)(2n + I)
S,,-a ') a)+ ., +--- 1
11~ 6 n·

,, _ 00
as

or

There arc a few properties of definite integrals that we should point out here.
Mos1 follow from 1he definition given in Equation I. If c 1 and c 2 arc constants.
then

(3)

Equation 3 is similar lo Equation 4.3 for differenliation. Jwa as we define a


differential operator in Section 4, we define an integrd.l operator by

l f(x) = lb
/l
f(x) dx

Equation 3 says that integration is a linear operation.

(4)

Some other properties th1..11 follow from Equation I are

1,
0
f(x)dx = far f(x)dx + c lb f (x)dx (5)

provided _f (x) is integrc.1.ble in la, c I and [ c, b J; in olher words. provided the limil
defining the two integrals exists. Also, if m S f(x) ~Min l t1. b ]. then

m(b - a) :S 1" _((x)dx ~ M(b - a) (6)

If f(.x).::: g(x) in [ a. b ]. then (Problem I 5)


1.7 lntt•!:_:r,1ti1111 41

f
IJ
bf (u)d11 .:'.:: {,-, g(z)dz.
• "
(7)

and (Problem 18)

It f(x)dxl :S [ lf(x)ldx (8)

Example 1:
2,"T cos ;u
Prove that lim
,; £
•O
- --,, rlx
x - + 11-
= 0.
so LU TIO N: We use Equation 8 to write

11)
2
:, <.:OS/IX ,dx
X2 + n-
I<12.Jr 1
_ ~+
0 X·
,
n-
1 <j.:b - dx
., dx _ , --., -<
0 x- + 11-
11."T dx
o
- ., = -2rrJ
n~ 11 -

which--,. 0 as 11 - oo. In going from lhc second tenn lo the third tcnn. we
used tl1e fact tlrnl I cos nxl .:5 I.

Suppose that j(x) is piecewise continuollS in [ n. b ] with a jump discontinui1y


al x = c. Then
I =
1/,
0
f(x)dx = Jim
f -
re-~
U• a
j(x) dx Jim
[h
, _,, 0 • I'+(
f(x) dx (9)

Example 2: .f
Find the area between the x ax.is and (Figure 1.51)
2
x<O
0;:Sx<I
I sx < 2
x~2
2 X

SOLUTION: Figure 1.51


= =
I= lim
€-•O 1 O
1-(
dx -t- lim
• -~ ·-o
!.:!-~'
l-1-£
2"x
The function f(.r) 0 for x 0; f (x)

/(_r)
=
I
forO _ .r < I: f(.r) 2 for I_ .r 2: nd
= 0 for x _ 2.
= l-.0
lirn ( I - E) +2
f
Jim (2 -
.t'-->0

1
- I + t:=) =3

We hav~ two jump disconi-inui1ies in Example 2, but the generaliz.alion of


Equation 9 Lo more thun one jump disconli nui1y is apparent. In fact. f (x) can have
42 Ch,1pter 1 / Functions of .1 Sin •le \l;i ri;ihl

an infinite number of jump discontinuities if it is a counlabl) infinite number. Also.


the values of .f(x) at 1he cn<l points of each interval in h.amplc 2 arc irrelevant.
Whether we write f (x) =1 for O S x < I. or n < .r < I. or O .:S x .:::. 1. or O < x .:S I
makes no difference at all.
We presented the mean value theorem for derivritives in Section 6. The very
name of the theorem hinL-; that there is a mean ,•alue rheoremfnr i11tegrols. If Jt.r)
is continuous in [ o. b J. then there is a point c in [ ". /J] such that

fII
h /(x)dx = (b - a)/(c) ( I U)

To sec that this is so. leL m ilnd M be the minimum and maximum values of f(x)
in I n. h I anJ use Equation 6 to write

m _ - 1-
b-a
1h fLt)d.r ~ M
0

Since j(.t) is continuous. it takes on all value-s between m and 1W. So there must
be 1-ome point c in f a. h ] such lhat
_f (c)

- I- JI> f (x)r/x = f (c) (I IJ


I> -{I "

X
,, f(x)dx is equal to the arc.a of the rcctan\:!lc
Eq u:uion I I sa y.s that Lhc ..11-ca .(" ' ....

Figure 1.52 define.d by x = a. x = h. y = 0. and y = j(c). Figure 1.52 gives a pic1ori..1I


A pictorial representation 11f rhc me;in represen1.1tiun of Equation JO.
value 1hoorem of integration. F.qu:i1ion 10. There is a generalization of the mean value theorem for inregTals chat say~ 1hat
The arc.a within the oli<.1 fel.'.ltmg k <:(Juab
the ~hadcd an:.a under the rnlorcd curve. if j(:r). x(x). and g'(x) are continuous on I a. h] and g(.r) does no1 change sign
in [ o. b ]. then 1hcrc is a point c in [ a. b I such that

! I,
j(x)g(x)dx = /(c) !/,
II
g(x)dx ( l 2)

No1ice that this reduces 10 Equation 10 if g(x) = I.


The integrals that we have been discussing so for have been between a a11tl b
and are definite integrals. If the upper limit. b. is replaced by x. then the integral
defines ;1 function of x and is called an indefinite iJllcgral. We write this as

F(:r) = [.1 .{(11)£111 ( 13)


• /l

Notice. incidentally, that we are using II and nor x as our variabk of integration so
that we c:an distinguish between !he variable of integration and 1he upper limi1. The
designatinn of the integration vi:lriablc is arbitrary and is what we call a dummy
variable. In other words.

, 1.( 1·{
F(.r) =
1 a
j(11)d11 =
fl
/(:)d~ =
tl
j(f )dr
1.7 lnt g r 1im 1 43

arc completely equivalent However. wri1ing

F(x) = 1.r f (x )d.r


a

is poor prac1ic:e Jnd should h~ avoided hecause 1hc x in the upper limit ,rnd the x
in the integrand represent difkrcnl quantiIies.
The Jimdomenwl rheorem cf calrnfos says that i r /(x) is continuous in the
interval I 11. /J J and if

( 14)

1hcn F(.r) i..; an a11tidcrin11fre of fLr): in 01herwords. F'(x) = J(.r) in the in1erval
(a.h).The fundamenlal lheorern of calculus give, u, the in\'ersc relation between
diffcrcniiation and intl!),!ra1ion. The u1ili1y of this tllL•orL·m cannn1 he ovcri-tatcd and
wa.s es~nrially unrl!cngnitL'd before Newton and Lcibnitz,
To ob1;ii 11 I (.r J from Equ:11 ion 14. we di ffl!rL·n1 ia1c with re peel to the upper
limi1, x. Write

F(.r + ~x) - F~x) = 1-•+,.\, j(11)d11 - f.- j(11)d11 = Jr . , f(11)d11


LI // \

Assuming that I (x) is continuous in 1he imerval (.r. x + ~x). we cm use 1he mean
value theorem of in1egra1ion 10 write

F (.r + ~.r) - F (.r) := J (~ l 6..r

If we di vidc by Ax and then lake the Ii mi I L\x _____,, 0. we obtain

dF I
- = F(x ) == f(x) ( 15)
dx

Equations 1.:1 and 15 'iumrnarize 1he fundamental theorem of calculus,


lncidcn1ally. we can generalize 1hi;: above differentiation to 1he case where
bo1h limi1s arc function-. of .r. If

G(x)=j·••w j (l)dt
Ul 1)

1hen ( Problem 9)

, dv du
C (x) := ((t'(x)) - - J(11(.x)) - ( 16)
· dx dx

Equa1ion 16 is called u:ilmir: ·s rule. Of coursi:. we arrc as~uming that 11(.r) and
11 (x) are suitably di ffercnt iablc.

gh
44 Ch,1pter I / Func tion o f a Singl , Va1iable

Example 3:
Suppose that

Evaluate l'(x).

SOLUTION: Using Equation 16,

, I I I
I (.r) = ---;;
x-
-2r - -
X
= X-

In your calculus course. you probably spent weeks learning how to evaluate
integrals. We start with a fairly complete table of derivative formulas and use Lhe
fundamental lheorem of calculus 10 find antjderivative.s. There is a bewildering
array of techniques or tricks that can be used to manipulate expressions into forms
that can be recognized a., ant iderivat.ives. For example, we have integral ion by parts
(see Problem 2). trigonometric substi1u1ions (Problem 3). partial fractions. and
·•mi~cellaneous" substitutions. With enough practice. mos1 students can become
pretty proficien1 in these 1echniques, bur integration is still somewhat of an art.
Although our derivative formulas allow us to evaluate derivatives of any
(differentiable) functions. Lhcrc ucc many intt:grn..ls thal can not be expressed in
terms of known functions. We"ll see in ChapLer 3 1ha1 a number of well-known
functions are actually defined in lerms of inlegrnls. For example. we·11 see that I.he
error function. erf (x). is defined by

3/2 u To evaluate crf ( 1.5). for example. we dcrcnninc the area hc1wcen the x axis
and the graph of e- 1? from O to 1.5 as shown in Figure 1.53. 1l1crc arc many
Figure 1.53 numerical routines to do this. You may remember learning 1he trapezoidal rule or
The area bc1wcen the 11-a.-.;is and 1he
g raph of e - ? from Oto J/2 is equal to Simpson ·s rule in your calculus course. Basically. these me I.hods provide numerical
r.J/- _.,:d approximations for areas. We"re nor going to discuss numerical melhods here
-O e "·
bcc:ause there arc many compu1cr packages avc1i lable nowadays that use much
more sophisticated routines than Simpson's rule. For example. I.he commercially
available computer programs. Mathematica, Ma1hCad. Maple. and Matlab. can be
used to evaluate integrals numerically. The one-line c.;ommand in Mathemalica

Integrate [ Exp [ -x • 2 ], {x, 0, 3/2} ] // N

gives

3/2 ,

10
e- 11 • d11 = 0.856 188
1.7 11111•;.:ratic)!I 45

Ccr1ainly owning a good 1able of integrals goes a long way to being able to
evaluate many integrals. The CRC Standard Mothemmical Tables mu/ Fvr11111/ae.
\\hich is a standard reference. lU !. ewer 50 pages of integrals. The mos! compre-
1

hensive tables (over 1000 pages!) are t;hc Tables of lntesra/3, Series. and Producr.~
by Grad-;hteyn and Ryzhik. which are indispensable for anyone who works in ap-
pli~d mathematics. In addition 10 Lhe~c- -.tnndard references. many of the CAS can
be used to intcg;ra1c symbolically. meaning 1ha1 1.hcy provide analytic expressions
for in1cgmls. For example. 1hc one line in MaLhema1ica

Integrate [ x~3 • Cos [ a • x ], x

gives the indefinite integral

• _ 3(a 2.r 2 - 2) cos a.r x ({rx· - 6) sin ox


x 3 cos ax d.\ - ..s +
/ n a3

and

Integrate ( x • Log [ a • x + b ], {x, 0, ~} ]

gives the definite integral

1
2ab- a 2 + 2b 2 In b + 2(a 1 - b2 ) ln(a + /,)
1 _
x \n(ax + b) dx =
4w
,.

01her CAS. such as Maple and Matlab. have the same capabilities. Almost every
academic science or engineering department or industrial research laboratory owns
a license for at k:i,1 one of Lhese CAS. Learning 10 u~e any one of 1h~,e programs
will not only save you hours of algebra. along wilh its concomitant errors, but will
also allow you to concenLrate on the central aspects of a problem r-.ither Lhan on
drudgery.

1. 7 Problems
I. Derive 1he fom1ula for integrarion by puns.

2. Use integration by pilll.'- to cvaluale

(a) J.\t' 'dx (b) / xsin.xd.r (d) / ln x dx

3. l 1,e 1rigonomc1ric suhstirurion to evaluate


. dr •(J J
(a)
)
.
Ja?. -x2
(b)
io Ja 2 +x~
{ X

4. Use hyperbolic substitution to evaluate

(a)/ Jx -- a- ~x ,, (b) f'(.,..2+l)1,2d.r


lo (c)
•I

Jo &>
d
(d) lo
rl /2 dx
1- x 2
46 Chapter I / Fun li on:- of a ingl • V.i ri Jhl

5. Find the area bounded by y = 2r and y = .t 2 from x = 0 10 2.


6. Find the area between the curve f(.r) = a 2 - x !. -a~ x ~a.and the .r axis.

7. Find the arc lengt.h of y = .1: 2 from x = 0 10 2.


8. Find the volume or a right ci ri:ular cone of base rai..lius r ani..l height h.
9. Prove Lcibnitz ·s rule ( £c.1ua1ion 16 ).

10. Show that f" -,\


f(.r)dr = 21' ()
1
/(.r) dx if /C.r) = .f(-x) and Iha! ['' /(x) dx
}_,,
= 0 if /(x) = - f(x).
, J
. x- v- .
11. Show 1ha1 tht: area of an ellipse. ---;;- + :...-, = 1. is rrah.
a- b-
12. Prove 1h::i1 lhc sum of the tirst 11 intc:ge~ 1)- 11(11 + 1)/2. Hi111: List 1hc 11 integers :,.cqucntially on one lint· .•rnd
under that line list rhem backwards. Do you see the trick here'!
13. In this problem. wc·I1 prove that the .~um of the tin-i II squares is n (11 + I )(211 + I )/6. Begin with the formula
(tJ + 1)-' - v 3 = 3 ~ 3 v + I and sum from I' = I 10 11 to obtain (11 + 1) 3 = 3 S1 - 3 S1 11 + I where

=L L /-. Now show that S~ =


t/ "

S1 j and S2 = 11(11 + 1)(211 + 1)/6.


j=:I i=l

14. Can you generalize Problem 13 to tind S_1'!


15. Prove rhe s1a1emcn1 of Equation 7. Hinr: Use Equation I.

16. Show that the sum of the first II odd integers i!- n ~. Hi11r: Evaluate I)2J - I) using the fact th;11
1~1

L 1= 11(11 + 1)12.
j =I

17. \ Ve 'II prove 1he triangle inequality. Ix+ y I _ I x I J y 1- in 1his problem. The triang le inequality i, used
often in applied mathem::i1ics. Add the inequalities -I .r I _ x ~ I x I and -I y I ~ y _ I y I and then t;1ke the
absolute value to obtain Ix + y I ~ Ix I I y I,
18. Prove the s1a1emenl of Equal.ion 8. Hi111: Use &7ua1ion I and Ihe 1ri angle inequal i1 y. I " + /J I ::::; I II I b I.
(Sec the prc\·iou~ rrnblcrn.)
19. Consider Figure 1.54. which shows I he hyperbola .x 2 - y 2 - a 2. Show tlrnt the area nf 1bc shaded region
is A =a 2 Jn Xo +Yo. This area divided by a 2 is a hyperbolic radian.
{/
which we denote by u, Show tJ1a1

cosh I/ ::;: -
x an d sin
. h II ,. H ml:
= ;_. - ll se I hC tac,
• rhal Xij -
, ' (.ro + YoH.ro -
Yo= ..,
Yo) = o·.
a a

Figure 1.54
t\n illu.'-lratinn o a hyperbolic r.idi:111.

g J
1.8 Improper lnt gr,11 47

Figure 1.55 .x x+ h x
A pinorial aid for Prnhlcm 20.

20. TI1is problem offers anmher proof of the fundamental theorem of calculus. Use Figure I .55 to show 1hat
F(x + h) - F(x) ~~ [J(x +I,)+ /(x)] for small values of I, and that F'(x) = J(x).
1-r sin 11 2d11
2 I. Evaluate -~~o CJ x3

· 'T / :. do F. . ., 0 I ,
i
17 .
22• Show that ,, _,, ., = _ ,, _,, . H111r: 1rs1 wn1e cm,- = 1 sec fl. no1icc that
o a-+ b-- co-.- 0 21ul(a- + b-) 111"'
d tan 0 = sec2 A d0. and then use rhc identity sec:? 0 = 1+1an 2 8.

Use tm_r CAS to t'1·a/11ate tire i11tegru/.1· in Pmhlem.1· 23 through 27.

23. f x\,--' cos dx

24. f (.r2 - X
X
1)1/ 2
dx

25. f xJ ln(ax + h) dx
x~ + 2r - 4
26.
f - - --(IX
x 2 + 3x + 2

27.

1.8 Improper Integrals

There are two types of integral!-- I.hat are called impmper integrals. The lirst type.
Type I. consists. of in1egrals with one or more infinite limits. which arc denned by

( I)

(2)
48

and

In Equation 3. ::: is any convenient poinl that you are free to choose.
The second type of improper iniegral. Type 2, consists of integrals where the
integrand" are unbounded at one or more poinis in the region of integr.11ion la, b J.
Three ca ..cs can occur: j(x) is continuous everywhere in r o. h I except at a, in
which case

fh
l o
b
f (x)dx = lim
'· •O. t:1 +-t
f(x)dx : (4)

f (.r) is contjnuous everywhere in [a. b I except at b, in which case

h
j(.x)d:r =,-0
lim
11>-1- j(x)dx; (5)
1 11 /1

and f (x) is continuous everywhere in [a. b I excep1 at some point c in (a. b), in
which ca~e
h
Jim
f'! 0 J (2
J(x.)dx (6)

provided that both limits on the righl exist.


We can also have improper integrals I.hat are a corn bi nat 10n of I.he above types.
where the integration limits are infinite ond the integrand is unbounded. When
the limiting process defining an improper integral exists. the integral is said to
converge. or to be com•ergem. Otherwise, lhe integral is said IO di\'crgc. or to be
di1·erge111.

Example 1:
E,camine the conven:?.cnce of f,x, d:c as a func,;1ion of p.
- 11 x 1'
SOLUTION:

f ""I·· - = I' ft,/ = [ 1·


I
t .\
xi'
im
,, __, - I
l X
-
x/1 h
1m
I I ]h
1- pxP - l I

If p > I. 1hcn we have

.
llm - I- ( - -
I - 1 ) = - -I ( 0 - 1 1 = -
I-
I - p hi' 1
b-")() 1- p , l- p

lf p < I, then

1
Um - -(1, 1-p - I) = oo
b-x I - p
1.8 lmprop r lnt r.i l 49
If p = I. then

J 1
oo dx .
- = lim
xf' ,,_
fI
h dx
-
X
.
= lim In J, =
h-oo

So we see 1ha1 f l
00 dx convell:!e if p > I and diverge if p :£ I. This result
xP ~
is worth rcmcmhering.

Example 2:
Examine 1he convergence of 1 00

c-udx as a func1ion of s.

SOLUTION:

. e- .ra - e-sb
= l hm .1·
(s > 0)

but equals oo if s= (I or ifs < 0. Thus the integral converges ifs> 0 and
diverge~ ifs ~ 0.

Nole that Equal.ion 3 does not say that

! oc,
'- /(x)dx = c~~ f' -c j(.r)dx (not 1rue)

oo udu
For example. consider / =
consider lhe two integrals
J-~--
. ·~ I+ u 2 ·
We choose : = 0 in Equation 3. and

and .
I1m
b-•!lO.
£"
n
-11du
--
I + ,,2

Let 11 ;:::: -x in r.hc fln;t integral 10 obtain

-
'
. 1'
ltm -
1
xdx
-
" o I + x2
=-
11- • -
.
hrn -I In(!+ a-)
2
., = -

I .mtcgra 1
so 11c f 'X)

-:x.
-udu
-=,, d.1verges. N
l + u-
I ' I h . .
1 ow et s ook at t e integral IIom Oto oo.

b ud11 . I ,
b
lim
. l --,
,1 I + u·
= llm -
1, ::,;:; 2
ln(I + h-) = oo
50 U,.,p!t•r 1 / Fu11ctions of a Single V,1ri.ihl._.

Thus, h01h con1ribulions 10 / diverge. (f we had u.sed

)~":-x. i
-a I
ll lldll

+ 11 2

Ihen we would nave obt;:iincd a value of zero I~ In( I + t/) -1 Jn( I + a 2 ) I for the
in1cgral.
ll is useful to have snme 1ools 10 detem1ine e,L<.ily if an integm.l converges
or diverges wiIhout having to evalua1c i1. If f(:r) and g(x) arc bounded and
0 :5 f(.x) :5 g(x) for x.::: t1, Ihcn Jt
J(x)dx convergc-s if Jt
g(xldx converges.
Comwscl:,i. if J(x) ~ g(x) ~ 0 for x ~ 0. 1hen ,(,~ J(x)dx divergc:s if ~(x)dx J/'
diverges. This test is called the comparison resr (Problem 22).
To see how 10 u.-.e the comparison text. let's investig..11e lhe behavior of

for O ,:5 X < ~

Using the fact 1ha1

we conclude 1lrnr / converges. Ln fact. iLc,; numeric.al value is 0.403 653 (Prob--
lem 23).

Example 3:
Show th,11 1he integral

converges.

SOLUTION: We lei f(x) =sin 2 x/x;i. and g(x) = 1/x·1. Then because
sin 2 x ~ I for all x. we have f (x) ~ g(x ) for all.\. ll~ing the foci tlrnt

we conclude 1ha1 the in1cgr.1I in qucsrion converge).. In fact. its numerical


v::iluc is 0.385 705 (Problem 24).
51

We also have a ratio comparison tesr for an integral such as

where f(x) ~ 0 for a~ x < oo. Now find a function g(x) ~ 0 for a_::::: .r < oo
for which the integral Jt
g(x) dx is readily evaluated. Then the test for Lhe
convergence of/ = ft
f (x) dx rests upon 1he limit

K = lim f(x)
.r g(x)

Three cases arise:

I. If K # 0 and is finite. then 1 00

f (x) dx and 1 00

g(x) dx either both converge


or both diverge.

2. Ir K = 0 ancl 1,=
ll
g(x) dx convefEes. then
l
f
00

f (x) dx converges .

3. If K :::::; oo and 10::.• g(x) d.x diverges. then .lo,;) J (x) dx diverges.

Given an integra.l f 0 J (x) d:r, it is often easy lo choose an appropriate function


g(x). as the next ex.ample shows. Let's invesLigate the convergence of

I= f
1
C<J f(x) dx

= f ro
11 (.\"
,,
l
x :?
+ I) 1/ -
,dx

J (x) as x .....;, oo. In this case. f (x) - I/ x


The ratio K involves the I imit of
as oo. so we'll choose g(x) = 1/x. in which case K = I. B.~1 we know
x ➔
(Example I) 1ha1 ~1 dx/x diverges. and so our 1es11ells us that fI - 6 ~! ,,.,dx
00
.. <x I J -
diverges also. The nice feature of this test is Lha1 you need only Lhe behavior of
f (x) a1 large values of x. which is oflcn fairly easy to see.

hample 4:
Use t..he ra1io comparison test to show that Lhe integral

= Inc.:; f(x) dx
1
00 .r2
I = dx
o x 4 + x-1 + 1 o
converges.

SOLUTION: Because /(x) becomes l/x 2 n.s x gets large. we cli1oo-.e


g(x) = \/x 2 . Because J;~ x- 2dx converges. we conclude lhal / =
·oo x2
Jo --1--,-dx converges .
•1 +x·+I
52 Chap! r I / fu n lio of J Sin :I V,iri, hi ·

Ano1hcr useful tes1, which is clo!-icly related IO 1hc previous test and is based
on lhc resull in Example I. is the following:
If lim xi'f(x) == K. 1hen
x-cx:.·

I. j ~ f (x )dx converges if p > I and K is. h nitc. and


2. J,,., j(x)dx diverges if p .:: 1and K :./a 0.

We shall refer to Lhis test al- 1he JJ li!II for Type 1 improper imegml~·- This test is
particularly easy to apply.

Example 5:
•,x lnx
lnvc.stigatc the convergence of
J 1
--dx.
x+a
SOLUTION:

In x
Jim X·--='"'-
.1 X + (I
Thus. p = I and K "/= 0 in lhc p 1csl and the integral diverges.

If.(.,._, lf(x )ldx converges. then_('· f(x)dx is said to he al,.w/111ely con\'C:r-


ge111. If J;, lf (x )ldx diverges. but J/
I (x)dx converges, lhen J;;'- f(x)dx is
said 10 be crmditionally com·l:rf.?1'/lf. If J.,''-- j(.r)dx is only conditionally conver-
gent, it converges because or i.:ancellalion of positive ,md nega1ive contributions to
the intcgr:1I. Clearly, if an integral is :.ibsolulcly convergent. lhcn it is convergent.
An example of an intcgral that is condi1ional ly convergent is J1'" ( sin x >/xdx.
So far we have discussed convergence only for Type I improper integrals.
1hosc whose limits arc infinite. What abou1 Type 2 improper in1egrals, those 1ha1 arc
improper because the integrand is unbounded in I a. b I'? The tcs1s for convergence
are no! very different from the ones above. For simplicity. we'll just slate 1he p
rest for Type 2 improper integraJs for the ca.c.e in which j(x) is unbounded at a,
but the other cases are essentially the same. Let lim (x • a) 1'f (x)
x-11+
K. Then =
/_,
I,
1 0
1,
f(.r) dx converges if p < I and K is finite

2.
1 rl
f (x) dx Jivergcs if p ~ I i.lJld K i=- 0
I .8 lmprop"r ,Integral 53

Example 6:
3
Investigate tJ1c convergence of
li
1
dx
J(x - ))(3 - X)

so LU TI o N: This integral is unbounded at both limiti-, o lel·s write ii. as

f· f 1·
1 1 5
dx dx dx
(x - 1)(3 - x) = . 1 J(x - 1)(3 - .r) + ~ ✓(:r: - 1)(3 - x)

and work on each one scparJtcly. Because

· I/"' I
lim (x - J) --;::==:::;::== = K =----;;;; < I
x-1+ J x - 1)(3 - x) ✓2

and

lim 3 - x ) 1f 2 I = K = - 1- < I
., - J<x - 1)(3 - x) J2.
both integrnls con erge.

1 .8 Problems

I. Evc1lua1e l
•0 x-
dx
+I
2. The square-well po1enIial for the interaction of two sphcric;;ally .symnu:tric rnolcculc~ -.cparatcd by a distance
r is given by (see Figure 1.56)

r < CJ

11(r)=1: a < r _ i.(T


0 r > >.er
where a. >... and e are con,tonrs Ihu1 nrc char.1c1cris1 ic oft he molecule. The second virial cocrncicnl of impcrfcct
ga., 1hcory is

where k 11 is the Boltzmann constant and T is the kcl,·in temperature. Derive an expression for B(T) for the
square-well potential

11(r)

0
Figure 1.56 r
The ,qu;lfl'·\~cll potl'llliaJ for the
interaction of 1wo ~rhcri~·:illy $ymmc1ric
111e1kt·uk~.
I
54 ( h,1ptt"r I Functions of a Single Variable

3. Evaluate 1: 1
sech xdx .

dx
4. Evaluate
1o v r,-- ..,..
I - x-

1 dx
5. Evaluate
1 0 J1-x·
12
6. Evaluate [1' sec xd.x.
le,
7. Use the compari:,.on test to show that
!"'
1
,,
dx
x · +xii-
, conve!"'6 es.

./x
1
N
8. Use I.he comparison 1csII0 show 1ha1 - -·-dx diverges.
I I +x

9. Use 1he comparii:on lest 10 show that f':.J ~ diverges.


}, I +x

.
10. Use the companson 1es110 show thal
r.
, I
00
I+ cos.r
.t ·1 +4
dx converges.

11. Use lhe ratio comparison test to show lhat (;o x , dx diverges .
Ji (x.i + I) 11 -
12. Use lhe ralio comparison 1es1 to show that J 00

(xl'> +
x
I) 112
dx converges.

13. Use the p teY.l


. .
for Type I unproper mlegrals 10 shO\V that
1~ I (x 6
x2 + I ., d.r diverges.
+ I) 1/-
.

. . foox2dx
14. Use the p lest for Type l improper mtegrnls to show that - 4- - converges.
I X +I
15. Use the f' test for Type 2 improper inrcgrnls to show Iha1
1 4
oo
x-1 J(x - 2)(x - 4 )
., ')
dx
dx converges.

16. Use 1he p 1esI for Type 2 improper imegrals to show Iha1
J l (x -
~- + -113 dx
1)
diverges.

·r . .
I 7. Show that
l 0
b d:c
- - - converges I p < I and diverge..-. 1f p
(x - b)P
~ I.

18. Show that fo 00


e- :,,.fud.x converges.

11
sinx
19. Show that
1 0
--dx converges.
X

. ·rfoi..,inx
20. Determine I Jo 7-dx converges.
00
sin.h ax .. . .
21. Show that
1 o
-_--d:r converges 11 0 < o ::: :rr and diverges 1f a _ rr .
srnh ,r x
22. Prove 1he compariwn 1es1 for improper integrals of Type I.
1.9 Uniform Con1,1erg('nc c: of lnlcgrals 55

Use any CAS ro evaluate rhe integrals in Problems 23 tl,rough 27.

23, 1,
o
e- x
(I+ x)2
dx

sin 2 x .
f
oc
24. - -- dx
1 X3

25.
i In x
(I+ x)2
l
dx

26. e- a
lo cos bx dx

27.
1 , - nrr-b1 /.ridx

1.9 Uniform Convergence of Integrals

Integrals that arise in physical applica1ioos are often functions of one or more
parameters. For example. later on we· IJ be using Laplace Lransforms to solve
diffcrcnlial equations. A Laplace tram,form is defined as

where s is a parameter that we can vary and manipulate.


We can express the general sirualion as

F(x) = J
11
00

f(x. r)d1 -"1 .S x .:S X2 (I)

When. for insrance. can we determine lim F(x) by writing


r-.•.•o

100 Jim
Jim F(x)
x-•.ro
= s-..r
lim
0 1
u
:,c.
j(.t. r)dt = u x-·•.111
J(x. t)dt ? (2)

Or. when can we find Lhe derivative of F(x) by differeniiating f(x. 1) under the
integral sign. as in

F'(x) = 1oc• ~f(x. I) dt? (3)


I/ dX

Since F (x). being an in1egral. is defined Lhrough a I imiling process and since the
derivative of F(x) involve~ anolher limiting process, we must wonder just when
these limiting proce,•;;ses can be interchanged.
Before we go on. we should mention a nice anaJogy concerning Lhe inter-
change of limits that is due to 1he 20lh-century Russian mathemat.ician, Vladimir
Arnold. Suppose we have a container of water and that there is a hole of radius r
56 ChJpler 1 / Ftin 110n o f i ngl • V ri, bi •

in 1hc bo11om of the container. Now consider 1he limit

lim {.imount of watcr in the container)


I-X.l'-0

If r ➔ 0 before r- -x,_ then there will be water left in the container. If. on the
01her hanc.l. r ➔ -x.., hefort! r ➔ 0. then therc will be no water left. This example
shows Lhat the order in which we take limiti; is cnici ..11 and thal obtaining 1.he ~me
resuh upon intcn.:hanging 1hcm is in no way assured.
To ac.ldrc~s the 4ue~1ions illuslrated by Equations 2 and 3. we first define the
notion of 1he 1111ifon11 com·erRence of improper imegmls. First. lei"s suppose thal
F (x) given by Equal ion I converges for each .r in I .r 1• x 2 ]. In other words. suppoi.:e
that

IF(x) - [ f(x. t) dt I« whenever h > N(e. x) (4)

where N is a number tha1 depends upon f and x with x 1 :::: x.::: x~. Equ:nion 4
is the fom1aJ way ot' expressing that F (x) converges for each x in I x 1• x'.! 1- Let ·s
suppose. now. 1hat Equation 4 is sillisflcd for a number N(€.) that depends only
upon f and not upon x. In other words. suppose that

where N ( ~) is i11depe11de111 of x. In lh is case. we ~ay that F (x) converge~ uni form ly


in I x 1• x:d- For example. lhe integral

c.:onvcrgcs uniformly to 1/x for x ~ I because

X
-1 - l"
O
-n
e · dr I ,,-.d, ::S"' -h
= --
t=

X
1

and e-" will be<€ if we choose N to be In 1/€, independent of.r for x _ I.


On the other hand. 1he integral

G (x) = [7'".) Xt -.tr dr


lo
converges for x :::_ 0. but does 1101 converge uniformly for x ~ 0. To see why this
is so. first note Lhat

= {~
x=O
G(x)
x>O
and that
1.9 57

.r =0
x>O

The i ntcgr.:i 1 is II n iform I y convcrgcnI for x _ x 1 > 0 because e-.rh :·: l' ' 1" < f if
we choose b ·::- N =
I/ .1· 1 In (I/ f ). As x 1 __,. 0. however, N - oc and so I he in1egrnl
is not uniformly comergcnt for .r ~ 0.
The following theorems Lell us why uniform convergence is so important:

I. If f (.r. t) i.1· co111i1111011s Jon 2: a a11d .r 1 _ x _ X 1. and if L.,._ f(x. t )d r conrn~t· 1


1111ifon11/y to F (.r) in I .r 1• .r~ ]. 1/icn F (.r) is n1111i11un11.x i11 //}( imN1·0I I .r 1• x~ i
Tlii.1 re.wit a/1011·x 11s ro ll'rirc

(5)

2. If f (x. I J .w11isjies !hi: ahnl-'l' cmuli1i11ns. lht'11

f..
~
("''
.I J
F(x )dx = r·l' [f
• l I <I
:X. J(x. r )dt ] dx = 1(\.. [
lJ ~
f, l I
J (x. r)dx ] di (6)

/11 orha 11•1ud.~. ire cm, i111ercha.11.f.tl' rhe 1H,kro_(i111egrmi,m.


J. ~(f (.\'. n w,d fJJ/ dx 11rr.: /'()llfill/10/1.l'fort ?=_ LI mu/ .\' I _ X ,'.:: .1·2, if_/~"'C J(.r, I )di
C0/1\'t'rg,•s lfJ F (. ) in I xi • .r~ ]. miff if .{ : iJJ / iJx di cmll't:t~(•',1' 1111ifnr111/y in
1 X1. x2 J, then

F
.
trJ = f -- - -,,
I/
rJf(x .1) I
,I.'(

Note that this la..;1 result requires more off (x. r) than do the tirs1 two theorems.

Example 1:
Show tlrnt

and explain why.

SOLUTION:
lim { x,, - ·' 1d1 = lim I= I
- •I lo r-•O
anti

The two limit" arc nm nccc,,arily equal because. as we showed above. the
integral is nn1 unifom1ly ~on-..L"rg,cnt for .r :=. 0. sn we shoulJ nol cxp<,"t:l lhal
Fix) will be con1inuous for x _ 0.

gh al
58 Ch.i JJlt:r 1 / Functions of a, Single Variable

To effcc1ively use the above rheorems. we need a sirnplc lest for the uniform
convergence of an in1egral. Let f (x. r) be conlinuous. fort 2: a amd x 1 :S x :S x 2 .
Now if you can find a funclion M(t) continuous fort?: a, such that l f(x. t)I ~
M(t) for r?: a and x 1 ::;'. x ~ x 2 , and if fu M(l)dr converges, then fa f(:r. t)dt
00

converges uniformly for x 1 :S x :S .x 2. This 1es1 is known as the Weierstrass M test .


l11e proof is fairly easy. Let F (x) = ft
f (x, l)d L Then.

IF(x) - t /(x, /)di I=11"' /(x, t)dt - [ /(x, t)dt I= 11~ /(x, I) dt I
100 M(r)dr
<
l
•h
:xi
1/(x, 1)1 dt ~
h

This last integ-ral goes 10 zero as h ➔ oo. independent of x. ~ lhc t.heorem is


proved.

Example 2:
Use rhe Weierstrass M test to show 1ha1 F(:<) = Jo[' ,
c.,-.r,- dt is uniformly
convergent for x 2:. a > 0.

SOLUTION: ff we choose M(r) e- r11 = 2


• then f(x.1) = :s
e-·0 ! e- a,
2

fort :::, 0 and .r :-~ a . Bccau~c e - at\Jt fo= = (rr /4«) l/~ is finite for a > 0,
F(x) = J0=e-_,,~d, converges uniformly for x ~et> 0.

Example 3:

1,
C\) '
Show 1ha1 G(.x) = t'2.e-.n- dt is unifom1ly ~onvergent for x ~ et > 0.
0

SOLUTION: Choose M(r) = t 2e-cn because j(x. r) =12e-.r12 ::::; 12e-a 12


fort:> 0 and x ::-: er. Bec.;ause J~"' r2e-a' ~d, =rr 1l 2/ 4<Y 3/ 2 is fin.iIe fora> 0.
G(x) = f000
r 2e-M: dt converges un..iformly for x :::, lY. > 0.

Example 4:
Evaluate F'(x) in Example 2 by differentiat..ing under Lhe integral ~ign and
then inIegru1ing and compare your result to the one you get by differentiating
F(x) af1cr tirst evaluating the inIegraJ.

SOL u TIO N: Differentiating under rhe inlc)!ral ,ign and then i ntegrati.ng
gi,•cs
1.9 Uni fo rm Com er •en e of lnt ·gr.:ils 59

If we evalua1c F(:r) llrst, we obtain F(x ) (;r/4x) 112 , which gives =


F' (x) = -ft 1-'~/4x 3/'!.. in ngreerncn1 with 1he fi rs1 resul1. We. obrni n the
2
sumc re:-iuh bec;ausc e - .rr and t 2e - x 1" are continuous functions of x and
, and ft~
r 2e- 11 'dr converges uniformly for x > 0. (See !he previous
Example .)

Before we leave this section (and this chaplcr) we should mcmion some
corresponding results for integrals with fini1c limits .

I. If f (x . 1) is ronrin11011s i11 the rc·cta11:~lc a :::: r :::: Ii, x 1 _:s .t _:s x 1. rhe11 F(x) =
J,;' f (x, I )di i., ('{))/lill/10/IS )or x , .'.:: X _:':: Xz. This thenrem o/1,rn·s JI.\"'" \\"ril

!J,
lim
x-•.10 l"
b
/(x. l)d1 = ltm f (x. l)dr
o .1-•.ro

2. Under tht· co11di1io11.1· of the previou.l· 1l1t·orem. 11·e /rave

1-"
-~I
F(x)dx = 1·r. [J" .Hr. ]
_rl U
r)dt dx =
~IJ
fb [ frr· J
•XJ
(x. t)dx ] dr

3. If f (:r.,) and of (.x. 1 )/8:r are co11ti1111011s i11 the rl'ctanglt' a ~ t ~ b . .r 1 :5 x ~


x 2 , then

F
, (X)= [1, Af(:x. l)d t
- --
• r. i)x

You can easily 1cs1 Lhcse three theorems with F(x) = .f01 t'-.x,dt (Problem 10).

1. 9 Problems

I. Show 1hat 10...• t·-x dt is unifom,ly l'Onvergeni for .r ~ u


1
> 0.

2. Sho"' that fooc ,,,,. r1dt is unifom1ly cOn\'ergenr for x ~ tJ. > 0 where II a positi\ C integer. 1

3. We showed 1hat ix,


Cl
ti.-.ridr = 1/x is unifonnly convergent for t ;~et> 0 in Problem I. Show 1ha1

1
0
I t'
11 .!." Id
t = -11!- .
xn I

4. Prove 1ha1 fox ,,-.ri cost dt converges unifom1ly (and absolutely) for" ::: x _ b. where O < a < b.
60 C'h,1p1i·r I / un io ns of,, Si ng ll' \i,'lri,1hh·

5. Example 2 ~hows is uniformly convcrgenl for x ~ a > 0 . Show that

cos O'.\"
1
,X, • •
6. Show 1ha1 - ,- 1s uniformly convergem for all real values of Ci..
o ., .. -1- I

7. There is .i standard Irick 10 evaluate fooc c- 1


• cos etx d:r. Differentiate with respect 10 et. integrate by parts.
2 4
and notice Lhe result. The answer is (rr 1 -J2) e 1

8. Show 1hu1 la''° e-a., cos x dx is a continuous funclion of a for a > 0.

9. Does
1 O
00
dx
---
1 + x:?
= .
hm
u ·O
1o
00
co-; ax dx .,
----,-.
I + .x-

lO. Verif)• the last 1hree theorems ut the end of the section wi1h F(:r) =[ 1 e- r,d, and O::: x:::: I.
. ()
ll. Show lhal Jt(sin x)/x dx = rr /2 by wri1ing
, -·i,jn-dx
x = 1"'.3 sin x (ioc t!-.i, dr
) dx
10 X O 0

and then inten.:hanging orders of inIcgraIion.


12. Show Lhur for:.:, c - ,u (sin x )/x d:r = co1- 1o by writing
,:.; . .· 1oc
1 0
,,
-a·
r sin .\' d
- - ,\'.=
X 0
e - n.r

and inierchanging orders of integr'Jlion .

. . ,, , ,
13. Eva\ua1e the integral in the previous problem by differentiating with respect to a.
7r 1/2
14. Show that /(a. I,)=-=
o 1 l.'
2a
:r = b/oJ.. and lh~n integrating with respect to b.
-,r. ·- t,· / r· ,1x = -' -e- 2" 1' by differemiaring with rcspccl to b. then k:Hing

15. Given 1ha1 f..._,,, "cos xu du


Jn
= --1- .,, show 1ha1
I+x-
1= o
ue-u sin x,, du =
(I
2
\'
+ x 2)-
., .
61

References

CALCULUS TEXTS:
Frank Ayre~. Jr.. and Etlioll Mcndelwn. 1999. Calrnl11s. 4th ed .. Schaum ·s Outline Series.
McGr.1w-Hill
R. Couranl. 1970. Dif{ercminl and ln1t·Rral Calculus. 2nd ed .. Inicrs.cience, John Wiley
C. H. Edwards, Jr., and David E. Pc-nncy. 1998. Calcul11.t and A11aly1ir Gt·nemtry. 5th ed ..
Prcn1 ice-Ha.I I
Witold Kosmala. l 999. Advanced Calculu.~: A Friemlly Approach. Prentice-Hall
Jerrold Marsden and Alan Weinstein. 1985. Calc11/JL1· I, II. and Ill. Springer.Ycrlag
Richard Si Ivcnnan. I 989, /:".1.11•111iu/ Colrnlux. Dover Public::uions
Murray Spiegel. 1963. Adwm1·1•d Calm/us. Schaum ·s Outline Serie.~. McGraw-Hill
Dlivid Widder, 1989. Ad1·a11ced Calc-11/11,1·. 2nd ed .. Dover Publica1ions
MATHEMATICAL TABLES:
CRC Standard Matht·nwtical Tal>lcs and Fon1111lai..', JOlh ed., edited by Daniel Zwillingcr.
CRC Press (19%)
Tables af lnte~mls, Series. n11d Producis. 41.h ed .. I.S. Gradsh1cyn nnd I.M. Ryi.hik,
Academic Prcs!i ( 1980)

COMPUTER MATHEMATICAL PACKAGES:


!'vlATHEMATICA. Wolfram Re.search. Inc .. Champaign IL. www.wolfram.com
MATLAB. The Ma1h\Vorks. Inc., Nnlick MA. www.malhworh.com
MAPLE. Wa1erloo Maple. Inc. Waierloo ON. www.\\>alerloomaple.cum
MATHCAD. MathSoft. Inc .. Cambridge MA. www.nw1hcad.com
SOME GENERAt MATHEMATICAL WEBSITES:
www-history.rncs.sl-and.ac.uk (The www- is correcl.)
www. mat h. un,cdu/rnath/math-wch
www. mat h. u J)<:JIJI .cd u/M al hSou n.:e,. h 1111 I

GENERAL:
Da\•id Berlinski. 1995. A Tour of !he Calc11/11s. VinLage Book..;
Eli Maor. 1994. e: n,c SIOry of a Number. Prince ion Univer,ity Press

A HISTORY:
Carl B. Boyer. 1959. Tiu· History of tlw Ca/cullis 1111d /is Conceptual Devclop111l'l1t. Dover
Pub! icat ions
CHAPTER 2
Infinite Series

lnfinile se1ies play an irnporlanl role in applied mathematics. The use of infinite
series is a s1andard method for solving ccnain types of ordinary differenlial equa-
tions and many of the famous functions of applied mathematics are defined in terms
of infinite series. Bessel functions and Legendre funclions are just two examples.
lnregrals can often be evaluated by lirs1 expandin g lhe inlcgr::md$ in a Taylor se-
ries a.nd in1egra1ing term by term. Also. you may ha e heard of solving partial dif-
ferential equations by the met.hod of separat.ion of variables. This frequently used
method (and one rhal we shall study in detail in later chapters) uses various type!.
of Fouri1..~r ~l.'ries, which are infinite series in sines and cosines.
Jn Section l. we discuss infinite sequences and the ideas of convergence and
divergence. We di~u~s the convergence and divergence of infinite serie~ in Sec-
tion 2, and then present a number of tests to determine whether or not an infinite
!\eries converges. Infinite series whose successive tcm1s have ahanaring ~igns arc
called Jllemating series. the topic of Section 4. In this section. we'll learn about the
difference between an ahsolutely convergent series and a conditionally convergent
series. Then. in Section 5. we'll introduce the important idea of a uniformly con-
vergent series. Unifonnly convergent series have the convenienl propeny !hat they
can be manipulated pretty much like polynomials under fairly generul conditions.
The most commonly occuring infinite series in physkal problems arc power series.
~

which arc of the form L 11,,(x - c)n where c aml the 1111 are constants. It turns
n <=O
(IUI that if a power seril!s co nverges for values of.,· on I o, I> ). then ii convcrg~s
unifomtly wilhin the inrcrval fa. b ]. Power series ocrnr frequently because well-
behaved func1ions. like 1hosc 1hal occur in physical problems. can be expressed
as power series using Taylor's formula. We'll devdop Taylor serie~ in Section 7
and then present a number of applica1ions of Taylor series in Section 8. In the tin;il
section. we'll discuss asymptotic .series. which are quite different from power se-
ries in the sense 1ha1 asymptotic series are useful for large values of x and. in foct.
become better approximations as x increase ·.

63
64 Ch, pi •r 'l. I lnfinir ,

2.1 Infinite Sequences

An infinire sequence is a function whose domain is the set of positive integers. We


dc.signate an 1nfini1c sequence by s 1• s 2• SJ, ...• where s,, is called the 11th term. or
simply by l s11 I. For example, the integers them$elves. I. 2. 3, ... form an infinite
sequence. a,; do their reciprocals. I. l/2. 1/3 ..... Usually the 11th term will be
given a~ an explicit function of 11. such as 1/n or (-I)";,,:,_
Let's consider the sequence t(I - t) J. You can see that s 11 - . I as 11 - oo in
this case. We say that the limit of an infinite sequence is I if lim s,,
n• .. '-
= I. In 1erms
I + e I,--. ....................... .......
! , - - - - - - - - - --
......,_.._..._.._ of€ a.nd 8. Lhe sequence sn converges 10 the limit I if for any f. however small,
there i$ an integer 11 0 such lha1

11 > 11 0 (I)

I Tio n Pictorially. Equation I means that if we plot s 11 against 11, then all Lhe points for
,, > "u will lie within the band/±€ (Figure 2.1 ). If there is such a value of/, lhen
Figure 2.1
An illu..~trJrion of 1hi: convergence of a the sequence convl!rges: otherwise, it diverges. For example. I (-1/2)" l converges
sequence. to the Jri mit 0, bur {11} diverges because I im 11 = oo. The sequence I (- I) n J also
"·"'-
divcrgl!~ ~ince it does not .approach a limit/: ins1ead it oscillntes between + I and
-1. Thus, a sequence doesn·L necessarily have to be unbounded in order to diverge.
Nevcnheless, every convergent infinite sequence must be bounded.
We say that a sequence is botmdedfmm above if there is a number c such that
s,, ~ c· for all 11. and ho1Jnded fmm beloll" if .1,, :,:: b for all 11. A bounded ~equence
is bounded from above and from below.

Example 1:
Sho"' that 1he sequence {- " - } ii: bounded from nbove and from below.
II + l
. I 2 3 4
SOLUTION: The sequence 1s -. - - - ...• The tcnns of the
2 3 4 5
sequence arc increasing, monotonically becnu.,e

I- t - "+ I - _ ,_
,- - ,,2 +II+ I '• 0
s,,
.r,, ''' - 11 +2 n + I - (n + 1)(11 + 2) -

1.0 The limi1


. . n I
hm s,,
n-•::-c
= n-,x,
hm - - =
+I 11
0.5 •
so 1he 1erms are bounded from above by I. Furthcnnorc.

10 20 30 n
when n ~ I

Figure 2.2
A plot of 11 /( I + f!) against n for 11 _ I. so 1/2 is a lower bound. Thus. 1/2::: .f.,_::: t for all n (Figure 2.2).
T11e limiring value i~ ~hu'\\ n a" a da.,,hcd
line.
2. I lnflnire Se<1u 'n 65

A sequence is said lo be 1101ulecreming if s,1•• 12: ·,, for all"· It is 1110110/on-


icolly increasing if S11 + 1 > s,, for all 11, 11011im:reasi11g if Sn-I~ s,, for all,, and
111mwro11ically deuensillg ir s11 + 1 < s,, for all 11. Clearly. every bounded monotonic
sequence is convcrgcnl. Alrhough every convergent sequence is bounded. 1he con-
verse is not true: there are bounded sequences [for example. I (-1)" \ J 1hat arc not
convergent.
You should realize that the convergence or divergence of a sequence is not
affec1ed by adding or deleting a finile number of terms of the sequence. Conver-
gence depends upon the large II behavior of the sequence. or on the far "tail" of
the sequence. The criterion given in Equation I explicitly shows that convergence
depends only upon the behavior of {s11 I for n > 11 0 •

Example 2:
Dc1em1inc whether the sequence { '",," ] is incrc:1sing or dccrc:i-,ing for
large"· De1cm1ine i1s limit as 11 -

lnx . 1-lnx
SOL Ul ION: Let f(x) =-.Now f (x)
X
= --,-
x-
< 0 for x?: e. so
11
the sequence { -ln,-, } decreases tor
• 11 ~ 3. Usini;
. l'H.opI1a
. 1· s ru Ie. we see I.hat

its limit is zero as 11 - oc·,

2.1 Problems
2
J. Show 1ha1 lim 3',
611
+ 2 = 3.
11- 00 11 2 +1
r-
2. Show 1..hat lim ~ = 0.
11
- ·~ /I + I
3. Shmv th:it lim
n-•oc
l/ n = I.
4. Show rh a.1 lim cos
,r-- ...... 11
11
= 0.
5. Show tJiat lim ~ ·= 1) for " > 0.
,, •: ,C

6. Show that ,, lim


.... ~
0, = I.
7. Show rhal 11 !/5" is increasing for n 5.
2" r"
8. Show that - - 0 as 11 _,, ;X:> . Can you show that this result gcncrjlii.e1- to :...__ _. 0 as 11 --. for any value
11! 11!
ofx?
9. A simple method for showing whether a sequence is increasing or decreasing is 10 u~e a continuou~ function
/(x} such that _((11)-.,,. ~,, for 11 = I, 2, . . . and show that either J'(x) > 0 for x ~ I (an increa.-;ing sequence)
66 Chapter 2 / ln1i ni:-, Serie.

or that J'(x) < 0 for x ~ I (a decreasing sequence). Determine if the following sequences arc increasing or
decreasing:

(a) 4n - I } (c) (d) In - -11+1}


{ 611 + 2
r ,,
. defined by the recur-ion
l 0. Suppo~e that s11 1s . tormula
. x,,+ 1 = -I ( -~n + -A) . Show 1ha11f. .
Jun s,. =I e i~t·. then.
2 ~ " ~
I= Al/2_

+ F,,- 1defines the Fibonacci sequence. where each tern, is 1he sum of the
11. The n:..--cursion formula Fn+ 1 == F11
two preceding 1cm,_,;_ The sequence is I. I. 2, 3. 5, 8. 13. 21. .... The lim F, 1 docs nor cxis1 in 1his case. bur
11-c:,.:,
F
nssumc that lim ~ does exist. Show thal this limil is equal to ~(I + J5°).
" o::: F,. ~

12. Which of the following sequences converge?


11
co. h 11 (..,) "
(a) . ,, = -.- .- (b) s,, = -"-
s1nh 11 11
nl
13. Determine lim __:__
11-00 11"

14. Use the €-6 nol.ilion 10 prove lhal { ~} convcr_\.!~S to 0.

. . s,, 0
15. If lim s,.
n •
= a and lim
n
111 = b, 1hcn prove th..11 Jim s,,r,,
n ·•".
= ob and 11
lim -
•". 1,,
= -b
(I .
prov11Jcd thol 111 ,=- and
I, ,=- 0.

2.2 Convergence and Divergence of Infinite Series

An infinite series is an exprt's~ion of the form

oc

L "" =
11,ec]
"1 + u~ + "J + ...

The pnnial sums of this series are

S3 = " 1 + II 2 + 11 J
and the 111h parrial sum is

If 1hc sequence of partial sums converges. then the series is said to converge. or 10
be convergent. Otherwise. 1he series diverges, or is divcrgenr. If

Jim S,,
1/~'X:,
=S
then S is called the .,um of the infinite series.
67

The smndard example of an infinite series is 1he geomelfic series. whose 111h
panial sum is
,,
Sn = L rJ- l = I + r + r 2 + ... + r'i - I
j= I

Note that 11 11 + 1 = 11,1 r. h turns out that ii is possible (and easy) to obtain a closed
form expression for S,1 • Muhiply S,, by rand subLract the result from S11 10 get

S11 - r S11 =I- rn

or

I - r"
S,,=-- (I)
1-r

It's ca~y to see here that


2 - - - - -.- .... - ..... - -. - ... --

lrl < I
lim S,,
It•'\...
= 11 - r
00 lrl > I
5 IU
Note that S,, = 11 -'), oo as 11 - oo ifr = I and oscillates bc1ween I and O if r 'I. =-
Thus. the geometric series converges if lrl -c: I and diverges if lrl ?: I. Fi,gure 2.3 Figure 2.3
shows the partial sums planed against II for r = 1/2 . The geometric ~cries is of1en A pl'.ot of 21 l - ( 1/2)" ). the, panial ~urns of
wrinen as
the geometric s.e,rics for r = 1/1. ag.ain.~1
n. The limi ~ing \'alut: i.~ shm, n as a ilishc<l
IX.- linl".
Ex''== _I -1_ _ .t
lxl < I {2)
Jl ~ 0

Nore that we start the summation with a.n " = 0 1cnn in this case.

Example l:
The partition function of a diatomic molecule moddcd as a quunium-
mcchanicaJ harmonic oscillator i:-.
X,

q(T) = L l, - t" ½>lr ~ur


11-=0

where I, is I.he Planck constant, 11 is the frequency or t.hc oscilla1or. k8 is


the Bollzmnnn constan1, and T is the kelvin temperature. Express q (T) in
closed form.

SOLUTION: Facror ,, - h L•/ "!..lluT from the sum und lei r = e - h i•/ LnT < I.
Then. using the geometric ~eries
X,

4(1') ~ "-"l!':..1,r I: r"


11-1)
68 Chap1er 2 i' lni:i ni'P . •ri ')

e-l, 1•/ 21..nT


=I_ e - ln•/ 1. nT

We can use the geomclric series to show tha1 any recurring decimal ex-
pression mus1 be a rational number. Consider. for ex.ample. the decimal a =
0.0909090909 · • -. We c.an write o in Lhc fonn of a gcomc1ric series

Example 2:
Show lhi.11 a = 0.083:"33 · , · is a rational number.

SOLUTION: We wri1e a as

a= 0.0800 • · • + 0.003333 · • •

8 ~ I
100 -1'°'
L - 10'' J
n=O

4 3 ( I ) I
= 50 + I000 I - I/ IO = 12

It is easy to prove that if two series converge, then thei_r sum and difference
both converge. In fact, if

00

S;:;:Ls,, and T=Lr,,


n=I 11==:I

both converge, !hen

:x.
c1S + c~T = Lk1,~,. + 1.::!1,,)
11=1

where c 1 and c2 are constants. Also if L 11,, converges. then lll'n's.,arily 11,, ➔ 0
a" n➔ oo, To see 1his. simply write

T) 11--I
s,, = L llj and s,,_1 =Lui
j=I j=d
2.2 Convergence ,md Oiwr1_:1 'nc l' 01 ll1iinitl' \l'ries 69
ors., - Sn-I= "w Bur lim Sn= sand lim s,,_, = s. so
II • '-. II ·· • " -

lim
11-""-
''n = lim (Sn n-
- S,,_ 1) = lim O = 0
n-""'-

h is interesting and imponan1 IO note that the requiremeni rhat ''n ._ 0 a-;
,, - oo is a ,wcl'.1.mry c.:ondition for convergence. but iI ism,, su fficicnt. The classic
example of a series whose nlh term goes to zero. but for which the sl·ric" docs not
converge, is the lwn11011ic .}·eries

00 1
5=I:- ,r;:: I II

Problem 5 leads you t.hrough the standard proor that Sn ➔ oo as 11 -+ oo.


As in the case of infinite sequences, the convergence or lack of convergence
of an infinite seril:~ is not affected by the addition or deletion of a finite number
of terms. For example, if we were to add 100 1enn, (which add up 10 c) to the
beginning of a series. then iL'- partiaJ sums would be S,, c instead of S,,, and the
sum. S_1_, of the augmenied series would be

S-1 = II
lim (S11
• '-
+ c) = S + c

2.2 Problems

= '°' -.
:--.. I
1. Evalualt! lhc sum of the ~cries S
L.., 311
11-(1

2. Evaluate the sum of the series S = L.., '°' ---.


..._

11 = 1
(-])"+I
.,,,
-
3. Show that 0. 142 857 142 857 · • · is a rntional number.
4. Express Lhe recurring decirn:11. 0.27272727 • · · as a fruction .

5. Here is a slandard proof that the harmonic series diverges. Show 1ha1

S,

= 1+ -2I = -23 s 3
I+ .-,

and so on. and then argue 1ba1 S11 is unbounded ~ 11 - . oo.

6. Evaluate the series S = t:-..,

u=l n(n
1
+ I)
. Hi11r: Use partiaJ fractions.

.
7 • Den"c . • h
an expression 1or t c nth
.
pan,al sumo
rs I
= -I·'.\ + - I + - 1 + --
I
+ •• •and show Lhat S = 1/2 .
3.5 5 . 7 7.9
/-/inr: Usl.! partial fractions.

8. Evaluate the series S '°' -·----.


x,
= L..,
+ 1'1
6"
')n

"== I
70 Chap! r 2 / In init ·

.
9. Evaluate rile scnes S =~ I
L... - - - - --
. . f .
. H,,11: Use panJal rac11on~.
,i=:I 11(11 + l)(n + 2)
.
10. Find an expres~10n for the cen£"ral tem1 ol 1he scnc!- S
~
. .
= -2I + -43 + -78 + -16
15 .
+ -· · and show thal the scnes

.
II. Evaluate 1hc lll'ru:, S = -2-'l + -27I + -291 + -21I I
+ -· ·.
12. Find all the values of l" for which each of the following series converges:
X-

(a) L('2x)11 (b) I:cx


----
- l)n
L~, (~\-' r
11=(1 11= 0
·,C

(cl
11.::-0
(d)
L"
11-11
,f'

-...
13. Show that the scrici, L( ~ - Jiil di\·erge., even though ,,lin~ 11,1 = 0.
n=O

2. Tests for Convergence

A series 1ha1 consis1s or positive terms only is called a positive seril'S. In 1his
section we prc-.c111 i,(,•vcral tcs1-. for lh1;.• i,:rn1, t'fgl:nC.e of pt.")Sili\ e -.~·riC'_.;, In the nex1
~cl:tion. we will di1scus:-. oltenwti11J: sai1'.1:. which arc seri-c:s whose succe~sive rcrms
alLcmare in si£n. The curwergence propcnie!', of pmi1 ,i w ~cries and allemating
,,-
lh
)':::: 1111
~cries an: somewhat dil'forcn1. so we will 1.rca1 them !',cp,ararcly.
2 J .i S 6 II
The imegml ft'.\'f compares a posirive scri ~, 10 an integral of a posi Live f unet ion
(a) .f (x) such that J (11) > u n for all 11 ?: I. The convergence of the irntegral wi 11 Lhen
a&si1rc 1,hc convcrgcnl.·e of the !'>Cfics. The ndvnmage of t.hc intt'grnl tes1 is that it is
usually easier to evalluare-an integral Hurn it is to sum .i. se-rie-s into a closed form.

Integral l'esi·: Let L ''n be n pm:i1in:' .\eries 1,111d /1'1 j(x) he LJ omrimw11,\,
positi1·c. decreasing fimctirm :mch that J (11) = 11, 1 for 11 = L 2, .... Thr11.
L,, li 11 t'tJlll 'er,1u·.~ if and rml_1· if {.-x, f Cr )dx cmw1'1g1·s.
2 3 4 5 6 II • I

(b)
ll1e proof of 1hc integral lest is illustrated in Figures 2.4a and 2.4b. \\l'c .:;ec in
Figure 2.4 Figure 2.4a 1h:11 the area under the curve from I 10 N i.'- less than the total- ar~a of
A geometric aid 110 the proof of the in 1eg_r.il 1he rccianglcs. which is 11 1 + 11 2 + · · · + 11 N· Thus. we have
IC',I.

Ii N
J(x)dx < 11 1 + 11:? + •• -+ "N =S.v ( IJ

If L 11 11 converges. then the sequence of partial sums will be bounded. and so


Ji'' J(x)dx will converge as N-, oo. Conversely. Figure 2.4b shows that
2.J li - fnr (( m, -•rg o e 71

or
N
SN <
J.
1
J(x)dx 11 1 (2)

So, if ft' J (x )dx conwrgi:.s. then S.v < _r;x. J (x )dx + 11 1 and so 1hc !>equenc.:e
S,-v will ~ bounded and the series L "n rnnverges.
Actually. 1he inlegral test doesn't have to, stan wi1h the 11, = I 1.cm1. lt"s ~he
··Jong tail"" bchavior of a serie:- that dc1cnnim1es whelher it converges or di vl'u_ges,
so we can slart with any finilc 1enn. say. M, and investigate

-x.
f.\t ;;::
1 M
f (x >dx

We'll usually t::.ikc M ;;:: I. however.


~

Let· s consider 1he series L ~. The function f


11-
(x) = I/x'!. equals I/ ~ when 11

11=.I

x is an integer. Punhcnnon:, j(.r) is c:onlinuous and mono1onically decreasing.


Now

= .f".\.. = h-lim• ~ f"l. ]h = I


f
. I
<'x..
J(x)clx
I
dr
-=;-
x-
dr
_:_
x-
= hlim iXJ
[ - -I
X I

X,

s.o 1hc ~cries '°' ,,-1; converges. Lnh1rtuna1cly. neither the integral
~
1cst nor any
,: "· I
01hcr lest. tells us what the sum is equal 10: 1h~y simply 1ell us whether 1hc ~cric~
converges or diverges.

Example 1:
lnvcstiga1e 1hc convergence of the .series

. ,x I
.S(p)=, _
~ 11/1
110:::I

as a func1ion of p. We shall call S(p) the/' .1·erics.

SOLUTION: Tak • j(,r) = l/.1J' IO he the 1cs1 funnion fur Lhc in1cgrnl lest.

= J<X. -t!x = h~h. m 1·b -dx


j •'.l<.i

I
_
J (x}il x
I X J> • I XI

= lim
-rl-p ]/,
- -·- - = b•lim 1,1-µ - 1)
b o.., [ 1-p I ( 1-p
72 C/1.iplr'< 2 / !nllllilc Series

If p > I. !hen lim {bl-p - 1)/(1- p) = I/(p - I). and so lhe p series
lt-xa
converges. If p < I. then lim
/,-,.x
1
(b • 11 - 1)/( I - p) = , and w the f' :.cries
diverges. We already know 1ha1 1he series diverge,; for p = I, 1he harmonic
~ries. but if we didn ·1. we would sec so anyway because f (x) = I /x in this
case and

)
•ex;
I
J(x)dx f
= ~ dx =
1 X
lim [ In .r ],.
h--... I
= '-

We can use the integral test to determine 1h~ error tha1 occurs when we
approximate the sum of a series by its panial sums. The error RN incurred by
using SN is (Problem 3)

[x;
lN+l
J(x)dx S RN ~ 1 N
00

f(x)t!x (3)

If we sum u,, = l/11'.\ to ten tcmis. 1hc error will lie between
dx f'X\ -dx
f
oc,
---:;- S RN S
11 x-' in x3

or between 0.00413 and 0.00500.


Equal ion :l cun be used to determine how many terms we need to use to uchicvc
·: ,O

a given accuracy. For example, achieve an ac·curacy of ±0.0002 in ~


10
L.., ,,_!_.
... we
.,._, 11=1
S.v . -Inn L
,r4
- - - -: -.- . "' - .... .....
- - -- -
need lo take 12 1erms. To achieve 1hi:: !-.arne accuracy for .,- , we need 10 take
11=1
11-
90 over 100 000 tenns (Problem .5)! We say that the fir,1 ,um converges much fa,_ter
than the second. l.n fact. we migh1 say that the firs1 one converges ra1her quickly
(see Figure 2.5). and Lhal the second one converges very ~lowly. Obviously. rapidly
converging series are more useful than very slowly converging series for numerical
10 N work. There arc. however. a number of numerical methods for acceleral ing the rate
Figure 2.5 of convergence of infinite series.
N The geometric serie:-. and the p serie$ arc frequently used 10 test for conver-
The pnnial sum.s S,_, =L 1/n~ plo11t:d gence of other series by a lest called the compariso11 1es1. Before we discuss this
11 ""- I
:iga.in,t N, T:he lhni1in~ valui= is shown as 1cs1. we display the geometric series and the p series for ref~rcnce.
a da.~hetl line.
00

geome1Tic series : Lx"


~
=-'-
I- X
txl < I (4)
I/

,:,a
I converges p > I
p series:
L nl'
nee;)
diverges p ~ l
(5)

The Comparison Test: Let L ,,., a.11d L P,, be Jwo positive saie.-. with
u,, ~ v,, for all 11 ::- N. Then L 11 11 com·,~r:i,:1 s {f L Un conve~e.~. anJ L u,.
di,•er~cs if L 11,, di1·erges.

rn
73
(Sec Problem 14 for an outline of 1hc proof.)
00

Docs 1he series L


,r ;;;; I 11· + 3
I
converge? Well. - 1 - - <
-:i--
11· + J
3
n
I
and we know that
I

C ~

L~ II"
converges (p series wi1h p - 3). and so 1
1
- - L - +3
converges. What if
II"
n=I n=I
the series started with then = 0 1crm? We wouldn·t be able 10 apply 1hc above
inequaltty for 1he " = 0 term. Docs this make any difference? All we have to do
·,s to wn1e
· then= 0. tem1 cxplic11\y
- . and then consider
. . I ~ I
the scncs - ..1.. L ---.
3 11= I l!J + 3
The point is 1ha1 we can apply 1he comparison tcs1 srnrting with any term because
the convergence of an infini1c series is dctennined by 1hc lnrgc-11 tail of 1he series.
so the first lini1e number of lerms has no effect on the convergence or divergence
of an infinite series.

Example 2:
E.xamine lhe convergence of
00
I I I
S=E-=1 +-+ - + ···
n=I 11! 21 3!

SOLUTION:

I
for
~ = 11(11 - 1)(11 - 2) ... 3. 2 ::: 211 - 1

So

But

and so S = L ~,, . converges. We'll see in Section 7 that 5 = e - I.


m= I

Ano1hcr useful test for 1hc converge.nee of a posi1ivc scric..-. is the

Limit Comparison Test: If L 11,, and L v,, art· hvo pMilive serit·s .rnch tlwr
lim ~ Lu,, and L v,, «:itlu:r boll, com·er~r. or
= / 11·i1J, 0 < I < oo. rhe11
14-,. .:x,. I'

horh di;:agt·. ,r I = 0 011d L v com·erRt•,·. rhe11 L u" couverges. If I = oo


11

mrd L dirages. 1he11 L


1)11 diverges. 11 11
74 Chapter 2 / lnf nit ri !S

Let's use the limit comparison tes1 to rest the convergence of L 11 11 where
11 11 = 11/(11'! +!). For large n. u 11 behaves as l/11, so let's use v11 = 1/n in the
lheorcm. In this case.

lim ~ = lim ~ = I
11-rx..v,, 11•"'<..>JJ2+J

We know. however. 1ha1 L v 11 diverges (the ham1onic series). so L 11 diverges as 11

well.
If 11 11 = 11/(11.1 + I) ins1ead of 11/(11 2 + 1), lhen 11,1 behave. ~ l/11 2 for large"
and we would choose v,, = J/11 2, in which case

3
. II . 11
l1m ---!!.
n-,:x, l'rr
= llm -- =
co n' + I
,,
I

But L
t• 11 converges (the p series wi1h p = 2) and so Lu,, converges.
The cwo examples 1hat we have just discu."sed lead us to anolhcr u~eful 1es1
(we"ll call it the p res1).

p Test: If lim 11f'u 11


,, •X>
= I. rhe11 L 11,, converges 1:f p > I (nnd I isfi11iU:', e1·m
0), or Lu 11
di1-erges if p.:::: I and I f:. 0 (hllf I may he i1~/i11ire).

Example 3:
Test lhc series
~
Inn
S= L-,
11= I
11-

for convergence.

SOLUTION: We'll mul!iply In 11/1/!. by 11P and ).Ce if there i~ a vnluc of


p > I that gives us a finite Ii mi 1. If we choose p = 3/2. for example. we have

. ln11 . ln11
I1m ,.-111- • -., = ,,1,-1m - , =0
" :x., n- ._ · 11 1 ~

( by r H opi ta I· s rule. for imaancc ). There fore.

converges. We <.·hose p = 3/2 for concretcnL''' above, llut note 1.ha1 we could
haYe cho ... cn any vulue sm:h that l < p < 2.
2 . .1 T!:$15 for Convergence 75

2.3 Problems

I. Does lhe series S =~


L
Inn
-
II
converge?
11= I

.
2. Doe.s I.he series S = '°'
L - ln 11
- converge?
11=1 If •

3. Derive Equation 3.

4. How many terms do we need to take to achieve an accuracy of ±0.00 I for L ,I ?


::o

n;c:; I ,,.

~ I
5. Show !hat you need 15 terms to achieve an accuracy of ±0.000 I for 1hc ~um '°' - . but Iha1 you need more
L,,-1
n=I
~ ln 11
Ihan 125 000 terms 10 achieve Ihe same accuracy for'°' -,- .
L n-
11=1

6. Show lhat the series S


.
=~
L - - - -I - - - converges.
n-=I (211 - 1)(211 + I)
7. Wri1e out n few terrns in 1he series in the previous problem ,md show that S = 1/2. This type of series is called
a 1elescopi11g series because most of its terms cancel pairwise. Hint: Use panial fracIions.

= L (1- - --
I.) diverges because L
,:,:,I- and L I di,.·ergc'? Why
00 00

8. Can we conclude 1hat the series S


11-1 II 11+2 n:::d"
--
nc::111+2
not?

9. Write out a few terms of the series in the previous problem and show lhal S = J/2.
10. Test each of the following series for convergence:
-:,c. X•
111 1
(a) '°' nc- (b) '°'
L Ln 2+3
n=O n=l

(c) (d) ' ° ' - -


L
,r-;;:I
J1.J, + I
lJ. Docs Loc --
(Inn)'
-
3
convcr£c?
,i:=,f II

00

12. Docs L~ ---


I

+ fa
ri==I n
converge? Wh:-it about L
n""l n
I

+11
., ?
I-

13. Doe~ L
u::c.l I+ 2 + 3 + · · · + n
I
converge?

14. Prove the comparison 1es1. (Hint: Appeal 10 1he fac1 !hat a bounded monotonic series converges.)

gl
76 Ch.:ipl!'r 2 / lnr mi t1• Seri1~~,

2.4 Alternating Series

An altemaring series is an infinite series whose successive terms alternate in sign.


The general fomrnla for an alternating series is
,-:,:;::

S = L(- l)"+'vn = Vi - u2 + 1J3 - V.,i + ... (I)


n=I

where v,, ::_ 0. We have a very convenient convergence test for allemating series
that is due to Leibnitz.

If O < V11 + 1 :S v,, and 11Jim


..... ,,0
u,, = 0. 1l,en S com'erges.
(See Problem 17.) For example, I.he altema1i11g hannonic series
00
1
( - l)" I I I
S = L -- =1 - - +---+ · ·· (2)
11 2 3 4

converges according to Lhe LeibnilZ Lest


Convergent alternating series have a propcny that allows you to estimate their
sums eusily. If Sis Lhe sum of the (al1cmaring) series and SN is its nlh partial sum.
then lhe remainder after N terms is RN= S - SN . h turns out that IR.vl < u 1:
in other words, the magnitude of the error that occurs from approximating S by
using just lhc firs1 N 1crms is not greuter thun the magnitude of tJ1e first omincd
tem1 (Problem 18).
Let's illustrate this propeny using the alternating series
00
1
S=L (-1)"+!
n.a.l
11
I
= 1-2!+3!-~+···
I I

which we show is equal 10 I - e- 1 in Example 2.7-2. Table 2.1 lists the vaJue.s of
uN+I• SN, and I RN I for N =
I through 6. Notice that I RN I < vN+I·

Table 2.1
The first six part.ial sums and remainders for lhe
(-1)"+1
series L ---
00

11!
= I - e- 1
= 0.63212.
11=!

N llN+I S,v R,v =S - S,v I RN I


0.5000 I .0000 -0.3679 0.3679
2 0.1667 0.5000 +0.1321 0.1321
3 0 .0417 0.6667 -0.0345 0.0345
4 0.0083 0.6250 +0.0071 0.0071
5 0.0014 0.6333 -0.0012 0.0012
6 0.0002 0.6319 +0.0002 0.0002
2.4 Al1erna1ing Seci1·~ 77
Let's see how many 1enns we shou.ld use in 1he above series to get five-place
accuracy. We'll require the first 1errn omined 10 be < 5 x 10- 6 in order to assure
five-place accuracy. Therefore, we choose N = 8 since 1/9! = 2.8 x 10-6. If we
add 1he firs1 eight tem1s. we get S8 = 0.632 1181 . The correct value to seven places
is 0.632 1206.

Example 1:
How many renns should we use to caJculate

to four-place accuracy?

SOLUTION: We wa.111 rhc first omjned 1enn 10 be less than Ix 10~,


so we want lj(2k + 1) 3 10 be less than 10- 4 • which will be so if k > 10.
Choosing k = =
11 gives S 10 0.968 993. We"ll learn in Section 3.7 that
S = rr 3/32 = 0.968 946 · · ·.

You should be aware th:11 many of the CAS 1ha1 arc available can be used 10
evaluate series such as the one in Example I. For example, I.he one instruct.ion line
in Mathematica

NSum[ (-1)"n I (2 • n + 1)"3, {n, 0, 100} l

evaluates. the sum numerically from JI= 0 to 11 = 100 (0.96894-6).


Another propeny of altemaLing series that is closely related 10 the fact 1ha1
I RN 1 _:s vN+I is that the value of the series S always lies in the closed interval

l SN. SN+I J. The proof that I RN I~ v '+I- which we relegated to Problem 17.
has as a by-product Lhal succc~sive values of RN have opposite sign~ (if they
don't happen to equal zero). So, if S =SN+ RN= S,v+i + RN...._ 1• and R,v::: 0
and RN+I S 0, then S ~ SN and S ~ S,\'.:.J• or

(3)
,r' I 32 .- ... ---- - . -- -·•- ----
0.94
Similarly. if RN ::: 0 and RN+i::: 0, then
10 N
(4)
Figure 2.6
Either way. we h::ave 1he result 1ha1 S lies wi1hin rhe closed interval l SN. S,v+i ]. N (-1}1'
The partial sums S.v =L + 1)·1
Table 2.1 shows that Equations 3 and 4 are satisfied (up to N = 6 at least) and the =(J (2.k
ploncd again$t N . The limi1ing value i:;
partiaJ sum." for Example I are ploued against N in Figure 2.6. The "exact"' vaJue
shown ; a dashed line. 101c the scale on
is 0.968 946 · · · and this value lies between any two consecutive partial sums. the venjc.aJ ;u:is.
78 Ch,1plN 2 / In/mile 51.•rie-.

The harmonic series diverges, bui according 101he LeibnilZ tes1, Lhe al!ernating
harmonic series converges. A series L 11 n is caJ led absolutely convergenr if L Iu 11 1
converges. If L 11n converges, but L 11.1n I does not converge. Lhen the series L u 11
is called co11dirio11ally convergenr. The alternating harmonic ~L·ric':-. is an example
of a conditionally convcrgen1 series. Surely, if a series converges absolutely, then
it converges condi1ional ly.
Let's consider the series

(-Woo
s-I:-
,, In 11
11=-
This (alternating) series is convergent because (-1)" / In,, decreases mono1oni-
cally with increasing 11 and I.he nth lenn goes to zero as n --+ oo. It is nol absolutely
convergent, however, because

I(-t)"+ l I
00 I N
s-"
L..., - In
1 -
-"
L..., -
Inn
11 -
11=2 w=2

diverges (use rhe /J 1cst with p = 1/2. Problem 8).

Example 2:
Tcsl u1c following series for ab.-.olurc convergence and condi1ional
convc:rgcncc.

SOL u TIO N: Noti£:e that cos ,m = (-1)". so S is an alternating series.


Since

l I=l~I
co11 -~;r n-

the series is absolutely convergent by comparison with a p series with p = 2.


Thu.~. S i.~ convergent.

We now present several tests for !.he absolute convergence of ::i series. Of
coun-e. 1he~e tesls can also be used to tesl for convergence of a positive series.

The Ralio Test: LRr L 11 11 be any series and let

· l"•i- -'I
Inn
11-c,c, 11,,
=p

Then L 11 11
cnnverges absolwely if p < I. di\'ergt·,· if p > I. and the 1es1 is
i11r1111,:lmi1·e if p = I.
79

(See Problem 11.) We'll use 1hc series

to illm:;lmle the ratio 1es1.

lim
11-•00
~
ll
I u 11
I= 11
lim
00
I2" 1(11
.,,,,,
- I
+ I} = lim
11 - • "XJ
I
=-=p<l
2

so the series converges absolutely.


How about the series

(5)

ln this l'.ase.

i-o the mtio test foils us. There is an cx1ension (.lf the ratio 1cs1, however. lhat is
often useful when p = I.

Raabe's Test: l.-<'I

Thi' series L u,, com•Nges ah.w/UJely if l > I and either di1·ery: f!s or com•erx.es
conditionally ~f I < I. As ,vith rlu, simple ratio lest. the r,·st td h· 11s nothing if
=
J I.

We"ll ~,pply Raabc's Lesl 10 1hc series in F..qua.rion 5. where I.he ratio test failed us.
Applying Equa1ion 6:

li n1
n -~ ':\;
[11 ( I
1
11/nln 'I)] = ,,lim
11 '.!.
2u2 + II
+ 211 + 2
=I = 2
Therefore, rhc series ini Equa1ion 5 converges abso'1 utcly.

hample 3:
ln\'c~tigatc 1hc com·l·r~1.:nce of

I ,/•

(n + I)!
80 Ch.:ipt •r 2 I l11finit • Seri~

SOLUTION: The ratio lest gives

I1m
.
n-• oo
l"n+l
--I
u11

= . (n+l)
lim
ri~OC,
-
,,
-
3
.
• hm --
11-00"
I
+ 2
=0
so 1hc scric!- is absolutely convergent.

Example 4:
Tes! the following series for convergence:

S= f= I - 3 - 5 · · · (2n - I)
u=l 2"(11 + I)!
SOLUTION: The ratio lesl gives

.
I1m l- - I=
Un+I r·1m I-211-+-I I = I
n-► 00 u11 n~"- 2(11 + 2)
w Lhc rario 1esr is inconclusive in this case. Raobe's 1es1 gives

. 11 ( I - -
lim 2n-+-I ) = . --
hm 3,, - = -3 >
" · "- 2(n + 2) 11-00 2(11 + 2) 2

Therefore, we see thal the series converges absolu1ely.

The above tesL<; for convergence give us sufficie111 conditions for the absolute
convergence of a series; in other words, when the conditions are satisfied. these-
ries definitely converges absolutely. The tesl.S. however. do nor give us nccl'ssary
conditions: that is, there are absolutely convergent series which do nor satisfy the
above conditions.
Before leaving rh.is section. we point ou1 a somewhat disconcerting property
of conditionally convergenl series that you sbould keep in mind. Consider the
allemaring harmonic series. which we know is conditionally convergent. Write
it as
l I I I 1 I I
S=I--+---+---+---+•··
2 3 4 S 6 7 8
Mulliply S by I/2 and write the result as

I
-s = -I - -I + -I - -I + ...
2 2 4 6
81

and add this to S above LO get


3 I l I I I I I I
-S =I+ - - - + - + - - - + - + - - - + ·· ·
2 3 2 5 7 4 9 11 6
This las1 series is actually just a rcam:mgcment of the series for S. so we are led
to the absurd result that S = is.
The message here i:-. that you cannot rearrange
the tem1s in a cond1Lionally convergent series without changing iLs value. In fact
if L "" is conditionally convergen1, it is possible to rearrange L 11 11 10 have any
sum or LO even des1roy i1s convergence. Absolutely convergem series. on the 01hcr
hand, are fairly well behaved in 1he sense t.ha1 reammgement does not aher 1he
value of the sum of the series.

2.4 Problems
oo (-t)"+I
I. Construi:1 a t.ablc like Table 2.1 for the series. S = "(""' 4
L - -4- . (We'll see in Section 3. 7 that S = 7rr /720.)
11
nc=I
Show lhat I RN I< "N+I and that Ec.1uations 3 and 4 arc s.a1isfied.

~
1
2. Construct a table like Table 2.1 for the series S =
L
(- I):' +
,,~
• (We"ll see in Section 3.7 that S = JT 2 / I 2.)
n.:.I
Show th,ll I RN I< "N+I and that Equations 3 and 4 are sa1isf1cd.
00

3. How many tenns should we use to have an error of ±0 .000 I for S = '°'
L
(- I)"+ 1 ~?
411
11=1
(-1)"+1
4. How many tenns should we use 10 have an error of ±0.00001 for S =L -1 '? We"ll see in Section 3.7
n
that lhc citact value of Sis 7rrJ /720. 11
=1

oc 1/2
5. Use any CAS to con.suucr a figure like Figure 2.6 for S =""<-It+'_. How many renns should you take
L 2"
lo have an error of< 10--1? n:::I

6. Use any CAS to consLrUcl a figure like Figure 2.6 for S giwn in Problem 2.
7. U5c any CAS to construe! a figure like Figure 2.6 for S given in Problem 4.

8. Show thal L -In,,l diverge5.


oo

n=l

9. Use IJ1c .intcgr.i.l lest to test the sencs


. L '°" -In,,I- for convergence.
cc

,, ,.. 11
2

.
IO. Use the m1egral 1cs110 tesL the sc.nc:; L . '°" - - -2 for
I convergence .
""" 2 11(ln n)

I 1. In this problem we shall walk rhrough the proof of the ratio tesl. Assume that Jim
11-00
I""+' I=
u,,
r < I. Choose,

I I
such that r < r < I. w Lhat ""+ I St if 11 > N. Now argue that 111 11 • 1::I
ll n
~ 111,, 1,.1.- and appeal 10 lhe geomet.ric
series. The proofs for the case r > I and r = I are similar.
82 ri es

12. Detennine whe1her I.he followin g series are absolutely convcrg~.nl. condi1ion:.illy convergent. or diver ent: 0

~ ( - I )11 I I
(a) L - --
. . ,r2 + I
I II
(b) L(-l)"+i _ _
,1 - I ~
(c)
-x,
I::(-1)" 1
--
(n + I)~
,1.,,, I 11c-el

(dl (cl (f)

X· 1

13. lnvcs1jg:i1c 1hc convergence of the scric1- S = L (-1 )"+ 1


~.
Jl= I ,,3 + I

14. Tesl the sene· S


.
=~ 1 · 3 · 5 · · · (211 - I)
L - - - - - - - - for convt!rgcnce.
1t=I 2-4·6···(2n)

15. There is another test for convergence culled the Root Tt•sr: Let r = /!
lim v'l11,, I.
cc

If r< L 1hen Lu,, convcrgc-s (absolutely).


If r > I, then L u11 diverges.
If r =I. 1hen the root 1es1 is inconclusive.
(In applying lhe rooI test, it is often helpful 10 remember th:11 lim
,,-~ 1,i = I.) Use the root test to discus, the
convergence of the series

11=1

16. Use the root lCSI 10 see if t c~,")


11==1 ·
11

converges.

17. Prove Leibni1z's alternating ~eric~ thee rem. Him: first show 1ha1i S 2,, _ 0 and that S1,, 5 v1• and then argue
tJ1at lim S,,, = S. for example.. Then ~how lh:lil lim S1,1 1 = S ;1I. o .
n -, - n - •,:,.:,

l8. We'll prove thal I R,-.,1 ~ uN+I in I·h is pmbl'em. Firs-1 consider Ihe error made in stopping after 2N l<!nns:

Show that R?.N?. 0. Now write

Show thal O _ R2N S vu.r+i• the magnitude of 1hc first omiued tem1. Using a similar argumen1. show thar the
error made by. topping of1t:r 2N + 1 tem,s is -1 2N + I R !. .V- 1 ::::_ 0. (Note that R~N :;ind R·uv+i have oppo~itc
signs if they are nol equal to zero.) Combine these rwo re:-ults to snow I RN I:::. 1JN • 1.

19. Apply the rcsul1 of lhc previous problem 10 the altema1ing scric"

I I l
S= 1 - - - - - + o+o+o+o+ ...
2 3 4
00

20. Test lhe serie!- S = L --------


I· 3 · 5 · · · (211 - I) I
n=I 2-4 • 6···(2 11) n
for convergence.
2..5 Lnirmm Cn1l\t'rg1•111 P 83
2.5 Uniform Convergence
Up 10 thi~ point. most of the series Lha1 we huve discussed arc scric, whose terms
are conslanLS. Now let's consider series whose terms are function~. f-'or example.
we might have

= L --,-
(XJ

C'( cos /IX


..) x)
11 ~
n-d
or

~ I
T(x)= L-,-
n~+x
w=l

The definition of convergence of such series follows immediately from our defini-
tion of convcrgent:e for serit~s of conslanl terms. The series
c-:
S(x) = L 11 11 (x)
n=I

is said to converge to S(x) in l u. b I if

]S(:r) - 511 (.x)] < t: for 11>N(t:,X) ( I)

for all x in [a, b I. In other words. 1he sequence of partial sums {S,, (x) I converge,
in I a. b ]. Note that the value nf N may depend upon both t and .r. If Equation I
is satisfied. we write

S(x) = I/
lim S,,(:r)
• ~
(2)

for each value of x.

Example 1:
Test the following ,l·rics for convergence;

SOLUTION: We'll use 1hc ratio test:

.
l1m l",,
-·1(X)I
-- = 1·1m I< - - - - - I= Ix -
,. : )11(11+1)
.x - . 21
1 1-H',:: 11 11 (_r) II- , (II+ ])(II 2)

Thus, the strics converges if Ix - 21 < I. or if I < x < 3. The series is


I
convcrgcn1 al b(.lth cnJ points since L ---
:x:,

+ I)
is convergent. so we
,1;;;;:I 11(11
wri1c I ~ x ~ J, We' II inc:ludc the poinL<: x = I and x = 3 in the interval of
converge nee.

C gl
84 Chapter 2 / Infinite Series

We will show in Example 3 that the ~cries in Example I coF1verges 10


I + (3-x)
x _
2
ln(J - x) for I ~ x ~ 3.
Now consider the series
00 .,
x2 2 x2 x-
S(x) =L ,, . = x + --., + ., ., + · · · (3)
J=O (I + x-) 1 I + x- (l + x+

_..,. If x = 0, then Sn(x) = 0 for all n and S(x) = lim S,1 (x) = 0. If x c/=- 0. then
11---,00
2
1/(l +x ) < I, and Lhe sum is a geometric series with

S(x) =x-1 ( I
1
) = I +.c,, X -/=- 0
1---
J + x2
- 1 X
Nole that lim S(x) = I. so S(x) has a discontinuity al x = 0 in 1his case (Fig-
x-o
Figure 2.7 ure 2.7).
A plo1 of the function S(x) defined by In Example I, S(:c) is a continuous function or :c. but the function represented
Equation 3.
by Equation 3 is discontinuous. The terms of both series are continuous. so why
is S(x) coniinuous in one ca-.e bul not in the other? The answer LO this question
lies in lhe idea or unifon11 com ergence. We say that an infinite series is unifom1ly
1

convergent if

1S(x) - S,,(x)I <" for 11 > N (f.) (4)


where N depends upon E, bu1 is indepe11de111 of.r. Let's look al Equation 3:
00 J
x-
S(x) =~ 1

L (I+ x~)J
1-=0

Recall that S(x) = 0 when x 0 and S(x) = =


I+ x 2 otherwise. Let's consider the
case where x f. 0. Then the nth partial sum is

r-l
and

IS(x) - s.(x)I = I(I; x'


Now let's see if we can satisfy Equation 4. The difference IS(.t) - S11 (x)I will
·
be < e 1f we choose
') N I
l 1/(1 + x-)1 - <"·or N I In" I
,, . Notice, however,
>
ln(l + .r-)
that N becomes unbounded as x ➔ 0. so the series is 1101 uniformly convergent at
x = 0. the poi.nt at wh.ich S(x) is discontinuous.
2.5 Uniform Convergence 85
lim S,,(x) = S(x) unifonnly in [ a.b).Then. given E > 0, there
Suppose Lhal ,,..,.cc y
is an N such that if 11 > N. IS(x) - S,,(x)j < E for a ::: x ,:5 b. l.n other words.
S(x) - E < S,,(x) < S(x) +€for a:::: x::: b if n > N. Thus. S,.(x) lies within the S(x) + E
band shown in Figure 2.8.
S11 (X)
The series in Equa1ion 3 does not confonn 10 the picrurc in Figure 2.8. The
S(x) - E
partial sums of Equation 3 for n = 20 and I 00 arc plolled in Figure 2.9 along with
S(x). Note tha1 the discontinuity in S(.r) at x = 0 forces the partial sums 10 reach
a b X
down from the curve I + x 2 to reach the value 5(0) = 0. None of Lhese partiaJ
sums can lie within a band S(x) ± € because of the behavior near lhe origin. They Figure 2.8
aJI break off from the parabola S(x) =I+ x 2 • and hence from lhc b.md S(x) ± E. Ao illustr:uion of unifom, con\'ergeoce .
as x approaches zero. V

What about the series in Example I, where S(x) is a continuous function for
I ~ x ~ 3? 1n order to address this question, we need a useful method to establish
1he uniform convergence of a series. The defini1ion given in Equation 4 is often
awkward to implement. The following 1cs1 is much more uscrul:

The Welers ..rass M Test: If a sequence of cnnJrams Mn rnn be found mcl1


-1 X
1ha1
Figur.e•2..9
The patti:LI sum..- of Equation J ,for " = 20
nnd 100 plotted agai,nsr .r.
for x i11 [a, h ]. QJ1d M11 converges. then L u,.(x) is 1111ifon11/ya11d 11bso/i11ely
crml'erRe111 for x in [ a, b i

Example 2:
Use the Weiermass M ,~s, 10 show that the scrie~ in Example I is uniformly
convergent.

SOLUTION: Choose M,J = 111-I . Then

] (x 2) 11
In(11 + I)
I
<
I
11 2

Then L M,, is a p series with p = 2 and thus converges. Therefore, the


series in Example I is uniformly convergent for I ::'.: x s 3,

We're hinting around that 1he series S(x) in Ex.ample I is a continuous func-
tion of x because the series is uniformly convergent. Well, it's true. Unirormly
convergent series have a number of nice propenies. and continuity i."- one of them .

J. If all the lt'nl1S in S(x) = L,1= I U n<x) Ort' conti11uo11s in f a. b l and


L~=I 11 11 (.r) is 1mifom1/y crmverge11t in [ a. b L then S(x) is comimwus in
f e1, b ) . fo other words.
86 Chap! r 2 / lnfinit

~ -X X

lim
x-a~
'°' 11,.(.r) =L lim 11 11 (.r)
i-~
= L 11 11 (0) (5)
11.=:1 n=I w=d

Thus, a unifonnly convergenl series of conLinuous functions is a continuous func-


tion. Not only is S (.x) contjnuous if a uni fonnly convergent series consisrs of terms
that are continuous. but the converse is true : if S(.r) is discontinuous. then lhe series
i~ not un.ifonnly convergent .
Two other useful propenies of uniformly convergent series are given in the
following 1wo Lhcorems.:

00

2. /fall thetem1s in S(.x) =L 1,, 1(.r) arern111i11uo11s in I o. b laud L 11nCx)


11=) 11=1
is w,iformly co11verge11t i11 I tJ, l> J. lhl'II

(6)

or

(7)

In other words. a uniformly conlinuous .series can be integrated 1crm by rcrrn .


X ~

3. lf L 11 11 (x) co11very.C.\' ro S(x) a11d all the rcm1s i11 S(x) = L 11 11 (:c:) arc
n=I
~

differe11riable in I 11. b I and if L 11 ;, (x) wm:erges u11ifnm1ly in I a. h I. then


n=I

00

S'(x) = I>;,(x) (8)


I/ -.1

or

d ~
r.-c ] ~
-
[
L., ll11(X) = '"' J (x)
L., - - -
11,,
(9)
dx n=I 11::I Jx

In other words. the series can be differentiated term by 1enn. Notice that the
conditions in 3 are more stringent than those in I and 2.

Example 3:
Stan wi1h 1hc geometric ,i:ri..,·s for 1/( I - x) am.I imegrah! twice 1erm by
term 10 dcrivl: an e."<prc.s~ion for

S(x) =~
L.,
x"
r1=-I Tl(ll I)
2.5 Uniiom1 Corweri:; n e 87

Compare rhis series 10 rhe one in Example I.

SOLUTION: Start wilh

_ I
I- z -
- L ,'l L
lz l < I
n==O

ln1egra1c bolh sides of this equation from Oto x (Ix I < I) 10 obtain

00 \'II
... =I>-
11=1 ,.,

Integrate both sides once again

- lor ln(I - 1)dI = f 1


1-r
In II d11 = [ 11 In 11 - 11
] 1-.r

x2 x.l -'.4
= x + (I - x) In(\ - ."I')= - +- +- + ···
1-2 2 -3 3.4
00 n
=XL= x
+ I)
=xS(x)
11 1 11(11

or

S(x) = I+ (1-x)
-x- ln(l - x) xl < I

The ~cries in Example I is expres~d in terms of x - 2 mlher than x. so


substitute x - 2 for x in lhc above expression to get

3 - x I (3 ~ (x - 2)11
I +- - n -x) = L.J lx-2 1< I
X 2 n=I 11(11 + I)

The great advantage of uniform convergence is 1.hat unifomlly convergent


series can be manipulated preny much like polynomials. TI1e most common series
in which the 1erms are functions of x are power series. which arc series of the fonn

00

S(x) = L a, (x 1 - c)"
n=CI

where the a 11 are constants. We are going 10 study power sertes in the next two
sections. where we shall prove that all convergent power series are unifom1ly con-
vergent. and so we can manipulate power series just as we manipulate polynomials.
88 Chap1er 2 / Infinite Ser ies

2.5 Problems

1. For what values or x do lhe following series converge?

(a)
oo
L (211)!
x2n
(b) L
XJ (.r - 2)11
112'1 (c)
oo .r'i
L ,,2 (d) f (.r + I)"
J,i
ri==I nc,,I nc.:.1 n::..:rl

2. For what values of x do lhe following series converge?


00
oc. n 1(x - I)"
(a) L "~rn (b) L 2"
n:I n=I

3. We'll learn in Chapter 12 1hat the series L (-l)"(r/2)~"


x-

,i;;:-0
· defines a Be. el fonction, which we denote
11! 11!
by J0(x). For wha1 values or .r does J0 (x) converge?
00

4. Use Equation 4 10 show that S(.r) = L .rn converges uniformly to 1/( I - x) for l.rl Sa < I.
n==O
oo N
5. We"ll prove the Weiersr:ra...;s M test in this problem. Let S(.r) =L u,,(x) and SN(x) = L 11,,(x). Now show
11.=:I
that
,-:,:) X,

I RN(x) 1-:- 1S(.r) - SN(.x) I .'.S L I 11,.(x) I .'.S L M,,

ll.Ild 1htH given ::m ". however small. there is a number Nu i-uch 1ha1 I RN (x) I _ if N > N 0 • independent of x.
6. Show 1ha1 I.he following series arc uniformly convergent:
oo xn oo I
(a) L
112
l x 1:5 I (b)
112
+ x2 I .r I L
11=.l 11=-1

7. The nature of the Weierstrass M 1es1 migh1 le.ad you Lo believe lhat if a series converges uniformly. then il is
= f,
1
ubsolu1ely convergcnl. Show 1hut S(x) (-!)" ~ is uniformly convcrgcnl for aU x. but only conditionally
n:I 11+x
convergent.
8. Prove tha1 if 5,,(x) is continuous in [ a. b I. and if S(x) = lim S,,(x) is uniformly convergent in
11-00
ra. b I, 1hen
S (x) is continuous in I a. b 1-
o:i

9• Show lhat f (x) = "'cosnx


L -- ., -
n~
. .
is conunuous tor all x.
.
11"'1
IO. We shaJI see in Chapter 15 1hat the series

S(x) =~ "£:
rr n=ll
sin(2"
211
+ l)x
+I
= I ~
_I
0<X<i1

7T
X

<
= 0, 7T
X -:::: 2JT

Is 1he series uniformly convergent?

= L -- where -oo < x < oo, converges uniformly for x


oo (a.x)"
1 l. Fl.f'SI prove tha11he series y(x) - , in any fin ire
11 1
n:.:0 '
inLcrval. Then show that y'(x) = ay(x) and in1cgra1e to get y(x) = c,1.1.r_
2.6 Power Series 89
12. Differentiate the geometric series term by term and derive a power series for (I - x)- 2• Justify your steps.
13. Integrate lhe series for I/( I+ .r 2) term by 1em1 to obtain a power series for lan- 1 x.
00
. ""'CO~IIX .
14. Does the senes L - - 2
- converge uniformly'?
,.,,,,] II
IS. Consider the series whose partial sums are S,,(x) = 11:re_,,_r: for 11 ~ I and O 5 .r ::: I. Show that

lor( II
lim S,,(u)) du i- lim
n-•OC lof' S11 (11) d11

Why are these two expressions not equal 10 each other?

16. Can you prove the a~serriun 1ha1 you make in the previous problem? Him: lnvcstigaic the bcha,.ior of
I 5(x) - s,,(x) I 31 X = 1/ J,i.

2.6 Power Se ries

An infinite series of the form


00

S(.r) = L a,.(x - ,·)" = a0 + a 1(x - c) + a 2 (x - c) 2 +... (\)


11=:0

is caJled a power series in x about I.he point c. A common special case is the series
about c =0
cc
S(x) = L a,,x" = a0 + a 1x + a 2 x 2 + • ..
11=:0

Let's apply Lhc ratio test to detennine the convergence of S(x) in Equation I.

where/ = nJim
->
Ia,,+ I· Therefore. we see that the series con.,,erges absolutely for
II 1
1

Ix - cl < I//= R a~d di.,,crges (in the absolute sense) for Ix - cl > I/ I = R. The
range of x for which the series converges. c - R < x < c + R. is called the inten-al
of co,wef"Rence of the series and R is called its rodi11:. of convergence.

Example 1:
Find the interval of convergence and the radius of convergence of

= '°' _x_
oo n
S(x)
L
n=l
"-2"

gl
90
SOLUTION:

11 · 2" I lxl
f (n + 1)2 11 +1 =2
The series converges absolutely if J.r l < 2 and diverges if lxl > 2. When
x = 2. we have the harmonic series (divergent) and whl·n x = -2. we have
lhc allcmn1ing hannonic series (conditionally convergent). Thus. the interval
of convcrgem:c is [ -2. 2 ) and the radius of convergcm:e is R 2. =

The radius of convergence of a power series may equal zero or infinity. in


whichcase it converges for only one value of x or for all values of x. respectively.
An example of Lhe first case i~

00

S(x) = L n!(x - I)"


11=1

which converge:,-. for only x = I (Problem I) and an example of the second case is
oo x"
S(_t) = L -11'
,.~1 .
which converges for every value of x (Problem I).
We see 1hcn that there are lhree possible situations for a power series
:xJ

S(x) = L a,,(x - cf', namely


11=-I

I . It converge~ for all vaJues of x: or


2. It converges for values of x in an open interval fr - R. c + R). bu1 nor outside
the dosed intcrv:l.l I,. - R. c + R ]; or
3. 11 converge.s only for :c = c.
In case I. the interval of convergence is (- oo , oo); in case 2. il is (c - R. c + R)
and possibly one or bo1h of its endpoin1s; and in case 3. it is only the poin1 x = c.
The radius of convergence in each case is oo. R, and 0. respectjvely.
A central theorem for power series is the following :

If a poirnr series L a11 (x - c)" com·erges for .r = f 1he11 it converges


absol111cly in the inlerval ) x l < I ~ I and uniformly in 11,e intuval
I x I ~ ,, < I ~ I. where 'I is some positive m,mhn.

In other words. a power series converges uniformly and absolulely in any interval
that lies entirely within its interval of convergence. (The proof of this theorem is
developed in Problem 14.)
As a direct consequence of the above lhcorcm. we have
2 .6 Power S ,nes 91
X

I. {( /(.r) = L u,,(x - c)'' co,wer~es in the imerval (c - R, c + R). rhen f(x)


n=O
i.,· crmrinunus in the interval (c - R, c + R).

2. If f(x) = La 11 (x - c)" com·ergt•.s in 1/,e i11rervnl (c - R. c + R). 1he11


n=O

x oc n,,(x - c)"+I
f f(x)dx= I:----
' n=U n +1
converges in the i111en,a/ (c - R. c + R).
00

3. lf f(x) = L a,,(x - (') 11 co111·1·ry:es in the imerval (c - R, t: + R). then


,r::cO

oc
J'(x) = L na,,(x - c)"-
1

,r=l

com·erges i11 the inrt•rool (c - R. c + R).

For example, the geometric series converges unifonnly for Jr I < I.


00
I ~ II 2
--::=. L t =l+t+t + ... It I < I
1-t
n=O

If we integrate bo1h sides of this equation from r = 0 to r = x wi1h I x I < I, we


ob1ain

.r"
= I:- =x + - + - + · · · -
00 .\"2 .r3
-ln(I - .r) I !;X < I (2)
,re::I
n 2 3

We wro1c - I ;s x < I here becau.sc if we invesripate 1he end poin1.-; x =±I.


separately. we sec that 1his series converges for x = - I (alternating harmonic
series) and diverges for x = I (hannonic series). We can also differentiate the
geometric series tenn by temi 10 obtain

I oo
- °'""' nx"- 1 =I+ 2.r + 3x::? + · · ·
- - = L-
(1-x) 2
(3)
"re I
1.n 1his case. no end point is indudcd in the interval of convergence (Problem 13).
We can also derive a power series expres.sion for 1an- 1 x. Lei's Sia.rt with the
si.andard inlcgrnl

du
1
.r
--=tan- 1 x
o I+ 11 2
Now expand the integrand as a geometric series lo write

- I-2 = I - ,
u- + II 4 - ll
6
+ ...
I+ 11

C gl
92 Ch.iprer 2 / l11r1nirt: St•ri1·s

and then integrate tenn by term to get

-1 XJ X5 X7
tan X =X - - + - - - + ··· (4)
3 5 7

hample 2:
Derive a closed expression for
00

f(x) == L nx"
ne:O

SOL U T I O N : We first note lh at the radius of convergence of thj s series is


=
R I since

.
hm
n-"~
I- -l)xn
(II +
IIXn
I
- - I =lxf 11m
. - -11=lxl
II IJ__.OC
In +
Now no1ice 1ha1 J (x) is similar 10 a geometric series. but with II in front of
.x 11 • You can arrive al this fonn. however. by di ffcrentiaring x" to gel 11x 11 - 1
and then mulliplying by x. So if we slart wi1h

I
- - =
I- x
Lx
oo

,,=0
n
=I+ x
.,
+ x· + · · ·

and differentiate both sides and then multiply by x. we gel

(5)

This series is used to calculate the average energy of a quan1um-mechanical


harmonic osciUator (Problem 12).

In this section we have derived power series for ln(l - x). (I - x)- 2 , and
lan- 1 x. all of which were deri'Jed from rhe geometric series. In the next section.
we shall study Taylor series, which furnishes us with a general method to derive
power series for any (suitably well-behaved) function.

2.6 Problems
00 oo n
1. Prove lhat L n !(x - 11
I) converges for only x = I and 1hal L ·: 1
converges for al.I x.
11::aal n;::Q .

2. Determine 1he interval of convergence of


00 ,,

(a) L _x_ (b)


,r=il n(11 + I)
2.6 Power Series 93
3. De1erminl! the interval of convergence of

(a) '°'L -+-


-..:.

n.c:.:I
(2.J,
xln I

I}!
(b) '°'L --
oo

n- 1
(x -

11 2
2)"

4. Determine the interval of convergence of


00 I/

(a) L 2nx - I
(h) L n1(x + I)"
n::::il 11=1
f)()

5. Substitute 11 1 for x in the geometric series L x" and then integrate from O 10 x to obtain a J)O\Ver series
n==O
expansion for tanh- 1 x.
."1/Ji d'<
6. Evalua1e
1 o
- - ·-.i to four decimu.1-place accuracy.
I +x
00

7. Show Lhat
(I
1
x)-
1
=~
2
L
,i;O
11(n + l)x 11
-
1
l.xl<I.

I I+ x
= L --
oo x211+1
8. Show thal - In - - lxl<I.
2 I- X ne:=O 2n + I
oc 2n + 1 oo 2,,
9. Consider lhc two power series S(x) = 2)-l)" x and C(x) ==- I)-1)"-x__ Show that
11.0 <211 + I)! 11:..0 (2n) !
S'(x) = C(:r) and 1ha1 C 1 (x) = -S'(x) . Do the~ n..-sults suggest anything 10 you? Can you show (at least for
1he lirs1 few rcnns) tha1 S 2(x) + C~(x) = I'?
IO. Use Equation 4 to calc11\a1c the value of rr 10 ~ix decimal places.

11. Evaluate L 11
2x 11 in clo,;cd form.
11,ad

12. The energy of a quan1um-mechani1.:al harmonic oscillator is given by En= (11 + !)hi). 11 = 0. I. 2, ....
where h is 1he Planck cons1ant and v i.s the fundamental frequency of the oscillator. The average vibra1ional
energy of a harmonic oscillator in an ideal gas is given by

=(I_ -hv/knT)
=
~ .. ,-nl,~/krtT
£..,,h e L'""1.
11=1J

where k8 is the Boltzmann constant and T is 1hc kelvin temperature. Show that

13. Show that the interval of convergence of Equation 3 is Ix I < I.


14. ln 1h1s problem, we'll prove that a power series converges unifonnly and absolu1ely in any interval thal lies
entirely within ii.-. interval of convergence. (c - R. c + R). First show that

10111 <'. - - - - for n > N


IR-cl"

gl
94 Chapter 2 / Infinite St•riL'.,

Use lhis resull ro show that


XJ 00

L la 11 (x - c)
11
I= L
Why does this series converge absolulely? To prove unifonn convergence, use the Wcicrstrass Mte.•a and leL
M = 111 - c In /IR - (I". where Ix I _ TJ < R.

2.7 Taylor Series

In Section 1.5. we fomrnla1ed an extension of the mean value 1heorem, which


resulted in the expression

, ,, (x - a) 2
j(x) = f(a) + f (a)(x - a)+ f (a) ! + ···
2
(I)
(r a)n (x a)n+l
+ J"(a) . - + /n+l)cn-----
,, I (n + 1)1

In Equation I we assume that J(x) and i1s first n derivatives arc continuous on
I a. x ] and that flt' 1)(0 is con1.inuous on (a, x l. Equation I is known as Taylor's
Jonnulo with remainder. The remainder tennis given by

(2)

In Equation 2. the nota1ion J< 0 >(a) means the func1ioo i1self; i.e. j(o). If Rn(.r. O
➔ Oasn ➔ oo. then

f(:c)
.
'°'
00
= L.flk>(a)
(
x - a)
k'
J:.
(3)
k=O .

is called the Taylor series or Taylor expansion of j(x) about x a. Jf a= 0 in =


Equation 3. the series is called the Mnclaurin series or Maclaurin expansion of
f(x).
=
Let's consider J(x) sin .r a, an example. For /(x) = sin x, all the deriva-
tives of f (x) are continuous for al I values of x . Funhermore.

because IJ<n+ 1>(~)I = I sin ~ I or I cos H bolh of which are .::: I. But
11+1
1
lim I x -a =0 (Problem I) and so lim IR,,(x.OI =0. The Maclaurin
11 ~ (n + I) ! " •
2. 7 Taylor S1 ·ri1 ·, 95
series for sin x is
2 l
sin x = sin O + (cos O)x - (sin O) ~ - (cos 0)~ + •••
2! 3!
XJ x5 x ( - l)n x :!n+ I
=X - 3! + si" + .. . = L(. 11::::,
(2,, + I )! (4)

Nor only does rhe series on the right converge for all .r. hut it is equal to sin .r for
all x. Similarly, 1.he Maclaurin series for cos x is
00
x·~ x4 x" l)".x2,1
cosx =I- -
2!
+ - - - + ...=
4! 6!
L. ---- (-

(211)!
(5)
n=O

hample 1:
Derive the Maclaurin expansion of f(x) = ~( -
SOLUTION: All the derivarivcs of e-"- are conrinuous and equal 10 e-r. The
remainder formula is

where~ lies between O and x. Cer1ainly O _ e~ _ el r l. so R,/r) ➔ 0 as


11 ➔oo for all values of x. Thus.

. .r 1 x3 oo x"
,~= I + .r + - + - + ... = ~ - (6)
2! 3! L
11:=()
n!

which converges for all values of x.

hample 2:
Show that

SOLUTION: Start with

'" ( I)" n oo (- l)"x" oo ( - 1)"+1.-n


e-.r=L - x =l+L - - = t-I: - -
..
111
n=O II! 11=1 """' n!

Let x = I and 'Olvc for the summation 10 get the equation in rhe statement
of the Example.
96
The ~cries 4 lhrough 6 are wonh remembering. All mathematical handbooks, such
as The CRC Standard MathemaricaJ Tables and Fonnulas, have lists of Taylor
expansions or Maclaurin ex.punsions of functions. Al.so, many of the CAS can
derive Taylor series with a one-line command. For example, 1he command in
Mathematica

Series[ (1 -Exp[ -x·2 ] ) / ( x Sin[ x"2+3 x ) ), {x, 0, 5} J

2
gives Lhe Maclaurin expansion of ( I - e- :r )/x sin (x 2 + 3x) up co fi flh order in x:
2
I - e-x I x I Ox 2 I 7x·1 3073x 4 124 79x 5
- - -2- - = - - - + - - + - - +--- + - - - + O ( x6)
x sin(x + 3x) 3 9 27 81 9720 29160

One of the most useful expansions in applied mathematics is the binomial


series. Recall that the binomial expansion is

where n is a positive in1eger. For example

(I+ x/ = I+ 3x + 3x 2 + x 3
and
(I+ x)-l = I+ 4.r + 6x 2 + 4x 3 + x 4
(see Problem 2). When " is a positive integer. ( l + x)n consists of n + I 1cnns.
starting with .r 0 =
I and ending with xn. When II is not an integer, however, lhe
expansion of (J + x)a (a not an integer) is an infinite series. To derive 1hc binomial
series, we use

f(x) =(I+ X)a

J'(x) = a( I+ x)"- 1
f"(x) = a(a - I)( I + x)a-:2

f'"l(x) = a(et - I) · · · (a - n + \)( I + x)"-n


11 tenns
Problem 5 ha.c; you show that lhe remainder term R,, (x, ~) - 0 for I x I < I as
n ---. oo, so we can substirute j("l(O)= a(a - I) · · • (a - 11 + I) into Equation 3
with a = 0 to get

" a(a - I) C1(a - !)(er -2)


(I + x) = 1 + ax + - - -x 2 + ------x·1 + • • •
2! 3!
oe, a(a - I) · • • (er - n + I)
= I+ L - - - - - - - - - x 11
(7)
n=I
11!

Jh n
2.7 Cwlor Series 97
When a is a positive in1cger, 1he binomial series terminates and becomes the
binomial expansion (Problem 4). To detennine Lhe interval of convergence of the
binomiaJ series, we look at

a(a - I) - · · (a - n + I )x..r
.
= ,,-hm l(cx-n)xl
- - - =lxl
x II+ I

Therefore. the binomial series converges absoluLely for Ix I < I. The behavior at
the end poims x = ± I depends upon the value of a.

Example 3:
Starting with

derive a series expression for sinJ,- 1 x.

SOLUTION: The binomial series with :c replaced by 11


2 is (we'll let
a = -1/2 l::11cr)

(I+ II
2
)
0
=I+
Lx- a(a - I) - · · (a - n
- - - - - - - - - 11' 11
+ I) ,

,1=! n'.

Integrating bo1h sides from O 10 x. letting a= -1/2 yields


CIC
-h-1
sm x = I + '°'et(et-l)•··(a - 11 + 1) 1n+I
~ - - - - - - - -- .c
n=I n !(2J1 + I)
I , 1-3 , 1-3·5 .,
=I- -r + --.c - ----x · + · -·
2-3 2-4-5 2-4-6-7

We should point out that not all funclions have a Maclaurin expansion. For
example. f (x) = In x has no Maclaurin expans[on because none of the derivatives
of In x exis1 al x = 0. The function In x does have Taylor expansions about poinl.5
=
other rhan x 0, however. For example. if we expand In x abour rhe point x I. =
we obtain (Problem 6)

(x - I )2 (.x - 1) 3
ln X = (X - I) - - - - + ---
2 3
Before we leave 1his section. lei's considerrhe Maclaurin expansion off (x) =
1/(1 + x2)
98 Ch apt r 2 / Infinite Se rie

f lxl < I

We obtained this expansion in the previous section by replacing II in 1/( I - 11) by


-x 2 and then using the geometric series. The above series diverges ,it 1he end point."
x =±I. yet 1/( I+ x~) seems to be perfectly well-behaved at x =± I (Figure 2. 10).
Where dill this restriction come from?
-5 5 X
In Chapter 4, we·re going to study complex numbers. the complex plane, and
Figure 2.10 !he behavior of a few functions where we'll allow 1he argument to take on complex
The funccion f<x) = 1/(1 +x 2 ) plo1ttd value~. You probably remember. however. 1ha1 a complex number is of the form
against ..r. .r + iy. where x and y arc real numbers and i 2 = -1. We can plot complex numbers
y in a coordinate system where .x represents the horizontal axis and y the vcnical
axis. Figure 2.1 I shows the poinl;: = x + iy plotted in this coordinate system caJled
. : =x+iv
, the complex plan('. If we vary.randy.::-. becomes variable and is called a complex
1·oriuhle. and we can in,.,estigate the hehavior of functions of:::. In particular. we
can replacex in f(x) = 1/(1 +x 2 ) hy: and write
X

Figure 2.11
A p(lint ~ =.r + iy in the wmpk:x pl:inc.
where by I :: I we mean the dislance of the point (x. y) 10 the origin of the coorcJina1e
system. or (x 2 + /!) l/1. The reg.ion corresponding to I - I I in the complex plane
is the region within a unit circle ccn1crcd at the origin (Figure 2. I 2). Now. if we
investigate the behaviorof f (:) as a function of: within Lhe region I z l < t. lcning
x and y vary within that region, we see that the denominator of J(:) equals zero
at ;: =±i . Thus. the region of convergence off (z.) = I/( I + ~:2) is restricted to
within the unit circle in the complex plane and even if we consider JU,) only along
X
the x-axis. where it equals 1/( I + x 2 ). the values of .r are restricted to lie within
the unit circle.
Thus, we ~cc 1hc.11 tht! bchavior of a real function f (x) is actually innucncctl by
how its corresponding complex function /(:) behaves as z. varies in the complex
plane. The calculus of functions of a complex variable : is an extremely rich
Figure 2.12
subject. and one that has many applic:11ions in applied mathcrnalic.:s (Chapters 18
The rcgioo Id I in rhc complex plane. and 19).
2.7 T;:iylor Yi 99

2.7 Problems
r"
I. Show 1hat :__ - 0 as n -> oo for all x.
II ~

2. The coenicients of lhe expansion of ( I + .r)" can be arranged in the following fonn:

JI

0
I
2 2
3 3 3
4 4 6 -l

Do you sec a pattern in going from one row to the next? The 1riangular amingcmen1 hl!rc is called Pascal"s
uiangle .

3. Prove 1ha1 L k!(NN-. k.)! = '!."'.


N

b:O
'

4. Show that 1hc binomial· series truncates if tt is an in1l·gcr.


5. Prove 1ha1 IR,,1- 0 as II for the binominl series. provided I .r I -::: I.

6. Show 1hat In x = L(-


"- .· (,; - I)"
1)"+ 1 ·
II
for O < .r < 2. Use l11is result 10 show 1hu1 the altcmaling harmonic

series is equul to In''""'


2.
7. Assuming 1hat Equation 6 is valid for imaginary numbers. show 1ha1 J• .;. .; cos x + i sin .r.
8. U,1! l·.qua1io11610 <:akulale the value of 1• 10 six decimal pbL·.:,. {Assumt' initi;1lly that t· ~ 3.)

9. Find the Maclaurin ex pans.ion of xe - x~. (Hint: Do nor use Equation 3.)
JO. Find 1hc Macl:iurin expansion or JT+x.
:-..
11. Prove that if/ (.r) :;7 L utt(x - l')". 1hcn this series is lhe Taylor series for f (.r). even if it is obwincd wi1hou1
11:=-0
using EquaLion 3. as. we did for Lan- 1x and In( I - x) in the previous section. (Him: Derive an expression for
the 11,,.)
12. Show that the derivative of EquatiL>n 7 gives a( I + x)u-l.
13. Show that

-er a(a + I) a(a + l)(a + 2) ,


( I +.t ) = I- CtX + - - - X 2 - - - - -- -X- + · · ·
2! 3!

= °"
,)0

(-l)"(et + n - l)tx"
~ 11!(a - l)!
11==0

Rcnicm~r that O! = I. (We will discuss foc1orials in the next chap1cr if you are not familiar with 1hem.)
14. Show lhal ( I - x 2) I/'! .= t( 2
")} (~)
n=O (11 !)- 1
211
100 Chapter 2 / Infinite S<•riC'~

15. Show that


4 6 8
(a) I+ x 2 - ~ + ~ - ::_ + •• · = l + x sin x
3! 5! 7!
x1 x4 x6 ,in x
(b) I - - + - - - + · · · = -
3! 5! 7! X
I x2 x4 x6 I - cos x
(c) - - ·- + - - - + ...
2 4! 61 8~ x2
=---
00 00

16. Consider two power series L a,,x" and L b,,x". ll is straigbtforward to show tha1 if we denote their product
::0

by L c, x", then
1

n=O

"
= Lai.h,,_1;
k::::0

17. Show that 1


o
00

e-.r.l cos ea dx = ./ii t. - a2 /4 by expanding cos cu


2
in a Maclaurin seril.!s and integrating tenn
by1erm.

18. Tn 1his problem, we'll show that even if all the dcriva1ivcs of a funclion exist and are continuous al some point.,
lhe function sLill moy not have. a Taylor cxp:1nsion aboul lhal poinL A classic example of this bchavior is giVl!n
1 2
by f (.r) = e- t/ r • x # 0. and f (x) = 0, x = 0 (Figure 2.13). Show that the Maclaurin expansion of e- l/.c is
e - t/ x? =
0 + 0 + • • • + 0 + R,, and 1ho.L the remainder is equal to the function itself for a.II values of II and
docs nor vanish. except for x 0. =

~ 5L
1.0

figure 2.13
The fonc.tion f(x) = (J - I / A: plotted aguin, l x. -I I -;

19. Use any CAS to find the Maclaurin expansion of ~o);(X sin t) to fifth order in r .

20. Use any CAS to find the Maclaurin expansion of tru1.h(x 3 - x) to fifth order in x .

21. Use any CAS to find the Taylor series of x 3 In x abou1 the point x = I to si,;th order in (x - I) .
22. Use any CAS to find the Taylor series of x 3 sec x ahout the point x = rr 10 fifth order in (x - JT).

C
2.8 Application~ of Taylor Seri es 101

2.8 Applications of Taylor Series

Taylor series have aumerous applications in practice. For ex.ample. suppose we


want to know if the integral

I=
1-.,-x
O
sin xd
x-

is fini1c. The region around x = 0 is problematic because the denominator ap-


proaches zero. Lei's ex.pand sin x about .x = 0 and write I as

{' I ( x3 x5 )
I= Jo x2 x - 3 + 5! +... dx

=
i 0
i
-dx -
X
1I O
x
-dx
3!
+ 1' 0
xJ
-dx
5!
+ -· •

All the inicgra.ls beyond the firn are finite, bu1 the first integral diverges because

ld\·
__:_ = [ In 11
]I = oo
!o
O X 0

and so / itself diverges.


We can aJso use Taylor series 10 evaluate integrals. The integrJ.1

tJ

1
00
I= -·-dx (I)
o eX - I

occurs in the theory of blackbody radiation. This integrnl does not appear in
Lhe CRC Mathematical Tables nor in Gradsh1eyn and Ryz.htk (see I.he references
al rhc end of Chapler I). We can evaluate it by multiplying the numerator and
denominator of the integrand by e-x and ex.panding the denominator in powers of
r using the geometric series.

The series here is unifonnly convcrgen1 for x O and so we can inlcn:lrnngc orders
of summation and intcgrn1ion to write

00 00
-6' I -6'_!_
- L (11 + l)J - L ,,4
n~O n=I

The summation here is in most handbooks (see also Section 3.7) and is equal to
Jr J /90. Thus. we see thal / = ,rJ/ 15.

C gl
102 Chapl •r 2 / In in1t

II rums ou1 that rhc integral

I (x) = lo r l,- • u
l
du

which occurs in 1hc kinetic 1hcory of gases and in many other areas. cannot be
evaluated in 1em1s of elementary f unclions. We can ob1ain a useful power series
for / (x) by expanding e-,j~ in a power series in II and tl1en integrating term by
1em1

6
,,t{ ]
/(.r) =
1
o
.1: [
I-
111.t
II-+ 11 11
- I + I + ... d11
-· 3. 4.

or

(2)

This is an aJtemar.ing scrie~. :-.o the error we incur by 1runcating is less in m;1gni111de
than the first 1cm1 ncgh:ctcd. If we keep four lt'm1s to C\'aluate / ( 1/2). the error
will be less than ( 1/2) 9/(9 • 4!) = 9 x Io-6 . We gel / ( 1/2) 0.46127 compared =
10 the ,,cceprcd ··exac(" ,•::ii uc of 0.46128 .

Example 1:
1
s-in r
Use the Tnylor series 10 evaluate
1
. -·- · dx to four-rlucc accuracy.
X

SOLUTION:

l
,0
I sin x
- -dx
x
= 1 o
1
[
I- -
x2
3!
+ -x°' - -x + xii
5!
- + · · ·]
7!
6

9!
dx

I I I I
=1 - - -
3 · 3!
- - - -7-7!
5 - 5!
- + 9-9!
- - + ···
If we trunc.ite this alternating ~eri s arrer the 1hird tcm1. we have
un nccuracy of 1/(7 • 7!) =
3 x 10 5 . So. if we use jusl three 1cmii- .
1

1 O
sinx .
- - dx = 0.9461 I compared lo the acceprcd v:.iluc nt 0.94608.
X

It is also of interest in the 1heory of blackbody radi ,llion to know the bchavior
of the integrand in Equntion I for small values of x . The inlcgrand i::, of the form
0/0 ar x = 0. bur if we replace 1r' by I + x + x 2 /2 ! + • · •. we Sc!e 1ha1
103
x3 x x2 3
-- = ---
., - - = - ----
X = x2 - ~ + ...
e' - I r t+-2 + ··· 2
x+-+···
2!
This is probably a good place to imroduce a frequently used notation thal is
very useful when working with power series. We write f (x) = 0 lg (x) Ias x - a
if f(x)jg(x) is bounded as x ➔ a: in other words . .f(x) = O[g(x)J as x ➔ a
if 1/(x)I .::S Mlg(x)I as x ➔ a. where M is a constant. Usually g(x) will be a
power of x. For example, we write f (x) = I - cos x = 0 (x 2 ) a.-. x --+ 0 because
cosx =I- .x 1 /2r + x-l/4! + • • •asx ➔ 0; and so ( I - cos x)/.r 2 - 1/2 as x---+ 0.
We will use this notation in a bookkeeping sense 10 keep track of powers of x that
we neglect when truncating series expansions.

hample 2:
One stalistical-mechanical theory of solutions of strong electrolytes (such as
an aqueous solution of sodium chloride) gives 1he energy of the solu1ion as

£ K R __ x 2 + x - x (I + 2:r) 1/2
(' )- 4rr{3R 1

where f3 = I/ kR T. R is the average radius of the positive and negative ions.


and .r = K R. where K is a known parameter. Show that E g0t.,-s as K·1 as
K-0.

SOLUTION; We need to write E as a power series in x. Using 1he binomial


series

= x-~ + x - x [ I + 2.x - -(2.x)2


- +0 3 ]
(x )
2 8

=x 1 +x-x-x 2

or

Anolher ex.ample from elec1rolyLe solutions is I.he following: One theory


expresses Lhe osmotic pressure of a solut.ion of a s1rong electrolyte in terms of
a f unc1ion CJ given by
104 lh,1p1t·r !. ,' lnfmllt· S · rie

rr = -33 [ I +x - - I- -2ln(I +x) ]


x I+ x

where once agnin x = K R. The small-kappa (dilu1e solution) behavior of a is g-iven


by

a = x'.l
3 I+ I x - [I - x + x .. - . x l
+ x 4 + 0 (x 5 ) J

- 4x°' + O(r)~ ] }
2 3
-2 [ x - x +x
2 3

No1c lha1 we keep I.rack of the powers of x in Lhe expansions of 1/( I + x) and
In ( I + x) by writing O (x 5 ) lo indicate the firs1 power of x neglected in each
expansion. Without this no1at.ion 10 remind us, it would be easy to make the mistake
of neglect.ing some power of x in one lem1 but not Lhe olher. This is particularly
important when a number of lower powers cancel, as they do (the/'. x 1, and .x 2
powers cancel) in the above two cases.
Sometimes power series expressions are a little easier to use 1han rHopitaJ's
rule to evaJuate indetenninale expressions such as 0/0. ror example. let's evaluate

• x - sin x
I1m 1
,r-,o x- tan x

using the series expansion for sin .x in Lhe previous sect.ion and

3 5
tan x = x + -x3 + -2l"15 + .. •

(This expansion may be found in any handbook.) We gel

3
x - [x - ~ + O(x 5 )]
. x - sin x . 6
I1m - - - = 11m
.r-•O x 1 tan x .r~o x 2 1x + O(x 3)1
I
=-
6

Example 3:
Use Taylor series to evaluate

SOLUTION: Firsl lei x = l/11 and look at the limit 11-> 0.


2. 8 App Ii r ,1t ions o( Taylor Series 105

. -
I1m (I + 311)1/2 - (1 + 11)1/2
-------- 1·1m -I
= u-0 {I + -31.1 + 0( u-") - [1 1)] l
+ -II + 0( u·
u-+0 II U 2 2

= u-0
lim ~=I
U

To end this section. we'll use Taylor's formula lo give a simple proof of lhe
second-dt:rivative criteria for an exLrcmum. Suppose thal a is a cri1ical point so
=
1ha1 J'(a) 0. Then Equation 7.1 wi1h n is =\
I ,, 2
J (x) -
.{(a)= -2·r (.;)(x - a)

s
where a< < x. If x is close enough 10 a so rhat J"(O has the same sign as
f"(a). Lhcn we sec that f (x) - J (a) and J"(O have Lhe same sign. If J"(n < 0.
then f(x) < /(a) and /(a) is a maximum. and if P'{{) > 0. then j(x) > j(a)
and f(ci) is a minimum.

2.8 Problems
t t·-• - I+ X
1. Doe~
loo ,,

dx converge?

2. Use any CAS to evaluate the integral in the previous problem.


1 1an- 1 x
3. Does
1
--.,-dx converge?
x·o
4. What happens when you try 10 evaluate I.he in1cgrnl in the previous problem wilh a CAS?
I/~
5. Ex pond ( I + x"') 112 in a binomial series and cvalua1e lo (I + x 4 ) 112 dx to four decimal places.

6. Evuluatc I.he intcgml in 1hc previous problem using any CAS.


7. n,c in1cgrjJ
K(k) = f'/2 _ __d_0_ _
lo (I - k 2 sin 2 0) 1l 2
called a cornplcle cllip1ic intcgrd.l of the firs! kind, arises in u '-ludy of a pendulum of arbil:rur)· amplitude
(Section 3.5). The in1egral cannot be cxprc~sed in terms of clemcnla:ry functions. but derive the first few tcm1s
of an ex pans ion of K in powe~ of k.
8. Use any CAS to do the previous problem.

9. Evaluate la I

sin Jx dx to five-decimal place accuracy.


10. Use any CAS lo do Lhc previous problem.
Xi

11. Evalua1e -
1
+ -2 + -3 + -4 + · · · = '°'n-1.
L - - 1n closed form.
2! 3! 4! 5! 11=2 ,1!

gl
106 Chapter 2 / lniinir Seri

.
12. Denve a power series for f (:c)
. = 1·t -11d11-.
o I+ u-t

13. Find J' IOJ(O) for sin Jt / Jx. Hint: Do ,wt differenLiate rJ1e expression ten times.

14. Use any CAS to find J'(O) for the previous problem. Do you run inlo any difficulty? Is /'(O) linitc?
·"\. i • Iii , ~
15. Show lh!lt
inlegrnting
1u
.:- " r- cos bx d.x
2a
tenn by term. Use the foci that
=~ e- b· / 4,r by cxpa.nd1ng cos bx inn Taylor scril:s aboul .r = 0 and

• I · 3 ·· 5 · · · ( 211 ·- I) r-
dx =- - - -- - -...;;:r
2n la2n+ I

(Problem 1.9.5).
00
b
16. Show 1hal
1 o
in1egraLing term by term.
e ·•" ~inh bx dx = -,--,,
a- - l>-
!bi < a by expanding sinh bx in a Taylor serie!s at x = 0 and

17. We defined a p series in Section 3 by S( p) = t ~-


oc

11=)
nP
Show that

where for now p = 2. 3..... (We'll gencl"Jli,ze 1J,1is formula in the next chapter. where we'll learn how to
handle foctoriaJs for non-integral value, of p.)

18. The 1hcory of aqueous solu1io11s of strong electro'l ytcs expresses. 1hc free energy of Lhe soluIion in 1crms of a
quantity r defined by

r = _3_
(K R)-'
I· ln(I + KR) - KR + K1,._R2
,
I

where R is I.he average radius of the ions and K is a known parameter. Show that

r-. I - .:.KR
3 3
+ ~(KR) 1
+ O[(KR) 3I
4 5

19. Use any CAS to carry out the previous pY'oblem 10 Ol(KRf'il.

21:1/2 r2 x-t
20. Show tha1 - cos(x sin f) d1 =I- :..... + - + 0(./ 1
).
;r o 4 64
21. Use any CAS to deterine the series exrpu:nsion in the previous problem throug.h O(xio).

22. Use Taylor series to derive l'Hopilaf s rule.


23. For the chemi<:al reaction A + B-+ C. the concentra1ions of A and 8
iL" a func.:tion of time are given by
1
,.= - - - n - - - . w ere Ao, an · 0 arc I. e 1mua cunccmralions an d ,;.'· .1s a con,1:mt. f.md I.he lim1L1
,.r I A(t)Bo h dB h ... I . . . .ng
Ao - Bo AoB(I)
expression if A 0 = B0 . Hin1: A(f) = B(t) if Au= 8 0 .
2.9 Asymptuti< F:-.p,1mions 107
2.9 Asymptotic Expansions

Taylor series are most useful for small values of (x - c) because Lhe tem1s usually
decrease steadily and only a few tem1s need be used to achieve a prede1em1ined
accuracy. Frequently. ho\Vever. we would like 10 know the hehavior of a function
for large values of x. For example. we might want to know how a sys1em behaves as
lhc number of particles increases. or how it behaves at long times as it approaches
equilibrium.
For example, the following integral arises in ::1 quantum-mechanical lrcatmcnt
of a hydrogen molecule:

(I)

and we arc ofccn imcrested in the value of Lhis integral for large values of x.
=
Integrate £ 1(.r) repearcdly by parts (with "11" 1/z." and "du"= -e-=dz.) lo gel
an expansion in inverse powers of x:

(2)

Equation I is an identity; Lhere is no approximation yet, buL there will be if we


neglect the integral on the righ1 side. Lei ·s look at the error involved if we do drop
it. Since z. > x throughout 1he range of integral.ion, we have

If x = 5, 1hc error involved is 5.2 x 10- 5. Thus we cnn use

X>5 (3)

to achieve four-decimal accuracy. Equation 3 gives £ 1(5) = 0.00112 ± 0 .00005,


or 0.00 I IO < E 1(5) < 0.00121. compared to the "exact"' vaJuc oro.oo I I 5.
II migh1 be tempting lo con1inue rhc integration by pans in Equation I and
wri1e

e-., = (-1)"11!
Ei(X)=-I:--
x n=(l
x"

but this series diverges for all .x. The ra1io test yields

.
I1m lll,,+1(X)
- - - I= 1·1m 111+11
-- :>
n-,oo 11,,(x) "-'"JO x

for any value of x. Nevertheless. we obtained an excellenr numerical approx ima-


Lion 10 £ 1(x) by using the truncated version, Equa1ion 3.
To see wha1 's going on here, wri1c Equation 2 more generally as

C
108 Ch,1ph•r 2 .1 Jnfinile Series

(4)

= S,,(x) + R 11 (x)

where Sn(x) denotes the sum up through the 1/x" term and Rn(x) denotes the in-
1egral. which represents rhc sum of rhe series af1cr the 1/xn term, or rhe remainder.
In this case

and

for a.fixed value of 11. The ratio of the error to the terms kept tends 10 zero as .r ➔ O;
in other words. the approximation improve~ a~ x gels larger. If we divide both sides
of Equation 4 by Sn (x). we see that

as x - oo (6)

for a tixed value of 11. We express Equation 6 by writing

where Lhe ~ sign means is a.nmptotic to.


GeneraJly, we say that

(7)

is an asymptotic expansin11 of j(x) if for each 11

as x ➔ oo (8)

The lcrm in bracket, here is R,,(x). lhc remainder after 1/x" term. Thus we can
write Equation 8 as

(9)

It is common practice LO write


Xl
tJ,,
f (x) ~~·
L x"
n=O
- (10)
2.9 A:;ympln!i1 f,p,Hl~ions 109

and 10 caJI Ihjs 1heasymptotic series of /(x).


We must bealen to the foci. however.
1ha1 you mus1 truncate the series at some value of 11. This subjec1 provides a good
example where we can 1101 interchange the orders of summation and integral.ion
in Equation I.
The asymplotjc series of£ 1(x) is

00
t' ' (- l)"n!
Ei(x)~-L - - - (I I)
x n=O x"

'e-.1
This series satisfies the requirement of Equal.ion 9, since I R,/r) I = '-'-··_ and
xn+I t S"
Ix n Rn (x) I ➔ 0 as x _. oo for each fixed n. It is convenient 10 rewri1e Equation I I
as

( 12) 5 10 n
-I

Table 2.2 lis1~ 1hc successive tcm,s and the corresponding successive partial sums
of xr E 1(x) for x =~-(See also Figures 2.14 and 2. I5.) Notice 1ha1 the terms first Figure 2.14
decrease to 0.0384 and then increase. This should come as no surprise because The- ~ucccS_..:ive terms of 1hc asymptor.iL·
.setit·~ .u' £ 1(x ) given by Equa1ion 12 for
Equation 11 is a divergent series. x = 5 pl,ottcd a~aimt 11.
Since Equation 11 is an alemating series, it$ actual sum lies be1wcen any two
consecu1ive partial sums. If we scan down Table 2.2, we sec that the tightc,1 pair
of pan ial sums is the pair (0.8704, 0.8320), so we expect the ··exact .. val uc ( which
s,,
is 0.85211) to lie between these values. 1.0
A sirrular calculation for x = I 0 shows that Ihe successive terms decrease
until n = I 0. and then increase arr er 1.ha1. The tighh.:,1 pair of pani al sums is 0,5
(0.915 46, 0.915 82), and the "exact" value is 0.9 I 5 63. For .x = 20, the successive
tcnns decrl!a~c unIil n = 20 and give essen1ially 1hc "exact" value. 0.954 370 91
5 JO n
(Problem 6).
It's possible to estimMe Lhe value of II al which 1he lerms in Equa1ion 12 have Figure 2.15
a mini mum value. For a fixed value of x. we wish lo determine the minimum value The ,ui:ce.s ·ive panial .,urn, of the
asymp101lc seri e. x r' E"ttx) given by
of n !Jx". To do I.his, first 1ake 1hc logari1hm of y = 11 !/ x" 10 get E<1uai•ion 12 for x = 5 ploncd again:.r n.
rt

In y = In 11 ! - n In x = L In j - n In x
}=I

We can find lhc minimum of y = n!/x" by finding 1hc minimum or ln y because


In y is a monotonic fonction of y. Now. approximate Ln

J=I
In j by 1"I
In;:; dz. and

differentiate In y wi1h respect to II a~ if it were a continuous variable. I.hen set the


result 10 7.ero. yielding

d In y
- - ~Inn - In x =0
d11
or n ~ x. This result is in accord with our above calcu lalions for x = 5. I 0, and 20.
al
110 Chapter 2 / lnimilt· Series

Table 2.2
The .successive term,;, and 1hc partial sums
ofxe£1(X) for X = 5.

" successive tcnns partial sum

0 1.0000 l.0000
-0.2000 0.8000
1 0.0800 0.8~00
3 -0.0480 0.8320
4 0.0384 0.8704
5 -0.0384 0.8320
6 0.0461 0.8781
7 -0.0645 0.8136
8 0.1032 0.9168
9 -0.1858 0.7310
JO 0.3716 1.1026
11 -0.8175 0.2851
12 1.9620 2.2471
13 -5.1012 -2.8541
14 14.2833 I 1.4290

Example 1:
Dctcnnioc the asymptotic bchavior of
,,- :cu
- 'l-r/11
u-

SOLUTION: Firsl lel:: = .rn to gel £ 2(x) into the form

Now iniegrnre by parts repea1edly. aJways using "d v" :::. e- ·d;. to gel

E2(x)
- - =('
-x [
-I2- -+
2!
-'( + .. · + ( - I) ,, -
(11 - I)!]
--
.\" x XJ XJ Xn

= S,,(x) + R,,(x)
Reph.1cc ~11 + 1 by x 11 + 1 in the remainder tenn and write

and so we see Lhat Ix" Rn (x) I ➔ 0 as .r ~ oo. and so Equation 9 is satisfied.


2.CJ Asymprotic bp.m,ion!- 111

Therefore. we can wrire

and in panicular that

as .r - oc

In 1he nex1 chaprcr, we \vill discuss some functions Iha1 arc defined by lheir
integral expressions. One of them is calle<I the error function. which is a central
function ia sla1istics and one lhat ocrnrs in 1.he kinetic 1heory of gases. An integral
that is closely related 10 the error function is

The asymptotic formula for / (x) is obtained by repeated imegration by pans


(Problem I):

oo _.!
I I I e ~
+ (-1) n+I 2 ( 11 + - )
2 ( 11 - -
2)
• · · -
( 2)
J x
- "1-
:-''
d:
1
( I 3)

l11e corresponding asymp101ic series is

e-,-~[1- -I , +-.1·3
l(x) ...... - - ,- : ... ]
2t lr- 2-x 4

t s,,
0.95 ~ •
or ■ ■ •••••••

o.94 I
(14)

5 10 (5 II
Figure 2. 16 shows I.he succL'ssive partial sums S,, ( x) of this series plotted agai ns1
11 for x == Js; they arc also listed in Table 2.3. figure 2.16
Tiie ucc·c"i \i! ~11ial _umi- o f lhc
If we scan down Table 2.3, we sec 1ha1 the 1ightes1 pair of partial sums is
2 ,l-~ymp101il· eric<1 2.xf'"..: /(x ) g1\cn hy
1he pair (0.94618, 0.94684), so we expect 1ha1 the "ex.act" value of 2re-'' / (x) lies Equation 14 for:<= Js plouc<l against 11.
between these values. The "exact value·· is 0.94661 . Noh: the scale on 1he vcnil-;;1J :i '" ·
11 2 Chaprer ::! / lnlinire Sl·rie-s

Table 2.3
The succes.sivc p!lrtial sums of
2
2.xe-'" /(.x) given by Equal.ion 14 for
x=./8.
II s,. n s,,
0 1.000 00 10 0.946 94
0.937 50 11 0.946 16
2 0.949 22 12 0.947 28
3 0.945 56 13 0.945 52
4 0.947 16 14 0.948 48
5 0.946 26 15 0.943 12
6 0.946 88 16 0.953 52
7 0.946 38 17 0.932 06
8 0.946 84 18 0.979 00
9 0.946 34

The asymptotic series 1ha1 we have discussed in this sec1ion are divergent
asymptotic series. Nol all asympto1ic series diverge; some converge for large
vaJues of x. But. divergent or not.. asymptotic series can be used lo obrain good
approximations to functions for large values of x.

2.9 Problems

l. Derive Equal ion l 3.

2. Show 1ha1 J (x) = lo -.,-


- .Tld/

/-+I
~-
1 iXi n (211)!
~(-1) -.,-.
x L., x-11
11=0

,; re-.r' dt
~ -x~I Loo + I)!
1
(211
3. Show I.ha! g(x) = -
I1
~
+ -I (-1) 11
x 2Jr ·
0 ,,=<>

4. The following integral arise!\ in a t.realmenl oflheradiation emiued from a linear antenna: C(x) = f
.l
00 cos u d11 .
/I
Derive an asymptotic series for C(x) in terms of f(.x) and g(x) as defined in Problems 2 and 3.

5. Derive an asymptotic series for lo e- 11


::;in II d II by letting - =.rn. then by using the Maclaurin expansion
1
of sin(zjx ). and Lhen by integra1ing tenn by 1ern,_ Compare your result to fxi e-.rn sin II d11 =- -,.
lo l + x-
6. Calculate 1he successive terms and !he firsl' 25 partial sum5 of Equation 12 for x = 20 and show 1ha1 the
successive terms decrease until 11 = 20 and give the result 0.954 3709.

7. Verify the entries in Table 2.3.


11 3

8. We argued just before Example I 1ha1 the value of II at which successive 1cm1s in Equation 11 have a minimum
value is approx.imatcly equal 10 x. Table 2.3 shows, however. thal Lhis value of n is approximately .x 2 (n == 8 and
x == .,/8) for Equation 14. Argue that 1hi~ ~hould be the case for Equation 14. Hint: Writ.e (11 - ½)(11 - ~) · · · ( j)
(2n - 1)(211 - 3) - · • ( I)
a s - - - - - - - - and Lhen use 1he relation {( 1)(3) · · - (2n - 3)(211 - !)} x 12 - 4 · 6- · · 211) == (211)!.
2n

9. We can derive an asymp101ic e...,pansion of F(s) == fo?O f(x)e-udx for large ,•a!UL:~ of s by inrcgrating by
parts repeatedly. Show that

Use this result to show tha1

e _ 51· cos x dx == -I - -I
s x3
+ -I - -I
s5 s5
1 o
e- r sin .x dx

Show lhat the remainder term (Lhe in1egral on the righ1) is le~s than s- 6 and calculate F( I 0). Show 1ha1
F(s) = s /( I + s 2 ) and Lhat F( I 0) = 0.090 09900 • • -. Compare your result.s.
sin x
i
oo
10. Use the method in the previous problem to calculate F(s) = t' -
1
--dx for s = 10 and estimate the
0 X
error. Given that F(s) = lan- 1
( 1/s). compare your result to the "exact" answer.

References
Frank Ayres. Jr .. and Elliou Mendelson. 1999. Cnlculllx. Sclumm's Ou1linc Series,
McGraw-Hill
C.H. Edwards. Jr.. and David E. Penney. 1998. Ca/c11lus a11d Analytic Geometry. 5th ed ..
Prcnticc-H al I
LS. Gr.1dshtcyn and l.M. Ry7.hik. 1980. Tahle nf Integrals. Serie,f, and Products, 41h ed.,
Academic Press
Elden R. Hnn~l'll. 1975. A "Jiih/i- of !jeries 1111d Proi/111'1,, Prentice-Hall
I.T. Hir.ichman, 1962. /11.ft11ire Saies. Holl. Rinehan and Winston
James M. Hyslop, 1954, Jnfmire Series, 5th ed .. Oliver and Boyd
L.B.W. Jolley. 1961. Summation of Series. Do\.cr Publitation,
Konrad Knopp. 1956. Infinite Serii•.f and S1·q11n1ces. Dover Publ ical ions
\\'itold Kosmola. 1998. Adwmi:,·d rnh-11/ux. A Fri(·11rlly Apprnncl,. Prcn1ice-Hall
Jerrold Mars.den and Alan Wcin,tein, 1985. Calrn/11s /, II. and I//. Springer-Vcrlag
Ea.rl D. Rainville. 196 7. Infinite Series. Macmillan Co.
Murray Spiegel. 1963. Advu11ct·d Calrnllls. Schaum 's Outline Series. McGrnw-Hill
Leonhard Euler (1707-1783). one of rhe most outsranding and certainly mos1 prolific mathematicians
of all time. wa...: born on April 15. 1707, in Basel. Switzerland. His father wa ...: a Protestant minister who
had smdied mathematics with the Bemoullis. Euler entered the Univer:-i1y of Bai-.el at age 14 10 prepare
for the ministry, but he soon discovered his talent for mathematics. Although he switched from religious
i.lUdies to marhcmatic.:.,, he remained devout throughout his life. He published many papers on a variety
of mathematical topics when he was a student. Upon tomplerion of his univc~ity i.ludies in 1726, he wa..;
offered u posit.ion at the St. Pe1ersburg Academy of Science in Russia. in part due to the efforts of the
Bernoulli family. Here he wa.~ surrounded by gifted scientists. and Euler was able lo study every facet
of mathematic:-.. both pure and applied. In 1734, he married Katharina Gsell. a Iota! Swi:,s woman. The
marriage was long and harmonious, producing 13 children. of whom only 5 survived p:i.,1 infant:y. In 1735.
he sufTcrcd from a serious fever. resulting in the loss of most of his vision in his right eye; however. his
produc1ivity was nol hindered. Due to political turmoil in Russia, Euler accepted a position al rhc Berlin
Academy of Science in 1741 at the invitation of Frederick the Great. During his 25 years in Berlin. he
published close to 400 articles, including a popular science book, Leners 10 u German Pri11cess, his 111<1.~t
widely read book. Personnl differences with Frederick led 10 his return to St. Peler.burg in 1766. In 1771,
he lost his wife and then the sight in his good eye. In spire of 1he.--c tragedies. he continued his prodigious
mathematical output. aided by his two sons and other assistants. He died of a brain hcmorrhage on Scpll·rnhcr
18. 1783. Arter his deal h. the Academy at St. Petersburg conti nuc<l to pub Iish his unpublished but finished
work for almost 40 years. There is no field of mathematics. bolh pure am.I applied. to which Euler did not
make important contribution.~.
CHAPTER 3
Functions Defined As Integrals

In applied malhematics, we frequently need 10 evaluale integrals such us 1x e- 1,1 ~du


.{ sin 11
1
. . . . •
or --du for various values of .r. but II tum~ ou1 1ha1 11 1s 1101 possible 10
(I II
express either of 1hcse integrals in tem1s of any of !he func1ions lhat we study in
calculus. There is no simple funclion whose derivative is e -· ~ or (sin x )/x. We say
that 1hc abo\'C in1cgr.1ls i:annot be cxpn:1'scd in terms of elementary functions. an
admincdly somewh:u vague 1cnn. This situation occurs oflen in applied problems.
and we use such integrals to d('./ine new (non-elementary) functions. which if they
occur frequcmly enough. become a, well accepted as the .. elementary" functions.
We sha 11 nlso sec in Chapter 12 that di ffcrent ial equal ions can serve 10 define new
functions . .!>Uch as Be.!,~cl func1ions and Legendre functions.
Jn this chap1er, we will di~uss a number of Functions thal are defined in one
way or ano1hcr through integrals. In lhc first two sections, we discuss 1hc gamma
function. and ils close relative. the beta function. We'll see that a gamma function
gcneraJizcs our idea of a foctoriul. in that it equals a fac1oria,I for in1cgcr values of
x, but is defined (and useful) for non-inicgcr values. In Section 3, we discuss the
error funcLion. a central function of s1a1is1ics. and one that occurs in a wide variety
of physical problems. In the next twu scc1ions. we discuss the cxponenLial integral
and then elliptic integrals. and in Seer.ion 6, we discuss the Dirnc delta funct,ion. a
powerful manipu la1 ivc 1001 used in a number of quantu m~rnecihanical problems. Jn
the lasl section. we discuss Bernoulli numbers and BcmouUi polynomials. These
arcn·1 really funclions defined hy integrals. but they are used in dealing wilh
summn1ions. which arc the discrete versions of integrals. The standard reference
for the material of this chapter (and much more) is Abramowitz and Stegun. (See
the references at the end of the chapter.)

3.1 The Camma Function

The expression 11 !. equal to I • 2 • 1 • • • 11. occurs when we enumerate pennutations


and combinations of Lhings. such as the number of ways that N molecules can 115

C gl
116 Chapler 3 / Functions 0(•finl'd As lnlegr.:ils

be disLributcd over 11 molecular quantum states. In the l 700s, Euler introduced a


function that yields"! when" is a positive integer, but is well-defined when n isn't
a positive integer. The function is caJled the gamma fimcrion and is defined by the
integral expression

x>O (I)

Notice thal the integ-rand is a function of x and z and Lha1 the resulting integral is a
function of x. If x is a posi1i.ve integer greater Lhan or equal to 2, we can integrate
f(x) by parts. Letting e - :.dz. be "dv" and ;.;.r-l be·',/". we ob1ain

If we compare the las1 integral here to r (x) in Equation I, we see that it is equaJ
tor (x - I). so we have

r(x) = (x - l)r(x - I) (2)

We can now substi1u1e r(x - I)= (x - 2)r(x - 2) and so on into Equation 2 10


get

r(x) = (x - l)(x - 2) · · - r(J) (3)

where

Therefore. Equa1ion 3 reads

f(x) = (x - l)(x - 2) ···(I)= (x - I)! X =2, 3. · · · (4)

Up to I.his point. Equation 4 is resuicted to integer values of x ~ 2, buL we


can use ii to define factoriaJs for olher vaJues of x. If we let x = I in Equation 4,
we have re I) = 01. But Equation I is perfectly well-defined for x = I. and yields
r (I) = I. Therefore, we can say that O! = f( I) = I. a relation that you may have
come across before. The fact that 01 = 1 shouldn't bother you; the familiar relation
11! = I• 2 - 3 · • · 11 is true only for positive integers.
We can use Equation 4 to define a factorial for non-integers. This extension.
or generali7...alion. of a factorial turns out to be very convenient in practice. We can
let .x = I/2 1n Equation 4 to write r ( I/2) =(- I/2) !. Now we can use Equation I
to evaluate f( 1/2). Letting z. = 11 2 , Equa1.1on 1 becomes
3.1 The G,u11ma Func1ion 11 7

so that r ( 1/2) = (- 1/2) ! = .fir (Problem I0). Wha1 about r (3/2) = (1/2) !?
Simply use Equation 2 to write

r(3/2) = ( l/2)f( 1/2) = fi


2
We can continue 1his process and write (Problem 8)

" ~ I (5)

for any integer value of n.


You might~ wondering why we would wani to consider factorials of numbers
other than positive integers. The definjtions of many functions that occur natuml ly
in physical problems involve quantities such as r( I /2) and r(I / 3). For example.
we' II study Bessel functions in Chapter 12. and 1he definiLion of 1hese func1ions
involves gamma functions. The following Example gives another reason.

Example 1:
Evaluate focc .t 1' u..rd dx in 1cnns of a gamma function (a > 0).

1
SOLUTION: Let 11 = ax.i.• then x = (11/0) 114 ' dr. = -4al/-l
-11-JJ-ld11 and

{00
Jo .xe
- nx'' I
l.X
I
= 4a1J2
100 -;;T7i
o
l!- u d r (l/2) •
u = 4a l/ 2
I (rr)
= 4 -;;
1;:i

The definiLion of r(x) given by Equation I rcs1ricts x to values greater 1han


zero (Problem 9). but we can use Equation 2 in the fom1

f(x + I)
r (x):::: - --
x
(6)

10e..x1e11d our delinir.ion of r(x) 10 values of x less than zero. For example. let's
use Equa1ion 610 calculate r(-1/2). (We'll s.tart abandoning the factoriaJ notation
now.) Equa1ion 6 gives

Using a combinalion of Equations I and 6, we can calculate r(x) for all values
ofx. other than Oand negative integers (see below).
118 h, pier l / Function Defin d " Integral

Example 2:
Calculmc f (-5/2) .

SOLUTION: We USC Equ:uion 6:

There are some special values of x that we have avoided up to now. According
X to bo1h Equation I (if we:: let O - O+) and Equation 6. f (0) is noL defined.
Furthermore. Equation 6 gives us r(- I)= r (0)/(-1), which is also not defined .
Figure 3.1 show~ r (x) ploncd against.r. Notice that r(x) has a vertical asymptote
when x is a negative integer. Thus. f(x) is not defined when x = 0 or n negative
Figure 3.1
The gamm:i function r(x) plotted number. The graph of r(x) is given by Equation I for x > 0 and Equation 6 for
ag:d n~t x. X <0.

Example 3:
Show that lim f (x)
x-,o-
= - oo and thar
lim f (.t')=limf(-1-t')= + · whcrc r 0.
x- • - 1- !'.:_,.0

SOLUTION: We use Equation 6:

.
I 1m r (.r) = 11m
· f(x + I)
--- = 1~( I) .1-.0-
1·1m -I = -oo
x-•0- x--.()- X X

Similarly. for € > 0.

.
I1m r (- 1 -E)= 11m
· -re---=
>
~-o ,-,o - 1-

The reasoning that we use in Example 3 can be used 10 verify 1he bchavior of r(x)
in Figure 3.1 at all the negative integers.
Using Equation 6. it's possible to reduce the cvalumjon of r(x) for any
(non-negative integrnl) value of x to the evaluation of r (x) for I ::: x .::: 2. For
example, f(6 .13) = (5.13)r(5.13) = (5.13)(4.13)(3 . 13)(2.13)(1.13)r(l.13) =
(159 .6) f(l.l3) . Therefore, mathematical lables list numerical values of r(x)
for values of x only between 1.00 and 2.00 (Table 3. 1).
3.1 The G.1mma Fun( 1io11 119

Table 3.1
Some values of r(x) for I:::: x :s 2.
X 1.00 I. JO 1.20 1.30 1.40
[(x) 1.00000 0.951 35 0.9'18 I 7 0.897 47 0.887 26

1.50 1.60 l.70i 1.80 1.90 2.00


r(x) 0.886 23 0.893 52 0.908 64 0.93 1 38 0.961 77 1.000 00

Example 4:
Use Table 3.1 to detennine the value of r (-0.800) .

SOLUTION: Use Equarion 6:

rc-o.soo) = rro.200) = n1.200) = o.918 17 = -s.ns


(-0.800) (-0.800)(0.200) ( -0.160)

There are a couple of useful formulas involving r (x) that we shall presem but
no1 prove here (see the next section, however). ·rnese two formulas are

r(x)r(I - x) = -rr- (7)


sin rr x

and

(8)

Equation 7 is called I.he reflecrionfomwla and Equation 8 is called the duplication


fomwla for the gamma function. We sha.11 use 1hese formulas in later chapters.

Example 5:
Use Equal ion 7 lo show that r( 1/2) c:: ../rr.
SOLUTION: Leu= 1/2 in Equation 71oobtain r 2c1/2) = 11 / sin(JT /2) =
,r, or f(l/2) = Jrr.

In many applications of the gamma function (particularly in statis1ical me-


chanics). we need value~ of x ! for large values of x (such a.<. I.he Avogadro constanl).
We can s1an with Equation I and derive an asymptotic series for r(x + I)= x!.
The derivation for lhe complel.e expansion is fairly involved. but we can derive lhc
most important factors here. S1ar1 wilh
120 Chap1er J / Fun tio ns nc in •d A lnl " •,r,1ls

f (9)
2
Figure 3.2 shows lhe in1egrand of Equation 9 ploued againsl z for x = 20. You
=
can see 1ha1 the integrand peaks sharply al x 20. Generally. the i.ntegrand looks
like a Gaussian curve and peaks sharply aboul its maximum value. which occurs
at z = x (Problem 11 ). Tbis suggests I.hat we write Lhe inLegrand as an exponential.
~ To do this, lcl zx = ex 1" : and wri!e
Figure 3.2
111e funclion f (::.) =e' In~-:. which i" the (10)
inr~grond of Equa1ion 9. plo111!d agait~1, •
fo r .r = 20.
Because the integrand peaks sharply at z = x. only values of z. near x wi 11 contribute
10 rhe integral. Because exp (x In z. - z.) i~ a monotonic function of x In z. - z.. we
can work with the simpler x In z. - z. instead of exp (x In z. - z). Let's expand
x In z - : about z = x to gc1 (Problem 12)

(z: - x) 2 (:: - x)·'


.r In z - ;: = x In x - x - - - - + - -2- + · · -
2x 3x
Because x is large and only remis where z :::::: x are imponanr. we neglect tcnns in
(;:: - x) 3 and higher. giving

(;: - X )2
x In z - z. ~ x In x - x - - - -
2x

Subs! itutc this back into r (x + I) to gel

Let (z - x) 2 /2x = u 2, s.o that r(.x + I) becomes

Because x is large and the integrand falls off rapidly about 11 = 0. we can
extend the lower limit here 10 -oo lo finally gel

( 11}

or

I
Lnx! ~ x lnx - x +- ln(2.Jrx) (12)
2
Equations 11 and 12 arc known as Srirling 's appmxi111a1i1.m. and arc used fre-
quently in statistical mechanics. Equation I I is at.:tua.1 ly the first tem1 of an a.symp-
3.1 The Gamma Function 1 21

totic series for r (x + I): 1har's why we used~ in Equa1ions 11 and 12. The series
itself is

Table 3.2 compares S1irling's approx.imaLion, Equation 12. 10 the exac1 resuh for
n = 5. 10, 15, 20. and 50. Even forn == 10, it is in error by only 0.05%.

Table 3.2
A numerical cvaluarion of Sti.rling's approximarion. Equation 12.

n In 11! Equation 12 (Inn! - Equation 12)/ In 11 !


5 4.7875 4.7708 1.7 X IQ-;!
10 15.1044 15.0961 5.5 X I0--t
15 27.8993 27.8937 2.Q X ]Q--!

20 42.3356 42.3315 9.8xto-s


50 148.4778 148.4761 \.I X \Q-S

3.1 Problems

fo 00
J. Evaluate e-nr, 11 312d11.

2. Evaluate 3r(5/4)/2f ( 1/4).

3. Evalua1c (-c z. 112 e-~-'d::. in terms of a gamma function.


lo
fo
00
4
4. Evaluare x 5e-~ dx in Iem1s of a gamma function.

S. Evalua.1c fo 1(In x )" dx.

<ix
1 Jx
1
6. Evaluate --;::== =·
o ln (l/x)

r(x + n)
7. Show thnl r(x) = - - - -- -- --for -11 < x < -11 + I.
x(x+l)···(x+11-I)
8. Derive Equation 5.

9. Show that Equation I diverges as x--), O+.


00
10. In this problem we will use a trick 10 prove that J = fo e-x~ dx = Jrr /2. First ~uare J and v,1Tite it :is
122 Cl1,1pter I / 1-urn_,t,iun~ Defined As lntegr., 1..

Now convcn to plane polar coordinates (.r = r cos 8, y = r sin 0) and use the fact that d:i:dy = rdrJO to show
that / 2 = rr /4, or that/= ft /2.

11. Show that f(x) = exp (x In z. - .z:) has a maximum ot z. = x. Hinr: Work with x In z. - z. instead of
exp (x In :. - ::).
2
12. Show 1ha1 the Taylor expansion off(:)= x In z - : . about ~ = x is equal 10 x In x - x - (z - x) +
2x
(:: - x).3
---+···
3x 1 ·

13. Use Table 3. 1 to cvalualc f(4.30).


00

14. Show that the binomial series can be wrillcn as ( I + x )0 = I+ L r (a + I) x".


11
-=I f(II + l)r(a - fl + I)
15. A factorial notaLion tha1 is sometimes u!-ed i:- n !! = n(n - 2)(11 - 4) •·•.Evaluate (a) 10!! and (b) 7!!.

16. Show that (2n) !! = 2n 11 ! and that (211 + 1) !'. = 2''r (n + 3/2) /rr J/'!..
17. Evaluulc l=
•0
x"'t,,-,.Jt dx. where m and II are positive integers. in 1cm1s of a gamma function .

18. Show that (I - x)- 112 = L --..


oo (X- )"
(2JJ)!
(11 !)-
11::0 4

19. u~c Equmion 7 10 show that r (11 + I) r (~ - 11) = _n


__ _
2 2 co .. 1111

fo
00

20. Show 1.Jia1 x 2"e-ux:dx = r ( n + {) /2a" 112

21. Show that focx, e ui


2
cos 2o dt = (rr /4a) 112 e- /a by expanding cos 2.n in a Maclaurin ~e.ri ~s; and tJ1cn
integrating Lerm by term.

22. The logari1hm funcfion In x can be defined by the integral In x =


},
r dt1. Show that In ab =Ina+ In band
"
that In r/' = b In a.

3.2 The Beta Function

ln this sec1ion we shall discuss another useful function introduced by Euler. It is


1he bew f1111crion. defined by

O<x. O<y ( I)

By leuing I - .: = 11, we find 1hat B(x, y) = B(y, x) (Problem I); in other words,
the beta function is a symmetric function in x and y.
J.2 The Oe1.i Funt1io11 123

ll 1ums out that B(x. y) is related to the gamma func1ion by the relation

B(x. ,·) = r(x)r(y) (2)


- r(x + y)
which aJso shows I.hat B(x. y) = B(y, .x). The proof of Equation 2 is outlined in
Problem 3.
The integral in Equation I occurs in a variational 1rea1men1 of the quantum-
mechanical problem of a particle in a box. In that case. x and y tl~e on t.he integral
vaJues 2. 3.....

Example 1:
Evalualc the integral f' z·\ I -
lo
z.)Jd;...

SOLUTION: We simply us.e Equation 2 with x = y = 5 10 gc.1

f I
zJ( I_ :)''d: = r(5)r(5) = (4!) 2 = 4 .3 -2 = _I_
lo · f(IO) 9! 9-8-7-6-5 630

We could have evaluated the integral in Example I by expanding (I - .:) 4 using


the binomial fom,ula. but that approach gels tedious prcuy quickly.
The beta function can be transfonned into orhcr useful fom,s by a simple
change of variable. lf we let z. = sin 2 0 in Equation I, then (Problem 2)

D<x. O<y (3)

Leu-ing 1 = z/ ( I - : ) in Equation I gives (Problem 4)

()0 ,..c-1
B(x. y) =
1
O
- - - -di
()+1)-'+)'
(4)

If we let x + y = I, Equation 4 becomes (ilfter using Equation 2 above and


Equation 7 of the previous sect.ion)

~ ,.r - 1 rr
B(x, I - x) =
1
o
- -dt
I+ I
= f(x)r(I - x) = -.--
Stn Jr X
0 < x < I (5)

(We'll evaluate this integral another way in Chapter 19.)


Equations 3 through 5 can be u.c,ed 10 evaluate a host of integraJs. These are
best illu.... 1.ra1ed by examples.
124 Ch, pr •r 3 / Fun 11ons D fined As ln1egrals

Example 2:
r12
Evaluate Jo (tan x) l/Jd.r .
SOLUTION: Let tanx = sin .xj cosx and use Equation 3 with 2x - I= 1/3
and 2y - I= -1/3:

Example 3:
.,
Evaluate 1- 1
u· (4 - u 1 ) 312 d11.

SOLUTION: First le! 11


2 = 4z.
so lhal (4 - u 2 ) 3 ~ = 8( I - z)'.1/'2..
u3 = 8z.Jl 2, and d11 = dz/z 12 . Then
1

1 2
3
11 (4 - u
2
/
12
du = 64 fo
1
z(I - z.)
311
dz

===
64
rc2>r(5/2> = 256
r (9/2) 35

Example 4:
Evaluate 1o·,, cos6 fJ d0.

SOLUTION: We could use Equation 3 if the limit,; were Oto rr/2 . Note.
however. tha1 cos6 {) is symme1ric aboui () = JT /2 (Figure 3.3). so 1ha1

rr/2 ,r X
f~7 = 2 ./~1 / 2. Now we u~ &1ua1:ion 3 to write
Figure 3.3
1bc function y(B) = cos 6
0 is symmetric
r
lo
cos6 fJ dB= 2 r/2
lo
cos6 0 dO = 2. ~ f(l/2)f(7/2 ) = 5rr
2 f(4) 16
about (I = ,r /2.

hample 5:
Evaluate f "° du
Jo0 I+,,~
l.l The Be•r., Fum lion 125

SOLUTION: Lc1 z. = u\ d11 = dz./4z. 3IJ. and wrile


f)O du If oo , - J/4
lo I + uJ = 4 lo 1 + zd:.
;r rr
4 ~in(.rr /4) = 2 /2
We used Equation 5 to evaluate the last integral.

3. 2 Pro blems
l. Show that B(x. y) = B(y. x).
2. Derive Equation 3.
3. (This problem invohl:'- polar coordinates.) You can derive Equa1ion 2 in the following way. Start wi1h
r(m) = J;zm-le ;dz and let;:= x 2. Do the same for r-(11). Now form r(m)r<n) a<: a double in1egral.
trnnsfonn IO polar coordinates, and use Equa1ion J.

4. Derive Equation 4.

s. EvaJuate l'n
O
- cos6 0d0.

6. Evaluate 1,
()
1
/11( l - u ) d11 .

7. Evaluate l' ( r/J


O
I- ;
1
d.r..

2
8. Evaluaic fo 1.1
2
-18 - u3 du .

9. Evaluate
11:c -e2"- - d11 .
-oo I + e 311

10. Evaluate 1"


()
dx
Ju 6 - x 6.

11. Evaluate fo:rf-sin l/J 2fJ d0.


12. Compute the area bounded by the curve x 213 + y 2f = I. H;n,: Plot rhis expre,,ion first.
13. We shall derive the duplication formula for I.he gamma function (Equation 1.8) in this problem. First use
Equation 3 lo wrile

I f(x + i >f (~)


1
:r/2 . 1..r I I
11 = sin 11du=-B(x+~. , >= - - -- -
o 2 2f(x + I)
126

{"/2
Now consider the integral / 2 = Jo sin 2.r 211 d11. Use 1he relation sin 211 = 2 sin II cos II to show thal

2.r-i r(x + ~)f(x + j)


-=2
/l
r (2r + I)
Now let 211 = z. in / 2 and show that 12 = / 1. Finally equate Lhc expressions for / 1 and / 2 above to get

3.3 The Error Function

One of lhe most commonly occurring and important integrals that canno1 be ex-
pressed in tenns of elemen1ary functions is of the fonn
1
ft
e-" du. Thjs type of
integral occurs so frequently that the function of x that it defines is a standard
function of applied mathematics. We d,'.fme the error Ju11c1io11 by the intcgra.l

erf(x) = -2 1·re-u du 2
-OO<X <OO (I)
ft 0

Even though we cannot express erf (.x) in terms of simpler functions. il is a


err perfectly well-defined function of x and can be evaluated by numerical integration .
Almost any mntllematical handbook will have tables of crf(x) and any CAS will
have the error function as a standard function .
The 2/ fa factor in Equation I is chosen so that erf (oo) = L Furthennore,
2 x it's easy to show that (Problem I)

erf (-x) =- erf (x) (2)

Figure 3.4 so lhat crf(-oo) = -1. Figure 3.4 shows crf(x) ploued against x and Table 3.3
The error function crf(x) plol!cd :Lgain,r lists erf(x) for a few (posiLive) values of x.
X.

Table 3.3
Values of e.rf(x) for a few positive values
of x .

X erf(x) X erf(x)

0.00 0.00000 1.20 0.91031


0.20 0.22270 1.40 0.95228
0.40 0.42839 1.60 0 .97635
0.60 0.60386 1.80 0.98909
0.80 0.74210 2.00 0 .99532
1.00 0.84270
3.3 The [rror F1111c !inn 127

Example 1:
The error fonction plays an important role in 1he kinetic theory of gases. The
fraction of molecules that have a componen1 (.x , y. or z) of velocity between
v and 1J + dv is given by

where III is the nia.'-.S of a molecule, T is the kelvin tempermure, and


k8 is the Bollzmann constant. C:.ilculalc the frJClion of molecules with
-(2kH T / 111) 112 :S 11 _ (2kB T / m) It:? = 1;0·

SOLUTION: The fraction is given by

•·o
= 2 j-1'0 f(v) dti
F =
f -1'(1
f(11) d,
0

because the integrand is an even functim1 of,,. Now let 11 = (m/2k 13 T) 1 nv


to write

F ') 11 '
= - '-
./ii ()
t·-
u-
d11 = erf(I) = 0.842 70

Tims. about 84'.f of the molecules ha\'e a nrngni1ude of a component of


veloci1;y iha1 i" iles~. than ~2k 8 T / m.) l/2.

Suppose we want 10 calculate lhc frnclion of molecules in a gas whose .r-


component of veloci1y i... grca1cr in m:.1gni1ude than v0 (perhaps 10 detemiine lhe
fraction of particularly energetic molecules). We would calculate the following
(sec Figure 3 .5):

F = 1-vo
-oo
f(v) du+ /
00

111
f(u) d1J =2 (-,n-)
2rrk5T
I/:? f
J,
00

1,0 1
e - "'
2
f ll9 T dv

Figure 3.5
If vn = (2k 8 T/ m) l/2, as in Exnmple I, 1hcn
The sh.'.lck.-dl area rcprc..~cnts the rniction or

fi') J1r:.J
mok<.: ul c , in a g.c; with an .r-con1poncrll
l
F = e _,, d11 of the \eloci1y tha1 C."<Geed,; i;ome value 1, 01

in m. grTi!ude.

which is a special case of

F(x) =- 2 1XI e- ,,2 d11


Jrr r

Equation I ~ays that F(x) =I- erf(x). Although F(x) = I- erf(x). 1his in1egrnl
occurs frequently enough that it is used to define the complemml(lry error ftmction.
erfc(x),
128 CIMp1er J / Functions Defined A$ Integrals

2 {x ,
erfc erfc(x) =I- erf(x) = fa lx e-"-du (3)

=
Note that erfc(-oo) = 2, erfc(O) = I, and erfc(oo) 0. (See Figure 3.6.)
The error function also occurs when we solve cenain differen1ial equarions.
In Chapter 17, for example, we'll learn how 10 solve 1hc diffusion equation. which
describes how a substance diffuses Lhroug.h a solution. Suppose we have a long
-2 -I 2 X narrow cylindrical tube ( -oo < x < oo) and we prepare the solution initially sucb
that the concentration of diffusing species. c(x). is (see Figure 3.7)
Figure 3.6
The complcmcntMy error fonc1ion crfc(x)
X<0
plottc:d against x. c(x) ={O x>O
1=0
en

We expect that as time evolves, rhe concen1.ration profile will smooth out and
eventually become unifon11. as shown in Figure 3.8. When we solve this problem
in Chapter 17. the concentration profile will be given by

c(.t. I) = co [ I + erf ( ~ ) ] (4)


2 2D,.;t
X

where D is the diffusion constant. which characterizes how rapidly the substance
Figure 3.7
1l1c initial concenl.r'Jtion profile (x) =0 = =
diffuses. Note lhal when 1 0. erf (x /2 D Ji) crf ( -oo) = - I when x < 0 and
for x < 0 and c(.t) = r 0 for. .,. 0 in a long erf(oo) =+I when x > 0, so I.hat
cylindric.al rubc:.
X<O
c(x. 0) = {coO X>O

C I in agreement with the given initial conditions. Also. as/ -. oo, erf (x /2 D Ji) . . . . 0.
I'= o so lhat c(x. 1) ➔ a unifonn concent.ra1jon of c0 /2 along the entire wbe. For
intem1edia1e values of t, Equation 4 gives 1he concentration profiles shown in
Figure 3.8.
The error function. or very closely related functions. plays a prominent role
in statistics. The Gaussian distribution or noTTTial distribution
0 .x
-OC<X<OO (5)
Figure 3.8
The concentration profiks in a Ion£
cylindrical tube. where the initial describes Lhe dislribulion of e1Tors in many types of experimental observations
cono:nr.rotioo prolilc wa.s c(x) =
0 for (Chapter 22). In Equation 5. µ is the mean. or 1he average. of I.he observations and
.r < 0 and c(x) =,·
0 for x > 0. er is the standard dcviaLion of lhe ob~ervations. In olher words.

(6)

and

(7)

(Problem 5).
.l .J The Error Funt. lion 129

Equation 5 is the famous bcll-shnpcd curve of statistics. and is ploned in f


Figure 3.9. T\vo important features of the curves in Figure 3.9 are I.hat Lhc curves
are centered at x = µ and that rhe widths of the curves are controlled by the value
of a; 1he smaller a is. rhe narrower and more highly peaked the curves are.
Equation 5 is used 10 predict the chance that an experimental observation will
lie wilhin a cer1ain interval about x = JJ.. For example. we might want to know lhe
chance that an obscrva1ion will lie within one standard deviaLion ofµ: this is given µ .r
by
Figure 3.9
1
The Gau sian dis1ribution, Equ:ition 5.
I - [' a f(x) dx ploutd agn.i ns1 .r for er = 1.0. 0. I 0. and
1,i-(1 0 .050.

Let (x - µ)/(Ji. a)= u to write

I = (-1)1/211/.Ji e -ul du= (4)'-~Ll/./2


- e
-u2
d11
iT - I/Ji. 1f 0

= ed(l/h) = 0.683

If you've had an introduction to statistics. you might remember lhal the corre-
sponding values for an observation being within two or three standard devia1ions
of the mean are 95.5% and 99.7%, respectively (Problem 6).
A word of caution: The functions 1abula1ed in most statistics sources are not
erf (x). as we hnve defined it in Equa1ion I. The function usually tabulated is

(8)

which is related to the error function by (Problem 7}

<!)(x) = !1 I+ erf(x/v'2)1 (9)


2

The error function has a useful Maclaurin expansion for small values of .r and
an asymptotic expansion for large values of x . The Maclaurin expansion of erf(x)
is (Problem 9)

2oo (-l)"x2n+I
erf(.x)::::: - L ----
fan~ "t(2n + I)
(10)

and we showed in Section 2.9 (in slightly different noLalion) that the asymproLic
series of crfc (x) is

erfc(x) ~- -
t! t2 [
I+
Loo (-1)11 l-3,5-- .,-(2n-l)] (11)
ft X n=I
(2l"-)"
130

Example 2:
Use Equa1ion I O to evaluate erf( 1/2) to four-place accur•.u:y.

SOLUTION: Equ:i1ion 10 ii. an alternating scriei;, so lhc truncation error


\viii be less than Lhe magnitude of the fir.._t term dropped. We wam four-place
accurncy. so we ~I

( I /2)2" I
----< ,0--1
II !(211 + I)

Thi~ will be :-.o for 11 = 4, i,.n we ha"c

~3 (~)) +~ ~
5 1
L'lf{x) = ___3_
.jii -
[{ -
_
({)
10 _
- (~)
4_ 2
]

= 0.52049 · ·,
The "exact'" value 10 five places i~ 0.52050.

A number of definite integraJs can be expressed in terms of 1he error func1jon


or it.s related funcl ions, For example (see Problems I I through 14 ),

and

3.3 Problems
I. Prove Lhat erf(-x) = - erf(x).
(2.fi. ,
2. Determine 1he value of Jo e-=-dz..

3. Refcrri ng 10 Examph! I. show th:11 Prob I- u_dl ::5 1 ,l· ::'.:: u.ro I = crf [ (111 /2k» r) 11~v , 0 J.
4. Thi.! probability 1ha1 the x-component of the veloci1y of a ga~ molecule exceeds a value of 11J 11 ii,. given by

Show 1ha1 this probahi lity is given by t:f"fc(11 0), where u 0 = (111 /2kR T) 112 vxo•
5. Verify Equa1ions 6 and 7.
J.J Th ' Error Fu nclion 131

6. Using Equal ion 5. show that the chance 1ha1 an cxperimt·mal oh, 1:: rvatinn wi 11 lie within two srandard de\'iations
from the mean is 95.5 ~; .

7. Dcn\'C E4uat.ion 9.
8. The kinetic theory ,if ga,c, rrO\ ilk, the following formula for 1he probability 1ha1. 1he speed of a ,!!a~ molecule
exceeds a \'llhlt' ,·u:

Prob f,· - t·r }


,
= _-_
r.:
") I( )
\I lT
__!!!___
,,,. r
"--"II
1/ 2 c e -mcu/ll,1T
o

9. Derive rhe first fcw ram~ of rhc Milcluurin ~cries for erf (x ).
I 0. Use Equal ion 10 to cakulatc erf ( I) 1:0 three-pl ace ail:tllr;:icy. The ..exacr" value is 0.842 70 to Ii ve places.

11. Show 1h:i1 { ,::,.;, .e -


lo
axl , ll,x : ·'dx = ( ;r
,-la
) I/.? / "~-{J /u crrc(~)-
Ja
e- (lf dt (;r)l/2 ~ .--
1 e"·" crfc( ✓ax).
::,c
12. Showth,1l - --,,== = -
0 f + x- (I

• :xi e- "' dt ...


13• Sh nw t h al
J. O
----
t 1n (I + x)
=
...; X
• r:-:
"r.:e (/ 1 erk(vax).

1•:x: iT) I/ ~ and use this result to i:.how thal


14. Show 1ha1
1 o
x Ct1!.11
· ~ du ==
v" o
sin
. • r.: t/11 = :._
'-'"
11 (

_I (:!.. )l,'2
2 2

Hint: Substitute -
L
=-I 1~ l
,-Ill
r: dt into the first intl'~rals and exchange orders of integration.
II ./ii (J ...;/

15. ShO\V rhal 1 ()


c - r:n - : f Ji rJ:r = :n 11-ea! erfr(o).
•~ , , - 11•
' t: - l •
,
>
16. Show thnt )
- .,- d 11 =- - - ,r I/_ crfc(x). X > 0.
x u- X

17. Two functions th:11 arc closely rclarcu to the error function arc 1hc Frc.l'llel sine i111egml :rnd the Fr1.._1·11el cosi11£'
imcgml (in the notation of Abra.mowirz and Stcgun):

S(x) = f (-T;')
sin d" ( 12)

and

cc,J = f (";2)
cos J,. ( I 3)

gl
132

y
0.8 j /,
I
I

Figure 3.10
The Fresnel .!>'lne in1cgrnl. S ( x). tlefincd by
0.4 r/
I
Eqwuion 12 ('-'>lid) und 1he Fte!>nel cosi rie
1

integral', C(.r). defined by Equati on lJ


(dru hcd1) p'loned ag.ainst .r. 2 3

Figure 3.10 shows S(x) and C(x) planed against x . As d1e names might imply, these inregrals arise in lhe
study of the diffraction of radiation. Show t.hm

C(x) = I: (-
,,=<J
l)"(JT 12?11 .r-1,1+1
(211) !(411 + I)

and

S(x) =L
:):) (
- I)" ( / ? )211 I
7r - x-ln+3
11
=0 (2n + 1)!(4n + J)
00 ,, 1/2
18. Starting with
IO
1 o
e- ,.n -dx
verify the in1egr:ils in Problem 14.
11
= --,
2a
ler a= (I - i)/2 112 and separate the result into real and imaginary parts

19. Differentiate /(a. b) = {


lo
e -hu·) t:oi-: r1u d11 = (!!..)
4b
1/2
e- 11 ~/-lh with rcspecl to 11 :md then integrate with

respecr 10 b from h 10 oo to show 1ha1

f e- b"~ sin a"


Jo
00

" d11 =2
;r
erf
(.
2;:,2 . )

20. The complcmcn1ary error func1ion also occurs in the quantum-mechanical dis.cussion of a hannonic osci llaror.
Recall thal a harmonic oscillator can be used as a model for 1he oscillarory bchavior of 1wo masses connected
by a spring (a model of a diatomic molecule). If the relaxed length of the spring is I. then tJ1c two masses
oscillate sinu~oidnlly about/ and the polential energy of 1he !,.)'!)tcm is given by V(x) 1
~(x - 1) . where = ')
k is the force constant of the spring. Show tha1 the maximum displacement of the rwo masses is given

by Xm:ix =I+( \t:) 112


, where £ is I.he total energy. 1l1e quanti1y Xm:u is called 1.he c/11ssical/y-allowed
amplitude. One of the many strange resulls of quantum mechanics is that there is o non1..ero probabilil)' thal
1he displacement of the oscillalor will ex.ceed its cla,sically-allowed amplitude. even though ir doesn't have
sutlic.:icnl energy. For a quan1urn-med1anical hannonic oscillator in its lowest energy state. 1his probabili1y is
given by

where a is a com,1an1 lha1 is charactcrisli( of Lhe oscillator. Show rh:ll this pmbahill irry is given by
=
Prob crfc( I) =
1 - crf (I)= 0.15730. Thus. we St."C t.hat 1here is almost a I 6 chance ,!hilt the di, placemcnl
will exceed iL'- c\a,!.ically-allowcd ampli1ude.
3.4 The Expon 'nlial lnlegral 133

3.4 The Exponential Integral


Another 1ype of integral tha1 occurs fairly often in physical problems has t.he
general fonn

x > 0. n = 0. I .... (I)


0.5 1.0 X

In par1icular. £ 1 arises in a quan1um-mechanical 1rea1ment of the electronic energy


Figure 3. 11
levels of a hydrogen molecule. Figure 3.11 shows graphs of En (x) planed agains1 The funclions £,,(x) dctincd by Equation I
x for various values of n. plo11ed against :r .
Some special values of En(x) arc

E/1(0) = - I- 11 ~ 2
11 - I

while £ 0 (0) and £ 1(0) diverge at x = 0. In fac1.

(2)

so it diverges as 1/x as x ➔ 0.
We can derive a recurrence relation between E,,+ 1(x) and £,,(.r) (Problem I)

0)

by integrating Equation I by parts. We can also derive the rela1ion

(4)

by differentiating with respect 10 x under the integral sign in Equation I (Problem • Ei


11 ).
Some authors call £ 1(x) the exponenticil i111egrol and others call the closely
related integral
X

X>O (5)

the exponential integral. Figure 3.12 shows Ei (x) plotted against x. Note that Figure 3.12
Ei(x) ➔ -oo as x ➔ 0 and--,. oo m; x ➔ oo. Both £ 1{x) and Ei(.r) are well The funt·tion Ei(:,;) dcfine-J by Equation 5
ploucJ against x.
tabulated and are built into most CAS. The two functions are related to each other.
but we must wait until the next chapter. when we ~1udy complex numbers. to see
what the relationship is.
The series expansion of £ 1(.r) introduces us to a fundamental numerical
constant ea! led Euler ·s co11s1a1ir. If we denote this constant by y. then

X>O (6)
134 <..h,1p1N J / Fu11< tio11, O •f ined As /11h•gr.1I<

where
,, I )
y = ,,_,,o
lim
( 'L..,, -k - In 11 = 0.577 215 (7)
. k,,_I
- ·-· = ·-· !--•~•-•,J\.,•.lt..• ..... ■ ..
r - -

The limit in Equation 7 is or the indc1ermina1e form oo - oo. bu1 1he limit does
exist. Figure 3.1 J shows lhe righ1 side or Equation 7 plo11ed a£ains111. Note 1hu1
5 000 10 000 II
Equ:llion 7 sugg ~ ts th:u Che .harmonic series diverges as In 11 . The series expansion
of Ei(x) is somewhut siim·; Jar lo 1ha1 of l-.: 1(.r):
Figure 3.13
,'X:J II
~ . Ln -I - In
The 1unc11on
J.:
II

ploned ngamst Ei (x) = y + lni x + L ~ (8)
k ·., I n==-1 II 11 .
n. The lirniling \1alue j,.. equal 10 E uler's
constn.nt.
Note that £ 1(x) is almost equal to -Ei(-x). extepl for 1he In x tern1. We say
'"almost·· because ln(-x) is undefined for posi1ivc values of x. so the ·'almost'"
relalion is meaningless for po~iLive vaJues of x. We'll see in 1.he next chap1er.
however. thal we ca11 give a mcan~ng to ln(-x) if we allow x to Lake on complex
values.
In Section 2.9. we showed that the asymptoLic series for £ 1(.r) (in diffcrcnl
noli:.llion) is

(9)

This. large x bchavior can be ~ecn in Figure J . 11. The asymp101ic expansion for
E,,(x) is (Problem 4)

.
En(.r)
e-:. [
~ -- I-
11
- + 11(11 +
., I) - 11(11 + 1)(11
1
-1- 2)
+ ... ] ( 10)
X X x- x·

A number of dcfini1c inicgrals can be cxprc!-scd in 1crms or £ 1(x) and f i(x).

Example 1:
Show lhat

SOLUTION: Let/,+ I = 11 and write


x. e - 111
--di = eub
le,.:, - - du
e-au

10 /, +f b II

Then kl 11 = h::. to obtain


135

Example 2:
Show that

oo e-ar _ e-hr b
l o
----dt=ln-
f (1

SOLUTION:

e - ot _
- - -- dt= lim
- br [1
lo ~➔O .r

= limf£ (a:c) -
.f )
1 £ 1(/J:c)]

Now use 1he series expansion for £ 1(.r) given by Equatjon 6 to wri1e

lim[£ 1(ax)- £ 1(bx)J= lim[- ln(o.r) + ln(bx) + O(.r)]


x-0 x-0

b
= ln -
a

Two other integrals 1ha1 are usually discussed 1ogcther with 1he exponential
integral [either£ 1(x) or E;(x)] are the sine integral and the cosi.ne integral. defined
by
sin u
1
.x
Si(x) = --du x>O (I I)
O U

and
00
Ci(x) =-
1 r
cos II
--du
/J
x>O ( 12)
- I '
f
I

The graphs of these 1wo func1ions arc shown in Figure 3 .14. Figure 3.14
The series e;,;paasions of Si(x) and Ci(x) arc (Problem 5) The- ,inc inll·grnl. Si (.x) (soliJ ). defined
by Equa1ion 11. und 1hc co,inc integml.
::,:. (-l)nx2o+l Ci(x) (da.,hcdJ. ddint·d by Equation 12.
Si(x) = L ----- ( 13) plor ti:d against .r.
"=1_1 t2n + I )(211 + I)!

Ci(x) =}'+In .r +
. ,,=l (2n)(2.11)!
Loo (-1)"
·r2" ( 14)

Example 3:
Show that

.r
liJn Si(.r) ==
~ i
o
x. sin"
--d11 == -
11 2
-;r

gl
136 lh,1pli:-r J / Funcriuns Dt'ii1wd As lnll·grals

SOL Ul I ON: Lc1 ~ = (';; e- 'dz. and inlcrchangc orders of integration


11

to get
11 lo

00
]x_
1 d;:_ [ _1 iT
- - - ·- - tan -
- o 1 + ;::? - .. o 2

3.4 Problems

1. Derive Equations 3 and 4. (See aJso Problem 11.)


2. Figure 3.11 suggi:,t~ thal En(.r) > £,1+ 1(.x). Prove i1.

3. Show 1hat Ei(x) can also be wriuen as £i(x) = -1:x- :.:_ J1.
-.r r
4. Derive an asymplotic expan . . ion of £,,(x).
5. Derive Lhe series expansion of Si(x). Equation 1 J.
00

= -y. show lhat 1.r 1- ,- ,


= y + In x + £ 1(x) .
6. Given 1ha1 lim
........ o 1f
e-.x In x dx ·.
O
---dt
I

= -y, show lhat 1.r e' - I


= Ei (x) -
l
(X)
7. Given thal lim e-.r In .x dx --dr In x - y.

.,
(-,0 t O I

COS/I - I
8. Given tha1 lim
,-0
,xi
1("
t· - .r Ln x dx

I
= -y. show 1hat
1 O
- - - - d u = Ci(x) - y - In x.
,,

9. Showrhat
1 (I
e-n'Ci(r)dr=--ln(I+</).
2n
10. The following integral occurs in 1he theory of 1:he conductance of solulions of strong elec1rolytes:
·:r:i - ma
S11 = R"- 2
1
R
- - dr
r "- I
II= I. 2, .. .

where R is the average radius of the ions and K (> 0) is a known parameter. Firsl show lhat Sn is a function of
e- KR
only KR. Now show rhat S 1(KR) = --
KR
and S,(KR) = £ 1(2KR).
-
11. Justify differentiating under lhe integral sign in Equation I 10 derive Equalion 4.

12. Anothl!r expression for Euler's constant is )' =- lim [ e- In x dx. Use a CAS to show that y = 0.5772
.-~o. t
lo four pl::icc:s.
00
I
13. Showrhnl
1 o
e-c_'E 1 (x)dx = - ln(l+a).
a
3.5 Ellip1ic ln1cgr,1ls 137
Joo _e-dx +In'=. Verify this relation.
f
xi -.t
14. To derive Equal.ion 6, you need to use the relalion e- 1 tn x dx =
' c X

15. Use the relation in the previous problem to derive Equation 6.

3.5 Elliptic Integrals

We'll start this section with a discussion of a physicaJ problem that leads naturally
to an elliptic in1egral. Consider a pendulum consisting of a mass m art.ached to the
end of a rigid. massless rod that is suspended from a fixed point (Figure 3.15).
Assuming that the oscillatory mat ion of the mass takes place in a single plane. the
k.inetic energy is given by

where s is the distance along the arc. I is the length of the rod. and 0 is the angle
Figure 3.15
that 1he rod makes with the vertical. The potential energy of the ma,s is mg times
The gc.-omecry of lhc pendulum discussed
its height above its vertical position, or in Equations I through 6.

V(0) = mg(/ - I cos 0) = mg/( I - cos 0) (2)

The total energy of I.he ma. s i - E, = X + V. or


1 )1 +
111/-
- 2 (di-
d0 -
,nr,J(I - cos0)
,":,
=[ (3)

We can determine £ from an initial condition, such as when the mass is staned
from rest ar 0 = 00 • so that J0 / d 1 = 0 at that Ii me. The ini1ial total energy £0 is

[, 0 = mg/(\ - cos 80 )

The energy is conserved (Problem 2). so f, = £0 in Equation 3. and we have


,,,,2 ( df)) 2 +mgl(l-cos0)=mgl(l-cos00 )
dt
2
at any time. or

/f)
~ = ( ')_g ) 1/2 (cos R - cos B0 ) l/'.2 (4)
dr I

The ma.~s will as.ciliate from B0 10 -00 and buck to R0 repeatedly.undone-hair of


the period of oscillation is I.he time that it takes lo go from 00 to -00 (or from -00
to f)0 ). If we ~lve Equation 4 for dt and in1egrate from -00 10 00 • lhen we have
138
that r /2. one-haJf the period of oscilla1ion, is given by

.:. = (_!__)11210/j _ _ d0_ _


2 2g _ , co 0 - cos 00 ) 112

or

2/) /219t)
1
(81) 112
{"o _ _d O _
= (g
d0 (5)
r - t>o (cos 0 - cos00 ) 112 ::= g lo (cos& - cos 00) 112

where in Lhe last slep we have used the fact that the integrand is an even function
of 0 because cos(-0) = cos(€/).
Equation 5 is a little awkward to use numerically in general because of the
singularily at fJ = 00 . It is cus1omary (and very convenient) to tram.form the integral
in Equation 5 so I.hat Equation 5 becomes (Problem 4)

1(2L;r/2 I
r=4 ( -/ ) {U
(6)
g o ( l-k 2 sin 2 11) 1 -

with k ::: sin(B0 /2).


We can investigate Equation 5 for small values of 0c,. where Lhc pendulum
undergoes ham1onic motion. If 00 is small enough [and consequently/.; = sin(80 /2)
is also small enough] that we can neglecr k 2 sin:! u in the dcnomina1or. then r0 =
2rr (// g) I(!., the period of a simple harmonic pendulum. Otherwise. r reprc~ents
1he period of a pendulum of arbitrary amplitude.
The integral in Equation 6 cannot be evaluated in terms of elementary func-
tions (unless k = 0) and is called a complete elliptic integral of the first kind. It is
usually written as

:r/2 df)
K (k) =K =
i -
-;:::===::;:=
J t - f? sin 2 0
(7)

The "complete.. in the name implies 1ha11he upper limit is rr /2. If the upper limit
is arbitrary, then we have an incnmplete elliptic integral of the fir.rt ki11d:

F (k. <I>) = 1,p -;:::== d=0 = ; = (8)


o / 1 - k 2 sin 2 0

As you may have guessed from the name. there are also complete and incom-
plete elliptic integrals of the second k.ind. namely

E(k) = E = Jor ~Ji - k 2 sin 2 0 dR (9)

and

( I 0)
3.5 F.11 ip!ll" 1111 r.il, 139

All these elliptic in1egra'ls are well tabulated and Figures 3. I 6 and 3.17 show K {k) K
and £(k) plotted against k. Some special vaJues of K (k) and E(k) are (Problem 5)
,r
,r
K(O) = -; K(I) = oo: £(0) = ~: £(1) =I ( 11)
2 2
1(

2
1-------
-1

0.5 1.0 k
Example 1: Figure 3.16
Evaluate F(O, rp). £(0. ,p). F( I. ,P). and£( I.</>). The conipkte elliptic integral o( the tirs1
kind. K (k). ddirll'lli by Equati on 7. plulli.:d
SOLUTION: &1uaticms 8 and 10, with k = 0 and k =I.yield ,1guin:~1 k.

E<O. </>) = 1,:. t!O =- <P


!I

F(l.</J)=
1 o cos 0
dO- = [ ln(sec8+tan0) ]<P
,p -
o
0.5 1.0 k
= ln(scc </>+tan</))
Figure 3.17
The cmnple1c elliptic integral of 1hc
sc::i.:011d kind . !-.'(/.:). defined by Equa1ion 9.
plollt...-d a~ain,t k .
Note 1hat we obt.ain Equations I I if <t, = Tf /2.

Returning to the pendulum problem 1.har we used to introduce this section. we l. i T/To
2
see lh.it 1he ratio of the period r. gi\·cn by Equation 6. to the period of a simple
harmonic pendulum. r0 = 2rr(//g/ '~ , is given by

( 12) 1.0

=
where k sin (00 /2). Equation 12 is ploued against o0 in Figure 3.18. showing
Figure '3.18
the deviation from simple lwrmonic mo1iun Lis H0 increa~c~ . 1l1e rlllio nf 1.he period, of a general
We can derive ~cries expansions of the elliptic integrals by using the binomial o~d ll:11or arid a ham11111ic· o..cill.110r plOlll·J
ngain.•a 1,11 . the ini1ial angh: in Figure 3.1.5.
series in each case to expand in powers of k 2. For example.

1'
7

K (k) = n df} ( I+ -k·


I , ., I ·3 4. I ·3·5 6 6
sin- 0 + --1-4 sin 0 + - - k sin 0 + • -·
)
o 2 2~2! 2:I J!

Using
2

1 0
;i 1 1
sin.:.,' 0d0 = I . 1 · S · · - (2n - I) ( rr )
· ·
2 . 4 . 6 .. . (211)
-
2
II = J. 2.
140 Chaptl'r J / Funclions De(ined As lnlegrals

we have

K (k) = -,r
2
[ I (1)
+ -
2
2
~ + ('·3)
k'" -
2-4
2
k4 (1-1-s)
+ -·-·
2,4 , 6
2
k6 + ...] (13)

Ax
Similarly. (Problem 8)

X
( 14)
Figure 3.19
The geometry u.<;SOCi111cd with 1hc
calculation of arc length.
The elliptic integral of the second kind arises naturally when we calcuJate the
length of the arcs of certain curves. Figure 3.19 shows the geometry aswciated
with the calculation of !.he lcnglh of 1he arc of a curve. If the Hvo ends of Lhe arc
are given by x = x 1 and x = x 2 • then l, the length of the arc. is given by

( 15)

If x and y are given parametrically by x = x(t) and y = y(r). then Equation 15


becomes

(16)

or, if y is given as a function of x. we can use the diffcrcniial form dy = J'(x)dx


to write Equation 15 a.s

(17)

Let's use these equations to calculate the arc lengths of some curves. For
example. let's use Equation 16 to calculate the circumference of a circle. The
y equation of a circle of radius a. x 2 + y 2 = a 2• can be expressed parametrically
by !.he two equations. x = a cos 0 and y = a sin 0, where O,::: (J ,:s: 2n. Substi1u1e
these .relations into Equation 16 with t = f) to get

2.n

X
I=
10
• 2
(a-' sm l 'J
f:J +a-cos· 0) 11-df)
'l
= 2rra

That was easy, Now let's calculate the circumference of an ellipse.

Example 2:
Figure 3.20 An ellipse can be described by the parami:tric equations x a cos 0 and =
An ellipse "'ith tJ1c m.ajor axis ulong 1hc y y = b sin 0 for O ~ 0 ~ 2rr. (Figure 3.20 shows an ellipse for lhe case in
axis. which h > a.) Calculate the circumference of 1he ellipse .
141

SOLUTION: UsingEquat.ion I6witht=8.

1L1f [ }r1 - rr"}) 1 1/2


=bfo I-
(
~ sin~o
]
r/8
This integral would be of the fonn of Equation 9 if the upper limil were rr /2
ins1ead of 2JT. We can ge1 an upper limit of ;r /2 by ei1her calculating l /4 of
I.he circumference (0 .:'.:': 0 ~ 11 /2) and 1hcn multiplying by 4. or by realizing ,r 2.Jr 0
thal the areas in the intervals (0, rr /2). (rr /2, rr ). (JT. -Y.rr /2). and (311 /2. 211)
arc I.he same. (See Figure 3.21.) Either way. we gel Figure 3.21
An illusr.ration of lhc fm.:1• 1hn1
y (O)-:::;; pi - k" ~in 2 0) 112 is periodic with
I= 4b£ ( Jh 2 - a 2 / b) = 4b£(e) a period ,r. In this figure k 2 = 0.50.

where e = b 2 - a 2 / b is the eccentricity of the elJ ipse. Note thal / = 2rr b


if a =b.

Suppose !hat a > b in Example 2. In that case, we have

I = 4b Jot'/2 [ I + ( a l ;i /2) 2 ]1/2


> sin 0 d0 ( 18)

This integral is not of the form of Equation 9, bul we can trc1nsform it into lhe form
of Equation 9 by first using rhc rda1ion sin 2 0 =I- cos 2 (~ 10 get

b2) cos 2 0]l/2 d0


0--;;2- (a2-----=;--
I= 4b
1 1r(1 [
lr b"
( 19)

To Lransform cos2 f) si n2 0. simply let 0


10 = n /2 - </) [because cos( 1 - tJ,) =
sin ipJ. Equation 19 becomes

I = 4b {~12
.fo
[a~b.. - (a2 lr~ b2 ) sin2 <P] 1;2 d<j)
=4a L"T/0 [,-(a' ,;;b') sin'rJ>f' d,J,
= 4a £ (Ja2 - b2 I a) = 4a £ ( e) (20)

where the eccemrici1y e = J o 2 - b2 / a in this case. Equation 20 is exactly Lhe


answer obtained in Example 2 with a and b interchanged.
142 Chaplt•r J _i Fun( t1on~ 0Pfi11 ..d As Integrals

We can use such manipulation 10 evaluate many integr.ils involving square


rooL<; of 1rigonometric functions in terms of elliptic integrals.

Example 3:
Evaluate the integral

in lerms of a complete elliptic integra.J or the second kind.


SOLUTION: Following the same tnmsformation as above, let sin 2 0 =
2
I - cos 8 to gel

Now tr.msform cos 2 (J to sin 2 0 by !citing 0 = rr /2 - </J to ob1ain

I = fo,r/\4 - 3 sin 20) I11d0 = 2 {'l/ I- ~ tf,)


2
( sin 2
111
d</J

= 2£(J3/2) = 2.42211 · · ·
(Sec Problem 11.)

Many types of integrn.Js involving square roots of 1rigonome1ric functions can


be evaluated in terms of ell.iplic integrnls. We won't illustrate any more here. but
the rde-rcnce to Spiegel at Lhe end of Lhe chapter has many different examples.
We can also use ellip1ic integrals 10 evaluate integrals involving square roots
of polynomials. Let x = sin 0 in Equations 8 and 10 to get (Problem 15)

(21)

and

:( I - fl.x2)1/2
E(k, ;:) =
1
(i I - x-
., dx (22)

where z =sin</). These become complc1c ellip1ic integrals if z. = sin(JT /2) = I. It


turns out thal integrals of the type

dx
1J
1,
I= ~ -
a P (:d
3.5 Elliplic lnti-gral, 143

where P(x) is a third or fourth degree polynomial with real zeros [i.e. real roots
=
of P(x) O ] can he transformed into elliptic integr.ils. This is a vast subject, but
we'll give just one example here.

Example 4:
Evaluate

in tcm1s of an ellip1ic integral.

SOLUTION: Lei x = tan 0 to obtain


I= ('/l sec2 Od 0
lo Jo + tan - 0)( I+ 2 lan 2 0)
= forr/'!. dO r/"2. dO
lo Jco 2 (J + 2sin 2 0 - lo J i+ sin 2 0
Thi~ Inst integral is similar to the one in Example 3. Using the same
substitution), gives

1-j..,;i t/0
---;:::===;:= - -
I 1 1112 d(I
- o J2 - sin 20 - Ji. 0 / I- ~ sin 2 0
\ -

= J2I ,
I( ( I/ v 2)
r,:;
= 1.3 I I 03 · · ·
(Scl! Problem 13.)

Once again, the reference to Spiegel presems many examples and the reference
to Hancock is un entire book on Lhe subject.

3. 5 Problems
l. Derive rhe cqu:uion of rnorion ( F = ma) of the pendulum shown in Figure 3.15.
2. Problem I ~hows 1ha1 the equation of motion of the pendulum shown in Figure 3. 15 ism/~~ = -mg sin 0.
dr-
Show that energy is conserved for this system. Hint: Muhiply bolh ~ides by d0 Jdr. and use- the rela1ion
2 2
-d (d0)
- = 2dO- -.,
d 0 and then .integrate both s1des
. .
tl! dr di rlt-
3. Let 0 be restricted 10 small angles in Equation 4 and show 1hat 1hc mo1ion is co1sinusoidal with a period
r = 2.Jr(//f:.)1/~_
144 ChJpter J / 1Fune1ions Deii!iL-d As Integral s

4. We'll derive Equation 6 in 1his problem. First use the trigonometric identity cos f) =I- 2 sin 2 (0 /2) to write
Equa1ion 5 as

4/ ) l/2 [°ci dfJ


t' =( g lo -I s-i-n2_(_0_/_2_)---s-in-2-(0_/_2_)_]1-/2
0

Now Lra.nsfonn the 00 upper limit in this equation 10 11/2 by defining a new integral.ion variable,, by 1.he
relarion sin(B /2) = sin" sin(Oo/2) and derive Equation 6, (Note that sin 11 = I or 11 = rr /2 when 0 = 00 .)
5. Show that K (0) = rr i2. £(0) = rr /2. K (I) = oo. and £(I)= I.

6. Calculate the arc length of the curve y = cos 0 from Oto 2:rr.
. . . . . x'.! \'2
7. Use Example: 2 10 calculate the c1rcumlerence of the ellipse whose equation 1s - + :.._ = I.
4 16
6
8. Show that E(k) = -rr [ I - ( -I ) k 2 + ( -I · 3) -k~ - ( -
I . 3-
. 5 ) J;.- + · · ·] ,
2 2 2-4 3 2-4-6 5

9. Evaluate / =
l ".,
0
d0
(3 - cos 0) 11•
. f . . .
,, m tcnns o elliptic mtegra s.
I

IO. The answer 10 the previous problem is K ( I/ v'2) - F( lj ..fi., 1r /4). U~ dlher tables or a CAS to a.-.sign a
numerical value to this result.

J 1. Use either rabies or a CAS 10 verify lhal the answer 10 Example 3 is 2.42211 , • •.

12. Use a CAS to evaluate the integral in Example 3 direc1ly and compare your unswer to thal in the previous
problem.

13. Use either tables or a CAS to verify that 1he answer to Example 4 is 1.31103 • • •.
14. Use a CAS to evaluate the integral in Example 4 directly and compare your answer 10 thal in the previous
problem.
15. Derive Equations 21 and 22.
3
16. Evaluale / =
1 dx
-;:::===;===::;;;::
0 J(l6 - x 2 )(9 x 2)
in 1cnns of elliptic integrals. Hint: Lei x = 3 sin H.

17. Evaluate / = 12rXI (x


1 I~.\" ,,
- 4) I (x~ + I)
I , in terms of elliptic integrals.
/~
Hi 111: Lei X = 2 sec O.

3.6 The Dirac Delta Function


ln this section we will introduce a famous function Lhat is no1 a function. Consider
the funclion defined by (Figure 3.22)

-a u X X < -Q

-a < X < (l (I)


Figure 3.22
Tl1e function q>0 (x) defined by EqU!ltion I x>a
ploued a.gainst x.
3.6 The Dirac Delta Function 145

where h and a are rela1ed such that


00

1-oc
tf>a(,x)dx == 2a/J == I (2)

Now let"s form the inLegraJ

where f {x) is a continuous funcLion. Because of lhe definition of <Pa(x),

Because f (x) is conti.nuous.. we can use the mean value theorem for integrals and
Equation 2 Lo write

where -a:::~:::: a. As a ➔ 0. ~--+ 0 and/ becomes

lim / == lim
a-+O a .... o
JCO
l-a
a
tf,11 (:<) dx == lim
n-+O
/(0 1
11

-a
-dx = /(0)
2a
(3)

By mul1iplying /(x) by t/>0 (x) and then lening a --+ 0 and h ➔ oo. such that
2ah =
I, we have sifted oul Lhe vaJue off (x) al .t 0. =
We can use a construcLion like </>0 (x) to sift oul J (x) at any value of x. To ".
isolate J (x0 ), let

x < x0 - a
x0 - a < x < x0 + a (4)
XO+ a< X x0 - a x0 .r0 + a X

with 2al, = I (Figure 3.23). Equation 4 simply defines tf>a(x) to be cenrered al x 0 Figure 3.23
rather than al x = 0. Clearly The function 4'>r,0(x) defined by Equation
4 plotted ngainsl x.

(5)

Jn the limiting process in Equa1ion 5. t/>., 0 (.x) is gening increasingly narrow


and increasingly tall such tha1 the area of Lhe rectangle 2al, = I. We denote this
limiting "function" by 8(x - x 0 ), and write

0 X ;af= Xo
8(x - x 0 ) = { 00 (6)
x =xo

always keeping in mind 1ha1 Equation 6 is a short.hand notation for t/>., 0 (x) as a --+ 0
nnd h -+ oo such 1ha1 2ah = I. Physically. Equation 6 represents n spike at x = x 0

al
146 Chaplt'r J / Fun tiom D med, lnt - rals

or an impulsive force if x reprcscn1s rime. In terms of 8(.r - x 0 ), Equa1ions 2 and 5


read
"X)

f
• -.; 8(x - x 0 )dx =I (7)

and

where j(x) is a continuous func1ion. Equa1ions 7 and 8 serve lo define 1he Dirac
dt•lrafimctirm, which was introduced by the 8ri1i:.h theoretical physic.;ist Paul Dime
in 1927. Equa1ion 8 illustrates 1.he sifting property of Lhe delia function.

Example 1:
Evaluali:! / = 1_: 8(.r - x 0 )P- 1 \1.r.

SOLUTION: The function e- i:- continuous. so we simply repLll-l' .r by


. .J
x 0 in , - , 10 get

Ano1her function that can acl a.-. ::i delta funct.ion in a limiting process is the
Gaussinn di--tribution function:

(9)

which we discussed in Section 3. Figure 3.24 shows p 0 (.r) plot1ed again~t .r for
y
severnl values of a. The function is ccntercd HI x = x 0 and the width about x 0
is governed by the value of a: the smaller rY is. the rnarrowcr and more peaked
is p"(x). The factor of l/(2m1 2) 10 in front ~Lssures 1h111

TI1c curve$ in Figure 3.24 gt.;1 both narn1w.:r ,md u1Her wi,th de(.·re:asing value$ of
Figure 3.24
The Gau-.,;ian di~1ribu1ion in Eqll..'.ltion 9 a because the areas under 1h~ curves arc all the same (equal 10 1i.
pkmcd _gain-I x for (T = 1.0. 0.10. rmd If we multiply a con1inuous function /(x) by f'rr(x) in Equ..11ion 9 nnd let
0.050 (sec Figurc 3.9}. rr ~ 0. then p 0 (x) approaches cS(x - x 0 ) :
J.6 The DirJc D •ltc1 Fun :lion 147

Example 2:
Evaluare

explicitly and then let a -). 0 to show thar lim /(a)= cos x0 .
(J'-,0

SOLUTION:

1/2
The first integrul is equal 10 2 ( ~) ae-n'f'!. and the second integral is
equal to zero because the intcgr.ind is an odd function or x. Thus.

and so
lim /(a)= cosx 0
,,_,.()

We can present Equations 4 and 9 in a more general way. From Equ.ition 4.


detine

n .r < - - ",,
II (I {I
<5,,(x) = 2a
-- <
II
X < -
II
( I 0)
{I
0 X > - ,, .----- 20

and from ~uution 9. define


------ 10
-OO<X<OO ( I I) -------- 5

The graphs or the sequences iS,iC-r - x0 ) in each case get taller and narrower as 11 Xo X

increases (Figure 3.25). In both cases. 1_: 8,,(x - x0 )dx = I for any value of n, Figure 3.25
The dch11 .~cqucncc in Equation 11 plotted
and against.{ for,, = ~- 10. and 20.

for any continuous function f(x). Con::.equently. we .,ay that Equations 10 and
11 are de/Ja :r.equences in the sense thal a" (x - x 0) ➔ <S(x - x0 ) as 11 ➔ oo. They
148 Chapter 3 / Functions Defined As ln1egrals

both have the propeny that

( 12)

and that
o,. 20
(13)
10

5 Another deha sequence is (Problem 4)

n
8n(X - :co) = ---.,----
rr [I+ n-(..r - x ) 2 ]
-OO<X<OO ( 14)
Xo X 0

Figure 3.26 Equation 14 is ploned against x - x 0 for various values of n in Figure 3.26.
The delta sequence in Equa1ion 14 ploued The delta function is nor a function in Lhe strict sense, but has meaning only
against x for n = 5, 10. and 20.
if it occurs multiplying a continuous function under an integral sign. For example.
we can assign a meaning to :co(x) by multiplying by a continuous function J (x)
and integrating 10 get

J_oc f(x) x 8(x) dx = 0 · f(O) · I= 0 ( 15)

and so one often sees the expression

x8(.x) =0 ( 16)

Keep in mind. however. that Equation 16 is a shor1hand notation for Equation LS.
Similarly. we write

x8'(x) = -S(x) ( 17)

and

I
8(ax) = -8(x) ( 18)
a

where, once again. you must be aware of rhe meaning of these relations (Prob-
lem 1).

3.6 Problem
1. Write out the actual meanings of Equations 17 and 18.
2. Show that x8'(x) = -8(x).
3. Show that 8(ax) = a- 18(x).
3.7 Bernoulli Numbers and Bernoulli Polynomials 149

4. Show that Equation 14 is a deha sequence; in other words, show 1.hat L: 811 (x - x 0 ) dx = 1 and Lhal

lim
n-oo
f c-:J S,, (x - x 0 ) f(x) dx = j(:r0 ) if f(x) is continuous.

5. Use the delta sequence in Equation 14 to prove Lhat j_..r S(11)d11 = H(x), where H(x) is the He:iviside s1cp
00
func1ion.

6. Multiply 811 (x) in Equation 14 (wilh x 0 = 0) by e-.r\ integrate from -oo to oo. and then let 11 - oo to show
oo 1:x: e-o,i
J
2 ,r l
thar Jim. . 811 (x)e-x dx =I.Hint; You need lo use - - - ,dr = -<· 0
..r erfc(Jax).
11 - • ' - -oo o I2 + x- 2x
00
7. Noricc that the delta sequences in Equations 10-12 arc of the fonn p(11x). where /_ p(x)dx = I, Show that
f: j(x)p[n(x - x 0 )\dx = f(x 0 ) i.f f(x) is conrinuous.

8. Evaluate /(a)= (2;ra 2) - l/ l £ 00


2
e -(.t - .tol f2a! sin x dx explicitly and then let a➔ 0 to show thal
llm /(a)= Sin Xo.
tJ-•O

3.7 Bernoulli Num bers and Bernoulli Pol nomials

ln this section, we will first inLroduce the Riemann zetaftmction, which is nol de-
fined by nn integral but by a summalion. If we dcno1e 1he Riemann zeta function
by {(s). then

((s)
.
=, -
ooI
LkJ s>l (I)
k=!

We know from Chapter 2 that {(s) diverges for s _=s I. being the hannonic se1ics
when s = =
I. We also used the fac1 that {(4) rr 4 /90 in Section 2.8. The zeta
function has been a wellspring of rich and extensive mathcma1ics when sis allowed
ro take on complex values, but we shall reslrict s to a posi1ive integer, n. We will
use { (,,) simply Lo discuss Lhe sums of reciprocal powers of integers. which arise
fairly frequently in applied mathema1ics. We shall evaluate {(2) and ((4) with the
aid of Fourier series in Chapter 15:

oo I ,
{(2) =' - =~6
L k2
k:::.I

and
I
~(4) =' -=
L k
co

1;,,,,1
4 90
4
!!.._

C gl
150 h.ipr r J / Fun, rroM D1•f111e-d As lnl<-•i:::1,11~

Table 3.4
The values of {'(11 ) • .l(11 ). 11(11 ). and /J (11)
(Equations I through 4) for 11 =
I. 2. 10.

II {(n) ,\.(11) r,(11) /i(n)

I 00 0.693 14 0.785 39
2 1.6449 1.2337 0.822 47 0.915 97
J 1.2020 1.05 I 8 0.901 54 0.968 95
4 1.0823 1.0147 0.947 03 0.988 95
5 1.0369 1.0045 0.972 12 0.996 16
6 1.0173 1.0014 0.985 55 0.998 68
7 1.0083 1.0005 0.992 59 0.999 55
8 1.0041 1.0002 0.996 23 0.999 85
9 1.0020 1.0001 0.998 09 0.999 95
10 l.OCHO 1.000fl 0.999 01 0.999 98

Abramowitz and Stcgun (see references al t.hc end of the chapll!r) define 1.he
following auxilary func1ions involving sums of reciprocal powers of in1cgcrs:

(2)

O(.J I
A(II) (J}
L (2k + I)"
J.:=O
and
oc (-l)k
/J(n)=E-- ( 4)
k==O (2k + I)"

The four functions {(11). >..(n). ry(11). and /j(11) are tabulaled in Table 3.4. Note that
all of them approach unity very rapidly as" increase.s.

Example 1:
Show that ,\.(11) = (I - 2- 11 )((11).

SOLUTION:

= A.(11)
J.7 Bernoulli Numb€r .incl Bernoulli Polynomials 151

Some speciaJ values of Equations 2 1hrough 4 are

~ I I :rr 2
17(2) =I- - 1 '-+ - 2 - - 2 + · · · = -
2 3 4 12
I I I 7rr 4
17(4) =I- - + - 4 - - 4 + . -. = -
24 3 4 720
I I rr 2
>..(2) =I+ 32 + 52 + ... = 8
I I ,, 4
A.( 4) = I + 34 + 54 + ... :-:: 96

I I I rr
/HI) = I - 3 + 5 - 7 + ... = 4
I I I rr}
/J(J) = I - J3 + 53 - 7·1 + .. . = 32

No1e 1ha1 17(1) and {i(I) are conditionally convergent series and recall from Sec-
t.ion 2.7 thal 11(1) = In 2 .
We occa."ionally need expressions for the ,.;urns of positive powers of rhc
integers. We !-howcd in Section 1.7 thal the sum of the flrsl n integers is given
by

tk = n(n,t
b,d -
I) (5)

and that the sum of the first ,1 square.c; is given by

t
k=l
k~ = n(11 + 1~211 + I) (6)

We derived these two formulas in Section 1.7 by what you might call tricks, but
there is act ua II y an organized. straight forward way Io derive t.he general form u Ia for
I.he sum for any integer power. There is an area of mathematics called t.hecalculusof
finite d~f!erences. which deals with sums instead of i n1egrals and differences instead
of derivatives. Just as we deal with differential equations in ordinary calculus, we
deal with difference equations in the calculus of finite differences (a common type
of difference equal.ion is a recurrence rela1ion. where the nlh term of a sequence
is expressed in 1erms of the preceding terms). II would take u.c; too much space 10
develop the tools of lhe ca.lculus of finite differences here. but we will finish 1his
chapter with some useful results from that field.
If we expand Lhe function

te'.r
f(x, I)= - -
e1 - I
152 Ch,1pl ·r ] I Func::lio n Defined As lnll' •rals

as a power series in r, the coefficients will be functions of x. In particular,

where 1he Bn (x )'s arc called Bernoulli polynomials and arc (Problem 2)

I .., I
B0 (x)=l; B1(x)=x- -; B,(x)=x---x+-
2 - 6
(8)
33x 2 X .3
B3(x) = x - - + -; Bix) =x J
- 2x
'l
+ x- - - )

· 2 2 30
and so on. Equation 7 is called a ge11emtingfimcrio11 for the Bernoulli polynomiaJs.

hample 2:
Show that B~(x) = nBn_,(x).
SOLUTION: Differen1.ia1e borh sides of Equation 7 wil.h respect to x .

=
-"'8
- L
1

n+l ·
(r)
,,,--1.J

(n + I)!
11""'0

where we used I.he fact thar B0 (x) = I. Bue

- ="'
12e1 ,r
,
e -
L I
"=-0
,ir+t
BII (.x)-
I
n.

Equaring equal powers of 111 + 1 in the 1wo sums gives

The calculus of finite differences gives us a general fonnula for the sum of the
mth power of the first n integers in terms of Bernoulli polynomials:

(9)

Nore that Equation 9 looks some1hing like an integration formula be1wecn Lhe
limits of O and n + I. except in I.his ea~. it's a summalion fonnula Let ·s use this
3.7 Bernoulli Numbers and Bernoulli Polynomials 153
formula 10 derive Equations 5 and 6. In the case of Equation 5. m = I, and so
Si= t k = B2(n + I;- 82(0)
l.:=l

=
(n + 1) 2 - (n + L) n(n + l)
=---
2 2
and form= 2

(n + I) 2 3 3
=- - ( n + 2n + I -
3
~n - ~
- -
+ ;.I )
l

= (n + I) ( 112 + ~) = n(11 + 1)(2n + I)


3 2 6
When .x = 0 in Equation 7, lhe resuhs are called Bemo11/li 11umbers (Prob-
lem 7):

I
Bo= I; 84 = --:
30
and

n~I

BernouJli numbers occur in a number of various formulas. For example. 1he


Riemann zeta function can be expressed in terms of Bernoulli numbers for even
values of its argumenr:

( 11)

If n = I, we have {(2) = (2Jr) 2 82 /4 = 11 2 /6, and 1f "= 2, {(4) =


(2rr) 1B4 1/2 · 4! = rr /90.
4 4

Bernoulli numbers also occur in the series cxpan$ions of certain functions.


For example,

x3 2x 5 17x 7 2 ( - 1)'1 -l22n(22n - l)B nx 211 - 1


1anx =x + - + - + - - + · · · + - - - - -- -----+ ···
3 15 315 (211)!

and

, x
CSCX=-+-+-+···+------Bu,.x
1x 3 c- w•1-1(22n - 2) 2,,-1
+···
X 6 360 (211)!

They also occur in the Euler-Maclaurin sumrnal.ion fom1ula. which we'll see
below is a scheme for approximating a summation by an in1cgraJ. For example,
154 Chaµl~r J / Funct ions D in ed As lnt · grals

the series
N
S = L(2J + l)e-aJ(J+I) ( 12)
1=0

where CJ i., a parameter occurs in the sta1istical mechan.ical Lrca1mcn1 of rotating


molecules. If a is small. then a plo1 of 1hc succcssi,•e tcnns in Lhis series is
esst:ntially the same as a plol of (2J + l)e-a-J(J +I). with J being treated as a
continuous variable (Figure 3.27). Figure 3.27 suggests that we can approximate
50 J
Lhc summation in Equation 12 by an integral, and write
Figure 3.27
The terms of 1h • series in Equa1io11 12 and
I he conti nuou5 function (2 1 ~ I)e 1 1 1
(13)
=
ploneu ag:iin~1 J for a OJJO I.
If we let x = J(J + I). then Jx = (2) + l)dJ and S becomes S ~I/CJ.If a=
0.00 I (a fairly typicaJ value), then Equations 12 and 13 differ by les., than 0.51:,.
Because it is usuaJly ca!-.icr to integrate 1han it is to find a closed cxprc~!-.ion
for a 1:;ummation. being able to approximate a summation by an integral is very
convenient. It is possible to express a summation as an integral plus correction
terms. The series

L" f(x) = Jn J(x)dx + -IJ(11)


I
+ /(1)]+ L oo 8
u [!( 2'-"- 1\n) - f2k-ll(I)]
.r""I I 2 i:,,,, 1 (2k)!
(14)

is called the E11/er-Maclouri11 swnmmio11Jon11u/a. Note that il t!Xprc,-.c._ a summa-


11on as .in integral plus a sum of corrccrion tem1s. The first fcw 1erms of Equal ion 14
arc

L" f(k) = f" j(x)dx + -IJ(11)


k=I I 2
I I
+ j(l)I + -u<n(n)
)2
- J1'>(1)]

1 1
- - -11°\11) - / 13 l(l)J + - - [ f 5 >(n) - f 51 (1)l + · · · (15)
720 30 240

where we assume that all the derivatjves of j(x) are well behaved.
Let's apply Equation 15 10 the summation

The various 1enns in Equatjon 15 are

!
11

1
j(x) dx = 1"d__::_ =
0 xl
I - -I
II

I J(n)
-[ + j(l)I = -I ( ---:;I + I)
2 2 11.:.
J.7 Bl'111011lli i'.umb1·r, .rnd Bernoulli Polynomi.1ls 155

I [ J rJl (fl)-J (3> (I) ]


-- = -I ( -I5 - I)
720 30 n

and so on to obtain

" I (\--nI) +-21(1,+I


'°'-=
{;; k 2 n-
) --61(1- - 1 ) 11 ·1

which give.,; 1.5620. compared to the exact value or 1.5498 (to four places) for
11 = 10, a difference of less than I%.

Example 3:
Use the Euler-Maclaurin expansion to c.:alc.:ulatc Inn! for n = 10.

SOL U T 10 N: First write In 11 ! as L In x and lhen apply Equation 15:


.t:::: I

Ii" In x dx = [ x In x - x I= n ln 11 - 11 +I

1 I
-I f(11) + f(l)I= - ln11
2 ~

and so on 10 obtain

" I
= ,, In n - 11 + 1 + - In 11
2

+-I (I--1 ) - -
I ( -I - \) + -I- ( __
I
5
,) - -
I ( -I - 1 )
12 n 360 ,.:i l 260 11 1680 n7

This formula gives I 5.0992 versus 1he exacl value of 15.1044 I (lo fi vc
places) for n = 10. a differcm:e or0.03 %.

Before we leave this sect.ion. we should point out one fact about Bernoulli
numbers I hat sbould be kept in mind. We gave the values of the 8 11 'sup ton = 6. and
156 Chapter 3 I Functions Defined As ln1t•gr.ils

lhey appear 10 decrease with increasing 11 (ac1ually 2n since B 211 + 1 = 0 for n ~ I).
If you look at a table of Bernoulli numbers, however, you'll see that 8 8 = -1/30,
= =
Bw 5/66, B12 -691/2730, and that lhey get larger and larger in absolu1e
vaJue as n increases. In facl, i1's possible to show that Lhe values of ]Bui I satisfy
I.he bounds

2(211)! ( 22n ) 2(2n)!


- -1 -21- - >IB, l > - - (16)
(21r )2, 2 ' - 2 ·" (2,r )2n

The factor 2211 /(22n - 2) goes 10 unity as " increases, and is equaJ to 1.000002
when n is only I0. Thus. Equation 16 shows that

(17)

For example. Equation 16 gives I8 20 1 = 529.1237 versus lhe exac1 value of


529.1242 (to four places). This behavior of the Bernoulli numbers does nol en•
danger lhe usefulness of the Euler•Maclaurin expansion, however. because the
coefficients in Equation 15 involve the ratio IJ.i.k/(2k) !, which Equation 17 shows
goes a, 2/(2Ir) 2k_

3.7 Problems
I. Show 1ha11J(11) = (I - 2 1-nJ{(n).

2. Use Equation 7 to derive expressions for 1he first few Bernoulli polynomials.
3. Show 1ha1 Bn(x + I) - B,,(x) = nx"- 1.
4. Show rha1 Bn(I - x) = (-1)" B,,(x).
Bn+i(x) - 8 +1(0)
1
.r
5. Show that B,.(u)du = --'-------'-- . 11

a n + I
6. Show that (- I)" B,. (-x) = B,,(x) + 11.x 11
-
1.

7. Use Equation 7 with x =0 10 derive the first few Bernoulli numbers.


I/

8. Use Equation 9 10 derive a formula for S3 = Lk 3


. Compare your re.suit 10 Lhc one in the CRC Ma1Jrematical
11~ l
Tables.
9. Use Equation 17 10 show that {(2n) - I a..~ n - oo.
00

10. EvaJuate L ~-
,r=I II

11. Use the Euler-Maclaurin expansion to evaluale Sm= L km form = I, 2. and 3. Compare your rcsull for S 3
k=l
to 1hc one that you get in Problem 8.
II

12. Derive an Euler-Maclaurin expansion for L x- 112 and use it to evaluale the sum for 11 = IOand n = 50.
x=I
157

13. Use a CAS 10 evaluate 1he sum in 1he previous problem and compare your answer 10 1he one ob1ained using
lhe Eulcr•Maclaurin expansion.
00

14. Com,ider 1he sum S = L e-"k = --


k--0
1
- (geomerric series). Expand Sas a power series in a. Then apply
I - e-a
the Euler-Maclaurin summation fonnula 10 1he sum and compare your result.
15. One of I.he most famous applications of the Euler-Maclaurin summation formu.la 10 a physical problem ics ~ls
applicalion 101hc ro1a1ionaJ partition func1ion of a rigid rotator (a good model for a rotating diatomic molecuJe).
0j

The ro1alionaJ partilion function 1s q = L(2J + l)e-<-:-JJ(J+IJ/T_ where T is I.be kelvin tempernturc and 0 is
J=-0
a parnmc1er 1..ha1 is characlerislic of the molecule. Apply the Euler-Maclaurin summation formula 10 thi!'. -~urn
and derive

Typically, 0 / T is small. so this mms out to be a very useful expansion.


. X oo 8,,xri
16. Use Lhc defining expression for the Bemoulh numbers, - - = ~ - - . to show 1hm
ex - 1 L 11!
II~)

( - 1)11'221'
cot x = L ---- B2.1,x 2"- 1.
,,=0 (2n )!

17. Ut-e a CAS 10 verify tha1 the ~ries in Equation 12 and 1he integral in Equation 13 differ by less lhan 0.05'¾ for
a= 0.001.

References

CRC Standard Matlienuuical Tables nnd Fon1111/ae. 30th ed .. edited by Dooiel Zwillinger.
CRC Press ( 1996)
Harold Davis, 1962. Summation of Snies. Principia Press ofTriniry Universi1y
William Dunham. 1999. Euler. The M(lster of Us All. Ma1hcma1ical A~soc,i:nio n of Am1::ric::i
HllITis Hancock. 1958. Elliptic lnu:grals. Dover Pub\i(:alions
Handbook of MnthC'maticol Functions, edited by Millon AbrJmowirz and Irene S1egun.
National Bureau of Standards Appliw Mathematics Series 55 ( 1964)
Hit,:her Tra11.sce11denwl F11nc1ions. 3 vols .. edi1cd by A. Erdclyi et al., McGraw-Hill (1953)
Wilhelm Magnus and Frirz Oberhettinger. 1943. Functions ~{ Mwhemalical Physics.
Chelsc.n Publishing
Murray Spiegel. 1963. Sclwum 's 0111/i11e of Advanced Calcu/11s. McGraw-Hi II
Murray Spiegel. 1971. Schaum 'J Ou1/ine of Finite Diffen.·11n·s and Difference Eq11a1io11s.
McGraw-Hill

gl
CHAPTER 4
Complex Numbers and Complex Functions

Complex numbers are usually introduced by considering a quadratic equation of


the type .r 2 - x + 1 = 0. where the quadra1ic formula gi,·cs

x=~± R=I±,./3
2 2 2 2
where i = J=1 is the imaginary uni I. A number of 1hc fom, a + i I,. where a and
b arc real numbers. is called ::i wmplex numbu. If a= 0. then x = ih is called an
imaginary number. The mc,-.-;agc here is t.hat we mus1 imroduce imaginary numbers
in order to be able 10 solve quadratic equations in general. LI shouldn't be surprising
that initially there was a great resisiance lo the inLroduction of complex. numbers
and th:it it 100k many years for 1hem to be accepted as legitimate members of our
number system. The very name "imaginary number" seems lo convey a certain
degree of mysticism 10 these number:-.
If complex numbers had arisen only wilh quadratic equations. Lhen it m[ght
have been easy to reject them by asserting that the equation x 2 - x + I = 0 has
no solu1ions. Aflcr all. we're probably comfonablc s::iying Lhal sin x = 2 ha.-. no
solution for real values of x. H is1orical ly. it was in the study of the solutions to cubic
equalion'i that iml.lginary numbers were mos I puzzling. Consider the cubic equal ion
x 3 + 2x 2 --- x - 2 = 0. You can vcrif y by inspection that Lhis equation has three
rcaJ roots. ± I and -2. Yet when you solve this equation using 1hc s1andard (fairly
messy) fom1Ula for calcula1ing the three roo1s. square roots of negat.ive numbers
occur al several intermediate steps. In fact. the very requirement that I.here arc t.hree
rcnl. dis1inc1 roo1s lead:, to ,t he . qu:irc 1ioo1 of a negntive numhcr. The linal results
arc 1he three real roots. so it i-. apparent 1hal the occurrence of imaginary numbers
doesn't invalidate any of the formu 'la~. Eventually ma1hema1icians not only ea.me
10 1olera1c imaginary numbers. bu1 to embrace them fully.
You might wonder if more complicated polynomial equations (such as 17th
degree equations) require the introduction of types of numbers .. beyond'' complex
numbers. It turns out that Lhey do 1101. There is a remarkable theorem of algebra
called nothing less 1han 1he frmda111e111a/ theorem of algebra tha1 says that every
Nlh degree polynomial equa1ion, oNxN + a., 1x"'- 1 + · · · + a 1x + a 0 = 0, even
with complex numbers as coerfic-ients. ha.'.' exactly N roo1s over the complex
159

C gl
160 Chap1er ,1 / C.11111pll'-. Numhr·r, and Complex Funcrions

numbers. In other words, complex numbers are sufficient to satisfy any polynomial
equation.
[n this chapter we shall first review 1.he aJgebraic rules for complex numbers
and then consider complex numbers as variables and define functions of complex
variables. The study of complex variables is one of 1he richest areas of mathemat-
ical analysis and has countless physical applications. We shaU learn only 1he basic
propenies of complex functions in this chapter. and wait until Chapter 18 to exploit
them fully.

4.1 Complex Numbers and the Complex Plane

We denote a complex. number by c =a+ ib where a and bare reaJ numbers and i
is caUed the imaginary unit. Tbc imaginary unit has the propeny that i 2 = -1, and
is sometimes wrinen as i = J=I_ (Because 3Il electric current is usually denoted
by i. electrical engineers often denote complex numbers by a + j h.) The real
numbers a and b are called the real and imaginary pans. respectively. of c, and
we write

Rec=a lmc:=h (I)

The real numbers arc clearly a subset of 1he complex numbers because c =a, a real
number. if b = 0. The complex. number,_.= 0 + iO corresponds LO 0. If a= 0 and
bf. 0. then c =ibis called an imaginary number. or a purely imaginary number.
l\vo complex numbers are equaJ if and only if their real and imaginary parts are
equal; i.e.

where<===? means "if and only if.''


We can do arithmetic with complex numbers by using the fol lowing rules: We
can multiply a complex number by a real number f3 according lo

f:)c = /Ja + if3b (2)

The addition of two complex numbers is given by

For subLraction, simply add c 2 = -a 2 - ib 2 to c 1. To form the producl of two


complex numbers. we multiply I.hem as we would multiply two binomials, using
the relation i 2 = -1. Thus,

c 1c 2 = (a 1 + ib 1)(a 2 + ib2 ) = a 1a 2 + i 2b 1b 2 + i(a 1b 2 + b 1a 2)


(4)
= (a1a2 - b1b2) + i(a1b2 + b1a2)
Finally. division is obtained in the following manner:
4.1 Complex Numbers and the Complex Plane 161

(5)

An imponant quan1i1y associated with a complex number c is ils complex


conjugate, defined by

(6)

(Some authors use c to denote a compleit conjugate.) Nore 1ha1 ccK = (a + ib)
(a - i b) = a 2 + b 2 is an intrinsicaJI y positive quanthy (unless a = b = 0). We call
~ the modulus. or absolute value. or 1he magnitude of c and write

(7)

Equation 7 also !ells us 1ha1


c•
(8)
C IC 12
Complex numbers cannot be ordered, in the sense lhal the inequali1y c 1 < c2
has no meaning. Nevertheless. the absolute values of complex numbers. being real
numbers. can be ordered. Thus. for example. lcl < I means tha1 c is such that
Ja2 + b2 < I.

hample 1:
Given c 1 =-I+ i and c 2 =2 - Ji. find 1,y 21and lci/r 2 !.

<"1C2 == (-1 + i)(2 - 3i) = (-2 + 3) + (1 + 3)i = I + 5i


Ct (-l+i)(2+3i) 5
-=
(2 - 3i)(2 + 3i)
= ---
13
-13
Then

1·~
c
1= J26
2 13

The rule.,;; that we have presented for mnnipulnling complex numbers are the
ones lhat are usually presented in introductory or elementary discussions. In more
162 Chapter 4 / Complex Numbers ;ind ornple x un :1 iom

•y advanced discussions. ii is desirable to treat complex numbers as orderetl pairs

. z= x+ir.
of real numbers (a. b) that sari sfy certain algebraic rnles. For example. we -;ay
1ha1 (o 1• h 1) = (a 2 . b2 ) if and only if a 1 = a 2 am.I b 1 = h 2. Similarly. if a is a
rc:il number. 1hen a(a. h) = (aa, ab). Addition and subtraction satisfy the rule
(a 1• h 1) ±(a:!., b:!.) = (ll 1 ± t1 2 , b 1 ± b 2) and multiplication saris lies the apparently

X complicated rule (a 1• h 1)(a 2 • b2 ) = (a 1a 2 - b 1b~. t1 1b 2 + a 2 b 1) . If you compare


these rules 10 Equations 2 through 4. you see that th~y are the very same. The
advantage of the more abstract ordered pair appronch is 1hat it is based entirely on
Figure 4.1 real numbers (the imaginary unit doesn't appear in any of the rules) and that it can
The compk:t plane . The rtal p.irt of
be used to develop an axiomatic foundation of complex number-:.
- = x + iy is plotted along the horizomal
axis and the i~ginary pan is ploued The fa1.:1 that a complex number is an ordered pair of real number.; suggest,
along the vcrtic.:.al axis. that we can represent a complex number as a point in a car1e.sian coordinate system.
=
From now on. we· II use the standard notation :-. x + i y. We· JI lel Ihe horizontaJ
axis represent the real part of:: ;rnd the vertical axis represent the imaginary pan of
.::. a~ shown in Figure 4.1. The horizontal axis is called the real t1.ris and the vertical
axis is <.:al led the i111ogi11ary m:is (al1hough the real number y is plotted along that
axis) . The xy-plane is called the complex plane. The distance of the point :-. lo the
origin is the modulus of: be1.:ause 1:1 = Jx 2 + y 2 is equal 10 this dist;.mce .

2 X

Example 2:
De1ermine thi:: curve in the complex plone that i~ de:-cribecl by I: - II = ~-
SOLUTION:

Figure 4.2
T11c gr-.iph of 1- - II = 2. or
(x - IJ~ + v2 = 4. in the complex plane. and so I.: - 11 = 2 com'..'sponds to

which is a circle of rndius 1 cen1ered al x = I. y = 0 (Figure 4.2).


y

Example 3:
Determine rhe set of points in the complex plant that is tlcscribcd by
.r lz-11 5 2.

SOLUTION: In this c.a."-C, we have

which is the entire region bounded by the circle (:c - I)~ + y ~ = 4, i nduding
Figure 4.3 1he circle itself (Figure 4.3).
The region in the comple x plane described
by I: - II::: 2. or (x - W -1- y 2 ~ 4 .
4.1 Compl • umbe ,incl 1h , C mpl i,: Plan<.' 163

\'

\'

, I •I
z.
I
I
I
_J
X
I X

I
I I

X
- 1111
I
I
•z
Figure 4.4 Figure 4.5 Figure 4.6
A ge11mc.1ri.,;.1l iflll•rprt'1a1i,in nf the: The nega'1i,1; ,if a l'. 11111pk __, m1111llc r i~ it~ The complex conjugate or a complc-11
adJi iion l,f IWo c11111plcx numhcr-. .•~1 rc11i:trion 1hn>ugh lhe ori~in . 111111,ber i., i1, rclkc1ion through the ., ax k
and .: 2•
The addition of 1wo numbers in the complex plane has a nict: gcomcuical
inlcrprc1a1ion . Figure 4.4 illustrates the adJi1ion of: 1 = x 1 + i_,· 1 and - :! = x 2 + i,r~ -
Thc poinl : 1 +::!completes 1hc p::irallclogr.un whose legs arc : 1 and : .:! - We say
,.
tbJt the addition of : 1 and :: 2 satisfies the JH1mlll'lngra111 lmr. Two other operations
have simple geometric interpretations: The ncg:nivc of a complex number is its
reflection 1hrough the origin (Figure 4.5) and the complex conjugate is a rcnection
through the x axis (Figure 4,6).
If we refer lL) Figure 4.7. we see that we can represent a complex number: in
polar fvrm. by lc11 ing r be the distance of =-. from 1l1c origin and f-l be the ~tng!c that
a Ii ne from the origin !o : makes with the real axis. Thus. we h,1ve

.x = r cos 1-1 y = r sin fJ (9) X

and figure 4.7


TI1c ~1lar form u f • lm:a1c:-. lhc poinr : by
~J'L'<.:11'~ in.~ r a11J /· .
:. = r{cnsH + i ,in H) ( 10)

where

(I I)

und

\"
Ian (-I=:... ( 12)
X

The angle 11 is c.1lled Ihc arg111m•111 or Ihc amplirut!e of: and r is irs modulus. We
ofrcn denote thc.'-C two quanti1ic~ by A= arg ~ and r I z. j. l:.quation 10 i, called =
thc110/arfon11 of:: (Figure 4.7). Equation 10 implies rhat :-"(r. 0) = :(r. - 11). 1hc
reflection of-~ through the .x axis.
164 Chapter 4 / Complex Numbers and Comple:io:. Functi ons

z=-l+i y
Example 4:
Express z = - I + i in polar fonn.

so L UTION: The modulus of z is .Ji. n.nd tan 0 = _!_ = - I or 0 = 3rr /4.


-I
Thus. lhc polar form of z is
X

z = v~L. ( cos 311 .. 3,r)


Figure 4.8
The complex number z = - I+ I in polnr
4 + sm 4 1

fom1.
(Figure 4.8).

The polar form of z provides a geometrical interpretation of the product of


two complex numbers. Write

z1z. 2 = r 1(cos 0 1 + i sin 01)r2(cos 82 + i sin 02 )


= r 1r2[(cos 0 1 cos 02 - sin 0 1 sin 02) + i (sin 0 1cos 0i + sin 02 cos 0 1) I
= r 1r 2[cos(0 1 + 02) + i sin(0 1 + 02)J
Thus z.= z 1z2 is obtained from ;: 1 and z2 by multiplying the moduli and adding the
two polar angle.,.

4.1 Problems

I. Pro't'C Lha1 (a) (z 1: 2)''" = zi z 2. (b)


= 1/z•. and (c) (z.n)" = (z.~)" where 11 is a positive integer.
(lh)"

2. Show lhal Re (z) = (z + z·)/2 and 1ha1 Im (z) = (z - z")/2i.


3. Show that \z 1z2I = lzdlz2!-

4. Does_!__=
I:!
1~1?
z
5. Determine the cwve described by I~ - 21 + lz. + 21 = 5. What type or curve is it?
6. Determine lhe region in the complex plane described by I < Jz +ii~ 3.
7. Dc1ermine Lhc region in Lhe complt-x plane described by ,r /4 ::S arg z ,:::s ,r /2.

8. Determine the region in 1he complex plane dc$cribcd by lz - 2i I < lz + i 1.


9. Determine the region in Lhc complex plane described by Re(;:+ i) ~ 2.
10. Evaluate
'\ . 3
(a) Re - +1 (I - i) I (d) Im ( I
1
- - - )
I -i (b)
I + 2i) 2
(3 3+i 3- i
11. Express the following complex numbers in polar fonn:

(a) 2 - 2i (b) -I (c) -2 - 2./3 i (d) -3 + J3 i


4.1 Functions o( a Complex Variable 165

12. Expre.ss the following polar forms of complex numbers in cartesian forms:

(a) 2 ( cos ~ + i sin ~) (b) v~-1 ( cos 2:rr .


+ i i.m "2:r)
3 3
( c) cos 3JT + ,. sm
. 3:rr (d) ?_ ( cos rr + ,. sin. rr )
2 2 6 6
13. Leibnilz was aware of I.he relation JI + R + JI - R = ./6 but <lidn 't know whu1 to make of ii. Prove
1hat this rcla1ion is correct.
14. Describe the sel of complex numbers {;J 1h:11 satisfy the cxpre""iun z. = T/Z i + (I - T})z. 2 where O::::: 11 :S I and
z 1 and ;:2 ore fixed.

15. Prove that j.: 1 + z.21::: l.:il + l:_1 . This is known as the triangle i11eq1J11lif)' because lz 1 + z.21- Jzi[. and lz2I
represent 1he lcng1hs of the legs of a triangle.
16. Show 1.har (i /2) 112 = (I + i)/2.

4.2 Functions of a Complex Variable

If we have a prescription from which we can cakula1.e a complex number w from


the complex number z. then we say that w is a function of::. and we write w = f ( z.).
=
If w is specified uniquely by z, then w f (:,_) is said lo be a si11gle-vn/11cdfimcrion.
Otherwise. w = f(::.) is multiple-valued or mu11y valued. We shall sec lalcr in lhis
section that a multiple-valued function may be considered to consist of a se1 of
single-valued functions, so we shall restrict our functions 10 be s1nglc-valucd (~e
Sect ion I. I).
Because : = x + i y. w wi II be complex and abo a function of .r and y. h is
customary to write w in lhc fonn

w = 11(x. y) + i v(x. _v) ( I)

For e,rnmple. if IJ) = f(z.) = z2• 1hen


w = z.2 = x 2 + 2i xy - y
2

so lhal

u(x,y)=x 2 -y1 and u(x. y) = 2xy

Example 1:
Suppose that w = f(z.) = 1/(1 + .:). Oe1cnninc u(x. y) and v(x. y).
SOLUTION:

I I (x +l - iy
w = J (z) = --
I+z
=- --- = -
(x + I) + iy
-- -'--
(x + 1)2 + y2
166 Ch,1r:iter 4 / Complex NumbNs ,md omµlex Fun ti ons

and so

u(x. r) = x+I
., ., anJ l'(X, \") = ____,_ _ _ \"

· (x + l)· · .v· · (x + 1)2 + y2

Figure 4.9
An illus1ra1ion of a mapping from che :-
plane to the w-planc. D represent" the We can visualize a funclion of a complex variahle a,; a transformation or
domain of w and R ~pn.'"S(:nl.s 1hc r.ingc
of w. a mapping of a set of points in the .:-plane to a set of points in the w-planc
(Figure 4.9). The set of points in the :-plane for which UJ is defined is called the
domain of u.1. and the set of corresponding points in lhe w-plane is called the range
of w. The domain and range of w arc illus1ra1cd in Figure 4.9.
Consider the function w = /(:) = :: 2. where O < x < oo and O < y < oo.
The domain off in this case is the first quadrant of the :-plane. ex.eluding the x
and y axe$. We saw above that w = 11(x. y) + iv(x. y) with u(x. y) = x 2 - y 2
and 11(x. y) = 2\'y. Because O < x < oo and O < y < oo. -oc < ,., < oo and
0 < v < oo. so that the range of u; is the entire upper half of the w-plane. excluding
the II ax i~. as shown in Figure 4. 10.

Example 2:
Determine 1he range of w = j(:..) = i::, for O < x < a and O < y <a.Map
the four line!- that are boundaries for 1.he domain in the :-plane into curves in
the w-plane.

SOLUTION: /(z.) = i::, gives u(x. y) =


-y and 1!(X. y) = x. Because
0 < x < a and O < y < a. 1hen -r, < 11 < () anJ O < 11 < a. The range
is shown in Figure 4.11. The boundaries of the domain are numbered in

l)
w = z2 u

X
u

Figure 4.10
The d()main and the range nf 1hc mapping 111 = f ( ·) = ;:~ for O < x < and() < _\"
4.2 Functions of ,1 Complex V,uiable 167

2 3
-------- (a, a)
w = iz
D 3 2 R 4

Figure 4.11
The domain and the range of the mapping
4
X II = =
w J (-) i ~ for O < .r < u und
0 < .I' <I.I.

Figure 4.11. Curve I. in the ;:-plane. (x = 0 and O < y < a) maps into v = 0
and -a < 11 < 0. which is curve I in the w-plane. Curve 2 in the :-plane
(y = u and O < .r < a) maps in10 u = -o and O < v < a, which is curve 2
in the w-plane. Curves 3 and 4 are also shown in the figure. Note thal the
mapping corresponds 10 a counterclockwise rotation or 90°.

Let's consider f (z.) = ;; 2 with -oo < x < oo and -oo < y < oo. In this ca-;e,
)'
the range of

and v(x. y) = 2.xy


is the entire u ll-plane. The curves
2 2
u(x. y) =x -y =uo and v (x • y) = 2x y = v0
X

define two families of curves in an xy-planc (figure 4.12). We' II now show that
these two families of curves are orthogonal to each other. Consider y to be a
function of.r and then differentiate u (x. y) and v(x, y) with respect to x.

dr
2r+2x......:....=O Figure 4.12
- dx
The 1wo fomilic.s of curves. x 2 - .1 = 11 0
solve for d.\'/ d.x in each ea~ and multiply the Lwo slopes togetherto get - I. Recall (dashed) and 2xy = l'o (. o lid).
that the result - I implies that the two curves are onhogonal.

Example 3:
Show tJ1at 1he 1wo families of curves 11(x. y) = u 0 and u(x. y) = i.:0 as...ociated
withf (z) = 1/z. are onhogonal.

SOLUTION:

iy
f (:.) -x y2- - -')--,
= :-I = .x- 2 + x~ + y~
168 Ch.:ipler 4 :' Complex Numh1•r, and Complex Functions

and so the rwo families of curves are

= --~-·\' - = v0
X
u(x. y) = -2--., = "o and v(x. y)
X + y·
1
x-- + y·
Diffcrcn1ia1e u(x. y) and u(x. )') with reSpt.~t 10 x 10 ob1ain

2r 1
--------
x2 + y2 (x2 + y2)2
and

2-ry dv 2Y~ dv
., , -=-- + , - , ., . = 0
(x- + _v·) dx (x- + y·)· dx
After a little algebra. the two equations give

dy y·' -- x-1 dy 2xy


dx
= 2.ry
and =
and so we see that their product is -1.

Lest you think that the families u (x. y) = 11 0 and u(x. y) = v0 are always
011hogonal. do Problem 15. II is true, however, tha1 if /(7.) is differentiable wirh
respect to the complex variable;::.. then the 11(x. y) = u 0 and v(x. y) = 1·0 families
of curves will be orthogonal. We have to wait until Chapter 18 before we define
just what we mean by ''differentiable .. with respect to the complex variable z and
why the families of curves" (.r, y) = 11 0 and u(x. y) = u0 are orthogonal in some
cases but nor in others.

4.2 Problems
I. Exprc~s e.acb of the following funct1ons in the form w = u(x, y) + i IJ(X. _,•):
(a) z.3 (b) I/{ I - :) 2

2. Exprcs:!i each of the following func1ions in the fonn w = u(x. y) + i v(x. y):
(a) I (b) -I (c) I-;::.' I
z. 2 + i Z
3. Detennincu = f(;_)if11(x,y) = .x/(x 2 +y 2)andv{x.y) = - iy / ( 2 + y 2).
4. Detem1ine w :::l /(z.) if 11(x. y) = l/(x 2 + y 2) and v(x, y) = 0.
5. Find I.be vaJues of w 1ha1 correspond 10 the points := I + i and;_= I - i under 1he mapping = /(:) = 1/:.
6. Find the values of w that correspond to the poinh: ,co 2 + i and z = I - i under t.he mapping w =JC:.)= .: 2.

7. Dctcnninc how the line that connects th~ two points in the ;:-plane in the previous problem is mapped in10 the
w-plane. Hinr: Take the equation of the line 10 be:= >n. 1 + ( I - ,, ): 2 where O::: 11.::: I and z I and z2 are fixed.
8. Determine how the line x = I mups in10 1hc w-plane under 1hc mapping ui = /{:_) = 1/:.
4.3 Euler's Formula ;ind the Polar Form o{ Complex Numbers 169

l'

a b
u I

1,

4 ' I~ I '
~
('

31 }' w = z2
2 /3
I ...; a

I 2 3 -I X - Ih --l -I 16 ll

Figure 4.13
The lines in the: ;:-plane 1ha1 arc 10 be mapped into lines in the w-plane in Problem 16 arc shown on the left.
The images in the w-plane arc shown on the righL

9. Determine the range of w = f (z) = 1/z for the domain 1::1 < 2.
10. Dcrcrminc the range of w = f (z.) = z2 for the domain O :":: x ~ I. 0:::: y ~ I.
11. Determine the range of w = J (z) = lz. - 11 + 2i if O < x < oo and -oo < y < oo.
12. Determine the range off(:)= z2 for I < x < 2 and I < y < 3. Map the four lines that are boundaries for the
domain in the z-plane into the w-plane.
13. Dctem1inc the range of f(z) = iz. + 2 for O < x < oo and O < y < oo. Map the lines that are boundaries for
the domain in the ;:-plane into the w-planc.
14. Show thar the two families of curves, u(x, y) = u0 and v(x. y) = u0 • a.<;sociated with J (z.) = i z are orthogonal.
JS. Show that the two families of curves. u(x, y) = u 0 and u(x. y) = v 0 , associated with /(z) = I l + i(y - x)
are nor orthogonal.
16. Consider j(:,) = ;: 2 for O < x < oo and O < _Y < oc. Map 1he line). shown in Figure 4.13 into the w-plane.
Show that the lines in the uJ-plane arc orthogonal. Notice tha1 the 90° angle inten.ections in the :-plane are
mapped inro 90° intersections in 1he rJJ-planc.

4.3 Eu ler 's Formula and the Polar Form of Compl Number

We learned in Section 2. 7 that Lhc series expansion or r;' is

-00<.X<OO (I)

We can exlend 1he definition of~ 10 complex values of x by defining e: by Lhc


series

(2)

C
170 Chap! r 4 / omplex Numbers and ompl x Fun tion

It is common in mathematics lO cxIcnd 1.hc definitions of runc1ions in this manner.


Equal.ion 2 certainly reduces l'O Equation I when :: is real. Problem 15 has you
show 1hat Equation 2 lead.,; to the relation e~,+~~ = e~ 1e::_
Leujng z be purely imaginary in Equation 2 (i.e.::= iy). we find that

The series in parentheses are cosy and sin _,·.however.so we arrive at the formula

/Y = cos y + i sin y (3)

Equation 3 is called Euler's form1Jla. and is extremely useful in dealing with


=
complex numbers. I r we lel y rr. Equation 3 becomes

(4)

one of the most remarkable formulas in all of maLhematics. Equation 4 contains


I.he imaginary unit. i. the two most fumous 1ransccndcntal numbers. e and rr (a
tr.rnsccndental number is a number which cannot be expressed as the roOL of a
polynomial equalion with rat:ional coefficients). and the two most fundamental
numbers of the real number system, o and I. The physicisL Richard Feynman,
used lo rcft'r to Equation 4 as a "jewel."
We can also ex.press Euler·s fonnula a"

(5)

From Equation 5. the real and imaginary pans or t': :ire

= 11(x. y) + iu(x. y)
ll ·s easy lo show that the two families 11 (x. y ) =11 0 and u (x. y) = v 0 a.re orthogonal
(Problem 14 ).
Euler's formula provides a convenient polar representation of complex num-
bers. We learned in Section I 1ha1 we can wri1c

.: = x + i _\' = r cos fJ i r sin 0 (6)

where the modulus r =


(x 2 + y 2 ) 111 is the distance from the origin 10 the point
=
(x. y) and O-= arg z is given by Ian O y/x. Using Equation 3. we have

;:. = r(cose + i sin 0) = reiFJ (7)


4.3 Euler's Formula and the Polar Form or Complex N11mlwr, 1 71

Example 1:
Express z = - I + i in polar form (Figure 4.8).

SOLUTION: r = Ji and Ian 0 = 1/(-1) = -1. s.o0 = 3rr/4. Thus,

Ln Example I, we took the angle 0 to be 3,r /4. We could also have taken fJ _v

10 be 3n /4 + 2.Jr, or even 3n /4 + 2rr n where II is any integer, lf we rcslficl 1he


values of 0 10 1hc intervaJ O .S 0 < 2rr. then 0 is called the principal argumenr of z
and is denoted by 00 or Arg z_ We'll denote genernl values of 0 by arg z. and write

l:) = arg z = Arg z + 2rrn = 00 + 2rr11 (8)


X
forn=O. ±1. ±2.
We can illuslrate 1he restriction O .::5: 0 < 2n for O(z) = Arg z. graphically as
shown in Figure 4.14. The two closely spaced lines depict a branch cur. Suppose
lhal fJ = 0 on t.be upper line. Then, as f) increases in a counterclockwise direction.
it reaches the lower line, where 0 approaches, but does not equal, 2rr. Theta must
Figure 4.14
cross lhe lower line in order to equal 2rr. which it does along the upper line. The An illusrrnrin [I ,of a branch cut for rh~
significance of Lhc branch cut is lo emphasize that Lhe function 0(.z.) changes as function 0(::.), - aF£ :,.
lhe branch cut is crossed. For example, as long as O ~ 0 < 2rr. then O(z) = Arg z.
However, according to Equation 8. if 0 s1ans al some value 00 = Arg: (where
0 :'5 0 < 2rr) and incrca."es by 2rr. and consequently, crosses 1he branch cul in
Figure 4.14, then

0 = arg ~ = Arg.: + 2rr


is no longer equal 10 Arg z_ The brnoch cut serves to specify the function 0(z).
Whenever 0 crosse s the branch cut, the value of II in Equation 8 changes and 0(z)
changes to a new branch. We depict the bmnch cut as two closely spaced lines
for pict0rial reasons, but in reality i1 is one line, and is somc1i.mes depicted as one
beavy Ii ne. Th'e upper Ii ne in Figure 4.14 corresponds to the top part of the branch
cut and the lower \,ine corresponds to the bottom part of t.hL'. branch cut. ln either
pictorial rcpre.scntation, whenever the branch cut is crossctl. i:J(:-.) changes to a new
branch.
The 1iII le circle around I.he origin in Figure 4.14 reflect;;; the fact that arg z is
undefined al z = 0. and serves to exclude 1he origin. The origin in 1his case is called
a branch point. We shall have more to say abour branch cuts and branch poinL"- in
Section 6.
The multiple-va1ued narure of arg z is due 10 the fact that 1an- 1(y/x) is a
multiple-vaJued funclion. You also have to be careful in computing Lhe value
of 0 from 0 = lan- 1(y/x) because il can give two different values of 0 even if
0 _s fJ < 2rr. For example. in Example I we chose 0 = Jrr /4 because Figure 4.8

C gl
172 Ch,:ipter 4 / Complex Numbers and Complex Functions

showed that z = - I + i lies in the second quadrant. However. tan- 1(-1) also gives
0 = 7rr /4, which would be the correct value of 0 for z = I - i. This ambiguiry
causes no problem if you keep in mind just where the poini z lies in the complex
plane.

Example 2:
De1crmine lhe polar representations of z 1 = I+ i and z. 2 = -I - i.

SOLUTION: In both cases, r = ./2. The point;:= I+ i lies in the first


quadrant. so lan- 1( I) = rr /4. and

The point z. = -1 - i lies in lhe third quadrant. so tan- 1( I) = 5}T' /4. and

Both z1 and z2 have the same value of y /:c.

Multiplying and d.ivid.ing complex numbers is easy in polar fom1 :

Z1 = ~e''''i -b';?J
z2 r2

For example. the product of z 1 and z2 in Example 2 is z,z 2 = 2e 6Tti/ 4 = 2eJ,7 i/ 2 =


-2i and their rat.io is z1/ z2 = l' .,-,, = -1.
We can aJso use Euler's fomrnla to derive I.he formulas (Problem 4)

e' - e-;o e;r1 + e-iO


sin0= - - - - cos0= - -- - (9)
2i 2

These formula<i are often usefuJ when eval uaLi ng in1egrals invo\ ving sin x or cos x.

Example 3:
Evaluate

(er> 0)

by using Equation 9.
4.3 Euler's Formula and rhe Polar Fonn of omplex Numbers 173

SOLUTION:

I= -
2i
11 o

I ( I I ) 2i I
= 2i a - i - a+ i = 2i(c/! + I) = c/1- + I

We can evaluate / in Example 3 anolher way. Because e;' = cos 1 + i sin , .


we can write / as

,= roc e - at sin, dt = Im
k
1~ u
e 1" , >I dt

=Im ( -I- ) = -
. I-
a- i a2 + I

This procedure gives us

100 . (- -.
1 0
00
e-"' cos, dt = Re
o
e -ta - 1), dt = Re
a-1
1 ) = -.,--
a
c:r+I

as a by-pro<lucL
Equarion 7 aJso gives us a fomrnla known as de Moivre'sfomwla. Tak_ing the
11th power of both sides of Equation 7 gives

or

(cos fJ + i sin&)" = cos nt) + i sin 110 (I I)

We can use de Moivrc·s formula 10 derive multiple angle formulas for the
trigonometric functions. Lening n = 2 in Equation 11 gives

y
cos 1 8 - sin 2 0 + 2i cos 0 sin 0 = cos 2fJ + i sin W
Equating the real and imaginary pans of this relation give.(;

cos 28 = cos 2 f) - sin 2 0 sin 20 = 2 cos 0 sin 13 ( 12)

Of course, we can use Equation 11 to derive fonnulas for cos 30 and sin 30, and
so on (Problem 7).
X
Equation 7 is also useful for evaluating powers of complex numbers. For
example. consider ( I + i)\ In this case. r = .fi and 00 = rr /4. so ( I + i )1 =
Figure 4.15
23l 1 e311 il 4 _ Figure 4.15 shows z. and z-' plotted in the complex plane. Nole Lhat The relation between z and :. < in 1hc
the magnitude of z is cubed and ilS principaJ argument is multiplied by 3. complex plane for :: = 1 •· i.

gl
174 Chapler 4 / Complex Numbers Jnd Comph·x Func1ions

hample 4:
Evaluate.;:= (I - i) 6 .

SOLUTION: Fir,t -.cc that,-= ./2 and that fJ = 7rr/4. Thus.

= 8 ( cos ,r + ,.sin
. ,r) = . 81
2 2

The equation x 2 = 1 has two distinct roots. x =±I. The cquaLion x"' = \ has
N distinct roots. called the Nth roots of unity. As we shall see. some of these root)s
• y
\Um out w be complex. so we·11 write the equatfon as :N = I. Now let z. = n.i 11
to
obLain ,-N e 1NI-! = I. The value of r is unity and so we have
elNO = cos N0 + i sin N0 = I
This equation says that cos N0 = I and sin N0 = 0: this is so only if N0 = 2rrn.
X
where 11 = 0. I. 2. . . . . N - I. (If n ~ N. cos N0 and sin N0 just repeal

_ _) themselves.) Thus we see that the Nth roo1s of unity are given by

tu = e l Jr /n / N " = 0. I. 2, ... , N - I

For example. if N = 2. we obtain


Figure 4.16
0
The lhrcc cube rooL" ofuni1y ploncd in I.he z0 = e =1 and
complex plane.
If N = 3. we get

.: 1
.., •11
= l'_n 2rr
, · = cos -
. 2rr
+ism - = --I + iJJ
-
3 3 2 2
and

4JT . JJ
Z2 =e
4.T; , J
=COS - +,. sin
. 4rr I
- =-- - , -
3 3 2 2
Figure 4.16 shows these Lhree roots plolled in 1he complex plane. No1c tha1

JI
X
all of them lie on a unit circle centcrcd al the origin because they all have un.i1
modulus. One root (z = I) lies along the x axis and 1hc other two .ire symmcl-
rically distributed about the origin. Figure 4.17 shows the four roots of .:4 = I
(Problem I0). GeneraJly the Nth roots of unity lie on the unit circle centered al
the origin; one of the poims lies along the x axis; and the others are symmetrically
Figure 4.17
dist.ribu1ed about the origin.
TI1c four 4th rooL~ of unity plo11cd in 1hc We can also find 1hc Nth roots of any complex number. We simply express z
complex plane. in polar form. and then take 1hc Nth root as we did c1bovc. In this case. we start
4.J Eul-•r's Fom1ul;i and the Polar Form o( ompl Number 1 75

with

n =0. I. 2, ... ( 14)

where 00 = Arg :. the principal argument of z. The following Example shows how
to find the 1hree cube root.s of:;: = I + i.

Example 5:
Find the three cube roots of : = 1 + i.
y
SOLUTION: First wrile : in polar fonn. Arg :- = ;r 14. and so
:: = I +; = 2 / W11 2:r n) = ,.fj_ / (n/ 4 lT nJ

Now lake 1he cube ro()t:

-1

The argument of : 1/3 will lie in the interval O.:::: (I < 2JT if 11 = 0. I. and 2.
171c re fore .

These three roo1s urc ~hown in Figure 4.18. They are symmetrically figure 4.18
distributed on a circle of radius 2 116• but none of them is direc1ed along The three cube: roor~ l)f ;:, · · I + i plo11cd
in the complex pl ane. The r:idiu., of the
the positive x axis. as in 1hc c.1.-.c of the Nth roots of unity.
cil'C"lt in 1hi~ case is 2 1/t._

4. 3 Problems
I. Determiner and 1he princip,·11 ,u-gumenl for

(a) 6i (b) 4- Ji; (C) - I - 2i (d) rr + ei


2. Exprcs~ the folllowi ng polar forms in cancsian form:

3. TI1is probkm gives a simple derivation of Eutt:r·s fonnula . Start with J(O) = cos n !· i sin 0. Differen1ia1e
=
with respect 10 0 and ~how that J'(l1) i.f (0) . Now inregrate to gel Euler's formub.
4. Derive Et1ua1ions 9.

5. Evalua1L· 1:x, ,,-"' c.:os r Jr (a > 0) using Euler's formula .

6. Use Euler's fom1ula 10 show rhat (Hand III are integers) 1~'
0
7

sin nx sin mx dx = 1
11
2
~ cos n.r cos mx dx =
£,~" ,in ".,cos m., dx = 0 if m ,' " .

qh al
176 Chap1cr 4 / Complex Numlx!rs ;ind Complex I-um lions

7. Use Equation I I to derive fonnulos for cos 38 and sin 38 .


8
1
8. EvaJuate ( - i) .
I+ i
9. EvaJuate(2-i) 10.
10. Find the four 4th roots of unity: in 01her words, find 1he four ~olulions to z-1 - I = 0.
11. Find the six 6th root" of z6 = 64 . Plot 1he six roots in the complex plane.
J2. Find 1hc four 4th roots of i.
13. Find the three cube roots of J3 - i.

J4. Starting with et= f"r cosy+ ie:r sin y = u(.r, y) + i v(.x. y), show thai thl! two fomiliei; of curves 11(.x. y) = u 0
and v(x. y) = v0 are onhogona l.

15. Use Equal.ion 2 to show lhar e --1+:2 = ezie.:z_


16. Show lha1 1he region -co< x < oo. 0 ~ y ::'.:: a in the z•plane maps in10 the upper haJf plane in the w-planc
under the trn.nsformatjon w = e' 1 z/n.

17. Staning with f 00 e-<h dx


2
= fi. lei a= (1 - i)/Ji 10 show that
lo 2a

lo1 00 2
cos x dx i
= lor-=• sin x 2 dx = ( ) 112

4.4 Trigonometric and Hyperbolic Functions


In the previous sec1ion. we derived the formulas

e;" _ e-ix
sinx = - - -- (I)
2i
and
eit + e- i.,
cosx= - - - - (2)
2
where x is a reaJ number. Equations I and 2 suggcsr rhat we define 1he sine and
cosine of complex numbers by

ei :. - e- i:
sinz.=-- - -
2;
and
e'i + e-iz
cosz.=---- (4)
2
We can show that these defini1ions are consistent with all OUI standard fomrnlac;
for the 1rigonome1ric functions. For example, Equal.ions 3 and 4 say that

sin(-::)= - sin z
4.4 Trigonoinl'tric and Hyperbolic Functions 177

and that
cos(-z) = cos z
Furt.hcnnore.

e2;~ + 2 + e-2i;; _ e2i~ +2 _ e-2;~


=----------=I
4

hample 1:
Use Equations 3 and 4 10 show th,H sin 2z = 2 sin z cos z..
SOLUTION:

. (ei: - e-lz.)(elz. + e - i~)


2sm ;: cos z = - - - - - -- -
2i
e2.i! - ,. -2.i:
= - - - - = s i n 2.::
2i

We can extend the definitions of the byperbolic functions to complex variables


by defining

(.5)

(6)

Once again. 1hese definitions reduce Lo those in Section I. I for 1he hyperbolic
functions of real variables. Just as Equations 3 and 4 sali-"fy all the Lrigonometric
identities for real variables. Equations 5 and 6 saiisfy all the iden1i1ies involving
hyperbolic functions. For example,

(e= - e-zf2
----=I
4

and

. e 2: - e- 2: (e' + e-z.) (e= - e-=)


smh 2z = ----
2
= 2- -2- - - -2--
= 2 cosh z sinh z.
There is a close relationship between the trigonometric and hyperbolic func-
tions when we define I.hem in terms of complex variables. If we substitute i z for z.
178 Lh,1ph•r 4 / Complex Numlwr, .111rl ( 1Jr11pl1"< Function~

in Equations 3 and 4, we obtain

. . e - e . e; e
sin ,z: = - - - =, - - - = i sinh z: (7)
2i 2
and
c-~ + e~
cos i;: = - - - = cosh::. (8)
2

Funhennorc. if we subst..itule i;: for;: in the right-hand sides of Equations 5 and 6,


we have

sinh i z = i sin .: (9)


and

cosh i-z. =cos::.

Equation 7 shows 1ha1 if z. is real. then

sin ix = i sinh x ( 11)

Thus. al1hough sin x is ourordinary sine function along Lhe re a.I axis, in the complex
plane it behavei; as sinh x along the imaginary axis. To see how sin.: behaves over
1he entire complex plane. consider

,/rx i ·) _ e-iCr+iy) ei.r-y _ e - ,' y


sin z = sin(x + iy) = - - - -- - - - = - - - - - -
2i 2i

=~ [e--''(cos x + i sin x) - ,·1 (cos x - i sin .x)]

= sin x cosh y + i cosx sinh y


We can also derive this result by using the 1rigonomc1ric formula for sin(a + /3):
sin z = sin(x + iy) = sin x cos iy + cos x sin iy
( 14)
= sin x cosh y + i cos x sinh y
When)'= 0. sin;:= sin x. and when x = 0. sin:= i sinh y.

hample 2:
Derive an expression for the real and imaginary parts of cos:..

SOLUTION: Use cos(a + fi) = cos er cos fi - sin a sin /j 10 write


4 .4 Trigonornclri , nrl Hyperbolic Fune lions 179

cos;: = cos(.r + iy) = cos .r cos iy - sin x sin iy


( 15)
= cos x co.;h y - i sin x sinh y

When y = 0. we have cos.:= cos x and when x = 0. we have cos:= cosh y


in &1uation 15. Figures 4.19 and 4. 20 show Lhe real and imaginary parts of sin :
plot1ed in the complex plane.

Example 3:
Evaluate cos(,r - i).

SOL u TION: u~ Equation 15 with x =n and y = -1:


cos(:r - i) = cos :r cosh(-1) - ; sin rr sinh(-1) X

= - cosh(-1) = - rnsh I y

Note tha1 Im (cos.:)= 0 along the venic:.il lines that are integer multiples Figure 4.19
of iT. The real pan of s.111 ;; ploned against x and
y. Note the pcrio<licily in the x din:ct.ion
and the cxponemial growth in the y
direction.

In Sectjon 2. we viewed w = J (:) as a mapping from a region of the :-plane


to a region of the w-plane. Let's see how the domain -1f /2 .:s x .:s rr /2. 0 ~ y < oo
=
maps into the w-plane when w f (z) =cos.:. The domain in the .::-plane is shown
in Figure 4.21. For w = /(':.) = rns ;,,

11(x. y) = co~ x c.:osh y v(x. y) = -- sin x sinh y

Along the venical line at x = TT /2 in the :-plane.

t>( ! . y) = - sinh y O.:::y<X·

Therefore, 1he venical line at x = TT /2 maps into the negative u axis in Lhe w-plane.
Along the venical line at .r = - i t /2 in the :::-plane,

11(-1,Y)=0 v(-f. y) = sinh y 05y<oo

and so the ver1i1.:al line at x = -JT /2 map1- into the positive L' axis. Now. lei's
Figure 4.20
look at poin1s within 1he domain. Because O 5 cos x 5 I for -rr /2 5 x 5 rr /2 The imaginary pan of ~in::. plorted again.,1
and <.:osh y :;;: I for al\ values or y. we~ 1hat 11(x. y):::: 0 everywhere within the x ~ncl y. Note the pcriodici1y in the x
domain. and so 1he domain in Figure 4.21 gels mapped into the entire right-h.ilf direction and the exponen1i.il growth in
the _\' direction.
plane in the lJJ-plane.
180 Ch;ipler 4 / Complex Numbers and Complex Funclions

I V

1-\' = cosz
((

Figure 4.21
The mapping of !hi.: region
-rr /2 ::S: .r ::S: ;r /2. 0 .:::: y < oo in
= =
lhe z-plane into w /(z.) cos z in the -,r/2 ,rf2 X

w-planc.

4.4 Problem
1. u~e Equations 3 and 4 10 show that cos 2z = 2 cos2 z - I.
2. Use Equations 3 and 4 to show that cos(z I + z2) = cos z 1 cos 22 - sin z. 1 sin z2.

3. Express cosh z in the form u(x, y) + i v(x. y).

4. Express sinh z in the form 1.1 (x, y) + i v(x, y).


S. Use Equations 5 and 6 to show that cosh 2z = cosh 2 z + sinh 2 z.
z211+1 ~ .,.2n
6. Use Equations 5 and 6 to show Lhat sinh ~ = L ----
+ !)!
n=O (2,1
and cosh.: = L -'"-.
(2n)! nc.cO

.J J.5
7. Use Equations 5 and 6 to show that tanh ~ =z- ~ +:::.... + O(z 7).
3 15
8. Show that sinh (z + 2rr i) = sinh z and that cosh(z + 2rr i) = cosh z. Interpret this resuh.
9. Evaluate cos( f - 2i).

10. Evaluate sinh (2 + i rr).


1J. Show that

(a) tanh u = -i UUl i11 (b) coth 11 = i cot iu (c) cos iu = cash u (d) sinh iu = i sin u
12. Show that

(a) sech i 11 = sec u (b) coth iu = -i cot u (c) sech u = sec i 11 (d) sin iu = i sinh 11
13. Map the region -rr /2 :'.: x :5 7r /2, 0 :'.: y < oo in the z-plane into the w-planc under the transformation
w = sin z.
14. Mop the region O:: ;c :::: rr /2, 0,:::: y < oo in the z-plane into the region in the w-plane under the transformation
w=sinz.
15. Showthar e": = (cosh t + sinh z) = cosh n::: + sinh nz. and 1hate-, = (cosh z -
11 1
~ sinh z)" =cosh nz. - sinh nz.

16. Use the relation in the previous problem lo show that sinh 2u = 2 sinh 11 cmh 11 and that cosh 2u =
2 cosh 2 u - I = I + 2 sinh 2 u.
4.5 The Logarilhms ol Complex Numbers 181

17. Show that the region O::: x < oo, 0.::: y ~ a in 1he z-plane maps into the upper half plane in rhe w-plane under
rhc rranfonnation w = cosh 1T z./a.
18. Show tha1 the region O _ x ~ a. 0 ;:s y < in rhc ,-plane maps into the first quad.rant in the w-plane under
=
the transformation w sin rr z./2.n.
19. Use !he defining expression for Bernoulli numbers (Equarions 3. 7.7 with x = 0) 10 show that
cot 11 = L~ - (-1)"22n
-- 8 21 ,u
2n-1
.
11=0 c2n)!
20. Show that the family of curves u(x. y) =u 0 and v(x, y) = 1·0 as ...ociatcd with J (z.) = ms z arc 011hogonaJ .
21. Use a CAS to produce figures like Figures 19 and 20 for cos:.

4.5 The Logarithm of Complex Numbers

We define I.he logarithm of a complex variable as the inverse of the exponential


function e:. and write

w =In: (I)

We have the restriction 1ha1 z. I, 0 because eu• cannot equal zero for any finite vaJue
of w. If we letz = rt! 8 and w = 11 + iv, then we find that

11 = In r and v=B (2)

where In r is the usual natural logarithm of a real number. But ,:''' = ei 0o+'lm,i
where 00 is lhe principal argument of.: (0 .::: 00 < 271') and 11 == O. ±I. ±2. . . . .
So we rewrite v as

u = 00 + 2rrn JI= 0, ±1. ±2 .... (3)

and

In :. = In r + i (00 + 21m) 11 = 0. ± I. ±2 .... (4)

Equation 4 shows lha1 In z has a diffcrenr value for each value of n. or lhal In z. has
an in Ii ni le nu mbcr of values. We can also v..'tite Equation 4 in terms of u (x, y) and
u(x. y):

In z. = ln(x 2 + ~•.:'2) 112 +; 1.an· 1 t (5)


X

where tan- 1(y/x) =00 + 2JTn when n = 0, ± I, ±2 .....


182

hample 1:
Determine rhe value of In (3 + i jj) _
SOL u TIO N: First express 3 + i J3 in polar fom1:

11=0.±1.±2, ...

and $0

In (3+ J3i) = i In 12 +i (~ + 11111)


i rr ,.., -, ]3,r i
= 1.242 ... + -. l...c4_ ... + --.
6 6
., I IJTi 25rri
1
1,_4_ ... - - 6- , 1.242 ...
6

and so on.

Before 1he devclopmenl of func1ions of a complex variable, mathematicians of


the 18th century were uncertain of the meaning that should be given to logarithm$
of negat.ive real numbers. To dc1crmine In(- I), we write -1 as ei::r and write

ln(-l)=ilr

a purely imaginary number. Because -1 is also equal 10 e;c, +:!:r,.) for 11 = 0.


± I. • • •. we have in gcm:ral that
ln(-l)=i(rr+2rrn)=rri, 'Jni. -JTi. (6)
X
which are all purely imaginary numbers.
Even though In z. ha, an infinite number of vaJucs, we l:an render it a single-
valued function by restricting the value of 0 . Just how we choose lo do lh.is is ralhcr
arbitrary. but there are two commonly used choice~. One of these restricts 0 to the
values O.::; 00 < 2:rr. where 0 11 is the principal argument of In :. We can ii lus1.ra1c
Figure 4.22 Lhis choice grdphically in Figure 4.22 by drawing a branch cul along the positive
The brnnch cul for In : in lhc complex x axis as we did for arg z in Figure 4.14. In this ca,e. 1he origin is a b,J.Jlch point
pl:inc thal re~lricts arF :: IO 1hc values
0 _ arg :: <. 2'.rr. The nrigia i~ al ·o cut out because In ;: is nol defined for z. = 0.
because In z: is not defined al z 0.= If the value of 0 is restricted 10 the interval I 0. 2JT ). then In z. is called the
principal value of ln ~ and is denoted Ln ;:. (Unfortunately. Lhere is no generaJ
consensus on 1he notation. Ln : and In.:. Some authors use In :: for the principaJ
value of In z. while others use Log :: and log ;:.) Thus. we write

Ln :: = In r + i Arg z. (7)
4.5 The Logarithms o f Compl x. N umbers 183

where Arg;: = 00 • 0 S 00 < 2rr. This single-valued function is called a branch of


In z. If 0 increases and crosses the brnnch cul shown in Figure 4.22, then

In z. = In r + i (OD+ 21l') = In r + i0 2,r :::: 0 < 4Jr (8)

and we have another branch of In z.. The function described by Equation 8 is one
of an infinite numher of branches of In .:. Each time A cros:-.es a branch cut. In ::
goes from one branch to another.

Example 2:
Evaluate In zi.: 2 and Ln .: 1.:~. if.: 1 = -1 - i and .: 2 = - I - i JJ/3.
SOLUTION: The polar fonns of : 1 and .: 2 are

and

\vhcrc II and k = 0. ±I. ± 2 ..... Note 1ha1 Arg :: 2 = 7,, /6 because :: 2 lies
in 1he third quadr.int. (Thi~ is an example of a case where a curcle~s Ul-e of a
hand calculator wi II give II = rr /6.) The produc1 of .: 1 and .:~ is

.:i: 2 = (8/3)1 l'.! /(19:r/ 12+2rrn -nk)

= (S/))1 /2 / (5:r / J_ 1;rm )

where 111 = 11 + k + I = 0. ±I. ±2 ..... Therefore.

Jn ;: 1;:, = -2I In -83 + i ( -5,r + 2rrm )


. 12

5Jti 29rri
= 0.4904 + - : 0.4904 + --;
12 12
' \"

0.4904 - 19:ri: 0.4904 + 53ni


12 12

and so on. For Ln z 1:: 2 , we choose only 1he principal argumcnl of In : 1;:; 2 , or
80 = 5rr / 12. and write

Ln ;: 1::->
-
= 0.4904 + -5rr12i .(

Note that we can move from one branch 10 another when we evaluate In z 1z 2 or
Ln (z 1: 2 ). Therefore. although In z 1~2 wi 11 al ways equal In ::: 1 + In : 2• Ln : 1z1 wi 11 Figure 4.23
The bran-eh cut in the compk.-.; pl;me
not equnl Ln z 1 + Ln .:: 1 unless O ~ Arg.:: 1 + Arg .:::! < 2rr. Anolhercommonly used
th:11 restricl'- arg ::: co The value,
branch cut for In z. is shown in Figure 4.23. In thi~ 1.:ase. /J is restricted to the values - JT < arg • _ .'1. The origin 1, ,11'.o cul out
-JT < 00 ~ Tf. You can use either branch cut. as long a~ you are consis1ent. because In : is nol dclincd :11 :: = 0.
184 Ch,1pter 4 / Complex Numhers ari d Complex Functions

4.5 Problems
1. Detennine In (-i) and Ln (-i).
2. Find In z. and Ln z for (a) ei1r/ 3 and (b) - I+ i ./3.
3. Find Ln I Ln ( I + i ./.3)1.
. iTr . i,r
4. Find Ln smh - and In ~tnh - .
2 2
5. Solve 1he following equations:

(a) In z =- 1 (b) Ln z:: - l

(c) In z = 0 (d) Ln z. =0
6. Does Ln .z: 1z. 2 = Ln z 1 + Ln z. 2 ifz 1 =-I+ i and z~ = -I?
7. Show that Ln z. = ,, + i11 where u = { ln(x 2 + y 2)•
and 11 = tan-l I.
X
Show 1hat the families of cuivcs
1'(X. y) = 11 0 and v(x. y) = vo arc orthogonal.

8. Show thal sin- 1 z. = -i ln(iz ± Ji - :1). J-linr: Solve (e1 ·t - e-ix)/2i = - for x.
9. Use rhe result of the previous prohlem to solve 1hc equation sin;::= 2. Nole that this. equation has no solution
if z is real.

10. In the previous problem. you showed that sin- 1 2 = !!.. - i ln(2 ± ../3). Now show I.hut
')

sin [ 1- i ln(2 ± v'3)] = 2. -

J1. Show that Ln;: = In r + i 00 give-; cl.n: = ,:.


12. Find all the values of In e~. where z = I + rr i.

4.6 Powers of Complex Numbers

We can use In z to evaluale general powers of complex numbers. and some of these
rcsulls may be surprising to you. Let

(I)

where both z and c may be complex. We can express., in lerrns of ln z by writing

(2)

where k = 0. ±1, ±2, .... The properties of w a_(i a complex number differ
markedly depending upon whether c is a (real) integer. a (reaJ) rational number, a
(real) irraLional number. or a complex number. so let's look at each case in tum.
4.6 Powers of Complt·x ~umbers 185

We discussed the cases where c = 11 and c = I/ 11 (n an integer) in Section 3.


but we include them here for comple1eness. If c is an inreger. then Equation 2 gives

(3)
since eiZ:rnk = I.
If c = 1/n, so Lhal we arc evalua1ing 1he nth root of:. then

k = 0, ±1. ±2. . . . (4)

In this case ;_I/" is multiple-valued. For 0 = 00 + 2rr k / 11, k lakes on the values 0,
I. 2. 3..... n - I before f::J goes through a complete cycle of 2rr. These n values
or n roots of z are disuibuted uniformly on a circle of radius r l/ 11 ccnrercd at the
origin in Lhe complex plane.

Example 1:
Find the four 41h roo1s of:'. = I +;.
SOLUTION: The polar form of;: is 2112 e1(;r / 4 • 2..-rl:J wilh k = 0. ±1. y
±2, .... so the 41h roo1s of z are z1: = 2 118eil.'l/l 6 nt/'J.l with k = 0. I, 2.
and 3. The four rooL<; are

Z1 = 2 118 (CO$.!!._+ i !-in !!.._)


16 16
X
118 9 9
Z1
-
=2 (cos "
16
+ i sin 16")

.,_ = -., 1/8 ( CO~. -17,r . . I 7rr )


- + I Sin - -
16 16

25rr
Z.J = 2 1/8 ( COS l6 + i Sl.n i6
25rr )
Figure 4.24
The four 4th roots of : =I- i ploltt!d in
rhc complc.\ plane. TI1e roiJ l u~ of lhc drdc
Figure 4.24 shows these four roots plotted in Lhe complex plane. i~, ii1ii_

Now let c be a ratiooa1 number and express it as c = m / 11. where n and m are
integers. A strnightforward application of Equation 2 gives

z."'! 11 = rm/ 11
/fm/n)tlio+'!.-rt> = r"' 1n [cos ~(0
,,
0
+ 2Trk} + i sin ~(00 + 2rrk)]
11
(5)

where k = 0. I. .... 11 - I (Problem 3). Nole !hat z"'I" is n-valued,jusl like;- 11 ".
186 Chapter 4 / Comple Numbers and Cu:nplpx Fun< rin11~

Example 2:
Evaluate (1 + i ) 213 .

SOLUTION: The polar form of z. is ;:;. = I+ i = -/2. ei(rr/ 4+'?.,,k) with


k = 0. ± I. ±2, .... and so

where k = 0. I, and 2. The three distinct values of /-1 3 are

z. 0 = 2 113 ( c:m, ~ + i sin ~)


3112 i
y = 2213 + 2"?./J
9 9
: 1 = 2 1/] ( cos : i sin : )

= -21/\
X
::,- = 2 1/3 ( cos -17:,r
6
. -1771")
+ i sin
6

TheJ.e three values are plo1tcd in Figure4.25. Nole that they are symmetrically
Figure 4.25 distributed about the origin and are the squares of the three roots that we
·n,e three vnlues of ( 1 + ; )!/-' ploued in found in Example 4.2-5.
lhl' complex plane. The radius of the circle
· 1 I' l
I.~ - .

Problem 4 has you show that z.c has an infinite number of values if c is an
irra1ional number becau$e 0 will never repeal ask = 0. ± I. and so on. Thus. an
expression such as ( I + i) ./2 has an infinite number of values

(I + i),.,~~ = v~Ji . '; 2


2 - e' '' - 1, / 4 + _,.,,: l
I.
k - 0, ±1. ±2 ....

Thus. the values of (I+ i),/2 will be densely dist..ributed in the complex plane on
the circle of radius ./2 ✓2 cen1ered i.H the origin.
Lastly. zc is aJso infini1e-valued if c i.s complex. even if a and hare integers.
Let c = t1 + i b in Equa1ion 2 to obrain

z.c = e<a . i/J) [ lnr - i( ~TkH

= {,a In r-b(fl. +2,Tk\/i b In r+o (0t_


1 1 .:.n k))

= ea lnr - b(Ou 2-Tk)(cos{ b In r + a(00 + 2rrk)I + i sin{ h In r + a(0o + 2rrk)]}(6)

rr
4,b Pnwers of ComplP:-.. 'lumhn.. 187

where k = 0. ± I. ±2 ..... Equation 6 looks rather messy, bul let's use ii to


evaluate Ji_

Example 3:
Evalua1e I'.

SOLUTION: Subs1itu1e r = 1. 00 = 0. a = 0. and h = I directly into


Equa1ion 6 to obtain

k = 0, ± I. ±2 ....

Thus. I; is nor necessarily equal 10 I; only fork = O is Ii = I. In fact, I raised


10any complex power will have an infini1e number of vaJues.

Before we finish tfas chap1cr. let's look al 1hc transformation w = ,_It~ from
Lhe z-plane to lhc w-plane. In polar form. we have w = r 1l 2eifJ / 2 . When 0 = 0.
z = r and w = r 112• As 0 increases (moving in a counlCr-clockwisc direction). z
varies as rc 11' and w varies a~ r 112eio ,- 1. When 8 bas made one complete revolution
=
in the z-plane. w = r 1l 2ei2.rr J":. = r 1l 2eirr -r l/2, and so 0 has made only one haJf
of a revolution in the w-planc. Then as 0 makes a second revolution in 1he .::-plane. X
0 goes from J'( Lo 2rr in lhe w-plane. This bchavior occurs because w = z 112 is a
double-valued funclion . We can make w single-valued by resuicring 0 i,n the z-
plane to the values O ~ 0 < 2,r. As we did in the previous section for Ln z. we
can indicate this restrict.ion on 0 by a branch cut along the positive x axis in the
z-plane (Figure 4.26). As long as 0 does nol cross the branch cul . w is a single-
Figure 4.26
valued function; ii is I.he principal branch of I.he double-valued function w =;:: l/2. The branch cul in the ;:-plane for the
If 0 crosse Lhe branch cut so that 2ir ~ 0 < 4rr. then w represents the second =
function 11 • /(z.)=- 12
/ . This bmnch cul

br.iach of w = z11-. The two branches restricts w 10 be singh!-valued.

0 ~ 0 < 2rr

and

2rr ~ fJ < 4rr

correspond to 1he two square rooLs of z.. ±z. 112. Further revolutions of fJ simply
repeat the values of w 1 and w 2 .

4.6 Problems
I. De1ermine nil lhe "alues of ( I - i ) 213.
2. Derennine all lhe values of ( I - i iJI 2 .
188 Chapter 4 / Compl 'X Nunr1hr•rc :i m1 Complex Functions

3. Argue that k = O. I, 2, .... 11 - I in Equation 5.


4. Argue 1hat lhere arc an infinite number of values of z.•· if c is an irrational number.
5. Determine all lhe values of ( I + i ); .
6. Determine al I lhe values of ( I + i) l+i.
7. Discuss 1he 1ransfom1ation w = z 1J from the .,:-plane to the w-plane. indicating the appropriate branch in lhe
1

z-plane.
8. Discuss the transformation w = z 112 using a branch cut along rhe negative .x axis in the w-plane.
9. Evaluate ;i.
JO. Evaluate 1.1.
11. Show that (i /2) 112 = (I + i )/2 .
12. Map the region Arg z. = 1r /111 (with m ~ 1/2) in the z-planc inro the w-planc under the transfonnation w = z"'.

References
R.V. Churchill. 1960. Complex Variables und Appfications. 2nd ed .• Chapter I, McGraw-
Hill
William Dunham, 1999. Euler. The Master of Us All. Chapter 5. The Mathematical
Association of America
Richard Fcynman, Robcn B. Leighton. and Mathew Sands. 1963. The Fey11ma11 UcrureJ·
011 Phy.l'in, Vol. I, Chapter 22, Addison-Wesley

Paul J. Nahin, 1998. A11 lmng inary Tale: nu: Srory of .J="i". Princelon University Press
Murray Spiegel, 1964. Schaum ':r 0111/ine in Complex Variables, Chapter I, McGraw-Hill
David Wunsch, 1994, Complex Variables with Applicutivm. Chuplt.-r I. Addison-Wesley
The Geometric Interpretation of the Complex Numbers

Before the 19Lll century, imaginary and complex numbers posed a problem for ma1hcmaticans, air.hough
they arose naturally in solving algebraic equations and proved useful in a number of 1:1pplications. The
in1erpretat.ion of complex numbers was very elusive. but at the end or the I 81h cen1ury. two obscure men
independently offered a bcautjf ul geometric in1erpret..arion .
The lirs1 to do so was Caspar Wessel ( 1745-1818). a surveyor. He was born in Norway. but he and his
1wo older brother::. allended Lile University of Copenhagt'n because there were no Norwegian univcr!-.ilic,
at lhis time. Alkr one year. Wessel left the Univcr-,ity for financial reasons 10 work as an a,sis1an1 to his
brother. who \\'a, a surveyor with the Royal Danish Academy. After 15 years working as a surveyor on Lile
topographical survey of Denmark. he received ~abbatical leave with full pay 10 complete his law degree.
Aflcr obU"lining his law degree. he retllJlled 10 hjs surveyor position. Wessel developed his ma1hcmaLical
skills in order to solve some of the difficulr problem." in geographical surveying. By 1796. he had produced
the first accurate map of Denmark. ln l 797. We ._cl presented his only rnathernmical paper 10 1he Royal
Danish Academy, in which he described the geometric interpretation of complex. numbers and 1hc addition
and multiplication or vectors as we know lhc.m 1oday. Unfortunately. rhe significance of his work wa, not
recog.nized. and the paper was not translated from Darush for ll wider distribution. Wessel\ work wa~
unknown until it was discovered in 1895.
The second one was Jcnn-Robert Argand ( 1768-1822). an accountant nnd bookkeeper in Paris. Very
little i.-. known about his background und eda·ca1,ion. How he came to be interested in l:Omplc:< numbers is
al~o unknown. Argand first published his ge.omcLric in1erpre1a1ion of complex numbers in 1806 in a book
thnt he had printed al his own expense. Surprisingly. he did not even put his name on it. Hjs work became
known in a ra1her strange manner. Legendre received a copy of the book and wrote favorably about it to
Fr.in<;ois Fran~ai~. n follow ma1hematican. When Fran~rus died. his bro1her. Jacques. also a mathematician.
discovered Legendre's letter and the book. In 1813, he published a paper based on A~arid'5 idea<; and asked
1.hr1t !he author of lhc book come forward to receive the recognition that was his due. Argand responded 10
Frnn~ais's request and submincd a refined version of his work.
ln 1831. Gauss publ.ished a geometric interpretation of complex. numbers leading to a gl'n~ral accepLance
of what is nO\V known as an Argnnd diagram. lnlerestingly. Gauss was also involved in survey work in some
of the ~ame regions Lhar Wl·~,cl surveyed.

189
CHAPTER 5
Vectors

Many quanti1ics in the physical scicnc:cs arc wctors because they ha\'e l')()lh a
magnitude and a direction DS!-.ociatcd wi1h 1hcm. Example arc ve\oci1y, force.
angular momentum. and the electric or magnc1ic field at some point. Contrast
1he.'-C 4uan1i1ic~ 10 1cmpcra1ure. dcnsi1y. or lime. which have magnitude only. and
arc called sot/(lrs .
We begin 1hc chapter with a discu~sion of vec1ors in 1wo dimensions bccau~
their properties arc easy 10 visualize and 1he results arc readily extended 10 three
(or more) dimensions . In Section 2, we discuss vectors that arc functions of a
single variable. such as Lime. Wc' I l show how a \'ecror function of time describes
the Lrajectory or a particle, and then show how its acceleration can he rc.~)lvcd
inro components tha1 arc tangential and perpendicular 10 it~ trajectory. We discuss
vectors in three dimensions in Section 3 1:1nd three-dimensional vector functions
in Section 4. wilh a number of applica1ions to cla-;sical mechanics. In lhc final
section. Section 5, we apply these vector methods to the propcnics of lines and
planes in three dimensions. Scctjon 5 nicely illustrates how simple and powerful
vecLor mcrhods arc in solvin~ f!COmer.rical and physical prohlcms. We'll solve a
number of problems in this !>CClion lhat wou Id be much more tedious wi1hout vector
methods .

.1 ectors in Two Dimensions


l\vo-di rncnsional vectors can be dcfi ned as ordered pairs of real numbers (11. b) thar
obey certain algebraic rules Lhal we shall develop below. The numbers a and I> are
c:alled compo11en1s of lhe vector. The vector (n, IJ) can be represented geomel'rically
by a directed line segment (arrow) frum the origin of a coordinate system 10 the
point (a. h). The length of rhe arrow represents I.he magni1ude of the vector und
the dinxtion of the arrow represents I.he direction of the vector. Vectors 1ha1 have
the sarne length and the sume direction ~•re equal. Thus. all 1hc vec1on. shown in Figure S.1
All the h'~lt•n- in thi., figure arc c:qual
Figure 5.1 arc equal. It makes no difference where the tail of 1hc vector is located.
lx-cou~l." 1hcy have 1hc same length and
although we often locate it at the origin of a coordinate system for convenience. ~1111c dircl."tion.
Vectors are usually deno1ed by bold face type and scalars by i1alic type. The
magni1ude (or length) of a veclOr is denoted hy r, = !al. Two vectors, u = (u 1.11 1 ) 191
192 Chapter 5 I Vectors

= = =
and v (v 1, u2) are equal ir and only if 11 1 t.• 1 and u 2 v2. Geometrically, this
u means that the two vectors have the same magnilude and the snme direction. but

/ can be located anywhere, as in Figure 5.1.


We can multiply a vector v by a s.calar c. the result being

(1)

Geometrically, the mngnitude ofv is changed by this operation. If c > 0. the length
Figure 5.2 of,. is scaled by a factor of c; if c < 0. the direction of v is reversed as well
The vector -u points in the opposi1e (Figure 5.2). If c = 0, we have what we call the zero 1·ector. 0 = (0, 0).
direction of u. All the vcc1or.i; pointing
The addition of two vectors is defined by the relation
downward in the figure are equal 10 -u .

(2)

In other words, we add vectors by adding their components. The addition of two
vectors has the simple geometric interpretation illustrated in Figure 5.3. Place the
two vectors rail-to-head and then draw the resultant vector as shown in the figure.
This procedure is sometimes called the parallelogram law of vec1or addition. Ei-
ther Equation 2 or the para llclogram law shows that vector addition is commu1a1ive.
The subtraction of two vectors is defined by

(3)

Figure 5.3 The direction of u - v is shown in Figure 5.4. Nole that it is directed from the tip
An illustration of I.he parn.Jlclogram law or
vector addition .
=
of v to 1he tip of u. This can be seen mos1 easily by writing w u - v, in which
case u = v + w. as shown in Figure 5.4.
Consider the two-dimensional cartesian coordina1e system shown in Fig-
ure 5.5. The two vectors i = (I, 0) and j = (0, I) have unit length (they are unit
vectors) and point along the x and _v axes, respectively. Every two-dimensional
vector can be expressed as an additive combination of i and j: in 01her words, any

Figure 5.4
An illu.,tration of the subrracrion of two Figure 5.5
wc1ors. Note 1ha1 w == u - v poin1s from The unit vcetors of a two-dimensional
I.he tip of v lo the tip of u. Cartesian coordinate system.
5.1 Vettor:,. in Two Dime11~io11~ 193
vector u is given by

Equation 4 represents the resolution of u inlo n horizontal component and a vertical


component. Because t1 1 and u 2 are Lhe components or
u in the x and y directions,
=
respeclively, u is usually written as u 11.ci + 11 _,. j. The length of u, or t.he 11onn
of u. in this representation is given by

(5)

Figure 5.6 shows the vectors u, v, and u - v, where u and v are perpendicular Figure 5 .6
10 each other. Pythagoras tells us that
The \'CCtors u. v. and u - v, "'here u and
v are perpendicular 10 c.ich other.

or that (from Equation 5)

or that

(6)

Equation 6 gives us the condition 1ha1 must hold if rwo vectors are perpendicular
to each other. We can define the dot product. an inner product. or a scolar product
of two vectors by

U · ,, :;;;;; U.xU.r + 1'yVy (7)

Equation 7 shows 1hat

(8)

and Lhal

(9)

Thus. 1.hc doc product is a commutalive operntion. Equation 6 implies lha1

if U J_ V (10)

Equalion I O shows that the vector u = 0 is perpendicular to all vectors. Vectors


thaL are perpendicular to eacb other are said to be orthogonal. Thus. the condition
u · v = 0 implies 1ha1 u and v are orthogonaJ. Note that u • v = 0 does not nec-
essarily imply that either u = 0 or v = 0. For example, u · v = 0 if u = i + j and
v=i -j.
194 Chapter / V ror~

Example 1:
Show that the two vectors u = (3. 4) and,,. = (4. -3) are orthogonal, Draw
( 3. 4) these two vectors in a ca.rtesian coordinate sy. rem.
SOLUTION: The dot product of u and ,. is equal to

U · V = (3)(4) + (4)(-)) = Q
and so the two vectors are orthogonal . Figure 5.7 shows these two vectors.

( 4, -3)

Example 2:
Figure 5.7 Prove that the line from the apex of :m isosceles triangle that bisects its base
The two vectors (3.4) :incl (4.-J). is perpendicular to the base,

SOLUTION: Figure 5,8 shows that

I .
m+-(u-vJ=u
2
I
m- -(u-v)=v
2

or that m = ~ (u + v). The ba-.~ is represented by u - v, and

m · (u - v) = -;I- (u + v) • (u - v)
L.

u - \'

Figure 5.8
since lul = lvl . Thus, we see thai m is pcrpcndiculur 10 the base of the
A pic1orbl aid to Example 2. triangle.

We can use the law of cosines 10 derive another expression for lhc dot product
of two vectors. Refer back 10 Figure 5.6 and write Equation 9 as

ju - vl 2 = (u - v) • (u - v) =11~ ""'!'" 1·~ - 2u • v ( I l)

Now the law of cosines says that

lu - vj.! = ,,~ + 1i - 2,rn cos f) ( 12)

where(} is 1he angle between u and v. and where u and v ar~ arranged ta.il-10-1.a.il
ns in Figure 5.6. Comparing Equations I I and I 2. we see thal

u · \' = lullvl cos0 ( 13 l


5. 1 Vt'< I r,;. in Two Dimf•nsion 195

Equation 13 is a slandard definition of the dot produc1 of two vectors. Either


Equation 7 or 13 shows that the dot product of two vec1ors is a scnlar quantily.
SomeLimes the dot product is called lhc scalar pmd11r1 because of this. Of cour~e.
Equations 7 and 13 are equivalent 10 each other.

Example 3:
I
Given 1ha1 u =I+ 2 j and w = 2 i + J. dc1cm1inc the angle <Pin Figure 5.9. I

SOLUTION: l11c angle fJ in Equal ion 13 i~ rhc angle bcr wcen u and v
when they arc arranged tail-to-tail. Thus. the angle 0. given by

cos0 =~
UL'

is equal to 1r - <j,. as shown in Figure 5.9. Using v =w - u =I - j in


Equ:11ion IJ. we have

or 0 = 108.4°
Figure 5.9
1l1c. gc1'lfl1(;tf}' :i.s~oci:itcd with brnmplc 3.
or <I>= 71.6°.

Equation 13 gives lhe following for the unit vectors i and J:

2
i · i = Ii I cos O = I =j · j
and
.I · J. = J. · I = 1·11 1·1
J COS -rr =O
2
so

in agreement with Equation 7. It's fairly easy (Problem 15) 10 show 1hal the dol
product sarislics the dist.ribu1ivc law:

U · (V + W) = U · V + U · W (14)

Example 4:
Show t.hal 1he vector u = a i + b j is perpendicular lo the slfaight line
expresse<.I by nx +by+ c = 0.

SOLUTION: Let (X1- , ., ) and (x2. Y1) be LWO points on the line. Then
n.r 1 + hy 1 + c = 0 and ax 2 - by2 + c = 0. Subtract these two equnrions 10
196 Ch.-1p!e1 5 / Vectors

get

But

The vector (.x2 - x I ) I + (y 1 - y 1) j is parallel 10 the slI'aight line


(Figure 5. l 0). so we see that u = a i + b j is perpendicular to it.
X

figure 5.10
If (x 1, y 1) and (.x 2 • n) are Lwo points on
the line cxpl'CSS(:() by ax + by + c = 0.
then the vector (x2 - x1) I + ()'2 - Y1) j is
coiocident with the line.

5.1 Problems
1. Let u = (2. l) and v = (- I . I) . Find u = Iu !. u = Jv . u + v. 2u - 3v. and Iu + v I.
2. Solve a(I. I)+ h{-1, 0) = (I, 0) for a and b.

3. Find the un.i1 vector in the same direction as

(a) (3. -1) (b) 21+3j (c) i+j

4. Given that bo1.h u =


2 i + Jj and v = -i + 2j si.art at 1he origin. as shown in Figure 5.11, calculate the distance
between the heads of u and v.

figure 5.11 X
The vectors lL~ in Problem 4.

5. Detem,ine the angle between the following pain: of vectors:

(a) u = ( l. 2) and v = (- I, 3) (b) u=3jandv=-l+J

6. DCLenninc the angle between the following pair of vectors:

(a) u = -I - j and v =I+ j (b) u = i + j and v = i - j

7. Find all the vectors lhat are perpendicular to u = l - J. but are twice as long as u.
8. What angles do the following vectors make with the positive ,r axis?

(a) I+ j (b) -i +J (c) -i - j


5.2 Vl'rtor Functions in Two OimPnsions 197

9. Consider 1he triangle sketched in Figure 5.12. Prove that the line joining the midpoims of 1he sides II and v is
one-half the length of Lhe third side and parallel 10 ii.

figure 5.12
A pictorial wd 10 Problem 9.

IO. Prove that the diagonals of a parallelogram bisect each other.


11. Resolve u into componenL'i. that are parallel and perpendicular lo any other nonzero vector"·
12. Find the perpendicular di~ta11cc from the poin1 (4. 3) to the line x + 2y - 4 = 0.
13. Consider a vector u = 11 1 I+ 11 2 J Ro1a1e u 90° in a countcrclockwisc direction to obtain u90 . and show 1hu1
U90 = -112 f + 1.11 j.

14. Show that u · v is 1.hc lcngIh of the projc{:tion of u on10 v time~ the lcnglh of\'.
15. Prove the distribu1ivc law geometrically for the do1 product of two vectors.

5.2 Vector Functions in Two Dimensions

The vector r == .r i + y j, directed from the origin 10 a point (x, y). is called a
position vecwr. lf the point (.r, y) moves with Lime [in other words. if .r and
y are fun et tons of time . .r(I) and y(I) 1- then r(t) = x (t) i + y(t) j is a (vector)
runcLion of lime. The function r(r _) traces out a curve in the plane as t varies. We
can denote a point on this curve by r(.r. y) = r(.r (t). y (t)) = r( 1 ). We can consider
a two-dimensional vecror funclion lo be an ordered pair of reaJ~valued funclions
(x(t), y(t)). Most of the concepts associated with real-valued functions carry over
to the case of vector func1ions. For example,

lim r(f)
1-•{I
= llm x(r) i + lim
1-ll 1-11
y(r) j (I)

and we say thal r(I) is continuous ut a if lirn r(t) = r(u) = .r (a) i + y(a J j. The
,-a
continuity of r(t) depends upon 1.he continuity of iLs components. We define the
deriva1ivc of a vector func1ion by

dr , . r(f + 6.1) - r(r)


- = r (1) = 11m (2)
t/ I .61-•0 6.t

provided this Ii mit exist-.. If r =x (I) i + y(t) j. we can find r' ( I) by diffcrentiali ng
r(r) to obtain

dr
- = r , (/) = -dx I+
_ dv j
. . .;. . (3)
d1 dt dt

al
198 Chapl r / V _o r~

(Nole that no dcriva1ives involving i and .i appear in Equation 3 because neither


r(/0 + tl.1) - r(r 0) their magnitudes nor their directions change with Lime. Their 1imc dcrivmivcs arc
I equal 10 7,cro.)
Lc1 ·s look at Ec1ua1ion 2 pictorially, as shown in Figure 5.1 J. Figure 5.13 shows
1hc vectors. r(/ 0 ) and r(r 0 + b.t), and 1he difference r(/0 + 61) - r(t 0 ). As 61 ➔ 0,
I r(r0 + tl.1) - r(,0 ) J/ D./ become~ tangent 10 the curve r( r) at r = tu and is called
lhc ta11ge11r to the curve :H 10 . The veer or r( 10 ) + r' Uo) (r - r0 ) defi ncs 1hc ru11&e111
li11e 10 1J1c curve r(t) at 10.

X Example 1:
Find the equation of 1hc langcnt line al the point r = I to thi: curve described
Figure 5.13
1
An illu~lrulion 1.tui1 r (1) i~ tangt:nt 10 1he
parametrically by x r. y 2. = =,
curve gt:ncrated by r.
SOL U 11 ON: The tangent vector nt rhe point t = 1 is given hy
, I) dx . dr . . ,., .
r( =-1+--=---J=•+-J
dt t/1
y
and the equotion of 1he Langen I line is given by

r( I) + r' ( I) (1 - I) = i + j + (i + 2 j) ( t - I) = , i + (21 - I) j

or by x = t. y = 21 - I (Figure 5. 14).

If r(I) describes lhe position of a pariiclc. then Equation 3 gives us the velocity
of the pa11icle-. The magnitude of the velocity is lhe speed of the panic le:

The Ii me deriva1ive of the velocity gives us the accelcrnt ion of the panicle:

,, d:.x d 2y • • .
a= v (t)
1
=r (I)= -
d1-1
I+ -.., J = a_, 1 + a" J
dr- ·
(5)
Figure 5.14
The pw-amcrric rnrvc · = f . y = t ~. and
the tangent line at tht: poin1 t = l.
E.xample 2:
D esnibe the motion given by

r(r) = 2 cos 1 I + ~in r j


Find the velocity. 1he spc.-cd. 1hc acc.:elcration. ::ind the magnilude of lhc
acceleration when r = 2:r :' 3.

SOLUTION: The parametric cqua1inns of the motion arc x(f) - 2 cos I


and y(I) =sin/. Thus. x~ = 4 co,~ 1. y - = s in 2 t, :ind x :! 4 y 2 = 4 . The
motion 1n1cci; out un ellipse in a t"ountcrdockwisc direction increase-:-. a., ,
5.2 Vccror Fu n ri ons in Two Dinwmion:. 199

starting at (2. 0) and completing a cycle for, = 2rr. 41r ..... Equa1ions 3
and 5 gi\•e

. . .
v = -dr = (- 2 sin,) 1 + (cos I) J
dt
11 = (4 sin 2 , + cos~ 1) 112 = (4 - 3cos 2 1) 1/ 2

a = -dv = (- 2 cos t) l - .
(sin f) J
di

Al I = 2:t /3. ,. =- (3) 11 ~ i - ( 1/2) j. D =i - (3 1 ·/2) j, v = fi°3/2. und


0 = ./1/2.
y

The distance travelled aJong the curve traced by r(,) from 10 10 t I is called
the arc le11grh of the curve. As Figure 5.15 suggests, the arc length is g·iven by
d.s 2 = dx 2 + dy 2• or (see also Equation 3.5.16)
dx
X

Figure 5.15
Nole that this equa1ion is equivalent to v = ds/dt A gc11mc1ric nid 10 1he calcularion of
rhc an: lcn~th of a curv~. Note thar
=
ds 1 Llx 2 + dy 2.

Example 3:
Calculate the arc length of rhe curve traced by r( r) = t cos 1 i + r sin , j from
I= 0 lO 2JT.

SOLUTION: Firs! detemlinc

r'(I) = (cos, -1 sin lli +(sin,+, cos 1)j


and

Therefore.

= n(I + 4rr 2 ) 112 + In ( 2:r +Ji+ 4rr 2 )


-- rr( I + 4rr) 11
1
+ -Arcs1n
I ,· h,,_,r -- ., '>563 . ..
.. I...
2
200 h,. pt r / Vectors

The velocity vector \' (1) is tangent to I.he curve generated by r(t). If we deno1e
Lhe arc length along the curve by s. 1hen the unit tangent \·cc/Or is given by

T(t) = v(r) = dr dt (7)


u(r) dt ds

where we have used 1he fact that v = ds /dt (Equation 6). We can write Equation 7
as

v(r) = v(r) T(t) = ds


-d1 T(I) (8)

The acceleration is given by

2
a= dv = Td 2s + dT ds = Td"2.s + dT (ds) (9)
dt dr 2 dt dr dt 1 ds dt

11 turns out that 1he magnitude of the vector dT / d.~ has an important geomet-
ric interpretation. To see what this interpre1a1ion is. start with T(t) - T(r) I (see =
Equation 7). It is ea.1:;y to show (Problem 3) that for two arbitrary, differentiable
vector func1ions u(I) and w(t).

d
-(u • w) = u • -dw + -du - w ( I 0)
dt dt dt

Differentiate T(t) - T(r) = I with respect tor to ob1ain


d dT
-(T(t) -T(r)) =T · - + dT
- -T
dt dr dt

=2T· dT =0 (11)
)' dr

Now. using the chain rule of diffcren1i::i1ion.

d ds d . d
-(T -T) = --(T-T) = v(f )-(T-T)
dr dr ds ds
dT
T =2u(t)T- - = 0 ( I 2)
ds
X
and .so we see that the vcc1ors T(s) and dT /ds are perpendicular to each other
Figure 5.16 (Figure 5.16).
The. \'tttor.; Ttn mid tlT /d,~ are
perpendicular 1·0 cac:h other. dT /ds
As Lhe point r =
(x. y) moves along 1he curve shown in Figure 5.16, the
mcnsurt:$ the rale of change of the magnitude of T remains cons1an1: 1hereforc. dT / d s measures the rare of change
dirccLion of T{.,) as a func1.ion of 1he arc of 1hc direction or T. If 0 is the angle that T makes wilh the x axis, then T =
leng1b .L cos 0 i + sin (} j. DifferenLiating both sides wi1h respect to .\· gives

dT . . . d0
-Lis = (- sm fJ 1 + cos 0 J) -
ds
5.2 Vector Funr 1ion~ in Two DimPn~iorh 201

Taking the ab!,olute value of both sides yields

(13) J'

The rate of change of 0 wi1h s is called the curvature K of a cuf\'e described


parame1rically by x(1) and y(r). The curvature tells us how rapidly the ctirection
of the 1angent line is changing as we move along Lhe curve. A large value of K
means that the curve is bending sharply, while a small value means thal the curve
is bending slowly. The reciprocal of K is called the radius of curvature. which
we denote by p. The radius of curvature has a nice geometric interpretation. Let a
curve haven given radius of curvarure pat rhe point (x. y). We define the osculating
X
circle 10 the curve at (x. y) to be the circle ofractius p passing through (x. y) having
the same tangent 10 the curve at (x. y) and whose centcr lies on the concave side
of the curve (Figure 5.17). If the curve is bending slowly, then the radius of the
osculating circle will be large. whereas if the curve is bending rapidly, Lhen the
radius of the osculating circle will be small. Figure 5.17
We can derive an expression for K in terms of the (nonpara.metric) equation An illusaut.ion of an oscul::uing circle.
of the cur.•e, J = y(x). Start with

.dv
. .: . . = tan 0
dx

Then

!!_ (dy) = d0 !!_ (dy) = dtan8d0


ds dx ds d() dx d0 ds

} d0
=sec-0-=(l+ian
ds
2 d0 [
&)-=I+-=-
ds
(dy)dx
2
] d0
-
ds
( 14)

But

d (d_\') dx d (dy) dx d
2
y ( 15)
ds dx - ds dx dx - ds dx 2

and (Problem 8)

dx

because
202 hapt r ,. / Vectors

FinaJly. after equating Equations 14 and 15, we see 1ha1

( 16)

We took rhe absolute value of d 2 y/dx 2 in Equation 16 because we have la.ken K


to be a positive quantity.

hample 4:
Find lhe curvature of the parabola y = x 2/2 at (a) x = 0. (b) x = I. and (c)
x = 2. and draw Lhc os.culating circle at x = I.

SOLUTION: Equation 16 gives us

I
K =----
[ 1 + :c 2t3i 2

X
=
The curvature is equal to I at x 0. 0 .354 at x =
I, and 0 .0894 ar x == 2.
The radius of curvature al x =I i.<. equal to 2.83 and the oscula1ing <.:ircle al
Figure 5.18 x = Ii$ shown in Figure 5.18.
The o:.culatlng circle c:ilcul111cd in
Ex11mplc 4.

Let's now return to Equation 9 for o:


2 2
d s- +dT
a=T - (ds
~) ( 17)
dt 2 ds dt

If we let N be a unit vector in the direction of dT /ds. then we can write

dT = IdT IN = KN (18)
ds ds

and Equatjon 17 becomes


dv ,,
ll =T- + /(lJ-N (19)
dr
Equation 19 resolves the acceleration vector into component~ 1angen1jal and per-
pendicular 10 the trajectory of the particle. The tangentiaJ component

dv
aT= -
dt
is the rate of change of I.he speed of the particle and the normal component
[JN ==: KV2

is n mc.asure of the rate of change of rhe direction of rhe mo1ion of I.he paniclc.
5.2 Vector Functions in Two Oin1Pn~1ons 203

Example 5:
The radius vector of a mass moving in a c:ircular orbit of radius R with a
unifonn angular speed of w rnL.lians per second is given by

r = (R cos. wt) I + (R sin llJI) j

Determine v. u. aT, and uN and interpret the result foraN .

SOLUTION:

V = -dr
dr
= (- Rw sm
. wr) I. + (R cos WI ) J.
(JJ

and

(This result for u also follows from differentiating the arc length s RO =
with resp<."Ct to time and using w = d0/dt.) Note that r · ,. 0 for circular =
moIion. The 1angen1iaJ and normal components of the acceleration are given
by

dti
aT= - =0
dr

and

Problem I 6 has you show 1ha1 K = I/ R for a cin:lc, so we find thal

which is calJcd centriperol an·elaatio11. If the moving particle has mas.s m,


=
then o force muN m tJ 2/ R din:cred wward the origin i~ required 10 maintain
the circular motion .

.5. 2 Problems
I. De1cnninc the equation of the IangcnI line al t =I to Lhc curve described by the parnmerric equations
x=2t 2 +1.y=t"J.+2r.
2. Determine the equa1ion of Lh~ 1angcn1 line al t = rr /4 10 Lhc curve described by lhe parametric e4uations
x = cos;\ r. y = sin~ r.
d df dw d
3. Show that -[J(t)w(r)] = -w(t) +/(I)-and that -(u · w) = -du •w + u • -
dw
.
dr dt dr dt dr dr
204 Chap! r 5 / V rors

4. We can integrate vector functions by integrating the component-:. For example. if \'(I)= x(r) i + y(I) j, I.hen

! u
b v(11) du= I fb x(u) du+ j fb y(t1) d11. Suppose thal a particle has an acceleration a(r)

and ini1iaJ values r(O)


L L
= 2 i and \'(0) = i + j. Find i1s position and velocity al any time,
= 31 I - 2, 2 j

S. Prove tha1 a(u) and da/du arc perpendicular if lal = c =constant.provided ncilhcr a nor da/d11 is equal to
z.ero.
6. Detenninc 1he arc lengrh of the curve described by r(t) = 2 cost I+ 2 sin t j from t = 0 tot= I.
?

7. Dctennine 1hc arc length of the curve described by r(t) = , I + ~ j from 1 = 0 10 r = I.


8. Show that if a curve is described nonparame1rically by y = y(x). 1hen the arc lenglh is given by
s = lbC:
II+ 1/(x)J 2 J 112 dx.

9. Find T for r(u) = (cos 2 11) i + (sin~ 11)j. Why is Ta constant vector?
10. Show 1ha1 dx/ds = [I+(:;)' rl/l
11. Dctenninc dx/ds and dy/ds on the parabola y = x 2.
12. Detennine lhe curvature of a straight line.
13. Determine Lhe curvature of I.he ellipse x 2/ 16 + y 2/9 = I al 1he poin1 (0, 3).
14. Detenninc the value of x at which y = In x has its greatest curvature.
15. The cen1er of 1he osculating circle. called the cenrer of cun.•ru11re. is givera by y =
r + p N, where r is the
position vector 10 the point (x. y) and N is a uni1 normal vectorto the curve y =
f(x; (Figure 5.19). Determine
=
the radius of curvature and I he cen te r of cu rval ure of the parabola. y 2.r 2 + I at 1he poi nl ( I. I).

Figure 5.19
X
An illustrarion of the ccnrcr of curvature.

16. Show that p = 1/K = R for a circle of radius R.


17. Find I.he tangent vector and the normaJ vector to rhe parabola y = 2r 2 + I al the point ( I. I).
18. Find I.he tangent veclor and the normaJ vector to the curve (.kscribcd by r(I) = r cos i I+ t sin t j al , = 7T.
19. Find the UIJlgentiaJ and normal components of accclemtion at r = 1/4 for a particle moving according 10
r(r) = (2 cos 2rr t) i + (sin 2,r r) j.
20. Find the tangcn1iaJ and nonnal componenLS of acce\er:uion nt r = 0 for a particle moving according 10
r(t) = , cos , I + t sin , j.
5.3 v~u "" in Three Dimensions 205

211. The para.me1ric e.quations of a cycloid are x(11) = a(u - sin 11) and y(11) = n( I - cos 11). Calculate lhc length
of one arc of a cycloid.
22. Cnlcmilale I.he area under one arc of a cycloid (see the previous problem).

5.3 Vecto1rs in Three Dimensions


Three-dimensional vectors can be treated as ordered triplets of three numbers
and obey rules very similar to Lhose obeyed by rwo-dimensional vectors. We ::
represent three-dimensional vectors by arrows and the geometric interpretaLion
of tne addition and subtraction of these vectors follows a parallelogram rule just
as it does in two d.iimensions. We define unit vectors i. j. and k along rhe x. y. and
z. axes of a cartesian coordinate system and express three-dimensional vec1ors as

u = "·· i + u _,. j +II: k (I) )'

In tem1s of ordered triplet~ of real numbers.

i = (I. 0, 0), j = (0. I. 0). and k = (0. 0. I) X

Figure 5.20 ~hows a canesian coordinate system and 1he three unit vectors I. j, and Figure 5.20
k. The coordinate sys1em shown is said 10 be a right-hnnded coordinate system The uni1 vectors of a three-dimensional
because as the x axis is rotated toward they axis through the smaller angle be1ween c.ane.--ian c,:c,ordinalc !>')'~1e m,
them (90° in this case). the positive z: axis is in the direction of advance of a right-
handed screw. A right-handed coordina1e system also obeys lhe right-hand rule:
if you curl the fin!_!ers of your right hand pointing in the customary direction of a
countcrclockv.'isc rotation in the xy-planc. 1hen your Lhumb will point in Lhe s.ame
direct..ion as the posit..ive z axis (Figure 5.21 ). It is customary 10 use right-handed
coordinate systems and we shall do so throughout 1hc book.
We can define lhe do1 product of two vccto~ by

U · Y = ,, · U = II l/ COS fJ (2)

where the length of u is given by

II = (U · U) l/2 (3)

As in the two-dimensional ca.,;;e. 0 is the angle between u and v. where u and v are
X
arranged 1ail-to-1a.il. According to Equation 2. the dot produc1s of the uni1 vectors
i. j. and k are Figure 5.21
An illustrution of the righ1-hand rule.
i-i=j-j=k·k= I
(4)
i -j=j•i=i•k=k•i=J·k=k •j=O

For u = u.x i + u_rj + 11 2 k and v = u,r i + u_\,j + V.: k. we have (Problem l)

U · V :::: 11, U,r + LI y V1. + II: V;_ (5)

m
206 Chapler 5 / V('t Ii ir~

and

(6)

Equa1ion 6 is a three-dimensional version of 1hc Pythagoras theorem. Equa1ion 5


can also serve as the definiLion of the dol product of two vectors.

z
Example 1:
Find the angle between lhe two vectors u = I + 2 j + 3 k and v = 2 i - 3 j - k.

SOLUTION: According to Equation 5.

~ U · V = (])(2) + (2)(-J) + ())(-[) = -7


y
II= (I+ 4 - 9_) 1/2 = -.,./j4'
11=(4+9+ 1i 1n=...l'i'4

0 -- 2:1 -- ]_,oa
u. V I
cos{;= - - = -- or
11v 2 J
p,,
X

(a)

Example 2:
Dewm1ine the angle:-. in the triangle fonned by t.hc lhree \•eniccs.
=
P 1 l], 2. 2), P2 = (3, I. 1). and PJ (3. 3, 3).=
SOL u TIO N: Ont' ,ide of 1he triangle is given by the vector associa1ed
with P2 - P1, or

u = (3 - 2)i + (I - 2) j + (I - 2) k = i - j - k
u
and ano1her by P_\ - P1:

v = (3 - 2) i + (3 - 2) j + 0 - 2) k = I +j + k
PI The third side is given by P_, - P2., or

W=(3-3)i+(3- I.Jj+tJ- l)k=2j+2k

Note th;H w = v - u. The triangle is pictured in Figure ~.~2a. As Figure 5.22b


show~. 1hc anpl~s between u and v (0,,,,) and hetwcen v and w (Ot...,) arc
( b )
given by Equation 2. Thus we have
Figure 5.22 u. V I
(:t) The wctnr~ u. \', and win Ex:1111plc 2. ('() " 111'= - - =- - 0/l ll = 109.47°
(b) The ,..ectors u. ,·. and win Example 2 I/ I 3
viewed normal 10 1he pbne of 1he 1rianglc U ·W 4
that they form. COS O[ U' = -ll ll'- = --
24
5.J V 1or in Three.• Dirnen ions 207
To calculate II""" no1e tha1 u • w/uw gives the complement of the interior
angle between wand u (i;ec Figure 5.22h) . Therefore.

, a .w ..i
(0.
1111 •
= --
IIW
=- - -
,J14
1
0Ill' = 144.74°
and so A,w, = 180° - 144.74 = 35.26°. Note that the sum of the 1hree angles
is 180°.

The direc1io11 rmglt'.1 of a vector u r.trc 1hc angles ex. /3. and y rhal the vecror
makes wiLh each of the coordinate axes (Figure 5.23). The cosines of the.,;.e angles
are called the direction cosim:x of u. For u = u..- i + " Y j + ll .: k . i · u = 11 cos a.
j • u = 11 cos /J. and k · u = u cos v. or Figure 5.23
The Jin.·,:tion angle~ of a vector u.
I.u k. u
= j- · U = --=-- :
II ( 11 1· II.
cos a= - - = --=--: cos fi cosy= - - =.....:. (7)
II II II II II II

Equation 7 shows that 1hc direction cosine-. are the components of a unit vector in
Lhc same dirccLion as u; or

-uII = COS CX I. + COS I'~·J + COS t' k (8) '-


Equal.ion 8 gives
n
V
cos 2 a + cos 2 f3 + cos 2 y = I (9)

We have defined 1he dot produc1 of two veclors, which is a R"aJa.r quanti1y. I
There is anorher usdul definition of the product of two vectors lhat results in a /
/ u y
vcc1or quan1ity. We de tine the cross prnduu or the vector p,vd11c1 of u and v by
I
i
u x \' = lullvl sin 0 n ( 10)

where 0 i~ tJ1c angle ( < 180°) between u and v (arranged tail-to-tail a-. before) and n
is a unit vector perpcndiculario the plane fom1cd by u and v, and thus perpendicular ,x
I
I
to both u and v. The direction of n is the same that a right-handed screw would
advance if u were rotated toward v through their smaller angle (Figure 5.24). figure 5.24
The unir vc.•cror n in the dircclion u x , ..
Because sin 0 === 0 when A = 0. Equation 10 s<1ys thnt

uxu=O ( I 1)

We also have

U XV = - V XU ( 12)

Equ.ition 11 says that the cross product of parallel vectors is equal 10 zero. Equa-
tion 12 re.suits because the direct.ion of n i:-- rcvcr.-cd if we imcrchange u and v.
208 ChJpler 5 / Vectors

1t is useful 10 apply Equations 11 and 12 to the unit vec1ors i. j. and k. (Refer


to Figure 5.20.)

ixi=jxj=kxk=O

i X j =k jxi= -k
i X k::::: -j k X i =j (13)

j X k =I kxJ= -I

The dis1ribu1ive law for the cross product of two vectors is more lengthy 10 prove
than for the dot product of two vec1ors, so we simply slate wi1hout proof (see the
references al the end of 1.he chapter) 1ha1

u X (v + w) =u X V +u X w

The cross product of u and v in terms of i. j. and k is given by

+ tlyVxj X I+ llyl\.J x j + u_,.v;j x k

+ ":'\kxl+11:v_ ,kxj+11:v:kxk 1

Can you see a pattern in 1he rcrms of Equation 14?

Example 3:
Show 1ha1 u xv given by Equatjon 14 is pcrpcndiculor 10 u.

SOLUTION: We wish to show that u • (u xv)= 0. Using Equation 14.


we get

The product v · (u x v) = 0, also. (Can you see this without carrying out the
algebra?)

Equation 14 can be rewritten concisely in 1cm1s of a dc1errrun1.uH as

j k
UXV=
".r Uy ll - (15)
l).f v_,,
5.3 V1•( !nrs in Three Oirnr.•mions 209
We haven·, studied determinants yet (we will in Chapter 9). bul you mighl rec-
ognize 1ha1 EquaLion 14 resulLs by expanding the detenninant in Equation 15 in
terms of cofactors of the first row. Recall that

j k
ll.x Uy ll z =i l".v
Vy
II ~
Vz
- 1-J, lu
U.r
V.r v_,, LJ_

Example 4:
E val ual e u x v if u = 3 i - j + 2 k and v = 2 i + 2 j - k.

SOLUTION: Equation 16 gives

j k
U XV= 3 -( 2
2 2 -I

=l~ ~1 l -I~~, IJ+I~ ~I lk


1
1

=-31+7J+8k

Note 1ha1 u x v is perpendicular 10 both u and v.

Cross products occur in a number of physical problems. For example. if a


particle is moving about a fixed center wilh momentum p, Lhen r x p. where r
is the position vector of the particle from the fixed center, is equal lo the angular
momemum of the particle about the fixed center. Similarly, if I.he pan.icle is acted
upon by a force F, rhen r x Fis the 1orque acling on 1he particle. Anolher example
is provided by the force acting on a charged particle moving i.n a magnetic field.
=
In Lh.is case. F qv x B, where q is Lhe charge on Lhe particle, v its velocity. and
8 lhe strength of 1he magnetic fidd. u
The magnitude of rhe cross product of 1wo vectors has a useful geometric
Figure 5.25
interpretation. Figure 5.25 shows a parallelogram whose sides arc I.he vcc1ors u An dlu~tialion or the geometric
and , .. The area of I his parallelogram is equal 10 the areas of lhe two triangles plus in1crpre1a1ion of u x v.
the area of !he rcc1angle. or

A= 2 (~) (u cos 0)(v sin 0) + (u sin 0)(11 - v cos 8) = 11v sin 0 (17)

Notice !hat Equal.ion 17 says lhat !he area is equal LO the length of one side times
1he height. Bui "v sin 0 is simply the magnitude of u x v, so we con write

A= lu x vl = uu sin & ( I 8)

gl
210

Equation 18 can also be used 10 caJculate Lhe area of a triangle formed by vectors
u, v, and v - u (Problem 17).
The cross product results in a vector. so now let's consider the rriple scalar
product u • (v x w). Using Equation 14. we have

This expression looks just Iike Equa1jon 14 with i, j, and k replaced by II x. 11 y. and
u!. however, sou• (v x w) can be expressed in the fom1

II X II ,I' u.
u • (v x w) = Vx u_\· U- (19)
Wx w.\· u:

u which is just Equation 15 with i. j. and k replaced by II x · 11 _,.. and 11: ·


Just as Iv x wl is 1hc area of1he paralklogrnm wi1h sides,.. and w, lu · (\' x w)I
Figure 5.26 is the volume of the paraJ\clepipcd with sides u. v, a.nd w (Figure 5.26). To sec
A parallelepiped with sides u. v. and w.
=
that this is so, note tha1 lu · (v x w) I 11 Iv x w I cos 0 and that l ,., x wl is the area
of the base of the parallclcpipe.d shown in Figure 5.26 and "cos fi is its height.

Example 5:
Show that the three vectors u =I+ j - k. v = 2 j + k, and w = 21 +4j - k
are coplanar.

SOLUTION: It suffices 10 show 1hn1 u · (v x w) = 0. for then 1hc volume


of the corre.,;ponding panillepiped will be zero.

i j k
vxw = 0 2 I = - 6 I+ 2j - 4 k
2 4 - I
and
u • (v x w) - 6 + 2 + 4 =0
We could also have used Equation I9 directly.

We summarize here some generaJ properties of vector producL,;:

U X \' = -V XU {20)

(cu) XV= u X (cv) = l'U X ,. (21)

u X (v + w) = u X V +uX w (22)

u . (v X w) = (U X V) . w (23)

The proofs of a.II these relations are str.1igh1for.vard.


5.3 Vectors: in Three Dime nsions 211

Equation 23 is called a lriple scalar product We also can have a lriple ,,eclor
producr. defined by u x (v x w ). It is straightforward. albeit somcwhal lcnglhy.
to show 1ha1 {Problem 13)

U X ( \' X W) = (U · W) Y - ( U · Y) W (24)

Notice that Equation 24 says that the triple cross product is an addirivc combination
of v and w of rhc form C1 ,, - /3 w where a = (u • w) and /J = (u • v).

5 .3 Problems
I. Use Equations ➔ to derive Equa1ion 5.
2. Find the angle between the vectors u =2 i - 2j + 3 k and v = 2 i + 3 j + k.
3. The poinr (I, - I. 2) is connected to 1hc points (2. 0. I) and (3. I. - I). Find the angle between the I ines
connecting these points.
4. Find the direct.ion angles of the vector u = i + j.
5. Lei u = (cos a 1• CO$ /3 1• co. y 1) anJ \' = (cns a1. cos /Ji. cos y2 ). Show that the angle betw<..-cn u and vis given
by cm O = co a 1 cos a 2 + cos /3 1 cos /3 2 + cos y 1 cos Y2·
6. Find 1he area of a parnllelogram with vertices (0. 0, 0). ( 1. I. I), and (2. ). 5).
7. Detem1ine the interior angle of a regular retr..ihedron. Him: See Figure 5.27.

Figure 5.27
The relation bctw~en a 1c1rahedmn and a culx.'.

8. Dctcnnine the dir<..-ction cosines of

(a) v =i - 2j + k (b) v = 2 i + 3j - k

9. Use vector methods to derive the law of sines of pl:.ine trigonometry. Him: Sia.rt with u + v + w = 0, the
condi1ion that u. v. and w form a triangle. and then take cross products.
10. Show I.hat the points P1 = (I. - I, I), P2 = (2. 0, I). P3 - (2. 2, 0), and P4 = (3. 3, 0) are cophmar.
l I. Show that the volume of a regular tetrahedron is given by ,./2£, 3/
12. where o is the length of an c<lgc, Refer
to Figure 5.27 for the geometry. Hin,: The volume of tl1e tetrahedron is 1/6 tl1e volume of the parallelepiped
formed by tl1e thr~ vectors forming lhc tetrahedron.

C pynght na al
212 Ch,1ph•r 5 / V«•t Im'-

12. Show that u • (u x v) = 0. Interpret this resulr.


13. Verify Equation 24.
14. Prove the Ctwcl,y-Scln-.·am inequoliry. lu · vj :::: lu!lvl.

IS. Use lhe resull of the previous problem to prove the triangle inequaliry, lu + vl ~ lul + IVI. Him: Use the
Cuuchy-Schwart.z inequality from Problem 14.
16. Derive a relation between the cross product of two vectors and the area of the triangle fom1ed by Ihcse vectors
and their difference.

17. Use the result that you derived in the previous problem 10 c.alculntc the area of a Irianglc with ven.iccs
P1 = (I, -1, I). P2 = (0, 2. I). and P3 = (3, I. 2).
18. Is u x (v x v) = (u x v) x v ?

5.4 Vector Functions in Three Dimensions


In Section 2. we discussed vector functions in 1wo dimensions. Many of the results
=
of that section carry over to here. The vector r x i + y j + z k, directed from the
origin 10 a poin1 (x, y. z). is called a position 1•ector. lfthe poinr (x, y. ~) moves
with time. Lhen r(t) = x (t) I + y(t) j + ;:(t) k is a vector function of time. The
function r(t) traces out a curve in space (a space curve) as t varies. We can denote
a point on this space curve by r(x. y. z) = r(.r (t). y(t). :(I)) = r(t). The velocity
of the point is given by

,•(/) = r, (t) = -dx


dt
, dy _ dz
1 + - J+ -
dr dt
k ( I)

As i.n the case of two dimensions, r' (t) = v(t) is tangent 10 the curve described by
r(t). The speed of a moving point is given by

(2)

The fonnulas fort he deriva1 ives of quantities such as J (I )u( t) and u (,) · v (r) arc
rJ1e same a') for two dimensions:

d df . du
- f(t)u(t) = -u(t) + j(l)- (3)
dt dt dt

and

d
-u(t) • v(t) = -du •v + u. dv
- (4)
dt dt dt

Fomwlas 1ha1 are new lo this section arc those involving the cross product of two
vcclors since the cross product is a three-dimensional quantity. It is easy 10 show
5.4 Vec1or Funcrrons in Three Dimensions 213

thar (Problem I)

d du dw
- U( /) X \V (I) ;:::::; - X W +U X' - (5)
dt dt dt
and that

d du d
-[u • tv x w)I = - •(\' x w) + u • -(,· x w)
dt dt dt

- . (v X w) + u . (dv
=du - X w) + u . ( dw)V X - (6)
dt dt dt

Here is a nice application of Equa1ion 5. Newton's equations of motion for a


body of (constant) mass m are, in vec1or form,

(7)

Multiply both sides of Equation 7 by rx to obtain

d1r
mrx-=rxf
2
(8)
dr

The left side of this equation can be obtained from

!!_
dr
(r dr)
x
dt
dr dr d 2r
=-x-+rx-
dr tit dr 1

d 2r
=rx-
dr1

because dr / d t x dr / d r = 0. Thus Equation 8 may be wrinen as

-d ( r x m -
dt dt
dr) = r x F (9)

The left side here is the time derivative of the angular mome11111m:

dr
L = r x m-
d1
and the right side is rhe torque. N = r x F. so I.hat Equation 8 becom~s

( 10)

Now suppose that the force acting upon m is a ce111ral force; that is. suppose that
F is directed along r. Then r x F = 0 and

dL =0 (central force) ( 11)


dt
Equation 11 says that the angular momentum is a constant of motion in this ea~.
214 Ch<1pter 5 / V to

But. since L is a vector quantity. both its magnitude and its direction are constants.
ConsequentJy. the path of the particle will remain in a single plane.

Example 1:
...
_\'
The force acring on a particle or charge q moving with velocity v in a
magnetic field Bis F = q v x B. Detem1ine the motion if B =Bk.where B
is a constant.

SO l U TIO N: Newton ·s equations of motion are

j k
d-.·
m-=qvxB=q l.'.r
dr
u,. 1-
Figure 5.28 0 0 8
The lk:I ix described by .r = a cos q B r / m,
y = o sin q 81 /m . .: = bt. = q l\Bi - qt!_r Bj

or

dt\ d u.
mdt = -qL\B 111 - · =0
dr

We"ll learn how lo solve lhcse cquatjons for v(1) and subsequently r(t) in
Chapter 11. and a solution is (see alw Problem 5 for a reminder of how 10
solve the. c e 1uations)
q81
x(I) = o cos - - )'(/) = ll SIil. -
qBt
- ;::(r) = bt
111 m

where a and b arc const:mrs that depend U!)(Jn the inirjaJ values of r(t) and
v (/). Thus, the trajectory of the panicle is a helix_ with a uni form speed in the
Figure 5.29
;:: direc1ion (Figure 5.28). The frequency with which the panicle revolves in
A rigid body reprc~nred hy a collccrion of the xy-direc:tion. qB/m. is called the uirmo1.1r Jrc411e11cy. or the cyclvtmn
ma....scs rigidly separated from each other. frequency.

Now let's consider a rigid body that is rotating about an axis pas.sing through
the center of mass. and let's represent the rigid body by a collection of masses
w
rigidly separated from each olhcr. as shown in Figure 5.29. We'll focus on the
motion of the ith mass and then sum over all of them al a later stage. Figure S.30
shows this mass roia1ing about the axis of rotation, which we denote by w. The
position vector r; of this mass is refere need to the cent er of ma.,;;s of 1he rigid body.
In a time interval 6.1. 1he mass rotates through an angle 6.0 lo a new position
specified by r + 6.r. Now. if D./ i~ sufficien1ly small, the magnitude of Ar will
approximate the circular arc length Rb.8. where R = r sin <I> and where <I> is the
angle between r and the axis of rotation w (see Figure 5.30). If we lei n be a
unit vector along the a.:ds w in I.he direction such I.hat 6.0 > 0 corre!>.ponds to a
Figure 5.30 right-handed system. then we can write
The rom1ic,n of mass m, about an axis of
rot.arion w.
215

Note that l~rl ~ r sin <I> 6.0. as we slated above. As ~, ---,. 0. thi:-- equation, upon
introducing diffcrenLials. becomes.

dr df)
v= - = - nxr=cvnxr ( 12)
dt dt
where w is equal 10 df:J /dt. Thil- result suggests that we define an 011g11lar velociry
vector by

w=wn (13)

and wrile Equation 12 as


dr
v=-=wxr (14)
dt
The angular momentum of 1he rigid body consisLing of II such masses is given
by
I) JI

L = L m,(r1 x v;) = L m;r; x (w x r;)


i=I

where we used Equa1ion 14 furv;. No1ice that w is not subscripted because we are
discussing u rigid body. where all the mass.es rot.aie wi1h 1hc same angular velocity.
We can now derive the equation of motion of I.he rigid body by differentiating L
with respect 10 , .

dL
- = L m,lv;
r.
x (w x r;) + r,- x (w xv;)] ( 16)
dr i=l

Using Equation 14. we see lhal !.he first term on the right side vsnishe~. Problem 6
It

has you show that L ri x (w x \';) =w x Lin this case. so Equa1ion 16 becomes
i=I

dL
- = wxL (17)
dr

which is a fundamental equation of motion for a rigid body rotating at a consiant


angular velocity w. Equ::11ion 17 i). C!-(sem-ially "F:::::; ma" for such n system.

Example 2:
Use Equation 17 to show tJrn1 L precesses about lhe z. axis with an angular
frequency hJ ir O.J wk. =
SOLUTION:

j k
JL
-=ruxL...: 0 0 (J) = -wly i + wl.,_ j
dt
Lx Ly L
216 Ch.1ph.:r :; 1 Vu torr.

or
w
dl1: dL'"
- - =-(J)L1· --· =wL-'
dr · dt
These are similar to the equations lhal we obtained in Example I. Certainly.
\ L.x(f) = a cos w1. L,v(t) = a sin wt. and LJt) = b. where a and b arc
\ constanL<; that could be determined from some initial conditions. satisfy
\ these equations (Problem 5). Thus. we sec that L: remains constant while L
\ traces oul a circular motion of angular frequency 1,J about the ::: axis (the w
\
\ axis in Figure 5.3 I). The angle lhat L makes with w is given by L/ILI.

Figure 5.31
An il1Ui,tr:1tit.m of 1bc prcl'e,.~ion of the The trajec1ory that a particle 1races out with time is described by a position
angulmr rnomenium vector L about I.he vector r(t) and a velocity vector v(t) = dr /dt. The::: vector ~•(r) is tangenl to I.he
rotation axis tJJ. curve at t and the unit tangent vector is given by

T(t) =~ (18)
lv(t) I
or
v(1) = u(r)T(t) (l9)

We can also express T(r) by (Problem 8)

T(t)
.
= -dr
ds
(20)

The speed of the pan.icle is also given by

ds
u=- (21)
dt

where s is the arc length of lhe curve. given by

s= 1b v(t)dt = 1b [(dx) 2+ (dy )2 + (d:) 2] 1/2 dt (22)


a (J d1 dt dt

Example 3:
Detennine the arc lenglh of one cycle of the helix described by

r(I) = b cos r i + b sin r J+ et k


SOL u TIO N: One cycle occurs when 1 goes from O to 2rr ( or \Vhen , goes
from to to to + 2,-r ). so
5.4 Vecror Fumrions in llirt"t· U1n1(•11~i1111, 217

As in Section 2, we define the curvature K by

K =ldT I= ldT I=~ ldTI


ds
dt
dt ds u dt
(23)

and the principal nonnal vector N by

dT
-=KN (24)
d.\-

Because T is a unit vector, T and N are perpendicular. (See Equalion 12 of


Section 2.)

Example 4:
Calculate the curvature of the helica.l curve described in Example 3.

SOLUTION: We sLan wi1h

v(t) = -h sin t i + h cos, j + c k


and

Thus.

T(1) = v(t) = b sin r i + b cos r j +ck


lvU)I (b2 +c2) 1/ 2

and

dT -b cos. t i - b sin t j

Finally. using Equation 23.

K = ~
I ldTI
dt = b2 + b
c2

Notice 1ha1 1he curvature is unifonn.

The acceleration of a panicle is given by

i 1·r

a=-=- dv
dr dr 2
(25)

We can resolve a into components that are tangenl and normal 10 the 1rajcc1ory
by using Equations 19 through 22. The result is formally the same as for the two-

gl
218

dimensional case (Problem 14):


dti ,,
a= -T+ Kv-N (26)
dt
The tangential component of a correspond~ to the change in the length of v while
the normal component corresponds to the change in the direction of v. The nom,al
component is caJled the ce111ripewl arcelnario11.

Example 5:
Determine aT and aN for Lhe lrajec1ory given in Examples 3 and 4.

SOLUTION: v(I} = (h 2 + c 2) 112 and so d1J/d1 = 0. Therefore,


d1•
aT = -tit =0

=
For a space curve traced out by r(t). T t/r / d.'i is a unit tangent vector and N
(the principal normal vector) is a unit vcclor perpendicular to T. ll seems natural
lo in1roduce a third unit vector perpendicular 10 bolh T and N to consti1u1e a set
of mu1uall y on.hogonnl uni I vectors all ached to the curve. We deli nc the binonna.l
vecltJr B by

B=TxN (27)

so that T. N. and B form a right-handed coordinate system (sec Figure 5.32 and
z Problem 22). As r(t) traL:c~ out lhe p.tlh of I.he curve. the T. N. B coordinate system
moves aJong the curve. AJthough the magnitudes of T. N. and B suiy fixed (they·re
unit vectors). their oriental.ion varies with rime .
Equation 24 says (ha1 dT /ds =KN.Now let's sec how N and B vary with s.
y
Differentiating Equation 27 gives

dB dT dN dN
-=-xN+Tx-=Tx- (28)
ds ds ds ds
The firsl term on 1.hc right equaJs z.ero because. according to Equal.ion 24. N and
dT/ds are paraJJcl. Equation 28 tells us that dB/ds is perpendicular to T. But B
X
is a unit veclor, so dB/d.\· is also perpendicular to B. Thus. we .see that dB/ds is
perpendicular to bol.h B and T. and so muse be parallel 10 N. (See Figure 5.32.)
Figure S.32 Therefore. we wri1e
The (right-handed) crutc~i:m coordinate
system formed by the unit 1ange.n1 vector dB
T. the principal normal vcc:ror N. ond the - = -r(s)N (29)
binormal Vl!"clor B.
d.\·
where r (s) is called Lhe torsion f~{ rhe curve. The nega1ivc sign in Equation 29 is
just a convention . It turns oul 1ha1 Lhe shape of a space curve is un.iquely de1em1ined
from ::i knowledge of Ii.. and r as functions of the arc length s.
5.4 Vector Fum tiom in l hrf'(' Dimensions 219

Equations 24 and 29 tell U$ how T and B vary with s. To see how N varies
with s. we differentiaLe N = B x T to obrain

dN dB dT
- = - xT+Bx-=-rNxT+KBxN
ds ds ds
= r(s}B - K(s)T (30)

Equations 24. 28. and 30 are called the Frenet-Serret equations and are
f undamcntal equations of diffcrcnLial gcomcLry. which is the study of space curves
and surfaces.

Example 6:
Calculate T. N. B. K. and r for the helical curve described by

r(t) = b cos, i + b sin t j + t'I k

SOLUTION: We s1nn with

\' = -dr = - ,, SIil. ' I + b cos I j + L' k


dr

and so
(J = -ds = (b-., + C), 111-
dt
Therefore,
T = dr = dr dt = -b sin t i + b cos I j +ck
ds dtds (b2 + c2)112
and
dT = dT dr = -b cos r l - b sin r j = ii:N
ds dt ds (h 2 + /!)
where
N = - cos I i - sin t j
and
b
K = -- --
(b 2 + r 2)1/2
Finally.
I i j k
B = T x N = - - - - -b sin t b cos r c
(b2 + c2)1/ 2 - COS/ - sin r 0
1
,, · (c sin t l - c cos t j + b k)
(b'- + C 2) 1/ -
and
dB = _d_B ~ = c coS/ I +, sin r j = rN
ds dt ds (b 2 + c 2)
where
C
f =·-----
b2 + ,2

C gl
220 Chapter 5 / Ve< lOr~

5.4 Problems
d(u x v) du dv
1. Provetha1 - - - = - xv+ u x
d1 d1 dr
2. Show thal if v =constant.then v and dv /dr are perpendicular (unless v or dv /dr = 0).
3. Given a(() = 1j +, 2 k. r(O) = 0, and v(O) = 3 i + 2 j - k, find r(t) and v(I ).

. d':!r d 2 r,, . . d 2r
m 1111-,
4. Slart w1Lh m 1-.,-1
dr-
= F 12 and m 2 ---::;<-
dr-
= F21 , and derive 1he equn11on
m +m
., = F2 1, where r
.. = r2 - r 1•
1 2 dr~
ln1erprc1 this (imponan1) resu It. Hint: Use Newton ·s third law. which tells us !hat F 12 = -F 21 •
5. Solve the equations for u.r and vy in Example I by differcntialing them wi1h respect ro lime to obtain rwo
d 1u
equations of the form -.,
dr~
+a 2
11 = 0. where = v.r or v· and a 2 = q B / m. Then show that u = A co::; at and
1..1 1•

u = B sin a1. where A and 8 are constanlS, satisfy th.is equation.

6. Show !hat the right side of Equal ion 16 is equal to w x L. J-Ji11r: Use the fact that r · v = 0 and w · v = 0 for
circular motion.
7. Let's look a1 the mOl"ion of a charged particle in a magnetic field from a different point of view than we did
in Example I. You might recall thnt a charged pa11icle moving with a velociry ,. 1n a cloi-cd orbit produces
· momen1 µ. given
a magne11c · by µ, = q( r ,x \' ) . w here q .1s th e <.:harge. Sh ow Ihalµ.= 1. A • w here ,. .1s the

rr
associated electric currenl and A is the area the orbit is circular. Expressµ in terms of the angular momcnrum
of lhe panicle and describe the motion ofµ, and the charged particle in a static magnetic fidd.
8. Show 1hnt the. unit tangent vec1or T is equal to dr /ds.

9. Use Equation 18 lo find Lhe uni I longent vector 10 lhe curve traced out by r(I) = r i + r 2 j + ,3 k.
10. Use Equal.ion 20 ro derive 1hc result of the previous problem.

11. Use Equation 18 to find the unit 1angen1 vector to the curve traced out by the helix described by
r(t) = Q cost i + a sin t j + bt k.

12. u~ Equation 20 to derive the result of Lhe previous problem.


13. Prove that T and N are perpendicular.

14. Derive Equation 26.

15. Stan with Equa1ion 26 and show that K = Iv x alfv 3.


16. Use I.he formula for K ia Lhe previous problem 10 find 1he curv-.u-ure of the curve traced by r(t) = t i + r 2 j +, 3 k.
17. Use lhe formula for K in Problem 15 10 find the curvature of the curve traced by the helix described by
r(I) = a cos t i + a sin t j + bt k.

18. The trajccrory of a panic le is described by r(r) = r I + r2 j + r 3 k. Find the rangent.ial and normal componen1s
or Lhe accclera1jon a ai t = I.
19. The Lrajcclory of a particle is describe<.! by the helix r(t) = a cos t i + a si.n t j + bt k. Find the tangential and
nom,al components of lhc accelcnllion a al, = 2JT.
20. Use Equal.ion 22 10 calculate the arc length from r = 0 to t = 2 of the curve wilh p:m:imetric equations
x(1) = r,.r, + b.x- y(I) = u_..r + h_,·· z(t) = a/ + h:,, where the a ·s are constants. Does your answer make sense?

21. Calculate the curva1ure of 1hc line given in lhe prc.vious problem. Docs your answer make sense?
5.5 Lines and Pl,111(•, in $pace 221
22. Show 1ha1 N = 8 x T and T = N x B. (See Figure 5.32.)
23. Find T. N. B. K. and r for the curve traced out by r(!) =, i + 12 j + ~, 3 k.
24. Find T. N, B. K, and r for the spa~ curve 1.n1ced our by r(/) = i +, j + 12 k. fa plain your result for r.
25. A nucleus wi1h a nonzero spin has a magnetic moment µ 1hat is proponional to the nuclear spin angular
momentum I. The proportionality constant y in the relationµ= yl 1s caJled the magnetogyric ratio. We can
describe the precession ofµ classicnlly in lhe following way. A magnetic momentµ in a magnetic field B
experiences a torqueµ x Band so I.he corresponding equation of motion is di =µ x B. Using the relation
dt
µ = y I, we have d µ = y µ x B. Show that if B is directed along the z. direction. 1hcn the direction ofµ x B
dt
is such 1hat rhe tip of lhe vector µ tr:ivels along lhe circle of the cone shown in Figure 5.33, resulting in
prc1.:es-.ional motion.

Figure 5.33
The prece.,,ion of rhe rip of the magneric
moment wi:111r µ along 1tk! circle of a
cone.

( x,y, z)
.5.5 Lines and Planes in Space
z
We can use the vector fonnalism that we have developed to easily solve a number
of problems in three-dimensional analytic geometry that would be fairly difficult
Lo solve without using vectors. These calculations are best illustrated by example.
so this section will contain a fair number of Examples.
Let's st.an off by describing a s1rnigh1 line in thrcc-dimcni;innnl space. There _v
are severaJ ways to do this. For example, we can specify the line by saying
that it passes through some point r 0 (.r0 • y0 , ;:0 ) and is parallel to some vector
=
v a i + b j +ck. Ifrlx. y. z.) is any other poinl on the !foe. then the vector r - r0
is parallel to,, (see Figure 5.34) and we can write 1.he cquarion of the straight line
as

r - r 0 = 1v
Figure 5.34
where r is a parameter that can take on any real value. Equation I, which actUBl!y The line defined by the two poinL, (x, y. ::)
represents three equations, is called 1he parametric fonn of a straight line. The and (x 0 . _v0 . 2<,).

C
222
three separate parametric equations are

x = x 0 + at Y =Yo+ br z = z.0 + er (2)

The numbers (a. b, c) are caJled 1he direction 1111mbers of the line and a /lvl. b/[vl,
and c /[vl are the direction cosines of 1hc line.
The following Example shows how to obtain the equations of a line if ii is
specified by two points.

Example 1:
Delerminc 1he equation of the straight line th:ll pa-;ses through the two points
r 1( I. -1, 2) and r 2(-2, 3. I).

SOLUTION: The line is parallel to the \'ector

r 2 - r 1 =(-2-1)1+(3+ l)j+(l-2)k

= -3 i 4 j- k

Choosing the point r 1• Equations 2 give

x =I- 3, y =-I+ 4t z=2-r

l.f we had chosen the point r~. tnstcad. we would have obt.,iined

X = -2 - 31 y = 3 + 41
The parametric equations of a line arc nol unique. (The second sc1 is obraincd
from Lhe first set by replacing , by r + I.)

The parametric equations give us an easy way to see where a line passes
through the xy-pla.ne, Lhe xz-plane. and the yz-plane (the three coordinate planes).
y Consider the line described by

x =I+ 2, y =6 - 2, z. = -2 + 4,
This line passes Lhrougb the yz-plane when x = 0. or when/ = -1/2. The values
of y and z at t = -1/2 arc y = 7 and z = -4. Similarly, the line passes through
Ix the xy-plane at x = 2. y = 5 (when t = 1/2) and through I.he .~;:-plane at x = 7,
Figure 5.35 z. = I O (when t = 3) (Figure 5.35).
The straight hne de.i.cribcd by the We can express the cquation(s) of a straight line in what is called Lhc point-
pru-JJTICl.ric cquar.ions x = I + 2t, direction fonn. Provided a. b. and care not 1..cro, we can solve for, in each case
=
y 6 - 2t. ;: =- 2 + 4/.
in Equa1fons 2 and write

x - x0 y - Yo z - zo
a
= b
= c
(3)
5.5 Lines .:1nd Pl,,ne~ in Sp;ic:e 223
lf one of the a. h. or c happens to be zero (let's say a = 0). then Equations 2 yield
Y - Yo :::. - -o
- --=--
h C

instead of Equations 3. The equation x = x 0 means that the line lies in the plane
perpendicular to the x axis at x = x 0 .

Example 2:
Determine both the parametric form and I.he point-direction form of the line
thal passes through the point, r 1( 2. I. I) and r2 ( -1. 0. I).

SOLUTION: 171c line is parallel 10 the vector

r2-r 1 =(-l-2)i+(O- l)J + (l- l)k

= - 3i-j+0k
171e parametric equations are (choosing lhe point specified by r 1)
;:
X =2 - Jt y =I- , :=I

and !he equations in the point-direction form are

-2 y-
X

-3
= _, I
:=1

or simply
x-2 1· - I
3

y
Geomet1ica1ly. a plane is determined by a point that lies in the plane and a
vector normal to the plane. Let the point be r 0 (x0 . y0 . : 0) and le! v = A i + B j +
, X
Ck be the vector nonnal to the plane. Jf r(x. y. z) i.. any other point in the plane.
then r - r0 lies in the plane (see Figure 5.36). Con sequently. r - r0 and v arc Figure 5.36
perpendicular. and so The vector r - r0 lying in a plane.

(r - r 0 ) • \' =0 (4)

is the equa1ion of 1hc plane. Writing (r - r0 ) • v in terms of recrangular coordinates


gives

A(x - x 0 ) + B(y - y0 ) + CL: - :::o) =0 (5)


or
Ax+ By+ C: + D =0 (6)

=
where D -(1\x0 + By0 + C: 0 ). The next Example shows that we can also
specify a plane by thriX point!s that lie in the plane.
224

Example 3:
Detem1ine the equation of a plane through the Lhree points r 1( I. I, 7),
r 2(-2, 2, 3). and r 3 (1. -1. 6).

SOLUTION: The (WO vectors

r 2 - r 1 =(-2- l)i+(2- l)j+(3-7)k

=-3i+j-4k

and
r3 - r 1 = (I - I) i + (-1 - I) j + (6 - 7) k

= -2j- k
lie in the plane. Using lhc fact that the vector u x v is perpendicular Lo the
plane formed by u and v, we see that a vector that is perpendicular 10 r2 - r 1
and to r 3 - r 1 is given by

j k
v = (r 2 - r 1) x (1":_1 - r 1) = ··3 I -4
0 -2 -1

= -9i - 3J + 6 k

Thus. ac{;ording to E.quatjon 5. I.he equation of Lhc plune is

-9(x - I) - J(y - I)+ 6(: - 7) =0


or
3x , - 2z + IO = O

Example 4:
Dcrenninc the equation of the line through the point r(2, -1, 3) that is
perpendicular 10 the plane .l' - y + 2.: 4. =
SOLUTION: According to Equation 6. the vectorv = 1-j + 2 k is nonnal
lo the plane and is lhus parallel 10 the desired line. The parametric equations
of the line arc r + v , in vector form. or

X =2 +t y=-1-r : = 3 + 2,
The point-direction form is given by

x-2 y+I z- 3
-!- =--=
-t
-2-
5.5 Lines and Plan~ in Space 225
Vector methods also provide us wi1h a sLraigh1forward way to find the perpen- z
dicular distance of a point P 10 a line. Let r be a vector from P to any point on
the line. and let v be a vector parallel to the line. Then Figure 5.37 shows !hat the
perpendicular distance from P to the line is given by

r xv I
d = lr sin 01 = Il-v- (7)

y
Example S:
Find the perpendicular distance from the point r 1( I, 2. -1) to the line joining
the points r2( I. 0, 0) and r3 ( 1, -1. 2).

SOLUTION: The line is parallel 10 the vector I r x vl


v = r3 - r2 = (I - I) i + (- I - 0) j + (2 - 0) k

= -j + 2 k Figure 5.37
Now chooser in Equation 7 to be r2 - r 1 1ne perpendicular distance from a poim
P 10 a gi\·cn line.
r = r2 - r 1 = ( I - I) i + (0 - 2) j + (0 + I) k
= -2j + k
Then
j k
rxv= 0 -2 I =-3i
0 -I 2

and

p
Of course. we obtain the s.ame result if we had chosen r = r3 - r 1 instead of
r2 - r 1 (Problem 11 ). I
J
, 1

I V
1,
Vector methods provide us with a straightforward method 10 find 1he perpen-
dicular distance of a point P 10 a plane. Lei r be '-' vector from P to any point in y
the plane. and let v be a vector normal to the plane. Then Figure 5.38 shows that
the perpendicular distanced from P to the plane is equal to

d = lr cos(rr - 0)1 = lr cos 01 = r ·v I (8)


! X
I-r>-

figure 5.38
The perpendicular dis.tance fmm il poinl
P to a given plane.

Example 6:
Find the perpendicular distance from the point ( I. 2. 3) 10 the plane described
by 1hc equation 3x - 2y + 5;:_ = I0.
226 Chi!pter / Vector ·

SOLUTION: According ro Equation 6. the vector normal to the plane is


given by

Let x = y = 1 and solve '.h - 2y + 5;:: =


10 for .:: to see that the point
( I. I. 9/5) lies in the plane. Thus. the vector r in Equation 8 is given hy

r=(l-l)i+(1-2)j+G-3) k=-j-~k

and Equation 8 gives

d=1~1=-4
/38 38

Once again the result is independent of the point that we choose in the plane
(Problem 13).

We can use vector methods to find the tangent line to a curve and the t..angent
plane to a surface. Let's look for the tangent line first. Consider a curve in three•
dimensional space (u space c11n>e) described parametrically hy x (1 ). y(t). and z(t).
The position vector is r(l) = x(1) i + y(t) j + :(t) k and dr /tit is the vector langent
to the curve l!accd out by r(1) as 1 varies. The vector r' ( 1) is called the rangent
vhtor of the curve at the poinl x(r), y(I). ;:(I) provided r'(t) i= 0. The tangent /i11e
=
is the straight line passing through the point r(r 0 ) x (r0 ) i + y(r 0 ) j + ;: (to) k and
parallel to

,
r (to) =(d.r)
- + (dy)
df
_:_ .+ (dz)
10
I-
dI 10
J
dt 111
k

z
According to Equations 2. the tangent line is dc$cribcd by the vector equation

r - r(ro) =t (dr)
dt 111
(9)

or by the pard.ITlelric equations

)' x -x0 =1 (dx)


-
dr 10
y - Yo =I (rh) d-, to ;: - lo = I ( -d::.)
dt 'o
( I 0)

where I can take on any real value.


lx The ,wm,aJ plane to rhc curve at the point P0 = (x 0 • y0 , z0 ) passc). through
Figure 5.39 P0 and is perpendicular 10 the tangent line (Figure 5.39). According lo Equa1jon 5,
A lipllCt: curv1:, ii~ tangent vector r-'(t ). and
its normal plane.
=
if v A i + B j + C k is a vector that il> nonnal to a plane, then the equation for
=
the plane is A(x - x 0 ) + B(y - Yo)+ C(:: - ;:0 ) 0. Using &.1ua1ion 9 or 10. we
see that the equation for the norn1al plane is
5.5 li11,•~ ,md Pl.1111·~ in Sp.11 ,. 227

(11)

Example 7:
DclCnnine 1he tangent line and the normal plane at 0 = 2JT 10 the curve
whose par.1mc1ric equations arc

x(H) = h cos 0 y(8) = b sin 0 z.(O) = c(I

SOLUTION: = b co,11 I+ bsin O j + c0 k and r'(0) = -b sin O i +


r(U)
b cm ti j At 0 = 2rr, r 0 =bi + 2.rrc k. r = I, j + ck. and so
+ (' k. 0
.Equations l Ogive

x=h y =hr

The nom1al plane is described by by+ c(: - 27Tc) = 0.

5 .5 Problems
1. Dctcnninc the par.1mc1ric C{1uaIions of the s1ra ight line thal pa,, sc, through the poinr.s (2. 0. 3) and (-1. 2. -1).

2. Detcnninc the point-direction equations of the straight line that passes through the point (2. I - 3) and is
parallel 10 v = 2 i - j + 3 k. Docs the point (0. L. -6) lie on this line'? What about the point (4. 0. 0)?
3. Dcrcnninc the equation of the plane that pas.~s through the poililt ( I.. -1. 3) and is pamlh.:l to the xy-planc.
4. Dctem1ine 1he equal ion of the plane I hat pa!>se-.s through I.he point (3 . I. 2) and is parallel 10 the plane described
by 3x - 2y + 4.: = 5.
S. Determine the equation of the plane that passes through the point~ ( I. 2. 1), (-1, 0, I), and (3, I. 2).
6. Detenni ne 1he equation of the plane that contains the two vectors u = 2 i - 3 j + k and v = -i + j - 2 k.
7. Define 1hc angle be1wccn two planes 10 be 1hc angle be1ween 1heir nonnal vec1ors. Find 1he angle be1ween 1.he
plant!s described by x - y + 2: = 3 and 3x + y + 2: = 7.
8. Detennine lhc equa1ion of Lhc line of in1ersecrion of the 1wo planc.s in Problem 7. Hint: Find one point on the
line and a veclor par.:i.111!1 IO lhc line:.

9. C:ilculatc 1hc pcrpendicu lar dist~mce from the poi nl 0. -1. -·I) 10 the line joining (0. 2. I) and ( - 2. - I, - I).
10. Find the perpendicular dis1ancc from the point (-2. I. 1) 10 Lhc plane described by 3x - 2y + 4;: = 6.
11. Calcul:ue d in Example 5 using r = r .l - r 1•

12. Why is Equ.i1ion 7 independent of the choice of the poim on the line?

13. Why is 1hc rc::.ult in Example 6 independent of tJ1e choice of the point in 1hc plane?
14. Show thnl the perpendicular di,tance from the point (x0 . _r0 . '<>->101hc plane de~cri,bcd by Ax + By c~ D=
0 ·-. ,· . b d _ IA.ro + B.ro + C zo + !)I
•· given y - (A 2 + 8 2 + Cl)l /2 .
15. Show that the angle berween the planes A 1x + B1.\' + C 1z + D 1 = 0 and A 2x + B2 y + C'2z = D2 is given by
A1A2 + 81B1 + C1C2
COS 0 = (Aj + Bj + C 2) 112(A -, + 8 + C ~ )112
? l l •
1 2 2 2
16. Dctcnninc the equations of Lhe tangent line and the nonnal plane 10 rhe space curve described by x = t + I.
y = 2, 2 - I. z. = 2,3. where I.he curves cross the yz-planc.

References

Frank Ayres, Jr.. and Elliou Mendelson. 1999, Calcullil, 4th ed .. Schaum·s OulJinc Series.
McGraw-Hill
C.H. Edward-.. Jr.• and David E. Penney. 1998, Calculus and A11aly1ic Geomell]'. 5lh ed ..
Prcn1icc-Hall
Wi1old Kosmnla. 1999, Advanced Calc11/us: A Friendly Approach. Premice-Hall
Jerrold Marsden and Alan Weinstein. 1985. Calc11l11s /, II. and Ill. Springer-Verlag
Murray Spiegel. 1959, Vt'rtorAnnlysis. Schaum's Ou1linc Series. McGraw-Hill
Murray Spiegel. 1963. Acfr1111ced Cnlmlus. Schaum·s Outline Series. McGraw-Hill
MATHEMATICAL TABLES:
CRC Srnndard Mmht•mmical Tables and Fom,11/ae. 30lh ed .. edited by Daniel Zwillingcr,
CRC Press ( I 996)
HISTORY:
Michael J. Crowe. 1967. A Hisrory of \l(•ctor Analysis. Dover Publicauons

m
The Concept of Fractal Dimension

Lei's consider what we mean by dimension. Suppose we have a continuous curve of unit length. The number
of Ji1tle s1raighl-line segmcnls of length€ 1ha1 ii will 1akc 10 ··cover" the cur.'t: is N(f) = 1/f. Similarly,
if we have a unit area. then the number of liulc squares of area f~ that it will take to "cover" 1hc area is
N(€) = 1/f 2. You cans.cc 1ha1 tht: dimcn~ion din cn1:h case is given by fd. which we can fomialize by
defining the dimension by

. In N(€)
d= lim - - -
t-0 In( 1/€)

Now le1·s consider 1he following figure:

N t:

I
2
·'
4
9

,
8 27

I
11 steps 2n
311

We s1ar1 with a unit line segment and remove 1he middle third. Then we remove the middle third from each of
the two remaining l>Cgments. We continue 1his proce,,. producing in the limit an infinite number of separale
pieces. each of zero length. This limiting result is known~ a Canior ,et. The figure shows N. the number
or :,,cg.menis, and f. the leni;1h of each one. ai each ~iage of subdivi ... ion. The dimension of the Cantor~, is
less than one. but perhap, greater than 1.ero. Using our fom1al dctini11on. we have
211
d = lim In = In 2 =0.6309
ln 311
11-•oo ln 3

Thus. we find 1hat the dimension of a Cantor set is a non.integer. Sets wilh nonin1eger dimension are called
fraL"tal:i:. TI1e Cantor se1 is just one of mnny se1s that are fractals. which play a key role in non linear dynamics.

229

al
CHAPTER 6
Functions of Several Variables

In Chapter I, we reviewed 1hc calculu~ of functions of a single variable. As 1he


1itle say~ in thil-- chapter we will discuss functions of more than one variable.
Many physical quantities depend upon more than one variable. For example, the
pressure of a fixed quantity of a gas depends upon the temperature and the volume.
The temperature of a body may vary from point 10 point and so depend upon
1he three spatial variables, x, y. and ;:. Even just a cursory look al any book on
1hem1odynamics shows tha1 the formulas of 1hermodynamics abound in panial
dcdvatives.
Many or the concepts of a func1ion of a single variable, such as limits of
func1ions and continuity, carry over to functions of two or more variables, except
that the domains arc regions in 1wo or more dimensions rather lhan intervals on
1he x axis. A significant difference between functions of a single variable and
those of several variables is I.hat you can form partial derivatives of functions of
severnl variables. which leads 10 mixed higher partial derivatives and a varie1y of
chain rules. After di~ussing lhese topics in Seel.ions 3 and 4. we go on 10 study
diffcrentials of functions of several variables in Section 5 and then directional
derivatives in Section 6. The study of direclional derivatives leads to the grndiem
vector of a function j(x, y, ~). We shall see that I.he gradient vector of J (x. y. z)
has the physical inlell)relation of pointing in I.he direct ion of I.he mos! rapid change
in J(x, y, z). Because of this properly, the grJdient vector occurs in !he descript.ion
of a number of physical phenomena such as Lhe direction of be.al flow or 1he
direc1ion of diffusive tlow. After discussing the extension of Taylor·s formula to
functions of s.everal variables. we discuss the important topic of the maxima and
mini ma offunct.ions of several variables ia Section 8. The invcs1igat.ion of maxima
and minima or functions of even two variables is a little more involved than for
functions of a single variable. The conditions 1ha1 'rlf/ox = 0 and fJJJoy= 0 at an
cxtrcrnum are necessary conditions (as is df /dx = 0 for a single variable). but they
arc not sufficient. We'll learn in Section 8 lhat the namre of a critical point (the
point where firsL partial derivatives equal zero) depend~ upon Lhc relative values of
the four different .-.econd partial derivatives, a2 f /cJ.x 2, a2J!oy2. 3 2J/Dxny, and
a2 JI ayox (al though usually a2JI ax ay = a2 JJa yox ). In Section 9, we djscuss
Lagrange's method of undetem1ined multipliers. which is used 10 maximize or
minimize Functions of several variables when the variables arc related to each other 231

al
232 Ch;ipJer 6 / Fun tion o{ Severol Vari.:ibles

by one or more constraints. Finally. in Section I0, we discuss multiple integrals.


y In most cases throughout the chapter, we shaJI restrict ourselves to functions of
b + c5 two or three variables. but most of the result~ are readily extended to any number
of variables.
(a, b)

b-o
6.1 Function
a-6 a+o X

Figure 6.1 We define a function of more lhan one variable much the same as we define a
A rectlln gular 8 neighborhood Jelincd by function of one variable. We say that ;:: is a function of x and y if there is a rule
Ix - al < S. y - hi < S. that allows us to determine one or more values of z for a given pair (x. y). We
shaJI restrict ourselves to real functions in this chapter. In this ca.Cie, the domain is
y a set of pairs of numbers (.x. y) and the rdllge is a set of real numbers;:. We denote
this relationship by z;::: f (x. y). or z:::::: z(x. y). In this second expression. z on
the left denotes the value of z: and.: on the right denotes the function. This slight
b • ambiguity causes linle problem in practice and is common notation.
If z. := j(x. y). we say that z is a depeT1dent variable and Lhe x and y are
independent variables. As with funcLiom: of a single variable, z is sin,:le-valued
{I X if one value of z. results from each pair (x. y) and multiple-i•aliu:d if more than
one value of z. results from a pair (x. y). As with functions of a single variable. a
Figure 6.2 multiple-valued function can be viewed a-. a collection of single-valued functions.
The circular S neighborhood defined by
As usual, we shall mostly restrict ourselves to singlc-valut!d functions.
(.r - u) ~ + (y - b)~ < .5 2 •
Because f (x. y) is a mapping from a set of ordered pairs of points (x, y) to
a set of single points. we need to define a few tenns that describe sets of pairs of
points. First we define a rectangular 8 neighborhond of the point (a. b) as the set
of all points such that l.r - al < 8 and IY - hi < 8. where 8 > 0 (Figure 6.1 ).
If the point (a, b) it- excluded, so that O < Ix - al < 8 and O < Jy - hal < cS •
• then we have a dele1ed recurngular 8 neighborlwod of (a. b). We can also have
a circular 8 neighborlwod, defined as 1he set of all points such that (x - a) 2 +
(y - b) 2 < l 2 (figure 6.2). If O < (x - a ) 2 + (y - b)'J. < c5'J.. we have a deleted
circular 8 neighborlwod of (a. b).
Figure 6.3
A point (a. b) is called an interior poinl of a set S if there cxisL" a neighborhood
An example of an interior poim. tl
boundary point, and an exterior point of a of (a. b) that lies entirely within S. 1f every ncighborhood of (a. b) contains not
set. only points in S but also points not in S. then (a, b) is called n boundary point. A
point (a, b) not in S is called an exterior poi11r of S if there exists a neighborhood of
(a. h) in which none of the point~ belong to S. Figure 6.3 iIlustratcs this diffcrencc
between an interior poinL a boundary poinr, and an e:,.:terior point
If a set consists entirely of interior points. then it is called an open set.
For example, the set of all points such that x 2 + y2 < a 2 is an open set. An
open set contains no boundary poin1s. Conversely. a closed set contains all of its
boundary points. Finally. a set is said to be connected if any two points in Scan
be connected by a piecewise smooth curve lying entirely within S (Figure 6.4).
Figure 6.4
An open connected set is called a domain. or an open region. Thus. the annulus
An iUustration of connected scl ·. a < x 2 + y 2 < h is a domain. A region i~ a domain containing some or all of iLS
o. ! Func1ions 233

,, ,,
,, .. - ...
\, I
I
' '\
I
I

,,
,--~ ... 'I
I

, '
' I
I

I
I
' ... _, I
(a) ' ...
\
I
(b)

Figure 6.5
(a) A non-<:'onnected set. The ~t of poinl~ wi1hin the two ~ts S 1 :ind S2 docs nor constitute a region.
(b) An open region. or a domain. (The da.,hcd lines signify 1hat the boundary points are nor inclU<Jcd in S.)
(c) A closed region.

boundary points. If ii contains all of its boundary points. it is called a closed region.
Figure 6.5 summarizes these definitions .
y

Example 1:
Describe the set SI (x, y)} such rh:11

in terms t)f the above definitions. ls the set connected? Is it a domain?

SOLUTION: The set is shown in Figure 6.6. It is connected because we


can join any two points in S by a piecewise smooth curve lying complclCly
within S. h is a domain. or an open region. because it does not include iis
two boundaries. Figure 6.6
The sc1 of all points such that
2 <.., 2 +y2 <9.

Example 2:
Describe the set S = {(x. y)} such lhat I~ x 2 + y'.! < 2 in terms of the above
definitions . Is the set connected? Is it a domain?

SOLUTION: The set is shown in Figure 6.7. It is connt.'\:tcd bc1,;uusc we


can join any lwo points in 5 by a piecewise smooth cur.·e lying completely
within S. It is no1 a domain because it contains some of iL..; boundary points
(x + y2 = I). hUl it is a region.
2
- 8- X

1
We can represent the functions that we shall encounter in this book by plotting
z = f (x. y) in a three-dimensional Cartesian coordinate system. The graph of
Figure 6.7
;: == f (x. y) is the set of all points with coordinates (x. y, .::) that satisfy the The set of 11II points such thar
equation;:= f(x. y). The graph of a function of two variables is called as111face. t ~ x 2 + y 2 < 2.
6.1 Fun lions 237

I
I
I
I

y y y

X ,
Figure 6.19
I
I X '

Figure 6.20
x,

Figure 6.21
The surface of a hyperholoid of one sheet The surface of a hyperboloid of two shceL'- The surface of !I hyperbolic paraboloid of
. -
~

v-
, ~

-- . ~2 .{ 2 \"2 xl vZ
descnbed by - + ;_2 - 1 = I. descnbed by--:; - - - :.-,= I. one sheet described by cz. = 1 - ;_2 •
a2 b c· c· a2 c- a· h

(Figure 6. 18)

(an elliptic cone) (5)

(Figure 6. 19)

(a hyperboloid of one sheet) (6)

(Figure 6.20)

:2 x2 )'2
-----=I (a hyperboloid of i-wo sheets) (7)
c2 a2 b2

(Figure 6.21)

x2 y2
C'."=- -
,. a2
-
b2 (a hyperbolic paraboloid) (8)

In rhis sectjon. we have limited ourselve~ to functions of two variables for


concreteness. Generally we can have functions of any number of variables and
write:= f(.r 1• x 2 •...• x,1 ). Then z = f(x 1• x2 ••••• Xn) represents a mapping
from an n-dimensional space to a one-dimensional space.
23 8 Chap1er b I Function, oi Several V;iri abl !S

6.1 Problems
1. Calculate the distance between 1hc poinis ( L -1. 2) and (3, 2. 6) in a three-dimensional space.

2. Dctcnnine 1he domain associated with 1he real funcrion sin -t x - Y.


x+y
3. Is the set of points for which sin x = 0 a connccred se1?
4. ls the set of points for I :::: x~ ·f xl + xl + x] 5 5 a domain; a region?
5. The equation .x + 2y + z. = 4 is the equation of a plane . Let x. y. and z. equal zero in rum 10 skc1ch the planar
surface.
6. Describe rhe planar surt·ace given by x + y = 0.
7. Equation 2 is the ~quation of an ellipsoid centered at the origin. Wri1c down the equtition of ::in ellipsoid
centered al (I, -J. 0) .

8. Sketch the level cu!"'es in the xy-plane for the surface in the previous problem.
9. Sketch some level curves of /(x . _v) =y - 2.x"".
IO. Sketch some level curves of /(x. y) = x2 + y 2 -6x + 2y .
l l. Identify rhe surface in 1hrec-dimen.~ional space that is described by the equal.ion .r 2 - 2x + y 2 + ;: 2 + 4: = 8.
12. Identify 1hc :;urfacc in three-dimensional space 1har is described by the equal.ion 4x 2 + 16y 1 = 32.
13. ldent·ify the surfai;e in three-dimensional space represented by .r 2 + 2.r + 4y 2 = 8z - 17.
14. Starting from Equa1ion 5. ~ketch the surface that it dei;cribes.

15. Sta.ning with Equ;nion 6. sketch the curve that it describes.


16. Detem1ine the distance between the points (I, 0. -1. 2 . 3) and (2. I. -1. 0 , 2) in u five-dimension:il space.
Hin1: Generalize lhe Pythagorean lheorcm.
17. Explain why 1herc is u gap between Lhc 1wo sheets in Figure 6.20.
~ ' If '
18. Use any CAS 10 plol the surface xe - cx· y-l • . (This func1ion is a hydrogen p orbit.al.)

19. Plot lhe level curves for thl! function in the previous problem .
1 1
20. U,c any CAS to plot the surface xy,, -(.r~ i l " . (This function is a hydrogen d orbital.)
21. Plot the level curves ror the function in the previous problem.
22. Use any CAS 10 plot the surf:u.:c "in(xy/ 10).
6.2 Limi1~ ,md Continuiry 239
6.2 Li mits and Conti nuity

The defini1ion of the limi1 of a function of several variables is $imilar to that of a


function of a single variable. We say that

~i!~ f(x. y) =I ( I)
Y- •I•

if for any pos111ve number r:. we can find a pos111ve numhcr 8 such that
!J(x. _\') - /I < t: whenever O < (x - a)~ + ()' - b) 1 < li2. The value of 8 de-
pends upon 1he value of€ and may depend upon the puinl (a. b). The func1ion
need not t>c defined al (a, h) for the limit to cxisl.
For example. suppose that f(x. y) ~ 3x 2 - y 2 . h appears that the limit of
f(:r, y) a.-; x ➔ I and y ➔ I is 2. We can prove Lhat this limit is 2 by finding a 8
such 1ha1

1/(.r. y) - 21= 13x 2 - y2 - 21 < € (2)

whenever

We can satisfy Equation 3 by writing O < (x - 1) 2 < 82 /2 and O < (y - 1) 2 < f-;2,
or by writing x = I ± 8/ ./2 and y = I ± 8/ ✓ 2 . Substilute 1hcsc cqua1ions in10
Equation 2 10 obtain

Solving rhis expres..,ion for 8 gives ,S ~· "/2~/:. for small vaJues of€ (Problem I).
The limit in Equal.ion I is carried our in the xy-plane. Just as the limi1 of n
funcLion of a single vari:ible must be unique. 1he limj1 of j(x. y) musl be unique. (x. y ) (x. y)
This means lhat the limit in Equation I must be i11depe1tde11t oflhc path along which
x and y approach the point (a. b) in 1he xy-plane (Figure 6.22). Let's reconsider Figure &.22
If the limit ot" f(x . y) e,i:)u. al the point
the limi1 off (x. y) = 3x 1 - y 2 as (x. y) - ( I. I). The family of straight lines that
(u , h), it must ha,•c thl': ~amc v:thu:
pass Ihrough the point (I, I) is given by y =
m (x - I) + I. where m is 1hc slope indc:pcnde111 of rhe parh along which
(Problem 2). If we subs11tutc y = 111 (.r - I J + I into I (x. y) = 3x 2 - y 2 , we have l r. y) :)ppro;icht",; {£1. h).

Now let x = I + € and lei€ -+ 0 10 get lx.,·1-,(1.n


lim /(x . .r) =2. indepefldenr ofm .
TI1is C\c:rcisc is no1 s1~/Jicie11t 10 prO\'C rhar 1hc limit exisL'-. however, becaus.e
the limir muse be independent of the pail, along which (x. y) approaches (I, 2). ll
is certainly necessary 1.hat the Limit be independent of any straight line palh. bu1 n
rigorous approach con,isr., of Equations 2 and J above.

C
240 Chapter b / Fu rr lions of Sev ral Variables

Example 1:
Delermine
lim j(x, y)
(.T. ,·)-• (0,0)
where
x-y
(x. y) # (0, 0)
/(X. )') = X +y
0 (x, y) = (0, 0)
SOLUTION: The family of sLra.ighl lines Lhat approach (0. 0) is)'= mx.
Substitute 1his inro f (x, y) 10 gel

1-m
f(x. v) =- -
- I+m

The limit here depends upon m. the direction in which x and y approach Lhe
y origin. so Lhe limit docs not exist (it is not unique).
~
( a. b)
~

,
We can also take the limit (x, y) ➔ (a. b) in a sequential fashion by (Fig~
urc 6.23)

Lim [ lim f(.x.y)]


:t-.a J-• b
or lim [ Jim j(x,
y-•b , •,1
y)] (4)

X
(x ' )' ) The sequential limits of f(x. y) = 3.\.:-! - y2 as (x, y) ➔ ( I, I) are

Figure 6.23 . [ Jun


lim
.r-• I
. (Jx·,, - y-)
y-• I
,, ] =.r-,
. (3x-,, -
hm
I
I)= 2
An illLL,;tration of the sequential limiL,; in
Equation 4. and
lim [ lim (3.x 2 -
y- I .(....., I
i)] = y--.
lim (3 -
1
y2) = 2

The two sequential limits come out lo be the s.ame in this case because the Limit
exists.

Example 2:
Determine the ~cquentiaJ1Ii mi LS of the function in Example I.

SOLUTION:

. [ 1·1m -
I.1m x - 1·
- - =1m(I)= I y]
.t--.0 y-0 X + y .r-0

Jim [ lim
_1·-.0 :t-,,0 X
x-+ yyJ= lim(-1)=-I
.r-0

The function has no limit a p:. y) - (0. 0) because 1he two limit-. differ.
6.2 LlmilS and Continuity 241

If the sequentiaJ limits d.iffer, the limit does not exist. IJ the sequenLial limits
arc the same. however, the limit may or may not exist. Thus. the equality of the
s.equentiaJ limjL.., is a necessary but nm sufficient condition for the existence of a
limiL Sequential Limits constitute only two directions. Wld the limits must be equaJ
for oil directions (see Problem 12).
Just a.-; for a f-unction of a single variable. the limits of functions of severa.J
variables have the following properties:

Jim
(.r ._Yi-., (a .h)
letf(x, y) + {3g(x. y)l = a f,r. y)-
lim
(n. h)
/(x, y) + /3
I r.
Lim
_1')-,. (a_/,)
g(x, y)
(5)
lim
(.r. y) ... (u .I.>)
J (x, y)g(x. y) = ( .\" .
Jim
) (a. b)
f (x. y) • lim
{x •.1·)~ (<1 .h)
g(x, y) (6)

J(x, y) limc.r.y>--(a./Jl J(x. y)


lim = (7)
(.r,n--1.,1,b) g(x. y) lim(Ly)-,.(a.h) g(x, y)

where a and /J are constants in Equation 5 and provide<l g (x. y) -:p O in Equation 7.
Equations 5 through 7 essentially tell us that if J (x, y) and g(x. y) are conunuous
at (a, h).then so are af (x. y) + fJR(X. y). j(x. y)g(x, y). and j(x, y)/ g(x. y),
provided g(x. y) #- 0.
The limiting processes discussed so far in 1his. section lead directly 10 Lhe
defini1ion of the continuity of a function of several variables. The function J(x. y)
is said to be.continuous at (a. b) if J(x. y) - f(a. h) as (x, y) - (a.b).In rcnns
off, and <=. J (x. y) is continuous at (a. b) if for any positive number E. we can
dc1cnninc a posi1ive number 8 ~uch tha1

lf(x. y) - J(a, b)I < <=

whenever

(8)

As usual. the value of 8 will depend upon the value off and may depend upon
(a, b).
As with functions of a single variable, the following lhrec conditions mus.I
hold for /(x, y) 10 be con1inuous al (u. b):

I. j(x. y) is defined at (o. b)

2. lim J(:c. y) exists


lx._1•)-(u.b) (9)
3. Jim
l.r.y)-(a.h)
/(.r. y) = J(a. b)

We can summarize these three conditions by writing

lim
(x ..l")-(a.h)
f(x. y) =f ( lim
Cc,v)-{a.b)
(.r. y)) = j(a. b) (10)

C gl
242 Chaprcr 6 / Funct ions of everal Variables

If f (x. y) does not satisfy these conditions. then f (x. y) is said to be discontinuous
at (a. b). For example, the function
xy
f(x, y) =
l ..,
x-; y-
..,

is discontinuous at (0. 0) because the limit of J (:c. y) :is (x. y) -


(x. y) :/:- (0. 0)

(x. y) = (0, 0)
(0. 0) docs not
cxis1.

Example 3:
Show that

, ., . I

j(.r. y)

i:- continuous at (0. 0) .


= .
I (r + r ) sin-,-
0 x- + y-
- , (x, y)

(x, y)
'#= (0, 0)

= (0, 0)

SOLUTION: To detennine the limit. lim f(x. y). lei x 2 + y 2 = r 2


(.r, 1·)-• ◄ 0,0)

and let r 2 ~ 0. Because I ~in 1/(./! + y 2)j ::: I,

independent of the manner in which (.r. v) approaches (0. 0) . The three


conditions of Equations 9 arc ~atisfied, so f (.t, y) is continuous at (0. 0).

We detined uniform continuity of a function of a single variable in Section 1.3.


In an entirely analogous manner. we say Lhat J(.r, y) is uniformly continuous in
a region R if to an arbitrary€ > 0 there corresponds a number f, such 1ha1 for all
poinL-. (x 1• y 1) and (x 2, y 2) in R

This occurs whenever (x 1 - x 2 )2 + (y 1 - y 2 )~ < o2 . The key requirement of the


rmifonnity of the continuity is 1hat 1hc same value of l, works for al I the poinrs in R.
We stale without proof that a function which is continuous in a closed region is
unifonnly continuous in that region.
Although we have limited our discussion to functions of only two variables
in 1his section, rhc resulL" arc readily extended Lo more than two variables by a
slight change in noration. For example. a funclion of three variables (11. v. w) is
continuous at (a, b. c) if

lim
l u, t•,u,) - •(n,h.r)
j(11. v, w) =f ( Jim
(11 ,IJ , U' ) 1 , /J,1·)
(11, l', w)) = f(a. h, c)
m
6.2 limits and Continuity 243

hample 4:
Does 1he limit

. ·! 2y - 3;:
j (.r' y. :) = X------'--
2x - y

as (x. y. z) ---+ (0, 0. 0) c:i.isl?

SOLUTION: The limit must be independent of the path along which


(x. y. z:) approac.:hes (0. 0. 0) . If we let z-,. 0 and then .r.. -> 0. we obtain
-2 as the limit. If we let z - 0 and then y - 0, we ob1ain 1/2. The limit is
not unique. and so docs not cxisl.

6.2 Problem

I. Show 1ha1 I 3 ( I ± }i)' - ( ± }i)' - 21< , g;ve., ~ «/2'"' forsmall values


I of,.

2. Verify 1ha1 y = m(x - I)+ I is a family of sLrnight lines pa.<;sing through rhe point (I. I) .

3. Use tJ1e €-S notation to show thol Jim (x + 2y) = 5.


(x ..1·)--tl.~)

4• Use •u1e
1
€-a" notauon

10 show that 1·1m xy-
-')-·- = 0.
(.t.y)-40.0) .x- + y 2
. x~ .
5. Show 1ha1 hm -.,-··- docs not exist.
tx.y)-•(0.01 x- + y 4

., ...
. x- r- 0 . 0 2 } " , ..,
6. Show Ihat I1m ----,::,;; . Hmr: Use :S (x - y)- to show thal L..rl_,·I : : _ x + y".., .
(.r , ) ' ) - • \0,0J XJ + y-
7. EvaJunte the following Limits. if they exist:

(a) lim x 2 sin ~ (b) lim


1, ••, ) (2,lT) X (x. _,1 •l,1./,)

8. Evaluate the following limits. if they cxis1:


(a) lim 2t - y (h) lim e- l/xy
11., )-• (0,ll) .f + )' (x ,_1•)- (0,ll)

9. Evaluate the following limits. if they exis1:


~ x_v
(a) lim 1an- 1 :!.. (b) lim
{.Ly)-(0.1) X (.t,yl •!O.O)x2+\'2

10. Evaluate the following limits. if they exist:


x2-y2
(a)
1,r . ,1
lim
·W.O) x~
.t 3
..,
+ _\':i
+ y- J (b)
,,.,1

11. Take .sequentiaJ limits of 1he following and detelitline if the limit cxi~1~:
x2 .t - 2y
lim
.,n.oJ
I ---
+
x 2 y2
0
X

x=y
'f=- _\'

(a) lim ---., (b) Jim - -


Ix •.'/ , ro.o, x2 + _\'- 2 <.• ,)' ) - •<- UJ x + y
244 Chap1e, 6 / Func1lons of St•1wal Variables

x:+- <)
2
12. Show 1ha1 the sequential Iimits of llrn ( are equal. Now show that the limit is not independent
(.r.y)-IO.Oi X )'
of m if y= mx. To whar values of m do 1hc sequential limits correspond?
13. Is J(x, y) = x//(x. 2 + y 2) con1inuous at (0. 0)?
1
(x, y) I- (0. 0) .
14. 1, f(x. y) ={ x:~ y 1 continuous at (0. 0)?
(x, y) = (0, 0)

15. Is f(x. y) = f ./f'y• (.r. V)


·
'F (0, 0) .
con1muous at (0, 0)?
(x. y) = (0, 0)
16. Which of the following functions is continuous at (0. 0)'?
X
(a) x 2 + y2 (b) xy +6 (d)
2x +y

6.3 Partial Derivatives

The evaluation of a partial derivative or a function of several variables is similar


to the evaluation of the derivative of a function of a single variable. Suppose. we
have a function j(x. y). Then. the partial derivarive of /(.x. y) with respect to x
is defined by

fx ==of;::; lim J(x + ~x. )') - J(x. y) (I)


iJx .lu-,.0 ~x
when this limi1 exisL'i. The partial derivative of J(.x, y) with respect toy is defined
in a similar manner:

f = rJJ = lim f (x. y + 6y) - J(x. y) (2)


-"' oy ov-0 6y

Example 1:
Use Equations I and 2 to delermine /.rand f_v if f (x. y) = 2x·' + 6xy + _v 2.
SOLUTION: Using Equa1ion I.

. f(.x + 6.x. y) - J(x. y)


f .r = I,m
6 .r-0 6x

== .
hm
2(.r + 6x)-' + 6(x + 6 .x)y· + y 2 - 2-c 3 - 6x,· - v 2
· ·
ar-o 6x

= 6x 2 + 6y
6.3 Par1ial Derivatives 245

Using Equa1ion 2.

. f(x, y + 6y) - /(x. y)


f 1,= I1m
· ~\1 ·•O lly

6x(y + 6y) + ()' + 6y) 2 - 2x 3 - 6xy - y2


Ay

= llm
. 6x6y + 2yA\'· + (6\')2
·
.\1•--.0 A}'

= 6x + 2y

Example 2:
Use Equations I and 2 to evalualc ft and / 1• at lhc point (2. I) if
f(x, y) = lr 3 + 6xy + y2.
SOLUTION: Using Equation I. we have

f, (2, I)= lim /(2 +Ax. I) - j(2. I)


. tl.c-0 AX

= lim 2(2 + A,t") 3 + 6(2 +Ax}+ I - 29


a.t-0 AX

= Jim 306x + I2(6.x) 2 + 2(6x ) 3 = 30


A -, Q 6..r

Using Equation 2. we have

Jy( 2 . I)= lim f (2. I + Ay) - f(2. I)


6y-0 6y

= lim 16+12( 1+ .l\y) +(I+ y) 2 -29


~y-0 Ay

. 146v+(l-.\') 2
= ~J-CJ
Inn · ·
A)'
= 14

Notice that we obLain the same result by using

and f'f = 6x + 2_y


and lening x = 2 and y =
I. As we pointed out in Section 1.4. this procedure is
valid only if fx and J>. are continuous functions of x and y. which is most oflen
1he case in physical problems. (Sec. however. Problem 14.)
246
Equations I and 2 say 1ha1 you can determine partial dcri\la11vcs by differen-
1ia1ing with respec1 to one variable while keeping Lhc other one as a constant. For
=
example. if j(x. y) er sin xy. then


f~
A
= -i'Jf
ox = c! sin xy + •,,t! cos .r1• 1

and
aJ = xe-
.{\.
_.= -ay cos x .r
1

Sometimes we write (af (dx)y 10 cmph.:isizc that y is held constant when we take
the derivative of J wilh respect 10 x. U~ually, however. it will be clear from the
con1ex1 which variables are held constanl.

Example 3:
The van der Wa:ils equation is an approximaLC equation for lhe pressure of a
gas as a function of i1s temperature and vnlume. The van der Waals cqualion
for one mole or a gas is

RT a
P= - - - -2 (3)
V -b V

whl'fl' R is the molar ga.., constant and a and b are tn11,1an1s that arc
charactcrist.ic of !he. particular gas. Determine (a P / 'cJ"/')" and (iJ P /o \I ) 7 .

SOLUTION:

p and

( ap) RT
i:JV r =-(V-b) 2 +
2n
VJ

Physically. r.hcsc equations govern how lhe pressure varies as we change the
1empera1ure at conslanl volume, and how the pre,;~ure chang1..·~ as we change
the volume at cons1ant 1empcro1urc. No11cc tha1 (oP/cJV)r is a func1ion of
both T and V.
V

Panial derivatives have a nice geometric ioIcrprc1aLion. Figure 6.24 1,hows


the surfa~ corresponding 10 Equation 3. Let"s choose some fi ., ed 1empera1ure T0.
Figure 6.24 171c condition To= const.ant is described by a plane parallel to the P V-plaI1e and
The pl\!,sure surface for rhe \'an der W;rnls
in1crsec1ing 1he T axis at T,, 1• Then. the par1ial deriva1ive (o P ;a V) 70 is 1he slope
cquatioa.
6.3 Partial De-iiv,1tiv1•\ 247

of Lhe tangent line to the surface in the T0 = cons1ant plane. Figure 6.25 shows (8P/8V)r
the intersection of 1he T0 = 500 K plane and the pressure surface in Figure 6.24.
a a
The slope of Lhe tangent Line is ( PI V >ro=SIXI K. Similarly, the partial derivative
( i) P / aT) v is the slope of Lhe Ian gent line 10 the surface In Figure 6. 24 in a plane
that is parallel 10 the PT -plane.
The tangent Iincs to a surface can be used to define a 1angent plnnc. Suppose we
hnveasurfacedescribcd by z = f(x. y). Then iJJ/iJx at n point P = (a. b. f(n. b)) V
on the ~urfacc is the slope of the tangent to the ltnc formed by the surface with the
Figure 6.25
plane y = b, as shown in Figure 6.26a. Figure 6.26a shows a vector of the fonn The partial dcrivati1•e ("d P fa V 17 al
=
T 500 K for the l'an der W:.wh equa1ion
u=i + fx (a, b) k is the slope of 1hc P- V cro-.~ sec1ion
shown above.

Similarly, as Figure 26b shows, the tangent line to the intersection of the surface
and the plane x = <I at the poin1 (t1. b) can be expressed as
.:

These two vectors define a plane called rhe ran gem plane 10 the surface:: = f (x . .r)
at the point P = (a, b, f (a. b) ). The imit nomial 1-'ector to the surface al P is given
by

j k
U XV k
n=---= - - - 0 fx(a' b)
uxvl luxv l
0 /_v(a,b)
(4) .{

f-.:(a. b) I+ !_1.(a, b)j - k (a)


= - - - - - - - - - 1-2
[! + J!(a. b) + J,.(a. b)] l
·' .

Note that the direction of n is not specified by Equation 4: in 01her words. ±n arc
both unit normal vectors.

Example 4:
Detennine lhe ve.ctor normal to the spherical surface .r 2 + y 2 • z2 = a 2 at k
lhc point (a/-./3. a/../i. a/.Jj).
SOLUTION: Start wilh J y
(b)
Figure 6.26
Then (a) 1nc tangcn1 line 10 the intersection
off the surfa~ ;: = /(,l. y ) ,in the plane
a .fia) = -(aJj' -/3a) = -I
l.r ( ..n· }y
y .,; b al I.be .P<lint P = (a. b. /la. b)), is
=
parallel to the vector u i ' /.(a, b) k.
(b) The 1angen1, linc- 110 1J1c iri1c:rst..--c1ion of
and so the surface ;: = /(x , y) in the plru1c .x = a
-l - j - k al the point P - (a, b. /(11. b)) is p:irnllcl
n=---- 10 Lbc vector v = J1+ [ y(a, b) k .
./3
gl
248 Chapter o/ Functions of Several VariablE'!>

l z This vector poinL'- inward from the spherical surface at the point
(a/ JJ. c,;..Jj_ a/ .j'j), The outward pointing vector is given by (Figure 6.27)

i+j+k
n=-....;;__-
J3

)'
=
The tangent plane to rhe surface at a point P (a. b. J(a. h)) is nonnal ton.
If we let r p be a vecror to I.he point P and r be a vec1or lo any other point in the
tangenl plane. then r - rp will lie in rhe tangent plane and thus,

X ,·
(r-rp)·n==O

Figure 6.27 where n is the unit normal vector al the point P. Using Equation 4 for n. we see
The outward unit normal \'t.'C!Or 10 the that the equation of the langcnl plane at the point P on the surface is given by
=
spherica.l surface x 2 + y 1 + z2 a 2 at the
point (a/./3. a/./3. a/.J3).
(x - a)fr(a, b) + (y - b)fr(a. b) - [z - J(a. b)l == 0

or by
z
z = (x - a)fr(a. b) + (y - h)Jy(a. h) + f(a. b) (5)

Example 5:
Find the tangent plane to the spherical surface described by x 2 + y 2 + ;: 2 = tJ 2
)' al the point (a/ ../3. a I ../3. a I ../3).

SOLUTION: We found in Example 4 that

so 1he equation of the tangent plane at (n / ./3. a/ J3. a/ J3) is (Figure 6.28)
Figure 6.28
Th~ tangent plane ro the spherical x+y+z=.J3a
=
surface x 2 + y 1 + z2 a 2 at the point
(a/./J. a/./J. a/../3).

You should realize I.hat fr and /" Lhem~lves can be functions of x a.nd y.
For example. if f (x. y) == y2~. then i.r =
y 21;r and fr= 2ye.r. Therefore. we can
form pani al derivatives of fr and / 1, just as we did for J (x. y). The second parti a I
deriva1ivcs off (x. y) are

(6)
6.3 Partiill Derivatives 249

Example 6:
Show that V(.x. y. z) = (x 2 + y 2 + z.2)- 112 !.alisfies the equation

everywhere exccpr for 1..hc origin (0, 0. 0). Thi~ equation is called
Laplace·s equalion. Among other things. Laplac:e· " equation dctcnninc~
the elcct.rost.atic potcnt.ial V (x, y, z) in a charge-free region. Notice that
V(x, y, z) = (.r 2 + y2 + 22)- 112 = 1/r. the Coulomb poten1ial (wi1hin a
multiplicati,,c const.anl) due to a charge situated at the origin .

SOLUTION :

X
V.r =

Because V (x. y, z) is symmetric in x. y. and z.. we can ob1ain Vn, by


interchanging .r and y in v.r.r and we can obtain V: : by interchangi~g x and
z_ Therefore.

(2x2 _ yl _ 22) + (2y2 _ .x2 _ z.2) + (2.z.2 _ x2 _ y 2)


V.o- + v,.'. '" + v== = ~ ., ., s i
(x~ + r + ;:-) /

=0

The types of derivatives in Equation 6 are similar 10 1he second deriva1ive of a


function of a single variable. but func:t.ions of more 1han one variable admit mixed
partial seco11d derimtives as well:

(af)
i)

ox a_\· = aa"
a21
X )'
=f1•.r

(7)
a a2J
aY (:~) - --f
- Jyrlx - xy

Example 7:
11
Find fJ.-.r- f."r f.rr and /,·.r if f (:r. y) = xy 2 + e-<- . ••

SOLUTION:
250 Chap1er 6 / Funcrions of ~ewr.:al Variables

- .., .
f.xy - -} + 2x(I + x 2 yk(~_\' /_1·.r = 2y + 2x(I + X 2y)e- r~Y
.

Notice that J.ry = fi·x in Example 7. Do you think that it is a coincidence?


Let ·s evaluate fx/0. 0) and f.\'x(0, 0) for

(x. y) -:/=- (0. 0)


(8)
(x. y) = (0, 0)
To evaluate fx_..,(0. 0) and f.,.,x(O. 0). we use Equations I and 8, which in 1his cai;e
give

and

so we need fr(x, y) and fyfx. y) for (x. y) -:/=- (0. 0). These tum out to be

4x 2 y3 + x"'y - y5
f.Jt. y) = 2
(x + y)
2 2

x5 - 4x·\•2 - xy"
JJr_.
.
y) = (x-
., + y .,- ) 2
Therefore.

. _ fx (O. 6 y ) - f x(O 0) - 6y
f ,r\' (0 • 0 ) = I1m - - - - - - -
) o 6.y
= "-'1·1m o - - =-
6.y
I

. f.
1.(tl.x,0)-Jy(O.O) . 6x
J,._A0. 0) = l1m - - - - - - - = llm - =I
• 6x->0 D.X lu--+O 6x
We see Lhen that f.ry(O. 0) i- f,.,r(O, 0) in Lhis case. Whai's the difference between
the bchaviors of j(x, y) in rhis example and J(x. y) in Example 7? We'll answer
Lhis question wilh a theorem:

If fry and Jy.r are continuou.f at a point (a, b). then fty = fn at (a. b):
otheni>ise f,_r 0110 fv.r may 1101 be equn/.

[I turns out that /ry


and /y.r are continuous for all x and y and so f.ry fy.r in =
Example 7. but Lhal neither f.ry nor f-;x is continuous at (0, 0) for Lhe function
6.J P.1rti,,I Derivatives 251

defined by Equation 8 (Problem 14). Fortunately, most of 1he functions that we


deal with in physical applications are continuous.
The equolity of mixed partial second deriva1ives is used often in thennody-
namics. For example, lhennodynamics tells us 1ha11he entropy (S) and Lhe pressure
( P) of a substance can be expressed as pa.rrial derivatives of a function called the
Helmholtz energy, A = A ( V. T). which is a function of the volume and the tem-
perature:

S--(oA)
ar v
p =_ (rlA)
av.,
(9)

Using lhe relation

a~:T = [a°v (!;) Js = [a~ (!i) Jv


a2 A
= aTav
we see thal

Equation IO is known as a Max well relation in thermodynamics. The derivation of


Equation 10 is a typical thermodynamic manipulation. Equation I Ois an important
and useful equation because ii allows us to calcuare Lhe entropy of a substance
(which is not a directly measurable quantity) in 1enns of the pressure-vo\ume-
temperature (P-V-T) dependence of the substance. which is readily measumble.

6. 3 Problem
I. Determine aJl the partial derivatives up Lo second order of J (x. y) =

(b) y sin x + .x 2
2. Dcrennine .all the partial derivatives up 10 second order of j(.r. y) =
(a) l.rul-1 !...
X

3. Show that /£y = frx for (n) J(x. y) = re- ' )'l
and (b) e->. cos xy.

4. Show that f.ur = f:1y.r = fyn for J (x. y) =


(a) x 4 y 2 + xy

S. Show 1hat if (a) f(x. y) = ln(x 2 + .v2) witb (x. y)-:/:- (0. 0). then fxx +In= 0. and if (b) f(x, y. 2) =
+ + wi1h (x. y. z) # (0. 0. 0). then .xf..,.= = Yf:.x = zf.Q,.
ln(x 2 y1 z2 ) ··
6. Show that .x/r + yf>' = 2J if J (x. y) = xy tan(y/x). x -::f. 0.

C gl
252 Ch,1p!t ir 6 / Funr rion~ of ~t·\HJI Variables

7. Show thal c(x. r) = (4n Dr)-ll~e- -' 1 ~, .4


D,
.
sansfies the equatrnn -
. ik
=
a2c
D--i .
a, ax-
2
8. Show that c(.x. r) = -1 [ I + e rf ( xr;:;-: )] satisfies
. _ the equa11on
. -;--
ik = Da-c •
2 2...,0, or ax-1

9• Show !hat f( x. y ) = sin. ax sin. h ay sat1sfie,


. - La I . . a2 f a2 r o
p ace s equalJUn -.. -, + ~ = .
ox- or
10. Shuw that Y(0. I/>)= e±i4J si.n 0 c~0 s::n.isfies the equation

sin 8 i_ (sin 0 aY) + a:?~ + 6 sin! 0Y(0, </>) = 0.


ao ao a<1>-
11. Find tJ1e equation of the tangent plane to the surface " = xy + 3y~ at the poini (I, I, 4 ).
12. Find the equation of the tangent plane to the surface z = x 2 /(x + y) at the point (2. 2, I).
13. Find the equation of the tangent plane 10 the elliptic paraboloid z = 2x 1 + y 2 at lhe point (I. I. 3).
4
x6 + 9x-'y2 - 9\''.!y - ~• 6
14. Show that J.,, 1• =
.
,
(.r- + y )-1
;
.
· = fi•.r for (x. y)-::/=, (0. 0) for the function defined by Equation 8.
hut that /.:ry and fvx are nol continuous al (0. 0).
xy

!
(x. y) 'I- (0. 0)
15. Show that f(x, y) = x-
1
+ y- 1
is an example of a func1ion where bolh firsl partial
0 (x. y) = (0. 0)
derivatives exist al (0. 0) bu1 rhat j(.T, y) is discontinuous there. Contrary 10 tJ,e case of functions of a single
variable. 1he existence of first derivatives is nor sufficient 10 guaranrec continuity.

The rrexl five problems i11l'o/ve applicarions of partial derivalives to rhemwdynamic:.~.

16. The isolhermal c.:omprcssibility. K 7 . of n substance is defined as Kr = _ ..!_


V
( i-l
BP
V) .Ob1ain an expression for
T
the ii-olhermal comprcssibi lity of an ideal gas ( P V = RT) ia terms of P.
The coefficient of 1hermal expansion. a, of a substance is deli ned as rt = _!_V ( uV) .Ob1ain an expression
rJT p
for Lhe coefficient of lhcrmal expansion of an ideal gas ( P V = RT) in tcnns of T .
17. Given 1ha1 U = k8 T-., (cJ- .In- Q), where Q(N. V. T) = -
I (2rrmk,, 8 T)JNJ'}. V N and k8 . m. and h arc
ar N.v N! 1i-
cons1an1s. determine U a::; a function of T.

18. The 1hermodynamic equation ( ~U )


av ' T
=T (
0
iJT
p) - P shows how 1he energy U of a system varies with the
V
volume in terms of pre, s ure. v•olumc, and (kelvin) temperarnre of the system. Evaluate (oU ;a V)r for an ideal

gas ( P V = RT) and for a van der Waals gas [ ( P ; 2


) (V - b) = RT]- where a and bare constnnts.
.
19. Given . at con.,;tanl vo I ume .,s dc fi ne d bv C v
Lhat the heat capacity = (;JU)
- .
and given 1be expression 1n. .
' aT v

Problem 18. derive the equation ( ac


av
v) r = T ( az ~) .
ar v
20. Thermodynamics tells us that the difference between 1he heal capacii-y at constanr pressure and the heat C::'.lfWCity
at constant volume is given by C r - Cv = T ( ~ p) (0 V) .Show Lhat C p - C ,; = R for an ideal gas.
aT v ar P
6.4 Chain Rules for Partial Oift(•M11i.11ion 253
6 .4 Chai n Rule for Partial Differentiation
Suppose that 11 = f (x. y) where x and y are functions of a single variable,. Then
1he composite funct..ion f(x(t). y(/)) = U(I) is a function of a single variable,
and we shall prove that

dU
-=--+--
au dx au dy
dr ox dt oy dt
provided 011/ox and 011/ay are continuous and .r(r) and y(r) are different.iable.
Equal.ion I is called lhe chain ,-11/e of partial differentiation. Before we prove
Equal.ion I. we should say a few words about the nolation. First nolc lhal we
wrote f(x(t), y(t));:::, U(t) instead of simply 11(1). We did this to empha~ize that
"= =
f (x, y) and U f (x ( t). y(,)) are ac1ually di fferen1 func1ions. For example.
=
if 11 x 2y and x = ,e-1 and y = e-ll, Lhen U = 12e- 4 '. Many aulhors don"t make
this distinction, nor shall we mosl of the time. but ii is occasionally helpful to keep
the distinc1ion in mind.
Equation I i). fairly easy to prove. We ac.sume 1ha1 x(t) and y(I) have finite
derivatives so that a change in t produces changes 6x llJld 6y which 1end 10 zero
as 6r ➔ 0. The changes 6x and 6y produce a change in 11. and so we write

611 = f(x + 6x, .r + 6_y) - f(x. y)

= (f(x + 6x, y + 6y) - j(x, y + 6y)I + lf(x. y + 6y) - f(x. y)] (2)

where we have simply added and subll1lcled j(x, y + 6y) to the first line. If fx
eitists and is finite in a region of the .xy-plane conl.ai.ning Lhe point (x(r), y(I)).
Lhen the mean value theorem for derivalives (Sectjon 1.6) says that

where~ lies in the open interval nx. Similarly. if f~- exists and is finite in a similar
region. then

f (x. y + 6y) - j(.x. y)::::: /)x. 11)ny (4)

where 11 lies in lhe open interval 6y. Combining Equations 2 through 4 and dividing
1hrough by 61 gives

(5)

Now as 61 - 0. 611/ 61. 6x/ /:J,.t. and 6y/ 6.t tend to du/dt. dx/dt. and dy/dt.
f
and if fr and 1• are continuous. then

d11 au dx dll dy
- =-
dt ax -
dt
+-
ay -
dt
(6)

gl
254 C. h.1 pi<' r b i ~ unt I ic m, c1f Sever a I Variables

Example 1:
Use Equation 6 ro evaluate d11/d1 if u = x!y + xy 2 and x(t) = ,e-r and
y =c-'.
SOLUTION:

au = 2rv + }' 2 a,, ,


-ax . . -
ay
=x· + 2xy

dx l ) _, dy -I
-=( -le -=-e
dt dr

and so

-d11
dt
.,
= (2.rv- + .y·)(l - l)e
-r
+ (x·, + lxy)(-e -, )

= f(l + 2r)(I - l) - 1(1 + 2)Je- 3'


= (I - t - 3r 2)e-J1

Of course you get the same result by substituting x (,) and y(t) into 11 (x. y)
and then differentiating wilh respect 10 r (Problem I).

Equation 6 is readily extended 10 a function of more than lwo independent


=
variables. If 1, = u(x 1, x 2••.. , x 11 ) and xj xj(t) for j =Ito n. then

(7)

Suppose now Lhat u = 11 (x, y) and that y = y(x). so that u is actually a function
of a single variable x. This is just a speci fie case of Equation 6 with r = x. and so
we have

du au au dy
-=-+-- (8)
dx ax oy dx
ln Equation 8. du/dx represents the total derivative of u with respect 10 x while
(uu/8x) represents the partial derivative of" with respect to .x.

Example 2:
Use Equation 8 to evaluate du/dx if u = y sin x and y = .1.e x

SOLUTION:

-au = }' cosx au = $1TI. X


-ay
ox
6.4 Chain Ruic·, ior Parti,11 Oifftn•n1i.1rion 255
du ~
-dx = .r cosx + (srn. x)(l - x)e-

Of course you gcI 1he same rcsull by subsIi1uIing y = xe-·' into 11 = y sin x
and then differcntinti ng wi1h respect lox.

Now suppose rhal 11 = 11(x. y)


and 1ha1 x =
x(s, r) and y = y(s, r). In this
case u is a function of two variables, s and t. or ,, (x. y) = U (,~. r ). We can ex rend
Equation 6 to write

dU OlldX 0/111\'
- =- - + - ~ (9a)
i-Js ox as ay i>s
and
JU ,)11
-=--+-...:...
ox 011 ih
(9b)
or Jx a, ay a,
We hold r constant everywhere in Equation 9a and s consta.nl everywhere in
&1ua1ion 9b. so EquaLions 9 are essentially equivalent 10 Equation 6. We made
a notational distinction between 11(.r(s. r), y(s. r)) and V(s, r) in Equations 9a
and 96. but you'll usually sec them wrincn as

c.Ju 011 i:lx au cJy


-
Js
=iJx
-- OS
+ay- OS
- ( IOa)

and

il11 ifo Jx au ov
-=--+-~ ( IOb)
a, ax a, ay a,
When II is differentialed with respect lo s or r. then u is rcgartlcd to be a runc1ion
of s and r and the other variable is held constant during the partial di1Teren1ia1.ion.
When II is differcnliated with respect 10 x or y. then II is regarded 10 be a runction
of x and y and !he other variable is held cons1an1 during partial differcntia1ion.

Example 3:
If u(x, y) = ye-.r + xy and x (x, r) = s 2t and y(s, 1) =e ·' ··I r. e\'aluate
011/as and ou/at.
SOLUTION:

Ju
-= -011 a., + -_..:a.
011 av
dS1
0.\ dS a,_\' as
= (-_w,-.r + y)(2s1) + (e-· + x)( - e- .r)
1
256 Chapter 6 / Functions o( Several Variables

a,, ai, a.x a11 ay


-at =-ax -+
a, --
ay a,
= (-y e- .r + y)s 2 + (e - x + x)

.\'
Example 4:
The expression

is called the Laplacian opcra1or (in two dimensions) and occurs throughout
applied mathematics. The Laplacian operator above is expressed in c.artesian
coordinates. we·re going to study various other coordinate systems in
Chapter 8. but you might remember from other courses that it is somct.imes
X
convcnicnr to use po/or coorrlinotn. where x and y are expressed in 1em1~
of r and fJ in Figure 6.29.
Figure 6.29
A point in a plane may be s-pccificd by its x = r cosB y = r sin 0
disumce from the origin (r) and the angle
Lha1 :i line from the origin 10 the poin1 Take the special case where r = o = constant and express v' 1 in 1enns of 0.
makes with r.he x axis (0). The quantities
r 11nd Oare called polar coordin::itc.$. SOLUTION: We first lei v' 2 operate on a function /(:c. y) and write

Now. using Equa1ion.s 10 gives

Bul 0 = Lan- 1(y/x). so (a0J8x),. = - sin 0/a, and

Using Equations I O again. we write


6.4 Chain Rules for Panial Di Elt•r1•nria1ior1 257

sin 2 0 ;3::! f sin 0 cos 0 of


=---+----
a2 ao 2 a2 ae
The result for a2 f /oy 1 is obIa.ined in a similar manner and yields

and so we find thal

We can express this result in operator form:

Equations I O are easily ex1ended lo any number of variables. If 11 = u (x 1•


x 1 , ...• x,,) and xj = xls 1• s2 ••••• s,,,). then,, is a func1ion of s 1• s 2 ., ••• s,,,
and

(II)

Becau.se k = I. 2..... m. Equa1ion 11 consists or m equations.


There is a theorem called Euler's theorem Lha1 is extremely useful in l.hermo-
dy namics and a number of other fields. First we defi nc a homoge11eo11s function of
degree pas one 1hat has the property that

( 12)

where ). is a parameter. For example . .f (x. y . .:) = x 2z + y z2 + x y z is homoge-


neous of degree 3 i.ince

f(i-..x. ,\y, J..z.) = (Ax) 2 (AZ:) + (Ay)(>,.:) 2 + (),_x)(>..y)(,\z)


= >.. 3(x 2z + yz 2 + xyz) = ), 3J(x, y. z.)
Not every independent varinble has to appear in Equation 12. The function
j(x. y, z. w) given by
X) '
f(x. y. ;:. w) = xy sin z + -e-u·-
)'

is homogeneous of degree 2 in the independent variables x and y because


2
J(>..x, >.y) =). j(x. y) (I 3)

gl
258 C h,1ptt•r b I Funt 11c111~ llf ~•vt·r,1! Variablc,s

The independent variables l and ware simply suppressed in Equation 13.


Euler's theorem says lhal

If J().x. ,\y) = )..P J(x. y). then


ar
nf + v....:_
pf(x. ~•)
. = x....:._
ox - Jy (14)

The proof of Euler's Lheorem is easy. S1ar1 with

f(h . .\._v) = ).'' j(x. y)


and let u = ,\.x and v = .>..y. Now differentiate both sides of

f ()..x, ).._r) = ).P j(x. y)


with respect 10 A to get

aJ dll
--+--=pJ...
uf au ,,-If(X,\')
011 a;i. au a). .
Bui J11/,-J)..:::;:. X and avjiJ'A.::::: y. so

I ;Jf iJf
p>..P- ((x. y)
. . = xau- + .,._
av
Because 1his equation is true for any value of)., it is lruc for).. = I and so 11 = x
and u = y and

We' II illustrate Equation 12 wilh a ··mathcmnlicaJ" Ex:ample for 1hose who have an
aversion to 1hermodynamics for one reason or another. bur Problems 15 through
18 illusrrarc some thermodynamic applications of Euler's theorem.

Example S:
. .r) - 1111 . .
Show that f = y s1.n z. + -e .. a11sfies Equation 12.
_y

SOL Ul 10 N: The runction f is homogeneous of degree 2 in the two


independent variables .r and y:
6.4 Chain Rul('S ror Parrial DiiinL-11ti.11ion 259
of aj . Jx 3 - wl . Xj _,,,1
x-+y-=..1.,·,1n;:+- e +x\·sm z- - e
i:lx uy . y - y

=2J
1
Note that sin;: and ,:,-u- play I.he role of multiplica1ive conslams in each
rem, of JU, y. :. w).

6.4 P o bl ems
l. Verify 1h:11 you gel 1he same resul! for Example I if you ..;ubs1itu1e x(I) and y(t) into 11 and !hen differen1ia1c
with rcl>-pcct 10 r.
2. Use 1he chain rule to evaluate du Jdr if u(x. y) :::::: cos(x 2 + 2y) where x(I) = r and y(t) = t 2 .
3. Use rhe chain rule to evaluate du /dt if u (x. y) = _\'l' '? where x(I) = r1 and y(r) = ,~.
4. U~ 1hc chain rule 10 t:valualt: df/dt if ftx. y) == t' 'J whern x(t) =sin, and y/1} = cos./.
5. Use lhe chain rule to cvalu~lc d11/d1 if u(x. y • .:) = + :::.e>" where x(r) = 1. y(r) = 12. and ;:{I)= 13.
x2

6. Use Equa1ion 8 10 evaluate du/dx if 11(x. )') = y cos xy ,md y = t,-.t. Verify your rcsuh by suhs1:itu1ing
y = , . ' into II fin-1 and then diffcrentiating.

7. Evaluale 011/ilr if u(x. y) =x2- xy + y2 and .r = r cos O and y = r sin 0.


8. Eval11a1e au/i:Jr if 11 = (x + 2
= r sin 8 cos</). y = r sin O !-in ,fi. and z = r cos 0.
_r 2 + : 2) 112. x

9. Evnluale J11 / os if u(x. y) = e-•+y_ X =It'''' and_\'= sins.


De1cnni11c 'iJu/rJ.~ al (s.1) = (I. -1) if 11,( x. _y) = .ry - y 2. x = se- and y = .~,t~. 1

11. Suppose that 11::;;:


..
.
ifo
j tx. ,·. s. I). where .r = x(s. t) and v = r(s. I). Show that -
. . ih
= -O8/X -rh
eh
ilf (h
+ _....;.. + -
a,· a
Jf
as
. h' ;31, nJ ox aJ ay of
andt a t - = - - + - - + - .
·
a, i:ix a, ay ilr a,
. . a2,.,
=c-., -.aax--,11 .
2
12. Show th::it 11{x +cl)+ 11(x - cl), where x and r urc variables and,· 1s a constant., satisfies - -
,Jr
. . a211 , J2,, . . .
13. Consider 1hc equatwn - = c· ;i.r2
- . whefe c 1s a consLanl. Dehne new vanabk~ E = x - et and r, = x + cl
0 ,2
and nnw ~how· 1hu1 lhc above cquaLion be.comes - -
az,, = ()_
· aea~
14. If 11 = 11(x. y) and x = r cos 0 and y = r sin 0. then show that

The applicahilily nf £uler ·s 1l1eornn 10 11,rmwdy,rrimic.t rrs/J upon the co,11 ·cpt of mi extensive thennodynami<.:
quamity, 11'/iich i.~ o q110111i1_v rlwr ix dirc.•crly 1m.,pvrrimwl ro tht: siz.,:• of rl,l' system. Exrensi1·c propertie.\· are
\"fJ/umc. mtLU, 1111mhcr of mo/,:•s, cuergy. and e,11rop_,: An intensi,•t> then11ody110111ic q,wnriry. such as
260 Ch.ipre-r 6 / Functions of Several Variables

temperature or pressure, is indepe,idenr of rhe size of !hi' .\_\·.1·1t·111. Fune/ions nf extensive i.;ariable.1· are
homogeneous of degree one. For e..xamplc. rhe l'Olwne of a sysrem depends upon !he number of moles of each
consti/Uent, and so

( I 5)

Physically, if we double the n,110111I1 of each co11.s1iwem, 1Jren Equation I 5 says that we double the vofllme of
the system. The next four problems are applicorions of Euler's theorem 10 thermodynamics.

15. Let r be any cx1ensivc propcny. Use Euler"s Lhcorem 10 prove 1ha1 Y (n 1• 11 2 •.••• T. P) = L j y·J' where
II

Y1 =(rJY/onj)T.P.nt,-,·
16. The 1hcrmodynamic energy (U) of a ~y~tcm can be expressed a'> a func1ion of I.he entropy (S). lhc \10l ume ( V).
and Lhe number of moles (n ). u~ Euler's theorem to deri,.,e U= S( ~ U) + V( ~ U) S,n + ~i:)nU) S.V .
11 (
oS V.n i:)V
Do you recogni1..e the resulling equation?
17. The Helmholtz energy (A) of a sys1em can be expressed as a function of 1he temperature (T), the volume
(V ). and the number of moles (11). Apply Euler's theorem to A= A(T, V. 11). Do you recognize the resulting
equation?
18. Apply Euler's theorem 10 V = V(T, P. 11 1. ,, 2).

6.5 Differentials and the Total Different ia l

)' Line rn Section 1.5. we defined lhe differential of a function of a single variable. We
shall ex lend lhese ideas to a function of several variables in this section. Recall from
at .x Sect.ion 1.5 lhar if dy and ~)' are defined a~ in Figure 6.30 (see also Figure 1.42).
then the difference bc1wcen dy and ~y goes as

~y = dy + € .6X

where e-), 0 as t:u ➔ 0. In other words. rhc difference between dy and ~y goes
to zero faster Lhan ~x as ~x ➔ 0. Because of this.

X dy = y'(x) dx (2)

Figure 6.30 is an excellent approximation. We can readily extend lhese idea-, to functions of
An illus1ra1ion of the di llerencc be1wccn
dy and l'l.y. more lhan one variable.
Let j(x. y) be a function of Lhe independent variables x and y. Now change
x by ~x and y by ny and lel ~f be tlte corresponding change in f(x. y), then

.6f = J(:r + nx. y + ~y) - f(x, y)

= f(x + ~x. y + ~y) - f(x, y + ~y) + J(x. y + ~y) - J(x. y) (3)

Because y + ~y is held const.ant in the first two terms in EquaLion 3 and x is held
constant in Ihe second two 1crms. we can now us.c the mean value theorem for
b.5 Differential~ and the Total Differential 261

derivatives of a single variable in Equation 3 and write

(4)

where x < ~ < x + ox and y < 1) < y + ll.y. Assuming that fx and /v are con-
tinuous. Equa1ion 4 becomes

where € 1 ---+ 0 and € 2 ---+ 0. as both ox - 0 and 6y -► 0. Equation 5 is I.he two-


variable analog of Equation I. It shows I.hat jj,.f -;. df even faster than tu---+ 0
or oy ---+ 0. Thus. we see that

(6)

is an excellent approximation ~ Lh and ll.y approach zero. Equation 6 is the


two-variable analog of Equation 2. We call df the Iota/ differential of f.

hample 1:
Determine E I and e2 in Equation 5 or 6 and show lhar f 1 -. 0 and E2 - 0 as
6x - 0 and Ay ._ 0 if /(;r, y) = x·\ + xy 2 •

SOLUTION:

ll.f = J(x + 6x. y + 6y) - J(x. y)


= (x + 6x/<.v + 6y) + (x + lH)(_\· + Ay)2 - x\ - x_r2
= (3x 2y + y 2)6x + (x-' + 2xy)Ay
+ [3x_r6x + (3x 2 + 2y)6y + y(6.r)~]6x
+ [x6_\' + 3.r(6x) 2 + (6x)(ll.y) + (6.t) 3 J6y
= f.r6x + f_,.ll.y + F1A.r + E26Y
where f 1 = 3.x_,·Ax + (3.x~.. + 2y)Ay + y(A.x)-., and €2 = x6y + 3x(6x)- + ')

(Ax)(Ay) + (6.r) 3. B01h € 1 -;. 0 and€: - 0 a.<: Ax - 0 and fly---+ 0.

If f(x, y) is such that € 1 ➔ 0 and E2 ----. 0 as both ll.x ➔ 0 and 6y ➔ 0, we


say 1ha1 f (.x. y) is diffcren1iable al the point (x. y). Contrary to what you might
gather from Equation I. lhe existence of fr and Jy is not surficicn1 to guaranlcc
that J(x. y) is differentiable; f.- and /\' mus1 also be continuous.
For the rest of this section. we shal I use 1hc pressure of a gas instead off (x, y _)
to illustrate differentials. Generally. 1he pressure of a gas depends upon the (kelvin)
temperature T and the molar volume V (the volume per mole). and s.o we write
P(V. T). Recall that one mole of all gases at sufficiently low pressure obeys 1hc

al
262 h,lp!c r 6 / Fune 11ons o f " •ver.1I V;11iahlt•~

ideal-gas equation of slate:


PV = RT (7)

where R is a constant. whose value is R = 0.0821 litcr • atmospheres per kelvin


per mole. At higher pressures. where devia1ions from ideal behavior occur. the van
der Waa\s equation is often used:

RT a
P=--- - - (8)
V -h V'.!

where a and b are consrnnts that are characteristic of the particular gi.ls. We shall
use these two equations as exi.lmples of functions of two independent variables.
As Equa1ions 7 and 8 imply. the pressure of a ga.-. depends upon its temperature
and volume. The total differential of the pressure for a lixed amount of gas is given
by

dP= "iP) dT+ (r1P)


(~ - dV (9)
aT v av r
We have subscripted the panial derivatives in Equation 9 to emphasize which
variable is varied and which is held constant. PhysicaJly, Equa1ion 9 says that it is
an excellent approximation to colculate the total change in the pressure of a gas
due 10 a l:hange in both the temperature and Ihc volurnc in Iwo steps - Iha1 due
to the temperature chan~t! keeping the molar volume fixed at its initial value and
then that due lO the change in the molar volume keeping the tempernture fixed at
its linal \'alue.

Example 2:
Evaluate the total differential of tJ1e pressure for an ideal gas.

SOLUTION: L",ing &1ua1iun 9. "'-' ,t·c that

dP= ( -aP)
iJT V
dT+ (aP)
-
rJV T
i/V

. (DP)
w11h-
,ff v
= -Ranti
\I
(~p)
-
i) V T
= -RT
-1 MJ
V

R RT
dP=-dT--dV ( I 0)
V \f2

We can U);C Equal.ion 10 to estimate the change in pressure when both the
tcmpcmturc and \'Olumc of an ideal gas are changed slightly. We write Equation I0
for finite 6 T and 6 V as
( 11)
263

Lei's use this equa1ion to estimate the change in pressure of one mole of an ideal
gas if the lemperature is changed from 273.15 K to 274.00 Kand the volume is
changed from 10.00 L to 9.90 L:

R RT
l:l.P ~ -l:l.T - -LlV
V V2
(0.0821 L • atm • K- 1 • mol- 1) .
: : : : : ; - - - - - - - - - ( 0 .85K)
(10.00 L, mol- 1)

(0.082 1L-atm·K - 1 -mol- 1)(273 .15K) OL -1


- -- -- - - - - - -..,- - - - ( - 0 . I • mol )
(10.00 L • mot- 1)~

~0.0294 atm
For comparison, in this panicularly si mplc case. we can calcula1c the cxac1 change
in P from

1 1 ( 274.00 K 273.15 K )
=(0. 08 21L·atm·K- •mol-) . ------
9.90 L • mol - 1 10.00 L • moJ - 1

= 0.0297 atm
You can see that Equation 11 give-. us a good estimate of l:l.P.
The total differential of the pressure of a gas described by the van der Waals
equation is (Problem 5)

R
dP=--dT+ [-
2a - - RT
- - ] dV ( 12)
V-h V3 (V-b) 2

We know that this expression is the iota.I differential or P given by the van dcr
Waals equation. but suppose we are given an arbirrary expression. say

-RT
-dT + [ - RT
- -2 - -a-2 ] dV ( 13)
V - b ( V - b) TV

and are asked 10 de1em1ine the equation of state P = P(T. V) 1ha1 leads 10
Equation 13. In fact. a simpler qucs1ion is to ask if 1herc even is a func1ion P( T. V)
whose total differential is given by Equation IJ. How can we 1ell?
If 1herc is such a function P(T. V ). then its IOlal dffferenLiaJ is

dP= (-iJP)
fJ T v
dT+ (:IP)

oV T
dV

Using the fact that mixed second partial derivalives are equal (provided they are
continuous functions of T and V), we have Lhc requirement that

2
aP ) = [ iWa (aP)
( avar ]
ar v r
264 C.h,1ph)r 6 / Functions of Sev~ral V.viables

and
2
a p ) [ a ( aP) ]
( c)ToV = aT av T V
must be equaJ. If we apply this requirement to Equa1ion I 3. we find that

a [ RT a ] R a
aT (V-b) 2 - TV 2 = (V-b) 2 + 72v2
and

a ( RT ) RT
aV V - b =- (V - b)2

Thus. we see that the cross-derivatives are not equal. so the expression given by
Equation J3 is not Lhe differential of any function P(T. V). The differential given
by Equation 13 is called an ine.xac:t differential.
We can obtain an example of an e.xact differe111ial simply by explicitly dif-
ferentiating any function P(T. V). such as we did for the van cler Waals equation
to obtain Equation 12. The mixed second derivatives in Equation 12 are equal, as
they must be for an exac! differential.

Example 3:
ls

( 14)

where Ci and fJ are constants, an e:wct differential?

SOL U T 10 N: We evaluate Lhe two derivatives

and

iJ [ RT a(2V + {3) ] l
{ oT (V - fJ)2 + 71/2 V2(V + fJ}'l f
R a(2V + /J)
= - -(V -- fj)2
-

These deri\'atives are equal, so Equation 14 represents an exac1 differential.


6.5 Difl~r~n1i al:s and the l otal Oiifo. rcnlial 265

We know how 10 tell if an expression is the tolal differential of some runction.


but is it possible lo detennine what I.he funclion is? Let's go back to Equation I 0.
which is the 101111 differen1ial of the pressure given by lhe ideal ga.-. equation of
stale:

R RT
dP= -dT- -dV
V V2

Because Equation 10 is an exact differential. we know Lhal

and ( 15)

Integrate tJ1e first of 1he1-e two equations with respect to T. trealing Vas a constam.
IO gel

RT
P=-+J(V) ( 16)
\I

where f ( V) is an arbitrary function of V that occurs because we integraled


with respect to T, but with V constant. The function f(V) corresponds to the
constant of in1egr.1tion in "'ordinary" integraLion. Now take the panial derivar:ives
of Equation 16 with respect 10 V 10 ob1ain

Equate this resuh lO the second of Equal.ions 15 and get

or

/(\/)=constant

Subs1i1u1c this result into Equation 16 to gel

RT
P =- constant
V

We can determine tJrnt Lhc value of the constant here mus1 equal zero because we
know that the pressure of a gas approaches zero as the volume increases without
bound. Thu~ we see I.hat
266 Ch, pi >r 6 / Funclions ol S ·•v ral Variables

Example 4:
Determine P from the exact differential

dP=-R-dT+[2a- RT ]dv
V - b v:t (V - b)2

where a and b arc con~tanL.;;.

SOLUTION: lntegrnLC

R
V-b

"partially'' 10 obtain

RT
P=--+f(V)
V-b
where f( V) is an arbitrary function of V to be dc1crmincd. Now differcntia1c
P partialJy wilh respect lo V and use the ubove expression ford P to write

RT df 2a RT
---
(V - h)2 +-=-----
dV V ( V - b) 2 3

or
df ill
d\l
= vJ
Integration gives

n
J(V) = - vi +constant

So
RT a
P =- - - ----::;
V - b V-
+ constant

Bul P -+ 0 as V --> oo. so "constant"= 0. and we have the van der WaaJs
equation.

6. Probl ms
1. Delermine '=I and E2 in Equal.ion 5 and show lhal E1 ➔ Oand ~::! -• 0 as D.x ->- 0 and D.y ➔ 0 and lhat ().y ➔ dy
if J(.x, _y) = .r sin y + x~eY.
2. Show lhal E 1 ➔ 0 and E2 -,. 0 in Ec1uation 5 a..~ tu-,. Oand t.y-,. Oand lhat t.y-,. dy if /(:r. y) = <:r cosxy 2.
3. Detcm1ine 1he total differential of (a) /(:r, y) = x:! sin y and (b) ~(u. u) = (11 3 + 11)ev.
xv.,.
4. Dc1em1ine Lhc total differential of (a) f (.r. y . .:) = .r 2y + y 2x + ;: 2x and ( b) f (x. y. z) = ~ .
I+ z~
6.6 The Directional Deriva1ive and the Gradient 267
· d Pi f (van der Waals equauon
S. Detem1me . .
tor one mole) P = -RT- - a
? .
V - b V·
6. Lei ;:(x, y) satisfy the relation F(.x, y. z.) =0 where Fis a ,:on1inuously differentiable function . Show that
aF/a.x d -az. = -aF/ay
-rJ: =----an - - . w1en::
1 :.iF; a~·.. --'-O
a -,- .
;,x i-.lF/iJ::. ay aF;az.
7. Use the result of the previous problem to find J::;/ax and 3::../oy ut the point (I, I, I) for xJ + x y 2;: + ;::- = ).
8. Use the result of Problem 6 to determine J;: irnplicilly for

(a) .r\;: .-:c 1

9. The volume of an ellipsoid is V = rrahc. where"• h. and,. arc 1hc leng1hs of the semia,r;es. Calculate the
1
uncertainty in V if a = b = I0.0 ± 0.050 and c = M.00 ± 0 .050 . Calculate the relative uncertainty.
10. The surface tension of a liquid may be determined by observing how high the liquid will rise in a t.:apillary
tube. For the case of water in a glass capillary, the surface tension y is given by y =
~f).f.!rh. where f> is the
denl-ily of water (0.998 kg • dm-J). g is the acceleration due to gravity (9.81 m • s-:!)~ r is the radiul.- of 1he
capillary tube, and /J is the height that the water rises. Calculate the uncertainty in y if I, 42 ± 0.15) mm =(
and r = (0.35 ± 0.010) mm.
11. Is rrr 2dh + 2,"Trhdr an exact differential? Is rrrdh + :r'1 1dr?
12. Is dx = C v(T)dT + nRT dV an exact or inexact differential? TI1e quantity Cv(T) is simply an arbitr.iry
V
function of T. Is dx / T an exact di ffcrcntial?

13. ,., (2xy + y 2 )dx + (x 1 + 2:ry)dy an exact differential? If it is. detem1ine J(:c. y) to within an addirivecom,tanl.
14. Given that df = 2.r sin y dx + (x 2 cos _y + eY)dy is an exac1 differential. determine f(x. y) 10 within an
additive constant.

6.6 The Directional Derivative and the Gradient


Consider some quantity such as temperature or electrostatic potential that varies in
space. which we express by wriling w = J(x. y, :) . The various partial derivatives
of j(x. y, .:) sho\Y how w changes as we change one of the independent variables
while the other two are held constant. For example. ('Jj/ux)-" ..~ expre~ses how w
varies in a plane perpendicular to the.,· axis . We wish to generalize these ideas and _\'
be able to detem1ine how 11.1 =
J(x. y. ::) varies in an arbitrary direction. rather
than just in lhe x. y. and z directions s.eparately. Consider some point (x0 • _v0 , z0 )
and a unit vector emanating from that point (Figure 6.31 ).
The uni l vector is specified by its three direc1ion cosines. a cos et. b cos /J. = = I
I
=
and c =cosy. and we write the unit vector a,;; u a i + /, j +ck . The parametric .r /
equa1ions of the straight line coincident with u are
Figure 6.31
A unit vector cm:rna.ting from a p1>in1 on
x = x0 + as a surface z = f (x, _1') . The uni I vtttor is
y =Yo+ hs (I) spc<.'ilic<l by ii · three dirtttion cosines.
;: = .Z:o + CS a = cos a , b = t·os fj, and c = co" y .
268 Ch.1ptc r 6 / Fm1cr1ons of ~·\·er.JI Vc:1r-iabl :s

where -oo .:'S s < oo. The potential now is a function of a single variables, and
the derivative off ( x, y, z) in the direction of u is given by I.he chain rule:

df of dx of dy c>J dz
-=--+--+-- (2)
ds ox ds oy ds oz ds
where all the derivatives in Equation 2 arc evaluated at the point (x 0 , y 0 • z0 ). The
right side of Equation 2 can be written as the dor product of Lwo vectors:

df
-=Vf-u (3)
ds

where

11f . aJ . af k
VJ =-•+-J+- (4)
ax ay az.
is evaluated at the point (.\· 0 . J'o, z. 0 ) and where

u=ai+bj+ck (5)

The derivative df /ds is caJlcd the directional derivative of w =f (x. y. z) in the


direction u. Note that if u =I.for example. Lhen the directional derivaLive is simply
(af j,J.x) 1••••. Thus. the directional derivative is an extension of paniaJ derivatives
such as (af/ax)_..,,: to an arbi1.rary direction. The vector in Equation 4 occurs
frequently in physical problems and is called 1.hc gmdiem veuor of f(x. y. z.) or
simply the gradie111 of f (x. y. z). The gradient of J is often dcno1ed by grad f.

Example 1:
Find the gradient of /(:c. y, ;:) = x2 - yz + x.: 2 al the point ( I. I. I).

SOLUTION: We use Equation 4:

grad f= V f = aJ i + iJJ j + rJ/ k = (2x + ;;;2) i - d + (2c: - y) k


ax a_v a:
At the point ( I. I. I). we have grad f = 3 i -j + k.

Example 2:
Find the <.lirectional derivative or f (x. y. z) = xy 2::i al the point (3. 2. I) in
tJ1e direction of the vector,.::; -2 i - j + k.

SOLUTION: The partial derivatives of .f(x. y . .:) arc


b.6 The Dircclionnl De:rh, Ii c ,md the Giadi ·n.1 269
and so

gnid f = V f = _v2:: 3 l + 2.ryz 3 j + 3x/z2 k


The unit veclor u in the same direction as v is
v -2i-j+k
U=-=----
!vi J6
Using Equat..ion 3. the direc1jonnl deriva1ive is

-8 - 12+36 16 (1
= J6 = ./6 al .. , 2• I)

Example 3:
Find the dirccrionnl derivative of </)(x. y. ::) =.rJ + 2.ry 2 y.: 2 from the
point P 1=( =
l. 2. I) I0\\1:ird the point P2 (-1. 0. I).

SOLUTION:

IP..- ( I. 2, I) = I I:
The vector from P1 IO P2 is

v=(-l- l)i+(0-2)j+(l- l)k=-2i-2j

and 1he unit vector in the direction of\' is

v I . I .
U=-= - - 1- - - J
!vl Ji. Ji.
Therefore.

-dip
ds
= v'4> · u = (II i + 9 j + 4 k) · (-i-j)
~
~2

II 9 20
= - Ji - Ji. = - Ji. at ( 1. 2. I)

The gradient vector VJ ha." an imporunt interpretation. If 0 i~ the angle


between V f and u. then Equation 3 says that

-df = 'v f · u = IV f I co~ fl (6)


ds

al
270 Chapter b I funclions of ~ewr,ll Variables

because lul = I. The maximum value of cos (I = I. and this occurs when u =
V f/lV.fl. Thus. we see IJ1at the maximum value of the directional derivative
occurs when u points in the same direction as V f. orlhar VJ points in the direction
in which f (x, y, z) increases mosl rapidly, and 1hc rate of increase is given by the
magni1ude of V f. Similarly. -V f points in the direction in which f(x, y. z)
decreases most rapidly.

Example 4:
Suppose that !he: tcrnpcrnturc T throughout a body varies as T (x. y. :) =
I 00 + x y:. Find the maximum rutc of incrc.a.-.c in tcmpcraturc (with n:spec1
LO disrance) at rhc point (I, I, 1) and the direction in whii:h it occurs. What·s
the direction of 1he most rapid ra1e of decrease in lemperature?

SOLUTION: The maximum rate of incrca~ in tempcraiurc at the point


( I. I, I) is in lhe direction of the gradient at that poinr

'vT=yzi-:-- .r:j-xyk
=i+j+k m(l.1.1)

=
The magnitude of the increase i~ given by l'vT I v'3. The direction of 1he
most rJpid rJle of decrease in 1empcrature is given by - 'v T. or -i - j - k.

We can give a nice physical interpretat.ion to the gradient of a function. Recall


that the level curves of z.:;;; j(x, y) are the curves z = constant in the .xy-plane
(Figure 6.32) . If we consider these curves 10 be expressed by the parametric
equuLions x(/), y(1), then the 101al d.ifferealial of j(x. y) = c al point (x 0 . y 0 ) =
(x (1 0 ). y(l0 )) is

Figure 6.32
df:;;;(~ 1)
tlx ( (-dx) (af)
+ - (d\')
~ =0
-Yot dr 'o 8y I l> •Yol d1 'o
A set of level cUJ"\'e;,; (coior) for rhe surface
- = f( x. y) and the pnlh of 'v I. which which says thar
follo ws the direction of ~lc.cpc~t. dcs.c c:11 1.
(bl:1ck).
"f
V = - (aJ) .1+ (aJ)
- .
j
ux (.ro-."ul oy {X(1,Yu )

and

v= (d-d1x) .1+ (dr)


ro d1
J. ~
io

are orthogonal al the roint (x 0 . y 0 . z0 ), provided V f -:;': 0. The vector v = tlr id!
is tangent 10 the level curve j(x. y) =,·,however, so 'v.f is normal to the level
h.6 Tiu· i1< 1im1.1I Dcrivaiive and 1h • Gr.:1di1:111 271

curve. The path trJced out by V f in Figure 6.32 is normal 10 each level curve Lhat
it cros~c~ ;md follows Lhe direc1ion of steepest des,.:ent. For a set of equipotential
curvL'S. -'il f represents 1hc corresponding clec1ric lield and deno1es the path thal
a charged panide will follow.

Example 5:
The elcc1ros1a1ic po1cn1ial produced by 11 dipole moment µ lu<:alcc.J :.11 the
origin mid directed along I.hex axi~ is given by

(x. y . .: f=. 0)

Derive an cxprc).sion for 1hc dcctric field E associa1cd wi1h thi:,.: potcniial.

SOLUTION;

Figure 6.33
The cquip.o1c:n1i11ls (<:olorl and the clcclril'.
Figure 6.D ~hows a set of equipo1en1ials (color) ancJ the force field (black) field ( hl:K kl nf 11n electric dipole fom1cd
given by E. by equal anJ opposi1c l'11:1rgc • .

Before we finish this section. we will show how to find a vector nonnal to 11
surface at some given point on the surfucc. we·re going 10 see in the ncxl clrnp1er
1hat 1he flow of a ;, uhstance or ,1hc llow of energy ;,is heat through a small surface
area ,c an be expressed in terms of a unit vector that is normal to the surface area.
Let the surface be given by J (x. y. :: ) = c. which we. may consider to be a :,;pccial
=
case of F (x. y . .: ) 0. By an ex tens.ion or our argument that VJ is nonnal 10 the
level curres f (x, y) = constant. we find thal V f (x. y . .: ) is no1mal to the surface
F(x. y . .:) ;:;:: 0. The un,i t nom1al vector n 10 the surface is given by

'v F
n - (7)
l'v FI

Let' ~ use Equal.ion 7 10 del.arninc the unit vector I.hat is nonnal IO lhe spherical
surface described by x~ + y ... +. ~;; : a 2 at the point (a/./3. aJ./3. aJ./3). The
normal vector is given by

'il f = 'v(x~-!- .r2 + .:2 _ a2)

= 2x i + 2y j + 2.: k
2o . u1 j 2o k
= v3r.;-•+ v3
r, + r-.3'
v.J
272 Ch,lpll'r & / Fum lion~ 11f Several VariJbleS

and the unir normal vector n is

VF i+j+k
n=--=---
IV FI .fi
in agreement with our result in Example 6.3.4 (Figure 6.27).

Example 6:
Find the equation of the r:mgcnt plane 10 the spherical surface described by
x2 y 2 :: 2 = a2 at the point (a/./3. a/./3, a/-/3).

SOLUTION: If r 0 is a vec1or from 1he origin to the point (x0 , y0 , z.0) and
r is a vecror to any point in the tangent plane, then r - r 0 lies in rhe 1angen1
plane. The tangent plane (x0 , y 0 • :: 0 ) is normal 10 1he gradient at thal point,
so we have

(8)

The gradien< of x 2 + y 2 + 2.1 = a~ at (a/./3. o/J°J. a/.JJ) is (2u/v'3)(i +


J+ k). so Equation 8 tells us that the equation of the t.1ngent plane is given by

2a (i
. .13
+j+ k) •[(x - .!!_)
v0
i. + (v - ..:!..... ) j + ( :- - !!..._) k] = 0
. Jj Jj

or

in ngrccrnenr with the resulr rh,ll we obt:.iine<l in Example 6.l5 (Figure 6.28).

6.6 Problems
I. Detcnnine 'v<J> for ,t,(x. y) = x 1y + 3xy 3.
2. Determine VtJ, for ,p(x. y. z) = sin xy + .rye~.
3. Find the directional derivative of (/>(x. y) =,/cosy at the point (0. 0) in t.he direction of u = I.
4. Find Lhc directional dcriva1ivc of rJ> (x. = sin x cos y al tJ1e point (TT /3. - 2n /3) in the direction of v = I -
y) j.

S. Find 1.he directional derivative of r/>(x. y. ~) = xyi: at the point ( 1. -1. 0) in the direc1ion of v = I + .i + k.

6. Find the larges1 dircct..ionaJ derivative of <J>(x. y. z) = x 3 + y 2 + z at the point (0. 0. 0).

7. Determine lhc largest directional dcriva1,ivc of ,J,(x, y. : ) = (x 2 + y 2 )eZ al rhc point ( I. I. 0) .


= x 2 + _r 2 + ;; 2 + 2.r y + 3y-
8. Find the region where Vi/) is parallel 10 the xy-pl::inc if tp(.x. y. ::) 8xz.
9. Use gradients to find the angle between the two families of curve~ xy = c 1 and x 2 - y2 = l· 2 .
6.7 Taylor's Formula In Several Vari.1bb 273
10. Suppose Lhe temperature tJ1roughout a region in space varies as T(x. y, z) == x 2 + y z. Find I.he direction of the
most rapid increase in 1cmpcra1urc.
II. Suppo$e Lhc electrostatic potential is given by t/J(.r. y. z) = 100- x 2 - y2 - z: 2. In what direction does it
increase mos.I rapidly from lhe point (3, -4, 5)?
1
12. Suppose that lhc surface of a hill can be described by::= IOOOe- u i+ )/2<'-IJ_ If you are standing at the poinl
x = 5. y = 10. in which direction should you go in order to descend most rapidly? Whal is your rate of descent?

13. Show that the paths of :::1cepcs1 ao::ccn1 for 1he sunace described by : = 1x 2 + _v2 are of the form y = k:r: 2.
14. The equipotential surfaces about a charge q located at the origin are
tf>(x. y, z) = .,q
4 ,r(; 0 (x-- + y 2 + z2) 112
• where !o is the perm.iuivity of free space. Show lhat Lhe lines of force

arc radial lines emanating from the origin.


15. The elec1ros1atic potential due to parallel lines of charge of linear densities ).. and -). is given by
r/)(x. y. l) = _.._ _ In 2.. where rJ is the perpendicular distance from the line of charge lo the point (x. y, z).
2n-Eu r1
Sketch the equipo1en1ial surfaces and the lines of force.
16. De1ennine the unit normal vector to the surface of the ellipsoid described by 2x 2 + y2 + z2 = 4 at tht: point
(I. I. I).

17. Determine Lhc cquarion of Lhc tangent plane for the previous problem.
18. Dclcnnine the unit normal vector 10 the surface described by .x_vz = I at the poinl (-1. I. -1).
19. Determine the angle with which Lhe line r(t) = (t. t, 1) intcrscl'.b the surface described by x 2 + y2 + 2;: 2 :::: I
in the first octant.
20. Find 1.he equal ion of the tangent line to the surface described by x 2 - Jx y + .,,:! = - I al the point ( I. 2).

2 t. Find Lhc equation of the tangent line 10 the surface described by y + sin x y = I at lhc point (0, I).

6.7 Taylor's Formula in Several Variables

Recall from Section 1.6 that if f(x) is a funct.ion of a single variable x and if
f(x) has coniinuous derivatives F(x). J"(x) . ...• J< 11 + 11 in a closed interval
et. S x S {J. then Taylor's formula wilh a remainder term is given by

x2 -en .
f(x) = f(0) + xf'(O) + - J"(O) + · · · + :._ l"J(O) + R11 {I)
2! /J !

where R,, is the rc:mainder term

(2)

where O < ~ < x.


Now consider a function f (x, y) of two variables all of whose first n + I
partial derivatives arc continuous in a closed region of 1hc xy-planc. Let F(1)
be dcfi ncd as J (x. y) = .f (a + IJI. b + la). where a. b. h. and k arc constants.

C gl
274 Ch,1ptt•r 6 / F1Jn ti ns o( Se r.11 V,rn.ibles

Applying Equa1ion I lo F(t) gi\'c,

(3)

where O < f:i < t. We can now evaluate the Ftjl(O) in EquaLion 3. First of all.
F(O) = f(a. h).Using x = 1.1 + lit and y = IJ + kt. the chain rule gives us
F I (f) aJ -dx + -cJf -d_r = I BJ + k -aJ = hf
= -ddtF = -ax l- + 1''·f 1• (4)
dr a\' dr Jx
. ih . .
1
·

It is convenient lo express Equation 4 in operator noiation:

F'(I) = ( h - iJ
Jx
k- a)
Jy
((x. v)
. -
(5)

in which the operator (/1a/i:Jx + ku/Jy) acts on j(x. y ). At 1 = 0. Equation S


becomes

(6)

We can evaluate F"(O) in Equation J in a similar manner:

F ,, ( / ) =
rJF'ui
- - = J- (11i-lf- + f,. (of)- = d- (If
I r+
kf ,)
dt dt rh i:ly dr -.

=hdf, +kd/1•
dr dt

We usi: the chain rule to evaluate dfJdr and df, / dt and obtain

I
f ·" (1)=1 (afr
- - dx
+ofi - + k· (:Jf.
- dy) - -+dx
-of. d_r)
-
dX dr ay dr d.\" dr a . dt
= U12 frx + hkfry) + (hkfrx + k~ fn.)
-- I~ (
1 • .r.t - /k· j . _I' + k 2 f .\"_I"
+ ?/ I (7)

We can write Equal.ion 7 in upcrntor notation by writing

i-l "/ )2
11
F (r) = ( Ji~ + k-!- J(x. y) (8)
dX O\"

In Equation 8. the square on the operator in parentheses means tha1 1hc opernlor
i) 'j
~
ax + k ay
h- ac1s scq1.1c11tiallr on j(x. \" ). In other words.
. .
275

ri a )2
( li~+k-;-: ( ;i a) ( h~+k----:
a a ) /Ley)
ox iJ., J(x.y)= h~+k-;-:
iJ.\ oJ ,.lx ,)_,-

(Problems I lhrough 7 provide you a liule pr-c1ctice with operator noral.ion if it is


completely new to you.) If we let, = 0 1n Equation 8, we have
2
F ,, (0)= ( h~+k-:-
a
i:h
j-) )
or·
f(x. y) I (9)
(x,)0 =(n,b)

A repeated application of lhis procedure shows that (Problem 8)

If we substitute the..-.e result-. for !he derivat[ ves of F (1) at / = 0 into Equation 3.
we ohwin

f(a + lit. h + kr) = f(a. h) +r a) f(x. y) I


(hdXa- -1- k-_
(})' .l, ,1' )= (11 ,bJ

+- .2!, ( oxa
,-
lr --;:- + k~
a -
ay
)., f(x. y ) I
( .r .y)=(a .h)
(II)
11

+··· +-
1 ( - )" J(x.v ) I
1, -a+ k o
11! ox rJ_,. • (.r •.vJ = (//, b )

where O < 0 < I. Letting, = 1 in Equation 11 finally gives Taylor'sformulafor a


./imc:tio11 of two variables:

f(a + h, b+ k) = j(a, b) + (1i -!!- + k;...) f(x.


ilx rl y
y)I
1.r._1·).e:Ca.h)

+ -2~ ax
iJ )., f(x. \' ) I
I ( hcl- +k-
ih .
( 12)
· (cyl = (a, /,J

+--·+-\ ( ha- + kJ- )n


11 ! 3.x ily
f(x.y ) I x .y) ::s(r, .b)
+Rn

where the remainder is


11 1
Rn= I . ( h -a +k-
i:) ) + f(x,y) I ( l.l)
(11 + I)! ax ih
· Ct. _11 1= u+oh.1,+
.
I. )

where O < 0 < I. In olher words. lhe remainder i$ evaluated somewhere along lhe
line segment Lhat connects (a. b) to (o + h. b + k).
276 Chap1er 6 / Fundions of Sevcr.,il Vari.,hlt•,

If we let n = 0 in Equation 12. we have


f(a + h, b + k) = _((a, b) + hfx(a + 0h) + kfr(b + 0k) (14)

where O < I} <I. Equation 14 is the mean value theorem for a function of several
variables and is a di reel extension of Equa1.ion I .4.1.
Equal ion 12 up to quadratic 1erms is

f(x. )'):;;: f(a. b) + (x - a)fr(a. h) + (y - b)Jyla. b)

(x - a) 2
+ fxx(a, b) + (x - a)(y - b)f.r/a. b) (lS)
2
(y - b)2
+ ---fn(a.
2 ..
b)

where we have used x =a +/randy= b + k.

Example 1:
Expand trr-" about (0. 0) up 10 quadra1ic tenns.

SOLUTION: We"ll USC Equation l 5.

f,(0, 0) = J_..(0. 0) == /u(O, 0) = f, 1 (0, 0) =0

l.n-(0. 0,) =I
Therefore.

t?Y = I+ xy + third order terms a11d higher


Nore that we can obtain 1he same result more easily by just using
e~ = I +.: + z2/2 + • • •. TI1is is not Lhe case for the next Ex.ample, however.

Example 2:
Expand ln(x 2 + y 2 ) abou1 ( I. I) up to quadra1ic 1enns.

SOLUTION: The necessary derivatives arc

2.r
J- - -
.r - .x-2 + y1

2r
f1•. =-,-·-,
x- + y-
2y2 - 2.r2
fu = ,; 2 + r 2 ) 2 and f_u(l, I)= 0
6.7 Taylor '.i. f'orm ula in Sc\ •r;:J'I Va.ri .Jhl • 277

and f_vy(I, I)= 0

4.ry
nnd .f.ry(I. I)= -1

Therefore. using Equal ion 15. we 11 nd !hat

ln(x 2 + y2) = In 2 + (x - I) + (_\' - I) - (x - l)(y - I)+ · · ·

= In 2 - 2 + 2.r + 2y - xy + third order lerms and higher

Example 3:
Expand sin x_,. in a Taylor expansion around a = 11 /2 and b = I up lo
quadratic terms.

SOLUTION: Let Ii= x - a and k =y - b with a= rr/2 and b = I in


Eq ua1ion 15:

j(x. y) = sin xy; f (~. 1) =I

fx (~. 1) =0
r\ = -x cos xy : fr(~- 1) = O
fx.r = - y2 sin xy: fu (~- 1) =-I

f"y ( ~. I) = - ( ~ r
f, _= -x.1· sin xy:
1. fr.1· (f. 1) = -f
. (x - rr/2)'!.
smxr=I- - -- -
. 2
rr~(r -
.
I):?
-
rr (
- X -
,r )
- ( \' - I) + . ..
8 2 2. .

You should be aware tha1 mos1 of 1he CAS cnn derive Taylor series in more
than one variable. For example, rhe command

Series [ Exp [ Sin ( {x+y) ] ], { x, O, 3 }, { y, 0, 3 } ]

in Mathematica gives the Taylor series of e~in (.t" +Y J about the point (0. 0) up to
278 Ch;ip1er (, / Fun :tion. of ver;il V;iri;ible-s

third-order rcrms in x and y:

and the line

Series [ Sin [ x • y ]. { x, Pi/ 2, 2 }. { y, ~, 2} I

gives the result in Example 3.

6. 7 Problems
I. Lei A be f.omc given operator. Perform the following opera1ions:
dl 2
A d~ d
(a) .4(2.x) A=- (b) A(x > A=-+-+3
dx 1 2 dx dx
- -a+ 1 -a a ;J
A<x.r 2 ) A=
A

(c) (d) A(sinxy) A=x-+y-


ox c)y ,fr ay
10 he linear if .4fci.f1(x) -1- r:d2 (x)I = c- 1A/1(x) + r 2 1\f2(x). where ,· 1 ;mu<":? urc (po:--sibly
2. An operator is suid
complex) constants and where all the indicated oper.ltors are well defined. Otherwise. A is said 10 be nonlinca.r.
De1em1ine whether the following operators are linear or nonlincar:

(a) in1cgr.11ion (b) diffen::n1ia1ion


(c) square root (d) take complex conjug:ile

3. The operator ,.PJ1x) means that A acL,; scriucntly on /(:r): in 01hcr words. A2f(.r) = /\(A/(.r)]. Write ou1
the opemcor ;P for A =
d2 d
(a) - 1 (b) - + .r
dx· dx
Hin1: Be sure to include f (x) in A2j(x) before carrying out lhe operations.

4. Evalua1e ~i!.(cos ax)(cos by)(cos e::). where the opera1or 'v 2 = -,.,
a2 + :--
a2
+ a2 is called the Laplaci:in
-;---
ar a_v 2 oz2
operator.
5. Two opera1ors are s...1id 10 comm111e if A.Bf(x) =
BAJ(x) for an arbitrary function f(:r). (As usual, we a,J.ume
1haI 1he inruca1cd operators are well defined.) Determine whether or nol the following pairs of operators
commute:
d d2 d . J J o
(a) - ; -., + 2- (b) mul11ply by x; - (c)
dx dx- dx rlx o.r By
Hint: Be sure to include J (x) in ABf (x) and 8 Af (x). where J (x) is a ~uiIably arbitr.iry function .
2
6. Evaluate (a) ( I - ~ ) .,in .r and (b)
dx
(h.;-
dx
+ k.E_)J
oy
X)). where hand kare constants.
6.8 Maxima and Minima 279
7. In ordinary algebra, (P + Q)(P - Q) = P 2 - Q~. Expand (P + Q)( fa - Q) where the supcrcarats denote
1hat P and Q arc opcra1ors. Under whar condi1ions do we find the ~me result for operator algebra a... we dn

r
for "ordinary" algebm?

8. Show thal F< 3'(1) = (1i }r + f.:. o~r f(x. y) in the Taylor expansion of j(x, y).

9. Expand x~ - xy + y 2 in a Taylor series about (1. I) up to quadratic terms and check your resull by al~l~brn.

10. Expand ,. ' 1' about ( I. I) through quadrntic tenm.


JI. Expand J (x. y) = sin xy about (0. 0) up 10 quadratic terms.
12. Expand f (x. y) = sin x y about ( I. I) up to q uadral ic terms.
13. Expand j(x. y) = )\in x cosy about (0. 0) up to third-order terms and compare your result to the product of
rhc Maclaurin expansion~ of sin x and cosy.

14. Use any CAS 10 verify your result in Problem 10.


15. Use any CAS 10 verify your resull in Problem I l.
16. Use any CAS to verify your resull in Problem 12.

17. Use any CAS to verify your rcsull in Problem IJ.

6.8 Maxima and Min ima

In Section I .4. we discussed how to find maximum or minimum vaJues of a


function of a single variable over some interval. The necessary condirion that J (x)
have an extrcmum al x = a is that f' (a) = 0, as.sumi ng that f (x) is differenl iablc in
a neighborhood of .x =a.Recall that later in Sec1ion 2.7, we used Taylor's formula
10 give a simple. direct proof of the .'it~fjicienr condilions. Briefly. if

and

( I)

and if j(2,r)(x) is continuous in some 8 neighborhood oi a. then

I. f (x) has a local maximum value al x = a if f 2ti) (a) < 0.


2. j(.r) has a local minimum value at x = a if J(lii) > 0.
The proof rest-. upon Taylor's formula., which for Lhis case [ I' (a) =
J"(a) = •••= /(2ti-l)(a) = 0 and Jtm>(a) -:f:. OJ is

(2)

al
6.8 Maxima and Minima 281

for 111 I < 8. But this statement simply says that f (x, h) is a local maximum al x = a
where f(x, y) is considered to be a function of x with y held constant al y = b.
Thus. the condition that a function of a single varinhle be a maximum 3t x = a.
namely j'(x) = 0 al x =a.becomes

fJf
-=- = O ac x = a, y = b (3)
ax
The same argument with h = 0 instead or k. = 0 gives
aJ = 0
-ay at x = a. -,. = b (4)

Funhermorc. the same argument applies 10 the case in which f (x. y) is a local
l z
minimum at (a.b).so Equations 3 and 4 must be satisfied for f(x. y) 10 have a
local extremum at (a, b).The point (a. b) is said to be a critical point of /(x. y) .
Just as in the case of a function of a single variable. however. Equations 3
and 4 are necessary co11ditio11s. but not ~ufficicnr conditions that / (x, _v) be a
local extremum at (a. h). A good example of the fact that Equations 3 and 4 are
y
necessary but not sufficient conditions that f (x. y) be an extremum a1 (a. b) is
=
provided by f(x. y) x 2 - y2 . We see that J~ = 2x and J_1• = -2y are both equal
to zero at the point (0. 0). so Equations 3 and 4 are satisfied. Yet, considered as
l:l function of x with y held cons1an1 at y = 0. f (x. 0) has a minimum at (0. 0)
because fxx = 2 > 0 at (0. 0). while considered as a function of _v with .t held
=
consiant ~t x = 0. f (0, y) has a maximum at (0. 0) because J~.... -2 < 0 at (0. 0). ,· X

Thus. lhe surface::= f (.r:. y) has a maximum in the yz-plane and a minimum in Figure 6.37
the .n:-plane a1 the origin. as shown in Figure 6.37. The critical point in this case The saddle point of:. = x2 - ,..,~ nt the
is called a Jt.uldle point for obvious reasons. point (0, 0).

Example 2:
Find the critical points for

j(x. _1') = XJ + _\'J - X - 6y + IQ


SOL u TIO N : The equa1 ions for the critical points arc

Jf 1
.....:.... = Jx- - I =0 and
ar .,
.....:.... = Jy- - 6 = 0
ax a_,.

which yields 1he critical points x =±I/ ./J and y = ± ✓2. The critical p<>ints
are at
284 Chapler 6 / Functions of Several Variables

Example 4:
Investigate the critical poinL<. of

I '>
f (x. .v) = -x~
2 - .n
•'

SOLUTION: The crir ical points are given by

f X =x-v=O
•• fv=-x=O
and so we see that there is a criLical poinl at (0. 0) . The second partial
deriva1ives evaluated at the critical point are

f,._.. = 0: f :r:_r =- I < 0;


Figure 6.43
The behavior of z. = {x 2 - xy around its Therefore. the critical point is a saddle point (Figure 6.43 ).
critical point at (0.0):

6.8 Problems
The first 6 problems review maxim.a and minima for J1111ctio11s of a single variable.

I. Test f(x) =I - /xl for a ma.'<imum value. Why doe..,;:n'1 the first derivative test apply?
2. Test _f (x) = 1 - x 4 for extrema.
3. Test J(x) = .c" for an extremum in the inlerval (0. I j. Whal about in the open interval (0. l)?

4. Test f(x) = for (1 2 - 1


1)· dt for an extremum in the in1ervnl (-2, 2) . (Do not evaluate the integral.)

5. Test j(x) = x(I - e-1') sin x for an exrremum at x = 0.


6. Show that if J<k>(a) = 0 fork= 2, .. . , 2n and J< 211 + I >(t1) f. 0. 1hen f(:r:) ha..<; an inflection poinl at x = a.
Hint : Use 1he fact that f(x) - [J(a) + f'(a)(x - '1)] is the dirferencc between /(x) and its tangent line at a.
7. Find all 1he cri1icaJ pornL'> of (a) j(x. y) = xy 2 (3x + 6y - 2) and (b) J (x. y) = x.:1 + y.:1 - 2(x - y) 2.
8. Find all the critical poinL-. of (a) f (x. y) = xJ - 4x 2 - xy - y'.~ and (b) f(x. y) = (y - .r 2)(2 - x - y).

9. Classify all the critical points of the 1wo functions in Problem 7.


10. Classify all the critical poinLl- of the 1wo functions in Problem 8.

11. Cla.::sify all the critical poinL<. of (a) j(x. y) = xy anu (b) f(x. y) = x 2 + 2xy + 2y 2 + 4x.
J2. Classify all the crirkal points of (a) f(x. y) = 3x 2 + 6xy + 2y 3 + 12x - 24y and (b) f(x, y) =
x 1 + y 2 + 4x - 2y + 3.

13. Derive Equa1ion 6.


14. Prove that j(a. b) is a locaJ minimum if D(a . b) > 0 and f:u(a. h) > 0.
15. The sum of lhree numbers is I50. What is 1he maximum value of their product?
16. Determine the shape and maxi mum volume of a rectangular para Ilelepiped whose total surface area is A.
6.9 The Method oi l.1~r.mg(' ,'v\ullrplicrs 285

6. 9 Th Method of Lagrange Multiplier


The final problem of the previous section asks )'Ull to determine the shape and
maximum volume of a rectangular parallelepiped whose total surface area is
fixed at some value A. Therefore, you have to maximize V = abc with A =
2(ab +be+ nc) fixed. We can use the equation for A to eliminate a from 1he
equation of V and then minimize:

V= -l,c. ( ~ - he)
h+c 2

Setting both a\I /iJb and i'J V /iJc equal to zero gi ves b = c = (A/6) 112• and 1hen
substiwting this resuh back into the equation for A gives a = b = c = (A /6) 112 •
Thus the rectangular shape that produces the maximum value for a fixed surface
area is a cube.
This problem involves maximizing V with the com1rai111 that A = constant.
The occurrence or the cons1.raim means that a. b. and care not independent. and
so we used the expression for A 10 eliminate one of them in terms of the 01hers
and then treat V as a function of Mo independcn1 variables. In this case, it was
easy 10 solve the expression for A for a in terms of b and c however. ii may not
always be so. In rhis section. we will present a gcner-al. rather powerful, method
10 maximize (or minimize) a function with one or more cons.1raints. This method
is due to Lagrange and is called the method of Lagrange 11111/ripliers.
Suppose we wish to find an exrremum of 1hc runclion /(x 1• x 2 , ... , x,,) of n
variables, where then variables must satisfy some auxiliary condition (a constraint)

(I)

If /(x 1• x~ •... , .r,1 ) is an exlremum al some poini P = (x 10 , x:w, .... x,, 0), then

(2)

at Lhut point. lfthere were noconstr.iint. x 1• x 2 , .••• x,. would be independent. and
so we could vary dx 1• dx 2 • .•. , dx, 1 in Equation 2 independently. For example,
we could let them all be zero except for dx1;, say, and find that of/flxk = 0
fork= I, 2 .... , 11. giving us the necessary condition for ru, unconstrnined
ex1.remum. Because of Equation I, however. x 1, x 2 •..•• x,, are not independent.
so the dxJ cannot be varied indcpcndcn1ly. and of /8xk = 0 fork= I. 2 ..... n
is 1101 the condition for an cxrrcmum in rhis case.
In the method or Lagrange multipliers. we multiply the total differential of g.

(3)

by a parameter (a multiplier) A and subtracl the rcsull from Equation 2 to ob1ain

al
286 Chapref 6 i Fun -ri ons o f Sever, I Variables

Equation 3 may be viewed ns a rela1ion berween then differen1iab dx j• so only


11 - I of the d x i in Equation 4 are independent. We have no1 speci tied >- yet. and
are free 10 choose ii as we wish. Let's choose dx,, to be the differential that depends
upon all the others through Equation 3. Then we'll choose>.. to eliminarc the term
in dx, 1 in Equation 4 by leuing ). be given by

(5)

at the ex1remum. Now lhe remaining differentials in Equation 4 are independent,


so make the usual argument and find that

j = \, 2 ..... 11 - I (6)

at the ex1remum.
Equations 5 and 6 may now be combined to write

j = I. 2 .. , .. n (7)

Equation I and Equations 7 conslitute a set of 11 + I equations inn + I unknowns,


(x 10 , x 20 • •..• x,, 0 ) and A. We will illus1ra1c the use of Equa1ions 7 with several
Examples.

Example 1:
Find the shape and 1he maximum volume of 1hc paralellcpipcd. whose lotaJ
surface area is A. using the method of Lagrange multipliers.

SOLUTION: We have

V =abc wilh f.:= ab +be+ oc = A/2 = cons1an1


Equations 7 become

c)\I i:Jg
- - ). .......;_ = he - )..(b + c) = 0
0(1 Oll
av . og
- - A - =ac-).(a +c) =0
ob iJb
i)V ;'J g •
-
oc
- l-
ik
=ah - A.(D + h) =0
&.<) The ,' vlelhod ol L,grange Mulliplicrs 287

It's easy lo solve each of these equations for i. and then tind that a= h = c.
Substituting this result into the e<1uu1ion for A gives u = h = c = (A/6) l/2_
Thus. the shape is a cuhe and its volume is V =(
A /6).l/ 2 .

_\'
Example 2:
The equation

x + y+: = l

represents a plane that cuts the axe$ al the points ( 1. 0. 0). (0. I. 0). and
(0. 0, 1) (~c Figure 6.44). Find the shonc.sr distance from the origin to this
, .r
plane .
Figure 6 .44
SOLUTION: We ~hall minimize d 2 = x 1 + y:? + ~2 with the constraint The plane x + y =
• I ~uls the axes al
g =x + y + : = I. Equations 7 become the p('inls ( l .0.0). (0. 1.0). anJ (0.0. 1 ).

a/!. ;J~
- - i. -=2x - 1=0
Jx iJx

which yid<ls x = y = ;:. Suhs1i1u1ing this result into x + y + ;: = I gives


x=y =:: = 1/3.andd= 1;.JI

Example 3:
Determine the shape of a rri,mglc of maximum area for a fixed perimeter/ .

SOLUTION: Start with Heron 's formula for the area of a triangle with
sides 11. h. and c:

A= ls(s - a)(s - b)(.f - r)] 112

where s - (a+ b + c)/1 . (This fonnula is in almost any mathematical


handbook .) We wan\ 10 ma.,cimizc A. ~ubjtcl to 11 + h +<· =I = 2.( =
constant . Equations 7 give

AA iJI [s(.,· - h)(s - d] 112


-
/Ja
- 1-
oa = - - -- -- -
2 (~ - 0 ) 1/2
-).=0
288 Ch,1pt1:r 6 / Fun1 !i(Jlh ot S1•\eral V,3riables

aA
- - A-=
a, [s(J - (l)(S - c)] 111
------ - -)..=
O
clb ab 2(s - b) 1f 2

aA . a, = -
- -
[s( s - a)(s - b)J 1f2
J,.--2(s-- -- -- - ,\ = 0
i:lc ac c) l/ 2

These three equations give

A A A
A=
2(s-a)
= 2(s-b) = - --
2(s - c_)

Therefore, a = b = c and the triangle is an equilateral triangle.

One nice feature of 1his method is that it can be readily generaLized 10 more
Lhan one constraint. If we wish to find an exrremum off (x 1• x 2 • •.•• x11 ) subject
lo two constraints

g(x 1• x 2 ••..• x,,) = constant (8)


/J(.r1, X2 •...• x 11 .) = cons1an1
the resulting equations are

j = I. 2..... 11 (9)

where both>.. andµ are Lagrange multipliers.

Example 4:
One of the mos1 well known applications of the mc1hod of Lagrange
multipliers is in statistical mechanics . Consider a gas consisting of N
molccuh:s. Each of these molecules can be in a quantum-mechanical slate
j with energy Ej wi1h j = I, 2 •. .. . M (M may be infinite). If there
arc N molecules in I.he stale j. rhen rhe fraction of molecules in stale},
1
Jj = Nj/ N. is given by rhe following procedure. Maximize

( I 0)

with respect to each/\\ with

L N1 = N = constant (11)
j

L N £ i =another consrant
j (12)
j

Show that rhis procedure leads 10


6.9 The Method of Lagr,1ngc.- Muhipliers 289
e-µ£1
M
L e-µEJ
j=::1

whereµ is one of Lhe Lagrange muilipliers.

SOLUTION: To maximize \V(N 1• N 2 , ...• N,.,). it is convenient to


1ake rhc logarilhrn of W and 1hcn use Stirling's approximation for N/
(Sec lion 3.1 ). This procedure is valid because In x is a monotonically
increasing function of x. Tnking logarithms. we have
M M M
In W = In N! - L N/ = N In N - N - LN 1 In N) + L Nj
j~I j =- 1 j=l

The two constraints are


M Al

g= LNj=N and h = L Nj Ej = constant


j==l j==I

Equations 9 give

a In W ag iJh
- - - ) . - - 1 . 1 - = - l n N - -1 l + I - A - µ E 1 =0
<JN·J iJNJ oN·J

Solving for Nj gives

We can eliminate .l.. by summing both sides of this expression over Ni and
using Equalion 11:
M
N =e -). "L..,e -11£ ,
jc:d
Eliminating e;·. we have

It turns out that µ = I/ k8 r, where k8 is rhe Boltzmnnn constant and T is


the kelvin 1empcmturc, and the resulting expreftsion for N 1/ N is called lhc
Boltzmann distribution.

You may have not.iced in the examples lhal although we found an extremum or
J (xi, .... Xn), we glossed over whether a result wa$ a ma.id mum or a minimum.
The dctcnnirn1tion of Lhc actual nature of an cxtrcmum requires quite a bil more
effort. Usually in physically or geometrically motiva1ed problems it's clear whether
an c:xtrcmum is a maximum or a mjnimum. but you mma be careful about it.

C gl
290 Ch.ip1t·r 6 / r un lion!, f> f S • -·ral Vari Jhle

6.9 Problem
1. Dctemiinc 1he maximum distance from lhe origin 10
the ellipse described by 3x 2 + 3/ + 4xy = 2 (Figure 6.45).

Figure 6.45
Thr gr.1ph of 1hc ellipse dcscrilx-..J by
3x 2 + :h 2 + 4xy = :2.

\'
2. Find 1hc recranglc of maximum orea 1ha11.:un be in~ribed
~ l ')

in the cllips.c described by \ + y: = I (Figure 6.46) .


o- h-

X
figure 6.46
The graph of the ellip~ describc,d by
.r2 )'~
- + ' - =I.
a b

3. Generalize the previous problem to a rc(;tang.ular par..1\ldcpiped inscribed in an ellipsoid dc~cribed by


x2 \'2 z..:!
-+:_+-=I
a 2 bi c2 ·

4. Find rhe rectangle of fixed perimeter f thnt has lhe shone.st diagonal.

5. Find the maximum value of ax by + c: on the ')Urface of a sphere of radius R.


6. Find the rcciangular parallelepiped of fixed 101al edge length with maximal surf;:ice area.

7. Find the maximum volume of a righr cylinder with Mia/ surface area A (including top and bollom).

8. Find the maximum volume of a right cylinder cone of


=
tixed lateral surface are.a A. I-lint: \/ ~ .rr R'lh and
A = rr Rs . (See Figure 6.47 .)

s
h

I
I
R
figure 6.47 i.; - - -

A right cylinder cone ., hl1wing 1he


quan1i1ics R. h. and.\'.
6 . 10 .'-·lulriplt' Integral~ 291

9. De1ennine 1hc minimum distance from the origin 10 1hc plane described by 3x + 2y +: = 6.
IO. Oe1ermine the minimum dislan,·c from the origin 10 1he plane de.scribed by ax + by + c;,: = I. Chcd your
re$ult ::ig:iinsr lhc previous problem.
11. Find 1hc shortest di.stance from rhe point (3. -2, I) 10 1he plane described hy 2x - )y + : = I.
12. Find the shonesl disrancc from the origin lo 1hc surfat:c Je.$cribcd by x 2 + y~ - 2c: = 4.
13. Find the shortest dis1.anc:c from 1hc origin to the line fom1cd from the intersection of the lwo planes Jcscribcd
by .r + y +: = I and x + 2y + 3: = 6.

14. Find the point on rhc plane describeJ hy 3x - 2: = 0 such that 1hc sum of the $QUarcd disI:i111:l', from lhc
=
poin1s P1 = (I. I. I) ;111d P2 (2, 3. 4) is a minimum.
15. Find the m;iximum value of~ on the ellipsoic..l described by 3x 2 + .h 2 + 2: 1 - IOxy + '.'.! r;; = 10.
16. The rwo r,lanes 2.x +y -
;; :-= I and x - y -1 ::. = 2 intersect to form n line in spact:. Find lht· ~lwrtesl tlislam.:c
froni 1he origin 10 this line.
17. Dc1cnnine the coordinates of the point!- on the spherical surface described hy .r 2 + y:! + -2 = I that are closest
to and farthest from the poin1 ( I. 3. 4 ).

18. M.aximi7.e the product x 1.r2 • • • x 11 • subject to x 1 + x1 + · · · + .r,1 = o. where the x_,-'s and a are positive.

6. 10 Multiple Integrals

ff J(x, y) is defined in some clo~e<l region R in the xy-plane. the do11hle i111egml
of J (x, y) is denoted by

I= ff
R
j(x. y)dA ( IJ

where di\ is an element of .m:a in the xy-planc. Just as the single integral

[" f (x) dx has a 1.!Comctric i111crprcw1ion of being equal to lhc net urea between
/(x) and the .r :n;is over the interval (a. h). a double in1cgral ha." a geometric
interpretation of being the net volume between the surface f(x. y) and the .ry-
plane. -"
Y:z (.r)
Equarion I ls rhc limit of a Riemann sum. The region R is subdivided imo 11
subregions b. R; of area ~A;. We rhcn form the sum
A
II

I,,= L j(x;, _r;)6A, (2) Y_i (.r)


1=1
a I, X

where 1hc point (..r,. Y;) lies inn R;. If J(x, y) is conijnuous in R, then the limi'I of
Figur,e 6.48
Equation 2 as 1he number of subregions increa,;,es and the size of each one decre, !.i.?~ An illu:-1ra1:ion of a two-climcn~ional
is equal 10 Equation I. region R whi:rc any line llat1allc! II) the
Consider the region ~hown in figure 6.48, where any line parallel to they axis _r axis <.To~~cs rhc boundUI)' orR at lw(I
points al the most.
cro),,~c, the houndary of R a11wo poims .ii the most. Suppose tJ1a1 the top•boundary
292 Chaprer 6 / Func1ions of Several Variables

of R (ABC in the figure) is described by y 2 (x) and the lower boundary (ADC) by
y 1(x). where y 1(x) and y 2 (x) arc conlinuous in a S x Sb. In this case, we can
evaJuate the integraJ in Equation I by letting d A = dxdy and integrating over x
and y in mm. Consider a ve11ical strip of width d .x in Figure 6.48. The contribution
to I of this strip between the curves ABC and ADB is given by

\',(f)

da(x)=dx
1 _} 1(.l')
f(x.y)dy

Note that da (x) is a function of x. We can find / over the region in Figure 6.48 by
adding the areas of all the vertical strips between x = a and x = b. wh.ich amounts
to integrating da (x) over x from a 10 b, or in an equation

I =
l lf
"
b ''2{.<l

.1"1(.r)
f (x, y) dy
)
dx (3)

Equation 3 is called an iteraied integral. As we said above. the y integration in


brackets produces a functjon of x. which is then integrated between the limits
a S x Sb.

ty Example 1:
Use Equation 3 to evaluate the area of the shaded region shown in Figure 6.49.

SO l U T I ON : In I his case J (x. y) = I in Equation 3. The shaded region is


bounded by y = x (upper) and y = x 2 ( lower).
Equation 3 gives

Figure 6.49
The shaded region whose area is
dcrem1incd in Example I. lr is the area
boondc..-d by y = x (upper) and y = .r 2
(lower) between x = 0 and .x = I.
Example 2:
Use Equation 3 to evaluate the integral

where R is the region in the first quadrant bounded by y = x 2 and y = 2


(Figure 6.50).
294
Lei's use Equation 4 10 cvaJuale the in1egr.1I

I= ffx dxdy
N

where R is 1he region in the first quadrnnt boumcJed by ihe curves y · x 1 and _r : .r
(Figure 6.53a). In this case. the limi1s of the x inrcgration arc :r = y 10 x = y 1, ~
and the limits of they in1egration are Oto I. Therefore.

X /=
.lo J
I
t r,.,.,: dx) dy = Jor
y
X I [ ~- \ ]
- -
2
dy = -¼
1_

We could also have used Equation 3 to evaluate /. In this case. the limits of the _v
1ntcgra1ion a.re x 2 to.rand the limi1s of lhc x integration arc O lO I (Figure 6.53b).
and so

1·", dy }dx = 11 x(x - ,


1
I = 0. 1 { x
x~ O
.r~) dx = -I
12

! n the first case. we find the arcus of horizontal strips and then add them up in
the vcnical direction (Figure 6.53a) and in !he second case. we find 1he areas of
X vcnical srrips a.nd 1.hcn add rhem up in rhe horizontal dire.et ion ,(Figure 6.5Jb).
In EquaLions 3 and 4, we used curly brJckeL, to emphas-ize which variable is
figure 6.53 integrated first. This is nol standard notalion. Equation J ii. often wrillen as
The rc~ion in rhc tiri;.1 quadrn_111 bounded
=
by the cur.'t·. y x ~' 111d y = x. ( J The /, f.\",(,ll
in1egra1ion ll- over. fi rst(. 10 y 112 ) and
1bcn ver \' (U 10 I ). (,b) The inl<:-Uifiili110 i ..
over y fi ~ I (.l'., 10 .\ / and 1hen ov-cr x (0
I=
1 " ." 1 n
.I (x. yldy dx (5)

10 1).
wi1h the understanding that the inner in1cgmtion is performed first.
Anolhcr notation. which is commonly used in physics. is 10 write Equation 3

I=
f"
• II
dx
f."('}dy j
."11 r)
(x. y) (6)

where the y integration is thought of .is nn opcrnror lhat acts on f (x. y) that
produces a funct.ion of x followed by the x integration as an opcraror. The order of
1he two operJtors is from lcfl to right as u:mal. For example. if /(x. y) = xy and
y 1 (x) = 2r and y (x) = x in Equation 6. then
1

I = /1, 1~\'
II
dx x
,t
dy y

=
1,. (4r2- r) = JJ"
II
dx x · ., ·
-
~
- u
d.r .\" ~ ,
= :. (b-1 -
8
0-1)
6.10 Mulliplc ln!t•,:.:r,1ls 295

The key point here is 10 realize tha1 you perform 1he in1cgra1ions sequentially from
u II= I
right to lefl; you wait for they integration lo produce its result before you inlegrale
over x. This nmation is very convenient and well worth using.
It is often benelicial lo reverse the order of integration in a double integrn.J. A II =X
double integral such as

r
I = Jo d11 Jo
f" dr l.'(l) (7)

occurs in the sta1is1ical mechanics of fluids. LeL's reverse the order of integration (a)

and integrate over II first. It is usually helpful lo draw a pkture illustrJting the
u u= I
integration region. Figure 6.54a shows this region for Equation 7. We integrate
over r from O 10 the line t = 11 for some arbitrary value of 11 (the horizontal strip
It = X
in Figure 6.54a) and then over 1, from O to x (we add up the horizontal strips).
Figure 6.54b illu~1ra1cs 1hc reverse order. We i.n1egra1c over II from r 10 .-· (lhc
vertical strip in F-igurc 6.54b) and then over t from O to x {add up Lhe verticaJ
slrips). Thus.

I= tJo' dt v(I) = Jor


Jot d11 dr ,,(1)
fx d11 = Jor dr(x - l)u(I)
, (8) (b)

Figure 6.54
We were able to reduce Equation 7 to a single inlcgrJJ by reversing the order of Pi~·11rr1aJ1,Jilt~ 10 lhc cv,1luu1,ion of 1hc
inrng:ra1ion. Example 4 provides another example where it is bcneticial to reverse intcglitll i.o 6.ju,11ion 7 by· reversing I.he
orders of ,in1egrn1ion. In (o.), we in1cg ra1e
the order of inte£ra1ion.
o\·c-r I fi r-t. anc:I in th ). we 1n1egrn1c over u
lirsr.

Example 4:
Recall from Seer ion 3.4 I.ha! the exponential integral £ 1(1) is defined by

Show 1har

SOL u TIO N: first we write

The in1egra1ion scheme here is presented in Figure 6.55a. We first in1cgra1c


over II from t 10 oo (the horizontal s1rips) and then over r from O to x
(adding up the horiwntal s;tJips). The reversed integration scheme is shown
in Figure 6.55b. l'n this cnse. we intcgra1e over r from O,w 11 ii 11 < x and 1hen
from O to x if 11 ~ x. Then we add up all I.he venkal s1.rips by integrating
over II from O 10 oc. Using Figure 6.55. we have
296 C ia pler 6 / Functions ui Several Variables

lo.r du - 111 rlr+ !oc d u - lox dr


1= "
10
.r
dr£ 1(r)=
0
e - 11

II O .r
l,-11

JI 0

I= X

(a) " Equations 3 and 4 are readily extended to three dimensions. For example.

tt (=II
I=
1 IJ"'(d [fi,:,(x.y)
h
f(x. y. z)d::. ] dy l dx (9)
I I= X
ll .f1(X) ,!11(.r,_1')

or in operator notation

(10)

IJ = X II Notice that in either Equation 9 or 10. the :: integration yields a function of x and
(b) y. then they integration yields a func1ion x. and finally the x integration between
a and IJ yields /. Equations 9 and I Oare best iIlustrated with an Example.
figure 6.55
Pictorial aids 10 the evalua1ion of
J1' d1 E 1(I) in Example 4. In (aJ. we first
inrcgrarc: over II from I 10 oo (horizon1al
wips) and then overt from Oto x (adding
Example S:
up the horizontal strip.,). In (b). we first The three-dimensional ref!_ion bounded by the planes .r 0. y 0. and :.: = = =0
intcgrnte twt~r t from Oto u if 11 < .i and and the plane .r + y + : = o is shown in Figure 6.56. Evaluate
from Oto x if 11 ~ .r (vertical slrips ) und
then we inregrate o\'er ,, from O 10 oo.
I :_
I= fjj xy: d.rdyd:
R

over thal region.

SOLUTION: Integrate over x first. The limits of integration arc O to


" - y - : (Figure 6.56). Thi!- gives

y
1= j}d:dy)
{{ r --\-:xyz.dx= Jjd::tly·
f{ \' Z(tl - \' - -)-
0 2
Now in1cgra1c over y: the intcgrntion limits are Oto a - ;:.

6 a 6 f(2)f (5) a6
i X l/
= -8(2. 5) = - -- -
Figure 6.56
24 24 rm 720
The rc:gi on bounded by the plane where 8(2. 5) is a beta function (Section 3.1 ).
.r y+ +·= a and the three coordinnte
planes.\" =0 .. = 0. and::= 0.
298 Ch.;ipler 6 / Fun tion ' of Several Variable~

7. Calculate 1hc volume of the region bounded by x = I. x = 2: y = 0. y = x 2 : and.::= 0, ~ = 1/x (see Figure 6.58).

Figure 6.58
An illustration of the volume lo he
X
de1erm.ined Jn Problem 7.

8. Evaluare the integral fff x\ 2


z dxdyd.:: over the same volume as in Problem 7.

. f f . .
9• Fand the centcr o ma-is o t.hc snl1d w1lh boundary planes x = 0, y = O. z = 0,. and -X + .:\'. . + -l = I. 1f. 11. has a
a b c
uniform density p = I (figure 6.59). (See Problem 3 for the defini1ion of center of mass coordinates.)

(0. 0. c)

(0. b , 0)
}"

Figure 6.59
An illus1r:uion of lhc body whose ccnrcr (a.0,0)
of ma..~s is to be dctcrrmned ,n Problem 9.
X,

10. The moment of inenia of a solid body about the::: axis is given by
I [[{ ") ' .
I~= M lll (.c + y-)p(x. y. :::) dxdydr.. Culcula1c I: for 1he body m Problem 9.

11. If o-(x. y) is the mass density per unil area in a two-dimen~ional sheet. rhe total mass of the sheet is given by
1\tl =ff a(x. y)dxdy. Calculate the toral mass of a circular sheet of radius a if a(x. y) = x 2y 2 .
R
12. Re-do Problem I by rever~1ng the order of tht! integration.
13. Evaluate the following intcgr.ib by reversing the order of imegra1ion:
(a) f1 dy t
lo lo
0

1
,,dx sec x (b) [' dy
lo
f_1·
I
dx ye-'·
x

14. Show that [' dy


lo
f-'" I dx ye_.. J ·=
6
~ (e - 1).

al
R •for r.i ,, 299
IS. Show 1ha1 1J1c momcnr of incn,ia 01· a uniform right circular cylinder or radius R and height Ii about its
longi,tudinal axis is equal to R.2/2.
oc eu: /~/, a
16. Show 1ha1
lo
t·-"'erf(bl)dt = - -erfc-. Hint:
o 2b
You need lhe in1egrnl (Problem 3.3.11)

.l e- <m~ 1b, >J, = (:)


I/ "!.
e bi - a l/"erfc 3a·
.{ I . :rx 2 . . . ~
17. Show that
1o
C(11)du =xC(x) -

1
- stn - - . where C(11) ts 1he Fre~nel tnlcgral (Secuon 3.J)
!t 2
C(u) = {" dz cos rr z. .
lo 2
•,1/2 .-,12 sin 11
18. Show 1ha1 j0
dx
1.r
--Ju= I.
II

19. Show 1ha11oc. e-·<1 crf l(af) 112 } dt = ~ (-a-)'l-


o s s+a

20. Use any CAS lo cvaluntc 1o I


d:r {
lo
.r 1
dy e·''l 1 •

21. Use any CAS to evaluaLe ( dy 1·-•· dx x 2 + y 2.


o lo
22. Use any CAS to evaluate the integral in Problem I Ja.
23. Use any CAS to evaluale the in1egrnl in Problem 11 .

Refe r nces

Frank Ayre$, Jr., and El lion M~•nd(·l,011. 1999. Calc11/11.1·. -hh ed .. Schaurn·l> Ou1linc Si.:nt·,.
McGraw-Hill
R. Cournnt. 1970. Differ('11tial and lt1t('8rt1I C11lc11l11s. :!nd ed .. lntcr:-cicncc. John Wiley
CH. Edward':-. Jr.. 1994, Ad11111<wl Cul<·11/11s nf St'~wol Voriobles. Do,cr Publications
C.H. Edwards . Jr.. and David E. Penney. 1998. Calcu/11s and Analytic G,·mllF/f)', 51h ed ..
Prentice-Hal I
Witold Kosmala. 1999. Ad1·1111cnl Calc ·11l11s: A Friendly Approwh. Prcn1icc-Hal I
Jerrold Marsdc □ and Alan Weinstein, 1985, Colrnh,s I. JI, and Ill. Springcr-Vcrlag
l'vlurray Spiegel. 1963. Ad1•011t:1·d Coln,l11.~. Sch::ium ·s Outline Serie.-:. :VkGrnw-Hill
David Widder. 1989. Arh-m,ced Culrnf11.1·. 2nd e<l .. Dover Publications
CHAPTER 7
Vector Calculus

The central theme of this chaprer is that of a vector field. which is a veer or function
defined at each point (x. y. :) in u region of space. For example. lhe electric
field 1hrougho11r some region or the velocity of a fluiJ at each point in the fluid
can be represented as a vecwr field. Generally, a vector function v = v(x. y. :)
represents a vector field. Vector lleld~ play prominent roles in all areas of the
physical sciences and engineering. The fundamental laws of cla..,sical mechanics.
electricity and magnetism. Auid mechanics. elasticity. heat flow, and other ar~a,
are expn::s::.ed in terms of vector fields. The vector field methods that we develop
in this chapter allow us to cxpn:~s these laws in a compact fonn and lo derive
many useful relations from them in ri precise. srraigluforward manner. We shall
first introduce the gradient. the diverg1.:ncc, and rhe rnrl opemtions, which are
1he three central differential operntor quunti1ies involving veclor fields. Then, in
Section 2. we discuss line integrals and show how they arc used to calculate t11c
work done in mcchanic;i\ sy,tems. We finish this ,e;:et.ion with a discussion of y
Green ·s theorem in the plane. which is used widely in mathematics. as we shall see
in later chapters. Af1crdiscu.-i~ing surface integral~ in Section 3. we go on to discuss
1he divergence 1heorem, which rel,ues a surface integral to a volume in1egral. and
Stokes·s theorem. which relarcs a line integraJ 10 a ~tufacc in1cgral. Many au1hors
refer 10 these 1wo theorems as the "big" lheorems of vector analysis . We shall
see that both of these theorems have a great \•aricty of applications 1.0 physical
X
problems.

7.1 Vector Fields - -I


As we said in the introduction. a vector field is a vector funcLion defined al each Figure 7.1
point (.r. y. :) in a region of sp,·,ce. Consequently. we can associate a vector wi1h A pi..:turi I repn::.cntmion of 1.hc vec tor
field described by,.= y I+ ..r j.
each point of the region. which we can represent graphically as we do in Figure 7. I
for the two-dimensional veclor function ,, = y i + x j. This ve.ctor field represents
11uid flow in toward the origin from the second and fourth quadrants being forced
out inro the firs1 and 1hird quadranrs.
In the previous chapter. we learned rhat the gradient of a scalar f unc.:tion
<J> = rt,(x. y. :.) is de lined by 301
302 Lh,1pt1·r ;' / Vc-clor C,11< ulus

a<t, . a,t, . n,p


grad <f> = 'v</> = - 1 +- J+ - k (I)
ax ay a.:
The gradiem arose naturally when we discussed the directional derivative of
</J (x. y. z ). and we lcnme<l that 1hc maxi mum rate of increase of </J is in the direction
of 'iJ,jJ and 1hat its magnitude is given by IV<t>I. We also saw 1ha1 if j(x. y. z) = c
describes a sunace, 1hen V f is normal to the surface. provided 'iJ f -:j:. 0. Of course
we are assuming that f(x, y, ;:) is differeniiable. for orherwise V f would not ex-
ist The gradient occurs frequently in physical applications. In classical mechanics.
if V (x. y. z) represenL, the potential energy, then 1hc corresponding force field is
given by

F(x. y. z) = - VV(x, y, .:)


In electricity and magne1 ism. if V (x. _,., .:) represents the ckctros1a1ic potential.
then the c.:orresponding electric: field intensity is given by

E(x. y . .:) = -'iJV(x. y, .:) = -grnd V(x. y. :)

For example. the electrostatic porential due to an electric dipole located at the
origin and oriented in the x direction is

V(x, ,,. :)
·
= 4rre (x-., J.LX ~ .,
+ y-+z.-)
(2)
0

where J-L is the magnitude of the dipole moment and e0 is the pem1it1ivity of free
space. The level curves of V (x. y . .:) in the x_v-plune arc shown in Figure 7 .2.
Figure 7.2 The gradiem of V (x. y. z) is
·n1e equip<>rential~ ll'.Olor) and rhe electric
field line$ (black) due to ar:I d e~rric dipole
si111:i1cd :11 the origi n and oriented in the x
dirc.."Clion.

Figure 7 .2 also shows the vector field a-;,ociated with 'iJ V in Lhe x y-plane. Nole
that 'iJ V is pcrpcndicul.ir 10 the level curves of V (x. y. :), as we learned in the
previous chap1cr.
The gr.idient is also related to heal flow. Let the temperature of a substance
vary according to T (x, y, :). If the temperature is not uniform, energy as heat wi II
flow from regions of higher temperature to regions. of lower temperature in the
direction of the maximum decrease of temperature. If q is the flow of energy as
heal through a unit surface area (perpendicular to q) per unit time. then Fourier's
/a11• n.f hear ffniv says lhat

q(x. y. :) = -K'iJT(x. y. ;:) = -K grad T(.x. y . .:) (3)

where K is called 1hc thermal conduclivity of the substance.


The diffu~ion of a subs1uncc due to a concentration gradient is govcmt!d by
an equation very similar to Equation 3. If J is the diffusive flow r.ite of a subst.ancc
through a unit area (perpendicular 10 J) per unit t.imc. lhcn J is often given to a
7.1 Vector Fi~kls 303
good i.!pproximation by

J(x. y. ;:) = -D'vc(,,·. y. ;:) = -D grad c:(x. y. ::) (4)

where c:(x. y. ::) is the concemration of 1he suhsr.rnce and Dis called the diffusion d
coefficient. whose value depends upon both the substance that is diffusing and
the medium through which it is diffusing. Equal ion 4 is known a~ Fick ·s la\\" nf n
d~/Jusirm.
The veclorial quanl ities q and .I in Equations 3 and 4 are ea! ledj/1Ht'S. Consider
the situation in Figure 7 .J. which shows a small area element dS whose orientation
is specified by an outward unil nom1al vector n. If q and n are pointing in the same
direction. then the flow rate of energy as heat across rhc surface d S is given by
q dS. and has unil.5 of energy per unit lime. If. on the other ham.I. q and o are Figure 7.3
not pointing in the same direction. then the flow rate of energy a.-. heat across the An area clement d S whose orientation
is s.pecified by the n(lNnal Vector n. The
su,i"ace d S is given by q • n d S. For example. if q is para Ilei to d S ( perpendicular vector J •~ J flux al'rO)S dS = ndS.
ton). then there is no flow across dS. We can define a vectorial surfoce element
(ii has orientation and area) by dS =
n d Sand wrire

flow rate across d S = J · dS (5)

We shall use Equation 5 sever.JI times in this chapter.


It is often convenient to view

. d . a i:l
'v=l-+J-+k- (6)
iJx ny i-);:

as a vector operator. called the de! operator. Just as w~ can operate with 'v on
a scaJar field. we can opernte wi rh 'v on a vector tie Id A (x. y. :: ) by taking the
dot product 'v · A. This quanrity is a scalar field called the di1·ergrnce rf A and is
wri11cn as

. iJA
d1v A = 'v · A = __x + -- · + -aA.-
i.lA r
(7)
Dx [)y cl:

(Note that Ax. A 1• and A: in Equation 7 arc 1hc components or A. and not partial
derivatives.)

Example 1:
Determine the expression for Liiv F if F'(x. y. :) = .ry I+ y: - j + x 2 y,-. k.
SOLUTION:
304 Chapter 7 / Veclor Calculus

The divergence has an important physical interpretation which makes it useful


for physical problems. Let's consider the total flow rate of A per unit volume out
-I ,I
of the small rectangular region of volume ll V = 6x 6y6z shown in Figure 7.4.
(X, y, Z)
=
For example, A might be a flux of maller. given by J p u. where p is the mass

\ •
y
density and u is the velocity of the tlow. (Note that J has units of mass per unit
area per unir time .) The normal unit vector facing out of the front face (located at
=
x + 6x) in Figure 7.4 is n i and the area of the face is 6y 6;:i:. We can write this
vectorial area as 6S = .o.y6zi, and thus according to Equation 5. the flow rate of
A ou1 of the front face is

d .\·
flow rate of A ou1 of
. . .
the front face m Figure 7.4
== f A. dS == 1· AJr +
·
.o..r. y. :) dyd:

.,, == A x (X + 6.x. }1, Z1)6y6.:


IX l
Figure
7 _4 where. according 10 the mean value theorem of integration. (x + 6 .r. y 1• .: 1) is a
TI\c geometl)' u.cd for a dcriva1ion of the point on I.he front surface. In other words. we have u~ the relation
divagence theorem .

if _f(y. z.)dyd: = f(v1, Z1)6y6::


S=oyu~

for a continuous function f (.v , ::). where y .:s: y 1 .:'.: y + 6. y and .: :S. : 1 .:'.: .: + ~Z -
For small values of 6..r:. we can writ.e Ax(x 6 x . y . ::) as A_,.(x. y. ::)
ctA
+ __ .r 6x + · · · and so
ax
now rate of A out of JA, ]
. .
the front face in Figure 7.4
= [ Ar(X,
·
y, :) + -.-· 6x
i:lx
6y6:

where aA _\.jox depends upon x. y . and:: .


Similarly. the normal unit vector facing out of the back face (located al x) in
Figure 7.4 is n = -i. Therefore. the flow rate of A out of the back face is

flow idle of A OUI of


the back face in Figure 7.4
= J A-dS=-
r
~
A ...(x.r.:)d\·d:
· ·
= -A.((x, Y1, :1)6yil;:
where. once again, y S y 1 S: y + 6y and : S :: 1 S., + 6:. The net flow rate of
A through the two faces perpendicular lo 1he .r axis in Figure 7.4 is given hy
(uA.r/Dx)6x 6y6:. If we let 6x. 6y. and 6: approach zero, then

net flow rate of A out of the 1wo surfacc!'-


= ilAx dxd\'d:
that are perpendicular to 1he x axis in Figure 7.4 Jx .

We obtain similar expressions fur the flo\v rate through the other faces in
Figure 7.4. and so 1hc lotal now rJtc or A out of d V = dxd_rdz is given by
7.1 Vector Field~ 305

net flow rate of A OU( of = (DA,r + aA y + aA~) dxdvd:.


the reclangular region in Figure 7.4 i"Jx oy ih · (8)

= div A dxdydz
Thus, we sec that the divergence of A represents 1hc llow rate per uni1 volume ou1
of a given region in space in the limit in which the volume approaches zero.
For the case where A is the flow of some suhstance out of a small volume V.
then the llux is given by .I = pu. and Equa1ion 8 says that

net Aow ra1e of mass per uni1 volume out of V = div J = div (pu) (9)

Note that div J has units of mass per unit volume per unit time, or density per unil
lime. The net flow rate of mass per unit volume out of V must be equal 10 t.he rate
of change of density within V. and so we have

op .
- = -d1v(pu) ( I 0)
;)1

where 1he minus sign simply says that if 1he density increases with Lime, then the
outward flow musl be negative or. in other words. inward. Equation 9 is called the
co111i11iiiry eq11mio11 and is one of the mos1 fundamental equations of the physical
sciences: il is nothing lcs.--. than an ex.press ion of conservation of mass in differential
form.
There is a conlinuity equation for each constituem in a muhiconst.ituent sys-
c,.
tem. If is the concemration of the kth constituent then Equatjon IO reads

(I I)

Equation I I is an ex press ion of the conserva1ion of the m.iss or the kth consti tucnl.
or the numlx:r of panicles of 1he kth constituent. Fick"s law (Equation 4) says that
the flux of the k1h constituent can be approximated by

( I 2)

where D1;. i, 1hi: diHi.hion constam of the krh cons1i1ucn1. Subs1i1u1c this approxi-
mation into Equa1ion I I (which is exact) to obrain

-DcA
i1r
= D k d.IV £r.1cl CJ; (13)

assuming Lha1 D(- does no\ vary with position. It 1Ums ou1 Lha1 div grad can be
cxprcss0d in a fairly simpl1e fom1. Proble111 1] I has you show that

( 14)

The operator expression in Equal ion I4 occurs frequently in physicaJ problems


and is called the Larlacian nperotor and is denoted by 'v 2 . Equ:11 ion I4 becomes
306

( 15)

Using 1.hi1- notarion. Equation 13 becomes

( 16)

Equation 16 is called Ihe diffusion eq11a1iu11 and models how the concentration
of a subs1ancc varies as a function of space and rime starting from some initial
distribution. It is a partial di fferenlial equation in the unknown c1; (x. y, ::. t). We
shall learn how to solve partial differential equations in Chapter 16. but in 1he
following Example we invcs1iga1c one panicular solurion of Equation 16.

Example 2:
Consider the diITusion or a substance along a long thin cylinder. In this c.:ase.
the diffusion occurs along only one direction and can he dcs.crihcd by a
one-di mcnsional version of Eq uarinn 16:

- 00 <X

where w~ have dropped rhe k subi;cripr. Show 1ha1

is a solution 10 Equation 16. ln1crpn:t thi~ Sl1lu1ion phy.~i<.:ully.

SOLUTION: The spa I ial derivatives of c(x. 1) are

ik r x e- .1.J/-H 11
-=-------
ax
0
-$ 0 ,r)(;r Dl) lf~

Co(' __,·1 / 4.Vr

(4 Dt )(:rr Dt) I/:.

and the time derivative i~


c(x. 1)
il c ,.· x 2 e- 2/•HJ, cue x~/.J.D1
= - 0- - - - -
a, (8Dr 2 )(n D1) 1 n 4( .!'I: DI) I/ 2,

Therefore. we find thar

Figur,e i.5 The solu1ion i.,;. plollcd in Figure 7.5.


The ~ulution 10 the diffusion cqua1ion
gi ven in Exampl.c 2 plonetl for \',irfo:m lni1iolly. .ill the diffusing ~ubstJnce i:- located ai the orifin, in the form or
values of Dt. a ~pike [r(x. I) -,. cnS(x) as 1 ~ OJ. Then. a.,, lime inc:reasc,. lhc :-ubs1ancc
7. 1 VC'nor Fields 307
spreads oul in both directions from the origin. and evemually becomes
unifom1ly distribu1ed.

Fonning the dot product of "iJ with a vc-c1or A gives us the divergence of A.
We can also fonn the cross product of V with A to give what we call 1hc rnrl of A.
written as
curl A::::: "iJ x A

We can write I.his expression fonnally in detem1inantal fom1

j k
i) a i.}
curl A ::::: 'il x A ::::: ( 18)
dX av J;:
Ax A_1' A.

Example 3:
Find curl,. if,,= .r;:: I+ .ry 2 j + _r 2;: k.

SOLUTION:

j k
i) a iJ
curl A=
ax Jy
-
a::
= i (2y:: - OJ + j (x - 0) + k (y~ - 0)

x ;:
J
.\" \"-' y~ - w

= 2y.::-, l + x j + /! k

If A = grad </J. we call cp (x. y. :) Lhe srnlar potential of the vector field
A(x. y. :). and if A= curl u, we call u(x. y. :) the l'ector potential of A(x. y. :).
We' JI learn how to determine the scalar potential of A (x. y. ~) in the next section
and we· II learn how 10 detenni ne the vector potential of A (x. y. ;: ) in Section 5.
Jus1 as 1he divergence has a nice physical in1erpretation. so does Lhe curl.
We saw in Chapter 5 1ha1 the velocity at a point r in a rigid body rotating with a
rotational vec1or w is given by (Equation 5.4.14)

v=wxr

(Figure 7.6). For ~implicity. let w be directed along the.: axis, ~o that w = wk and Figure 7.6
The ro1:11i on of a rigid body abour 1hc w
j k axis. illus1ra1 in2 Lhar ,. ""w xr.
\' = () 0 C,J = -wy i + wx j
X \' -
~
308 Ch,1ptc•r 7 : Vl:'clor Calculus

Therefore,

/ j k
a a
curl v =
rlx a_v
-a;:a = 2wk =2w

-wy wx 0

x Thus. curl vis twice 1hc angular velocity of 1hc rot:uing body. Figure 7.7 shows
the vector field v = -wy i + wx j. You can see tha1 this field represents circulatory
mo1ion abou1 the origjn_
We illustrated the physical meaning of curl v by considering a rota1ing rigid
body, but the curl plays a prominent role in fluid dynamics aJso. Generally, if v is
the velocity field. or the 11ow Ii ncs, of a fluid. then curl v. called t.he vorriciry \'ector
Figure 7.7
A pictorial represen1a1ion of the vector
of rhefluid. points in the direcLion about which a vortex motion lakes place and is
field dc~bcd by,.= -er)_\' I+ wx j. a measure of the angular velocity of the tlow. The flow of a ft uid ia a velocity field
in which curl v = 0 is caJled irrotnrio11al Jim.,,_ lrrotational flow has the following
special propeny: Suppose 1ha1 v is given by 1he gradient of a ··velocity potential".
=
l/1. so that\' "vl/J = grad r/J at any point in Lhe nuid. Then

curl v = curl grad y,


But it is easy to show that

curl grad ,J, =0 ( l 9)

for any function that has con1inuous second partial dcrivaLives (Problem 12).
Equation I 9 tells us that lhe now must be irrotationul for there 10 be a (scalar)
velocity potential whose gradient gives the velocity at any point in the fluid.
Because of lhis imponant property. fluid dynamics 1exts spend a fair amount of
Lime on irrolat1onril flow.
There is another relaiion similar 10 Equal ion 19 1hat you should know: namely.
(Problem 13)

div curl v =0 (20j

where the components orv have continuous second partial deriva1ives. Equation 20
is consistent with the physical interpretations of the divergence and the curl. The
curl represents circular motion. and so Equation 20 says that the flwd i.s contillned
within a volume enclosing the mo1ion (no flow in or ou1 of the volume).
To summarize the key results of Lhis section. we have 1he three opera! ions with
the del operator. 'v:

grad <I>= 'vrp = -At/> •1 + -


iJ</> il,t,
j+ - k (21)
a.r ay a::
.
d1v v = 'v • v =
au. + --
_.t
01.\. Ju_
+ -~ (22)
ax ay az;
C gl
309

and
j k

-a
fj
curl v = V xv = (23)
i:Jy a:
u_, 1,1_

and the t.hrce "combination" relations:

(24)

curl grad rt, =0 (25)

and

div curl \' =0 (26)

There is also a number or other rcla1ions Lhnt can be proved s1raighrforwardly


from Lhc definitions (Problem 14 ). such as

div ,pv = 'v • tf>\' = rJ,'v • v + v • 'v</J (27)

div (u x v) ,...: Y · 'v x u - u • V x v (28)

(29)

curl tf>v = 'vrJ, xv+ </J curl v (30)

Most of the CAS can be used 10 cakula1e rhe gradient. divergence. and curl.
Problems 20 and 21 ask you 10 use any C AS for such calculations.

7 .1 Problems
1. Recall from 1hc previous chapter that if f(x. y, :) = c describe., a surface. then 'il f is normal 10 I.he surface.
=
Find the uniL normal vector to 1hc surface of 1he clllptic parnboloid described by : lY~ + y 2 at lhe point
( I. l. 3),
2. Find the unit nonnal vector 10 the surface of 1he circular cone described by x 2 + y 2 = 2:: 2 al the point (I. I. I).
3. Find div A and curl A for A =xy 2 i + 2.ty;::j- x 1 : k.
4. Determine div A and curl A for A = (.t - co~ _\·:;) i + (y - cos x:) j + (z - cos xy) k.

For 1/u· next fi,·e problems. tnke r = x I + y j + z k.

5. Show thnt 'v • r = 3 and lha1 'il x r = 0 (provided r t 0).


6. Show that div (r / r-1 ) = 0 (provide<.! r -:/= 0_).
7. Show that grnd ( 1/r) = -r/r 3 (provided r-:/=- 0).

8. Show I.hat '7 2 (;) = 0 (provided r =,:- 0).


9. Sliow that 'v x IJ(r) rl = 0.
310 Chap1e, 7 I \l('(:lor C.llculus

10. If the density of a fluid remains constant. the flow is s.aid to be incompre.uiblt>.
(a) Is the now incompressible if,.= y I - x J?
(b) Whal if v = 2x i + y j - 2z k?
(c) Is either flow irrotational?

11. Prove 1hat div gr.:id = \7 2.

12. Prove that curl grad i/, = 0 where t/J ha.s continuous sec.;ond partial d~rivativc~.
13. Prove 1hat div c:url v = 0 where the componenus of v have continuous second pnrtial derivatives.

14. (a) Prove that div r/)v = <J,'v • v + v • 'vt/J.


(b) Use this result to evaluate div {J)v if <t, = xy and v = y 2 i + x: k.
(c) Evaluo:1te it by applying div directly to 4J,• and compare your result.

15. Prmrc that div (u x ,•) = v • ('v x u) - u • ('v x v).

16. Consider a point P surrounded by a region of volume V with a surface S. Let d S be a small area element of S
with a nonnal outward vector n. as shown in Figure 7.8. A more fundamemal definition of 1he gr.idicnl of </J at
the point P is

grJd tJ, = lim ( /


v .... o
l n ,J>J S)
V

This definilion is independent of the coordinate system usctl. Show 1hat 1hii. definition is c.quivalent to
Equation I if we use a cartesian coordinate system. Hint: Use an argument like 1he one we u...ed with Figure 7.4
10derive Equation 8.

17. Referring to Figure 7.8 as we did in Problem 16. we can state a more fundamental definition of div v by

divv= lim
l'-,0
ln_•vdS)
( --'-
f
\I

This definition is independent of 1hc coordinate system used. Following Problem 16, show 1hat 1his defi n icion
is equivalent 10 Equation 7 if we use a cartesian coordinate system. Him: Use an argument like the one we
used with Figure 7.4 10 derive Equation 8.

n
tl.

Figure 7.8
The geometry a.<,sociated with I.he
fundamenial dctini1io11 of 1hc .Qradicn1
presented in l>J'oblem 16.
7.2 Line lntt'gr.ils 311

18. Referring 10 Figu.--e 7.8 a, we did in Problems 16 and 17, v.•c can sw1c a more funJamcntal deli ni1 ion of the
curl by

curl v = lim
V-fl
(fl n >n"
V
dS)

This definition is independent of the coordinate ~ystern u~d. Following Problt!m-. 16 and 17 ..,how 1hal this
detinition is equivalent to Equation 17 if we use a cartesian coordinate system.
19. Show 1ha1 curl curl v = grad div v - v''.\,_

20. Use any CAS to calculate grad f if f(x. y. z} = (.:~ - x 2 )e-Y 1 .


21. Us!! any CAS to calculate div u and curl u if u = x 2 ~~ i - xyz-:! j + t' -c, !
1· .:~) k.

7.2 Line Integral


The work done by c1 force F on a body that undergoes a displacement dr is
dW = F • dr. Now suppose that the body moves from point a to point h along
some curve C. The total work involved is given by

W:::: /
C
F · dr == f
i
F.rdx +
C
f Fydy + J
C
F~dz (I)

where !he C under the integral signs emphasizes that the integra1ion is canied out
along the curve C. The in1egral in Equa1ion I is called a line imegral or a path
inll'gra/ because it is canied out along a given path. We can describe Lhe curve
parametrica.lly by a position vector r(u) = x(11) i + y(11) j + t{11) k. which we
will always require 10 be piecewise smooth; in other words. Lhc curve consisLc; 11f
a finilc number of segments each of which has a unique tangent at each point and
whose direction varies continuously as the parameter u varies. Th.is requirement
allows us 10 write
dr
f
b

C
F · dr =
l 0
F·-
du
d11 (2)

where r(a) is the ini1ial poinL and r(h) is the final point on the in1egra1ion path.
The path dependence of Lhe i n1egral is renectcd in the factor dr /du.

Example 1:
Evaluate
J F-dr
C

where F = (x + y) i - y j and x = I - 11, y = 11 2 • from 11 =0 10 u = I.

gl
312 . hapl r 7 I Vi -1 r Cal ulu

SOLUTION: Using Equation 2.


}'

.I f · -dr du=
du o
ii
[x(11) + y(11)J.r (11) du -
1 1m l y(11)_v'(11)
1
. d11
C

=
10
1
(] - II+ ,i2)(-d11) - 2 ii
0
11
3du = --
3
4

Figure 7.9 show~ why 1he answer is nega1ivt!.


Tiic path of inrcgration is going against Lhe force field. so F · dr < 0 and
1hc integral i$ negative.
X

Figure 7.9
A pic:torial reprt'~nLaticm of the force field Let ·s cvalua1c JA • dr around the triangular pa1h shown in Figure 7.10 if
and the p3th of integration in Example I. A = (x - y) i + (x + y) j. We· 11 calculate the integral along eacb of 1hc three paths
in Figure 7.10 and 1hen add the results. Along path I. y 0 and dy 0, so = =

Along path 2. x = 2 and dx = 0, so

f A· dr = 1\2 + .\')
n
dv
. =~ ')
-
c~
Figure 7.10
A Lriangular path usc<l for the cviuuation Along path 3. y = x /2. so
of l A · dr. where A = (x - y) I+
f A · dr = f l(x - y)dx + (x + y)dyj
(.r • )') j.
CJ

Lcu-ing y = x/2 and dy = dx/2, we have

= Ji[ (x2 + 4
Jx)
0

f
C1
A. dr d.r = -25

(You should verify right now !hat you obtain the same result if you let x = 2y and
dx = 2dy and I.hen imegraLe over y from I to 0.) The integral around the triangu.lar
pa1b shown in Figure 7. 10 i~ equal to 2 + 5/2 - 5/2 = 2. We can express this
result symbolrcally by wri1jng f A · dr = 2, where the circle on 1he integral sign
emphasizes 1ha1 the path is closed. Furthermore, unless we stale otherwise. we
shall always traverse a closed path in a counterc\ockwise direction. as indica1ed
by the arrows in Figure 7. I 0. The area cnclMed by the closed path ~1lways lies to
1hc left as you traverse 1he palll. Note that _( A · dr in this case is ,wr zero. The
path integral around a closed path need not equal zero. It may equal zero, however,
as the next Example shows.
7.2 Line ln1t·;:r,1ls 313

Example 2:
Ew1lua1e f A• dr around the closed path shown in Figure 7.10 if
A= (y - x) I+ (x + y)j.

SOLUTION:

/A·
C1
dr = -1 2
x dx = -2

/ A · dr = lo \2 + y) dy =~
c~

/A· = f dr I()· - x)d.:c + (x + y)dy l


C.1

_3x) dx = -~
4 2

The sum of lhe 1hrcc path iniegral,; gi\'CS f A • dr = 0. The integral around
the triangular path in Figure 7. I O is equal 10 zero in 1his case.

Before we examine why the integral around a clo~ path was equal to zero
in one case but nor in the other, let's notice one other propeny of line integrals. If
we reverse the di reel ions of the three paths in Figure 7. I0, then the values of al 1
the integrals that we obtained would rcvcn.l.! sign. Thus. the value of a line integral
depends upon 1he d1rec1ion IhaI we rake along 1hc parh. We can exprc:-s 1hi,; r~sulI
symbolically by writing Pa

f
-C
A • dr =- f
C
A • dr (3)

We'll now prove that iff A· dr =0 foraJl closed paths in a domain D. 1hcn the
value off A · dr depends 011/y upon the end point<; PI) and Pb and is independent
of the curve connecting P,, and P,,. Figure 7 .11 ~hows an arbitmry closed curve C Figure 7.11
and 1wo arbitrary poinL<;. P,1 and P1,. which break C into lwo (arbitrary) curves, C 1 An aid 10 lhl! proof IIUll if f A · dr = 0
fo rr ;111 clo~ed ipmh, in a durJ1ain D. I.hen
and C.,. Then .f A • dr is puth independent.

f A · dr = 0 = J A · Jr + J A . dr
C1 C2

Using Equation 3.

f A • dr =- f A • dr

C gl
314 hapter 7 I V ror Calculus

so

/ A • dr = f A - tlr (4)
C1 - C~

But C I and -C2 are two arbilrary palhs from P11 to Pi,. so Equation 4 says that the
line integral from P" to Pb is independent of 1hc path.
Let's apply lhis result by cvalualing JA •
dr from (0. 0) 10 (2. I) along 1he
parabola y = x 2 /4 for A given in Example 2:

f A· dr = f [(_y - :r:)dx + (x + y)dy I


C

I
=2
which is the value of JA - dr for the sum of paths I and 2 in Example 2.
Furthermore, it is also 1he negative of the result for path 3.
A line integral J
A · dr between 1wo points Pr, and Ph is said 10 be parl,
indepn1de111 if the value of JA •
dr does nol depend upon lhe path connecting
the 1wo end points. Its value depends only upon the end poinLS. and we express
this by writing

f A · dr = 1h A · dr (path independent) (5)


C

It is easy to prove 1hc converse of the above theorem; namely. that if .f A • dr is


palh independent then

f A-dr=O (palb independenl) (6)

Example 3:
(0. I) C_-'_
. . . , 1. . , _ :_ _ __, (I. I) EvaJuare f A - dr around lhc closed path shown in Figure 7. 12 if
A= (y 2 + 2xy) [ + (x 2 + 2xy)j.

SOLUTION: Along. pa1h I. y = 0 and dy = 0, so


'• JA -dr =0
(o. o) I (I. 0) X C1

Along paLh 2. x = I and dx = 0. so


figure 7.12
The closed parh used 10 c,·alu:ite f A • dr J A-dr= 1 1
(1+2y)dy=2
in Example .l
C2
7. 2 Line lnlt•gr,il~ 315

Along path 3, y = I and dy = 0, so

f
.
A· dr = f
11
0
(1 + 2x)dx = -2
CJ

Along path 4, .r = 0 and dx = 0, so

JA -dr=D
C.1

Therefore. f A· dr = 0.

We haven· t yet addressed the quesr ion of why f A • dr = 0 in some cases but
not in others. but we have seen that an equivalent question is why J A · dr is path
independent in some cases but not in others. Notice that if A • dr = d</J, where
<t> = <f>(x. y. ;::), then

JA· dr = Jd<t> = </J(b) - </>(a) (7)

Now. A• dr would equal dt/> if

o</J o<t> a<1>


A· dr = -dx
ax
+ -d\" +
oy -
-dz.
cl;::

But this will occur if

(8)

or if A= grnd </J. Thus. we see that it is sufficient that f A • dr be path independent


if A = grad <f>. It turns out that this is a necessary condilion as well. We can
fonnalize this result by the following theorem:

Let A be comi1111011.s in a domain D. Tl1t'n. the line imegml JA · dr is palh


imlcpe11de111 in D if and unly if A = grnd t/J for .wme f1111c1io11 t/J(x. y. ;:)
defined i11 D.

How can we determine if a given vector function A can be wrinen as the


gradient of a scalar funcrion <J>(x. y. z)? Equivalen1ly. how can we detem1ine if
=
A · dr is an exact differential A · dr d</)? It"s !empting to say t.ha1 if A can be
wriucn in the form of Equarion 8. then

f)tp
Ay=-;
ay
and so

C gl
1 \11·1 tm C ,ilt ulus
316 C h,tph·r ;-

nA .r a2<1> i'l2</J JA1,


c.ly
= oyJx = OX<ty = --·
ox
oA_r a2<1> ;)2rJ> aA. (9)
i:l:
= i.Jz.i)y = i)yiJ:: = -ih-
8A, a2</J a2,JJ oAx
iJx
= i)x a: = iJz)Jx = cl z.
wherever the mixed second panjal derivatives are continuous. However. Equa-
tions 9 are necessary conditions thar A • dr = dq,. but they arc not sufficient.
The classic ex.ample of a case where the mixed second partial derivatives are
equal. ye1 A • dr is not an exact differential. is for

}' j X j
A= ---+--- (x. y) :/= (0. 0) ( 10)
x2 + y2 xl + y2
It's easy 10 show 1ha1

2 2
uA x = aA_ = 1. y - x
ay ax (x 2 + y 2 }2

but le1·s calculalc f A · dr around a unit circle ccntered al 1hc origin. If A · dr is


an ex.act differcn1ial, lhen the result should equal zero. The integral to evaluate is

1_ J. ( y dx + x dy )
-r x2 + yl x- + y2
( I I)

Let x 2 + y 2 = I. x = cos 0. and y = cos 0 (0 :5 0 5 2rr ). Then

The result is 1101 equal to zero. and so A · dr is not ;in exact diffcrenlial.
The problem here is that A and ils partial derivatives are not defined at the
origin, and t.hc integration path encloses the origin. If we integrate A• dr around
a path rhat does not enclose 1he origin. then we do get zero for the resulL
y

Example 4:
X
Evaluate the integral in Equation 11 around a circle of radius b ,.:entered at
thepoint (o. 0) (Figure 7. 13).

SOL u TI ON: TI1c circle shown in Figure 7.13 can be described parnmetri-

Figure 7.13
caJly by the equations
A circle: of radiu.~ I> centcred m the point
('1. O)T dc.,;,cribed by the rpararne1rk x(O) =a+ b cos O, y(0) =b sin 0. 0 .:: 0 ::: 2rr
c~l1atio ns x = " + b co~ 0 and' _r = b "'in /J
forO :E. 0 ~ 2,r. Substituting x(O) and y(O) into Equation 11 gives
7.2 Li Integrals 317

I = {cl~:r . , h2 ~ ab cos{} dO y
Jn a-+ h- + 2ahcosf)
= b2 ( ;,, ____,_10____ + ab (:r cos O d()
(12)
lo a2 + b2 + 2ab cos 0 lo a2 + b 2 2n/J cos 0
Figure 7 . 14 shows 1hat these integrals arc ~ymmclric ::ibout (} = rr, so we can
write 1hem a" 2 rimes 1he integrals from O 10 7T.
11,esc integmls arc in rhc CRC Morhemutii:£Jl Tables (see also
Problem I .7 .22) and give

l=,r-2 [ l(1
tan-
1 {/·-b-
02
+ b1
2
I
. , ) tan 0 J.7 I>

Figure 7.14
Now. if a > b. then we get rr - 2(rr /2) = 0. and if a < h. I.hen we ge1
The functions 1/(r/ b 2 + 2ab cos 0)
71 + 2(rr /2) = 2,r . Thus. / = 0 if the integration path does not enclose the
(color) and cos f) /(1.1! + b 2 , 2uh cos I})
01igin. but / = 2;r if it does . (bl ack) plo11cd :igainst & (a -:j:. b) ,

Generally. A · dr in the integrand of Equation 1 I is an exact differential in any


region that docs not include the origin. However. it is not an exact differential in a
region that docs include 1hc origin. These results lead us to the following definition:
A region R is sa.id 10 be simply com,ecred if any clos.cd curve lying within R can
be coni-inuously shrunk 10 a point without leaving lhe region. Roughly speaking. (a) (b)
a simply connected region has no holes in it. Figure 7. 15 illustrates some various
types of regions . Figure 7.15
An illustrn11on of (a) a simply connct:tcd
We can now state the following result :
region and (h) a region thal i., not simply
1:onnected.
If the compont!nls of A are cor11in11011s and /,ave conrinuous first partial
deril•arive.~ in a simply connected region R. thn1 A • dr wifl be an exact
dffferenlilll ifand only if Eq,wrions 9 are satisfied. Furthermore. if A · dr = 0
am1111J a11y .dmple (11011i11rcrsecri11g) closed pnrl, lying ll'ithi11 R.

Lc1·s use Equations 910 see 1hat A in Example 2 is the gradient of some sc..ilar
function. In that (two-dimensional) case. A_r = r - x and A_1• = x +yin a simply
connected region. and

JA oA
__
x = I=---"
ay ax
Therefore. A is equal to the gradient of a scalar function. We can even detemiine
1his function <f>(x. y) by writing

a<J) a<j)
A .= - = ,. -x and Av=-=x+v
·' ax . . ay .
lntegrnre o<J>/Jx ··partially"" with respect 10 x with y "cons1ant·· to obtain rf:,(.x. y) =
xy - .r 2/2 + j(y) . where f(y) is the "constant" of in1egr.1tion. Now fom1 o</J/oy
and equate the result to 'd<J,/"d_,. = A_1• = x + y to get
318 Ch,1p1. •r 7 / Vcclor Cakutu

o<P df
-=x+-=x+y
iJy dy

or df /dy = y. Integration gives us /(y) y 2/2 = + c. where c is a constant. Thus.


we see lh al q, (x, y) = x y - x 2 /2 + y2 / 2 + c.

Example 5:
Is the integral f
A · dr path independent ir A= (2.r - y) i + (x + )') j'?
f
Evaluate A · dr around n unit circle centered at lhe origin. What is the
value or the integral if we take a clockwise direction?

SOLUTION:

aA aA,.
_x=-1'#-· =I
iJy iJx

so the imegral is not p:nh indcpcndenl. Thi.s means that integrals around
closed paths will generally not be equal to zero.
To evaluat1: f A • dr in a coun1crclockwisc direction around a unit circle
centered at the origin. let x =cos" and y = sin u and write

f A• dr = fo
2
rr [(2 cos 11 - sin 11)(- sin u) + (cos u + sin 11) cos u] du

{2lf
= Jo (I - cos II sin 11)du = 2rr - 0 = 2;r
The vaJuc or {A• dr in a clockwise direct.ion is -2rr.

Equations 9 give the necessary conditions Lhat A = grad <t,. Do these condi-
tjons look familiar? If you compare them lo Equal.ion 17 of Lhe previous section.
you'll see that they are similar to the terms in 1he definition of curl A, and in fact,
Equations 9 say that curl A = 0. Thus, Equations 9 arc equivalent to the identity
curl grad (/J = 0.

A necessary and sufficienl condition 1'1111 A be palh i,ulepe11denr for uny parh
lying wirhin a ximply co1111ec1ed regio11 R is thor curl A -= 0 i11 R.

For the vcct0r A in Example 3. curl A= 0. but in Example 4, curl A= 2 k i=- 0.


J F • dr is the work done in going from a to b
If the vector Fis a force. then
along some pnt.h C. The work will depend upon the path unless F = -grad t/J (the
minus sign here is just a convention), in which case F is sajd 10 be conservLJtive
(or a co11servative force field) and </)(x, y, ;:) the potemia.l. To see why F(.r. y, z)
is called conservative. consider Newton's equations of motion

( 13)
319

Take 1hc dol product of bo1h sides of Equation 13 by dr and imegralc. The left ,ide
becomes

m
J
2
-d r • Jr
dr 1
=m • r -d-:!.r - dr
rfr'!. d1
-di = -2 III J- ('J-r •-
d
tf I dr
dr) dr
d1
")
Ill v-
= --
2
+ cons1an1

The left side of Equation 13 becomes

-j 'v </J • dr = -<P + constant

Equa1ing 1hc results of the two sides of Equation 13 give"


)
m v-
- - + <j)(x. y . .:) = constant
2
or that the total energy is conserved. Thus. a conservative force field implies that
energy i~ conserved .
Although we have llsed an example from dassical mechanic~ to in1roduce the
idea of a conservativi.: field. any vector field v(x. _r. :-). which can be expressed a.-.
the gradienl of a scalar field </>(x. y. :). is called a conscn·{l/i\:c, ,,coor field and <J,
is called 1he scalar pote11lial.
Before we fini~h this section. we shall discuss an important relation between
line integrals and surface integr..ils in a plane, Let R be a clo .. c<l region in the
xy-plane and let lhe closed curve C be the boundary of R. The :trrow!i on C in
Figure 7. I 6 indicate a cuunlcrclockwisc dircc1ion on C. where the region R is
Figure 7.16
always on the lefl a1_; we go around C . A closed region in the .ry-planc and iL~
If C i-; described by the pamm~tric equations x(I) and y(I) and if F(.r. y) = houndary curve.
P(x. y) I "1 Q(x. y)j. then the line in1cgral off around C is given hy

f
r
f
F · dr = I P d.x + Q dy I
C
( 14)

As usual. the integral in Equation 14 is 1aken in a l'Ounterclodwise direction . We


now stare Gret:11 ·s lh(>orn11 in rhl! pla11e;

If R is a simply co1111ected region in the xy-plane hounded by a piecell'ixe


smoorh cun•i: C, and 1f P(x. y) c.md Q(x. y) art' continuous u-irh co111i11uo11s
firs/ partial dt·rivori,·e.\· in mi npt'/1 rt:gio11 cnn1oi,ii11g R. then

aQ
1J( - - -".,,,)
f
C
l P dx + Q d .,. l =
R
i-)x
- dx d .v
Jy
( 15)

(Problem 15 takes you through a simplified proof of this theorem.) Equation 15


will be used a number of limes in later chaplers.

C
320

Let's verify Green's theorem in 1hc plane for P = x 2 + y 1 and Q = x + 2 over


the boundary curve of a triangle with vertices at rhe poinLS (0. 0), (0, I). and (L, 0)
(0. 1) =
(Figure 7. 17). Using y I - x along the segment 2 in Figure 7.17. we have

rO [x 2 +(1-:d-(x+2)ldx + j
1
1 •O

f [Pdx+Qdy\=
0
x'!.dx+
~ I I
2dy

1 ll l
=-+--2=-
3 6 6
(0.0) 1 ( J. Q) X
Now.

Figure 7.17
A triangular region in 1hc xy-pluni:: with
vertices :it the points (0, 0). CO. Ii. and
ff(~; -::)
R .
dxdy = ff
R
(I -2y) dxdy

(I. 0).
I [1 ('~ I 1 I
= 2- 2 Jo dx lo Y dy = 2- 3= 6
We finish this .-;ec1ion by expre~sing Green's theorem in 1he plane in vccror
no1ntion. We can write P(x. y) dx -i- Q(x. y) dy as

P dx + Q dy = (Pi+ Q j) · (dx i + dy j) =A· dr

where A= Pi+ Q j. Now fonn 1he curl of A:

j
a i)
curl A=
ax ay
P(x.y) Q(x. y) 0

Equation 14 becomes

f
C
A · dr = fJ
N
(curl A) · k dxdy (16)

Equation I6 is the planar version of S1okes's lheorem, which we shall discuss in


Section 5.

7.2 Problems
I. Evaluale j F · dr along Lhc par.imc1ric curve x = u~, y = u from u = 0 10 11 = I if F = -y i + x 2 j.
C

2. Evaluate/ F • dr from (0. 0) 10 (I. 2) along the parabola y = 2.r 2 if F = .ry I - _\'~ j.
C

3. E,·aluorc f A · dr countcrclockwise around a unit circle centcrc.d al the origin if A= y i - x j.

4. Evalunre j F • dr from (-1. -1) 10 ( I, I) along the curve x = y 3 if F = ,, 2 1+ .r j.


C
321

5. Evaluate f A • dr along the stra.ight line (0. 0. 0) to ( I. I. 3) if A= x~y i + y 2.;:j + ;:::2.x k.


C

6. Evaluate f F • dr countcrclockwi~ around a unit circle ccntcrcd al the orig-in ir F = (-y i + .r j)/(.r 2 + y 2 ).
1
7. Evaluate A• dr countcrclockwise around a unit circle cente-red at the origin if A - xy I y 2 j. Is I.he vec1or
field A conserva1ive? Now evaluate !he integral coun1erclockwise around the square whose c.;omers arc (0. 0).
(I. 0). (I. Ii. and (0. I).

8. Docs A(.\". y) = (3x 2 + 2i) I+ (4xy + 6y 2 ) j rcpre:;ent a con:-crva1ive force field? If so. tleterminc 1hc
potential c/J in A= grad </).

9. Find a potential function corre~ponding to F = (x I+ y j)/(,\" ~ + y~) V~.


IO. Docs F(.r. y. :::) = yz I+ .r;:J + xy k represent a conL1-ervntivc force field"? 1f so. dctennine (jJ in F = gratl ,P.
11. Docs F(.r, y, ;:) = 2ry.: i + x 1 ;: j .... ( x 2 y + 4.:) k represent a conscrv:uive force field? If so. dctcnninc <JJ in
F =grml t/>.

12. In t1u id dynamics. the quantity K .;::::: f v • dr. where v is the velociLy vector or the fluid, is callerl tJ1e circ11/,11ion.
and is w,ed to describe I.he character of the fluid flow. The c:ircula1ion depe:nds upon the inlegration path as
well as tJ1c vector field. Detcnninc K counicrclockwisc around a unil circle centercd at 1he origin if

(a) "= v0 I (b) v = 2xy i + (x:! - y:!) j

13. Consider the line in1cgrnl / F · dr, where F(x. y) ~ i + ,....::---, J.


= - x- I.~ 1his line in1cgral path
.
C
+ \"• .\". + ,..
- .
independent? In whal dornnin? Do you think 1.ha1 f F • dr = 0 if the closed integration path enclo,cs the origin?
.
14. Consider the vcc1or F = - ·.,., I + X, j . Does F(x. y) represent .1 conscrvnt1vc
.
force field? Now calcula1e the
x- \'-
work along a uni1 circle ccntercd at the origin. Whal happened? Why?

15. We sh;:tll prove Green's theorem in the plane for 1hc special ca-,c in which the closed region Rand i1s boundary
curve C have 1he property that any straight line parallel 10 1hc coordinate axes cuts C in al mos! two place"
(~ee Figure 7.18).
Let the upper curve ABC in Figure 7. l8 be described by _,· 2 (x) and the lower curve ADC be described by
y 1 (x). Now show that

11, /,'',(d dy-_-= 11,


Ji
R
'JP
'-_.-dxdy=
<Jy a
dx ·-
\' 1(X l
.
JP
0_\' I/
d.r[P(x.y 2(x))-P(x._,· 1(x))I

X
Figure 7.18
TI1c region u,.-d in Problem 15 10 prove
Gn:t!n"s theorem in the plane. D

C gl
322 Clnp1cr 7 / Vr! ror ·alculu~

an<l 1ha1 this result is equal to i


P dx . Similarly. lei che right boundary curve BCD be described by x 2(y) and
the left boundary curve BAD be de:-cribed by x 1(y) and show that

j {[ 'JQ
j 'iJx d.r = f Q Jy
R C

and hence prove Equation t 5.


16. Verify Green's theorem in th~ plane for P = y -x 1 and Q = 2x +_,.:!over 1h1:: boundary shown in Figure 7.19.
\'

D
figure 7.19
The region and bounc.fary c:urvc used in
Problem 16.
-1 4 X

17. Use Green ·s theorem in the plane 10 shllw thill the area bounded by a simple closed curve C (a closed curve
that does not cross itself) is given by A = lf (x dy - y dx).
C

18. Use rhe result of the previous problem to find the area enclosed by the ellipse described by .r~ /u 2 + y 2 /J:J. = I.
Hint: Use the parametric equal.ions x ::::: o cos 0 and y =
h sin (i for O ~ 0 :::: 2;r.

19. Given 1ha1 <I> anc.l if, arc continuous functions of x and y with con I inuous first panial deriva1 ives. use Green ·s
theorem in the plane to derive the relation

11( i'Jx
Ot') dxclv=
01' + ,JJ-.'..
IJ>-
Jv ·
f v(q>d\'-1/Jdx)-
· ·
ll P -+ -[liJ,)
(iJ<f,
ox ·-
or• dxrh
·
R . C R

Compare rhis formula to the formula for in<egration by parts. Hint: Set P = v,J> and Q = -L'lf in Equ:11ion 15 .

7 .3 Surface Integrals

We have discussed line in1egrals in the previous section and in this section we
shall discuss surface integrals. Before doing so. however. we shall discuss surfaces
a li11lc more lhoroughly than we have previously. Ree.ill 1ha1 ..i surface in three-
dimensional space can be represented parametrically by

r(11. u) = x (11. u) i + y(11. v) j + :::(u. v) k ( I)


7 . .J Surf.lee lnregr .1 Is 323

- I
,, \'

, I

Figure 7.20
A m:ipring fmm 1he 11tJ-plane 10 x. _1. ~
i.patt. In panicular. lhis mapping is
\'
from O 5 tJ> _ 2.'1' . 0 _ (} 5 ,T 10 a sphere
by the rrrnpping x = 11 sin fl cos rJ,.
II
y = 11 sin A sin 1/i . .ind:: = a cos 0.

As 11 and v vary over a region in the 11v-planc. the tip or r = .r i + y j +: k maps


ou1 a :-urfacc in.,. y.: space. Pigure 7.20 illus1ra1cs a mapping from 1hc II v-planc
to x . _v. :: sp;.u.:c.
We s..iy tha1 r(,,. l') = x (11. l-') i + _v(11. v) j + ;:(u. t•) k describes a smooth
swface if each of the functions .r(u. Li). y(11. u). and :(11. ti) and each of their first
panial derivatives are continuous over the region of,, and 11. and if

dr Jx Jr a-
-011 = -811 I :- -=- j + __::_ k
011 i-Ju
and

are nonzero and nonparallel over the domain of II and LI. Recall that ilr ji)11 and
ar / J t' arc ta11gcn1 to the surface. and so
rlr Jr
- x - :;= 0 (2)
i-l11 ov
is nonnal to the surface. This last condition assures thar the surface has a t::mgl!nt
plane at all point, (x. v. : ) and is the analog of a space curve having a non zero
derivative everywhere. We say that a surface is piece,n'se smooth if it consis1s of
a finite number of smooth surfaces. For example. we say that a cube is piecewi!--e
smooth because it con~ists of :>ix smooth surfaces.

Example 1:
Describe 1he surface deli ncd by r( 11. 1•) =11 i + L' j + ( I - 11 - t') k. Evaluate
-Jr x -Jr f or this
. sur f :!l'C and .in1crprc1 L I
Mc rcsu I.
au ih•
7.3 Surface Integrals 327

There is one other variation of the formula for surface area that you commonly
=
see, particularly if z f (x. y) does not change sign as .r and y vary over some
region R. In tha1 case, R is rhe projection of S onto the xy-plane. as shown in
Figure 7.27.
Let y be the angle between the upward-directed normal to a surface elemen1
!:!,.S and the z: axis. so that cos y = In - kl. Then /J.S and its projec1ion onto the
x y-plane. /J. R. arc approximate] y related by /J. R = /J. S cos y. or /J. S = /J. R sec y.
This rda1ion between /J. S and /J. R becomes exact in the limit. in which case the
surface area can be written as

A= fJ = fj
(J
dS
R
sec y dxdy = f( ~r_d:·I
R
(5)

Equation 5 is equivalent 10 Equation 4. To see this. note that 1he normal vector
=
to the surface .: : h (x. y) is the gradient of </J(x. y . .:) = z: - I, (x. y) = 0. which
IS X

iJI, • i)lz • Figure 7.27


'v</J=--1--J+k or a ~urfm.:c clcmcnl tlS
ox iJy The project.ion
onto the xy-plane.
and the unit normal vector is

But co~ y = n - k. so we have sec y = 1/n . k. or


r11 ~~ a, 1] ~ 1/2
sec y = [
I + ( ~)
Dx
+ ( ~)
rly

which shows the equivalence of Equation~ 4 and 5.


So far we have considered only the calculation of surface areas. lntcgr.ils of
the type

ff
s
.t(x. y. ::J dS

oc.:c.:ur frequently in physical problems. To evaluate these types of integraJs.. we


simply use the generalization of Equal.ion 3:

11
s
a
a I d11dv
j(x.(11. L~), y(11. u). ;:(11, v)) __..!:. x _i:
l dll dU
(6)

if the surface is given in parametric fonn, or we use


328 Chilp\ r 7 / Vector Ca l ulus

;: ff .r<x, y, :) I+ ( ~:) ~ + ( ~:~ )-


1 ']l/2 dxdy =ff f(x. )',:)sec ydxdy
[

R R
(7)
if 1he surface is described by an equation of the form : = h (x. y).

)' Example 4:
Evaluate the ~uriacc integral

I= ffxy: dS
s

I X overt.he plane x + y +: = I in the first octant.

Figure 7.28 SOL u TIO N: The plane and its projcc1 ion on10 1hc x .,·-plane are shown in
The plane, S, decribe<l by x + )' , ~ = 1 Figure 7.28. lfwe use Equation 6, we parametrize the plane by x = 11, _\ ' v. =
and i1s projection. R. onw the xy-plane . and::= I - 11 - t!, ., o 1ha1

r = 11 I+ 11 J+ ( I - 11 - v) k
and

ar I .
ar = 11+ J + kl =
I
h
- X - V .1
011 ov
Equation 6 give~

I= J3 !!
R
1111(1-11 - t1)d11dv

u
where 0 < u < I am.l O ~,.<I -11 (see Figure 7.29). Thus./= ../3/120
(Problem 15). Of cour~c. we would have obtained t.he same result if we had
Figure 7.29 used Equation 7 instead of Equation 6 (Problem 16 ).
The intl-grntion region R in Example 4.

Recall that a flux J is the flow rate of a substance through a unit area pcr-
pcndicu lar to J. Consider lhe flux of a substance (or even the flux of some kind
of vector field. such as an electric field intensity) through a surface element dS
who~c normal oul ward unit vector is n (see Figure 7 .30) ,
Then the flow ra1c through d S is given by J • n d S, and the 101al flow rnte out
of a volume bounded by the surface 5 is given by

I= ff
s
J · n dS (8)

If we represent the surface dS = n dS. then Equation 8 takes the form


7 .J Surface l111ewals 329

(9)

Surface integral!; of the fonn of Equalion 8 or 9 occur frequently in physical


problems.

Example 5:
I
Let the llux of a fluid be given by .I = ptr0 k. where p is the mass density of "
the fluid and v0 k is its vi::hxity. C:ilcularc the flow rate of fluid (mass per
unit time) through a hemispherical surface of radius 11 (Figure 7.31 ).

SOLUTION: The equation of the surface is:= (a 2 - .r - y 1 ) 1i 2. and so I X


n is given by 'v q, /I 'v 4, I. where </J = ;: - fo 2 - x 2 - / 1) 1/ 2 . Therefore.
Figure 7.30
a,JJ • n</J ilt/J .r • r The t1u.x 1hrough a direc1cd surface
'v<J, = - I+ - j +- k= - I+ :... j + k element dS = n dS.
ilx c1y il: :: :::

and

J = p vo k j -

Therefore.

'vt/> xl + yj + :::k
n= - - = - - - - -
l'vit>I o

and J · n = pi:0 k · n = p1•1):./n. Equation 8 gives us

I =ff Pt:•·o: d5 IX
s
Figure 7.31

= -/H'I I
a
iJ [ (ax
: I+ -J/i )] + (
ar.
2
-i)/1 ) ] IO d.rJv
.
The flux of li4uid 1hruugh a hcmi~phcrical
don~ of radius 11.
R

We sec rhat the total now through the hemispherical cap is just the upward
flux times the area of the projection of the cap onto the .ry-plane.
330 Chaprcr 7 / Vccior Calculus

7.3 Problem
I. Express I.he following surfaces in parametric fonn:

(a) \' = ;:2 (b) x + 2y - ;: =2


(d) x2 + y 2 = z
2. Sketch lhc surfaces given in Problem I.

3. The vector r(11. v) = e" I+ t1 j + (e2.i' + v~) k represenis the circular paraboloid x 2 + y 2 = ;:. Docs it represent
the enlire paraboloid'!
4. Use Equation 3 10 dctcm1inc Lhc area of the cylindrical surface x 2 + .: 2 = I from y = 0 toy= I.
S. Determine lhe area of the par1 of the plane ;: = 2x + 3.r that lies inside the elliptical cylinder described by
., ")
x~ )'M
- + :_=I.
4 9
6. Determine the area of 1hc plane 2x + 3y + .: = 6 rhat lies in rhc first octant.
7. Find the surface area of the torus described by the parametric equations. x(0. tj,) =(a+ b cos. 4>) cos 0.
y(O. </J) =(a+ b cos </J) sin 0. and z.(0. r/)) = b sin 0. with O < 8 < 2,r and O < </> < 2Jr (see Figure 7.32).

:. I

)'
I
y

Figure 7.32
(a) The parametric.: equal.ions lha1 generate
a torus. ( b) The torus res ul ts from rotating
the above circle about the y a:<is in (a).
X I (b)

8. Show that Ii111ar x or I= [ 1 + (oh)


i:J v ox oy
2
+ (!!!!_)
2
] I/~ if r = x I+ ,. j
.
+ Ii (x. \')
.
k.

9. Evaluate the surface integral ff (: + 2x + ~ .:-) d S over that part of the plane 6x + 4 y + 3z = 12 that lies in
s
the first octant.

IO. Evaluate the surface integral ff xd.S over1ha1 part of the sphere x 2 + y 2 + ~2 = a 2 I.hat lies in the tirst octant.
s
11. Evaluate 1he surface integral / / F · dS over 1he triangular ~urfoce bounckd by thl! point:- (2. 0. 0). (0. 2. 0).
s
and (0. 0. 2). where F = x I + y j +::. k and n is 1he outward normal unit vector from the triangular surface.
7.4 The Oiveri;ence Theorem 331

12. Evaluate the surface integral ff F · n dS over the cube O < x < l. 0 < y < I. 0 <.: < I, where
s
F = .x i + y j + 2 k and n is the omward nom,al unit vector from each face of the cube.
13. Evalua1c the surface integral ff F • n d S over the surface of a unit sphere cemered al rhe origin. where
s
f = x I + y j + : k and n is the outward nonnal unit vector of the sphere.
14. Ev.iluale the surface inteh,•rnl If (6x + ;: - y 2) d S O\'er the surface deli ned by r(11, ,~) = " i + 11 j + 11 k and
s
for O < 11 < I and O < i> < I. What is the surface?

15. Finish the calculation in Example 4.

16. Use Equation 7 to evaluate the surface integral in Example 4.

7 .4 The Divergence Theorem

In Section I. we saw that the divergence of a vector rcpresenls the flow rate of some
quantity out of a given region in space. and in the previous section. we learned tha1
a surface integral of the type

f
(J
F- n dS = f
(J
F · dS (Il

represents the now rate of the quantity represented by F through a surface S.


Therefore. it may not be surprising that there is a relation between div F and a
surface integral like I.he one in Equation I. This relation is given by the divergence
1heore111:

If S ix a pieceu·ise smooth .rnrft1ct.' enclosing 1.1 1l1re(.'-dimensional n..·gion V.


1hen

ff f. n dS = !ffV
'V · F dV (2)

ll'here n is the 011/wnrd 1111ir normal 1·ector to S. i.e., rhe 1111ir 11on11al vector
pvi111i11~ m,·oyfmm V.

I
The divergence theorem was obtained independently by Gauss and Ostrograchky. / x
but is referred to as Gauss's theorem in the Western literature.
Figure 7.33
Given the physical meanings of the two sides of Equation 2. Lhe divergence
A d osed !-Urfocc consis1ing of an uprcr
theorem is almost self-evident. but we shall give an ou1linc of its ma1hcmatical surfoc:c S 2 dc¥ ribe<l by .:: = /:( r. yl.
proof. Let the surface S be such tha1 it can be decomposed into an upper surface <1nd a lower surface S 1 <.lesc.:ri ~d by
S2 described by.:= h_(.x. y), and a lower surface S 1 described by;:= j 1 (x, y) - = J1(x, _I'). The projection of1he surface
onto the xy-pbnc is shown by the doM.--<l
(Figure 7.33). If F(x. y. ;:) = Fr(.r, y. ;:) i + F_.,(x. y . .:)j + F:(x. y. ;:) k. then curve.
7.4 The Div rgen e Theorem 333
The left side of Equa1ion 2 is

ff
s
F-ndS= ff
lop put
:dS+ ff f-ndS- ff
honorn p:in
:t!S

= (4rr)(4) + ff
~-un'~d pm
F - n dS + 0

where we used the fact that : = 4 on the top ·urfac:c und;:: = 0 on the bouom
~urface. For the remaining in1cgral. we use <J> = x 2 + v2 - 4 to get

'il<J, 2xi+2yj
n = --
[V<J,[
=- - - - - = xi+yj
(4x 2 +4_r~) 112
- -2 -

anJ f • n = (x
2
+ y 2 )/2 = 4/2 = 2. Therefore.
-, n
.ff F-ndS=2(27T -2)4+ 16,; =48JT
s
s
The divergence 1heorem is used to derive many of the cqua1ions of physics
and engineering tha1 involve some son of conservation condition. such as mass
balance or energy balance. Lei p (x. _r. .:: . 1) be the mass density of ::i fluid at the
poinr (x. y. :::) at lime t. and let V be an arbi1rary fixed volume located within the
fluid and let S be the boundary of V (Figure 7.36). The total mass within V is Figure 7.36
An :ubitrary fi,;cd volume V located

M = !ff
V
pdV
within a nuid: Sis the boundary or V.

and the rnle of change of lhe m:.iss within V is

(7)

since V is fixed in space.


Now. because of 1he conservation of mass. the ra1e of change of mass within
V mus! be balanced by the net r,lle at which mass flows through 1he surface S. This
rate is given by

dM= -
-
d1
11 pu-ndS .
s
where n is the outward unit nonnal vector at dS and u(.x, y. ;:, 1) is the velocity
of the fluid. The negative sign on the right of Equation 8 accounts forthe fact that
d M / d, < 0 if the net flow is outward. Equating d J\,f / d I from Equatjons 7 and 8

al
334 Chapler 7 J Vector Lile ulus

gives

JJJ:~dV=-
V
ffs pu-ndS (9)

Apply Lhe divergence Lheorcm. Equa1ion 2. to the right of Equation 9 to obtain

fff ~dV=- ffJdiv(pu)dV


V \I

or

The volume V in Equalion 10 is arbitrary. however. so the integrand musl equaJ


zero. or

ap + div (pu) = 0 ( I I)
a,
Equation 11. which ex.presses the conservation of mass in differential form,
is called the co11ti11riity equation. We acmally derived this equation in Sec1ion I
for the !-.pecial case of a rectangular parnllelpiped. but the derivation here is more
general.

Example 2:
Let T(x. )', z. t) deno1e 1.hc temperature ofa homogeneous body al 1hc point
(x. y. =:) at 1hc rime , . Fourier's law of heat now ~ays that q. the flux of
energy as heat. is given by

q = -K grnd T

where K is the thermal conduc1ivi1y of 1he body. Derive I.he heat equation

where a 2 =
K / cv p. where cv is the specific heal of I.he fluid ( heat capacity
per uni1 mass) and p is the mass densily.

SOLUTION: We're going 10 express the conservation of energy in


differenlial form using the divergence theorem. bur before doing so. we
must express the energy in terms of lhe temperature T. The product of the
specific heal rv and the mass density p is the hent cupac11y per unil volume.
If we let II be lhe energy density within 1he volume I/. then

11 = cv p T + cons1ant
335
where the con.'-lanl simply J-ets the 1..ero of energy. Now. the rate of change
of energy U wi1hin an arbitrary volume V is given by

-JU
J,
= 1JJ1/ -d'-1
011 . =
a, .
ft}. aau ara,
-
T
- dV =cvP 1IJ. -dV
uT
a,
V V V

where we used 1hc nbovc cqua1ion to replace au/ 'd T by cv p. T11c flow rate
of energy acm~, the surface S due 10 o 1cmpera1urc grndienl is given by

ff
s
q · ndS= fff \I
divqdV

where we have usetl the divergence theorem. Equation 2. Equating the rwo
rnres of change of energy gives

CvP fff ar;~


V
dV =- ff
S
q. n dS =- fff
I'
c1iv q dV

,vhcrc nncc again the ncgaLivc sign accou1.11s for the fact thar n is a unit
nom1al owward vector. Because the volume \/ is arbilrjry, we have

cvp-
ar = - d1v. q
a,
=
Fourier's law of hear flO\~ ~ay~ that q - 1< grad T. however. so lhe righ1
side of this equation is K div grad T = KV-T and we have the heat equation:

fJT ,,
cvp - - K'v-T
clr

Tile heat equation models the Iempera1ure distribution throughou1 a homo-


geneous body. Notice 1.hat the he.it equation is es:-.cntially lhc same as the diffu-
sion equation (EquaLion 15 of Section I). The reason lhcy arc ~n similar is that
1he diffusion of a ..;ubslance or the flow of energy a, hl:at hav~ :similar molecular
descriptions. Like the diffusion equation. the hear cqualion is a paniul differential
equaLion, which is the suhjec1 of Chapter 16.
The divergence theorem plays an important role in clcctros1atics. Recall that
1he elcclfic fielu intt.·n,ity produced by a charge q locarcd al 1hc origin of a
coordinate system is given by Coulomb's law.

( 12)

where er is lhe uni1 vec\or (x I + y j + ~ k) / ,- and i 0 is the pcrmi11ivity of a vacuum.


Let's consider the nux of E through a closed surface surrounding lhe charge. The
336 Chapter 7 / V to r Calculus

z total flux et>£ will be given by

q 11·e,..
cl> £=
ff E · n dS = --
4rrc 0r~
n
--')-dS
s
dS Before going on, lei's review rhc idea of a soli<l angle. Let S be an area on a unit
sphere ccntcrcd at the origin. All the rays starting at the origin and passing through
S fonn a cone. which is called the solid nntle Q. We say 1ha1 Q is subtended
\' by S. The units of solid angles are steradians, just as the units of planar angles are
I radians. Figure 7.37 .shows the .solid angle dQ subtended hy dS.

/I Just as the arc length ds on a circle is related to the angle d0 (in radians)
that ii subtends by ds = rdO. where r is the rndi us of the circle. d S is related lo
I
IX
the solid angle dQ (in steradians) that it subtends by dS = r 2dQ, For example. if
r =a= constant. then the total surface area of the sphere is S = 4rrn 2 , su that a
Figure 7.37 complete sol i<l angle is 4JT, just as a complete angle for a circle is 2;r.
A solid angle clement JQ. Unless S happens lo be a sphere. er and n will not be parallel, but e,. • n dS
wi II be the projection of d S onto the sphere of radius n (Figure 7. 38). so that

[ [ E · n dS = _q {f e,. / dS = _q {[ dQ = J... ( 11)


11
s
4rrE0 JJ
s
r- 4rrf 0 JJ
s
Eo

Equation 13 is a fundamental equation of elecLro-.catics called Cam·.1· ".I· luw. We


can transform Equarion 13 into another well-known equation by writing

q = f p,.tlV ( 14)

where Pc is t.he charge density within V. Applying the divergence theorem to


Equations 13 and 14 gives

div E = Pr- ( 15)


figure 7.38 t:o
The projection of er • n ilS onto the
surrucc of a sphere of radius o. which is one of Maxwell ·s equations.
lfwe define the elec1rosta1ic po1en1ial by 'v</> = -E. then Equation 15 becomes
Poisson·.~ equation.

( 16)

which gives the electrostatic potential </J (x. y . .:) due to a charge dist.ribut ion
pJr. y. ;:). Lastly, for a charged free region. Equation 16 becomes wplace's
e1Jtwtio11.

one of the fundamental equations of physics.


7.4 The Divergence Theorem 337

7 .4 Problems
The first four pmhlems are mean/ as a re1r'iew of triple integration.

1. Calcula1e the volume of the solid body bounded by the three coordinate planes (x = 0. y = 0. and z = 0) and
x + y + ;: :;: a for a > 0.

2. Evaluate the integral/= fff dxdyd:. (x


2
+ y2 + z2) over I.he same volume as in Problem I.
V

3. Evaluate the integral JJf dxdyd:. xyz over the volume of a tetrahedron wilh vertices (0. 0. 0). (I. 0, 0),
(0. I. 0). and (0. 0. I) (Figure 7.39).

Figure 7.39
A tetrahedron wi1h vcniccs !O. 0. 0).
( I, 0, 0). (0. 1, 0), and (0, 0. 1). ' X

4. Evaluate the intcgr.i..1 /=fff dxdyd: x:\ 2 1


:. over the volume of an octahedron with vcniccs (±1. 0. 0).
(0, ± 1. 0). and (0. 0. ± I).
5. Evaluate the surface intcgr.11 / = Jf F • n d S if F = x:. i + y: j + : 2 k. where S i~ the surface or the sphere
s
des.cribed by x 2 + y2 + ~2 = 9. and n is the outward unit nonnal vector.
6. Verify the divergence theorem if F = .r I + y j +.:: k and S is the surface of the cube bounded by the rhree
coordina1e planes and the planes x = I. y = I. and ;: = I.

7. Evaluate the surface integral / = jf F • n dS over the surface of the sphere described by x 2 + y 2 + : 2 = 16
if F = ,, y2 i + y: 2 j + :x 1 k. s
8. Verify 1hc divergence theorem if F = xy 2 I + .r 2 y j + y k and S is the right circular c.;yJinucr ue~cribcd by
x~ + y2 = I. - I < - < I.

9. Use l11e divergence theorem to evaluate 1he surface. integral / =/ J F · n dS. where F = xJ i + y-1 j + :: 3 k and
s
S is the surface of the cylinder x 2 + y 2 = 4 between - I < :: < I.

Copy 1gh al
338

I 0. 1n Problem 17 of Section I. we in1roduccd !he definition or div ,, given hy div v = lirn ffs " · v d S. Use the
V-0 \,"
divergenc:e theorem 10 verify this relation. Hi111: Use a ~cncrnlization of the mean value theorem nf integral ion
lo 1hree dinwm,ions.
I 1. If J and g are :-ea Jar functions suc.:h that F = .f V f.! has continuous first partial derivatives in some region R.
show that fff(f'v M+ 'vf ·'vg)dV =ff f ;~ dS. where iJ1,:/iln deno1cs 1he directional dcriva1ive or gin
2

V S
the dircc1ion of the ou1cr normal 10 S. which bounds V. This relation i~ known a:,; Gn·en ·.1·.fir.H ide111iry.

12. Using 1he rc.<.uh of Problem 11. prove Green's se<.'(md idenlity,

jffuu g-.~u ndv= f/(1~!-,<~)ds.


2 2

V S

13. U$c Green's first identity to :,;how 1hat !ff l'v fl~dV = !! !:, dS.
V S

14. Use the divergence theorem 10 J


show that// V f dV = Jf Jn dS. No1e that 1he resull here is a vector. Hi11r:
V .f
Cm1,idcr 1hc vector F = Fe. where c is an arbitrary consrant veclor.
15. Use the resull of the previou.:; problem 10 show that jf n d S = 0, where S is any piccewi~c ~moo1h closed
s
f;Urfoce :md n is the ou1ward unit normal vector.

16. Use Equationl'.i D and 14 to show 1ha1 if p, (.r, y. ;:) is spherically symmetric. 1hen the electric field E 1har is
ob:-erved at any point that is outside the charge density is the same as tha1 due lo a charge q = f Pcdt, located
at the origin.

17. Show that the result of Problem J 6 also appliC', to the g-ravita1ional force due to a mass distribution.
18. Show 1hat the divergence theorem in two dimensions is or 1hc fonn
ff div v dS = f n · \' ds. where v = i 11.~ +j L'y- Hi11r: Consider a slab-I ike volume V of height I, with
R C
base R in the::. direction, and then apply the divergence theorem to v in V.

19. Show that .ffB · n dS = 0 for any closed surface if B = curl A.


s

7 .5 Stokes's Th eorem

The divergence theorem relates a surface inle~ ral to a volume integral. There also i$
a rheorem, called Stokes ·s rheon·m. thal relates a Iine integral to a surface integral.
we·ve already cni;oun1ered a relationship be1wcen a line integral and a surface
integral. Al the end of Section 2. we presented Green's 1hcorem in lhe plane in
vector notation:

f
C
A· dr = f
R
/(curl A)· k dxdy ( I)
340 Ch.1pll'r 7 I Vector Calculus

region ointt1ini.11g S. then


GGGGGG f jj
G~C1GGG C
v • dr =
S
(V x ,,) • n d S (2)

cG s GG where C is trm•ened in the dircc1irm with re.1pect to n in accord wi1h rhe

G €, .I . GG righl-lwnd rule.

G G The proof of Stokes·s theorem is fairly involved but we'll give a suggestive
physical argument of Stokes"s theorem. Figure 7.43 shows a surface S and a
G __ GGG boundary curve C. where the surface is partitioned into a mesh of surfaces dS 1
with boundary curves c1 . Just for simplicity nt this point. let's evaluate f v - dr
GGGC·G for a small rec1angle in the xy-plane. First we'll determine the contributfon from
sides I and 3 in Figure 7 .44.
Figure 7.43
A surface Sand :i boundary curve C.
where the surface is p:i.rtitioned in10 a
mesh of 1,urfoces d SJ wilh boundary ! 1
v - dr
{
+ 1, v. dr = 1-r+6..r
J .r
I)~ (.x'. y)dx'
1·r
+ .
.,+j.1
l-'_, er'. y + fi_v)dx'
curves c1 .
.r+6..t
=- J

{v_((.r. y + 6y)
I
- t1_r(x . y))dx
,

1x

y 3
y + ll. y aVr i •
Now vx(x. y + 6y) - v.,.(.r. y) =-a· 6.y + [(6_rfl and so we can wrne
y
4 2
= _avx dxd\"
y
X f l+J
V. dr
Dy .
X X + 6.X Similarly. 1he in1egrals over side.s 2 and 4 in Figure 7.44 give
Figure 7.44 ;:)i\
A pic1orial aid lo 1.he evaluation of
f v · dr around one of I he ~urfaces d S J
1~+-I
\I • dr = - -- -dxdr
i)x .

in Figure 7.4~. and so

(3)

The right-hand side of Equa1ion 3 is 1he ::-component of 'v x v dxdy. and so we


can write Equation 3 as

f ,. · dr = (V x v) · k d x d y (4)

C)

Generally d Sj in Figure 7 .43 will not Iie in I he x y-plane. and so we replace k by


n and dxdy by dSj in Equation 4 to give

! v-dr=('v x v)-ndSj (5)


CJ
7.5 St ,, ' Theor •m 341

We're almost finished. If we sum over all the area elemenl!-1 in Figure 7.43. we
see that all the interior line integrals vanish because the integrals along the borders
of adjacent area elements are in opposite directions along lhe same line. The only
contribution 10 the sum of the line integrals on the left of Equation 5 arc due to the
curve C that encloses S. Therefore. Equation 5 becomes Equation 2. our statement
of Stokes·s theorem.

Example 1:
Veriry Srokcs·s theorem by C\•alu:.irjng both sides of Equation 2 where
=
v - y i + x j + k. Sis the hemisphere described by:= (4 - x 1 - _,· 2) 1/ 2 • -- ...
and C is J circle of radius 2 centered al the origin in the x_,·-plane (Figure 7.45 ). ,. .,
~
I
SOLUTION : The out ward unit normal vector to 5 is given in tenns of I
</)(x . y, :) =: - (4 - x2 - y2)112 by I y

x. r. k
'V</J - '+ ;_ J +
n = -- = _...,-_ __,,~ --
i + y j +;: k
= X__ .;.......c.._ _

2
1v,;1 ( ~: + -~: + ,) ''' x,
Figure 7.45
and so the right-hand rule tells us to lrJvcrsc the circle in J countcrc:lockwise The surfac~ und its boundary ,:urvc 1ha1
are use<! in Example I.
direction . Let the circle be described by the parametric equarions x(R) =
2 cos O and y(B) = 2 sin B. 0 =:: 0 =:: 2rr. Then. r(f,l) =
2 co:- 0 I+ 2 sin 0 j
and v(0) = -2 sin O l + 2 cos0 j k, and so

f1 ... · dr = lo(!r dB rc-2 sin 0)(-2 sin 0) + (2 cos 0)('.:! wsH)]


2
=4 / .-r dO = 8rr
~ ()

Now. 'V x ,. = 2 k. and so

Jj<vxv)-ndS= f{::ds
s s
= {{ __::__dxt.l\' = 2 f [duh·= 8rr
lJ In· kl - Jl - _\'
R R

where we have used Equation 5 of Section 3.

X,

Figure i.46
Example 2: Tile circular paraboloid ;: = .r 1 + y 1 and
Verify Stokes·~ theorem for,.,= i+xj+ y2 :, ; 2 k.
where 5 is the circular the tw" planes::= 0 and;:= I used in
paraboloid described by;:= .x 2 + _v 2 with O ~ • ~ I (Figure 7.46). faampl c 2.
342 Ch,1p1c:-r 7 / \',•< lor Cil< ulus

SOLUTION: Figure 7.46 shows t.hat the boundary curve is the circle
dcscri bed by x 2 + y 2 = l and ;; = J. We cun describe this c ire 1e para mel ric aU y
by the position vector

r(11) = cos u I -i ,in u j +k


fur O .::S 11 .5 2JT. The left side of Equal.ion 2 gives

ri V · dr = ln(" du I - sin
3
II + cm:2 11 J = JT

TI1e unit no1maJ vec1or is given by n = 'v J/l'v JI where f =:: · x 2 - y 2.


or

- 2.x i - 2yj + k
n = (I 4x - -, + 4_r)' II -,
and 1he curl v is given by

j k
a a a
curl v = Jx i)y oz = (I - 2y) k
.,
yv X z:2

Therefore. the right side or Equation 2 becomes

= Jf
R
I - 2y) dxdy

where we have used Equation 7.3.5. The above integral is

Stokes's theorem s.iys !hat the surface integral of curl F · n over a capping
surface S is equal 10 the line integral of F around the boundary curve of S. No1ice
tha1 this implies that jj A• n JS is independent or 1hc surface bounded by C.
s
provided A = curl F and Sand n satisfy Lhc requirements of S1okes·s 1heorem.
Let's go buck to Example I. where the boundary curve is a circle of r-.idius 2
centered a1 the origin and the capping surface is a hemisphere. We integrated
l:url F · n over 1hc surface or 1hc hemisphere in l:xampk I. but we coult..l jusl as
wel I have inlegrated over the unil disk itself (sec Figure 7.4 I ). If we do 1..hat, n = k
and we have
7 S S1okes's Thc..'Orc m 343

fJ
s
curl F - n dxdy =2 ff dxdy = 2(4rr) = 8rr

in agreement with the result in Example I.

Example 3:
Use Stokes's theorem 10 cvalua1e 1hc surface integral ff curl F · n d S.
s
where F = 3.r i - 2.r j + xy k and S is the hemispherical surface described
by x 2 + y2 + z2 = 4 with z 2:, 0 (Figure 7.45).
SOL u TI ON: Stokes·s theorem allows us to us.c any capping surface. so
let·~ use the disk of radius 2 given by x 2 + y 1 =s 4 and ;: = 0 . In this ~a.~.
= =
n k. and using curl F x 2 i - yzj - 5 k, we have

ff
s
curl F · n dS = -5 ff d:r:dy = -5(4rr) = -2Q;r

Problem 16 has you show that you obtain the same result using the
hemispherical surfacc.

According to Stokes\ theorem, if V x v = 0 in some region containing S,


then f v - dr = 0 for all simple closed curves in that region. In that case. ,, is the l i.'.
gmdienl of a scalar potential </>. or,,= Vtf>. TI1is is such an impor1an1 result in
physics and engineering 1ha1 we should be sure lo be aware of the strict conditions
involved. Thus we slate the following theorem:

If v(x. y, z:) is a vector field with comi11uo11s first-order panial derivariws


in u simply c:o1mec1t:d re~ion R. the11 V x v = 0 if and 011/y if v = Vrt, fo.r
y
tjJ(x. y. z:) dr.fined in R.

The operative phrase here is "simply connected region·· (sec Section 2). .......__.... C
A vector field F for which V x F = 0 is called irrorarional. Le1·s u~e Stokes's
theorem 10 sec why this is so. Consider a circular disk of radiui- a cen1ered al a
x,
point P with a boundary curve C (Figure 7.4 7). Then Stokes ·s theorem says that Figure 7.47

f/
A circular disk ccntcred at a point P with
f
C
F • dr =
S
curl F • n d S (6) a bound~ry curve C.

where n is perpencticular 10 the disk in Figure 7.47. Using the mean value theorem
of intcgr..uion on the right side of Equation 6, we can write

f
C
F · n = rra 2 [curl F - n),
344 hapt r 7 / \'t'< tor C,111 ulus

where ( is some poim in the disk. If we divide by ;r a 2 and let a ➔ 0, 1hen we have

1
Jim - - 2 J. F • dr
= 11-,0 rro r
tcurl F · nJp (7)
C

The rig.hi side of EquaLion 7 has a nice: physical inlcrprc1ation . Suppose that F
repre~ents (s1eady-stale) fluid flow. so Lhat F = pv. where p is the mass density of
the fluid ond v is its vcloci1y at any point. Then fc F · dr represents 1hc flow r-,Hc
of nuid around 1he curve C. Thus. the quantity [ curl F • n If', which represent, the
flow rate of nuid arotmd the point P. is maximal when curl F and n point in the
same direction. Therefore. 1hc axis about which the fluid rotates most rapidly is in
the same direction as curl F.
Stokes's theorem 1el15 us that I.he vector fie kl is irrotatfonal if and only if it is
=
a conservative li~ld; in other words, curl F 0 if and only if f = 'vtj), where the
potenlial func1ion </> is defined in a simply connected region R.
Stokes's theorem is often used 10 derive other results. For example. a wire
carrying an electric current generates a magnetic field. The relation between the
current and lhe magnetk field intensity is given by Ampere's law, which ~ays Lhat

f
C
1:1-dr =I (8)

where H is 1hc magneijc in1ensi1y. C is a closed curve enclosing the wire, and /
is the steady currcn1 crossing any surfocc bounded by C. If J is the steady currcm
crossing a unit area perpendicular 10 J (the flux of electrical charge). I.hen the
cum:nl crossing S is given by

(9)

Applying Stokes's theorem to Equa1ion 8 gives

I= ff
s
'v x H) · n dS ( 10)

Because 1he surface S in Equations 9 and 10 is arbitrary. the integrals must be


equal. or

'vxH=J ( 11)

which is one of Maxwell's equations for the case where Hand J do not vary with
time.
We can summarize much of this section by saying that if v has continuous
firs1--orcler partial derivaLives in a simply connected region R. then the following
five staremenLc; are equivalent:

al
7 .5 SrokC$ S Th1•1m ·m 345
0

I. V xv= 0 at every point in R.

2. f v • dr =0.

3. i'' "· dr is path independent.


4. ,. - dr is an exact differential.
5. Y = gr.id <f>, where ,J> is a ~l·a.lar potential.

7.5 Problems
I. Verify Stokcs·s theorem if"=::: i + ., j + y k is takl!n over the hemispherical ):iurfocc x 2 + y- + :: 2 = I, z > 0.
2. Verif-y Stokes 's theorem if F = y i +;: J+ .r k un<l S is the suriacc of the circular paraboloid described by
;: = I - x 2 - y 1 with ~::: 0.
3. Evaluate ff r • n d S where S is the surface of a sphere of radius I ccntercd at the origin.

=
4. Verify Stokes 's theorem ifv ( ,V - - + 2) i + (y: ---1- 4) j - x;: k is taken O\'er a cube bounded by planc:--x = 0,
r = 0,::: == 0. x :::: a. y =a,;:= a, excluding the face on the plane:: = 0.

5. Use Stokes's theorem 10 evaluate the integral ff<'v x v) • n dS over the surface of the circular raraholoid
s
: = 16 - (.r 2 + y 2 ) above the xy-plane if v = (x + y - 2) i + xy j + .r.: k.
6. Verify Stokes's theorem if F = (2y + ;:) i + (x - ;) j + (y - x) k and Sis the plane x + y +;: = I in the fml
octant.

7. Verify Stokes's 1heorcm ifF = 2x i ~- ~y 1 j + .r;: k and Sis Lhe su,i"ace of the cin.: ularparaboloid x ~ T \ ~ =2 - :
with x:! + _
\-~< I.
8. Use Stokes·s theorem 10 evaluate.( f · dr. where F = -_,. i + .r j + 2:. k nnd C ii- the unit <:irdc x 2 + y 2 :::: I
in the : = 1 plane.
9. Use Stokes's theorem 10 e,•,Lluatc .£. F • dr. where F = y! i - .r- J- (y · ) k and C 1s the 1rianglc fom1ed by
the poinl\ (0 . 0. 0). ( I. 0. 0). and ( I, I, 0).

10. Cakulatc f. f •dr. where F = (x 1 + /) i + .ry j - x;: k and C is the circle described by x 2 + y 1 = 4 in the
xy~plane.
It. U:,.e Sroh.\.·~·~ 1hcorcrn to evaluate the lint: in1cgral f. F • dr. where F = x 2_\.:.t i + j +·;: k around the circle
described b) x 2 + y2 =u 2
in the :c \·-plane.
12. Use Stoke ·~ 1heorem ro evaluate :fc F · dr. where F = (;: - 2y) i + (3.r - 4y) j + (;: + 3y) k and C is 1he
boundary of Lhc triangle fonned by the poinls (I . 0. 0) , (0. I. Ol. and (0. 0. I) .

13. Verify Stokes's theorem if F = y 2 i + x J + z2 k for the surface dt::!.eribed by :: = .r 2 + y 2 ,;: S I.


14. U:.e Stoke,\; theorem 10 cv:ilunre rhc line i111c.:grnl f F · dr. where F = (x + y) I + (:: - 2r + y) j + (y - :) k
and C is the unit circle x 1 + y2 = l in 1he plane - = 5.
IS. Use Stokes·s theorem 10 evaluate the line inregral _( F • dr. where F = (y - x) i + (x - :) j + (.r - y) k and
=
C is the part of the plane x + 2y + z. 2 in the firsl octnnl.

al
346 Chapter 7 I Ve tor C il ulus

16. Carry out Example 3 using Lhe hemispherical :mrfoce and show that you obr::iin the J;ame result.
17. Use Stoke~·!> theorem to show that curl grad <I>= 0.
= - y i + x j and a surface in the xy-plane bounded by a smooth
18. Apply Stoke~ ·s tbeorem to the vector field v

simple dosed curve C to show that the arcn bounded by C is given by A = ~ (x dy - y dx ). f


19. Use the fomwla derived in Problem 18 to detennine the area of an ellipse de.-.crihed parametrically by
x = a cos 0, y = b sin 0, 0.::. H::: 2rr.

20. Show that f v · dr = 21r for v given in Example 2 for the closed pa1h awund the square whose vertices are
(-1.-1).(1.-1).(1. l).and(-1. l).

21. Faraday·~ law of electricity and magneli~m i.ays that the elec1romo1ivc force around a closed cum::nt-cu.rrying
loop is equal to the negatin: of the rn1e of change of the magnetic nux through the loop. In tenns of nn equation.
Faraday's law is f
Jc
E ·di=_!!_
Jr
fs B · n dS for any fixed surface in the field. Us.e Stokes's theorem to derive
;JB
one of Max wcll 's equations, V x E = - -i}J .
22. Use S1okes·s 1heorem to show that 'v x (V j) = 0.
23. Derive Green's theorem in a plane from S1okcs's theorem.
24. We're going to derive a 1wo-diml.'.nsional \'Cf$ion of lhc di.,,ergcncc theorem in this problem. Lei v be ::i two-
dimcn,.ional field with components v.~ and v_,. and let F .;...:. - v_,. i + u.r j. Show Lhat ff curl F • k dxdy =

fJ
R
V • v dxdy. Now apply Green·s theorem in the plane (Equation 7.2.16) to the field F to oblain
R

ff
R
curl F · k dxdy = f
C
l-vydx + P_tdy] = f
C
v · (~::: l - ~:: .i) ds
= f
C
v · n ds

The two-dimensional di.,,crgcncc theorem is If 'v · v dxdy = f v · n dx.


R C
25. In this problem, we'll show how 10 find a vector w such 1ha1 ,. = curl w if we are givc:n v. In other words. we·11
determine ll vL-ctor potenrial for v. First show that

cJ wr iJ m. a _,· Olllr
. = --·
a: - --~ •
I.\, 11.= - - - -- (12)
iJ x ~ Dx ay

Now lake W.r = 0 (arbitrarily) and show thal lll: =- r


1.ro
VydJ.-' + fj)(y. z) and w_,. = 1•).' v~dx' + ijl(y. z).
· 10
where tf, and 1/J are arbitrnry differentiable func1ions of y and ;;:. Now s.ubst:itute these re ults into the first of
Equations 12 to obtain
References 347

Show that div v = 0 if v = curl wand use this result to derive t'J.{x 0 • y. z.) = aip - aifJ. This equation can
rJy a:
be sa1isfie<l by letting 1/1 = 0. in which case we have ·
tp(y. :) = J.·'· ll_c(. O• Y', :::) tly'. Show lhat one possible solution for w is then
Su

26. Show 1ha1 Lhe solution w 10 L11e equation v = curl w for a given vec1or field v is given by w in the previous
problem plus a term Vu. where u is an arbitrary scalar function with continuou~ )..ccond partial deriva1ivcs.
27. Use lhe result of Problem 25 10 find w(x. y. z) such 1hat ,. = (z - y) I+ (x - .:) J + (y - .r) k = curl w.
28. Use the resulr of Problem 25 10 find w(x. y . .:) such 1hat v = x I + y j - 2.: k = rnrl w.

Refer ence

Frank Ayres, Jr.• and Ellion Mendelson. 1999, Calc11l11s. 4th ed .. Schaum·s Ou1line Series.
McGraw-Hill
C.H. Edwards, Jr.. and David E. Penney, 1998, Calrn/11s and Arwlytic Geomnry. 51h ed.,
Prentice-Hall
\Vitold Ko~mala, 1999, Advanced Calrn/11s: A Friendly Appmach. Prentice-Hall
Jerrold Marsden and Alan Wcin..-:tein. 1985, Calculus I. JI. and 111. Springcr-Vcrlag
H.M. Schey. 1997, di1•. grod. rnrl. a,uJ 111/ that, 3rd ed .. W.W. Nonon & Company
Murrny Spiegel. 1959. VectorA,wl_\'Sis. Schaum's Outline Scrici;, McGraw-Hill
Murrny Spiegel. 1963. Adrnnccd Calrnlux. Schaum·s Ourlinc Series. McGmw-Hill

MATHEMATICAL TABLES:
CRC Standard Marliematical Tablc.t <m,I Formulae. 301.h ed .. edi1cd by Daniel Zwillingcr.
CRC Press ( 1996)
HISTORY:
Michad J. Crowe. 1967. A Hi.~tory of Vei·10r A11alysis. Dover Public.alions

C gl
C H APTER 8
Curvilinear Coordinates

Although cartesian. or recwngular. coordinaIes arc Lhe first ones we learn LO use
and use most often. they are not always the mosl convenient. For example. if the
system bas a natuml ccnter of symmetry, as in the case of an a1om wilh irs m;issivc
nuckus al its cenLcr. it is much more convenient to use spherical coordinates. which
are constructed wilh exactly such systems in mind. There are many examples of
problems that become much easier by using the appropriate choice of coordinate
sys1em. UsuaJly the symmetry of the system of interest will suggest which of a
number of available coordina1e syslems 10 use. Jn this chapter. we"ll study plane
polarcoordinares, cylindrical coordinates. and spherical coordinates and learn how
to express vector quantities, such a-. the gradient. the divergence, arn.J t.he curl. in
these coordinate systems. We'll see how we can unify the results for these various
coordina1e systems by introducing Ihe ide.1 of a melric coefficient In Section
4. we'll learn how 10 conven differen1ial volume element." from one coordinate
system to another by means of Jacobian detenninants and. in SecIion .5. we'll
generalize most of our previous results in10 one se1 of equations by introducing
curvilinear coordinates. In the last section. we'll introduce two other coordinate
systems involving spheroidal coordinates 10 give a little experience in lesser-used
coordinate systems.

8.1 Plane Polar Coord inate

Instead or locating a point in a plane by 1he two coordinates (x. y). we can equally •y (x. y)
well locate it by spt:cifying its uistancc r from the origin, and the angle() wilh whi~h
the line from the origin to the point makes with the positive x axis (Figure 8.1 ).
The coordinates r and 0 are called pnlar coordinmes. We shall restricl r 10 r :::: 0
and allow 0 to take on any value, although often 0 will vary from O lo 2rr. You can
sec from Figure 8. I that 1he relatjon between rectangular coordinates and polar
X
coordinates is given by
Figure 8.1
x = r cosO and y = r sin 0 (I) The spcci llc.:11ion or the locatinn of a poinr
in a plam: by polo.r coordinutc~. (r. 0 ).
and
and (2)
349
350 Ch,1p1i-r 8 / Curvilinear Coordinates

Thus the point ( I. l) in rectangular coordinates become.-; the point ( J2. rr /4) in
polar coordinates. When calculating 8 from the nrctangent fonnula in Equation 2,
you must bear in mind in which quadrant the point lies. Using Equation 2 blindly for
= -1. y = - I) give~ 0 = rr / 4. bm real i7..e lhal tan 5rr / 4 = t.an(225°) =
the point (x
I, also.

Example 1:
Equations I give
y

and

and Equaiion 2 says that

Use each of 1hc.se relations to caJculate O for lhe point (x = -1. y = - J3)
(Figure 8.2).

s O LU TIO N: In this case, r = (x 2 + y 2) 2 = 2. Using a hand calculator,


1
/

you· II fi.nd that

(-1,-\'3)

Figure 8.2
= -Ji
The poin1x = -1. y
uLl -- . -!
sm (-./3)-
2
- -600
.

and

none of which i.-. correct! The point Iies in the third quadrant. and the correct
answer is 180° + cm,- 1(1/2) = 240°.

Many of the famous classical curves of mathematics are conveniently ex-


pressed in polar fom,. Figure 8.3 shows a polar grid of coordinates for ploning
functions expressed in polar coordinates. Because lines of cons1an1 r (circles ccn-
tcred at the origin) and 1.incs of constant fJ (straight lines emanating from the origin)
intersect al right angles. the polar coordina1e sys1cm is said
10 be onlrugcmal . A
circle of radius a ccmercd the origin is described by r = a. This result fol-
a1
lows immediately from the equation x 2 + y 2 = n 2. What about a circle centered
Figure 8.3 =
somewhere else, say al (x = n, y 0)? In thal case. we have (x - a) 2 + y2 a 2 =
A polar grid of coordina1cs for plo11ing as I.he equation in rectangular coordinates, or (r cos 0 - a )1 + r 1 sin 2 0 = {/ 2 • or
func1ions exprcs..-.cd in polur coordirtatc,~. r 2 = 4a 2 cos 2 0. You should verify by plotting that r 2 = 4a 2 cos2 0 is indeed the

1h m r
8. I Pl,1111• Pol,1r c-◄ )tnr linalcs 351

equation of n c:ircle of mdius a centered al (a. O) and 1hat r 2 = 4a 2 sin 2 0 is a circle


)'
of radim, a ccntcrcd at (0. a).

\"
Example 2:
Plot the equation,~ = cos 20 for O _: (J < 2rr. The rcsul1ing curve is called
1he Lemniscare of Benwu/li.
Figure 8.4
SOLUTION: Let's firs:1 make a small table of values of r (remember lhat
The lcmnisc.::itc of Bc:moulli. r 2 =c ::; 2/J.
we restrict r tor ~ 0):

fJ
r +1.00 +0.93 +0.71 0 y

This gives 1hr upper half of the righl lobe in Figure 8.4. In the open interval
(4.5". 135°). ,. ~ . -:: 0. and so there are no real values of r in this range. From
135° 10 225°, we obtain the entire left lobe in Figure 8.4. In 1hc open interval
(225° , 315°). , 2 • 0 once again. and linally we ob1a.in the lower halt' of 1he X
right lobe in Figure 8.4 for 31 5° ~ 0 < 360°.

If you look al a handbook such as the CRC S1andarcl Ma1hematicul Tables.


you'll :-ee a number of beautiful cur\'cs with names such as cardioid (Figure 8.5) Figure 8.5
or the folium of Descarte~ ex.pressed in polar form. Beller yc1. the School or
A cardioid. r =11(cos 0 + I).
Mathematics at the University of SL Andrews has an outstanding lutorial website
(www-history.mcs.st-and.ac.uk/history/) with a "Famous Curves Index.'' which
gives internctive acces_., to almost 100 "famous curves". wiLb names such as
Freeth's nephroid (Figure 8.6). Fermat's spiral. and the conchoid of de Sluze.
V
ll's easy to determine the slope or a curve that is expressed in polar coordinates.
r:::::: r(0). In this case x = r(0) cos 0 and y:::::: r(0) sin 0. and using

dy = dy dx
d0 dx d0

we see Lhal I.he slope is given by


X
d_,.
dy d0 r'( 0) sin 0 +r cos 0
- = -= (3)
dx dx r '(0) co 0 - r sin 0
d0

Example 3:
De1em1ine the slope of the cardioid described by r(()) =I t co~ /I (Figure 8.5) Figure 8.6
at any poin1. Frec1h ·s ncphroid. Tht: polar equal ion is
r = +
u( I 2 :.in 0/2 ). (In this plot. r is
SOLUTION: Using Equation~. i.!llnwcd 10 Lake on negative v;1lm:s.)
352 h, pi r 8 / Curvilin ar Coordinate

d_v ( - sin 0) sin fJ + cos 0 + cos1 8


y - =
dx (- in O) cos O - si n - sin 0 co~ 0
cosB + co· 20
.sin O + sin 20

For example, Lhe slope al rr /4 is -0.4 I4 (sec Figure 8.5).

Figure 8.7 shows a curve described by r = e/J0 (-oo < 8 < oo). called a
logarithmic spiral. Trus curve has the unique propeny 1hat Lhc angle between 1he
figure 8.7 extended radial line OP and the tangent to lhc curve at Pisa constanl. This angle is
A logarithmic :.pirnl, ,. = c,. ' . denoted by a in Figure 8.7 . Let's prove this property of a logarithmic spiral . fir~l.
we need an equation for 1he angle a. Problem 5 helps you prove 1hat

)' I dr
cot a= - - (4)
r dFJ

If we substi1u1e , = eflP inio Equation 4. we find 1ha1 cot a = f3 = constant.


Furthermore, because ( 1/ r)(dr /d0) = constant. we see tha1 the logarithmic spiral
is the 011/y curve with lhis property.
A logarithmic ~piral has ,mother intcres1ing property. The tangent line at
0 = 2rr 11 (n = integer) is shown in Figure 8.8. r1 tums out that 1he lcng1h of rhis
X
line bc1wecn where it cuts the x and y axes (be:) is equal to the length of the spiral
from the origin (where 0 ~ -oo) to 0 = lrr 11. To prove this result we need a
fom,ula for the arc length of a curve rhat is described by polar coordinates. Th.is
is not difficult. Simply write d;) ~ = dx 2 + dy 2 and use Equations 2 to obtain

ds 2 = d:/!- + dy 2 = (cos 8 dr - r sin O d0/· + (~in 0 dr + r cos 8 d0) 2


figure 8.8
The 11.111gcnt lint: of a logarithmic ~pira1 a1 = dr 1 + r 2d0 2
8 = 2.Jrn (n = integer).
or

ds = [ ( ~~ r
.,
+r
2
] 1/.2
d0 (5)

Therefore. the arc length of a logari1hmic spiral from the origin (where 0 --1- -oo)
10 0 =2rr n is given by

s = (2.7n (fi2e:.JN + e2/W) l/2d0


. '\..

Problem 8 has you show thar this is equal to the lengtJ1 of the line be in Figure 8.8.

n
8.1 Plane r>olJr Coordinates 353

Example 4:
Detennine 1he length of the perime1er of 1he cardioid described by
r = I+ co. 0.

SOLUTION: We use Equation 5:

(2;f
s = Jo (sin {)
2
+ I + 2 cos 0

.t O 1.'ljl
=4
l
0
cos -dB= 8
2 0
cos II d11 =8

Now that we know how to calculate ~lopes and arc lengths of curves that
are de. cribed by polar coordinates. let's calculate the area of the cardioid given
by r(B ) = I+ cos 0. In rectangular coordinates. the diffcrcn1ial area clement is
dxdy. but it is 1101 drd0 in polar coordinates. We' II dc1cnnine just what ii is both
geometrically and analy1ically. Figure 8.9 shows an area element gcnera1tXI by
varying r by 6,.r and 0 by 6,.0. If 6,.r and 6,.0 are small (which we anticipate), lhcn X
the area of the area element is ( ~r)(r ~0). or rdrd0 in c..lifferential fonn. Thus, we
see that d A = rdrd0. The area of a figure described by r = r(O) is given by Figure 8.9
A differential area clement in polar
coordin:11e generated by varying r by t>.r
I /
f
{r(IJ)
A= dB Jo dr r =2 2
r (8) d0
~nd 8 by l!:.0.

For the cardioid r(0) = I+ cos 0 (Figure 8..5).

A = -2I 1( +o
2
:r
I .,
cos f:J)-dfJ = -I
2
( 2rr + -211 ) = -3JT
2 2

Example 5:
Determine the area of one lobe of the lemniscate of Bernoulli shown in
Figure 8.4.

SOLUTION: The equation of the curve in polar coordin:ues is/!.= cos 20 .


The area of one lobe is given by

A = -I 1,T/4 cos 2 W d0 =
lrr/4 cos 2 2B df:J = ~rr
2 -,7/J .0 8
354 Ch.1pter H : Curvifinr.a r Coorchn, 111-·,

Before we finish this ,cction, we"ll show hov,: to evaluate the integral / =
J0 e ox? dx using polar coordi ni:ltes. W rile ( 2 as

Now convert 10 polar coordinates, realizing 1ha1 the area of integration is tJ1c first
quadr.:int. or O .:5 r < • and O .5 FI :S rr /2. Thus.

100 ' .,.,.1•,0


1
.,. ;'/.(2 -ar- n e - (W J.,..

/-= df) £' rrlr=- - -du = -
o o 2u 2 4a

anti so/= (rr/4a) 1l 2_This is a slandard Irick forcvalua1ing this inlcgral.

8.1 Problems

I. Plor each of the following curve~ in polar coordinates. In each case. 0 _ (} < 2rr. Rcc:ill that we ha\'e clccrcd
10 allow r 10 lake on only non -ncga1ivc values (r:::. 0).

(a) r=2sinH (b) r = 2 cos O + s in 0


1T 21T
(c) r = ~in (-I cos 2 I) (d) r= - 0 -
sin U + c:os 0 6 3

(e) r = I + 2 cos (J (f) r = v cos 20


2. Some aulhors (particularly of calculus tex1s) ,illow r to take on neg.alive values. Plot the wn.•cs in Problem I.
allowing r 10 lake on negative \'alues .
3. Find the ~lope of r = =
sin 20 al f:J ,r j4 an<l 5;r /4 . (This represents a lwo-lcavcd rose if r:::. 0. and a four-leaved
rn~c if r is allowed to assume lll'ga1ive values.)

.
4. Find the slope of v~
2 r cos ( I! - -:r ) = I. - -:rr6 2,-r at any value of I) in 1he interval ( - -;r . -2rr ) .
< fJ < ·---
4 3 6 J
5. Derive Equulion 4. Hiw: First show that the ~mgle a in Figure 8.7 is equal lo y - H. where slope= rnn y and
1hcn u..e rhc relation Jy / dx = Ian y along with Equal ion J.
6. Dctem1ine 1he angle o between the cxrcndt't:.I rJdial line OP and 1he tangent to the curve al P for

(a) ,- = c 012 al nny t) (b) r = I + sin O at(} = ,r /2


(cJ r = sin 38 at 0 = 1T / 12
7. Calculate the arc length of the pc-rirne1cr of the lima(;'on of Pascal described by r = a + b cos O with a > h.
(Refer to Section J.5.)
8. Show that the length of the line be in Figure 8.8 is e.qual to ( I + {3 2) 11'2e 'rrfl 11 / /j.

9. Determine the area bounded by the LeminS<.:ate of Bernoulli (E.--:amplc 2).


IO. Oe1cnnine the area of I.he lima~on of PD.Seal described by r = a + b cos 0 ,vith a > b.
8.2 V. ctors in Plane l'nbr Coorrlin;iles 355
IJ. Calcularc the ~rca enclosed by the 1wo-lca\•ed rose r = 2 cos 2A (r ~ 0).
12. Detennine the arc-a between 1he curve given by r~ = 4 .sin 2 0 y
and the circle given by r = I. (See Figure 8.10.J

Figure 8.10
The Jrca betw~n rhc i:un·c gi\"cn hy
r~ = .t ~in "' /I ant.I the circle given by r = I.

13. Oercrmine the volume of rhe par..1buloic.J given by ;: = 9 - x~ - y~ above the xy-planc.:.
-, ' " I.,
14. A circular di.sk of radius II has a density 1ha1 varies ;1s p = e- - 1 , ·;- i · l ·•. Calculale the mass of tht= Jisk .
15. Use polar coordin-atcs to calculate rhc volume of a hemisphere of radius 11.

16. Show th:ll the f11rmul:1 A = f f (.r d_,. - y dx) lx.-cnmcs A =~f r~d(I in polar cnordinares.

8.2 Vectors in Plane Polar Coordinates

In Chapter 5. Wt! expressed the location of a particle in two dimensions by the


vector r(I) = x(J) i + y(I) j. The velocity and 1hc accclcra1ion of 1hc particle arc
given by

\' (f) = -dr


dt
= dx . Jv .
-tit I + .dr. .:. . J ( I)

i.lnd

(2) ' )'

Lei's look al the corresponding expressions in plane polar coordin::ites. and write

(3)

where e,. is a unit vector in the direction of r . Before we differentiate 1his expre~sion
X
with respect to , . let "s reali7.c 1hat unlike i and j in Equation I. the direclion of
er change. as r(r) varies with time (Figure 8. 11 ). Thus, the time derivative or Figure 8.11
Equa1ion 3 is Tht.' unit w cror c, =
L',(1 l in polar
coordinates ul rwo uiffert:nl time:-.

-dr = dr
- der
er + r ( l ) - (4)
dt dt dt
356 Chapler 8 / Curvilin ar Coordinate•

Al1hough irs length is always unity, Figure 8.11 shows that er changes with time
because iLs direction changes wi1h time if 8 changes with time. Thus, we sec that
e, = er(0). The time derivative of e, is equal 10

de,, dfJ de,


dt
= dt d0
(5)

We'll encounlcr a number of examples where the unit vec1ors in coordinate


systems olhcr than rectangular coordinate systems depend upon lhe coordinates.
so we're going to have to learn how to evaluate derivatives like de,/dfJ. Perhaps the
simplest way is to express the unit vector e, in terms of the Cartesian unit vectors
i andj:

e, = cos 0 i + sin 0 j
sinOj
Then
-sin Bi cos0i
.
.\" -de,
d0
= - .
sm 0 1 + co:-; u J
.c1 •

Flgu,re. 8. 112 This unit vecror is perpendicular toe, and is shown in Figure 8.12. If we define
The polrui coordin:11c u11i1 \-·ec1ors
e,=cosO I si11V'Jand
e0 to be a unit vector perpendicular toe, and in the direction of increasing 0. then
e) = - in 0 I co-. {l j c.xprcssed we can identify de,/d0 wilh e0 and write
in 1crms of the cartcsirm coordinalc unit
vecrors I and j.
-de,
tJfj
=- . .
sin 0. 1 + cos u J :.= e0
/'I •
(6)

Equation 4 tells us tha1

dr d0
v(t) =- e, +r- e0 (7)
dr dr
The Lwo unit vectors e, and efl con.s1i1u1c an orthogonal pair of unil vectors
(e, • e8 = 0) and any vector in the plane can be written as

(8)

Tak.ing the dot product of u with e, and e 0 shows that ur = u · e, and 11 0 = u · e0 •


so Equa1ion 8 can be ,vrillen a_.;

(9)

The two vectors e, and e 0 arc said 10 fomi a basis and u, = u · e, = 11,(r. 0) and
11 8 = u - en = u ,1 (r. 0) are cat led the comp(mellls of u in this basis. The vectors i and

=
j also form a basis in 1wo-dimensions since u can be expressed as u 11_,· i + u1• j. It
so happens thal any pair of non-colincar vectors can form a basis in two dimensions,
bu1 each coordinate ~ys1cm has a .. natural" basii- which is most convcnien1.
The accelcra11on of a particle is given by tht: time dcrivu1ive of Equa1ion 7:
8.2 V 1ors in Plane Polar Coordinatt.'.~ 357

We need to evaluate de 0 /dt. or dc 0 /d0 because dc 0 /d1 = (de 0 /d0)(d0/dl).


Problem I has you show 1ha1

den
-=-e ( I I)
dfJ r

Using Equations 6 and 11. Equation 10 becomes

which may be familiar lo you if you·ve had a course in cla.i;sical mechanics.

Example 1:
A force that can be expressed as F = f (r) er i:- called a cr,ura/ force . Show
that the angular momentum J (mr~df. / dt in this ca.-.e) is conserved .

SOLUTION: Newton's equation:<-. ma= F. re:id

111
d~r
-, -
[ d,~
r (d0)
-
dt
2
] = f(r)

and

But this equat ion c:an he written a.-.

I
- d ( , d(J) =0
- mr--
r,lr d1

which gives J = mr 1dt1/ dr = constant.

=
So far we have shown that der / dA e,. illld I.hut t!e,,Jdll = -er. Figure 8.13
shows pictorially thai er and e" arc indcpcndcnl or r because \ arying r simply
moves both unit vectors along r v. ithout changing their directions. Therefore. we
have th111 de,/dr === 0 and de 0 /dr = 0. Thcs.e results are summari1..cd in Table 8.1
at the end of this sec1ion .
We can u~e plane polar coordinates to introduce I.he idea of a .\t:al,~fac10r. or a
111erric ni~Oicit'111. lf we sta.n. with r = x i + y j = r cos 0 i + r sin ii j. Lhcn you can
see that iJr/or lies nlong er (actually, it"s even equal toe,.. but ir's not nccl'~ sary .\"

for what follows) and ar/o0 lies along elJ. We define the proponionality factors
Figure 8.13
hr and li 1i. called scale factors or metric coefficients, by A pic111r1;1l argumcni 1hu1 c, und e11 :ire
indepcnucnl ol r .
Jr
and h0 e (13)
358 C h,1plt•r 8 1 C11rvilinv,1r Corndi11.11t •,

=
so that 1,r = IAr/arl and hfl lar/30(. We can determine h,. by diffcrcn1ia1ing
r = x i + y j with respect tor and then taking it~ absolute value.

( 14)

Similarly. di.ffcrcn1iating r = x i + y j wit.h respec110 fJ gives

( I 5)

Nole Lhat Equa1ion 13 allows us to write

( 16)

and

( 17)

which in the ca~c of plane polar coordinates gives ds 2 -== dr 2 + /2.dr/1·.


We're now ready to derive expressions for Lhe gradient. divergence , and the
L.aplacian operator in polar coordinates (remember 1ha1 the curl is essentially a
Lhrcc-dimcnsional quan1ily). Start wilh

which defines Lhc components of grJd J in polar coordina1es. Substitute this


expression for 'v f into

df = 'vf · Jr ( 19)

with dr given by (sec Equation 16)

dr = h,.dr e,. + h0 df:J e, 1

= e,dr + rd8 ef, (20)

to obtain
df-== [ ('v f),.e,. + ('v f) 0 e0 1 ·le,. dr + r d0 e11 ]

= ('v f),.e,. . dr e,. + ('v no eo . rdfl Cl)


=('v f),.dr+('v f)ord0 (2 l)

If we compare Equa1ion 21 wir.h

= -of Jl
df
ar dr + ....,:_
;)(i
du

we see that
8.2 Vectors in Plane Polar Coordi11,11t·~ 359

(V flr= -
aJ and
I of
(V J)o= - -
or r afJ
or that
of I af
V f = - c, + - - e0 (22)
ar r a0
or
i) eo a
'v=e,.-+-- (23)
ar r a&
in operator notation.
Notice also that we c.an express Equation 22 in tenns of J,r and h 0 by writing

(24)

We'll see in Section 5 that this is a general resul1. Equation 22 can also be derived
in 01hcr ways (Problem 12).

Example 2:
Evaluate V / where f(r. (n = r 2 sin 0.
so LU TIO N: We simply USC Equation 22:

'iJ f = e, rJf + Co of
ar r a0
= 2r sin 0 t'r + r cos 0 eo

We t:an find an expression for div u in polar coordinates by writing

and using Table 8.1 for the derivatives of e, and e0 . The final result is

Example 3:
Evaluate div u if u = cos 0 l'r - sin fl en.

SOLUTION: Use Equation 25:

co · O - co. n
V-u = - - - - - = 0
r
360 C apl r 8 / Cun ii i1,c,1r Cc" 11·rli 1 ,1 1v ~

Finally, let's find Ihe expression for the Laplacian operator in polar coorcli-
na1es. We'll do this by using 'v?. f = 'v · 'v J and the above rcsulrs.

'v-1 r = 'v . 'v f === (


e -a + -e, -a ) . ( e -aJ + -en -af )
· r <lr r a0 ' ar r a0

e, (
+ -r - -
i) ~ f
- e + - - +- - 1
iJf iJer ) 2
I iJ f
o0 iJr r i:Jr of} r':. cJ0

or

2 J2 f I iJf I iJ- f
.. -
or· + -r -+,-;::-:;-
'v f='v•'vf=- (26)
or ,- - i) -
or. in opern1or form.

(27)

Example 4:
The equaIion ·,/! f = () is c.:illed I..J1p/11n· ·.\ ,·quarirm. Show that f (r. 0) =
r 11 sin n0. where II is any positive or ncg . 11 ive i ntcger (r =fa Oif n < 0). :wislies
uplace·s cqua1ion.

SOLUTION: We USC Equation 27 to write

=0

Table 8.1 summarizes 1he results of this section.


We've spenr some amoun1 of lime on plane polar coordinates because it's.
pedagogically uscful lo illustrate some general mc1hods in 1wo dimensions first
You· II see in lhc :-cct ions 1ha1 follow that it wi II be relatively easy lo derive
expression::. involving lhc 'v opcralcir in any coordinate system using Ihc mcIhods
that we have developed here.
8. 2 Vt '<. tnrs in Plane Pot ar Coard i na1es 361

Table 8.1
Some u,dul fon11ul.1s in plane polar coordinates.

rJJ I 8J
h, =I hu =r grnd J =VJ=
or + -r -aoe 0
-e,

ae, =0 ae,
-= e
.
d1v u = V · u
I o(r11,)
=- -- + - I 01,li
-
ar ao r ar r M

aeo = 0 rJe,J
-a- =-c , '\1
2 a2 / 1 a/ 1 a2 J
J = - + - - +2- -
ar or1 r or r af)'l

8.2 Problems
I. Show thar de 0 Jd0 = -e,.
2. Determine du/dt if u = sin 0 e, - r 2ll en and r =, 2 and 0 = 21.
3. Show that I,,= I and ho= r.
4. Newton's equations in rectangular coordinates for a particle moving in a plane under the influence of a
. . d 2x kx d2v kv
coulomb1c potcn11aJ are m -., = - . ., ,, and m--:;- =- '2 ·: ,, . Express these equa1ions in
t/1- (x~ + y )3 dr- (x + ·-) 311-
polar coordinates.
S. Derive the equa1ions in Problem 4 s1arting with a coulombic potential.
6. The trajectory of a panic le in the xy-plane i_ described by x = r. y =- , 2. Derive an equation for its velocity in
terms of e, and e 0 .
7. Evaluate grad J if j(r. 0) = r 2 - a 2 cos fJ where a is a constan1.
8. Evaluate div u if u = r cos 0 e, - r sin 0 e0 .
9. Show 1ha1 j(r) = ln r (r -:j:. 0) sa1isfics Laplace·s equation.
10. Show that V - u given by Equation 25 sarisfies rhe equalion

'\1 · u = - 1- [.!!__<ht111,) + !__(h,11 11 )].


h,h 0 ar 30
11. Show that V 2 f given by Equation 26 satisfies the equal.ion

v2 1 _ 1
lr,ltR "r +a0a (",iJf)]·
[aor ("·of) hr, a0
rJr
12. We can derive Equation "22 in a more pedes1rian way than we did in using Equal.ions 18 and 19. First write
nv 3/ . d tJ1en show Lh at
J- = -af I + -=--Jan
,h d)"
JJ
-=--
aJ fi r aJ cJfJ Jf sin O aJ
- - =cos - - - - - an d
cl.\' ar ax ao ax r r ao
afar 1Jf 08
-;y = - -+- - . ;)J
=s1n /7 -
cos0
+ ---.
aJ
ay rJr a,y a Dy rlr r oR
Figure 8.12 show~ that e, = co. i + sin, 0 j and e 11 = cos(': + 0) I + sint + 0) j = - sin O i + cos 0 j (note 7
that e, • e0 = 0), Solve 1hesc two equations for i nnd j and substitute the result and the above equations into

V f = aJ j + Jf j 10 get Equation 22.


rJ.r o_\'
362 C h,1ptc·r 8 ' C 111-..ili1w.11 Corirclinates

13. This problem a,:;ks you to convert v 1 .f from rectangular coordinates 10 polar coordinates \virh r fixed by
repeated applicalion of the chain rule of par1ial differcnIia1ion. St.arr with

DJ of or rJJ i":JIJ
-=--
ox 3r ax +-
ao -
8x
( IJ

and

aJ aJ or of a0
-a_, =- - +-
oray -
u0&y
(2)

and show that

aJ =_~in 0 iJf of cos0 af


= -- - (r fixed)
clx r t.ifJ a_\' r afl
Now apply equations I and 2 again 10 show that

iJ 2 f 2 sin 0 cos 0 rJf


- 2= - - 2- - - + -2 - -2
sin 2 0 a1 f (rfixed)
ax r ao , a0
Similarly. show that

and so

;i2 I i a2_r
- =- - {r tixed)
ih '.! r2 lf/2

14. Genernlizc Problem 13 to the ca.,e in which r i" not fixed.

8.3 Cylindric I Coordinates


p
A direcl extension of polar coordinates to three dimensions is 10 use r and 0 to
locate I.he projection of a point onto Ihe .x y•planc and.: 10 locate the venical disIance
z )' of the poinI from 1he xy-plane (Figure 8.14). The three coordinates. r. 0. and z. are
called rylindricol coordinale.1·. A surface given by r = constant is a right circular
cylinder; a surface given hy 0 = constant is a half•plnnc with its edge being 1he
.~. axis; and a .surface given by ., = consLanl i.!, a plane perpendicular 10 the ::: axis

I
IX

Figure 8.14
A point specified by the cyltndrical
{hgure 8. 15). Each of these three surfaces is perpendicular to the other two. and
so I.hey are mutually onhogonnl.

r =
The curve formed by the intersection of a 0 = cons1un1 surface and a ~ =
cons1..1n1 surface is called an r rnn-1'. The curve formed by 1.hc intcrsccIion of a
constant surface and a :: =
con:-tant surface is called a A cun•e: imd 1hc curve
formed by the intersection of a r = constant surface and the 8 = conslant surface
coo rt.Ii 11a11:~ r. (I, und : . is called a ::: cun•e (Figure 8. 16 ).
364 Chapter 8 / Curvilinear Coordinat "

to Figure 8. I 8, which shows such a volume element. You can see from this figure
z:
that

dV = rdr dtJ d: (4)


dz.
We'll discuss a general analytical method to find the appropriate volume elements
y
in Section 5. but we· 11 jusl use geometric arguments for now.

Example 3:
Evaluate the integral:
X

Figure 8.18
A picrori:il argument Lhat Lhc volume::
I= fff xy: dxdyd:
element in cylindrical coordinates is given
=
by d V rdrd0d:_. over the region x ~ 0. y ~ 0. 0:::: • < b. and x 2 + _v2 .::: o 2.

SOL Ur ION: The conditions x 2::. 0, y 2:. 0 imply that O :5 0 < rr /2. so

I = f{fcr 2 cos fJ sin 0):r drd0d:


12
=
1"tlz..: lJT
() 0
t/0 cos 0 sin 0 1"
()
dr r 3

.z:'
.co. 0, h) 2 2 4 16

- (!!) (a. 0, 0)
No1ice that the triple integral in Example 3 factored into a product of three single
inlcgrnls because the limits are consL1.nts rather Lhan involving the olher coordi-
nates . This will often happen if you empll)Y the ·•right'" or the "natural"' coordinate
system for the problem. Problem 4 asks you 10 evaluate the above integrnl using
rectangular coordinates and you'll :.ee how $Orne of the limits involve the other
,x coordinates.
We' II evaluate another integral using cylindrical coordinates. Figure 8.19
Figure 8.19
shows a right circular cone with base x'.! + y'.! ::: a:. and apex at (0. 0. /,). Let's
A right circular cone with base
x 2 + _r::::: "~ and apex at (0, 0. J,). find lhe volume of this cone . The upper limit of the : in1cgra1ion depends upon
the value of r = (x 1 + y 2) l/2. The surface of the cone is given by the equation
h
: =- h - - r. and so
a

V =
l(
l" b
d0
0
dr r
1/, -ird: = -rra
0
]
3
2
h

The limits on :. involve r here, but Lhe integral is m11ch easier to evaluate in
cylindrical coordinates than in rew.mgular coordinates (Problem 5).
8,J Cylin{Jrical Coordinate~ 365
Figure 8.15 shows the surfa1.:es r = conslant, 8 = constant, and:= constant
for cylindrical coordinates. The surfaces are mutually ortho~onal at any poin1 of
intersection. and cylindrical coordinates are another example of an onhogonal
coordinate system. We can prove analytically that a cylindrical coordina1c system
is orthogonal. and in the process inlroduce the unit vector.; in Lhi~ system. The
position vector of any point is

r = x I+ r j +: k = r cos 0 i +,. sin 0 j + :-. k (5)

\'
The vector i3r/or is tangent 10 the r curve in Figure 8. 16. Similarly. orJa0 is
tangent to the 0 curve and ar;a: is tangent lo the:: L:urve. Thus. ar/Jr. ar/30.
and or /o: form a set of mutually orthogonal vectors. They aren't necessarily unit
vectors. but if we divide each one by i1s length. then we have

or/iJr ar;a0 iJr/a:


e. = - - - (6)
e - ---
r - !or/orl ' c,., = lclr ;a01. ~ 1ar;a:1
x,
Substituting Equation 5 into Equations 6 shows that er. eu. and c: arc indeed mu- Figure 8.20
tually orthogonal unit vcclors (Figure 8.20) (Problem 6). lt"s easy lo sec from 'lne un;1 \"tX"tor.,. e,, c 1J , ,rnd e. Df ;:i

Equation 5 1hat J,r = I. h,, = r. and h: = lor /J·l = I. The metric cocfficicn1s for a cylindrical L'oordtnatc sys1cm.~No1~ 1.har
they fonn a right-handed coordina1e
cylindricaJ coordinate system arc given in Table 8.2 at the end of the section. No1e
~y:-rcm .
that ds- (Equc.1t1on 2) can be wn11en as ds~ = h;dr- lr,;d0· + J, ;d:- (Problem 7).
l • · l l 'l 'l '1 -, l

The difference be1wecn the operator 'v in plane polar coordinates and cylin-
drical coordinates is just the k a/'J:. tcm1, and so we can write

(7)

in cylindrical coordinates. (In this case. c~ = k.)

Example 4:
Evaluate grad .f if f(r. 0. :-) =:: cm, 8/r.

SOLUTION:

:: c.:us i> :: sin f,J cos FI


grad f = 'vf = - --..,-
r-
e, - --.,- e11
r-
--
r
c.

We can ohtain expression., for div u and 'v~ f in cylindrical coordinates by


the same method that we used in the previous section . The only new factor is that
e= is a cons1.:1nt vector. so that 'ded or = 3e)af) = aejJ-:. = 0. Furthennore, er
and e11 do not vary with :, w acr/o: = iJe 0 / tJ: = 0 . You can see this pictorially
from Figure 8.20 or analytically (Problem 8). The results for div u and 'il 2 f are
(Problems 9 and 10)
366 Lh,1p!t•r 8 ,' C urvili:w,ir Coordinates

div u = 'il. u = ~ o(ntr) + ~ au 0 + 811::_ (8)


r or r a0 iJ:.

and

(9)

We didn't consider curl u = 'il x u in the previous sect ion because the curl is
essentially a three-dimensional operation. In this case.

(10)

Equa1ion I O is an intermediate result. To arrive at Equation I 0, we have used


the known expressions for the derivatives of the unit vectors with respect 10
the coordinates and er x er = e,, x e0 = e., x e.- = 0 (Problem I I). Referring 10
Figure 8. 20. we see 1ha1 (Probll·m 12)

( 11)

Using Equations I 1 in Equal ion IO gives ( Problem 13)

Example 5:
Evaluate div u and curl u if u(r. 0. z.) = :1 er+ r 2 sin 0 e +, re_..

SOLUTION: Equation 8 gives

~2
div u = 'il · u = - + ,. cos () + r
r
and Equation 12 gives

curl u = 'il x u =;_en+ Jr sin 0 e~

Problcmi; 20 ..ind 21 ask you to use :.i CAS 10 evaluate Lhc gradicn1. divergence. and
curl in cylindrical coordinates.
8.3 Cylindrical Coordi11.ih•, 367

Lct"s finish this section with a warning regarding the curl in cylindrical coor-
dinates. Although

j k
c) ,J a
curl u = ( 13)
Dx iJy iJ=:,
y II -
"·' I(

in rec1angular coordinat~s.. there is no obvious corresponding detenninantal ex-


pression in curvilinearcoordina1es. The correct formula for cylindrical coordinates
is

er re 0 e.
I
curl u = -
a i'J cJ ( 14)
r i:lr i:10 a:
II r r11a "-
which can be obtained by working &1ua1ion 12 backward" (Problem 17).
'The key fonnulas of this section arc summarized in Table 8.1.

Table 8.2
Some uscful fonnula." in cylindric::il coordinates.

J,_ =I
iJe, =0
a,-
ae0 =0 :J,e,, iJe J =0
-=-t'-
or i-W ' ;) :

Fle:: =O iie: =0 i)c: =0


a,- rW o:
grad f = 'v f ar 1 "f
= -=- e, + - - c0 + -af e.
iJr r [JiJ i)- ~

. II au,,
= "v = -rI -
o(rn,.) OIi~

or- + -r -ao + -ii;:


I

d I\' U ·U

e, re(! e.
i) a a
= ,.
or -V0 -a-
II r ru 11 II -
368 ( h.1pte1 /1 .' Curvilinear Coo.rdi11.1rt•~

8.3 Problems

I. Describe the graph that each of the following equarions represenlS in cylindrical coordinalcs:

(a) r =a (b)

(c) r = cons1an1 (d)

2. Use Equations I to derive Equation 2.

3. Determine the arc length of a space curve given by r = at. 0 = b1, : = er, from t = 0 to r = I.
4. Evalua1e the inrcgn1I in E:~amplc 3 using rec1angul,1r coordinates.

5. Use rectangular coordinates to calculate the volume of :.i right circular cone with ba:.-e .r 2 + y 2 _::: a 2 and
height /1. (U!.C a CAS to evaluate the integral.)

6. Substitute Equation 5 into Equation 6 10 show that er, Cu, and e_. constilute a set of mutua.lly orthogonal unir
vectors.

7. Show that dr1 = lrdr


r
+ h-.,UJ-
..,
(,
..,
+ li:d;.-,
~
1 ., 1 .,
where 11~"'
r
= (ax)
-or
2
(ar) + (J:)~
+- ...,:_.
a,-
2

ur = I.
--

h~=(ih)2 +(a-'"):.,_+ (0:)1 =r2. andli:= (ih)'.! + (a_,.)2 + ((l:_.)2 = l.


~ a0 a0 a0 · a: a: .J:

..
8. U~c: 1he de fi n111on e. = -I -iJr to show that e. .•~ a con-.1an1 vc~tnr.
h.- ii::. •

9. Show 1hat div u is given by Equation 8.


IO. Show thal 'v 2J is gi\ien by Equa1ion 9.

11. Derive the in1ennediate result given by Equation 10.

13. Finish the derivation of Equation 12.

14. Show 1ha1 / (r, ;:) ,_ (r 2 + .: 2) - I/ M(r 2 + .: 2 ~ 0) is a solution 10 Laplace\; c.qua1io11.

15. E:<prcss the velocity of the space curve I r(r). 0(r). :(r)I in tenns of e,. e8 , and e~ .

16. Here·s a fairlv easy way to ex.press grad J in cylindrical conrdina1es. Write elf= of dr + aJ d0 + of d-:..
J or ao a:
Now u~ 1he facr 1hat df = (V f) • dr 10 determine 1he components of V f.

17. Shnw that Equations 12 nnd 14 arc cquivalcnr.

18. E.xprc~, the vector u = 2: i - xj +y k in cylindrical coordinaLes.

19. Determine V · u and V x u if u = er + cos O e~.


20. u~ a CAS 10 evaluate fi) f if f (r. 0 . .,. · = (r 2 . : 1 kr: CO\~ Ii.

21. Use n CAS to evaluate div u and curl u if u(r. 0 . :) =,.~cos 8 er - r: 1 sin~ f) eH + e~ e1;.
B.4 S1 h >ri ·al Coordinare 371

V =
1 1" . 1'1..1
u
11
r 2dr
O
sin 0dfJ
O
d</J = -4rra·-J
3
:

Notice !hat 1he triple integral for V fac1ors into three sep:uate single integrals.

Example 2:
Evalnatc

J = iff :
\'
2
dxdyd:
r sin(} dH
\"

over the spherical region of radius a cemered at the origin.

SOLUTION:
I
I
["
I = lo r
4 1·"1 cos 2 0 sin fl dfJ lo[1.7 d,p
dr
O
/
I
5 5 ,l X
= -o . -2 . 2;r = -
4no
-
5 3 15 Figure 8.26
A piclorial argumen1 1ha1 the volume
The evaluation of this integral would have been quite a bit messier in
clement in spherical coorclina1cs is given
rectangular coordinates ( Problem 4 ).
by d\l =
r 2 sin O drJOd,Ji.

Example 3:
Determine the volume of the part of a unit sphere cen1ere<l al the origin that
lies within lhe right circular cone with its apex at the origin and making an
angle /j with the positive: axis (Figure K27).

SOLUTION: )'

2.-, 2,r
inOdO [
d</J=-(1-cosfJ)
. () J

Note that V - 0 if /j = 0, and V = 4;r /3 if /J = rr.


Figure 8.27
Figure 8.26 can also be used 10 convince yourself that an clement of surface The volume to be d etc rm irtcd in
area on the surface of a sphere of radius a centered at the origin is Example 3.

dS = a 2 sin 0 d0dtj) (5)

Equation 5 tells us 1hat 1bc area of the polar cap wi1h 0 ::S /3 is
[/J {'1.;r
S = a2 j sin 0 d 0 j d ,J> = 2rr a?. ( I - cos /3)
O O

Ir /3 ;::, rr. we ohtai n the surface area of a sphere ( 4rr a 2).


373

as in Equarion 2,
We can derive a fonnula for the gradient in spherical coordinates by letting
'iJ f = (V f)rer + (V noeo + (V f),jeq> and comparing 1hc following !WO expres-
sions for df:

df
aJ
= -dr+ aJ aJ
-d0+-d<J>
ar a0 o</)

and

df = 'iJ f · dr = ('v f)r er · drer + ('iJ f),,~,, · rd0 C1i + ('v f)q,etf> · r sin 0d,P e,tJ
= (V f),dr + (V f) 8 rd0 + ('v 1)1,r sin 0d</)
where we have used Equation 11 for dr. Comparing the two expressions for df.
we ~ee that
I of I rlf
('vf)o=--: (\7f) ~~ -- -. - -
. r a0 ,. sin 0 (}'+'
'.lA,

Thus.
cJJ I ;Jf I Jf
'v f =- e + - ._'.__ e 11 + - - - ei/J (12)
or ' r ;w r sin 0 iJ<I>

or

( 13)

in operator form.

Example 4:
Evaluate grad f if f (r, 8. ,p) = ,.J sin tp.
SOLUTION: Using Eqi1at ion 12. we obtain

grad f = V f = 2r sm. t/J e, + -r sin. -0</J e~


C 0!-1

To derive expressions for the divergence and curl, we have to see how 1hc uni1
vectors. c,. e0 . and e<' vary with r. 0, and rf>. We'll do this by first expressing e,.
e 0 , and e4> in tcnns of i. j. and k. According to Equations I.

e, = sin 0 cos rJ> I + sin 0 sin q, j + cos 0 k (14)

al
374 C h,11)1t•r 8 / Cun 1l1nc'.1r Coordinale~

Figure 8.28 shows that e 0 = iJcr/o0. in which case we have


er-i = cos 0 cos tf> i + cos 0 sin q, j - sin 8 k ( 15)

Notice 1hal e can also be obt.ained by adding rr /2 10 0 (Problem I I). Figure 8.25
shows that et/> = er x e0 • and so we lind 1hat
e4> =- sin tf> i + cos <J, j ( I 6)

None of these unit vectors depends upon r. so we have immediately thal

oe, oeu rJe4i


-=-=-=0 ( 17)
ar i-Jr ar
varying r simply moves I.he entire set
You can also .see this result from Figure 8.28;
of unit vectors along r without changing their din:ctions. We'll leave the details 10
Problem 13 just 10 ~ve space, but it is s1raightforward (and fairly brief) to show
that

( I 8)

iJe,t,
- = 0
80

The~ results arc summarized in Table 8.3 al the end of 1he section.
We're now ready to find 1he spherical coordinate exprc.'>sions for div u.

fin-+
aco
-ifr
[Ju t/)
eA,
Je,p )
+ 1' ., - ar '
v i:fr y, ~·

r
(a.
+ -e/j • - 1'.r
[)I)
e + I.I aer
- + -auo c,., + u0 -iJey·
a0 ao " il0
-
dll ip
e.;.
r r A0 "'

Use Equations 17 and I 8 Lo get

. 011 ,
dJVU== - +
11,
- + --
I 011 ,~ "r
11 cos fl
+-+ -11 - - + -- -
I i1Ltif,
ar r r iJ 0 r r sin fJ r sin 0 iJ</)

I rJ(r 1 11r)
0 I i-l(u si n H) I iJu,J,
==---+---_.:..__--+---- (19)
2 r or r sin 0 a0 r sin O or/J
8.4 Sµherical Coordinal«-, 375
To find v' 2 , substitute 1/r = 'df/ar, U(j = ( I/ r)Wf/00). and "<Ji =
( I/ r sin 0)af a<t, from Equation 12 into Equation 19:

., = d1v. grad f
<v-J a r + --
= -·
2
2 af
+- I
-- a ( sin. 0-
aJ) + - -1 - a J 2

ar2 r iJr r 2 sin (} ofJ ofl r 1 si n2 0 il</) 2


(20)

Example 5:
Show that f(r. 0. </)) = r sin 0 cost/, s.alisfies Laplace·s e<1ua1ion.
SOLUTION:

. . ,, 2 sin (J cos <I> r cos rf>(cos 2 8 - sin 2 0) r sin 0 cos <I>


v- 1 = - -r - - + , .
r- sin 0 r 2 sin 2 0
2
cos tf> ( . cos'.! 0 - sin 0 sin 0 )
=- - 2 Sin O + - -- - - - - -
2
r in0 sin 0

= sin 0 cos <I> (2 sin. 2 B + cos"" 0 -


1
1 . ,
sm· 0 -- I)= 0
r sin- 0

We'll leave the derivation of curl u to Lhc problems: the result is (Problem 15)

curl u = 'v x u =

or
er r e0 r sin 0 eip

curio= - --
I a a
-
iJ
(22)
r 2 sin 0 ar ao iJ</>

llr r11 0 r sin 0u"'

Example 6:
Evalua1e div u and curl u fur u =re,.

SOLUTION: Equation 19 gives


div u = 'il · u = 3
and Equation 21 or 22 gives
curl u = 'il x u =0
376 h,iph'( n/ Cun,ilin1~,1, Cumdinales

Table 8.3
Some useful formulas in spherical coordinates.

l1r = I h9 =r h~~ = r sin <J,

oe, =0 aer Jer .


-=ev -=SIii 0 ei,1,
or a0 i:l<J,

iJe,, =-= Jc ac(I


0 - = -c - = cos O c
cir a0 r o<J, IP

Je,,, ae.,, cler1,


-=0 - = 0 -· = - sin 0 e, - cos 0 ea
or '10 D<J,
rJf I iJJ I iJ/
grad f
· = 'v f = -ar c
r
+ --
r iJ8
e0 + ---
r . in 8 o<J>
c.9

. " I i)(r 2u,) I a(ur, : in 0)


dIVU= v · U = - - - - + - - -- ' - - - - + - -Ollit,
-
r2 ur r sin 0 ae r ·in 0 o</)

0 -rJ.f)
2
n2/
V
8 f
= -- + -2 -of + 2- I-- -
iJ ( •·
t,10 + -2 -I 2- a°~-J
ar'! r i:Jr r in 0 J0 aB r sin 0 i:Jcf/·

curl u =V x u =- I- [ -a sm . - -a"o] e,
. 0 11~)
r sin B ao c)</J

Problems 22 and 23 ask you LO use a CAS 10 evaluate the gradient, divergence.
and curl in spherical coordinates.
We summarize the key fonnulas of this section in Table 8.3.

8.4 Prohlems
I. What is lhe relation between geographical la1iwde (a) and longitude (/3) and the angles 0 and tp:
2. Use Equation 2 Lo calculate I.he arc length of the equa1or of a sphere of radius o .
3. Show 1ha1 d.<. 2 = dr~ + r 1dB 1 + r 2 sin 2 Odt/>1· in spherical coordinates .
4. E\'alua1c the integral in Example 2 using cartesian coordinates.

5. Evaluate Lhe in1egral / =// f x 1 dxd_vdz. over a sphere of radius n centered al rhe origin and compare your
rcsul1 lo rhe one obtained in Example 2. Why are the answen:. lhe same?

6. Evaluate lhc integral / = .!ff r


1
dxdydz over a sphere of radius a cemered at the origin and compare your
rc::.sull to those obtained in Example 2 and Problem 5. Why is the answer to thi~ problem three times the an~wer
l() Problem 5 and Example 2?

7. Use spherical coordinates to evaluate the integral/= fJf e- u cos~ 0 JV over all space.
8.4 Sphericc1I Coordin.:ilc$ 377
8. Consider a sphere of unit radius so 1ha1 d S = sin 0 d0dtj). We call the region connecting the origin with JS a
sn/id angle. dQ (see Figure 8.29) . We express 1his by writing dQ = sin 0 d0dct, . Notice lhal a complete solid
angle Q = 471 (called 4;r srerodia11s) ju.'.I as a complete circle = 2n radians. Evaluate }! sin2 8 cos2 0 dQ
s
over lhe surface of a sphere of unit radius.

Figure 8.29 /
An illu~trntion of a soli<l :tnglc dQ . iX

9. Staning with r = x i + y j + : k = r sin 0 cos cp i + r sin 0 sin t,b j + r cos O k, show 1ha1 or/ Jr. ar / o0 , and
ar /ur/J are mutually orthogonal.
10. Show that h 11 =rand l14i = r sin l:J.
1 I. Show that e~ can bi! obtained from er by adding ."f /2 to 0 .
12. Evaluate grad J if J = r cos ,p.
13. Derive Equations 18 hy s1anj ng with Equal ions 14 through 16.

14. Show that f(r. 0. tJ,) = r 2 sin~ 0 sin 2</) satisfies Laplace\ equation .
15. Derive Equarion 21.

16. Show 1ha1 Equation 22 is equivalent to Equarion 21.


17. U~ the metri<: cocfficienls J,r = 1. l, 11 = r, and h4, = r sin Band rhe entry in Table 8.3 10 show that

if 11 1 = r, 11 2 = 0. and u .~ = rp.
18. U1-C the me1ric cocfficienis It, = I,""' = r. and hi:,= r sin O and the eniry in Table 8.3 to ~how that

'11 e1 /,'2 e2 J, J. e3
I a a a
turl U = 'v X U = ---
li1h2h 3 OU1 011'2 ou 3
I, 1/l I l1111~
"J" 3
378 l.h,1plPr H .' (. wvilin1•,ir ( ourdin,11!•,

19. An inLcgral that occurs in o number of applicmions (and one thal we will encounter in Chapter 17) is

r{{ lr{{
00

nin kr . .
}} j(r)e' •rdr. Show that F(k) = }} f(r)--k-dr by lclllng k • r = kr cosO and using a
"k
F(k) =
1
spherical coordinate system with k along lhe ··z axis." The no1a1ion dr means r 2 sin 0drd8d<J,.
20. Show that the great circle distance between two points of lati1ude and longitude (a 1• {J 2 ) and (a 2 • /j 2 ) is given
by d = R cos- 1[ cos( {) 1 - fh) cos cr 1 cos 02 -! .,in a 1 sin a2 ].

21. Evaluate Lhe integral IfJ(lu +by+ cz)"!.i'dxdydz over the region x 2 + y 2 z~ s R2 . Hint: Recognize that

ox + by + r;: is r • v where v = tJ i + h j +ck and take the··.: axis" to be directed along v.


22. Use ony CAS to tind grnd f if f = e_,.i cos 2 B sin</>.
23. Use any CAS to find div u ond curl u if u = e_,J cos 0e, - r 2 sin tp e 0 + cos fl cos tJ, e.p.

8.5 Curvilinear Coordinate

In the previous sec1ions of this chap1er, we deduced the fo1m of the differential vol-
ume elements in cylindrical coordinates nnd spherical coordinates geometrically
with the aid of Figur~s 8.18 and 8.26. We found 1ha1 d A == rdrd0dz. in cylindrical
<.:oor<linafcs and thnl d A = r 2 sin 0drd0dq, in spherical coordinates. We can actu-
ally derive these relations analytically using vector meLhods. We can handle both
cylindrical coordinates and spherical coordinates simuilanc.ously by considering
rhrcc general 011hogon a I coordinates. 11 1• " 2• and 11 3• where 11 1 == r. 11 2 == fJ. and
11 J == z in the case of cylindrical coordinatc.s and u 1 = r. 11 2 == fJ. and u 3 == rp in the

case of spherical coordinates. We say that u 1• 11 2• and u 3 are orthogonal coordi nales
if the surfoces u 1 ;::: constant. 11 2 = conslant, and 11 3 == constant intersect at right
angles. Let

(I)

be Lhe equations Lhat rela1e Lhe x, y. : coordinates 10 the u 1• 11 2 • u .l coordinates.


(Compare Lhcse to Equa1ions l of 1hc previous 1wo sections.) U the functions
x(u 1• u2, u 3 ). y(u 1. 11 2 • u~), nnd z(u 1• u 2 , 11 3 ) are continuous. have continuous first
partial derivatives. and have single-valued inverses. then Equations l represent a
one-to-one correspondence between points in the x, y. z coordinate system and
I.he u 1• u 2 • u :i coordina1e system. The position vector r in either coordinate system
is given by

Now consider a differential volume element in the xyz coordinate system.


where dr = dx I+ dy j + dz k. We learned in Section 5.3 that the volume of
a parallelepiped formed by lhree non-coplanar vectors u. v. and w is gi\"cn by
8.5 Curvilinear Coord in a.t s 379

fu • (v x w)j, so that rhe volume element in the xy::: coordinate syslem is given
=
by dV = ldx I• (dy j x dz k)I Ii - (j x k)I dxdyd:.. Bui i,j. and k are mutually
orthogonaJ unit vectors. sod \I = dxdydz.. Now consider a differential volume ele-
ment in spherical coordinates. where dr = dr e, + rdfl e0 + r sin 0 e4i. Recall that 11 1 C 11 :-'i ll ll l

ar / ar is tangenr 10 the r cur1e and tha1 ar / o0 is tangent 10 the 0 curve, and or/ cJ<J, is
tangent to rhe <I> curve. The differential volume element in spherical coordinates is
=
given by dV = ldr c, - (rd0 e0 x r sin() c1 )1 r 2 sin(:) drd&d</JI e, · (ell x e41 )1.
But e,. e 0 . and e~~ are orthogonal uni I vectors. so je, - (e 11 x C,t.) I = I and d V = u ~ con !ant
/ 2 sin 0 drd0d<J>.
Now let's consider the corresponding differential volume element in terms
of 11 1• u '.!.. u 3, which we wi II call curvilinear coordinote.~. The vector or/ Ju I will
Figure 8.30
be tangent to the u I curve. which is 1he curve that is formed by the intersection The inlcrsc:<:tion of two coordinalt:
= =
of the u 2 constant and 11 3 constant surfaces (Figure 8.30). Similarly. ar;a11 2 surfaces produces a coordinate curve.
is tangenl lo the u 2 curve and 'dr / 011 J i~ Ian gent to the 11 3 curve. Thus, the three
vectors rJr / nu 1. rlr /u11 2• and nr /011 3 are mutually orthogonal (Figure 8.31 ). They
are not necessarily unit vectors, but we can form unit vectors by writing

'Jr/ou1
eI -
- -
, - - ·' (3)
lor/iJu ii
or

(4)
p

where h_; = [c3r/ou 11 for j = l. 2. 3. Using Equation 3. we can wri1e

(5)

Example 1:
Show tha1
Figure 8.31
1ne coordinate surfaces of an onhogonal
coordinatt: system are mun.rnlly
perpendicular at any point of inrerscction.

SOLUTION:
380 Ch,1pler 8 / Curvili•w,u CoorcJin,1lc•,

The differemial volume clcmenl in the 11 1• 11 2 • 11 3 coordinate system is given


by

Bui e 1• e2• and e 1 are mulUally orthogonal unit vectors. so le 1 • (e 2 x e3)1 = I and
Equation 6 be(:1.1mes

(7)

which is our desired result.

Example 2:
Show that Equation 7 i::; consi~tcnt with our earlier result.c. dV = r drd9d'tj) for
cylindrical coordinates and dV = r 2 sin 0 drdOd<J:, for spherical coordinates.

SOLUTION: Recall that hr = I. h .. = r. and/,~= I for cylindrical


coordinates and 1ha1 hr = I, h0 = r. and h4' = r sin 0 for spherical
coordinates. Therefore,

h I h2 h-' du I d111 d113 = r drd0d:.


and

2
h I h2Ii,1d111 c/11"1 du'!-= r sin 0 drdBd,P

for cylindrical coordin..11es and spherical coordinates. respectively.

Equa1ion 7 is of1en wri1ten in the form

(8)

Recall from Section 5.3 that the expression in brackets in Equation 8 (called a
scalar w:ctor producl) can be w,inen in determinantal fonn (Problem 7)
ax Dy a.,
iJu I au1 8 111

nr ar ar ax r)y a..,
- · -X-= (9)
clu1 0112 8113 0112 0112 0112

ax ~ az
OUJ 0113 OIIJ

(We haven't dis.cussed dc1em1inan1s "officially'· so far (see Chapter 9), but we
assume that you remember how 10 expand n 3 x 3 de1em1inant. That's ..ii.I you·11
need to know here.)
8.5 Curvil i11P,i r Coordin,11 •1,
381

The type of determinant in Equa1ion 9 occurs fairly often and is called a


Jocol>ion detumimmr. or usually just a Jarnbia11. In particular. the Jacobian in
Equation 9 i~ said to be lhe Jacobian of x. y. z wi1h respect to 11 1• 11 2, 11 3 and is
wriuen as o(x, y. z.)/rl(u 1, 11 2 , u_1) so that Equal.ion 8 become~

( JO)

hample 3:
Evaluate the Jacobian for polar coordi:na1es x = r cos O and _r = r sin 0 .
SOLUTION: We want 10 cvaJuale cJ(x , ·)/rJ(r. 0).
ax ay
il(x, y ) 8r or
--- =
;J(r. 0) ax
- a_r
i)0 a0
= Icos0
-r sin fJ
sin 8
r cos0
I

Therefore. d V = r d rdO in polar coordinates .

We can also deri\'e expressions for differential area elements in curvilinear


coordinates. Let's co.nsider the area element involving the 11 2 and u 3 coordinates
=
on lhc " 1 constant surface. For example. 11 2 and 11 3 would be: 8 and <I> on rhe
surface of a sphere of fixc{I r.tdius in spherical coordinates. Figure 8.31 shows thal

dA 2 = l(h 2 d11 2 e2 ) x (1, 1 du 3 e3)1


= h~ 11 3\ e2 x e3 I du 2 du 3 = /r 2 '1_1 du 2 du 3 (I I)

since e 2 :md e3 are mutually or1hogonal unil vectors. The area elements on other
surfaces are similar 10 Equation I I.

Example 4:
Ui.c Equ::11ion 11 to derive an expression for the differential area element on
the surface of a sphere.

SOLUTION ; According 10 Equation 11. we use

in ::agrL-emcnt with Equation 5 in the prcviou.~ .-.ection.


382 Chap1er 8 / Curvilint•,ir ( oorrlin.:iles

We've derived equations for arc length of a space curve in terms of Lhe ej and
I, j in previous sections. To do this in general. start with

and form

( 13)

If u 1• u 2• and u3 depend parametrically on some variable , • then 1he arc length is


given by

We can derive an expression for the gradient in curvilinear coordinates by


recaHing that

df = grad / · dr =V/ · dr ( 15)

( 16)

and using Equation 12 for dr gives

df = 'vf · dr
= [('v nu. e1 + ('v /) .. 2 e2 + (V f),,l e3] ·(/,I e1 du I+ "2 e2 du2 + li3 e3 d113)

(17)

Comparing Equations l6 and 17. we sec that

or

(19)

in operator fonn.

Example 5:
Show how Equation 18 is consistent with our previous results in cylindrical
nnd sphericaJ coordinates.
8.5 Cun·rlin ',I f Coordin,lh"' 383
SOLUTION: for cy1Ji1ndrical comdinates. 1 = r. 11 11 1 =f).and ,, 3 = z.. and
hr = I. h8 = r, and I, :. = 11, so Equation I8 givc.s
V r = ---:--- e,
aJ I aJ
+ - - e,. +
ar CJ.
..d=o
• iJr .r afl ;J,:::

Similarly. 11 1 = r. 11 1 =(},and u 3 =</).and h, = I, hfl = r. and h 9 = r sin 0,


so
I ri/ fJJ
'v J = -ilf
clr
c
r r nO
I
+ - - e0 + - - - e~
r in Ooff>

We can derive an expression for the divergence by evaluating 'v • v using


Equation 19. When you do this. you must remember 1hat 1.he unit vectors cj are
not tixed in space, and so may depend upon 11 1• 11 1 • and UJ, We've e\'aluated lhe
derivatives iJe ;/ ouJ..- for cylindrical and spherical coordinates in previous sections.
bur Problem 17 helps you show that

i = I. 2. 3 (20)
k :::::: I. 2. 3

and

i)ci
-=--
e j oh i
i :f. j
i = I. 2. J (21)
j = I, 2. 3
in general. Thu~, the various derivatives of the cj depend upon the meLric co-
erticients (a.s does everything else). The following Example !.hows how to use
Equa1ions 20 nnd 21.

Example 6:
Use Equations 20 and 21 10 deri\'c the resulrs in Table 8.2.

SOLUTION: First nore that hr = I. 11 11 = r. and I, : = I. From Equation 20

;-J er I i:l hr I i'I/Jr


-.-
rJr
= -J,t,- -e -
JO 0
- -.-
,._ o~
e_
L

= -er
iJc. I i:J/,. I oh.
_ L = - - -' c - _ _. co
il ::: l,r ;Jr r lta c)tJ
384
Usim?. Equarinn 21.

oer C a/1 0
- =- - =eo
ao It, ar
oc, = !: air.: e_ = 0
a~ hr or ~
oe 0 = er ohr = O
i)r Ii ;)8

i)e.- = e, ah, = O
ar Ii ~ a;:
~c.-= ~ a111J =O
ao Ji . a:.
Problem 12 a~k.., you to do this for spherical coordinates.

Using Equations 20 and 21 along with div v = V · v gives (Problem IO)

(22)

We can also derive Equation 22 by starting with the defini1ion

. . f,·-ndS
d1v v = hm -·- - - (23)
.w-o b.V

and considering the llux in and out of the volume l:l. V (Problem 11 ). Similarly, it
turns out thar (Problem 13)

curl,,= V xv

+ CJIJ
.
/ [ i:J(h~t•~)
--- - ---
Jui a112
i)(I, IV1)] l
'11 e1 h2e2 l.r3-e3
a a a
- - -- i) 11 J
(24)
h1h2h3 0/11 i.J112

h1V1 h2V2 l, ., v:,

Finally. we can derive an C)(prcssion for 1.hc Laplacian operator in curvilinear


coordin:.ites by using 'v 2 f = div gr.id J = 'iJ • 'OJ and Equations 18 and 22 (Prob-
lem 14):
8.5 Curvilinear Co..irdinares 385

(25)

Example 7:
Use Equation 25 to derive Equation 20 of Section 4.

SOLUTION: Recall 1ha1 h, = I. hn = r. and h,:, = r sin 0.


p

v-., f = ---
2
1 [ -a (.,.
r sin O
r· sm 0 -aJ) + -i:J
or nr o0
( sin
. a/) + -o<J>a (- -0 -&<J,)
0-
u0 sin
aJ\]
1

= --I ()d/) + - -I - -
iJ
r2 ilr
r·-
i}r r 2 sin 0 ilA
i.J (sm0-
· of) + - -i - a J
J0 r2
2

n2 0 cJcp2

Figure 8.32
The volume fonned by a SC I of in1cr.-ec1ing
orlhogon,11 coordinate surfaces and used
to help derive an exp~ ion for div v in
8.5 Problems curvi linear coordinate. in Problem 11.

I. Dc.-.cribe the coordinate surfaces and the coordinate curve .. in cylindrical coordinate~.
2. Describe the coordinate surfaces and 1he coordinate curves in spherical coordinates.

3. Show that cylindrical coordinates form :m orthogonal coordinate sys1cm by ~howing thar the or /011 j are
orthogonal.

4. Show that spherical coordinates fonn an orthogonal coordinate system by showing I.hat the ar/cJ11j are
or1hogonal.

5. Express the vecror v = xy i + : k in a cylindricul coordinate system.


6. Show that i · (j x k) =L
7. Show that u • v x w can be cxpre!,scd in terms of a detcnninanr.
8. Evaluate i)(x. y. :)/iJ(r. f), ;:) for cylindric:ll coordina1c~.

9. Evaluate i:l (.r. y. :) /'d (r. 0. </J) for spherical coordinaJes.


10, Derive Equution 22 using 'v • v and Equations 20 and 21.

t I. We can derive an expres-"ion for Lhe divergence in curvilinear coordinates by s1aning with the definition
f.., v • n dS
div v = lim ·· . where the surface S encloses the volume V. Let V be fonne<l by a set of intersecting
v-o V -
onhogonal coordina1e surfaces. as in Figure 8.32. and calculate the now of v through all the surfaces to derive
an e:(pression for div v.
12. Use Equations 20 and 21 to verify the corresponding entries in Table 8.3.
386 Ch.1r1c r 8 / Cuniilin(w Coord in:i r

Figure 8.33
=
The 11 1 c1,ns1an1 curvilinear surfat·c that
is us d to derive ~in exprc-.~,ion ror curl v
in curvilinear coordinate, in Probkm 13.

13. We can derive an expression for the curl in curvilinear coordinates by starting with the definition
} ,, · d
curl v = lim - - - . where 1he curve s surrounds the area S. Consider the 11 1 = c 1 coordinate !--Urf:.ic.:c
S-·O S
in Figure 8.33. EvaJuarc J v • ds arounJ 1.he: path in Figure 8.33 to derive an expression fur rnrl v.

14. Derive Equation 25 using 'v 2 J = div grad f = 'iJ · 'iJ f and Equations 18 and 22.
15. Use Equal ion 11 to calcu la1e the surface area of a unit sphere.
16. Use Equ~irion I I to calcuh1te 1he 101al surface area of a right circular ,one of sl.rnt height s with a base of
radius R.
17. This problem helps you derive Equations 20 amJ 21. S1art with Equations 4 and use the fact that
a2r/a11,011j = o2r/011jau, to gel

al, j ae j a1,, ac;


e · - +lr•-
1
=e--
1 +h·1 - ( I)
1 J11·1 011·I 011·J iJ11-j

lei E;j = iJe;/ou J so 1haI (I) can be wrinen as

(2)

with no restrictions on i. j. or k. Now di ffercntiatc e; · ei: = 8;A· ( KJoncckcr delta) and show that
(3)

with no restrictions on i. j. or/.;, Use (3) 10 show tha1

(4)

Convince yourself that (4) represents nine etiuations. Operate nn (2) with ej (J '=fa i) and use (4) to gel

() # i) (5)
387
Similarly, operate on (2) with e,. (k -=I= i, -:f j) to get

(6)

Now ,uh"'lilule (J) into (6) to get

(7)

Let i. j. k = ( I. 2. 3). (2. 3. I). and (3. 1. 2) and show 1ha1 E;j · c1; = 0 for i ,=. j =fa k. C,i11~ 1hc,c resulL..;.
show that

I oh~
= - -- c,
I, I U/11 -

0C1
and - = -1 -iJh ~c1 and -ik,- = - I -;JI, 1 c 1• or cncrally
1
0

au, "1 a,, 1 · au, h1 0 11 2 °

· c, I M, i e
-=- -
;:i,,. )
1,.;:ii,.
I r
J

Now show 1h;11

== -<E~ 1 • e 1) e.! - (E3 1 • c 1) e.1


I iJI, I iJ/i 1
= - - -1e l - - - e
h: a,., 1
1
- 1,-' i:111 .1 -

and
I ii//., 1 A//.,
-i:le,- = ---- e 1- - - .-- eJ
i.lu1 h 1 iJ11 1 h_1iJ11_1

i:l e3
= - -I ilh 1
. Ct - I ilh 1
au "i nu, -
"~ i>111
- · C,
-

8.6 So me Other Coordinate Sy tem


T he only coordinate systems 1hat we have discussed so far are plane pol'a r cour-
dimates. cylindrical t: )Ordina1es. and spherical coordinates. If a physic,tl system
has a certain degree of sym metry, 1)iicn choosing the "right" coordi111ah.: ~ystem
10 describe Lhe system can greatly simplify the probfcm. For examp le. spherical
comdina1cs are namrnl comdirm1cs in dealing wi1h sy ·terns Lha1 have a cerHer of
symmelry. There are a number or well s1udjcd coordina1e. ys1cms. each of which
388 h,lpl r 8 / urvilin .1r rdinates

is appropriate for problems with ccnain symmclry. In this section. we shall briefly
introduce. a few other coordinate systems 10 give an example of what's available.
A prola1c spheroid is obtained by roiating the ellipse shown in Figure 8.34
about its long axis. Suppos.e we want 10 determine the electrostatic potential or
the clet·tros1a1ic field about an isolated me1;1llic prolate spheroid whose surface is
at a potential V. The narural coordinate 10 u~' iin 1l~is ,case ~s a prolme spheroidal
1

coordinate system. We shall also see that this same coordinate system can be used
10 calculate 1.he propenics of a molecular hydroge.n ion. rHi.
which con i. t. of two
(mas i1ve) nuclei separated by a di~rancc R wim or,re elec1ron imcracting with them,
Figure 8.35 shows the geometry for this system.
We can generate a prolate spheroidal coordinare system by r0l'a1i11g a family
of ellipses and hyperbolas having the same foci (confocail ) abour rhe major axis of
Figure 8.34 Lhc ellipse, as shown in Figure 8.36. The distance between 1he two foci is 2a and
A prolurc sphcroid is ob1aincd by rotating
1he cUip~c in lbe figure abou1 i1s long axis. 1/, 8, and rt, are 1he prolate spheroidal coordinates. The relations between x, y, z.
and 11. 0, </> arc

x = r1 sinh ,., sin fJ cos ,P


y =a sinh 'I sin [I sin f/J (I)

z. = o cosh ,, cos 0

Figure 8.35
The geometry :u.~,Jci.11ed with a h,)tt1 rngcn
molecular ion . .H ~. ·111e IWCl nuclei :uc
~ep.rrulcd by a. di~l~mL·c R = 2'_i and the
electron i.s locakd uc lh~ point (.r. _\', ;:).

d>=tc / 2

✓ y

Figure 8.36
A prolate s.phcroicbl coordinat.c system.
The coordin,al(' ,urface are prolate
;,,pheroidi. •iven hy n - con,tanr. Fixed d,
hypt•rboloid_ o.f ,t1\' h et., giH,·n by = ~
constunr. and pi rmcs cont.ainin~ tb ~ :::. axi ~ fl = ,r
f X
gil'cn by tJ, = c ,o-1ant.
8.6 Some Other Coordinate Systems 389

Let's look a1 each prolate spheroidal coordinate surface in tum. The surfaces
Y/ = constant are prolate spheroids

c>b

where b = a sinh ,, and c = a cosh ,,. and 17 varies from O to oo. The surfaces
0 = constant arc hyperboloids of two sheets. The angle 0 is 1..he angle between
the asymptotic cone of a hyperboloid and the;: axi~ as shown in Figure 8.36, and
varies from O10 rr: 0,:::: fJ < ;r /2 describes liic upper hyperboloid in Figure 8.36 and
;r /2 < fJ S rr describes the lower hyperboloid. When fJ = rr /2. the two hyper-
boloids degenerate into lhe x y-plane. The angle <J> simply represents the angle
abou1 1he: axis and varies from O10 21T. The ,P = constant surfaces arc ha.If planes
containing the ::. axis. Equations 2 summarize the full ranges of the three coord1-
na1cs:

Q _:5 IJ < 00: (2)

As wi1h uny new coordinate system. it takes a lirile practice and experience to
become cornfonnble with ii.

Example 1:
Show 1ha1 prol:ire spheroidal coor<lina1cs are an orthogonal coordinate
.._,,1cm.

SOLUTION: Write r = x i + y j +: k 111 rerms of,,. 0. t/>, using


Equations I.

r( 11. 0. <J,) =v sinh ,p,i n O co~ (jJ i + a sinh 11 sin 0 sin (/, j + o cosh 11 cos O k
Then.

Jr = a cash 11 sin O cos <p i +" cosh 11 sin 0 sin 4> j + tJ sinh 11 cos O k
i:lJJ

-r1r = ll s Inh ,, c:os O cos 4> .J ·a •


:-1nh 11 i:os it :-.1n 1:/J J - 11 .
co~h 11 :-in (J k
80

-ur = -a sin. h ,, sin. fJ sm. </J I + a smh . n,., cos </J j + (} k·


. TJ sin
u</J

and

i)r ar i:Jr ar a r ar O
-
a~ ·ao- =an
- · - - - ·--
a~ ao a~
390 (.h.1p11•r 8 / Curvilinear Coordin,11t·,

We can use Equations I to evaluate the scale factors I, w 11 0 , and /Ja,. For
eitample.

= (a 2 co~h 2 11sin 2 0 cos 2 tj) + o 2 cosh 2 17 sin 2 0 sin 2 ,p + a 2 sinh 2 ,, cos2 8) 112
= (a 2 cosh 2 ,, sin 2 0 + a 2 sinh 2 11 cos 2 8) I/?.
= a [( I + sinh 2 17) sin 2 0 + sinh 2 r,( I - 2
sin 0) 1
1
n
= (sinh 2 11 + sin 2 0) 112
11 (3)

Similarly {Problem I),

h 0 =h,1 and J,w = a sinh sin fJ (4)

We can use lhe scale factors Lo calculate I.he volume of a prolate spheroid.

hample 2:
Use prolate spheroid;il coordinates ro calculate the volume or a prolate
spheroid.

SOLUTION:

1 1
8;r ,r
= 4;ra·1 (cos.h·
--- 110
- cosh 11 0 + -2) + --(cash 'lo - I)
3 l '.'

4rr11 . , 3
=- - sinh~ 'lo cm;h 1/o
3

Bui h = o sinh 'lo and c = o cosh 110• so


4 tt ,
V =- Ire
3

Problem 2 nsks you lo c.alculatc the surface area of a prolate ~phcroid in u similar
way.
8.o Some Other Coordinate Systems 391

Laplace·s equation in prolate spheroidal coordinates is (Problem 3)


2
v' 21 _
- . h" 1 . (;J2J + co th r,-
of + -.,
a f + cot 0 -af)
£J 2 (sm ~ 1J + sm 2 !7)
11
a,i-
-..,
011 iJ0- u8
(5)

This equation may not look too inviting. but it reduces to a fairly simple equation
under certain (realistic) conditions. Suppose we want to determine the electro-
static field about an isolated metallic prolate spheroid 1J = TJo. which is at a fixed
potential V0 . (Recall that Laplace·s equation governs the electrostatic potential
throughout a charged-free region.) In this case. the potential will depend upon
only JJ and so Equation 5 will become

, d 2V dV
'v~V = - -1
+ coth ry- = 0 (6)
d,,~ dYJ

We haven't studied differential equations yet. but surely Equation 6 is a lot simpler
than Equation 5. and this simplification will occur only in a spheroidal coordinate
system. The solution to Equation 6 is

. In tanh(17 /2)
V(11) = \/o----- (7)
1n tanh ( 1Jol 2)

(Equation 6 is actually easy to solve and Problem 5 helps you solve it.) Figure 8.37
shows that the equipotential surfaces are spheroidal surfaces 17 = c 1• Figure 8.37
There is another way of expressing prolate spheroidal coordinates that is used The equipotent ial surfaces (color) and 1he
in molecular quantum mechanics, Consider two nuclei separnted by a distance corre~ponding electric fie ld (black) about
a prolme spheroid held at a fixed po1entiol.
R = 2a and an electron located at a point (x, y, z). as in Figure 8.35. Let r 1 and r 2
be the distance of the electron from each nucleus. Then r 1 + r 2 = constant maps out
a prolate spheroid and r 1 - r 2 = constant maps out the hyperboloids in Figure 8. 36.
To see this analytically. we use

and

and Equations 1 10 get (Problem 6)

- r-,
and Jl = r1--~ = cosfJ and r/>=<I> (8)
2a
The two coordim1les >..andµ and</>. the angle about the.:. ::.ixis as in Figure 8.36.
constitute an orthogonal coordinate system equivalent 10 the prolate spheroidal
coordinates '7, 0. <fJ (Problem 7). This coordinate system. which is sometimes
called a bipolar coordinate system, is shown in Figure 8.38.

al
392 Chap! r 8 / Curvilinear Coordin.ill',

--
y

Figure 8.38
A biJX>lar coordimitc system. where /
:,_ = (r 1 + r~)/2a, µ = (r 1 - r:! / 2a. and / x
<J> i$ the angle :iboul 1he inti:rfocal :1.i,is.

Because >.. == cosh r, aml O _s 17 < , Lhen I :::: i.. < oo (as you can also see
pictorially from Figure 8.38). Also. becauseµ == cos(). then - I ~ µ :::: I. Of course.
</>. being the same as 1he <I> in Figure 8.38, varies from Oto 2rr. Thus, we have

0 .S <I>,::::: 2rr (9)

Notice that Equa1ions 8 say that .A.== cons1 ..m1 (11 == constant) surfaces are
prola1e spheroids and µ == constant (0 == consrant) surfaces are hyperboloids.
Problem 8 has you show Lhal

( I 0)

Example 3:
Use Equal ions I Oto cukula1c the volume of ;.i prola.1e spheroid.

SOLUTION:
B.6 Some 0th r rdinatc Syst m 393

because >.. 0 = cosh 'lo= c/a and ). 2 - I= sinh 2 110 = b"2./a 2.

An integrnl that occurs in .1 qunntum-mccha.nical 1rcatmcn1 of a hydrogen


molecule j_,_

J =; fff e-r, ,- r1dv ( 11)


all '-I cc

where r 1 and r'"! are as depicted in Figure 8.35. This in1egral is fairly awkward to
evaluate (see Problem I 0) unlc~~ we use the bipolar coordinate). A, 1.1.. </J. in which
case it's simple. In tenns of A.. 11. and rt,. I becomes

= 40 -1
f l
x d'Al' -2a;. ( J.-
. 1 - -1)
3
3
=~ - ( R) - + -2
e- R(2 + -2)
2 3R R2 R3

( 12)

Physically. / i~ a measure of the overlap of the wave function centered on one


nucleus with Lhe wave func1ion cen1ered on 1he other nucleus und is called an R
m·erlup i111egml (Figure 8.39).
Another coordinate system thar is similar 10 1he one we have discussed here is
figure 8.39
The o,·crlap imc~rJ). Equation 12. which
an obla1e spheroidal coordi11nte sysu:m (Figure 8.40). The three oblaIe spheroidal i\ ..1 111ca.~ure uf rhc ovc.rlap of the wuw
coordinates n.re related to x. y. ~ by funclion tcn1crcd on one nu<.:ku~ wirh
the waw t1111 ~1i,in ccntcred on the other
x = a cosh ri sin 0 cos tJ:, nuclcu~.

y = o cosh 11 sin 0 sin </J ( I J)

~ = a sinh '7 cos 0


We can generale an oblate spheroidal coordinate sysIcm by ro1...11ing an onhogonal
family of confocal ellipses and hyperbolas abou1 1hc minor axis of 1he ellipse
(compare to prolate spheroidal coordinates). As in Ihe case of prolate spheroidal
coordinates. 2t, i1, lhc distance between 1he foci. The 11 = constam surfai.:t.' '- arc

n
394 Chap! r 8 / Curviline r Coo rdinates

Fixed µ

2 y
JI,

figure 8.40
An oblate spheroidal coordinate system.

oblate spheroids

h>c

=
where b a cosh 11 and c =" =
sinh '7- The surfaces fJ constant are hyperboloids
of one sheet and <I> is the angle about the z axis. The coordina1es range from

( 14)

We leave the discussion of this coordinate system to a set of problems (Prob-


lems 12-16).

8.6 Problem
1. Show that J,fl = £J (sinh~ T/ + sin 1 0) 112 and J,4> = lJ .~inh T/ sin (:) for prolate spheroidal coordinates.
2. Us.c prolate spheroidal coordinates 10 calculate the surface area of a prola1e spheroid.
3- Show that Laplace ·s equation in prolate sphcroi<.lal coordinates is

.,2/ _
v - , . h.,
I
. l'l)
(:i'!-j +corh 11-
- ~
il/
2
af
+ - .. cotO- af)
a-(sm - ,., + sin- u
?
aw a,, ae~ aB

4. Show that j(r,, 0. </>) :- sinh ri sin 0 sin </J is a solut..ion to Laplace\; equation in prola1c spheroidal coordinates.
5. Here's how to solve Equation 6. Fil":\t let g = dV Jdri and then separate variables and in1cgra1c to gel g. Then
Rei1•11·11< •·~ 395
in1egr.11e again 10 gel V. You can dc1cm1inc 1hc two imegrn1ion con,tanb by using 1he fac1 that V(11) = 0 as
= Vo,
1J ----... oo and that \I I /Jo)
6. Start wilh 1hc equations r 1 = f(o + ;::) 2 + x 2 + _,.~ )112 and r2 = [(ll - .:) 2 + x 2 + y 211l 2 to derive Equations 8.
7. Show 1ha1 l. µ. rp in Equalions 8 fom1 an orthogonal coordinate system.
u(>. 2 _ 112)1 / 2 a(j. _ J1. 2) 1fl ,
8. Show Lhat Ii ;_ = .1 , : '1 1,
(A - - 1) 1/ -
= - (l-- -µ2)1
- -:
-
and 1, 41 = a().· - I) 112 (1 - ,i·)1/2 for the .A,µ.</)

coordinale system defined by Equal ions 8.


9. Use the prolate spheroid::i.l coordin:ues l . µ . tJ, to calcul.ite 1he ::.urt·:u:c area or a rrola1c spheroid.
10. Show 1ha1 tfrle integral in Equa1ion 11 is given by

I (R } = _!_ {:,;, dr 1 e-r 1r~ ,f 2.rr d(/> f'.'I d0 sin fh,-r! using spherical coordinates ccntered on nudeu:- A.
H k 1, k
.1, :-.hown in Figure 8.41. To evalua1c 1his integral. use the law
of cosines 10 e.\pre r 2 in terms of r 1• fl. and R.

figure 6.41
The gcome1ry used in Prot,Jem I O 10
C\'alu:111: the i111e!=ntl in Equali r'm 11. A

11. Another integral that occur.. in a qunntum-mcchanical 1.rcatment of a hydrogen atom is J = fJJ 1
' ~~
1
d V.

where the integr.i1ion is over all space. Use the ), , 11. </) bipolar c.:oordinatc !.ystern 10 show 1hu1

J = *- e- 2H (I+ "'ii). where R is ,the distance be1ween 1he nuclei .


12. Show that 1he oblate spheroidal coordina1e sys1cm is orthogonal.
13. De1ennine 1he scuk factor.- h 11 • h,., . and h,~ for oh]alc sphcroidnl c.:oordinalcs.
14. Use the scale facwrs that you dete.rmined in the previous problem to calculalc the \'olume of an oblate spheroid.
15. Use the scale fac1ors that you derermincd in Problem 13 to calcula1e 1he surface area or an oblate spheroid.
Show tha1 your answer reduces 1n that or a sphere if b c. =
")

16. Show that Lapbce 's equation in oblalc ~phcruiual coordinate:-: reduces 10 d- ·( + tanh r, df = 0 if J depends
d,r d11
only upon '1· Can you give an example of a physical problem where 1hi~ would be the case?

References

Parry Moon and Domina Spi:nccr. 1961. Field Thevryfur Engineers. van ~c i,trand
Parry Moon and Domina Spencer. 1971. Field The(Jry Ha11dhook: i11cfll(fing co,mli11aft:'
;)•.,·rt·m.~. dif/i'r1·111ial equations 011d rht'ir so/nrions. :!nd ed .. Spri nger-Vcrlag
1-1.M . .Sd1ey, 1997, dil·. xrrul. rnrl. and aJJ 1}1111. Jrd ed .. Nonon
M. Spic-gel. 1959. \, •c11Jl'A.1wl_,·1is. Sdmum·~ Outline Serie~ . McGraw-Hill
GENERAL:
Eli Maor, i 998 . Trig11110111t·rri(· D1•/ig/11.1·. Prin(clon University Prc~s
CHAPTER 9
Linear Algebra and Vector Spaces

Although we learned how to solve ~imul!.aneous linear algebraic equations in high


school. the fundamental theory behind the manipulations we learned consti1u1es
lhe study of linear algebra. one of the most sophisticated and beautiful fielru; of
ma1hema1ics. We can only touch on some of 1hc more rclcvanI 1opics for our
purposes in 1his chapter. In Scc1ion I. we introduce determinants and show how
they can be used 10 s-olve simultaneous linear algebraic equations by Cramer's rule.
As elegant ru; Cramcr's rule may be. it is no1 well sui1ed computationally. and in
Section 2. we use Gaussian elimination to not only solve 11 linear equations in "
unknowns, but other cases as wel I. The key quantity in Seel ion 2 is the augmenicd
matrix. In Section 3. we discuss matrices more fully. and learn how to multiply
matrices 1oge1her and to find 1heir inverses. among olhcr things. Section 4 deals
with the idea of 1hc rank of a matrix. one of lhc mosI important quantitic, for
dctem1ining the namrc of the solu1ions to ~ls of linear algebraic equa1ions. Closely
related to rank is the conccpl or Ii near indcpendcnce of a set or vectors. which leads
naturally to Section 5, where we introduce and discus.,;; abstract vector spaces. After
presenting the axioms of a vector space. we define a basis of a vector space and i1s
dimension. When we define 1he operation of an inner product between the vectors
in a vector space. we then have what is called an inner product space, which is
1hc subjccI of Section 6. Introducing an inner product allows u.'- 10 discuss the
lengths of abs1rac1 vectors, the ungle between them. the disrance betwec:n 1hem.
orthogonality, and a number of 01her geometric quantities. Finally. in Section 7.
we generalize the results of the previous two sections 10 include complex inner
product spaces. which play a particularly i.mpor1.an1 role in quantum mechanics.

9.1 Determinants
We frequently encounter simultaneous algebraic equations in physical applica-
tions. Let's start off with just two equations in two unknowns:

(I)
397

C gl
398 Ch.,pni:r <J / Linear i\lgehr :i ,m d V, tor S1 ;i ("S

If we muh..iply the firsl or lhesc equations by a 12 and the second by 0 12 and then
subtract. we obLain

or
x = a22h1 - ar2lz2
(2)
a 11 a 21 - a, 2 a 21

Similarly. if we multiply the first equation by 0 21 and the second by a 11 and then
subLiact. we get

Y= a 21 h 1 -a, 1h 2 (3)
lJ1 I 0 22 - a,2 °21

Notice that Ihe denominators in both Equations 2 and 3 are the same. If we had
started with three equations instead of two as in Equation I. the denominators
would have come out 10 be a 11 a22 a33 + t113 a2, a31+a1202.3031 - a13a-n_a31 -
a,2 a21 a33 - a, I a23 a32-
W\": represent a 11 a 22 - a 12 a 21 and the corresponding expression for Lhrce
simultaneous equations by

I 011
021
a1 2
0 21
I= a, -
l a22 - a,2 o2, (4)

and
aII 11,2 a,J
0 1l a22 ll33 !· 0 tJ 0 21 a32 + an °n a31
021 a22 0 23 =- a13 °22 031 - 0 12 °21 <133 - 0 11 a2.' 0 :12
(5)
ll31 032 0 33

The quan1i1ies introduced in Equations 4 and 5 are called a 2 x 2 dt!tem,inant


and a 3 x 3 dnem,inant. respectively. The rca!-.011 for inLioducing this no1a1ion is
that il readily generalizes to II equations in II unknowns . An 11 x II determinant,
called an 111/1 order de1em1i11011t. is a square am1y or 11 2 elements arrJnged in 11
rows and II columns. We'll represent a detenninant consis1ing of elements alj by
IAI . Nore t.har the element a,'j occurs in the i1h row and 1hc jrh column of IAI.
As of now. Equations 4 and 5 simply introduce symbols for the right-hand sides
of these equations. but we shall develop convenient procedures for evalualing any
size de1c1minant.
Lei's rearrange the righl side of Equation 5 in the following way;

a 11 a12 C113
:;:: n, 1(a2 ~ " 3J - a23 °:.nl - 0 12< 0 21 n:n - 0 :D 031)
IA I = a21 0 22 a2.1 (6)
+ a 13 (a 21 n_, 2 - an a _H )
03, (/'.\2 a .,3

Notice that each term in parentheses in Equation 6 is equal 10 the 2 x 2 dclcnninam


1ha1 is obtained by striking out the row and the column of the foc1or in front of
each set of parentheses. Funhennore. these factors arc the members of the first
399

row of IAJ. Thus, Equ<11ion 6 shows us that we can evalua1e a 3 x 3 de1crminan1 in


lcnns of 1hrec 2 x 2 de1em1inan1s.
We can express Equalion 6 in a sys1ematic way by first inLroducing the mi,wr
of an clemcni of a II x II dcterminuni IA[. The minor M;j of an clement 11,; is a
(11 - I) x (n - I) determinant ob1ai ncd by dclc1i ng the: i th row and the j 1h column.
We now define 1hc cofa 'IOr Ai) of alj by A,j = (-1)' .i MiJ. For example, A 12 •
the cofactor of a 1:, in Equation 6 is

The introduction of cofactors aJlows us 10 write Equation 6 as

(7)

Equation 7 represents IAI as an expansion in n~fi1,:tnrs. In particular, ii is an


expansion in cofactors about the firsr row of IA;. You can stan. wirh Equation 5
10 show 1ha1 IAI can be expressed in an expansion of cofocrors about any row or
nny column (Problem 5).

Example 1:
Expand
2 -1
IAI = O 3 -1
2 -2 I

in an expansion in cofactors about the second row and about the 1hird column
oflAI.
SOL u TI ON: We. USC Equa.1.ion 7:

IA I = -0( - 1 + 1 > + 3(2 - 2) - ( - J H -4 - 2> = -2


IAI = 1(0-6) - f-1)(-4 + 2) + 1(6-0) = -2

We shall show below 1hal Equation 7 readily generalize.-, to detcrminan1s of


any order.

Example 2:
TI1c determi11w11al eq11atio11

X O ()
I x 0
0 1 .x 1 =0
0 0 1 X

C gl
400 Ch.i pler 9 / Linear Algebra and \'tot lr:i r Sp,u ·,

occurs in a quantum-mechanical 1reatmen1 of a bu1adiene molecule. Expand


this dercrminantal cqua1ion in10 a quanic t:"qu::ition for .r.

SOLUTION: Expand about the tirsl row of elements to obtain

X 0 I 0
X I X 0 X =0
0 1 x 0 I x

Now expand about the first row of each of the 3 x 3 determinants IO obtain

(x)(x) I~-~ 1-(x)(I) I~ -~ 1-(1) I~' 1-(-1)10.


X O ' 1= 0
X

or

or

x4 - 3x~ +I= 0

Note 1ha1 because we c:an choo c any row or 1.:olumn to expand th~
de1enninan1. it is easiest 10 take I.he one with the most zeroes.

Up 10 now we have used 3 x 3 determinants to illustrate how to evaluate de-


termjnants. We're going to discuss a number of general properties of determinants
below, and so we need to discuss determinants more generally at this poim. FirsL
consider the product of elements of :m 11 x n determinant

where the j's are distinct and take on the values I through 11. Note that there is
only one clement from each n)w and oneclcmcnl from each column in this producL
and that Lhe first subscripts are in their .. natural order." Also note 1hat there a.re n !
possible producL-. because ) 1 can lake on one of II values. h one of 11 - I values,
and so on. Now consider the process of interchanging the eleml'nts in the above
product successively until the second set of subscripts is in its "na1uml order." This
will require eilher an even or an odd number of interchanges. For example,

require one and two interchanges. respectively. Now define the symbol

depending upon whelher it takes an even or an odd number of interchanges 10 order


the,, j's into their ··na1ural order:· For example. € 12 =+I. E~ I = -1. E 1J 2 = -1.
401

and so on. Finally. the general definition of an II x II de1erminan1 is

(8)

where the summation is over all Lhe 11 ! pemmlalions of I.he ) 1 • h_. · · · , Jw


Equation 8 is the formal definition of a determinant. It may not be familiar 10
you and may be even a little fom1idable. but we won'L have 10 use it to evaluate
a detenninant. Its primary use is to prove some imponant general properties of
de1em1inantc;;. First~ let ·.s use Equation 8 10 evaJuate a 2 x 2 detenninant. For a
2 x 2 detem1inan1. Equation 8 gives

IAI = LL €j1h alJ'i a2h


h h

In the. summations. j 1 and h wke on I.he value$ I and 2. but there are no 1em1s
with jl = h because the rs
are distincl. Therefore,

Using lhe fact that € 12 == I and t 11 = -1. we have

in agreement with Equation 4. Problem 11 has you show that Equation 8 yields
Equation 5 for 3 x 3 detenninant.
The real utility of Equation 8 i!- that we can use it to derive general results for
determinants. For example. suppose that we interchange rwo adjacent rows.rand
r + I. If we deno1e the rcsuJting detenninant by IA, r+ il• then

We can get the order of the fir.;t suh<-cripts back into natural order by une pennu-
tation. however. so IA,-,+il differ:, from lAI by one im·er:-ion . Therefore we have
that IAr.--.r . 11 = -IAI, To analyzc the interchange of two adjacent rows is fairly
e,L<;y. Suppose now that we want lo interchange two TO\\ s separnte<l by one row. I-or

concrcleness, le! lhesc rows be rows 2. 3. and 4. We can interchange rows 2 ,rnd 4
by the protcs~ (2, 3. 4) - (2, 4. 3)-. (4, 2. 3) - (4, 3. 2). which require:. three
steps. This is a genernl result and so we can wrile IA,..,.,+_I = -I AI. Problem 12
ha, you show that it requires 2k - I steps to interchange row. rand r + k. so that
we see th;11

(9)

or 1ha1 IAI changes ,ign when any two rows are exchanged. (This is prope11y 3
below.)
Equation 9 says that IA, •r+d = -IAI even if the two rows are identical. If
the 1wo rows are identic..:a.l. howcv~r. then the value of IAI cannot change. The only
402 C'h.1 plL·r CJ I lihe.ir Alt\ ·lir - .:i nd VPcl m Sp,L ·,

= =
way thal IAI - IAI is if IAI 0 . Thus. we see that IAI 0 if two rows (or 1wo =
columns) are the same. Thi.., i,;, property 2 below. The rcsr of the properties listed
below can be proved using Equation 8 in a similar manner.

Example 3:
Show that

2 2
IAI = o -1 o = 0
3 6

SOLUTION : Expand lhc cofactors about the first column 10 obrain

JAI = 1(-6) -0/lOJ + 3(+2) =0


Notice 1ha1 the third column is twice the tir~I column. We"ll see below rhat
!A: = 0 because of Lhi!..

Some properties of determinants that are useful to know are :

1. The value of a dctcrminam is unchanged if the row, arc made intn columns
in lhL· ,ame order: in 01her words. Cirsl row bccomt'" ti~t col11n111. seconJ row
becomes second \;Olurnn . and so on . For example.

1-P
- I
-I 51= 11
2 0 ~31= - 6
2 5 -1

2. If any two rows or <::()lumns arc the ~unc. the value of 1hc determinant is zero.
For example.
4 '}
-I 0
I J I

3. If any two rows or columns are interch anged. the sign of rhe dc!L"nnin:rnt is
<:hanged. For example.

3 3
-6 -6 - 511
,...,
I I 2

4. If every clement in a single ro1>.' or column is multiplied by a factor k. the value


of 1.hc dctcnninanl i~ multiplicJ hy k (Problem 13). For example.

I - 6I 81= -J I - 3I
2 41
2 = -'JO

5. If any row or column i~ wrinc11 as the sum or di ffcrcncc of 1wo or more term:.. the
dctl!m1inant can be wrincn a.<; the sum nr difference of rwo or more dctcnninants
403
::u.:cordi ng to ( Pruhlcm 14)

£11 I± a;, {I 12 aI a 12 , ,., I± a;\ I


0 11 ai:! 0 13
1/21 ± £1;1
l111 i_ 11 .~I
a22
llJ2
an
a 3:;
= a,,
I""
OJI
Ll'.!2
(I )2
t1 2
fl .33
I
(IJ J
0

a1 ~
22 ll '.! J
OJJ

For cx.:iniple,

3 JI = I - 22+14 3!
= I - 22
3
+I '
12 6 6 61 4

6. Thl' value of a dc1aminan1 is um:hangcd if one row or column is added or


subtracrcd 10 another. a.-. in

a J.J I I"1 1 + 11 t '2. ll a 12


0 2J = a 21 n a22
a_1 3 a31 + a:1'.! 11_,'.!

For example

-I
0
2

where we added column 2 10 column I. This procedure may be repeated 11 1irnes


to ob1ain

(11 31 1"11 ' llfJl2 au I


ll:!J = " 21 : llll].2 a23 ( 10)
11:13 0 _11 T IIOJ.2 (133

This result is easy to prove:

0 21
I""
IIClt'.!
J1<r22
t.11:::
0 12
,,,a 13 I = I.,,
0 21
01:::
ol:.. ""
(/ 2J I+ I.,,
" 2. II
"1 2
u22 a,
"" I
a, I nn:12 "J'.! 0 3 _1 (13 ) 11.~2 a 33 lt 2 032 a 33

(1 I2
= I•11
02, (/2'2. a;.u + o
""I
a\ l a:i~ (l3J

where we used Rule 5 to vmte the first line. The second (klcm1inan1 on 1hc righ1
sillc equnls zero because 1wo columns are 1hc same .

faample 4:
Show that lhe value of

3 -I 2
IAI = - 2 I
4 3

is uni.:hangcd if we a<lu 2 time~ row 2 to row 3.

C
404 Cf1,1ptcr 9 I linear AIRPhr,1 ,111(1 Ve tor Sp,tl ,,,

SOLUTION: FirstofaJJ. IAI = 3(-1)- (-1)(-7) + 2(-9) = -28. Now


3 -I 2
- 2 I =3(-1)-(-1)(-7)+2(-9)=-28
- 3 6 5

Simultaneous ljnear aJgebraic equations can be solved in tenns of de1crmi-


nan1s. For simplicity. we will consider only a pair of equations. bul the finaJ resulr
is easy to generalize. Consider Lhe two equations

a1,x + 012Y = h1 ( 11)


a 21 x + auy = h2
The determinant of the coefficients of x and y i.s

According to Rule 4,

Furthermore. according to Rule 6.

a, ix+ a12Y a,2 1= x IA(


l
n21.t + n22Y 0 22
( 12)

If we subs1itu1c Equal.ion 11 inro Equation 12, Lhen we have

Solving for x givc.s

I"I I a12

.,. -- "2
G22
(13)
I a, 2 1
011
021 ll22

Similarly, we gel

I a,,
a21
,, I
h2
I
y= ( 14)
Ia1,
D21
a,2
a,,,
I
Notice 1hat Equations 13 and 14 arc identical to Equations 2 and 3. The solution for
x and yin tenns of determinants is caJled Cramer's rule. Note lhal I.he detcrminan1

rn
9.1 Delermin.1111~ 405
in 1he numera1or is obtained by replacing the column in IAI that is associated wil.h
the unknown quantity and replacing it with Lhe column a.c.socia1ed with the right·
sides of Equations 11. We shall show after the nex1 Example Lhar this result is
readily extended 10 more than 1wo simultaneous equat-ions.

Example 5:
Use Cramer·s rul~ to solve the equations

x+y+:=2

2r - y ··-:.=I

x + 2}' - z = -3

SOLUTION: The extension of E{1uations 13 and 14 is

2
-I -I
-3 2 -I 9
r - = - =I
<)

2 -I -I
2 -I

Similarly.
2
2 I - 1
-3 -I -9
_\'= =-=-I
9
1
2 -I -I
2 -]
and
I I 2
2 -I I
2 -3 18
.. - =-=2
9
I
2 -I -I
2 -I

Before we finish this section. let's discuss Lhe expansion of a determinant


in 1c1ms of iLc; cofactors and how it can be used to derive Cramer·s rule for 11

simultaneous equations. Equation 7 says that


406 C l,,1pli~r (J ,1 I ine, r Algebra and V1•1 !or Sp,H 1·,

where Au is the cofactor of a;j• We derived Equation 7 from the expansion of


a 3 x 3 detenninant, bur it is a general resull in the sense that if IA : is an n x 11

determinant. then

or, in su111n1ation notalion.


J/

IAI = I:o,jAlj ( 15 I
j-=-1

Equation 15 can be derived dircctJy from Equation 8. bu1 the general proof is a
liu\e long. (See "Lipschutz·· in the References.)
Equation 15 represents an expansion of IAI about i1s first row. More generally,
we can expand about the ith row of IAI. and so we also have
f/

IAI = LaiJ!\iJ ( 16)


J=I

If we had expanded about the i th column. Equal ion 16 would rem.I


II

IAl=Lo1;;A1;,· ( 17)
k=I

Now if we replace the ith column by some olhcr column. say !.he jth c..:olumn, then
!Al= 0 because IAI now has two identical columns. The cofactors in Equation 17
don't change. however. so we have the result
II

L(JJ:j;\ki =0 ( 18)
k:I

Notice that we can combine Equal.ions 17 and 18 to read


II

L Ot)Aki = IA[ 8,) ( 19)


k=I

where E,ij, the Kronecker delw

1 i=j
8;_; =0 i #j
(20)

We will refer to Equation 19 several times. in later chaplcr1;.


We can use Equation 1910 derive Cramer's rule. Stan with Lhe set of equa1ions

(21)
9.1 De1ermin;in1s 407
Multiply lhe first equation by A 11 • lhe second by A21 , and so on. and then add 10
obtain

/I
"
L aj1Aj x 1 1 + I:a 12A;1X2 + · · · + L a1 ,,Aj 1x 11 =L Ai,hJ (22)
}=I j=I j=l j=I

According 10 Equation I 9. all the tem1s on 1hc left side cxccpl the first one vanish.
and so the left side is equal to IA! x 1. According 10 Equation 17. the righl side is
the original detenninant IA I but with lhe first column replaced by Ii 1• h 2 • •..• Ji w
Thus, we see lhat

h1 D]2 n1~1
h2 a22 a 2,1
!Alx 1 = (23)

Ii" {/112 n nn

Similar equations for x 2 rhrough x 11 can be obtained by multiplying Equation 21


by 01hcr cofactors (Problem 19).

9.1 Problems
2 I I 3
I. Evalua1c the Jc1cnninan1 IAI = -l 3 2 . Add column 2 10 c..:olumn I 10 gc1 2 3 2 and evalua1e i1.
2 0 2 0
4 3
Compare your result wi1h I.he value of IAI. Now add row 2 ro row 1 of IAI to get - I 3 2 and evaluate ir.
2 0
Compare your result with rhe v;ilue of IAI above.

2. lnterchunge columns 1 and 3 in IAI in Problem 1 and evaJuate the resulting determinant. Compare your resuh
with 1..hc value of IAI. Interchange rows I n11d 2 and do the same.
6
3. Evaluate 1he dc1crminan1 IAI = -2 4 -2 . Can yt)u dc1em1ine i1s value by inspection? Whal ahoul
-'.\
2 6
IAI = -4 4 -2 .,
2 -3
4. Starting wi1h IAI in Problem I, udd two limes the 1hird row lO Lhe second row and cvnluate I.he resuhing
de term.i nant.

5. Use Equation 5 to dcri\'C an expansion in cofac1ors about the third column.


I sin x cosx
6. Evaluate !Al = 0 COS.\' - sinx
0 - sin x - cosx
408 Chap! •r 9 / Lin • , Algebra and V lor Spaces

-" I
I x 0
7. Find the values of x 1hat s.a1h;fy the dctcrminanu1I equation.
0 X
0 () X

X 0 I
I X 0
8. Find the values of x that satisfy the dc1cnninantal equation. =0.
0 I .x
0 I x

co· 0 - sin 0 0
9. Show 1ha1 sin() cos {} 0 = I.
0 0
10. Evaluate (a) f I 32 45. (b) €32 1~5. and (c) E'5..1J21-

l I. Show tha1 Equation 8 yields Equation 5 for a 3 x 3 detcm,ina.nt.

12. Show tha1 it requires 2k - I steps to intcrchnngc row.~ r and r + J: in a determinant.


13. Use Equa1ion 8 to prove property 4.

14. Use Equation 8 to prove propeny 5.


IS. Solve the following set of equations using Cramer"s rule:

x+y=2
3x - 2,· =5
16. Solve the following set of equations using Cramer"s rule:

X + 2y + 3,: = -5
-x - 3.r + :: = -14
2x + y +.:=I
17. Use Cramcr's rule 10 ~olvc

X + 2y = )
2.r + 4y = I
Whal goes wrong here'? Why?

18. Verify Equ:nion 19 for the determinant in Equation 5 for i = 2 nnd j = I.


19. Derive an equation for x 2 starting with Equation 21 .
2 5
20. Use any CAS to evaluate 3 I 2
-2 I 0
I 0 3 -2
6 I -I J
21. Use any CAS to cvaluulc
2 0 I I
4 3 2 5
409
9.2 Gaussian Elimination
Although Cramer's rule provides a systematic. compact approach LO solving simul-
1aneous linear algebraic equalions. it is not a convenient compu1a1ional procedure
because of 1he necessity of evalua1ing numerous de1erminan1s. Nor docs Cramer's
rule apply if the number of equations does nol equal the number of unknowns.
Ln this section. we shaJl present an alternative method of solvLng simultaneous
equations tha1 is not only compura1ionally conven.icnt. but is not Limited 10 11 x n
systems. Before we present th.is method. however. we shal.l discu s ome general
ideas about systems of linear algebraic equations.
Let ·s start off aga.in with 1wo equations in two unknowns.

aI1XI + a11X1;::: '11


a21xI + a21x1;::: h2
Geometrically we have three po~sibilities: I. the graphs of the two straight lines
intersect and we have a unique solution: 2. the lines are parallel and we have no
solut..ion: and 3. the lines coincide und we have an infinite number of solutions
(Figure 9.1 ). An example of the first case is

2'" 1 + x 2 = 3
X1 - 3.r'.? = -2
with x 1 = I and x 1 = I as its unique solution. An example of the second case is
2xl + X'.? = J

2.r1 + -"2;::: 5
These 1wo lines are parallel and have no point in common. An example of the third
case is

These 1wo lines acrually coincide and 1he solution can be wriuen ac; x 2 = '.\ - b· 1•
where x 1 can take on any value.

(c)

Figure 9.1
=
l11e lhrtl' gl."Otnl'lric pc1~,ibili1ics of two linear L"qu;1ti1111, in two unknown~ .., 1 and x 2 . (a) TI1c: colored line (2. 1 x_ 3)
and lhc blad,: line (x I - Jx2 = -2> huvi: u unique poinl of in1cr-.cc:1ion. (b) Th.: L"uk1rud line (2x 1 + x 2 1) and 1.hc bl:ick
=
=
line (2.,: - .t: 5) arc parallel and have no poinl of inlc~c~tion. (c) TI1c 1wo linc, (2x 1 - ·'! = =
I) and (4.r 1 - 2., 2 2)
supcrirnp~c. and so rherc is an infinire number of solution~.

_1'1 I
410 C h.ip1 r 9 / Line-.:ir AlgebrJ and VtX1o r Spa es

The geometric interpretation for a 3 x 3 set of e4uations involves planes. If


the three planes intersect at one poinl. there is a unique solution (Figure 9.2a).
If the three planes intersect as shown in Figure 9.2b, !here is an infinite number
of solutions. And if the three planes have no common point of intersection. as in
Figure 9.2c. there is no solut..ion.
Let's now consider a general II x 11 .system:

a11X1 + a12X2 + · · · + ll1nXu = I, I


C121-t 1 + a 22 x 2 + · · · + a 2,,x, = 11 2 1
( I)

If all Lhe Ii j in Equations I arc equal to zero. the system of equations is called
honwgeneo11s. If at least one h j i= 0. the system is called 11onhomoge11em1s. We

.,.l[l ["Il
may re-expre~s Equations 1 as

XJ 0 12
x, h~

(°''
a,1 "22 (lln X2
AX= ~
= .- = H (2)

a~, 1 au2 ll:w _'(:n hn

if we agree that the rth cqu.ition in Equa1ions I is fo(llled by multiplying each


element of the rth row of A by the corresponding element of X, adding the resuhs,
x~ um.I then equnl ing the sum to I he r I h clement i11 H. For example. the second Ii nc
in Equations I is given by

(3)

(b)
Thus. we can wri1c Equa1ions I as AX= H. where A is called rhc wefficienl ma1rix.
X is called lhe column vector of unknowns, and H is called the constant vector of
the system. Note that the lefl side of Equation 3 can be vie\ved a~ the dot product
of the rlh row vector of A and the column vector X.
The quantjty A in Equation 2 is an II x II matrix. which is an array of elements
that obeys certain algebraic rule~ such as Equation 3. We shall discuss matrices
and their algebrJic rules in some detail in the next section. The important point
here is that a matrix is 1101 equal to a single number. However. we can associate a
determinant with a matrix and write

a11 al:! (Jin


x~
a21 an a2,,
det A= IAI = (4)

a,,1 {Jn!. a,,"


Figure 9.2 which is a single number.
171c 1hrce geometric possibiliticl-> of the Clearly, 1he cocllicicnl matrix A must have a lot 10 say abou1 the existence
graphs of three line;:ir algebraic equations:
and m.llure of Lhe solutions 10 Equa1iom I. Cramer's rule contains IAI in the
(a) aunique i;olution: (bl an infinite
number of solutions: and (c) no solutmn. denominators of the equation~ for the unknown:;. so [A I cannot equal zero if there
9.2 G,rn sian Elimination 411

exists a unique solulion to a nonhomogeneous sysrem. If IAI = 0, Lhen A is s.aid


10 be singular: if IAI f. 0. then A is said to be 1w11si11gular. In fac1. we have 1hc
following Lheorem. which we shall prove later:

The II x " .1_nft'IN AX = H has a u11iq11e solwio11 tf w1d 011/y ifA is nomingular.

= =
If H 0. tha1 is. if 1he system is homogeneous. then x 1 x 2 = • ••= x,. 0 =
(called the trivial sol111ion) is always a solution. But the above theorem says lhat a
solmion is unique if A is nonsingular. so if A is nonsingular. there is only a triviaJ
solu1ion 10 a homogeneous system. To have a nontrivial solution to an II x 11 set of
homogeneous equal.ions. the coefficient matrix must be singular.
We shall no\v spend lhe rest of 1his section actually finding solutions to systems
of linear cqua1ions. even if the coefficient matrix is no! square. Let's consider the
equations

2x 1 + x1 + 3x3 = 4
2x 1 -.. 2x 2 - x3 = I (5)
- 2.r 1 + 4x 2 + x 3 = I

The coefficienl matrix and lhe constant vector arc

and

We now form a new matrix, called 1he 011gme11ted matrix. by adjoining H 10 A so


1hat ii is the las1 column

~
3
AIH =( -2 -I (6)
-2 4

Clearly 1his matrix contains al/ 1hc information in Equations 5, and is just a succinct
expression of 1hem. Just as we may multiply any of the equations in Equations 5
by a non1..ero constant wilhoul jcop::irdizi.ng 1he solutions. we may multiply any
row of A]H wirhout altering ils con1cn1. Similarly. we may interchange any 1wo
rows of eirher Equations 5 or AIH and replace any row by the sum of thal row and
o consIant times another row. These three operations arc called l'leme111nry (row)
operations:

I. We may mulliply any row by a nonzcro constant.


2. We may interchange any pair of rows.
3. We may replace any row by 1he sum of Lhat row and a consuint 1imes another
row.

The key point is that these elementary operations produce an equivalent system.
1ha1 is. a sys1cm wi1h the same solution as the original system. Matrices that differ
by a set of elementary operations are said to be eq11ivole111.

al
412 Ch, pier 9 / Linear Algebra and V L r pa

We are now going to manipula1e AIH by elementary operations so that there


are zeros in the lower left positions of AIH. Add - I times row I lo row 2 and add
row 1 to row Jin Equation 5 10 get

(~
I 3
-3
s
-4
4 -D
Now add 5/3 time,<; row 2 lO row 3 10 get

(~ -3
I
-4
3
-3 4)
0 -8/3 0

Write om the corresponding system of equations:

2x 1 + x 2 + 3xl =4
-3x2 - 4x_1 = -3
-8x 3 /3 =0
and work your way from bottom to lop to find x 3 = 0. x 2 = I, and x 1 = 3/2.
This procedure is called GaussiaH elimination and the finaJ form of Al His said
10 be in eche/011/orm. The following Examples provide 1wo 01her applications of
Gaussian elimination.

Example 1:
Solve lhe equations

X1 -rX~ - XJ = 2
2x 1 - x 2 + Jx 3 = 5
3.r1 + 2x2 - 2:r_, = 5

SOLUTION: The augmented matrix is

I -]
-l 3
2 -2

Add -2 limes row I to row 2 and -3 Limes row I to row 3 10 ob1a.in

-1
- ) 5
- I

To avoid introducing frncLions, interch:mg<: row~ 2 an<l 3 and then add -3


times the new row 2 10 1he new row 3 to get
9.2 Gaussi.in Elimina1ion 413

-I
-1
0 2

The corresponding set of equation~ is

X1 +x2 - X:, =2
-x2+x 3 =-I
~-:i=4

Solving these equations from rop 10 bonom gives ., J =-= 2, x 2 = 3. and x 1 = I.

Example 2:
Solve the equations

SOL LI TIO N: The augmented matrix is

Add -1 limes row I


u
10 row
-I

2 and add
I

row
-I

I 10 row 3 10 obtain

u -2
2
I
0
0
-~)
-4
Now add row 2 10 row 3 to get

(i -2
0
I
0
0
-D
In lhi.s case. 1hc corresponding sc1 of equal ions is

X1 + X2 + X3 = -2
-2r 2 =4

The solutions are -'"J = mnbimny. x 2 - 2. and x 1= = - .,-


3• so the rolu1ion is
not unique. Note that lAI = 0 jn this ca.e. ~owe should 001 expect a unique
1

solu1ion.

al
414 ( h,1 p!f'r 9 / Linear Algebra and V e 1 tf 11 Sp,1 c-,

Example 3:
Solve the equations

2r 1 -x;i=-1

Jx1 + 2x1 = 4
4x~ + 3x.l = 6
SOLUTION: The augmented matrix is

(
2
3
0
2
-I
0 -1)4
0 4 3 6

Add -1/2 t.irncs row 1 to row 2 to obtain

2 0 -I
0 2 3/2
(
0 4 3

Now add -2 times row 2 lo row 3 to gel

(
2
0 2 3/2
0 -I
11/2
-1)
0 0 0 -10i2
This las1 line says Lhat -10/2 = 0. meaning 1ha1 there is no s.olut·ion to lhe
above equations. They are inconsisten1.

Up to now we have considered only " x equations. Suppose


II sys1cms of
we have a sys1cm with more equations than unknowns. (Such systems arc called
overdetennim>d.) For example. consider

X1+X2=4
3x 1 - 4x2 = 9 {7)
5x 1 - 2x~ = 17

The augmented ma1rix is

-4
-2
Multiplying row I by -3 and adding 10 row 2, and then multiplying row I by -5
and adding 10 row 3 gives

(
~
0
- 7
-7
- ~)
- 3
9.2 (.,1u ..,i.in ! /imination 415

Now multiply row 2 by - I and add to row 3:

-7
0

The corresponding algebraic equa1ions are

x 1 + x 2 =4
-7x,, = -3
and the solution is x 1 = 3/7 and x 1 = 25/7. The coefficient matrix of the final set
.
o f equations .1s ( I _; ) and is nonsingular: thus the solution is unique.
0

Example 4:
Solve thi:: equations

SOL U T JON: The augmented malrix i~

J -2

(
-6
-3
4
2 -D
Addrng 2 times row I to row 2. 1hen adding row I to row J. :md then
interchanging the resultant rows 2 and J gives

-2
0
0

The ~cond line here claims 1ha1 0 = 4. so there is no solu1ion.

Example 5:
Solve the cquation5

x 1 - 2x_ =3
2.\· 1 - 4x 2 = 6
- 3x 1 + 6x 2 = -9

al
41 6 Chapler 9 / Lill ;:i r Al!-(e l11,1 ,11til Vector Sp._10:~

SOLUTION: The augmented matrix is

-2
-4
6

Add -2 times row I to row 2 and 3 times row I 10 row 3 to get

The corresponding algebraic equarion~ arc .r 1 - 2x~ = 3. or .r = 2r 2 + 3.


1
Thus. lhere i:- nn infinite number of solutions.

In summary, if we have more equations than unknowns, then there are three
possibleourcomes: I. there is a unique solu1ion; 2. there is no solution: and 3. there
is an infinite number of solutions. In each case. Gaussian eliminarion leads us 10
the correct result.
If we have more unknowns than equations. as in

X1 + X2 + X3 + X4 = 0

X1 + 3.x'.! + 2X3 + 4.X.i = 0 (8)


2.r 1 +x 3 -x.i=O

then the system is called 1mderdetenni11ed. The augmented matrix is

I
3 2
0
_,4 2
0
I
0
3
0

where we have wriuen ~ to i.ndicacc Lha1 the firs! mauix is equi,•aleat to Lhc
~et.:ond: that is. ii can be manipulated into the second by elementary operations.
The corresponding algebraic equations arc

x 1 +x 1 +x 3 +x4 =0
2x2 + X3 + 3X4 =0
Solving for x 1 and x 2 in terms of x3 and x 4 • we have x 1 = (x 4 - x 3 )/2 and
x2 = -(x 3 + 3x 4 )/2. Thus. there is an infinite number of !iOlutions in this ca.-.e.

Example 6:
Solve the equations

rn
9.2 G,u1~,iM1 flirnir1,11ion 417

SOLUTION: The augmcn1ed matrix is

u
I 2
3 -I
5 3
-2
I

0 0-U
I
I -5
0 0
2
-4
0
-!)
-5

The last line corresponds 10 0 = -5. so there are no solutions.

There arc only two possibili1ies when Lherc are more unknowns than equations.
Either there is an in fi ni le number or soluiions. or 1hcrc are no solutions. Either way.
Gaussian elimination will give the correct answer.
Before we linish this section, we should point out 1har any CAS can be u.,;ed
10 solve simultaneous Ii near equations ( Problems 14 lhrough 16).

9.2 Problems

I. Solve the equn1ions 5. Solve the equal.ions

X + 2y - 6z =2
x + 4y + 4;:: = I

3x + IOy + 2;: = -I

2. Solve the cqunlions 6. So lve the equmions


1

1x + 5y +;: = 5
x + 4y + 2:: = I =5
4.r + IOy - : = I - 2x4 = -I
-.r 3 + 4xJ = IJ
3. Solve the equation~
·- x.1 ..:.. 3x4 = 1 I
x+y =I
,'( 7. Solve the equations

2x I- y + z = 0

4. Solve the equations

8. Solve the equations

x 1 - 2x 2 + 3x - =0
XJ

.r 1 - 4.\~ · · 2x 3 + 2x~ = 6 -X1 + 2.r3 + XJ = 0


4x 1 + x 2 - 3x3 + 3x-1 =- I 2x1 + X2 - 2XJ =0

al
418 h,.1 pll'r CJ / I.i n ar lg bra a nd V ·c ID r SpJ ,

9. Solve lht: cquarions

X1 - 2.r'.! + .r, --- .r.1 + 2x_; = - 7


.r:? + ,\3 + 2.r-1 - .\'"5 = 5
x1- .\ 1 - 2.r) + 2r 4 + 2.r 5 = - I

IO. For what value.,;, of ,l,. will the following equal.ions have a unique solution"?

X +_I"= is
-x + y = ).y
11. For what values of>... will the following equations have a unique solution?

x+y+:::=6
x +A)'+ i.::.= 2
12. For what values of i.. will 1he equations in rhe previous problem have an inlinire number of solution<'
13. For what values of A will tJ1c: following c.quaLions have a unique solurion?

5.r + 1._1· = 4
4x + 3y = 3
A..X - 6y =3
14. Use any CAS to solve rhe equali()ns in Problems 2 throug_h 4.

IS. Use any CAS I<> solve rhc equations in Problems 5 through 7.
16. Use any CAS to solve the cquatinns in Problem." 8 and 9.

9.3 Matrices

Up 10 now we have used matrices only as a reprcscnl,Hion of the cocfficicn1s


y r( cos(a+U). sin(a+B) )
in systems of linear nlg.ebraic equations. The utility of matrices far exceeds that
use . however, and in lhi, ,cc1ion we shall present some of 1.hc basic properLies of
matrices. Then. in Chapter 10, we will discuss a number of imponanl physical
applications of matrice.s .
l\fony physical opera1ions such .is magnification, rotation, and reflection
through a plane can be represcn1ed malhemalically by quanlilies called matrices.
Consider the lower of 1he 1wo vector.. shown in Figure 9 ..l The .r and _r compo-
Figure 9.3 nents of the vector are given by x 1 = r cos ex a.nd y 1 = r sin er. where r is rhe leng1 h
A pictorial n::pn.."SC:nlalion of the ror:ition of r 1• Now. lcr·s roiarc the vector counltrclodwisc through an angle&, so 1ha1
of the \'Cctor r1 through an :i.nglc: (I in a
countcrdockwi~ ditt!Clion . The result is x2 = r cos(a + 0) and y 2 = r sin(cr + II) (sec Figure 9 .3). Using trigonometric
the vecror r2 , formula.'-. we can write
x2 = r cos(a + fJ) = r co~ a cos H -- r sin a sin tJ
y 2 = r sin(a + 0) = r cos a sin 0 +r sin a cos 0
<J.J ,1aIri 419

or

.r:2 = x 1 cos O - y 1 sin fJ ( I)


Y2 =.r 1 sin {:) + y cos 0
1

We can display the set of coefficient. here in 1hc form

R = ( cos 0 - sin 13)


(1)
sin 0 cos f1

We have expressed R in the fonn of u motrix. which is an array or numbers (or


f unclion:- in this case) that ohey a cenain set of rules, called mlJtrix algrhm. Unlike
determinants. mar rices do 1101 have to be square arrays. The matrix R in Equation 2
corresponds to a rotation of a vector through an angle FJ.

y
Example 1:
Show th:il lhe m.ilricc,

A=(l0 -I0) .ind B= (


-1
0

correspond 10 rcflcc1ioni- or a vcc1or 1hrough the x axis and y axis. X

respcc1ivcly.

SOLUTION: If we reflect the vector r 1 .\' 1 I + .,· 1j throu~h the .r ,n:is,


we ob1ain =
the vector r :! = x ~ i + _,·! j x I i - y 1j ( Figure 9.--b). Thus. we
can wri1e (a)

_\"

The :;et of coefficients can be expressed .is

A=(I, n -10)
(h) X

so we see 1ha1 the matrix A c-om:: ·ponds to a reflection of a veL·tor through Figure 9.4
1he x ux i.s. Similarly, ti,r a rl!nccl' ion duough they axi:-:. A pi(: 1ori:il n:pri:.,c111 :i.1 inn of the re 11 cd inn
(if a vec.:lor tlm ,ugh 1111 Lhc x a>. i~ ;111<.I (b)
th y a.,i;i:-..

-1
so 1ha1 13 =( O 01) com:sponds to a reflection lhrough the y axis
(Figure 9.4b).

We shall see that matrices usually correspond 10 physical 1rnnsforma1ions.


420 Ch.1p1er 9 / Linear Al g >bra and V ror Spac ·

The enrrics in a matrix. A are called its marrix elemenls and arc denoted by a;j
where. as in the case of detenninants. i designates 1he row and j designates
the column. Two matrices. A and B. a.re equal if and only if they are of the
same dimension (that is, have the same number of rows and the same number
=
of columns), and if and onJy if a;J b,) for all i and j. [n other words. equal
malrices are identicaJ. Ma1Tices can be added or subtracted only if 1hcy have 1hc
same _number of rows and columns. in which case the elemems of the result.ant
matrix. are given by aij + bij. Thus. if

6
A=(-~ 0 and B= ( 2
-6 4

then

7
4
5)5
If we write

-6 12
A+A=2A= (
2 0

we see that scalar multiplication of a mar-rix means tha1 each elcmcnl is multiplied
by 1he scalar. Thus.

(3)

Example 2:
Using I.he matrices A and B above. form the matrix D = 3 A - 2 B.

SOLUTION:

D = 3 (-~ 0 :!
6
4)-~( -62 ;)
- 4

=(-9 3
18
0
12)
6
( -12 4 2
8 ~)=(-;; -8
16 10)
ll

One of the most important operations involving matrices is matrix multipli-


c..11ion. For simplici1y. we will discuss I.he multiplic:.ilion of square matdces firs!.
Consider some linear transformation of (x 1, y 1) into (x~. Y:)_):

X2=or,x1 +011."1
(4)
Y:! =: £1 21 X1 -1- "::2 YI
421

represented by the matrix equation

(5)

X3 = b11 X2 + b12 Y2 (6}


YJ = b21 X2 + h22 Y2
represented by the malrix equal.ion

BV, = (h11 (7)


- h21

Let the transfom1ation of (x 1• y 1) direc1ly into (_\·3 , y3 ) be given by

X3=c1,x,+c1~Y1
(8)
YJ = c21 X1 + C,22 .\-'1
represented by rhe marrix equation

(9)

Symbolically. we can wrire 1.ha1

or 1ha1

C=BA

Lct"s fint.l the relation between the elemcn1:c; of C and I.hose of A and B. Substitute
Equa1ions 4 in10 6 10 ob1ain

.x} = b, i(o, 1 x, + t112 Yi) ' h,2(n21 x, ' t1~2 .V1) ( I 0)


Y ,\ = b21(a11 .,· , + 12 Y1) + hn,(<121 X1 + n Yi)
0 0

or

XJ = (b, 1"11 + b12 a21)x, + U>11 £112 + b, ~ a22)Y1


Y:t = (b21"11 + h21 a21)x1 + (h21 a,2 + b22 02.1).\'1
Thus. we see that

( 11 l
422 hapter 9 / Li n ar Aln"br, • nd Ve 1or Sp,

This resuh may look complicated, but ii has a nice pallem which we will
illustrate two ways. Mathematically. 1he ijth element of C is given by the formula

C;j = L biJ: l1J.:j ( 12)


k

Nole 1ha1 the righ1 side of Equation 12 is the dot product of a row of B into a column
of A. For example,

c1, = Lb11:ak1 == h11°11 +b1~a21


I..

as in Equation 11. A more pic:torial way is Lo notice 1ha1 any element in C can be
obtained by mult.iplying dcmcnlS in any row in B by the corresponding clcmenis
in any column in A. adding 1hcm. and then placing them in C where 1.he row and
wlumn inters.eel. For example. c 11 is obtained by multiplying the elements of row I
of B with the elcment.s of column I of A. or by Lhe St:heme

:)

Example 3:
Find C = BA if

SOLUTION:

=
(-3 +2+I 0+0-1
0+8+1 -1+0+1)
-9+ 0 - I + -1 0- I
3- I 2 0-4+2 1+0+2
9
(
= -]~ 0 -]
-2 -n
rn
9.3 Matric • 423

Example 4:
The matrix R given by Equation 2 represenL-; a rolation through the angle €l.
Show thal R2 represents a rotation through an angle 20.

SOLUTION:

- ,in 0 ) ( cos 0 - inO)


cos8 sin 0 cos9

cos 2 f1 - sin~ 0 -2 sin n cos O )


=( 2 sin 0 cos 0 co~2 H - ~in 1 II

Using slalldard trigonometric identities, we ger

R2 = ( cos 211 - sin 20)


sin :.H cos20

which represents rotation through an angle 20.

Matrices do nor have to be s.quare 10 be multiplied together. but eiLher Equa-


tion 11 or the pictorial method illustrated above sugges1s that the number of col-
umns of B must be equal to the number of rows of A. When this is so. A and Bare
said to be cvmpatihle . We call a matrix having n rows and m columns an II x m ma-
trix. Thus. an II x m matrix can multiply into only an m x p matrix and produces
an II x p matrix .
For example. the product of a 2 x 3 matrix and a 3 x J matrix produces a 2 x 3
matrix :

-I
3
-4
17
10)
-4

An important aspect of matrix. multiplication is that BA docs nor usually


equal AB . For example, if

and
-1
0)
then

o)=(o -1)
-I 2 0

and

so AB -:fa BA. If i1 does happen 1ha1 AB == BA, then A and Bare said to comm111e.
424 C::h,1ptE•r ':I l LineJr Alg bra ,rnd \'!"Clot Sp, e~

Example S:
Do the malrices A and B commute if

A-(2
- 0 :) anu 8= (~ :)
SOLUTION:

AB= (~ ~)
and

BA-( 2 ~)
- 0

so they do no1 commute.

Anolher property of matrix multiplication thal differs from ordinary scalar


mult..iplication is thal the equation
AB=O
where O is the zero marrix or the r,11// matrix (all elements equal to zero) does 1101
imply thar A or B necessarily is a zero matrix. For example,

Although matrices correspond 10 transfonna1ions and should notbe con Fused

A=
[ 011
G)I
-

a:,1
a22

a,12
, .l
with determinants. we can associa1e a determinan1 wilh a square matrix. In foci, if

ll I.!

a111

fl,,n

then
QI I 0 12 a1,,
a;;n {1~2 a2,,
de1 A= [A l = ( 13)

af,J a,,2 a,m

If det A -:/= 0, then A is said to be 11011si11g11lar. Conversely, if det A = 0, I.hen A is


said 10 be si11g11/ar. A useful property of the determinants of malrices is Lhat

de1 AB := (det A)(det B) (14)

(See Problem 21 ). Of course. A and B mus I both be square matrices and of the
~ame dimension.
425

Example 6:
Given

A=O ! ] B=n OD and

Show rhat IABI = IAI IBI.


SOLUTION: First calcu\urc AB:

4 0
AB= I l
(2 0

Then !Al = -2. !BI = 3. and IABI = -6.

The de1em1inan1 of a matrix A is used in the consln.lction of the inverse of A. which


we define below.
A I ransforn,ation that leaves (x 1• y 1) unahered is ea.I led the identity trans-
fom1a1 ion. and the corresponding marrix. is called the idemity ma1rix or the unit
motri..x. All the elements of the identity matrix are equal to zero. except those along
I.he main diagonal. which equal one:

I=[~ p ~1
0 0 0 I.

A unit ma1rix is net:l!!:,,ari]y a square matrix. The elements or I arc 811 , the Kro-
necker de !ta, which c4uaJs one when i = j and zero when i -j:. j. The unit matrix
has the propeny lhal
IA=A 1 {15)

The unit matrix is an example of a diugona/ morrix. The only non zero elements
of a diagonal mutrix arc nlong i1s main dingonnl. Generally. 1hc clc111cn1, on the
mnin diagonal of a matrix arc called dia~rma/ elt.·m(~nrs and 1hc others arc called
off-diagonal eleme11t.\·. Thus, we can say that all the off-diagonal elements of :'I
diagonal matrix are zero. Diagonal matrices are necessarily square matrice~. Also.
any 1wo 11 x II diagonal ma1rices commute with each 01her.
If BA =i\ B = I. then B is s..iil.l 10 be the im·erse of A. and is denoted by A - I.
Thus. A- 1 h.:u. Lhc property Lhat
AA - I = A - IA = I ( 16)

If A rcprc~nrs sonie tran:-.forma1ion, then _,,-J


undoes that Lransformalion and
restores the original state. The foci 1ha1 AA 1 = /\ · 1A implies that A must be

C gl
426 Ch,tph·r CJ ,' I inear Al g bra . nd V lor Space

a square matrix. It should be clear on physical ground,; Lha1 Lhe inver!.e of R in


Equalion 2 is a rotation 1.hrough 1he angle -0. Thus. we write

sin
cos 0
0) ( 17)

which i~ ob1ained from R by replacing 0 by -0. lt"s easy lo show lhat R(0)R- 1(0) =
R- 1W)R(0) = I.
We found lhe inverse of R(t)) in Equation 2 by a physical argument. but how
do we tind the inverse of a (square) matrix in general? II tum..:. out Lhat we essenti-
ally derived the formula for the inverse of A in Seel ion I. Equation 19 of thaI sec-
tion is

( 18)

The quantities A.1; 1 are thecofactorsofthea1.:,- of A. and Ihc right side of Equation 18
are the elements of a unit matrix.
Let's first define a matrix of wfacrors of A by

( 19)

Now we dcfinl! the transpose AT of a matrix A to be Ihe matrix Iha! is obtained


by in1erchanging the rows and columns of A. In terms of the matrix elements of a
general ma1rix (al}). we have a~ = a Ji" For example. if

-l
-2)
-1
2 -3

Notice that we can also fonn AT from A by simply flipping A about its main
diagonal.
We can now write lhe term in parentheses in Equation 18 as the i kth element
of AJ0 ,/IAI. so that Equation 18 becomes, in matrix notation,
AT
-..!::!iA= I (20)
IAI
where I i!- a unit mat:rix. Thus. we see Lhat if det A f. 0, then I.he inverse of A is
given hy
Al
A-l=~ (21)
IAI

Some authors call AJ°0 r tbe adjoint of A, written as adj (A). One clear implication of
Equation 21 is Iha1 A must be nonsingular. Singular matrices do no1 have. inverses.
9.3 M,11ri1 L'~ 427

Equation 21 may look awkward lo use, but ii's pretty Sliaigh1fonvard. Le.Cs
find 1hc inverse of

-I
I
2

The determinant of A is equal 10 -4 and Lhe matrix of cofactors is

Using Equation 21. we have

0
0
-4

It's readily verified that A - IA= AA- 1 = I.

hample 7:
Find the inverse of

0
A=(-~ -2
1
D
SOLUTION: del A= 16 and

1
Awr= (-;
-I
2
-7 D
and so

A-'=_!_C 16
-2
1 -1),,
-7
2 4

An example of a ma1.rix 1hat has no inverse. which occurs in a number of


physical appLica1.ions. is a matrix 1ha1 corresponds 10 a projccrion of a vector onto
a coordinate axjs, For example. the matrix.

C gl
428

y
projects the vector r = x i + y j onto 1hc x axis: th,.u is.

Mathemmically, P has no inverse because IPI = 0. Physically. P has no inverse


becau,1! any vector r with an x componem x 0 will yield Lhe same result, as you
X
can ,cc in figure 9.5.
Figure 9.5 Finally. we mention thal any CAS can readily find inverses of matrices (Prob-
An illustration of why a murrix lems 23 and 24).
CD/TC:1'ponding to a projeclion ,d oc, no1
ha\·c ru, im cr~. Both \£.~.: ton; have the
same projection onto the :c a..,,is.

9.3 Problems

I. Given the two matrices A

0=6B-A.
=( -i ~ -~) and B = (-~ ~ n. forn1 the matrices C = 2A - 38 and

2. Given the three mwiccs A=~ ( ~ ~). 8 = ~ ( ~ -~). and C = i (~ _~).show tha1
A2 + 8 + c 2 = ¾t, where Ji, :.i unil llllllrix. Ali-o show !hilt

AB- BA= iC
BC - CB= iA
CA - AC =iB

~). show Lhat


-I

AB - BA= iC
BC-CB= iA
CA - A = iB

where I is. a unit matrix.

5. A three-dimensional rotation of a vector about the::: a.xis c:in be represenIed by the malrix

Cf!'< , - siin tJ O) ( COS fJ $in 8


R= ~in O i:os O . Show 1haI dct R = IRI = I and 1h~1 R- 1 = R - ) = - sin 0 co. 0
(
0 0 I 0 0
6. Show1har(a) (AT)T = A:(b) (A+Blr =Ar +BT:(c) (etA )T = ct AT ;(d) (ABJT =BTAT,
9.J Matrices 429

7. Given the matrices C, =( ~


show r.hat a..,C-y. = a;. 1
• C3av = u~. a:,'a~ = C3 • and C.1a:' = a,.,. Calcula1c t..hc determinant associated wir.h each
malrix.
8. If AT= A- 1. then A is said IO be onltogonal. Which of the mnlrices in Problem 7 are onhogonal?
9. Find the matrix of cofoctors. Arne• and the adjoint. of A. adj (A). for

(a)
u
10. Verifyt.hm(AB/=BTATifA=(~
0l 2)I
0 1
(h) (: i fl
-~):mdB=(-~ ~)-

11. Prove that A - 1 is unique.

U. Let A= ( ; ~ )- (u) Find a nonzcro matrix 8 such that AB = 0. Dol.'~ BA:;;::: O? (b) Can you find B such tha1

AB = 0 1f A = (: ~ ) ?
J3. Provc1.hatta) (A- 1)- 1 =A;(b) (A1 ) - 1 = (A- 1)T_
14. Prove 1.hat dct (A- 1) = (der /\)-J. Hinr: Use I.he relation der AB = (dct A)(der B).
JS. Prove that (AB)- 1 = s- 1A- 1•
16. Use A= (; ~) and B = ( ! -~) to verify the relations in Problems 13 1hrough 15.

~)
-1 0
17. Us.eA= ~ 2 and B = ( -: 2 - ~) lo verify 1he reia,;ons in Problems IJ U,rough 15.
( ()

~
-2
~
I' ( I
18. Find the inverse nl (a) ( I
: )- (b)
0 2
19. Soll..-e the equations

.\" + y;:,J

by wri ring !hem as AX = H and then X = A- I H.


20. Show 1hat 1wo II x II diagonal ma1ricc.s commute. Docs an II x II diagonaJ matrix nece,,arily commute with
any II x II matrix?

21. ll1c general proof that dct (AB) =


(det A)(dc1 B) is fairly long and so we shall not prove it here. Ntverthele.~s.
verify that ii is true for 2 x 2 matrices.

22. A matrix 1hat satislii:s the relation A1 = A is called idempote.111. Show that if A has an inverse. 1he-n it must
be the identil) matrix. Arl,!lll' 1ha1 a projection matrix mus! be idcmpl)tcn1. Docs a projection matrix have an
inverse?

C gl
430 Ch,1p1er 9 Linear Al 1('hr,1 ,11111 V,·elof Spaces

~
5
23. U::-c any CAS 10 find !he inverse of (
-2

24. Use any CAS to find the ;nve,se of ( ; ;

9.4 Rank of a Matrix

In Section 2. we used Gaussian elimination to solve sets of linear algebraic cqua•


tions and saw thal we could have a unique ,olution. an infinily of solutions. or no
~olutjons. Gaussian elimjnation leads directly 10 the correct result in each case. but
it would be nice to have a general theory 1ha1 1ells us beforehand what to expec1
about 1hc ~olutions. Surely. the narure of the solutions depends upon some prop-
erty of I.he coefficient matrix and/or the augmented matrix since they describe 1he
sy.c;tem of equations completely. This property is the rank of a matrix.. which is the
subject of this section.
There arc several equivalen1 definitions of rank. One definition of rank is
expressed in terms of square subma1rices of A. A square subnwrri.'r of A is any
square ma1rix ob1ai ned from A by dele1ing a ccrtai n number of rows and columns.
If A happens 10 be square. then A is a submaLrix of itself. obtained by deleting no
rows and no columns. For example. consider the matri.'<

The po1>sible square submatrices of A are 1he 2 x 2 matrices

( 3I 2)
I (3I 43)
and the I x I matrices (I). (2), (] ). and (4 ).
The rank. r(A). of A is rhe order of the largest square submatrix of A whose
dclenninant is no1 equal to zero. The rank of the matrix A. :..ibove. is 2. The rank or

2
0

• also '...
1s "> because the subma1r1x
• ( I 2).
1 0
.
1s nonsmgular. even though 1he other 1wo
2 x 2 subma1riccs arc singular.
'J.4 R,111k of ,l •V\al,ix 431

Example 1:
Determine the rank of

SOLUTION: The largest thal r(A) can be is 3 since the largt·,1 possible
square submatrix or A is 3 x 3. and there arc four 3 x 3 square subma1rices.
The detenninant of the 3 x 3 submatri.x obtained by striking out !he fourth
column is zero. but 1he determinants of 1he other rim~ 3 x 3 submatriccs arc
not equal to zero. Therefore. r{A) = 3.

Another, perhaps more convenient bul nevertheless cquivalcnl. definition of


rank is 1hc number of nonzero rows in the matrix after it has been rransfomicd
into echelon fomi by elementary row operc11ions. Since we are going to base the
den ntion of rank on the echelon form of a matrix, we shou Id give a fonnal dcfi nirjon
of what we mean by echelon fonn. Wi: ~ay that a matrix. is in echelon if

I. All rows consis1ing of all zeros appear al the bollom.


2. If lhe first nonzero clement of a row appears in column c-. then all I.he clernent.s
in column c in lower rows arc zero.
3. The first nonzero dtmcnl of any nonzcro row :ippears to the right of the lirsl
nonzcro clement in any higher row.

All the linal versions of the augmented mat.rices in SeL:tion 2 were in echelon fonn.
Let's determine 1be rank or the matrix in Example I by this me1hod. In obvious
nolalion.

6
0
9/2
0
5 ) ---
-13/2 (?
0 9/2
0

7 9/2 -9/2 0 0 0

There arc three non zero rows. so r( A) = 3.

Example 2:
De1cmiinc 1he rank of

J 2 -4
A=~ 3 0 -I
( -6 3 -8

s O Lu TIO N: Rearrange 1hc rows so that the left-most column n:aJ~ 1.2.3
(this avoids imroducing fractions). Now add -2 times row I to row 2 and

C gl
432

-3 times row I to row 3 10 obtuin

-6 3 -8
15 -6 15
20 -8 20

The lasr two rows arc a con51un1 muhiplc of each other. ~o if we multiply
row 2 by -4/3 and add the resulL to row 3. we get

-6
15
0
-6
J

0
-8
15
0
-I~)0

Ther~ arc two nonzero rows. s.o the mnk of A is 2.

There are ten 3 x 3 submat1ices of the m<1trix in Example 2 (can you show
Lhis?). so you would have to cvaluatl! ten 3 x 3 dctenni nanl~ just to find out the rank
of A is not equal to 3. This result sugge~ts that the row echelon method is usually
much eas icr to .i ppl y lh an our fi rs1 de h n iti on of the rank of a ma 1.ri x. Nevcn.he less.
lhe deli nil ion of rank in 1cnns of the largest nonsi ngular square submarrix of A is
a standard definition.
There is another dclinilion of rank that we will present here for completeness.
We say that the 111 non.zero vectors v 1• v 2 • .... v,,, arc linearly dependnu if there

exist con$tants c 1• c 2 • ...• c 111 not all zero such thar

(I)

Linear tlepcndence means Lhal on.c of the vectors can be wrillen as a linear combi-
mil ion of the olhcr .... If Equal ion I b -.ali .... licd only if al I the cj = 0. 1hen Lhc vectors
arc said 10 be linearly i1ulep,•1uh•n1. In Lhrcc dimensions. the un.it vectors i. j. and k
are linearly independent. bul any other vector in three dimensions can he written
as a linear combination of i. j. and k (v = v,f i + y j + v~ k).
We now define the rank of a matrix in terms of linear independence of vectors.
Recall from Chapter 5 that a vector can be represented by an ordered 11-tuple of
numbers. (v 1• u> .... 1Jn). where we can think of I.he L'J as rhe components of v
in some coordinate system. We think of the rows of Lhe matrix A as vectors. If A is
n x m. then we haven m -dimensional vectors consti luli ng the rO\,..!-. of A. The rank
of A is the maximum number of linearly intlependenl vectors Lh,H can be formed
from 1hesc row vectors. In prac1.ice, this fundamental dclinit.ion of rank isn't that
useful because it often isn't easy 10 use Equa1ion I to de1crminc if a set of vectors is
linearly indepi..!ndcn1 or not. In fact, the ea..:;icst way tn determine if a !>Cl of vectors
is linearly independent or linearly dependent is to use the echelon mauix. procedure
to dc1crminc the rank of A. and hence the number of linearly independent vectors.
Nevenhcle.ss. this definition is useful in 1heorelical discussions. Of course. the
three definitions of rank that we have presented arc cquivnlcnt.
9.4 R;ink oi .i Ma1rix 433

To see more clearly the relation beLween rank and the number of linearly
independent rows of a mauix, consider the following matrix in echelon form:

-I
1 I
0 3
0 0

Row l cannot possibly be a linear combina1ion of rows 2 and 3 because they have
zeros in their first entries and so any linear combina1ion of rows 2 and 3 must have
a zero in its lirst position. Similarly, row 2 cannot be a multiple or row 3 because it
has a zero in its first and second entries. Working from Row 3 upwards now, notice
1ha1 no row can be a Iinea.r combinal ion of higher rows because of Lhe posi lions of
Lhe leading zeros in each row. Thus. there are three linearly independent vectors
in this rank 3 matrix.

Example 3:
Determine whether the three vectors (3. 2. I. -4. I). (2. 3. 0. -1. -1). and
( I. -6. 3. -8. 7) are linearly independent.

SOLUTION:

-6 3
3 0
2
-8
-1
-4
-:)
(We have arranged Lhe rows in this way to avoid fractions.) Thi!- is the
same matrix a:- in Example 2. where we determined that lhc r.ink of A is 2.
Therefore, only 1wo of the three vectors are linearly independent.

We. now present a theorem on the existence of solutions to a set of m linear


algebraic equations in II unknowns in 1erms of rank.

u•r A b,· the m x II co,_lfi,:i,•nr /Jlllfrix of rlu· .r,•r of m lint·ar al):;t-hrai,·


eq1wrim1.1 AX = H mu/ lt·r Al H be them x (11 + I) a11x111t'nfed morrix uf rhe
system. If

/. r(A) = r(AtH) = 11. 11,ne i.,· l1 llllll/11'-' solwion.


2. r(A) = r(A:H) < n, rhere are i11fi11irely 111n11y sol/I/ions, apressibh• in
terms of11 - r(A) parameters.
3. r(A) < r(AIH). rltere are no ~•o/11tions.

This theorem summarizes aJI the possible ca~es for all linear systems. ho-
mogeneow; or nonhomogeneous. Let·~ go b~ck and examine each of the cases in

C gl
434 Ch,Jp!t)r fJ / Li!ll"cll ,\I :.-hr,1 .11 ,d Ve'( lf>f pa

Section 2 in terms of the ranks of A and AJH. For Equa1ions 5,

~
3
A=(~ -2 -~)-2 4 I
and AIH= (
-2
-2
4
-1

Both r(A) and r(AIH) = 3. so Equations 5 have a unique solution.


For Example I.

-1) ~
-I
A=
u=
In this case. r(A)
-I
2 -2

r(AIH)
3 and AIH =(
J

= 3. and so lhe solution is unique.


-1
2
3
-2 D
For Example 2.

A=
(_: -1
_:) and AIH = (_: -I
I -I
-~)
-2

In this case. r(A) = r(A[H) =


2 < 3. and so 1hcre are infinitely many solutions.
expressible in tem1s of one paramc1cr.
For Equalion 7.

I
A=(; ~) and -4
5 -2 -2

= =
rn 1his case. r(A) r(AIH) 2. and so the sol mi on is unique. The rest of the cases
are left 10 1he Prohlems.
Before leaving this sec1ion, we shall present a 1heorern regarding homoge-
nCOI.J.') !>Cls of linear algebrnic equations. Even though the above theorem applies to
both homogeneous and nonhomoge11cou.s systems. homogeneous sysrcms occur
quite often in physical problem~. ~o we·11 present the implicat[ons of the above
general Lheorem 10 homogencou~ sys1cms.

T/11: m x 11 linmogeneous .~_ntem AX == 0 always Jun· l1 JriFial sol111io11.


X1 = x 2 == · · · =Xn =0. /It i.,· alway.1· rnnsisre111 becm,se r(A) = r(A]H)./
If r(A) = n. then Lhe trivial ..;olution is the only solution. If r(A) < 11. 1hcn the
genernl theorem assures 1hc c:.\is1cnce of non-trivial ~olurions. In panicular. these
non-trivial ~olutions constitute an (11 - r( A) J-parnmcter family of solutions.
An II x II homogeneous system ha~ a property rhat we shall emphasize here
by selling it off:

The II x n lw11wge11em1s system of lim:or algebraic eq11mio11.s AX = 0 has a


11mHri1·i11/ so/111io11 if and 011/y ~f de1 A= 0.
9.-l R.111~ o( .i .¼arrix 435

This lasL theorem follows directly from everything above. bur it is important enough
to emphasize.

Example 4:
De1em1ine 1he values of., sud11..hn1 the equa1ions

('2 + .l'/'3 + <"4 = 0


c, -1- XL".j =0
have non-1rivial solutions for 1he cr (This sel of equal ions occurs in a
quan1um-mechanical cakula1ion fur a buiadiene molecule.)

SOLUTION: To a...;sure a non-trivial solu1ion. the determinant of the


coefficient ma1ri.'I( mus1 vanish.

X O 0
] X O =O
0 ] X

0 0 I x

Expanding in cofactors about the first row gives

x(x.l-2r)-(x 2 -1)=0

or .r-1 - 3r + I= 0. or x 1 = (3 ' ~)/2. or x = ± 1.61804 and ±0.61804.

9.4 Prubll'ms
Use !he cuncepr of rm,k ro i11.ves1igare the 11arure ,f the sul11tim1s in Pmbfrms 1 thmugh 12.

I. Example 4 of Section 2. 7. Problem 4 of Section 2.


2. Equations 8 of Section 2. 8. Problem 5 of Section 2.
3. Example 5 of Section 2. 9. Problem 6 of Section 2.
4. Problem I of Section 2. 10. PTOhlem 7 of Section 2.
5. Problem 2 of Section 2. 1l. Problem 8 of Section 2.
6. Problem 3 of Section 2. 12. Problem 9 of Section 2.

gl
436 C!uph•r q / Linear Alt:dlr.i ,md Ve·, tor Spaces

13. For what values of x will 1he following equa1ions have non-trivial solulioni;?

X CI + C~ + C"-I = 0
c 1 + xc~ + c3 =0
r2 + xr.3 + c-1 = 0

Ct + CJ + X C4 = 0
14. De1crmine the values of x for which the following equations will have a non-trivial s.olution:

C1 + XC_l + C-1 = 0
CJ + C~ + C.3 + XC-1 = 0

9.5 Vector Spaces

Although we didn't poi.nt it out exp]jcitly in the previous section, matrices obey
the following algebraic rules:

1.A+B = B+A
2. A+ (B + C) = (A+ B) +C
3. a(A + B) = aA + aB
4 . (a+b)A=aA+bA
5. a(bA) = (ab)A
where a and bare scalars. II so happens that many at.her mathcmr1tical quantities
obey Lhe same set of rules. For example. complex numbers. vecrors. and functions
obey these rules. There is a mathematical fonnalism that treats all these quantities
in an ab. tract unified mGJilner and allows us lo see lhe similarities between them.
We dc,fine n wxtor space V to be a set of objects ( which we· II call vecto~) for
which addition and murltiplication by a ~caJar. eilher real or complex. are defined
and satisfy 1he following requirements:

I. Two vectors. x and y. may be added 10 give another vector, x + y. which is also
in V . (We say Lhat 1he ~et is closed under addi1 ion.)
2. Addi1ion is commutative; in other words, x + y =y + x.
3. Addition is associative; in other words. (x + y) + z = x + (y + z) = x + y + z.
4. There exists in \I a unjque z.nv 1'ecror, 0, such that x + 0 = 0 + x = x for .my
x in V .
5. For every x in V. there is an additive inverse -x such that x + (-x) = 0.
6. Any vector x may be multipl.icd b) a -.1.:alar c such Lhal ex is in V. In other
words, the se1 is closed under s<.:alnr mullipl.ica1ion.

_1'1 I
9.5 V, to r Spa "' 437

7. Scalar mult iptieation is as~ociativc: in other word~. for any Iwo numbers o :ind
b. n(bx) = (ab)x.
8. Scalar muJ1iplica1ion is distriburive over addition; in other words.

u(x + y) =11x +uy and (a+ b)x = ax + bx.


9. For the uni1 scalar. Ix= x for all x in \I.

Properties I through 9 are called rhe axioms of a 1•ecror space. If the scalars are
real numbers. \I is called a real vec10r space; if they are complex, V is called a
complex vector space.
The geometric veciors that we discus.-,ed in Chapter 5 satisfy all the above
properties of a vector spa<.:e. and fom1 what is called a Euclidean vector space,
in pankular. The element ... or members of a vector space. howe\·er. need not be
geometric vec1ors. For example. the set of all 11th order polynomial-.. Pn. with real
or complex cocfficienrs. forms a vcc1or space. as long as we consider mth order
polynomials (m < n) to be 11th order polynomials with cenain zero coefficients.
Another vector space consists or all tHuples of real numbers. where 1he sum
of (u 1• 11 2 , •..• 11,,) and (v 1• v 2 •...• r,,) is defined as (u 1 + v1• 11 2 + v2 , .•.•
u 11 + u11 ) and lhe product of an 11-tuple by a :-.ea.Jar is ddi ned as a (u 1• 11 2 , .•• , u ,1 ) =
(a11 1 , 011 2 • •..• 011 11 ) (Problem I). (We"ll us.e ordered n-tuples fairly orten 10
ii lustra1e the properties of vec1or spaces. so we' II designate the space of all ordered
n-tuples of real numbers by R" and that of complex numbers by C11 .)

hample 1:
Show thul the set of functions whose Ii rst derivarives are continuous in la, b I
and sati.'-fY

df + 2J(x) =0
dx
form a vector sp-ace.

SOLUTION; To show that 1he scr is clo~ under addition (I). let f and
~ be two element~ of V (in other words. both f and g satisfy the above
equationi. Then.

d df dg
-<f + ,::J + 2<J +id= - + - + 2J + 2g
dx dx dx

__ (df
dx 2J ) + ( :::
I + 2g ) = 0 + O=0
To show that the :-et is closed under ~calar multiplication (6). consider

d
-(aj)+2(af)=n
dx
(df
.....:...+2J) =0
dx

The other axioms arc ,ati,fied by any continuou!:- f unc1ion.

C gl
438 C'h.1pr1 ·r 9 / Linear Al ebra and V1•1 rm Sp,H ,-,.,

Example I suggesL" th::11 the set of solutions lo any Ii near homogeneous differential
equaLion forms a vector space (Chapter 11 ).
It often happens that a subset of the vectors in V forms a vector space with
rcspec1 to the same addition and multiplication operations as V. lo such a case.
Ihe set of vectors is said 10 form a subspace of V. A simple gcomc11ic example
of a subspace is the xy-planc of a three-dimensional Euclidean ·space. The ·r t of
all vccrors 1hat lie in the xy-planc forms a vector space. To see another example
of a subspace. consider lhc veclor space R" made up of ordered 11-tuplcs of real
numbers (u 1• u 2•.... u"). The set of 11-tuplcs (o. a . .... a) fom1s a subspace of
R11 (Problem 5).
An imponam concept associated with vc-ctor spaces is the linear independence
and linear dependence of vecIors. We 1ouchcd upon this idea in tJ1c previous
sec1ion, but we shall s1udy it more 1horoughly here. Lc:t {v.i; j = I, 2 ..... 11 I
be a set of nonzcro vectors from a vecIor space V. We say that 1hc ::.e1 of veclors
is Ji11eurly independent if the only way that
II

_L Cj\'j = 0
j == I

is for each and every c i = 0. If the vectors are not independent. Lhen rhey are
linearly depe11de111. There are several convenient ways to determine if a set of
vectors is independent or not Let"s test the 1hree vectors (I. 0. 0). (I. - I. I). and
( I, 2, -1) for linear independence. Is there a set of c 1. not all zero, such Lhal

II

L c)vj = (c: 1• 0. 0) + (c 2• -c1. c 2 ) + (c.1. 2c 3, -c 3) = (0. 0. 0)


j=I

or is Lhere a non1rivial solution to the equations

-c2 + 2c_, = 0 (I)

C2 - C:3 =0
The dctcnni.narH of the matrix of coefficicnLS is nonzero, so the only solution is
Lhc solution c 1 = c1 = c-' = 0, an<l so the three \'CCtors are lineurly independent.
We could also have arranged the three vectors a." the rows of a ma1rix and then
transformed ii into echelon fonn:

0
-I
0

which shows that the three vec1ors are linearly independent.


lt's easy to sec from the echelon fonn approach thal a set of m 11-tuples n111s1
be linearly dependent if m > n because the bottom m - n rows will always be
439
zeros. For example. consider the three vectors (I. I). (I, - I). and ( -1. I). Then,

and so only two of the vectors are linearly independent. We can also see that lhc
rank of A can equal 2 at the most

Example 2:
How many linearl.y independent vectors are there in 1ne: set l(l, I, 0. !),.
(-1. -1. 0. -1). (I. 0. I. l). (-1. O. -1. -1))?

SOLUTION:

(-i ~)
0 I 0 0

-I
-I
0
0 -I
0
-: )-( i
-] 0
0
-I
-1
0
i)-(~ -I
0
0
I
0
0

where we pl::iced the two zero rows at 1he bonom. Thus. I.here arc 1wo linearly
independent vectors in the se1.

If v 1, v2 • .. . , vn arc vectors in a vector space V and if any other vector u in


V can be expressed as n linear combination of v 1• v 2 • ...• v,, so that

U = C I \' I + C2 V2 + · · · + C 11 V 11

where tile c; arc consrants. then we say that v 1, ,· 2 , ...• vII spa11 V. For example.
Lhc three unit vectors i. j. and k span the three-dimensional space RJ. as does any
three non-coplanar (linearly independent) vectors. If lhe vectors v 1• v2 • ... , v11 in
a vector space V are linearly independent and span \/. then the set v 1• v2 . ...• vn
is called a bm:is or basis set for V . The unit vectors i, j, and k. or any three non-
coplanar vectors. form a basi~ in R-' . The number of vectors in a ba-,is i5 defined to
be Lhc di111L'l/.lio11 of the Vl:L'lor ~pace.. The dimension of a vcc1or space V is cqua.l
to the maximum number of linearly independent vectors in V.
Suppose that Iv J; j = I. 2, .... 11 l is a set of l.inc.arly iadcpcndcnl vectors
in an 11-dimcnsional vector space V. If 1he set composed of the v I and any other
(non-zero) vector u in V is linearly dependent., then the set {vf j =
I. 2 .... , n)
is said to be nuuimal. Thus. the maximum number of linearly independent vectors
in V is n. Because u and the vJ ::ire linearly dependent. we can write

where the c's are not all zero. [f c11 = 0, then the set v 1• ,..2 ••..• ,,,, is linearly
440 Ch,1p1<·1 9 / Linear Al•~cbr,i ,ind Vector Sp.1 lc!i.

dependent. conlrary to our assenion. Therefore. c,, f. 0 and we can wri1c

C1 c, Cn
U = -- "1 - ....:. \'2 - ' . . - - vn
Cu C11 Cu

We sec. then. thal u can oc


written as a linear combination of the set of linearly
independent vectors. and so the v J constitute a ba.,:;is.

Example 3:
Show that I.be vectors (I, 0. 0). (0. I. OJ. n.nd (0, 0. I) form a basis in R3 .
What familiar vectors do they correspond to?

SOLUTION: They form u basis in R 3 bccaui:.c any vector u = (x. y, .:) in


R'J. can be written as

u =x(I. 0. 0) + y(O. I. 0) + .:(0. 0. I)

TI1esc vectors correspond to Lhe unit vectors i, J, and k.

Example 4:
Show tha1 l.he vec1or spac.:e \I of ordered 11-tuples i~ an 11-c.lim~n~ional spac.:e.

SOLUTION: Then vectors (I. 0, . . . . 0). (0. I, .... 0) •...• (0, 0 .... , I)
constitute a linearly indcpcndcnL set of vectors in V. Funhcrmorc. I.hey span
V because any other vector in V can be wriucn as a linear combination of
these vectors according 10

u = (u 1• "~· ..•• 11 = 111(1. 0 . ... . 0) + u2(0.


11 ) 1..... 0) + ···
+ 11 11
(0, 0 ..... I)

Thus. the II vectors cons1i1u1e a ba~i:s and 1hc dimension of V is 11.

Suppose that lv1 ; j = I. 2 ..... 11\ is a basis of V. Then any vector u of V


can be wrinen as a Ii near corn bi nation of Lhe v/
,,
u= I: ,.j "j
j= t

We say that u 1 is the jlh coordinate of u with respcc.1 to Lhe given basis set.
Problem 19 ask.s you to show that the coordimHcs of u with respect to a given
basis in a given bnsis arc unique.
When \VC study differential equa1ions in Chapter 11. wc·n sec that 1he set of
s.olutions to an mh order linear homogeneous differential equal-ion forms a vector

m
9.5 Ve tor Spa 441

space. Consequently. it's not unusual 10 inquire abou1 the linear independence of
a se1 offunctions over some interval /. Ln other words. we ask if the only way Lhal

J/

'"'C·
L J-1 (-(X) =0 (2)

is for c1 = 0 for j = I. ., . ... n . If 1ha1·s 1.he case. we say 1ha1 then functions
f/x). j = I. 2. . ... " are linearly independenl. Orherwise. they are linearly
dependent.
Thert! i._ a convenient way to rest for the linear independence of a set of
functions. Start wilh Equation 2:

(3)

ff 1he f/x) are differentiable up 10 (n - l)th order over the interval /, differen1ia1e
Equa1ion 3 n - I times to obtain

+ c~f2(x) + + c,,_r,;cx) = 0
(4)
. r<n-1)( .r )
<. 1. 1 + f 'n .(ln- l)(x) = 0
II

The coefficient matrix of thc~e equacion~ is

h(x) J,,(x)

f2(x) f~ (.r)
W(x) = (5)

ll1e determinant W(x) in Equation 5 is called Lhe \Vrrmskia11 of Lhe func1ions fix).
If W(x) f= 0 at any point in the interval/, then the f/x) are linearly independent..

hample 5:
Test the functions sin x and cos x for linear independence oYer lhe interval
< .r < oc.

SOLUTION: The Wronskiun is

W (x) = sinx . .
CO'-\" I=-I
I .r
cos - sin .r

fnr all x. and so sin x and cos x arc linearly indepcndem for all values of x .

Unfo11unat.ely, the converse of the above result is not trne. If W(x) = 0, the
J;<x)may or may not be linearly independent, as the ncxr Example shows. (Sec
Problem 18 also.)

al
442 0 1apte-r 9 / Linear Algebra .1 nd Ve to r Sµaces

J, •. f 2

X X

Figure 9.6
11,c: functions / 1(.r) = x anti h(x) = lxl
in Example fi are :linearly indcpenden1
over the interval - I _ x :;; I. (a)

Example 6:
=
Show that / 1(:r J x and fi(:r) = lxl arc linearly independent over 1he
interval - 1 :s x ::; t. but linearly dependent over the inierval O ~ x ::: I.

SOLUTION: For the interval /-1. I ].

Ix
I
x
-1
I= -2r
IV(x) =
1-; ~ I= 0

W(x) ¥- 0 for - I::: x .:S 0. so J1(x) and h_(.r) are linearly independent over
the: interval 1-1, I I (Figure 9.6).
For the interval [ 0. I I

W(x)-lx
= I .x =0 I
I
and so this test docs no1 1dl us any1h.ing. However. fi(x) and h(x) are
idcntkal over the interval I 0. I J. so lhey arc linearly dependent (Figure 9.7).

X -I {b) X

Figure 9.7
1l1e funeri on:- =
J 1(x) x and h (,t) e I t I
in failJTlplc 6 a.re linearly dependent over
the intcrvnl O ~ x !=: I. (a)
9. S Vector Spa e 443

One final comment before we leave this sect.ion. It's ea.~y to show (Problem 17)
that if a set of differentiable functions f 1(x}, fi(x) . . .. • fn(.x) is linearly depen-
dent on an inter\'al /. then the Wronsk..ian of these functions vanishes over the en-
1ire interval.

9.5 Problems

I. Show 1hat the set of all ordered ,1-tuplcs of real numbers forms a vector ~pace if addition of two ,1-tuples
(11 1,11 2 •..• , 11,,) and (u 1, u2, .. . . Vn) is defined as (11 1 + v 1• 111_ + V_! •. .• • ''n + u,,) and scalar mulliplic:ation
is dcflncd by c(111, u2, . . .• 11") "" (cu,, ct12, ... , cu,,).
2. Show 1hat the set of all two-dimensional geometric vectors forms a vector space.
3. Show tJiat the se1 or all polynomials of degree less than or e(1ual to 3 forms a vector space. What is ir.s
dimcni;ion?
4. Show that the set or functions that arc continuous in lhc in1erval (a. h) fonns a vec1or space.
5. Show that the set of n-tuples (a, a, ... , a) is a s ubspace of Rn .
6. Lee S be a subser of R'J. spanned by (I. I. 0) and ( I. 0. I). Is 5 a subspace of V:
7. Test the following vectors for linear independence: (0. I. 0. 0). (I. I. 0. 0). (0. I. I, 0). and CO. 0. 0 . I) .
8. Test the following vector~ for linear independence: ( I. I. I), (I, -1, I) . and (-1, I, - I).
9. ls lhe vector ( I. 0. 2) io 1be set spanned by ( I. 1, 1). ( I. -l. -1). (3. I. I)'!
10. Show lhat the set of vectors {( 1. I. I. I). ( 1. -1. I. -1). ( I, 2. 3. 4). ( I. 0. 2, 0)1 is a basis for R-1.
JI. Show that ( I. I. 0) and ( l. 0 , I) arc linearly independent in R3 and find a third linearly independent vcc1or.

12. Find 1he coordinates of ( l. 2. 3) with respect to the ba.,;is ( I. I. 0), (I, 0, I). (I, I. I) in R 3.
13. Use the Wronskian to test the three functions I. sin x. and cos x for linear independence.
14. Use the Wronski.in to test the three functions ,r, c-:r . and sinh x for linear indcpcndcna.
JS. Evaluate the Wronskian of .f1(x) = x 2 and h_(:r) = l:rl:r over the interval [-1. I]. Are the functions linearly
independenl over the interval f - I, I ]? What about over the open interval (0, I I?
16. Use the Wronskian 10 test for the linear independence of f 1(x) = I. h,(x) = sin 2 x. and fi(x) = cos2 x for
all x. U" W = 0. can you check for linear independence by any other method?
17. We'll prove that if a set or (n - I) limes differentiable funcrions f 1(x). f 2 (x} . .. .. f, 1 (x) in nn interval/ is
Ii.nearly depeadenL their Wronskian vanishes identically on /. Argue that there must be nonzero const.anls in
the expression c I JI (..r) + c2 h_(.r) + · · · + cnfn (x) = 0 for every x in /. Now form the set of n cqu3lions in
Lile JJ<x) und their ftr.-t " - I derivatives . Why must the Wronskian equal 7.cro'?

18. This problem show~ that the Wronskiun of linearly independent functions may Cl1ual zero. First show lhal
2
0 .r < 0 f
Jl (.r) = { .r~., .r ::: 0 and -,(.t)

= { -x0 .r.x :::< O
0
. .
are lincarlv independent over ( -x.. -,c).Now show that
.,
W(x) = 0.
19. Prove that the coordinates of a vector in a given basis arc unique .
20. Show lhac 1he coefficien1 matrix of Equations I is made up of the three vectors in question arranged os column1,.
Using I.his observation. tC.'-1 l11c vectors ( I. -1. I. -1). (2, 3. -4, I), and (0. -5, 6. -3) for linear independence
in R 4 .
444 Ch,:1ph•r 9 / I ir,i car Algebra and V1•u11r 5pJ l!!~

9.6 Inner Product Spaces

The idea of a vector space generalizes the spaces of two- and three-dimensional
geometric vectors I.hat we discussed in Chap1cr 5. In those spaces we used a dot
product to define lcngtJu, of vectors and the angle between 1wo \'ectors. These
concepls are so useful 1hat it is desirable 10 introduce them into our general vector
spaces . We sh.ill now introduce a definition of an inner produc1 for 1he vec1ors of
a vector space.
A vector space is called an inner producr space if in addition to 1he nine
requirements that we listed in 1he previous sec1ion. there is a rule 1hat associates
wilh any two vcc1ors u and v in V a r~1I number. wrinen as (u. v) [some authors
use (u. v)]. that satisfies for all vectors in V

I.
(I)

where a and bare s.calar.;. (The inner product is a linear property.)


2.
(u.v)=(v.u) (2)

(The inner produc1 is commurn1ivc.)


3.
(u. u) ~ 0: and {u. u} =0 if nnd only if u=O (3)

(This property is known tL., po.~i,ivt! deji11ifr.•1u· .,·x .)

Problem I has you prove that the dot product that we denned for geometric vec-
1ors is an inner producr. so thal the Euclidian space of two- or threc-dimensionaJ
geometric vectoffi with a defined dot produc1 i~ an inner product space.

Example 1:
Let u = t11 1• 11 2 •.... u,,) and v = (u 1, u2 •...• v.,). Show 1ha1 rhc producr
defined by

in the vec1or space R" is an inner produc:r.

SOL u TIO N: We shall verify each of rhe above rhree properL.ies in tum :

= a(u. w ) + b(v. w)
2. {u. v} = II 1U1 + . .. + u,,u,, = ll1ll1 + .. . + u,.11,, = {\', u)
~
_,_ {u. u) = 11 1
1
+ · • •+ 11;' > 0 unle...-:s ,, 1 = 11 2 = •••= '·'ri = 0.

Jh n
9.6 Inn r Produ cr Spa 445

Example 2:
Let V be the vector space of real-valued functions that are continuous on the
interval [a. tn
Show Lim!

(f.~)= l ti
f(x)g(x)dx

is ..in inner produc1.

SOLUTION:

I. (af1 + h,h. g) = l" (aj 1 + bfi)gdx = a (11• R) h (f • g )

2. (/. g) = lfl
/:t.
J(x)g(x)dx = lfi
• fJ
g(x)f(x)dx = (~ . .f)

3. U. f) = ill J\r.)dx ~ 0 and is equal 10 zero if and only if j(x):;;; 0


in jc:Y. ,n

Mo1iva1ed by geometric vectors. we define the length of a vector in V by

llu ll = (u, u) 112 (4)

We aJso ea.II I u (I the nurm or u. For 1hc case of R". the nom1 is given by

Uul = {u. u) I('- = (11-1· + 11; + • • •+ 11'") 11•1-


-i -,

-
,,

II
(5)

The inner product satisfies an imponant incquali1y called the SchwaT7. i11-
eq11ality:

(6)

The proof of lhc Schwarz inequality goes as follows: S1ar1 with (u + h.


u + >..v) ~ 0, where A is an arbit:rary cons1an1. Expand 1his inner produc1 to write

(7)

This inequality musl be true for any value or>.. so we choose),= -(u. v)/(v. v).
Substi1uting this choice of ;.. 1mo Equa1.ion 7 gives

(u. •.f =5 (u, u)(v. v)


The square root of this result g·ives Equation 6. Notice from Equation 6 that we
can define the angle between u and,, by cos 0 = (u. v) /llu II llvll. where O:::: e .::: rr
because -I::: (u, v) / !lull Uvll::: + I.
446 Chap!.•r ') I linear ,\lgebra a nd Vecror Spaces

The norm in a vector space V s.a1i.sfies the following propenies:

11,·11 = 0 if and only if v = 0 (8)

(9)

(10)
llull
Equation I O is called the triangle inequality (Figure 9 .8 ). It can be readily proved
Figure 9.8
An illustration of tl1e trfangle inequaliry using the Schwarz inequality (Problem 4).
presented in E(!unrion I0.

Example 3:
Verify I.he Schwarz inequal iry for u = (2. I. - I. 2. 0) and\' = (-1. 0. I, 2. -2).

SOLUTION: WeuseEqua11on6:

(u. v) = -2 + 0 - I + 4 + 0 = 1
(u. u) =4 + I + I + 4 + 0 = I0
(v. v) = I + 0 + I + 4 + 4 = I0
The inequali1y reads I ::: I O in tit.is case.

w lfu and v represent geometric vectors from an origin to points given by the rips
of u and v. lhen lu - vl is 1hc gcomeuic dis1ancc between these points. We define
the distance between vectors u and v in a vector space V by d(u. \')=Jiu - vii.
which you can show sa1jsfies the following condi1ions (Problem 8):

d(u.v)=llu-vll:::0. which =0 ifandonlyif u=v (II)

= d(v. u) ( 12)

::: d(u, w) + d(w, v) ( 13)

where w is a 1hird vcclor (figure 9.9). Equation 13 is another fonn of the Lriangle
inequality.
If (u. v) = 0, u and v are s.aid to be onhogonal. For Lwo- and three-dimensional
geome1ric vectors, onhogonality means 1ha1 the vectors arc perpendicular Lo each
Figure 9.9
An illus1ralion of the triangle inequality olher. but onhogonaJity is more general lhan that. Using the definition of the inner
pre.-.entcd in Equation \J. product of 1wo funcLions as given in Example 2. we say the 1wo functions. j(x)
and g(x), arc onhogonal over [a. bJ if

1,
(f. g} =
1
(/
J(x)g(x)dx =0
Suppose tha.1 v j for j = I . 2. . ... n is a set of on ho gonal vectors in a vector
space V. lhen we can express orthogonality by writing
9.6 Inner Proclw I Sp,11 t•, 447

where 8; j is lhc Kroncderdelta. !ft.he lengths of all the vectors are made to be w,ity
by dividing each one by its lcnglh llvjll• Lhen Lhe new sel is caJled or1/1011ormal.
An onhononnal set of vectors saLisfies

(14)

Example 4:
Show that 1he gcome1ric vectors v 1 = (i + j)/J2. v 1 = (i - j)/ J2. and
v == k fom1 an orthonormal set of \'Cctors.

SOLUTION:

-
( V1,V1 ) -
(i + j) -,. (i + j) - I
-

It'.s cas) 10show that an orthononnal set of vectors is linearly independent.


Lei v 1• \· 2 • •• .• v,, be an onhonormal scr. and fonn

(15)

where the c i arc 10 be de1cnnined . Now fom1 rhe inner product or Equation 15 wit.h
each of 1.he vectors ,, 1• \' 2....• "" in ium. and find that c1 == 0 for j = I. 2 ..... 11.
Thus. the se1 of vectors is linearly independent.
We shall now show that every 11-dimensional veclor space V has an orthonor-
maJ basis by a<.:tually constructing one. Let v j for j =
I. 2 • . ... 11 be any set of
(noni.ero) linearly independent vectors in V. Start with v 1 and call ii u 1• Now lake
1he second vector in the new set 10 he o Ii near combin:ition of v~ und u 1:

( 16)

Now 1ak~ the third member or the orthogonal sc1 10 be


448 Ch, pi •r 9 / Linea, ,\I ~ br, and Vi ctor Sp,

and require that (uJ, u 1) = 0 and (u:_h u 2) = 0. This gives (Problem I 5)

( 17)

The gcncr;1\ pancm ii- evident now, and

( 18)

The uj do not form an onhonorrnal set because Liley are not normalized, but it
is easy 10 normalize each one simply by dividing by its length. This procedure
for generating an orthonormal basis from a general basis is called Gram-Scl,midt
orrhogo11alizarim1.

Example 5:
The three function~ / 1!x) = I, /~(x) = x. and h(.rj = .t 2 form a basis for
Lh~ vcc1or :-pace of all polynomials of degree equal 10 or less 1han 2. Using
Ihe detiniIion of an inner product given in Example 2. llnd an unhonormal
ba..--is over the interval 1-1, 11.

SOLUTION; We ~tnrt with \'I= I,\'~= x. and v 3 = . . ( 2 . Take u 1 = I and


write

Using Equation 16. we llnd 1hat u2 =.r. Equatjon 17 requires that we evaluate

2
J
I >
{u,. u,)
- ~
= - 1
:r~ dx = -3

which gives DJ = .\ 2 - i· We can nom1aJ i ze u 1• u::! · and u3:

2
and {u3. u3) =
JI (
-I
x - -l )
1
3
dx = -458
and sot.he three onhonormal vectors are
9.7 Complex Inner Produ I Sp.1 ' 449

9.6 Problems

I. Show 1hat rhe dot producr thar we defined in Chapter 5 for geometric vccton; is an inner producr.
2. Show that the two gcomclric vc.clor.; u = i + 2j + k and v = -i + j + k satisfy rhe Schwarz inequality.
=
3. Show thal the two functions / 1(.r) I+ x and f2{x) = x over the interval [ 0. I I satisfy rhe Schwan. inequality
given the delini1ion of lhe inner product in Example 2.
4. Prove rhe triangle inequaliry (Equation 10).
5. Show that llu1 + u 2 + · · · u11 12 = hu I~+ llu/1~ + · · · - llu,,li~ if the ui arc orthogonal. l11i$ is rhc
PytJrngorean theorem inn-dimensions.
6. Let V be 1he vector space of real functions tJ1al are continuou~ on [-,r. 11 I. Using the inner producr defined in
E."-ample 2, show 1.hat I. cos x. cos"' r ... . . sin x. ·sin 2x . . . . form an onhogonal sub.set of V .
7. Suppose llrnt u 1, U:i, .. • , u,, is an orth.ogoaaJ ba.~i ~ of V . Show rhal
(v. u ) (v. u ) (v. u,,)
v = - - -1 u 1 + - - 2- u')+ · · ·+ - - - u 1 .
(u 1• u 1) (u 2 • u 2) - (u,,. u, 1 ) '

8. Show 1ha1 the di~1ance func1ion d(u. v) surisfies Equariuns 11 10 I J.


9. Using lhe inner produc1 defined in Example I. show Lhal rhc nonn of an ordered n-tuplc of real numbers is
~
11(111 . "1· • . . , 11,1 )11 _- 2
(11 1 + ll .
2 l/~
+ · · · + 11,,) .

10. We defined the so-caJled Euclidean nomi in Problem 9. We could also define the nonn of an ordered 11-tuplc
of real numbers by llu 11 = 111 ii + lu 21+ · · · + l11n 1- Show that this definition salisfie! Equations 8 through I0.
11 . Show lhal if we define the nonn of an ordered 11-ruplc of rcul numbers by flull = max (lui i) . then this definition
satisfies Equations 8 through 10.

J2. Ca.Jcula1e the nom1s of the vectors (I, -2. 3) and (2. 4. - I) according 10 the definition given in Problem 10.
13. CaJcul..i1c 1hc norms of the vector~ ( I. -2. 3) and (2. 4. - I) according 10 the definition given in Problem 11.
14. Calculate the distance func1ions of the vectors in Prnblern 12 using the 1hrcc definitions of a no1m I.hat we have
presc111cd.
JS. Derive Equal.ion 17.
16. Cons1rucl an or1hononnol basis from I.he three \'Cttors (I. -1. 0). (I. I. 0). and (0. I. I) .

17. The functions J 1(.r) = I. h(x) = sin 2.r. and h(x) = cos 2, over 1-rr. rr I are a basi:-. for a lhrcc-dimcnsional
vector space. Con.--1.rucl on onhononnal set from lhe!!--l! three vector:-.

9.7 Complex Inner Product Spaces

In our discussion of vector spaces. we have tacitly assumed that the scalars and
vectors are real-valued quantities. It turns out that quantum mechanics can be
formulated in terms of complex vector spaces with complex inner products. so
in this brief section, we shall extend 1hc rcsult.s of the previous sections to include
complex numbers.
The central notion of linear independence is not altered if we arc dealing
with a complex vector space instead of a real one. Nowhere in lhe previous
450 CIi. ple1 l) / UnC'ar Al~chra ,md Veclor Sp,1ct.-..;

section did we specify that the vectors or ithe set of conslant.s in the defin.i1ion of
linear independence had 10 be real. Let's de1em1ine whether rhc vectors (I. i. - I),
( I +i. 0, I - i ). and (i. -1. -i) are linearly independent or linearly dependent.
We fonTJ a matrix with the vectors as rows and then 1ransform the ma1rix into
echelon fonn:

-I
0
-I )
1-_i
-,
~( I
0
0
1-i
0
-I)
I+ i
0

Thus we sec that the vcc1ors arc nol linearly independent. (Sec also Problem I.)
The primary difference hetween a real and a complex inner product space is
in our definiIion of an inner product. If we :illow the sc,tlars and vectors to be
complex. Equations I through 3 of the previous section tx.---come

( I)

(2)

(ulu} ~ 0 and (ulu} =0 if and only if u =0


We are using a venical line raIher than a comma to separa1e the two vecrors en-
closed by lhc ungutor bra~: kct~ in Equation~ I lhrough J to di s1ingui11.h between T'C!lr

and complex product spac~s . (This is standard no1a1ion in quan1um mechanics.)


If we take the complex conjugate of Equation I .ind use Equation 2. we obtain

(4)

In panicular, if we let v2 = 0. then we have

(ula v) = C1(ulv) (5)

from Equation I and

(6)

from Equa1ion 4. Note that these two equations say that a scalar comes out of rhc
inner product "a.s is'" from Lhe second position, bu1 a<; iL<; complex conjugate from
lhc firs! position. This is standard notalion in quanrum mechanic.<; lcxts. but not
in all mmhemarics 1cx1.s. Some ma1hcmutics texts dclinc a complex inner product
such lhal (ula v) = a~(ulv) and (a ulv) = a(ulv).
If u and v are ordered n-tuplcs of complex numbers. then we can define (u I\')
by

(7)
9.7 (11111pl1·, lnn<'r Produ Sp.1 451

and the length of u. l]ull by

( 8)

Equation 7 is sometimes called a Hc'n11i1in11 in11er pmtl11c1. Problem 9 has you


show 1ha1 Lh.i~ definition sarisfit'.s Equarions I lhtrougb 3.

Example 1:
Given u = (I + i. 3. 4 - i) and v = (3 - 4i. I +i. 2i), find (uM. (vlu).
!lull. and 11 '1 11 .
SOLUTION:

(ul,·) = (I - ilO- 4i) + 3(1 + i) + (4 + i)(2i) = 4i


Mu) = D + 4; ) ( I + i ) + 3( I - i) - 2i (4 - i) = -4 i = (u I\') •

11 n II = l ( I + i ) ( I - i ) + 9 + (4 + i) ( 4 - i) 1 ~ ✓28 1
'""'

11,·11= 1(1- 4i)(3 - 4i) + (I+ i)(I - i) + (2i)(-2i)J 112 = Jfi

If a set of vectors vj for j = I. 2 .. . . , ,, satisfies the condi1ion (v1lvj) = 8ij,


\\'C say rhar 1hc set is orthonormal.

hample 2:
Show 1ha1 the 1l,rec 3-ruplcs u 1 =(
I. i. I + i). u~ (0. I - i. i ). and =
UJ = (Ji - 3, I +i. 2) fonn an or1.hogonal set. How would you make them
onhononnal'!

SOLUTION:

{u ii U1} = 0 - i ( I - i) + ( I - i )i = 0
{u ii u _1) = (.1i - J) - i ( I + i) + 2( I - il =0
(u 2 lu3} = O + ( I + i)( I + i) - 2i =O
To make them orthononnal. divide each one by its nonn:

llni!I = (udu 2 ) 111 = f<l)(l)-i(i) + (I - i)(l + i)l 112 = 2


1102 I= (u21u~} 112 = I( I+ i)( I - i) + (-i)(i)J 112 "-'- 3112

llu II= (u_,lu3) 112 = [(-:'i - '.\)(3i - 3) + (I - i)( I+ i ) + 4J 11- = (24) 112
452 Ch,1ph·r 9 .' Lirn•,lf Alg bra and Vector Spac

Example 3:
Show that the set of functions

for 111 = 0. ±I. ±2 .... is orthonormal if we define rhccomplex inner product


by

SOLUTION:

= 11 and O if m :/=- 11. If m -# 11, rhcn it i~ un


The int.egral here equals I if 111
integral over complere cycles of the complex exponenlial function.

The Schwarz inequaliry takes on rhe same form for a complex vector space:

(9)

and i1s proof parallels 1he one for a real vector space (Problem 11 ).
The Gram-Schmidt procedure is also valid for complex vecror spaces. Let\
cons1ruc1 an orthonormal basis from lhc two vectors v 1 = (-1. I) and v2 = (i. -1).
We s1an wirb u 1 = v 1 = (-1. 1) and write

Form the inner product with u 1 from 1he right (see Problem 13) to get (u 2 iu 1) =
0 = (v 2 !u 1) + (o udu 1>. wh..ic.:h in !his c.:ase gives

or a = (I + i )/2. Thus. rhc two orthogonal vectors are u = (-1. 1 I) and

u, l+i
= (i. -1) + --(-1. I)= (i-1 i-1)
---, - I -
- = -(i t. i - 1)
- 2 2 2 2

The two orthonormal vectors are

and u-,- = -..,I (i - 1• i - 1)


453

9.7 Probllems
l. Show 1hat the three vectors ( I. i. -1 ). ( 1 + i. 0. I - i J. and (i. - I, -i) are nol 1inearly independent by
express~ng one of them as a linear combination of the olhl·rs.
2. Determine if l!hc vecwr l I. I, -i ). W. i. i ), and (0. I. - I) are linearly independen1.
3. Dcrcrmine if the vecrors (i. 0, 0), (i. i. 0). and (i, i, i) are linearly independenl.
4. Suppo~ that {u I v} = 2 +i. Dctem,inc (a) (( I - i )u I v) and (b) (u I 2i '').
. . 1· . I . . . b (u Iv) v C 1 I
5. .Show th :u lh e prOJccuon o a geometric vector u onto anot ,er geometric vector,. 1s given y - -- . a cu ate
{\'Iv )
!he projection of u = (I - i, 2i) onto v = (3 + 2i. -2 + i) in a complex inner product spacl!.

6. Detcnnine if the four matrices] = ( ~ ~). 17_~ = ( ~ ~). 17 _ = ( ~ 1. ~i ) . and ,,. .- =( ~ O ) arc
-I
linearly independent. These matrices arc called the Pauli ~pi11 matncc-. .

7. Verify Equation 2 explicitly for (a) u = (I+ i, I): v =(-i. -1) and (b) u = (3. -i. 2i ): v = (I.);. -1) .
8. Let u = (I. I) and v = (I. -2). Verify Equations 5 and 6 explicitly foro =i.
9. Show that 1he inner producl deline<l by Equation 7 ,afr,fies Equations 1 through 3.
10. Find the inner products and the lengths of the vectors in Problem 2.
11. Prove the Schwarz inequality for a comple,t vector space.
12. The two vectors (I. i) and ( l. I) arc the basis of a two-dimensional complex vector space . Cons1ruct a pair of
onhogonal vecwrs in this space.
13. Re-do !he derivation al the end of Ihe sect ion by forming the i nncr product of u I from the left.
14. Wed ·ri\·e B '\se/'~ ill'eq,wliry in this problem. Let ,p 1. j = 1. 2. ... . 11 be ~in onhononnal ~l!l of vectors in V
and lei v be any vector in V . If
( 10)

show Iha! c1 = {,•l,f> ,-) . Now form u = ,. - L cJ t/>.i. where,. .::: 11, and ,how that II ,,f ~ L c;.
J=I i=-1

Reference·

H. Anton. 2000, Eleme111ory Li1w11r Al1:t:bra, 8th ed .. Wiley


H. Anion and C. Rorre~. 1994, Eh·mtmm:\: U111:ar Algehm: Applim1/rms Ver.firm. 7th c<.l ..
Wiley
P. Denncry and A. Kryzywid:i. 196,7. Muthenwric,1· Jlir Physicists. Chapter 2. Dover
Puhlrcal'ioos
F.B. Hildcbr.md, 1992. Methods of Applied Mmlu:matics, Chapter I. 2nd ed .. Dewer
Publications
S. Lip!>chuu .. :woo. U11£•ur Algebra. 3rd ed.. Schaum·l- Outline Si:ric . . , McGraw-Hill
S. Lipschut7.. 1989 . •WOO Snfrnl Pmhh·mx i11 l.m£·nr Algebra. Schaum·s Outline Series,
l\kGraw-Hill
C Rorrcs and H. Anton. 1984, Applirntim,s of Li11eur Al,i;ebra, Jrd t:d .. Wiley
CHAPTER 10
Matrices and Eigenvalue Problems

In the previous chapter, we learned that a matrix is a rectangular array of elements


1hat obeys a cenain set of algebraic rules, called matrix algebra. For example, we
learned to add and subLract ma1rices and to muhiply 1hcm 1oge1hcr. We saw 1hal we
can associate a matrix with an operation or a transformation such as a rota1ion of a
vector through some angle fJ about some axis or the reHection of a vector 1hrough a
plane. Matrix multiplica1ion corresponds to applying two or more Lransformations
in succession. The idea of undoing a 1.ransformalion leads to the idea of Lhe inverse
of a matrix. so that if S represents some transformation. then s- 1 represents the
effecl of undoing that transformation.
We shall continue our discussion of matrices in this chapter. Seel.ion I deals
with orthogonal transformations. A matrix that represents a rotation or a rcncct.ion
is called an orthogonal matrix and satisfies the convenient rcla1ion A- 1 ===AT_ This
relation implies Lhat lhe rows (columns) of an orthogonal matrix are orthogonal. In
Section 2. we introduce one of 1he most important malrix equations, those that have
the form Ax= ;,x. where,\ is a scalar and x is a (nonzero) column vector. Values of
>.. that satisfy this condition are called eigenvalues. and the corresponding vectors
x are called eigenvectors. We'll learn how to determine the se1 of eigenvalues and
eigenvectors of a given matrix: (this is called an eigenvalue problem). Many diverse
physical problems can be formulated as eigenvalue problems. and we discuss
several illus1ra1ive applications in Section 3. We shall see Lhat lhe values of lhe
elements of a transfoITT1alion matrix A depend upon the particular basis bci ng used.
If we change I.he basis, then we change the elemems of A. It is oflen possible Lhat
if we choose Lhe basis appropriately. A will be a diagonal matrix, which is very
convenieni compu1ationally. In Section 5, we learn how to determine the basis
for which A is diagonal. a process called diagonalization. Many mathematical
and physical problems involve the diagonaliz.ation of malrices. and we discuss
a number of lhese problems in lht: final section.

10.1 Orthogonal and Unitary Transformations

In Section 9.J, we showed that the malrix

R(A) = ( c~s 0 0)
- sin (I)
Sin fJ cos0

corresponds to the rotation of a two-dimension.ii vector through an angle 0 in a


counterclockwise direction. In other words. Ru=\'. where u and v are shown in 455

al
456

y Figure 10.1. We say 1ha1 R Lransforms u in10 v. Similarly. 1he matrix

\'
(2)

u corresponds to reflecring a vec1or u through they axis because Su= (-"x· 11,)T
(Figure 10.2). (We wrire the column vector wirh components -11.r and 11 1. a-.
{-u (. u_\,) T for convenience in 1ypese11ing.) ·
The malrices R and S represent not only transformations. but 1hcy represent
linear transfonTilltions. A linear 1ran.~(omullim1 A on the vectors of a vector spal.'C
\/ is a function that takes one vector in V into another vector in \/ such that for ull
X
u and\' in V
Figure 10.1 A(au + /3v) = aAu + fJAv (3)
The roiation of :i VL>ctor c:ounten::lockwise
through •1111rnf!lc II. where a nnd fJ arc scalars. Le.i's show 1hat R given by Equa1ion is a linear
1ransforma1ion. Lei u = 11 ... i + 11_,.j and v = u_,. i + v_\'j. Then.

R(au + /3 v) = R [ (a 11 .r + ffo r) l + (cm _ + /J 1• y) j I

- sini0) ( Ci.llr + /J ll:r )


COS 0 (t.lly + {foy
(aux + /311_r) cos() - (au , + /h\,) sin 8 ) 4
=( (cw., + ft11_,.) sin i:i + (O'u : + /hi_;.) 4-•.os O ( )

X
Now Ru and Rv are given by
Figure 10.2
The rdlcction of a wdor 1hrough the .I' Ru-= ( co. 0 ".i - . in 0 u.r ) and R,· = ( cos 0 t\ - sin {J u_1. )
axi.-.. . in 0 11 r + co::. 0 11 y .sin fl Vx + co 0 v 1•

Thus.

(5)

in agrecmcn1 wi1h Equation 4.

Example 1:
Show that

A(etu + fjv) = aAu + fiAv


if
0
I
-\

SOl U TI ON: Th~ expression A (au+ /J v) in mairix form is


10.1 Orthogon.il and Unitary Tran forma1f n 457
I 0
0 I
(
2 -I

11 _, - U: )
=a ".v+".: +fi
(
211 ,T - 11_1•

= aAu + f,Av

Jf we introduce a basis se1 into our vector spaces. then we can investigate the
result of 1he t.ransfom1.itio11 in 1cnns of the basis vectors. For concreteness (and
simplicity). we will choose the so-called standard basis in V = R" . By a standard
ba..,,is. we mean .1 set of basis vectors e 1 = (I. 0. 0 .. .. , 0) T_ e 2 (0. I. 0 ..... 0) r_ =
= =
e~ (0. 0. J. ... . 0) T_ and so on up to e,, (0. 0. 0 . . . . . I) r. where 11 is the
dimensionality of V . No1e 1ha1 e 1• e 2 . e_, corm,pond 10 i. j. kin three-dimensional
Euclidian space . We shall restrict our discussion to transformations \' = Au such
that v is in 1he same vector space as u. Because v Au is in V. we can write =
II
"
u=L 11 iei and V = I: Vjej
}=I }=I

Funhermore.
n n

v =Au= A _L u1c1 =L11 1Ac1 (6)


_1=1 j=I

But thl! veer or Ae i can he ex pressed as a I inear combi nu lion of !he Ie;) . so

I/

AeJ = L alje,- (7)


i=I

In Equation 7. a;j is I.he ith componcnl of 1hi.: \cdor Ae., in rhc standnrd basis {c, 1-
Substiluting Equation 7 inlo Equation 6 give~
,, 11

v= L i L a,1e,-
II
i=I i=I

= t (t
i=l j= I
0 iJ 11 j) e,-
Thus. the ith component of,, in the standard basis is given by
n

u,, = L O;jll j

i "" '
458 Ch cl'l)lt:' r 10 / M,ltfil es•~. ,tnd Eig rwalue PriJb lts>111,

Let's write out Equation 8 in full so we can view the components more clearly.

(9)

Notice lhal if u = e 1 = (I, 0, . .. , 0)1, 1hen v = (a 11 , a:!J• . ..• a,, 1) \ if u = e 2 =


(0. I. 0 .... , 0) l, lhen v = (a 12 • a:22 •... , a,. 2 )1, and so forth. Thus. the j 1h column
of A is given by Ae j • In other words. the matrix A is complctel y detem1incd by
how ii t.ransforms I.he vectors of the standard basis.

Example 2:
Delerminc rhe ma1rix A if Ae 1 = (I. 2. -l)T. Ae 2 = (0, I. l)T. and
Ae 3 = (I. I. O/. Show that A is nonsingular.

SOLUTION: The Jth column of A is given by Ac j, so

The dctem1inant of A= 2 . .so A is nonsingular.

The linear lf'c.UlSformations that we have considered in this sec1ion 1r.rnsfonn


given vecrors into olher vectors. For ex.ample. the ma1rix described by Equal.ion I
corresponds 10 a reunion of a vector lhrough an angle 0 in a countcrclockwise
direct.ion. Si nee Ru corresponds to a rotation of u, we should expect rhat lhe length
of u does not gel aJtered under R (Problem 7). Lei\'= Au be a lioear Lrmsfonnation
of u tha.l presen•es its length. Then

(v. v} =(Au.Au)= {Au)T (Au)= t? AT Au (10)

= I. or that AT = A- 1.
I r v and u are 10 have lhe same length. then we must have AT A
A matrix with this propeny is said to be orthogonal. and v = Au is ~aid 10 be an
orthogonol transformation . We can state this imponant result as follows:

A linear trr.m:.fonnarion preser1·es le11grhs if and 011/y ,firs matrix is


orthogonal.

Example 3:
Show that R given by Equalion I is an orthogonal mJtrix.

rn
10. 1 Or1hoi:un,1 l and Unilal)' Transr< 11111,ll i, ms 459
SOLUTION: We need 10 show 1ha1 R1 = R- 1 or 1ha1 RRT = I.
~in U ). ( cos 0 - sin 0 ')
cos O sin 0 cos f)

= ( cos2 0 ~ sin 2 0 ., 0 ') ) =( I


sin-11+cos-0 0

ll is also 1ruc that RR1 = I.

No1 only arc the lengths of vec1ors preserved under onhogonal u--ansformn-
tions. but the angles bc1ween vectors are preserved as well. Recalll that the angle
between two vectors in a vector space is defined by cos 0 == (x. y) . To
(X. x) lf2 (y, y) l/ 2" .
prove that the angle is preserved under an orthogonill transformation. we simply
need to show that (Ax, Ay) = (x. y}. Now

(Ax, Ay) = (Ax) 1 Ay = x1 ATAy = x 1y = (x, y) ( 11)

Furthermore. the distances be1ween 1he ends of vectors are also preserved by
orthogonal 1ransfom1a1ions (Problem 8).
The orthogonali1y cxpn:s.sion

( 12)

or
implies that the rows A1 and the columns of A are orthonormal. The rows of Af.
howc\"er. are 1hc columns of ,4.., so Equal ion 12 impl ic.." that the column vectors (also
the row vectors) of an orthogonal matrix are orthonormal. To see this analy1ically.
insert the fact that o;: =
aJi into A 1 A= I to get

n "
L CJ~.ajl: = 8il, = L a11llJJ.. ( 13)
i=I J= I

Equation 13 says 1ha1 the ith and k1h columns of A are orthononnal vectors. If
we use A A 1 = I instead of AT A= I, we find i.hat 1he ith and kth rows of A are
orthonormal vectors (Problem 9).

Example 4:
Firs! 1-how thal

is onhogonal and lhl·n show l'X.plicitly that it$ rows (column, 1arcorthonom,al
vector-.

al
460 Ch.1pr •r 1O / Malri 5. and Ei ·•n ;;ilue Pro blems

SOLUTION:

A1 A= _!_
81
u -4)DU(
-4
-7
4 8
-4
I
-4)
-7
4 Ci Sn
= -
81
I
0
81
0

IC 8 0
4

AAT = 81 : -4
I
-7
4
8I -4
-4 -7 !) Ci 8n
= -
81
81
0

The columns are normalized because

I I I
-{1+16+64)=1. -(64+16+1)=1. ::md-(16+49+16)=1
81 81 81

Column I and column 2 are onhogonal vec1ors because

Similarly. columns 1 and 3 :md columns 2 ;mJ J ::ire onhogonal. For rnws 2
and rows 3.

(4, -4, - 7) (!) = :12 - 4 - 28 =0

Row.) I and 2 and rows I and 3 arc also onhogonal.

=
Using the fact !hat de! (A B) det (A) deL CB). it is easy to show that the
detem1 in ant of an orthogonal matrix is equal 10 ± I (Problem I I). If det (A) I. =
then A corrc~ponds lo a pure rotation. and if dct (A) == - I. A corre~(X)nds to a
rotation and an inversion through a plane.
Up 10 thi, point. we have consickrl·d only matrices whose elements arc ri:!al.
We frequently deal with vector spaces over the field of complex numhers in
quantum mechanics. and .<.o we shall end this ,e<.·tion wi1h an extension of our
rc,ulr.s 10 matrices whose elements are compkx. Recall from Sec1ion 9.7 that we
dchnl·d an inner product in a complex vector space by

{ 14)

s.o that the lcngtJ, of a vector is given by

( 15)

A linear Lransfonnation that preserves length~ must :rntbfy


10, 1 Or1I u >s< 111,1 I a,,d l l11it.11, Tc,rn,ic ,m1,11 :r >•h 461

or

(16)

A maLrix that sat.is fie., Equation 16 is said 10 be 1111itary. We shilll denote (At) T by
A"' (called the Hermitian crmj11gare of A). so Lhat Equation 16 becomes

(17)

or A - 1
= A". Note that a unitary matrix is the analog of .in onhogona I matrix in a
complex vec1or space. The analog of Equation 12 is A-t A = AA t = I. Equation 17
shows that the row.,;; (columns) of a uniwry matrix are onhononnal. Using 1he fact
that a:i = aj;• Equation 17 becomes
I/

L a;jajk = L a>ij/,.:....: ,\,'t


i=I j=I

which shows that the rows of A'· are onhononnal. The second expression in
Equation 17 shows 1ha1 the columns arc orthonormal. Problem 14 has you show
that the dc1crminan1 of a unitary matrix is of absolute value I.

Example 5:
Show that

A=-I
5
(-1+ -4-2-i
-2i)
2i
2 - 4/

i~ unitary.

SOLUTION:

A = (A~) T = -5I (-1


-4
- 2i
2i
2 +4i)
-2+i

and

-4 . ' '21 ) = ...!_ ( 25


-~ - i 25 0

AAi = _!_ (-1 + 2i -4 - 2i ) ( - I - 2i 2 + 4i ) = _!_ ( 25


25 2 - 4i -2-i -4+2i -2 + i 25 0

Note also that the rows and columns of A are orthoaonnal and that det A =
(20 - I Si)/ 25, which is of uni I magnitude,

al
462 Chapter 10 / M, 1r:, , .•• ind fif:Pnvalue Problems

10.1 Problems
I. Show 1ha1 the matrix S given by f-4uat ion 2 represents a rcflcl:1ion of u Ihrough the y axis.
2. Consrruc1 a 2 x 2 matrix that represents the refleclion of a two-dimensional vector through the origin. (This
operation is called i111•er.fio11.)
3. Determine the geometric result of the following mairin:s ac1ing on a two-dimensional vcc1or u:

(al (-1 0)
0 -I
(b) (c) I ( I
.Ji I
- t)
1
4. Prove that a non.singular lrJn~formation 1mnsfonns linearly independent vec1on; into linearly independent
vec1ors. Hint: Assume 1hc contrary.

5. Detenninc- the matrix A if Ae 1 = (0. I. 1) 1 . Ae 2 = (I. 0. l)r, and Ae 3 = (I. I, 0) 1 . Show that A is nonsin,gular.
6. Suppose the linear 1ran.sfom1::itfon v = Au transforms u 1 = (I. 0. I) into ,· 1 = (2. I. -1). u2 = (0. I, I)
into v 2 = (I. -1. 0) and u3 = (I. 1. 0) in10 ,.3 = (0. -1. I). Find the matrix that corresponds to 1his linear
transformation,
7. Show that u and Ru have lhe same length i( R is given by Equation I.
8. Show 1ha1 Lhe disrnnce between 1wo points is preserved under an onhogonal transformation: in other words.
show Lhat orthogonul t.ran:-formations preserve distances.
9. Show that the row~ of an orthogonal matrix arc onhogonal vectors.
10. Prove tha1 1he produc1 of two onhugon:il matrices is onhogonal.
l L Using the foc.;t thal uel (A 8) = dct (A) dcl (B). :.how 1.h::11 the dctcr111i11a11l of an orthogonal matrix is equal
10±1.

12. Show 1ha1 A= -


I ~ 2 _;) is orthogonal.
3 (
-~ 2 -1
13. Show that the columns of a uni1ary matrix are orthogonal.
14. Show 1ha1 the de.terminant of a unitary matrix is of abi-olute value unity. Hint: You need the rela1ion
=
dct (A B) dct ( A) det ( 8).
I ( 2 - 4i 4i ) . .
15. Show 1hat A= - ➔i _ _ _; 1s unitary.
6 2 4

16. Show 1hat 1he rows and columns of I.he unillli)' matrix in Problem 15 arc orthonormal.

I ( : _ 0 -I
17. Show that A = .Jj
1 1 0 -1) is uni1ary.
() -i

·1 0.2 Eigenvalues and Eigenvectors

Most of the linear transformations that we have discussed have been of 1he form
Au= v. where u and v are vectors in 1hc same vec1or space. be it re~1I or complex.
It turns out that linear transformations of the fonn
463
Ax =)..x (I)

play an imponant role in a wide varie1y of physical applicarions. In Equation I. A


is a given square matrix. )... is a scalar that we do not know beforehand. and x is a
vector Lo be determined. Certainly x = 0 s.atislies Equation I. but we wish to find
solutions 01..hcr than trivial solutions. Values or). that su1..isfy Equation I are called
eige11v{l/11es and nontrivial vec.:tors that s.atisry Equation I are called ei[::em·ecrors.
The problem of determining the sets of>.. and x that satisfy Equation I is culled an
eige,walm: prublem.
We c.:an rewrite Equalion I in tbe fom1

(A-Al)x=O (2)

where we have inserted the unil ma1rix I so that the factor in parentheses is a
well-defined matrix. If A is an n x II matrix. Equation 2 reprcsenL,;; a ,;;ystem of n
si multancous Ii near algebraic cqua1ions in II unknowns, x 1, x 2 , •.. , x," We know
from Chapter 9 that the condi1ion that there be a nontrivial solurion to Equations 2
is that

de! (A - i. I) = IA - ), I I = O (3)

Upon expanding this II x II dclerm i nan 1. you· 11 find that il yields an nth degree
polynomial in (1hc unknown) i... The polynomial equation given by Equation 3 is
called the clwrauerisric equation or rhc sl'C11/ar l'q1mrio11. The 11 ~olutions 10 1hc
characLeristic equation given values of>,. for which Equation I will be satisfied.

Example 1:
Find the ei1genval11cs of lhe matrix

-1
A= (
4

SOLUTION: The characteristic equation is

1
2 - )..
I = ),~ -
.. i, - 6 =0
or A= 3 and -2. Thus. lhis: 2 x 2 e,igcnvalue problem yields 1,vo eigc1walues.

Once the eigelilvaihiC!-. arc dc1errnined. we can determine 1he corresponding


iJ r \ c ubs1 itute >. = J in10 Ax = ). x, we have
ei!!c1wcctors.

-x + y = 3x
4.r + 2y = 3y

al
464 Chapter I O / M a tric and Eig rwalue Problems

or

-4x +y =0
4.t = y =0
These two equa1ions are esi.en1iaJly lhe same. and yield y = 4x. Thus. tJ,e l!igen-
value corresponding 10 >.. = 3 is (a. 4a) T. where a is any nonzero consran1. For
). = -2. we find 1ha1 Ax= -2 x gi ves
x+y=O
4x + 4y = 0
and so the corresponding eigcnvL-ctor is (b. -b) T, where h is any non zero cons1an1.
It shouldn't be surprising that both cigenvec1ors arc dc1crmincd only 10 wi1hin
an arbitrary fac1or because you can see from Equa11on I lhat if x is a soluLion.
so is any multiple of x. Thus. Equution I can determine x only to within a
mul1iplica1ive const:rnl. Nori cc tha1 the two eigenvectors (a. 4a) and (b. -b) a.re
linearly independcnt.

Example 2:
Find rhc: eigenvalue~ and eigenvector:- of the: matrix

SOLUTION: The charac1cri~1ic equation of A is

or ( I - 1) = ± 2i. or ), = I =i= 2i. For). = I + 2i. we have


2ix + 2y = 0
1x -2iy =0
or x = i y. Thus. the corresponding eigenvector is (i a. a) T_ For )._ = I - 2i,
we have

2x + 2iy =0
or x = -iy. The corresponding eigenvector is (-ib. b)T_
10.2 £i~t•m ·,1lues ;:ind Eigt•nw·1 tw, 465

Example 3:
Find the eigenvalui!s and cigenvec1ors of

A= ( : ~ -: )
2 2 3

SOLUTION : The charactcri~,ic equation of A is

I- A 0 -t
I 2-)..
2 2 J - i.

3.Jld so;., = I. 2, and 3. For).._ = I. Ax= x gives


;: == 0
x+y+ ::= 0
X _\" ::- 0

or x 1 = (11, -a, o?. For).= 2, we get


X +~=0
X - =0
2:r + 2y +:: = 0
or x2 = (-2/,, h. 2h)T _For,.= l we gc1 x_1 =(,.-c. -2,-)r _

If the eigenvalues of a mmrix are distinct. we say that they arc 11011-dt•gn,erare.
If an eigenvalue repeats, we say that it is d£•gt·11crotl'. and if it repeals k times, we
say that i,t is k-fold degcnaore. All the eigenvalues !hat we have found so for arc
non-degenerate.
You may not have noticed ii. but1the product or 1he eigenvalues in each case
above is equal to the dcterminam of A. To sec why this is so. write the characteristic
polynolillial as (). 1 - ).)().~ - A) · ·· ().11 - ) . ). where ). 1, ).~ • . . . • l,1 are the,,
ei1;?em alues. Th.is [P roduct is equal 10

dct (A - ), I)= (), 1 - ). )( A_ - >...) • · • (). ,1 - i.) (4)

If we lei A= 0. we sec that ), 1). 2 . •• ,. ,, = det (A) . This rt·,ult 1ells us that A is a
singular matrix if and onl)' if at leas, one of its eigem alucs is equal to zcm.
The .:ibove calculations of cigcmalucs also sug~c~, that the sum of the eig~n-
valucs of A is equal to rile sum of 1hc dia~onal element-' of A. In Section 4. we'll
prove that this is true in general. The ~um of the diagonal clemen1s of u matrix A
is called the 1ma of A. and so we can write

A1 + ).2 + · · · ), 11 = Tr A (5)

C
466
Another observation from Lhe four calcula1ions that we did above is 1ha1
the eigenvectors corresponding lo different eigenvalues arc linearly independent
(Problem 7). We can formaJize this observation with Lhc following Lheorem:

If x 1, x1 , .... '!IC 11 un: rlw c·igem·ecturs corrt·!i:po11di11g to the dis1inc1


ei,:e11w1lues >.. 1• ;. 2 , . .. • A11 • 1he11 x 1• x1 . . ... x,, are li11early i11depe11denr.

Problem 8 works you 1hrough a proof of this theorem.


If Lhe eigenvalue:-. aren' I distinct. lhcn there is no assurnnce thal 1he eigen-
vectors will be linearly indcpcndcn1. They mny or may nol he. Problem 3 treats
a 2 x 2 matrix. with equul eigenvalues for which I.here is only one linearly inde-
pendent eigenvector. whcrca~ Problem 4 treats a 3 x 3 ma1rix with a dl'g.cnera1e
eigenvalue wilh three linearly independent eigenvec10rs.
There is a cenain class of matrices. however. for which the eigenvectors are
linearly independenl even if 1he eigcm•alues arc no1 all dis1inc1. Jn a real vector
space. 1hc!.e matrices are symmetric. meaning I.hat
A 1 ;;:: A (symmetric matrix) (6)
In tem1s of the elements. "ii;;:: a ji. The corresponding propeny for a complex vec-
tor space is tha1

(AT)""= A =A (Hennitian matrix) (7)

or that A equals its Hermitian conjugate. In tcm,s of the clcmcn1s. aiJ = c{ = ll ;; ·


M.:i1riccs 1.h:11 s:11 isfy Equ:Hion 7 are called Ht:rmitia.11. A Hcnnitian matrix i~ the
complex vector space analog of a symmc1ric matrix in a real vector space. IL 1urns
out that symmetric and Hcnni1ian ma1riccs arise frequently in applied problems.
In fac1. the formalism of quamum mechanics is based upon Hermitian mat.rices.
and Hermitian operators in general.

Example 4:
Show 1ha1 1he ma1rix

I -i )
A= ( ~1 0 -l+i
l+i -1-i J

is Hermitian.

SOLUTION: The complex conjugate of A 1 i-.

At = ( -,I.
l+i
0
-1-i
-lti
1- i )
=A

and so A is Hcrmilian. Also note that a;i = a,, = nj;-

qh
I 0. 2 IS ,,wa lu • • nd ig nv ror~ 467
Symmetric and Hcnnirian matrices have a number of imponant and useful
properties. Since a symmetric ma.t-rix is a. special case or a Hennitian matrix if
its elemenL'- happen to be real. we shall illust.rate the.-.e propen.ies for Hem11tian
matrices.

The eige111•a/11c.1· of a Hem1irio11 matrix ore real.

TI1e proof of this is fairly straightfor.vard. Let A be a Hermitian matrix with


eigenva.lue ), and corresponding cigcnvec1or x. Then

Ax= ;._x and

Multiply the first equation from the left by (x '" ) :::: xt. to gel

(8)

Now, transpose the second equation

and then multiply the result from the right by x to oh1ain

(9)

Now :rnbtract Equations 8 and 9 to get

But A = At because A is Henn i1ian, and x t x O si nee x =f:. 0. so i. = ). •. or. in


01.hcr words, A is real. In quan1um mechanics, mc.isurablc quan1ities correspond
to eigenvalues of matrices tht1l are required 10 be Hermitian matrices bec.lusc
mca))urable quantities must be real.
The proof that we jusl prcscntct..l 10 prove thut I.he eigenvalues or a Hcnnitian
matrix are real can be slightly modified to prove that

Th,· 1·igc111"1'c/Ors corrt•s110111/ing to disri,11•r eige11wi/11e.\· ofa Hermirirm matrix


are ortltogoiwl.

To prove this. let A be a Hermitian matrix with distinct eigenvalues >.. and µ and
corresponding eigenvectors x and y, re~pectively. S-O that

1\x :::: ).x and


468 Chapler 10 / MJtri e and Ei •enva lu Probl m

Multiply 1he firs1 equation from the left by y~·. then transpose the s-ccond equation
and multiply it into x 10 get

and

Subtract 1hese two equations and recognize that A= At_ because A is Henni1ian.
to arrive at

(A - J.l) y·x =0
Because A f;. J.L. we have ytx = 0, meaning thal the eigenvalues of distinct eigen-
values arc onhogonal.

Example 5:
Find 1he eigenvalues and eigenveclo~ of

and show that the eigenvectors are mutually onhogonal.

SOLUTION; The chorac1cris1ic cqu111ion of A ii-

\ -)._ 0 I
0 I -J. 0
0 I - ,\

or).= 0. 2. and I. For)..= 0. we have

.f +z=D
y =0
X +:::= 0

or x0 = (a. 0, -a)T. For ). =I and 2, we get x 1 = (0. b, O)T and


x:i = (c. 0. c) T. respectively.

and
10.2 Eigenvalue and [ ig n\' tor~ 469
Even if the eigenvalues of a Hennit..ian mallix are no1 distinct, we can con-
sLruct an orthogonal set of eigen\'eclors. Suppose Lhat the eigenvalue >.. 1 is k-fold
dcgenera1e. In this case. we have

for i = I. 2 ..... k (10)

Although we shall no1 prove it here. if). is a k-fold degenerate eigenvalue of a


Hem1i1ian malrix. then there are always k linearly independent eigenvecrors cor-
responding to ).. Because of this. we can use U'le Gram-Schm id1 orthogonal izalion
procedure that we introduced in Section 9.6 to construct k. or1hogonal eigenvec-
tors from the J.: linearly independent eigenvectors in Equation I 0. Consequently.
we can say that

An 11 x II Hem1iria11 ma1rix has 11 11111t11ally onhogonal eigenl'Olues.

Example 6:
Find an orthogonal set of eigenvectors of

3 I
A= I 3
( 0 O

SOLUTION: The characterislic cqu:11ion is (2 - i.)(). 2 - 6).. + 8) = 0 or


).. = 2, 2. and 4. Thccigcnvec1orcorrcspondingto .i.= 4 isx 1 = {a, a, O/. For
A= 2, we get x = -y and:= z. If we le! x =Cl.and:= {3. the cigenvec1or
can be written as

Therefore. x2 = (I. -1. 0) T and x3 =- (0. 0. J/ are (linearly independent)


eigenvectors eorrcsponding to)._= 2. as you can show by dim:1 calc.:ulation.
h turns oul thal x 2 anc..l x 3 as ,ve have wrinen 1hem here are orthogonal ro x 1
and orthogonal 10 each 01her.

As is often 1he case. we conclude this !;Cclion by pointing out that any CAS
can be used 10 calcula1e the eigenvalues and eigenvectors or a matrix.. For example.
U'lc one-line command in Mathematica.

Eigensystem [ { {1,0, 1}, {0, 1,0}, {1,0, 1} } ]

or Example 5. Problems 17 through 19 ask you to use any CAS


gives the rcsuils
to dewrmine the eigenvalues and eigenvectors of some matrices.

al
470 Ch pier 10 / Malri - and i • •nvJlu Problem,_

10.2 Problems

I. Detcrn1inc the eigenvalues and eigenvec1ors of A = (; I) . H


2
. . .
ow many lrncarly independent e,gem•ectors
are there?

2. Determine lhe cigcn,•al ues and eigenvec1ors of A - ( ; _ ~ -i)- How many Ii neurl y in dependenl

eigenvectors are there?

3. Detem1ine I.he eigenval'ue: ~md t!i_!!envcctors of A= ( : - ~). Show thai there is only one linearly indcpcndcnl

eigenvector.

4. Determine 1he eige·nvalues .ind ei1g envcctors of A =


(
~1
,
-2
-4 -1)
-~ . Show I.hat there :.ire 1hrcc linearly
-4
indepcnde111 eigenvectors.

5. Show that Lhe eigenvalues ob1aincd in Problems I lhrough 4 ,ari,fy Equations 4 and 5.
6. Show that Equation 4 is 1rue for a general J x 3 matrix. Do you see any pattern?
7. Show that 1the eigenvectors in Examples 2 and 3 are linearly i.ndcpcndcnl.

8. Prove th3t if X.1, x~ .... , x,, are the eigenvectors corresponding to the di!ajncr eigenvalues .l. 1• >.._• ... • An•
I.hen x 1• x:! . ... , x,, are linearly independent. Hillf: Suppose 1.hal c· 1x 1 ~ "-~ ~:? + · · · + c, 1 x,, = 0 for som~
nonzero cl:,. Show thar (A - }.~l)(c 1 x 1 + c 2 x2 + -· · + c,, x, 1 ) is missing the lerm ,·2 x2 . Now open.lie wiIh
(A - >..1l)<A - 1....11) · ·· (A - ). 11 1) and show thal the result is c 1().. 1 - ) .~)(). 1 · J,, 3 ) · · - (), 1 - ). 11 )x 1 = 0. Now
argue 1h01 1bec·uu-..e all 1he >.. ·s are distinct anc.l x, -:/= 0. then c 1 = 0. Derive a similar result for all the x j and show
1hn11·J =Ofor j = I. 2.... ,11.

9. Show that the eigc·nvalucs of AT are the same as those of A .


10. Show that ,if A i.s a real mal"Jiix. wilh a complex eigenvalue 1. and cigenvcclOr x, 1hen the complex conjuga1c-. :i.•
and x• arc aJso L'igcnvalues and eigenvectors.

l I. Show that the eigenvalues of a unitary matrix arc of absolute value one.
12. A ma1rix. is caJled skew-Hermitian if A= -(N )T = - At .Show that tJ1e eigenvalues of u skew-Hermitian
matrix are pure imaginary.

13. Show that the eigenvalues of A - 1 a.re ). -:: 1•


J
14. Show thal A" x = AIll( ir Ax= ).x.
15. We can fonnally c.lcline a !'unction of a matrix by a Maclaurin expansion. If a function has a Maclaurin
~ ~

1.!Xpa11-,irn1 J (x) = L on:< 11


• which converyes for Ix I < R. then the matfr.; ~cries /(A)= Lon A" convcitc~
11=0 11=:0
if each of the eigenvalues of A is less than R. Show thal if Ax= AX. Ihcn j(A) x = /(A.) x.
16. The Cayley-Hami/1011 theo,.em says tha1 every si:Iu~rc matrix ,:11i,1ics iL~ own characteristic equaIion. Verify
this theorem for the mnr.rices given in Problems I and 3.

17. Use any CAS 1n deiermine lhe eigenvalues and eigenvwors u; A= ( :


I 0.3 S11n11· Applied Eig nva,lu · Prohlen'1~ 471

18. Use any CAS to dc1ennine I.he eigenvaJues and eigenvectors or A = ( !~


() 0
19. Use any CAS 10 determine the eigenvalues and eigenvectors of
ij./2
2
-i/...fi.)
0 .
0 2

10.3 Some Applied Eigenvalue Problems

In this sect.ion, we shall present a few examples of eigenvalue problems that arise
in physical systems. A couple of these will involve differential equations (wl1ich
we don'I discuss fonnally until the next chapter). but only in an clemcnwry way.
A wide variety of applications of eigenvalue problems involve sys1cms of
simuhaneous first-order linear differential equations wi1h constant coefficients.
For example. a ~equence of rndioac1ive decays from a parent isotope through
subsequent genernt-ions

A - B---+ C-----+ o- •· · __, X

or the time dependence of the occupalion of energy levels of 1he molecules in a


lasing material. or a srudy of the stability of equilibrium systems. or calculations
of currents in elecLricaJ networks lead to se1s of differential equations of the form

(I)

where the";/• are constants. We can write !his set or equations in matrix. form:
dx
- =l\x (2)
di
Let's consider a two-dimensional case first:

(3)

where I.he overdoLc; are slandard nolation for time derivatives. In the one-dimensional
case, dx /dt =
ax, r:hc solution i~ ).imply x(t) = x(O)e"'. Therefore, let's lfy a so-
472 Chap1 r IO / Marri - and ig -nva lu ProblPms

Jut.ion of the fom, x i') = 11 1e). 1 • If we substiwte this into Equation 3, we obtain

or
" 1<1 - >..) + 211 2 = n
(4)
211 I + U2( I - A) =0
To have a nomri vial solu1 ion for 11 1 and 11 2• the determinant of the coet'ficients must
equal zero. so that we have

1:,.),, ? I
, I~)_ =0
1

Note thar tJ1is i., just the detem1inantal charac1cris1ic equation of the coefncicnt
matrix A in Equation 3. The eigenvalues are A = - I and ).. = 3. To dcteITTline
11 1 and 11~ a.-.sociatcd with each value of A, we sec ).. = - I in Equation 4 lo find
that u 1 = -u 2 , or that u = ( I. - I/. Similarly, for).= 3. we have u = (I, 1) 1 .
Therefore.

and

arc both solutions to Equation 3. We ' ll learn in the next chapter that !he general
solution is a linear combination of v 1(r) and \' 2 (n. or

(5)

We can deterniine the values of c 1 and c 2 by employing the initial values of x 1(t)
and x 2 (r). Suppose tha.r .r 1(0) = I and x 2 (0) = 0 . Then Equa1ion 5 reads
X1(0) = C1 + C1 = I
X2(0) = -("I + £'2 :::; Q

or ,· 1 = c2 = 1/2. Thus. Equation 5 becomes

x(t)
-
=- I( I)
2 -I
e- 1 + -l ( l ) e·'.J1
2 I

or, in terms of .x 1(t) and x:!(t ).

It is a simple mancr 10 show 1hat x 1(t) and x 2 (1) su1isry Equation 3 and 1hc initial
condi1ions.
l O.J Some Applil'd Ei1•,t•rw ,1lul' Probl m 473

In general. we subs1·i1ute x(!) = ue ~1 into Equation 2 10 ob13in


(A- ),I) u =0 (6)

Let the eig.cnva1l.ues and lhe corresponding eigenvectors of A be >- 1, • • , • ).,., and
u, ..... Un. so that
j = I. 2 .. . . , n (7)

Substituting x(,) = u je)·J' into Equation 2 shows that this is a solution to Equa-
tion 2. TI1c general solution to Equa1ion 2 can be wrinen as a linear combination
of the individual solutions u1/·/:

(8)

To prove that this is indeed a solution to Equation 2, we substirute it into Equarion 2


and show that the two sides arc the same (Problem I). Equation 8 expresses the
solution to Equation 2 in tem1s of the eigenvalues and cigcnvcc1ors of A. The c;
in Equrilion 8 depend upon the initial conditions .l' 1(0). x 2 (0) ... . , xn(O).

Example 1:
Solve rhc sy::.1cm of equations

.i·2 = Jx 1 + 7x 2 - 9x;
,i-3 = 2.t-2 - X3

SOLUTION: Let X1(.r) = /I je>-- 1 10 obtain


- ).. 0
3 7 - ). -9 =0
0 2 -1- >.
(See Equation 6.) The three cigcn\'alucs arc ), = I, 2. an<l 3 and lhe
corresponding eigenvectors are

According to Equation 8, the gencrJI solution is


474 Ch.,pll'r I O ' Marri · · nd ig nvalu Prohh•m,

Example 2:
The radioactive decay of a certain nuc.:leu~ :,atisfics the equa1ions

dA
- =-k 1 A
dt
dB
- =k A -
dt
1 kJB
-

dC
dt
= k~B

= 2 and k2 = I with the initial conditions


Solve thi!- :set of equations for k 1
A(O) = Ao and 8<0J = C(()) = 0.
s O Lu TIO N: We write these equations in matrix natal ion

The cigcm-·.a\uc,; and corresponding eigenvectors of Mare >. 1 -2. ),2 -1. = =
= = = =
A.J 0 and u 1 (I. -2. I) T_ u2 (0 . -1. I) T_ U;t (0. 0. 1/. Thus, the
gcncr,i:I wlution is

Ao Using A(O) = Ao and 8(0) = C(0) = 0 gives c 1 .,_ A0 • c:,. = -2A 0 . and
c 3 ::::; Ao, so 1hat

A(t) = Aoe-2,r
B(t) = 2A 0 ( e_, - e·- 2.J)

C(r) = Ao( I - 2e-, + e- 21 )


Figure 10.J shows A(t). B(t). and C(t) ploncd ;1gai111~1 t. Can you explain
the shape or 1he curve B(t)?

Figure 10.3
The !,olu1ion.~ 10 rhc radiooctivc dec·ay mlc The examples 1ha1 we have considered so far have had real eigenvalues . What
IXJlllllions in Example 2.
if the eigenvalues come oul
10 be complex?

r hample 3:
Solve the system of equations
10.3 Some- Apr>lied Eigenvalu Problems 475
SOLUTION: Let x = u e"- 1
lo obl.:t.in

The eigenvalues and corresponding eigenvectors are -2 ± i J3 and


u 1 = ((-1 - i J3) /2. I). u 2 = ((-1 + i J3) /2. lJ. Proceeding formally. f
the :-;olution is

We can use Euler's formula <e"1 = ms O + i sin 0) 10 write x as


lf
x = k1u1 + c2u 2k- 21 cos ✓3 r + i (c 1u 1 - c2u2 )e-
21
sin J3 t
Figure 10.4
= a c- 2, CO!- J3 t + b e- 2' sin ✓3 r
An illustration of exponentially damped
harmonic behavior. llli; function t·-, co~ r
Thus. we sec that complex roots lead to exponemially damped hannonic is ploucd against 1.
behavior (Figure I0.4 ). We· 11 discus~ this type of behavior in more detai 1 in
the ncxr chapter.

Although we illusr.rnted this approach with 2 x 2 and 3 x 3 sys1ems. the size


of Lhe sys1em is really irrelevant, particularly wlt.h the CAS Lhat are currently
availahle. (Sec Problem 21.)
Vibrating or oscillating mechanical systems can be fomrnla1ed as eigenvalue
problems. Our first example will consist of lwo particles of ma,s m connected by
three identical springs of rela"<ed length / and consLrained to move horizontally
as shown in Figure 10.5. We shall displace the masses from their equilibrium Figure 10.5
posit.ions, then lel I.hem go and investigate 1heir subsequent motion. If x 1 and x 2 Two particles of mass III connected by
thn..-e identical sprin[!S of relaxed length/
denote small displacements of the two particles from their equilibrium positions. und con.<-tr.iincd to move longitudinally.
then the potential energy of lhe system is given by

k k? ? k ?
V = -x - + -(x, -
2 1 2 -
x 1)- + -x;
2 ~
(9)

where k is the force constant of each spring. Newton ·s equation for each panicle is

d 2x 1 oV
m -- = -- = k(x, - 2x 1)
dt 2 Jx, -

d 2x-,
m --:f
a-
v == k(x -
= -8x 2x2)
1
dr- 2

Write Equation I O in matrix notation:

d 2x
m-? =Ax (11)
dr-
476
where

A= (-2kk k )
-2k
( 12)

We know that this system osci II ates in lime, so we· 11 assume that x:( r) = u e'w.1.
Substi1u1ing this expre~ion into Equarion 11 gi s

k ) (")
-2k u~

Thus. w:. turns out to be given by

2k - mw2
-k ~ I =0 ( 13)
1 -k 2k - mw~

and so we find two charac1erisricfreq11e11cies

k ) 1/2
W1= ( - and (14)
m

with corresponding eigenvectors

and U1
-
=( -{I
ll) ( I 5)

where a is an arbitrary (possibly complex) consLanl. The moLion of lhe two masses
is given by

( 16)

or

( I 7)

where b 1 = ac 1 and b 2 = ac2.


Equa1ions 17 say that the ma.,,cs will vibrate in phase if b~ = 0 (b f. 0)
1

and 180° out of phase if b 1= 0 (b 2 'I- 0). To prove rhc~c las1 ~,a1emen1.s. write
b 1 = A 1ei 4' i. so that the real part of the solutions associa1ed wi1h frequency w 1 are

and (18)

Equation I R shows that the two ma.-;~c, vibrate back and forth in unison. Similarly.
write b2 = A 2ei~ and find thal the real part of the solutions associated wi1h
I 0.3 Sorn , App'lied Eig nvalue Prob l 111s 477

frequency u12 are

.r 1(f) = A 2 cos(u, 2r - rj>2)


( I 9)
.r 2 (1) = -A 2 cos(~/ - <t, 2) = A 2 cos(lV-_21 - </J 2 + n)
Thus. in this case. the two masses vibrnte in opposite directions.
Equations 18 and 19 are called rhe ,wrmul mode.\· of vibrntion of Lhe two-
mass sys1em. If the initial conditions arc just so. the system will vibrate either in
phase with a frequency w 1 or 180° out of phase with frequency w 2 (Figure 10.6)
(Problem I0). Usually. however. the motion of the system will be de~cribed by a
supcrposi1ion of the two normal modes according lo

and
Figure 10.6
An ii lu straiion of the nom1al mode:.- of the
system shown in Figure 10.5.

The four constants. A 1• A2 , </., 1• anc.l cp 2 , can be de1crmined from the values of x 1(0).
x 2 (0). .i- 1(0). and .\· 2(0).

Example 4:
Figure 10.7 shows two pendula of length/ constrained to move in a single
plane coupled by a hannonic spring with force constant k. Problem 9 ha., you
show 1hat the e4uations of motion for small displacement:- f mm the venic:al
position are

Figure 10.7
Two fi)\!"ndula of length / coupled by a
hannon ic: spring anti cons1 rn.ined to move
and in a sing le plane.

where s 1 and s~ measure lhe distances of lhe masses along their arc of motion
from their vertkal positions, Solve for 1he char.:i<:tcristic frequencies and the
normal modes of this system.

SOLUTION: The equations of motion in matrix form are

C
478 Chapter I O / Mar ri ce- and ig n aluc Problem~

Substitute s = uei".>1 into the~e equations to get the eigenvalue equation

-mg + k - mw
2
-k
I
=0
-k mg+ k - mw 2
I

which gives w 1 = (,:./ /) and ru 2 = (g/ /) 1n( I + 2k/ /m>:) 112.


112
(a) For w; = g /I, the eigenvector is given by

or 11 1 = "2· For w 21 = g( I+ 2kl/mg)/ I. we get 11 1 = -u 2. Thus, lhe two


nonnal modes for th is system are
~
(b)

Figure 10.8
An illus1ra1ion of the rwo norm11l rnod~ of The motion corresponding 10 rv 1 is 1n phase and 1ha1 corresponding to W::! is
1hccouplcd pcndul11 shown in Figure 10.7. 180° out of phase (Figure I 0.8).

Eigenvalue problems also ari~e in t.he analysis of elecrrical circuits. Consider


the circuit shown in Figure I0.9. Using the fact lhat the sum of the voltage drops
around each closed loop is equal to zero. we have

d/1
L-+R(l 1 - 1-'J) =0
R dt -
(20)
-1: f 12 dr+R/ 1 + R(h - / 1) =O
Figure 10.9 Differentiat..ing the second equation wilh repsect to time gives lhe pair of simulta-
ll1e electrical circuir described by neous linear equations
&1u1jlion 20.

Ldl 1 +R(l 1 -1,)=0


dt -
(21)
l,+cR'"2+cR(dl2 _d/1)=0
- dt dt dt

Before we go on. we can make Lhese equations look cleaner if we choose to mea-.ure
lime in units of CR. ff we lei r =,/CR. Lhen Lhese equations become

L d/
----
2
1
+ /1 - /-,=O
C R dr -
(22)
l,+2dl2 - d/1=0
- dr dr

Notice 1ha1 Equations 22 coniain only one apparent pa.r.imeter (the CR is absorbed
into r ). We can write Equations 22 in matrix notation as follows:
I (U Som Applied ig rwalue Pr blem 479

(
l/C R2
-I
0)2 I+
· (0I (23)

where 1hc dot rcprc.-~1:nts differentiation with rcspcc1 to r = r / RC . If we let


I = u I:';_,. we obtain

(
a).
-'J..
0)
2). u + ( 0I -1)
I u=O

or

a).+ I
( -,\
-I
I+ 2J.
) u = (). (24)

where a= L/ C R 2. For concrc:1enes$. take C = 20 microfara<.ls. L = 100 milli-


henrys. and R = 50 ohm .... s11 that a= 2. In 1ha1 case. we have the condition

2A+ I -I ) u=O
( (25)
-1 I+ 2>.

which gives A± = (-3 ± i ./7 J/8 . .so tha1 I = u ,, ' 1 e ·i 1 r / X. Thus. ,, l! ,t:t: that
the current will undergo damped harmonic motion.
No1icc that 1hc equation of the eigenvalues in this case is nol of 1he fom1
I A - )../ I = 0 because the coefficient matrix of i I and i2 is not diagonal. We could
have manipulated Equa1ion 2J into the form i =AI by multiplying by the inve~e
of the matrix multiplying i in Equation 23. Of course. we mu$1 end up with lhe
same resuh ( Problem 19).
We have presented just I wo diffcrcnt examples or the :ippl ica1ion of eigenvalue
problems in 1his .-.cction. but 1hey occur frequently in various physical problems.

1 O.' · Pro blems


I. Show th31 Equ::iti,on 8 j,- u solu1ion 10 E!juarion 2.

2. Solve the simullanc.o us cquali(lns


.fl = -X1 -;- .r~
.i·~ = -Jr1
by a:,;suming 1ha1 .rjCI) = 11/,i.r_
3. Solve the simultuncous cquu1ions
.\'I = X1 + X2
.i-_ = 4x 1 +x2
4. Solve the simull:meous equation~
.i 1 = 6x1 + 8.r2
481

Figure 10.11
A double pendulum. One pendulum is
,us:pended fmm rhe other and ooth are
cons1rnined 10 mun~ in 1hc same vertical
plane.

18. Show Ihut the total energy associated with lhe normal mode of lowesl frequency of the system shown in
Figure I 0. 10 is con~rved .
19. Com·cn Et:1uation 23 into 1hc fonn i = Al am.I :,;how thal the eigenvalues of A are the same as those we obtnin
from Equn1ion 24.

20. Figure I 0.11 -~hows a double pc:ndulum. which wnsists of one pendulum suspende<l freely from another. with
both consrrained 10 swing in the same venical plane. The equations of motion for smaJI amplitude arc

where M = m 2./(m 1 + m 1 ) and I is Lhc length of each pendulum. Show that 1he two characteristic frcqucncie.-.
of thi-. :-ystem are given by w 2 = g/ /(I± JM).
21. Use any CAS to solve the simularnneous equaiions

1 0.4 Change of Ba is

Up to this point. our linear trJnsformations have ac1ed upon vecton; to produce
new vecrors. all with respect to the same basis. For concretene~ and simplicity.
we have used the standard basis in R'1, e 1 = (I. 0 ..... O/, e 2 = (0. I, ... , 0) T.
• • -. en == (0. 0 . . . . . I) r 1hroughout. lt is often convenient to Lhink of the linear
482 C h,tp!r•r 1(l : .\-1.itric ,- nd EiRcnva luc Pro! I m s

1.ransfom1ation as acting on the basis to produce a new basis. Consider the quadratic
fonn

j(:r, y) = x~ + 4xy + y 2 ( I)

A quadr.uic fonn in 11 variable), i), an expression of the fom1


,, '1

Q(x) =L L a,jx;x 1 (2)


i=I j,:I

con1aining bo1h square 1em1s and cross producls or1hc ,1 variables . We can express
the quadratic form in Equation I in matrix fonn:

J(x, y) = xT Ax= (x y )( l
2
(3)

as you can verify by direct calculation. If we plol 1he equaLion

= (x y) ( ~
)'
J (x, y) ~) ( :: ) = x'.! + 4xy + y 2 = I (4)

we see I hat i1 rcprcscnrs a hyperbola ( Figure I0. 12 ), The occurrence or 1he cross
term 4xy tilts the hyperbola al an angle with respect to the x and y axes . It would
appear from Figure I0. 12 1ha1 if we were 10 rota le I.he axes Lhrough :;ome yet
X
umktem1ine<.I angle, 1hcn 1he plot of the hyperbola would look like rhc one: in
Figure I 0.13 .
We· II refer to Figure I0. 14 to derive the tr,msforma1ion nf coordi natc axes that
results when we rotate the .r. y axes in a countcrclockwisc direction through an
angle fJ . ff we denote the fixed vector by r' when we express it in the rotated (lhe
primed) coordinates. then according 10 Figure 10. 14,
Figure 10. 12
The hypcrbolu d '• c:ribt:d by the cqua1ion
x 2 + 4x y + y 2 = t. r' = x' i' + y' j'
= r cos(u -f}) i' + r sin(a - O)j'

= r cos a cos 0 i' + r sin a sin & i'


+ r sin a cos 0 j' - r cos a sin 0 j 1
= x cos 0 i' + y sin 0 i' + y cos tJ j' - x sin 0 j'
x' where we have used lhe fact that x = r cos a and y == r sin a. Thus, we have

x' = x cos 0 + y sin 0


y' =y cos 0 - x sin l:J

We can ex.press the relation between x', y'. and x . .\' in matrix fom1:

(<)= (
Figure 1 0.13
The hyperbola 1h;t1 resul1s from ro1a1ing c~s 0 sin H ) ( x )
the ..r. ,\' a.."cs in Figtm I U.12 . y . - sm 0 cos U .r
483

\'
r'

R(O)

Figure 10.14
An illu.s1rn1ion of rntari "!' the x. y ax~
cotm1~·rd<i...·k1.1, i,c through ;111 a11glC' + J.
The vccior r' is fixed.

Notice that 1hc malrix in this c:.u.c is R ( -0) from ScC"tion 9.3. where R (0)
is the matrix I.hat rotate:. r keeping the axes fix.cd. Ro1a1ing r 1hrough an angle
+fJ is equivalent to rotating 1he axe_.; through an angle - 0. Thus. Equation 5 c:.m
be wrincn as x' = R(-0) x. Bui R(-0) = R- 1(0). and R- 1(0) = RT(0) because
R(0) is or1hogonal. Solving Equation 5 for x in terms of x' gives x = R(0)x'. If we
sub~titute this result into

f(x . .,·) = X TAX= I

we have

(6)

= (.r , .\'),. (
-
cosli
.
Sin f-i
sin !7 ) ( I
cos f) 2
2) (
I
cos li
sin fJ
- sin H ·) ( ._,r.:)
cos 0

_ ., ., ( l + 4 cos fJ') sin• fJ . 2 cos 2 I:) - 2 sin~ fi ) ( x )


- {.\ I ) --.
· .t. co~- H - _ sm 0 1-4cos/-isinfJ y'

= (x i·)
. , (I+ 2sin W 2 COS.2fJ
I - 2 sm 20
) ( x', ) =x ,T Ax
_,.
, ,
· 2 cos 20

= x '1 2.x' 2 sin 10 + 4x'y' cos 20 + y r2 - 2y'~ sin 20 =I (7)

Note that we u~c the no1alion t(x'. y'l 10 rcprcscnl f (x. y) whc11 .r und y arc
eliminated in terms of x' and y'.
If we cl imi natc 1he cross term in Equation 7. then g (x'. y') wi 11 he oft he form
g(x'. y') =,a•.!..:.._ b_r' 2• which i~ the standard form of a conic section. We can
clirnin::ilc rhc cro~., 1enn in Equation 7 by choosing 0 such lhiJt cos 2fJ :::: 0. or by
choosing Ii :...:: rr /4. In this case. Equation 7 becomes
., ~

g(x'. /) = 3x'- - y'- =I (8)

It's ca:-.y to sec now that g(x'. y1) = I is a hyperbola,. wl,1crcas. the na1urc of
x 2 4xy + y 2 = 1 i!- not at all clear. The equation g(x'. l) = 3x' 2 - y' 2 I is =
484 Chapwr 10 / M,11ri c s nd Ei~:1c•nwilue Prubl 1•flh 1

)'
plotted in Figure I 0. l 5. and you can see thal it is simply the same curve as in
Figure I0. I I but plolled in a coordi natc system I ha! has been rotated by 45°.
The relation between rhe original coordinates and 1he new coordinates is given
=
by Equation 5 with 0 rr /4. or

x' ) I ( I I) ( x )
X
( _r' = J2 -I I r
or

(9)
Figure 10.15 .
' - I( - x
v:::;
-/2 + .\')
The hypt:rhola dt:SL'riocd by the equlllion
/(.r, y) = .lt2 - y~ = I.
If we subs1in11c Equations 9 into Equnlion 8. you get Equation 4. We sec that rhe
quadroric form in Equation 4 has only quadratic terms (no cross 1cm1 s ) if it is
ex prcs~ed in 1cnns of (x'. y ·) in~IL'ad of (x. y). We wi I.I develop a more efficient
way of dc1em1ining coordinates like (x'. y') in the next section. but the message
here is that ii is often desirable 10 transfom1 coordinate systems. or basis se1s into
new bas[s se1s.
We can summarize !he above procedure by writing x = Rx'. where
R:::;;: ( cos o - ~in fJ )
sin 0 cos0

Now substilu\e x = Rx 1 into

y 10 obtain

(I I)

Generally, g(.r'y') will contain a cross term whose coefficient depends upon A. as
in Equal ion 7. Now choose O so that there are no cross tem1s in g (x'. _v'). This
x amounts lo choosing R such that RT A R is diagonul.

Example 1:
De1cm1inc the rotation of coordinaie ax.i.:~ 1ha1 clirninatc:s the cross rcnn in
rhc equarion
Figure 10.16
A plot of9x 2 2 J3x y ' Jy - = I.
~howing 1ha1 its grnph is nn cllip ·c who,~'.
maj xis is not aligned wirh either
rd innte . i• . ( Scl' Fit:',urc I 0.16.)
10.4 ChangeofB i~ 485

SOLUTION: First wrire the L'qUUlion in matrix fonn:

Now use &1ua1ion 6. where

A'= R1 AR= ( co~ l:i sin 0) ( ./3 9 - ·in0)


- sin 0 rnsO cosO

_ ( 9 cos~ r, + 3 sin 2 I! + 2Ji sin O cos 0 J3 cos 2 fJ - J3 sin 2 0- 6 ~in fJ cos 0)


- .fi cos 1 0 - J3 :-in~ 11 - 6 <:in 0 cos fJ 9 cos 2 l:J + 3 sin~ 0 - 2J3 sin 1.1 cos fJ

= ( 3 + 6 cos~ 0 .J3 sin 20 JJcosW - 3sin2A )


J1 CCI). YI - 3 sin W JJ sin '2.V
3 + 6 sin 2 () -

Sc1 JJ co, :JI - 3 sin 20 = 0, or tan "!.IJ = ./3/ 3. to see that W = -n /6. or
0 = n .' 12. Substituting ,, = ~ / 12 into A' give~
y
A'= (9.4 641 0 )
0 2.5359

so that

g(x'. y') = 9.464 Lr'~ + 2.53.59/~ = I


.r
This cq11::i:t,ion is plont!d in Fig.urc I0. 17. The t:r.1nsforma1ion that produces
this re_ uh is. given by Equution 5 wit.h 8 = ,r / 12.

(.r'y' ) -_(-0.2588
0.9659 0.2588) (
0.9659
.r)
y

or Figure 10.17
Th(' figuf(• in l"Urc 10.16 r111a1t:-dl 1hn,1 u~h
x' = 0.9659.r + 0.2588y 15 . o that ii :l..'\e · are aligned w11h !he x
axis and 1hc y axi,. .
/ = 0 .258 'r + 0.9659.r

We"ll n0\\ ~pl'nd the rest of thi:- section discu~ing ):!.l'ner..il linear 1ransfom1a-
1ions of ba,i~ ~er.,. or coordina!e systems. There are ma.ny instances where it is
convenient 10 analyze a problem in one ha-.is. but 10 repo11 the results in another
ba,is. For example. in studying the dynamics of a rotating body. it is mos1 conve-
nient to use a coordinare system 1hat is rotaling wilh the body, but you might wunl
10 express your result in the coordinaw ,y!-tcm of a fixed observer. Or. if you arc
analyzing rhc dynamics of rhc collisions beLween partfrles. ii is conveninel to u.se
ccntcr-of-mass coordinates. but you would repo11 your results in labora1ory-fixed
coordina1cs.
Tht: only ba,i, ,c.:1. 1ha1 we have dealt with so far is the standard basis, c 1 =
CI, 0, . . . , 0)\ c 2 = (0. I. .. .. O)r. - · ·. e,1 = (0. 0 ..... l)T in V = R'', Now let
486
lz; J be ano1hcr basi~ in V. This new basis can be expanded in tenns of lhe slandard
basis

z,. = I: :.:::jiej i=l.2 ..... 11 ( 12)


}=I

A vcc1or u can be cx.pn:~sl'd in this ba~i~ :1~


II

U.
~. = ~
L ,
/l. 1Z; (13)
i=l

\vhere we subscript u wirh a:.: to emphasize tlrnl we are expanding u in the ;:-b~i~.
We can ex.prcs.-. u expande.d in Lhe standard basis le j I as

f)

u£ = L uEjeJ ( 14)
j=I

Realize 1hat uE and u: represent the very same vector. u. expressed in tcm1s of
different ha.-.cs. For example. Figure I 0.18 shows the same vector in two coordi nnrc
systems that differ from each ot.her by a rotalion 1brough +60°. In Figure 10.18.
Figure 10.18
The same \'CC tor in rwo coorclinnh::
the componcms of u in the { z_; I basis arc related to 1he components of u in rhc / eJI
sys tem s (ba.scs) I.hat differ by a rotaLion basis by
Lhmugh +60°.

(":1)= -~J3-
/l.1
( 1) 2
-
2
I

(Problem 6).
We'd like 10 know in general huw the components of U: are related to those
of u£. Substitulc Equation 12 into Equation 13 to obtain

( 15)

If we compare lhis result with Equation 14. we see that


n

"EJ = L ~jill~,- ( 16)


i::ad

We can express this result in matrix notation by writing

(17)

By referring Equation 12, you coin see 1ha1 the ith wlumn of Z consists of 1hc
10
components of Z; in the standard basis. We can express Equation 17 in a more
=
..;ymmetric form by writing I uF. Z u:, where l is the unit matrix.
487

Example 2:
Supposewehaveabasisz 1 =(1.0.0)T.z 2 =(1. l.0) 7 .andz3 =(1, I. l)T.
Detennine u. if U£ = (l. 2. 3)T .

s O LU TIO N: The malrix Z in Equation 17 consis1s of the ,·olumn vccwrs


z 1• z2 • and ZJ, so

According 10 Equation 17. u~ = z- 1u£, and so to find u: in 1crms of ur.. we.


need z-1• which is

-1
1
0

Therefore.

Let's check 1his result. Wc'vcjus1 :;hown that

Now substitule z 1 = e 1• ~ = e 1 + c'.!, and z 3 = e 1 + c2 + e.1 into lhis result


loobiain

Example 3:
Show that u1: and u~ in Example 2 have the same length.

SOL u TIO N: Bec:rnse u£ ii- cxprc$~cd in tcnns of an onhononnal ba,;is.

=1 2 +2:?+3~=14

so 1ha1 Lhe (Euclidian) length of u£ is ( 14) I/~. No1c. however. that 1.hc: basis
is nnr ort.hogonal. Nevertheless.
488 Ch,lph~r 10 / M, lri .:ind Eig rwaluc Prr,li!L·111,

= Z) · Z1 + Z1 · Z~ + 9 Z3 · Z3
+ ~ Z1 · Z2 - 6 Z1 · Z3 - 6 Z_? · Z_l

= I+ 2 + 9(3) + 2l I) - 6( I) - 6(2)

= 14
so that 1hc (Euclidc.an) length of u:. is ( 14) 1 ~. Of course. u F. w,d U: must
huve the same length because they are the same vector expr~,.,ed in different
coordinate sy:-rems.

We ea n genera Ii ze Eq ua Lion I 7 to r.he case where u is ex pressed in terms of l wo


(arbitrJJ)') coordina1e systems z and w. instead of in the standard basis £ and one
(arbitrary) coordinate system. Ir l"°Jl is yet ano1hcr basis in V. then Equation 17
becomes

( I 8)

where 1he ith column of W consists of the components o( w1 im the standard basis.
Thus. according to Equation 18, if lz,) and {w j] arc any two ba,i~ : et-. in, \I, then

( 19)

where P = z- 1 VI/ is a linear transformation that rela1es u: 10 u ....


One la..~t topic: Suppose we have a transformation mulrix A relative to some
given basis, nnd we wish to detennine Lhe fonn of A relative to some other basis.
In other words, suppose we have A in a basis lzj}, so 1ha1

v.=A.u.
- . ..

(Lhe z. subscripts here are just for emphasis) and we i ntroducc a new basis I w 1 )
rclared to \z 1I by

and (20)

as in Equa1ion 19. Then,

(21)

where A,, = p- 1A~P. Therefore. we sec 1hal

(22)

where Pi:- defined lhrough Equation I 9. Thu~. if we can conslfuct a malrix in one
ba.1;is (such as the standard basis). we can coosrruct the matrix in aoother basi:-. ifwe
I 0.4 hangc o i 13,1-i, 489

know the linear transforma1ion that relates the two ba5is sets. The transformation
depicted in Equation 22 is called a similari1y trm1.\fon1101io11 and two marrices that
are related by a similarity transformation arc said to be similar. We shall see in
the next section I.hat similarity 1ram,formation~ play a central role in transforming
matrices into diagl>nal fom,.

Example 4:
Given the representation of a mauix

in the standard basis. determine A: in lem1s of the lw,i, in Example 2.


SOLUTION: We f'irsl wrile

Equal.ion 18 gives us uF. = Z u: and ,·1. =--' Z ,·~· TI1ereforc.

Using Zand z- 1
from Example 2, we have

= (~ -: -~) (
0 0 I -I
~ 0
O
:) (~
O O I
I :)

-(-1 -2 -2)
- 3 3 3
- I O I

No1ice that the detem1in_anL,. of Ar: alilu A.- ar,e e4ual (Prohfcm 23).
In Lhe next ~ec1ion. we"l 'l learn how to transform quadrati'c fom1s into canonical
fom1 by using the eigenvalue.s and eigenvectors of A.

1 0.4 Problems

I. De1crminc the rotation of coord1na1c axes 1h::it climinate·s 1he cro5~ 1crn1 in the cqunrion .f (.r. y) =
2x + 2.ry 2y! = 1.
1

2. Deu:nnine the rot:itinn of coordinate axes 1ha1. ellimi11;·11c thl! cro~-~ tcm1 in the qundrntic form f(x , y) =

al
490 Cha p! r I O / Matric and Elg nvalue Problems

3. Show that if f (x. y) = (x y) ( : ~) ( ~::). then a roration of the axes 1hrough nn angle 1hnt satisfies 1he

.
relatrnn tan 1J:) = - 2-b ·11 e 1·1minate
w1. . I cw~!-. term .tn f (x. y).
tie
a- c
4. Derermine the ro1ation of coordinate axes 1hat climinotcs the cross term in the equal.ion 1r 2 + 6xy - 4y~ = I.
5. Derennine the rotation of coordinate axes 1ha1 eli min::ucs the cross rcnn in the equation 1r 2 + 2 J3 x y + y~ = I .
6. Show that the components of the vectors 1n Figure lO. J 8 arc rclarcd by

7. Suppose that we have a basis se1 z 1 = (I. 0. 2/. z2 = (-1. I. 2) r. and z 3 = (0. I, OJ T wi1h U: = (-1. 2. I/ .
Find u£.

8. Show 1ha1 D:-. and uE in the previous problem have the same (Eutlidian) length

9. Referring to Problem 7. calculate u~ given th:ir U£ = (-3. 3. 2)1°.


JO. Show 1hat u= and uE in the pre\-'ious problem have I.he same (Euclidian) length.

11. Suppose we have a basis z 1 = (2. -4)T and Z2 = (3. S)T. Given uE = (I. I) r_ find u.:·
12. Show thn1 u~ and u£ ,i n lhc previous. problem have 1hc same (Euclidian) length .

=
13. ConsiLlerthe 1wo basi s !-cL" e- 1 = (I. O)r. e 2 = (0, or. and z 1 (2. 1) 7 • ZJ '-' (-3. 4/. (aJ Find the matrix that
will express u; in the ba~i~ ~er e 1• e~. (b) Find I.he matrix tha1 will cxpre~.-. u£ in the basi, scr z 1• z 2.

14. Suppose1ha1u£=(1.2, l)T _Findurelativc101hcba..,issetz 1 =(1. I.0) 1 .z 2 =(1.0, 1)1 ,andz 1 °,(I. I. l) 1 .

15. Consider two basis sets. z 1 =(


I. I. O)r. z2 =(
I. 0. l)T. and z =(
I. I. l)T. and w 1 (I. 0. O)r. w 2 = = (I. I. O)T_
and w3 =(
I. I. l)T _Dctcnninc the ma!Tix tbar transfonns u in the r. ba.-.is to u in 1he w basis.
16. Find Lhc coordinates of u1: = ( I. I. 0) T relative to the basis z 1 = (I. I. 2) T. z2 ::.: (2. 2. 1/. and z3 :::. ( l. 2. 2) T_

17. A certain marrix A 1ran~forms u 1 = (I. 0. l)T inro (2. 3. -l)r. u2 = (I. -1. I) r inro (.l 0. -2)T, and
u 1 = (I, 2. - I) 1 into (-2. 7, -1) r_ Determine the representation of A in Lhe standard basis.

18. Let A= ( ~O -~) be a mauix expressed in the standard basis and lei w 1 =(
I. 0. 0) r_ w 2 = (0. I. 2) T.
I2 I
and w 3 = <O. 0. 1) T he onolhcr ba-;is. If u E =(
3. 0. 2J T. find the vec1or A uE. in the w basis.

19. Referring to Problem 18, find Lhc tran~fonnation Uu, =8u 11 , corre.sponding to,. £ ·= A uE.

20. Consider \WO ba.."is sets of an ,r-dimensional vector sp.ice z1• z~ . .... Zn ,md w 1• "°:?· ... , wn. If Ai~ a ma1rix
reprc~cnia1ion in the z ba~i-:. show 1ha1 its representation in the w basis is A,,. = vv- I 2 A.:z- 1w.

21.lfA_ -(2,,_I ~ 21 :!.


:~) is a matrix rcprescnta1ion i.n the : basis of Problem 15. determine the corresponding

matrix representation in t11e w basis of 1hn1 problem. Use 1he resuh of rhe previous problem .
10.5 Diago na lization of Ma1rices 491

22. If A= (oil ~ 0:) is a matrix representation in the basis "'i = (0. -1. 2)T, w 2 = (4. I. 0)1, and

WJ = (-2, 0. 4)T. find the corresponding matrix in 1hc: basis z 1 = (I. -1. l)T. 22 = (I. 0, -l)T, and
ZJ = (] , 2. l)T_

23. Show that the determinants of AE and A, in Example 4 are equal to 3.

·r 0.5 Diagonalization of Matrices


Consider the eigenvalue problem Au j = A1 uj in which A is II x II ond j =
l, 2 .... , n. II turns out that u matrix whose columns are the eigenvectors of A
has a very useful property. Let
(I)

l
where the jth column of Sis LI_r- More explicitly,

II 21 ") I 11

[""
II 12 1122 '')2 11,.2
nI
(2)
s = "( "'.!J U33
":')
II 111 II 2u 1.13,,, 11 11n

Now nolice that

(3)

0 0
(Problems I and 2). The matrix D is a diagonal matrix whose elements are the
eigenvalues of A.
If the 11 eigenvectors of A arc linearly independent, then S is non-singular.
Therefore. s- 1 exist,; and we can multiply Equation 3 from the left bys-• to obtain

(4)

Thus. we see that s- I AS yields a diagona.l matrix whose elements a.re the eigen-
values of A.

Example 1:
Diagonali7..e

A= (-3 2)
-I 0
492 Ch,1plc·r IO / t-.l.1!r1t 1•~ ,md Ei genvalue Pmhlt•ms

SOLUTION: The eigenvalues of A arc ). 1 = - I and A. 2 = -2 and rhc


corrcs(X>nding cigenve('IO~ arc

and

where a and b arc arbitrary conslanL.;, which for convenience we'll lei be
equal to one. Therefore.

and

and

-2
0) = D
Notke 1hat the elements of Dare the eigenvalues of A.

As we said at the end of lhe previous section, two maLrices A and B 1hn1 are
related by a rel:u.ion such as

(5)

are said 10 be similar and Equa1ion 5 is said to be a similariry 1ra11sforma1io11. A


matrix S. who~ columns are lhe eigenvectors of A. is called a modal matrix of A.
We shall now prove that similar matrices have the same characteristic equation.
and consequently the same eigenvalues. If A and 8 arc two similar matrices. then

Take the de1erminant of these matrices and use the prope11y that det AB =
(det A)(det B) 10 obtain

(6)

Thus. A and B have the '-:tme characteristic equation. Write 1he charactcris1ic
equation of B and A as

Mul1iply out 1he ld1 side of this cqu;:it.ion and compare like powers of), 10 obrain
10.5 Diagona liz,11irn1 111 M,11, ii 1•~ 493
f:J l = A1 + ). ~ + ... + >-11
fh = A1A.2 + A1A3 + · · · + ,\n-lA,1
(8)

The fjs do nol change under a similarity transfonnation and arc said to be invoriams
of the similarity transformation. Of particular interest are {3 1 and /3,,. which are
!.he 1racc of a ma1rix (the sum of its mr1in diagonal terms) and its dc1cnninan1.
respective! y.

Example 2:
Show that Tr (AB) = Tr (BA).
SOLUTION:

,,
Tr (AB) =L L ti;/J ji
i=I j=I

n /1

= LL hjiaij = Tr (BA)
j=I i= I

Example 3:
Show explicitly tha1 the trace of a malrix is invariant under a similarity
transfonm11ion.

SOLUTION: We wish 10 show that Tr cs- 1A S) = Tr A. Using the facl


thal Tr (AB) = Tr (BA) from faample 2. we ha\'\:

We 're going ro sec a number of applications of the diagonnlizar ion of mar rices
in the remainder of this chnpter. As our first application. consider 1he rate equations

tlxi= - 3X1
- + 2X,
dr ~

C gl
494 ChJplcr I O / M aLri • nd E,g nvalue Probl m

which we write in matrix. form as

or

dx
-=Ax (9)
d1

1.n tbe one-dimensiona.l case. the solu1ion 10 Equation 9 is x(r) = x(O)e 1


". It is
tempting to write 1hc solu1ion to Equation 9 in genernl as

which would be useful if we can find a meaningful expression for eA 1 •


We can actuaJly define a function of a matrix through its Maclaurin expansion.
If f (x) has a Maclaurin expansion
00

f(x) = L CnX,i
11=0

which converges for Ix I < R. 1hen the matrix series


'-
f (A) = L 1.·,, A'' ( l I)
11=0

converges and is a well -<leflned function of A. provided 1hn1 A is square and each
of i1s eigenvalues has absolute \'alue less than R. Now. if D =s- 1AS where Dis
diagonal. I.hen A= sos- 1 and A" is given by

( 12)

Becm.1..<-~ Dis diagonal, 0'1 is simply each diagonal element raised to the nlh power
(Problem 20).
Lei's apply Lhis result to Lhe matrix in Equation 9. We Jetermined Sand s- 1
for A in Example I and fouod that

D=(-10 -20)
Using Equation 12. we c:m write
495
and so

is the solu1ion 10 Equa1ion 9. Al1hough Equation 9 is just a 2 x 2 system. our


mc1hod of solution i~ ea~ily ex1cnded 10 11 x II systems. Must CAS have routines
lo find eA (Problem 21 ).
Realize 1ha1 1hc cigenvcc1ors or A musl be linearly independent in order
to diagonalize A. We saw in Section 2 that if an II x II malrix has n distinct
eigenvalues. 1hcn its II eigenvectors are linearly independent. Consequently. we
can say that

(( 011 11 x ,, 11wrri.r haJ II disti11c1 ci~t.·m·alues. tltt'II ii h diaKn11ali:.ablc: hy a


similtlriry rra11.1:(ormario11 s- 1AS, 1rliol' 1he columns of Sare rhe eigc,wecrors
tlA.

The eigenvectors of a matrix wit.h degenernre eigenvalues may or may not be


!~nearly indcpenden1. For example. the matrix.

A= (13)

ha,,; eigenvalues).= I. 2. and 2. and corresponding eigenvectors (-1. -1, 1) 1 .


(2. I. 0) 1 • and (0. 0. 0) T_ These eigenvectors arc not linearly independent. so A in
Equation 13 cannot be diagonalized by a similarity 1ransfoITI1ation. Another way
to see Lhis is lo realize that if the cigcnvecwr."- are not linearly indcpendcn1. then
ISI = 0 and s- 1 does not cxis1.
We .c.aw in Section 3 1hat the eigenvectors of a symmetric mntrix (or a Her-
mitian matrix in complex vec1or space) are nor only linearly indcpcndenl. but can
he made 10 he orthogonal. even if rhc ma1rix h.is repeated roo1s. Consequently. we
can state !hat

A symmelric or r1 Hermitian malrix A is dit1go11a/i:able hy ,1 similarity


tra11sfor111a1io11.

Furthermore. if we normalize the. eigenvectors of A and then construct the modal


matriit of A (which we"ll call a ,10n11ali2nl modal matrix), then

5-1 =51 ( 14)

In other words, S. the normalized modal ma1rix of A. is orthogonal.


496 Chapl-er 10 / M arri , nd Eii,:Pnv, luf'· Prohl f' rT1

Example 4:
Firs! show Lhal the nornrnlized modal mnrrix: S of

is on.hogonal, and 1hen diagonalize A by a similarity lransfonnation.


s- 1AS = sr AS.
SOLUTION: TI1ccharac1eris1iccquaLion of A is A2 - 25 = 0. od. 1 = 5 ::ind
). 2 = -5. The co rresponding normalized eigcm•cctors arc u 1 = (2, I) / J5
1

am.I U1 = (- I. 2),T / ✓5. The normalized modal marrix of A is

and ii is easy to show that 51 5 = 5S1 =I.Nore Lhat sr ::f:. s- 1 if we had not
nonnoli;,..cd the eigenvectors of A.
Lei's now diagonalize A:

Nore 1hu1 .1.. 1+ .1. 2 = Tr A and that ). 1). 2 = dc1 A.

Symmetric mauices are diagonalized by similarity transformations s- 1AS =


5T AS. where 5 is an orthogonal matrix. Hermitian ma1riccs, which arc the analog
of symmetric mc1trices in complex vector spaces. are diagonalized by similarity
transfonnar ion of the form s- 1AS = S7AS, where S is a unitary matrix.

E>cample 5:
Firsl show that the normali;,..ed modal matrix of

A=( I 1+ ;)
I- i 2

is unitary. and lhen tliagonalize A by rhc 1ransfonna1ion s- 1AS = s-r AS.


So Lu TI o N: or
The eigenvalues A arc .l.. 1 = 0 ancJ -\ 2 = 3. The correspond-
ing cigcnvccwrs an: u 1 = a(- 1] - i. I) 1 and u 2 = b( I+ i. 2) r _We normalize
= =
lhesc according 10 u:u 1 Uand u ~ul I, so we huvc u 1 = (- I - i. I) T J.jj
and u 2 = (I + i , 2) T/ .J6. TI1erefore..
10.5 DiagonalLi:ation of Matric _ 497
_I:;
S=
( v3
I
~)
J6
2
and S1 =
(
I- i

J'3
I- ;
./3 J6 ./6
and

Note that Tr D = Tr A and det D = det A.

Our last topic of 1hjs section is the following: It is often important in quantum
mechanics 10 diagonaJize two matrices A and 8 by the same similarity transfor-
mation. What are the special properties of A and B that allow this 10 be done?
We'll now show that if two matrices can be diagonalized by lhe same similarity
transfonnation. they necessarily commute. Let

and

Mu!Liply from the left by 5 and from lhc right by s- 1 and find that AB =
BA. Al-
though we shall nor prove it here, the converse of thjs s1atement is al~o true: If r.vo
matrices commute, lhey can be cliagonaJjze<l by the same similarity transformation.

Example 6:
Show 1ha1

and

commute and diagonali7.e both with rhe same simil::u-ity 1ransfornrn1ion .

SOLUTION:

8 7
AB= ( ) =BA
7 8

The eigenvalues and corresponding eigenvectors or A art.> ;_ 1 = I and A~ = 3


and u1 = (I. -l)T/ ✓ =
2 and u2 (I. nT;J2. Tho..c of Bare ).. 1 = I and
).. 1 = 5 and u 1 = (I. -1)T/ v'2 and u 1 = (I. l)TJJ2. Thus.

S= - 1 ( I 1)
✓2 -I 1
498 Ch.ipr,·r I O / Malri and Ei envalue Probl m

in both easel- and

and

1 0.5 Problems

I. Substitute Equation 2 into Equation 3 10 show 1hut AS= SO.


2. Here is ano1hcr proof of Equation 3. First show 1ha1 (AS);j = >-;"' j/· Now u,;;c the: fac1 lhnt D,j = A/\1 to show
Lhnl (5D)ij = ;...i"J,·
. . A
l. 01agonalizc =( 6
I

4. Diagonahzc A= ( ~ O

6. Which of the following marrices is diagonalizable"!

0D
0
(a)
(~ ~) (b) 2
I
(c)
(~ ~)
7. Diagonalize A

8. Diagonalize A
=(

=(
~
n
! n
0
0
0

0
0

9. Diagonalize A= ( -1
_- ; 3+3i)
3 3 1 .

10. Diagonalize A= (
1
~i 1+
2
i) .

11. Diagonalize A = 0 :} 0

12. Find eA if A= ( =~ ~ )-
gh
10.6 Q uadratic Forms 499

13. Find "in A if A = ( ~ 2


~)

14. Under what conditions does cA+R = eAc 8 ?


15. Find e''-r if A= ( -~ ~ )-
16. Solve rhe kineric system

that we solved in Seer ion 3. but by evaluating e'\J.


17. Show rhat your an~wcr to the previous problem i._ the same as the resuh that we obtained in Section 3.

18. Suppose tlrat S = ( ~ - ~) is the modal matrix of A= ( ~ -~). Show that SST = 51'5 #- I unlCS$ S i1- ,l

nomiali::.<'d modal matrix.

19. Show 1hat the cornmu1ing matrices A = ( : ; ) and B = ( ~ ~ ) can both be diagonalize<l by the same
similarity transfom1a1ion and show the similarity 1ransfonn::11ion.

u=; =i)-
20. Show that if Dis diagon::il. then D is <liagon::il an<l
11
iL..; dcmcnL, are the elcmen1s of D rnised to the nth power.

21. Use any CAS to find ,Air A=

10.6 Quadratic Form

The expression ox~ + bx 1x 2 + cxf which ctw,i,1s of squares of variables or


products of two variables. is called a qwulraticform. The general e.'(pression of a
quadr..itic form is
n "
Q :::: LL {l;j-''C,X J ( I)
;,_ 1 1"" I

where the nu are real constants. Quadrntic fonns occur in a wide variety of
applicaLions. For ex.ample, consider 1hc simple one-dimensional system shown in
Figure I0. 19. Let the equilibrium separation of each pair of ncighboring masses
be/. Then the potential energy of the system depends upon the displacements of the
ma<;ses about the equilibrium positions. x 1, .r 2 • x1• and .r 4 . For small displacements,
we can expand V(x 1• -":!· x_1 . x~) in a Taylor series about the equilibrium positions
(.r1 = x 1 = x-' = x 4 = 0) 10 obtain

Figure 10.19
Four ma ·ses .:onnc.:11:d hy .springs.
The mo1ion i$ constrained 10 tlc only
lnngitudinal.
500 Chaprer 10 / Matri - and Eigenvalue Problems

where the zero subscript means tha1 the derivatives are evaluated at x 1 = .x2 =
= =
x 3 x..i 0. The firs! term is just a constant. which we can let be equal to zero,
and the second terms vanish because the (aV / ax j ) 0 are forces. which equal zero al
= =
t.he equilibrium position x 1 = x 2 = x 3 x,i 0. Consequently. if we neglect cubic
terms in the x,-. then

I 3 3
V(x1, X2, X3. X4) = 2 L L kux;XJ
i=I j=I

is a quadratic form. which reflects Hooke ·s law for this system. The.re are many
examples in physical applications where a quadratic form occurs because we
expand some quantity in a multidimensional Taylor series and truncate afier the
quadratic terms.
We can write Equation I in matrix notation:

(2)

For example, if Q = 3xf + 8.r 1X2 - 3xr then we can express Q as


(3)

Notice tha1 we split the cross tcnn 8.r 1x:? into two equal tcnns when we wrote Equa-
tion 3. This makes A a symmetric matrix. which we know from the previous sec1ion
has 1wo l.inearly i.ndependenl eigenvectors. We shall always write Eqmllion 2 wirh
A expressed as a symmetric matrix (Problem 2).
We now wish 10 express x 1• x 2 •... , x11 as a linear combination of new
coordinates x;, x 2, ... , X:1 such lhal Q becomes a sum of on.ly squares of lhe
.x'.• there being no cross terms. In other words. we want to express Q mca11onical
1
fon11. Lei lh.is transformation be denoted by

x = Sx' (4)

where S is the 11 x n nonnali2ed modal matrix of A. Substitute Equation 4 into


Equation 2 to oblain

(5)

which will be in canonical form because the matrix A' is diagonal. We know from
if the eigenvaJues of A are >-. 1• >-. 2, ... , A11 , then Q will
the previous sec:Lion LhaL
be in Lhe fom1

(6)

This procedure is similar 10 the rota1ions that we carried out in Section 4 but is
more general.

rr
501
Let's apply lhese results lo Equation 3. The eigi.:nva.lucs of A arc A= ±5. Thus.
the canonical form of Equation 3 is Q = 5x/ - 5x; ~. The normalized eigenvectors
of A are u 1 =(2. I) T / Js and u1 =(-1. 2) T / Js and so the matrix Sin Equa1ion 4,
rhe normalized modal marrix of A. is

(7)

Therefore.

or

(8)

If you substitute Equations 8 into Q = 5x/ - 5:r;-2-. you get Equa1ion 3. as you
should expect.

Example 1:
Whal type of conic section (ellipse or hyperbola) is de.-.cribed by the cqun1ion
Jx 2 + 8x_,. + 3y:! = I?

SOL u TIO N: TI1e symmetric matrix of the quadratic form is

A=(·34 4)
3

The eigen\'aJues and corrcspondir,ig t:i gcnvcc1ors of A arc ). = -1. 7 ~nd y'
u 1 = (-1. \) T and u2 = (I. I) r _The nom1aiized modal matrix is

x'
and

Thus. the 1ransformed equation i · g(:c'. y') = -x' 2 + 7y'~ = I, or thar of a Figure 10.20
hyperbola ( Figure I0. 20).
A plot of the hyperbola -.\''"' 7y' 2 =I
de1cm1incJ in Example I .

al
502 Ch.:iprc, 10 / Mar rices and Eigenvd lu Probl~ms

Example 2:
Reduce I.he quadralic fonn

f(x. y. z) = 2x 2 + 3/~ + 23;: 2 + 72.r: +I= 0


to canonical form. Describe the re:-.ulting 4uaJric surface.

SOLUTION: The m.itrix A in xTAx is

The eigenvalues and corresponding eigenvectors of A are).. = -25. 3. and


50. and (-4. 0. J)T. (0. 1. O)T_ and (3, 0. 4)T_ The nnrmali1.cd modal matrix
of A is

Then

I
an<l the t:anonical forn, of/ (x. y. z) is

which i~ a hyperboloid of 1wo sheels (Figure I0.21 ).

Figure 10.21
Diagonalizing a quadratic fonn allows us to see more clearly the 1ype of curve or
A plot llf Lhe hyperboloid of two sheets
determined in Example 2. surface 1hat it describes. It's. nor obvious that the quadratic form in Example I is a
hyperbola or that the quadratic form in Example 2 is a hypcrholoid of 1wo sheets
until they are expressed in canonical form.
The more general expression

ax~ + 2bxy + c/ + dx + ey + f = 0 (9)

is called a quadratic equation. and ax 2 + 2b:r:y + cy 2 is called I.he associated


quadratic fon11. Equation 9 represents a conic scc1ion who~c major axes do not
t:oincide wit..h the x and y axes if h ;e. 0. d -:/= 0. ore t=- 0. For example, consider
the quadratic equation

2
x - 2x + 3/ + l2y + I2 = 0 ( I 0)
10.6 Qur1dra1ic Forms 503
The linear Lenus in x and y show !hat the graph or this equation is no1 cenlered at y
the origin (Figure I 0.22). The lack of cross tenns shows Lhat the major axes of the
graph are parallel 10 the x and y axes. To express Equation IO a.-. a sum of squared X
terms. we complete 1he squares of the x and y tenns to get (x - 1) 2 + 3(y + 2) 2 = I,
which is the equation or an ellipse centcred m x = I and y = -2 (Figure 10.22).
We' II call I.he equation (x - 1) 2 + 3(y + 2) 2 = I 1he srandardform of Equation I 0.
To conver1 a quadratic equation such a~
-2
9x 2 - 4xy + 6y2 - !Ox - 20y =5 (11)

into standard form. first diagonaJize lhe quadratic part (the associated quadratic
Figure 10.22
form) and then eliminate linear terms by completing the square. :\ gra~h 01· lh~ l'.(]Uati.o n
x 2 - 2.x + 3y 1 + 12y , - 12 =0.

Example 3:
Express Equation 11 in standard fonn (Figure 10.21).

SOL LIT ION: l11c associated quadr3tic fom1 is 9.x 2 - 4xy + 6y 2 . The
eigenvalues and eigenvectors of the symm.:tric coefficieor matrix are). 1 = 5.
).. 2 = 10, u 1 = (I. 2) 1 / ./5, nnd u 2 = (-2, l)T / Js. The rnarrix that ro1a1es Lhe
graph of Equation I 0 into canonical fom1 according lO x = S x (Equation 4)
1

is

or
Figure 10.23
X=
I
r;(X
I '
-2',') A graph of the equation
-.,5 .
9x 2 - 4x_,· + tiy 2 - [Ox - 20_,. = 5 ul-Cd
I I ,
in Exomplc 3.
.Y=-(2x
./5 +v)
.
Substituting x and y into the quadratic equation give!'.

Completing the square by adding 5 10 bo1h side gives


X

Thus. the quadrnric equation reprcsen1s an ellipse c.-entcred at x = ./5.


y = 0 (Figure I0.24). No1ice that we have rotated and shifted lhe ellipse in
Figure 10.24
Figure 10.23. The graph of the ellipse described by
(X - J5) l 2y2 = 6.

There are a number of physical applications where quadratic forms occur. Re-
cal I from Sect1on 6.5 tha1 Lhe rotation of a rigid body may be viewed as 1he rotation

al
504 Chapter t O / Matrices ,,nd Eig •n alue Probl ms

with an angular velocity w abou1 an axis passing through s.omc origin. which for
simplicity. we take to be the ccntcr of mass(~ Figure 10.25). Equation 5.4.15
says 1ha1 the total angular momentum of a collccrion of massc m,- located by the
position vectors r; is given by
n ll

L= L r,- x (m;\';) = L mir; x (w x r,) ( 12)


i=l i=I

If the triple cross product in Equation 12 is written out (Problem 9). Equation 12
becomes

L = I ( l_rx Wx + /.tyW + l_c_w . )


Figure 10.25
+ JU1-xw. + l _1-yW1. + l_~7 w . ) ( 13)
A rigid body rotat.i ng about an axis pas1-ing
through the c.cntcr of mw;s of I.he body.
+ k ( I. W .r +,_\,WV+ l .:.W- )

where
II II II

/1·1·
·· = "m,·(r
L.,_; · ~ + z~)I I
/1·y
. .
= '°'
~
111j(X2
I
+ .:::2)
I
/-•.. = '°' 2
L.,_; m -(x + v~)
1 I • I

•=I i=I i=I

11 n II

IX_\"= - L llljX;)'; l.r: =- L m;x;:.i ly:. =- L m;_r;z.;


i=l i=I i=I

( 14)

Equation 13 can be written in matrix notation by writing

L=lw ( 15)

where the elements of the moment of inenia maLrix I (not 10 be confused with the
idcnIi1y matrix) arc given by Equations 14. [n Problem I 6, we show thal the kine1ic
energy T of the rotating body is given by

I T
T = -w lw ( 16)
2
The axes in Figure 10.25 were chosen arbitrnrily. and that's why there are
nonzcro non-diagonal elcmenIs in I. It should be clear from our discussion of
quadra1ic forms, however. that we can find an onhogonal trJnsfom1ation of the
axes in Figure I 0.25 such that I becomes diagonal. The axes in which I is diagonul
are called principal ares and Lhe diagonal elements of I arc called the principal
moments of inertia of rhc body.
Another imponant application where quadratic forms arise is in the theory of
the thermodynamics of irreversible processes. Consider an abiahatically insulated
system. which is described thermodynamically by a set of stare variables. The
entropy S of the system is a maximum when these slate v<1riables assume their
cquilib1ium v:.ilues. say A7. A2. .. . , A~ . Suppose now that the system undergoes
10.(, Quadr,11 i, r, ,rms 505
a fluctuation from its e<1uilibrium stale, where the stale variables have (nonequi-
librium) values A 1• A2 •... , A,,. Then the deviation of the entropy of the system
from its equilibrium value is a function of the quantities {a j =
Ai - A j). and if
the fluctuations are small so that lhe {er j I are small, then we can write ~S as

where .l.- 8 is the Boltzmann constant. There are no linear term$ here because S is
a maximum when the a j = 0. Using the relation be1wccn the entropy of a sysrcm
and its disorder, it turns out the probability of observing the values lai I is given
by

=exp
(
-
I
L Lg,p.;aj
n
1- i=l
n )

j.:l

All the cenlJ111 results of the thermodynamics of irreversible processes can be


derived from this basic relation. which is a muhivariate Gaussian distribution.
RecaJI (Section 3.3) that a Gaussian distribution is of the form

-OO<X<OO ( 17)

A general multivariate Gaussian distribution is of the fonn

( 18)
J=l,2, .... 11

where c is a normalization constan1. Multivariate Gaussian distributions O{;cur


in statistics. in fluctuation theory in s1a1is1ical mechanics. in the theory of non-
equilibrium thermodynamics. and in a number of other areas. (See Chapter 21.)
We often need to evaluate integra1s involving f (x 1, x 2 • ...• x 11 ). For example.
the de1ermi nalion of lhe slati sl ical correlat 10n between I.he variable.-. x 1, x 2 • ...• x,,
requires thnt we evnluate integrals like

oc

Mij = ff x;.xjf(x1. x 1 , ...• x,,)d:c 1 ... dxi 1 ( 19)


-oo

The cross terms in the exponential make these integraJs difficult 10 evaluate.
However, if we write the quadrat-ic form in the exponenriaJ in canonical form,
then f(x. 1• • • • • xn) becomes just a product of independent factors and integrals
like Equation 19 can be re.1dily evaJuated.
506 Ch,1ptcr I O i ,\.latricc.-s and Eigcn alu Problems

We'll evaluate the normalization cons1an1 c in Equation 18 firs!. The nomial-


ization conscanl ,ati!.fies the relation

I·f
00

c f(x1,X2, . . .. x,,)dx 1,i:t:'!. ... dxn


-00
(20)

-x

Write x = Sx1 or x' = 5-- 1x. Then. xT.Ax = x' S1 AS,i:' = x'1 Dx'. where

0 0 JJ (2 I)

Th ere fore, xTA x. -_ x,ro x1 -_ 11.1 1x ,2.


1
+ ""!x
, ,_2 + · -· +).,,.t,,.1'.! an d j( x 1• x 2 • . ..•
x,,) ::::: g(x;, x~, ... , x;/ Note that all lhe eigenvalues of A mus1 be grea1er Lhan
7..ero in order that integrals involving g(_r;, x~ . .... x~) converge. We conven the
"volume element" dx 1dx 1 - - - dx,, 10 dx;dx; ... dx;, by inrroducing the Jacobian
detenninant

Jx, ax, ilx 1


ox; i-Jx~ rlx'It
J (Xx' ) -- (22)
,lx,, ox/I ax,,
ax; ax; ax'
"
Bui x = Sx 1
• so lhis de1em1inant is equal lo JS! (Problem 19). Therefore,

because S is orthogonal.
Equaiion 20 now reads

I ! ,'
00

C • •• g(x 1• X.,,
i.,,
•.• , X rlI ) d x 'd I
1 x,- ... d X ,,I
-oo

= I· f
'X)

c e-!rJ,ix;-+>-i ?+··· j ,,.. dx;dx; ... dx;, = I


;,i 1
-oo

This n-fold integral is act-uaJly the product of integrJls. so we can write


1O.G Quadratic Forms 507

(2rr t l 2
= (,'------=\ (23)
(A 1.l.2 . . . J..11) 1/2

But >.. 1>.. 2 · - · A,, = det A. so we finally have lha1


(dei A) 1/ 2
c=-·- - - (24)
(2rr )n/2

Example 4:
Evaluntc the integral

ff
(X.

I= dxdy e-½ c1ri - 2.x , 3J 2J

-00

by diagonnfojng the quadratic form explicitly and in doing so verify tha1 rhc
Jacobian de1crminan1 in Equation 22 is equal to one and that your tinal result
agrees with (2rr)" ~/(del A) 112 (with 11 = 2).

SOLUTION: The matrix A is

A=( -1) 3
-I 1

The eigenvalues of A arc >. = 2 and 4. and its corrc.sponding normalized


eigenvectors are (I. I) T / Ji. and ( - I, I) T/ Ji.. Its normalized modal matri ~
is

and so

x) l ( I - I ) ( x' )
( y = J2 I I y'

or

I I ' ,
.r = -(.x
I ,

X = .Ji_ (X - y ) and
Ji. + Y) (25)

The quadratic fonn becomes 2rr2 + 4y 12 when expressed in terms of the


primed v.iriahles .
508 Ch,1p!l'r 10 / M alri · .:md £igen \',1l 1w 1Pr1Jhle111~

To evalua1e lhe Jacobian de1erminan1, we differen1iate Equation 25 to


obLain

8:c ax I I
ih' ily' Ji - v'2
l =
oy ay = I I
= I

eh' c.l y' ../2 ../2


We'll now detennine / from

ff
CC.'
1
I= dx 1dy'e-~c2x';+4 / >

-X)

For our final manipulation of multidimensional Gaussian integrJ.ls. let's eval-


uate M 11 in Equa1ion 19. We're going lo see thal M,'j is the ijlh elemenl of A- 1.
First. write M;,1 a"

and use the rela1ion x = S x' to wrire


II

and xj = I:spx; (27)


l=I

Substitute Equations 27 in10 Equation 26 10 get

(28)

The only 1enns 1har are nonzcro in this double summa1ion arc the terms in whil:h
k ;:: I because lerrns such as

00

ff xye-¾C.•- ·' +:...i~·~>dxdy;:: fxcc.


1
00
xe- A.ir /''!.dx _(.: ye- J. 1 v1f 2 dy;:: O
-:,o
10.b Quaclrarir I orms 509
Thus, Equation 28 becomes

=
12
(det A) 1 "
('Jrr)"/2 L ·'ns1, (2rr)
.
-).
(2;r)
""i:
112
l/2
·· ·
( 2rr)
d
112

·..
(2rr)
-:;--
112

- · I= 1 I 2 ,.,_, 11 •

(29)

We"ll now ~how that this summation is equal 10 1he ijth element of A- 1• SI.art
wilh

Take the inverse of both sides and use 1hc relation (A B)- I = s- IA- 1and 1he fact
that sr = s- 1
10 obtain

Now multiply from the left by Sand from the tight by 5T 10 get

(30)

The ijth clement of A- 1 according to Equa1ion 30 is

'°'"'
(A -I );; = L_,;
11 H

L_,; S;1; D-11


kl slj
k=I /=I

(3 I)

If we re.aJize tJiat s~ = s JI• then Equation 31 is 1hc same as Equation 29. so we


have M;j = (A- 1
);
1, or

M=A- 1 (32)

in matrix notation. The quant:i1jes MiJ are called covarinnces and M is called the
covorinncc matrix. The value, of 1.he covariances arc a measure ()f the statistical
correlatio □ between the x;.
Because of the rclatjon A= M- 1 given by Equation 32 and the relation

al
510 Ch,1ph•r 1 O / Matrlcl', ,1nd l igenv;ilue Problt-1m

det (A) = 1/det (A - 1


). 1he multivariate Gaussian distribution is often written as

(33)

Example 5:
Use Equmion 33 to write out a Gaussian distriburion in two variables, Use
lhe noiation

{,;v2)= a2.
_\" '

SOLUTION: The covariance marrix is given by

a~,.
( -rrr,.a_,.

2
- - -2r-
X _
+ y-2 )
X )' ]
'
( r, ;: a_, a.,· a)~ '
Notice that f(x. y_) = p(x)p(y) when,-= 0. when x and y are sta.tiscically
independent or uncorrelated. Because or this. / 2 is called the correla1io11
coefficie111. Also note that r must satisfy - I ~ r :: I (Problem 13),

Ano1her imponant applico1ion of the diagoaalizalion of a quadratic form is lhe


following. RecaJI from Sec1ion 6.8 Lhat we i.nvesrigated the na1ure of the exLremum
of a function of rwo variables al a point (a, b) by considering Taylor's fonnula.

f (a + h. b + k) = f (a. b)
I 2 f.cx<~.
+ 2-[Ii 1/) + 2hkfn(~. 2
11) + k fn,({.
·
11)1 (34)
. ..

where a - '1 < ~ < a + h and b - k < 11 < b + k. The nature of 1he extremum
depends upon Lhe sign of the quadratic fom1 in Equation 34 for 1he various values
of It and k about the point (a. b). If we convert this quadrn1ic form 10 canonicaJ
fonn. I.hen we have

(35)

where the sign of Q now depends entirely upon the signs of .l.. 1 and >. 2•
1O.<> Quadratic Forms 511

A quadratic fom1 is said to be.posirive de.fin ire if Q =,?Ax> 0 for all valucsof
x # 0. It iscalled pm:irive semi-dejiniu if x I Ax~ 0: 11eg01ive de/mire if xTAx < O:
negarfre semi-definite if x 1 Ax ~ 0; and imlefinite if x TA x has both positive and
negative values. You can sec frorn Equation 35 1ha1 ,c TAx is positive definile if and
only if>.. 1 > 0 and >.. 2 > 0 and negative definite if and only if A 1 < 0 and .l. 2 < 0.
Thus. rhc bchavior of I.he sign of Q =
x T Ax and the quadratic fonn in Equation 34
depend upon the eigenvalues of

which arc

You can see from Equation 36 that both eigenvaJues wiU be positive if .f.u +
/,·y > 0 and fuf"r - /;, _.._, 0. or equivalcn1ly. if either / 0 > 0 or f"Y > 0 and
fvxf, ..,. - f,;" > 0 (because if fxx > 0. then f'"-" > 0 if frrfi•y · · J}1 > 0). If this
is so. then /(a +h. b + k) > /(a. b) and J (a. h) is a local minimum.
Similarly, if fu + f •n-• ,;;;: 0 and f:r.Jvi·
••
- / ,C.}2 > 0. or equivalently, if either
JJ.-x < 0 or /i•.r < 0 and f u J~·y - f}I' :;. 0 (because if fu < 0. then f,·y < 0 if
f.>. .J 1,y - f;\. 0). both c·igcmm•l,ucs will he ncga1ivc. In this case. f(a + Jr.
b + k) J(a. b> and f(a. /,) win be a locul maximum. lr the eigenvalues have
opposite signs. then J(a. /J) is a saddle poinl.
We derived lhese results for a function of 1wo variables in Chapter 6 without
1hc use of the Lheory of quadratic fom1s, bur for rhe '-.'asc of II variables. the
char..1cteriza1jon of an extremum in tenns of 1hc eigenvalues of A is indjspen~ablc.

10.6 Problem
I. Express lhe foltlowing quadra1ic forms in matrix notalion xTAx. where A i, a ,ymme1ric matrix.:

2. Show thar writ ~ng .a quadra1,ic fonn in terms of a symmetric matrix i$ equivalent to replacing the original matrix
A by the symmetric matrix (A+ AT)/2. Now show 1ha1 x·1 Ax = x 11(A + A 1 )/2Jx.

3. Find the standard lorn, of the equation 5x 2 + 4xy + 2y 1 = I. Find the or1honom1al basis which yields this
s1andard fom1.
4. lden1i.fy the grnph of 2t' 2 + y2 - 4xy - 4y:-...: 4.
5. ldcnt.ify the type of surface described by .h 2 + 6y 2 + l:: 1 - 6xJ - 2y;: = I.
6. Identify the Lypc of surfocc described by 2.x 1
+ Sy + 5.:: + 4xy -
1 1
4xz - 8v: = 10.
7. Express 3x 1 - = o in standard ronn.
8.x_r - 12,r.! - 30x - 64,r
8. ldeniify 1he type of -;urfacc describcd by 2\',I' + 2\': + 2y- = I.
512 Ch.ipler I O / Matric ~ and Elg nvalu · Probl ms

9. Derive Equation 13 from Equation 12 .

10. Show 1hu11hc probability distribution in Equa1ion 17 is nomialized; in orhcr words. show lhar 1= f(x)d x =I
1
-CIC
00
and 1ha1 {x) = 0 and {x 2) = a- 2, where (x" ) = x"J(x)dx. The quantity {x) is 1hc mean. or I.he average.
-=
of f (x ) . and {x '.!) is its variance lthe square of the st.andard deviation or J (x)].
2
11. If (x) = J.L :/=- 0, Equation
17 becomes j(x)d.r = (2Jto 2 )-'l 2e- 1x - µ l f'lo \ Jx for -oo < x < . Show that
{.r) = tt as inferred. Now show that {(x - µ ) 2) = a 1 . Interpret ((x - µ) 2) physicaJly. h is called the second
a•111ro/ 111omenr .

12. Use the relarinn det (AB) = det (A) det (B) to show 1ha1 dc1 A- = 1/det A. 1

13. In 1hi~ problcnl, we prove tlrnt the correlation coefficient r defined in Ex.ample 5 must satisfy - I ~ r ::: I. First
...
show tha1 {(et.r + /jy)'") -e"'ra;.., + 2afjra.ray + fJ·a_;
' ,., ~ 0. Now lake a= <\7,, nnd /J" -ra,roy an d show 1ha1 =
, 2 :S I. or - I ~ r ~ I.

14. In thi~ problem, we show I.hat

(I)

Le! S be the normaJized mo<laJ matrix of A. so !hat S1 AS = D and x1 Ax= x'Tox-'. where x = Sx'. cme
quadratic form is now diagonalized.) Now ler u = s- 1I = S1 t so Lhat t Tx = (Su) Tx = uTS1 x = uTx 1
Thus. we
.o.:,f .
.

= n,; JOO
_ . e i11 · l e
I
I • I ·'Q . t' - I ,_ r' ~
have • - • e'u x -11: x d.r;dx; . .. dx~ "! r ; tlx:. where the>.. J arc the eigenvalues of
. -oo }=I -:,.:.

= (2,7)
1/2
- u 2f'2J.1
A. Now use the fact thar (sec the next problem for a proor)

use uTo- 1u =IT SD-ISTt = tTA-It 10 verify cquu1.ion I.


!-:,c
e
iu:r'
I - J.l.x'•
Je - i I d:c
I

1
~
),_J
e 1 • and

15. Show that/=


! ·- .
_ I J,
e' 1x ! .r dx
~ = - (2ft)
h
l(J.
e
-1 ~/ '1 . · .
_, by expanding ettx and rntcgr,111ng 1enn by Lenn.

16. We derive Equation 16 in this problem. First. write the kinetic energy a$ T = L m,.;} /2 where i-1 = dr;/dt .
I
Now use r, = w x r1 lo get T =; LII

m 1(lu x r,-) • (u> x r; ). Now use the scalar triple product fomiula
- J-1
Vi . (v2 X V3 ) = (v, X V2). \'3
. T = 1w
= V1. (,· ( >< ,., ) lO \Vrlle I
Lj=:!I m;r; X (w X r,) = 2W .
. ,-..,, I L = 2W
I
' I. w =
~wTJa, .
\'
17. Thi!-i problem illus1rates how the principal moments
of inenia can be obtained as an eigenvalue probkm .
We. will work in 1wo dimensions for simplici1·y.
Con !\idcr the "molecule" rcpresen1ed here.

X
513
where all I.he ma._,~, are unj1 masses and the long and shon bond lengtJ1s are 2 and I. respec1ively. Show I.hat

I u: = 2 cos2 0 + 8 sin 2 0

/_Q = · - 6 cos 0 sin()

The fact that lxy ¥- 0 indicates that these I; arc no1 1hc principal moments or inenia. Now solve 1he secular
de1enninantal equation for).,

and compare your result with I.he vaJues of l.u and /JY Lhar you would obtain if you align the "molecule" and
1he coordinate sys1em such that 0 = 90°. What does this comparison tell you? What are the values or /.rx and
,_,·y if 0 = 0°?
18. Which of Lhe following quadratk forms is positive defini1e?

(a) x 2 - 4xy + 5y 2 (b) x 2 + 6x_,, + 3y 2

19. Show that 1he Jacobian de1enninaa1 in Equation 22 is equal to IS I,

References

H. Anton. 2000. Elemmrury Li1111ur Algebrn. 8th ed .. Wiley


H . Amon and C . Rorres. 1994. Uemt•11wry· Li11eor:\lxebra: Applicmio11.,; \"<',1io11. 7rh ed ..
Wiley
F. Ayre~ . 1962. Tht'ory mu/ Probfoms r,f Mar rices, Schaum·~ Our line Series. McGraw-Hill
R. Bron:.nn. 1989. Matrix Opero1inns. Schaurn's Outline Sc.:riL·s. McGrnw-Hill
F.8 . Hildebrand. 1992. Method.\· nf Applied 1Wt1thc111a1irs. Chapter l , 2nd ed .. Dover
B. Noble and J.W. Dnnicl. I 988, Applied Lifltar Algebra, Prentice-Hall
C. Rorre~ and H. Anton. 1984, J\pplimtion.t n( Linear Algt·bra. Jrd ed .. Wiley
G. Stephenson. 1965. A11 J,11rod11rtion of Marrin ·,\, Si•ts. and Gm11pJjor S1·it·m ·,· Stllllems,
Dover

C
CHAPTER 11
Ordinary Differential Equations

Many scicn1ilic laws can be expressed in 1enm, of differential equations. In fact.


diffcrl'ntial equa1ion:-. arc 1hc mos1 common and mosr useful means of formulat-
ing 1hesc 13\, s. A di ffl'l'l"ntial equation is an equal ion invol vi ng dcrivati ves of an
unknown func1ion that depend$ upon one or more indcpcndcn1 variables. If the
unknown func1ion dl.'pcnds upon only one independent variahle. then Lhc equalion
is called nn ordinary differe111ial eq11mi1111. Ordinal")' diffrn:ntial equations nec-
essarily involve ordinary derivatives. E.'<amplcs of ordinary differential equations
!.IJ'C

(a) (b)

dr) 2
d·'\·
[ (
] :-/'!.
(a) I+ -=-- =k - · (h) (2)
dx dx 3

In each ca.'-c, there is only one independent variable. x. and one dependent vari-
able, y. If the unknown function depends upon more than one independent variable,
then iL-. equation is called a partial diffen.•mial equtJtion. Partial differential equa-
tion .., nen.·ssarily involve panial deriva1ivcs. Examples are

and

We shall not discuss panial differential cqua1ions un1il Chap1cr 16.


There arc several terms that we often use when discussing differential equa-
1ions. w~ say that y(x) is a so/111io11 of an ordinary differcn1ial equation if it ,;ati<;11cs
!he equation identically over some in1erval (a. /1) of x when it and iLc; derivatives
are substituted inio the equation. The order of a differemial equation is the order
of 1he highest derivative thut occurs in 1.he equation. Equations Ia and 2b are fin,t-
order equations. I b is a s.econd-order equation. and 2a is a third-order equation .
The deJ;:rec of a differcnliaJ equation is 1.hc power of ils highest-order derivative 515
516 Ch;ipter 1l / Ordinary Diff rential Equa1ion~

when the equalion is wriuen a.s a polynomia.l in all Lhe derivatives involved. Equa-
tions Ia, I b, and 2b are easily seen to be first-degree equations and Equation 2a is
second-degree becau...;;e il lnkes the fonn

when ii is wrillen as a polynomial in its derivatives.


One rea-;on tha1 we classify differential equations according 10 order and
degree is hecause the properties of their solutions and 1he methods that we use
10 determine the solutions often depend upon this cla.. sificarion. The 1j1les of the
sec.:tions of Lhis chapter specify the types of equations that are LLiscussed. Section I
deals with differeniial equations of first order and first degree and Section 2 deal$
with linear first-order equations. A differential equal.ion i.-. said 10 be linear if
1he dependent variable (the functjon to be detem,ined) and all its deriva1ives occur
only to the firs! power and there are no cross tem,s. Equation lb is the only linear
differential equal.ion of Equations I and 2. Equa1ion I a is non Ii near because of Lhe
y2 tenn; Equation 2a ls non I inear because of Lhe (d y / dx ) 2 tem1; and Equation 2b
is nonlinear bet:ause of 1he _y 2 dy/dx term. Sections 3 and 4 deal exclusively wi1h
linear differential equations. Section 6 deals with systems of fina-order differential
equations. such as the simultaneous equal.ions

dx
- =.r +4v
dt -
dv
--'- =
dt
-2r + .v

In this case. we have two differential equations to solve simultaneously.

11.1 Differential Equations of Fir••-t Order and First Degree

A first-order differential cqua1ion can be wri11cn in the form

.dr
. . :. . = f(.x, y) (l)
dx

If f(x. y) = -M(x. y)/N(x, y). then Equation I can be written m 1erms of


di fferenl ials:

M(x. y)dx + N(x. y)dy = 0 (2)

In Equation I, we say that x is the independent ,,ariablc and tha1 _v. which depends
upon x, is Lhe dependent variable. In Equal ion 2. we don·t nccL·ssarily distinguish
between 1he independent variable and the dependent vari:.ible. In either case, u
solu1ion is a relation bc1wcen x and y 1hat s.ati,;fies 1hc differential equation.
For example, 1he function y (x) = c/ ( 1 - ex). where c is a tonst:mt. sa1 isfies the
11.1 Diil r n1ial Equarion o Firs1O rd er and First D •r 517

differential cqua1ion

dv
---=-- = ,,-., (3)
dx .

if x ¥= 1/c. You can see thal 1hi.'- is so by differentiating _,·(x) c/( I - ex). It =
turns out that e1 ·ef}· solution 10 Equation 3 is of the fonn y (x) c / ( I - ex). and =
so we say that y(x) = c/(1 - ex) is a general so/111io11 of Equation 3. If we are
given additional information. such a.-; y -= I when x I, then we can determine=
the constant c. which in lhis case would be c = 1/2. The solution _r(x) I/ (2 - x) =
is called a particular so/111io11 of Equation 3.

Example 1:
Show that y(x) = c f!"t - 2 - 2.r - x 1 • where c is a constant. is a solution of

dr
---=--
dx
= .\" + x-,
Fin<.I a particular solu1ion if y = I when .x = 0.
SOLUTION: Diffcren1ia1ing _\'(x) gives

dv ,
.dx
. .:. . . = c~ - 2 - 2.r = ,.
.
+ x-

Bccau~L· y(O) = I. l" = 3. and so the particulur solution is y = 3e-1· - 2-


2.r - ., : .

Before we go on. we should address the question of whether a given differ-


ential equation even ha ... a solution. For c-..:amplc. the equation (/) 1 + y2 + I =0
doesn't even have a real solution because the lefl side is nect:,~.:uily positive. The
following important theorem. which is proved in most textbooks on differential
equations. addresses not only the exi .. ll'llt:l' of a solulion. but its uniqueness as
well.

Cul/.\idcr tht'" jirst-orc.lt!r di//t'rt.·111i11/ eq,w1iu11

d"
---=--- = f(x. .,·)
dx

along ll'ilh 1he 1mriliary co11di1io11 y(x0 ) = y0. If f (x, y) and nf/il_\' are renJ.
fi11ire, sin[:lc-\'{llued, and conri1111011s in .wmr re,1:JiCm s11rro1mdi11g tlu• poim
(x 0 . _\"i)), 1he11 there is 0111' and 011/y 011e solution to the above cqunlion in mi
i11tt·,.-al -Ii ~ .l 11 :;; I, lyi11R within 1/u• r,·gim1.

Applying thi1- theorem 10 1he equal.ion in Example I show~ that the solulion is
unique for all values of x.

al
518 Ch,1pt,•r 11 / Ord it1,1ry Differ •nri .:i l Equa tion

Example 2:
The fonctions. _v 1(x) =0 and y 2(x) = x 3 arc solutions lo
y(O) =O
How do you rcroncile this with lhe above theorem?

y SOLUTION: Because iJf /ily = 2/y 1n i~ not <.:onrinuous :.ilong 1he line
y = 0. there is no guarantee tha1 Lhe soluIion is unique in any region
<.:ontuining the line y = 0. In fact. !here are infinitely many solutions bccau~c

-00 X k
k < .r

X is a solution for any value of k ~ 0. Figure 11. I shows this family of t:urves.

Figure 11.1
=
A family of snlu1iom, of y' '.\y~I-' with
=
_y(O) 0 gjvcn in Example 2. We should men1ion al rhis poin1 that 1he continui1y of J(x. y) und i)J/;Jy is a
sufficient but not necessary condition. In panicular. the condition on 1hc derivative
can be suhsLnntially relaxed. hut 1he con1inui1y of i)f/cly will cover all 1he case.s
we shall consider.
Let ·s go back lo Equation 2. If 1W (x. y) and N (x. y) are such that Ec1uation 2
c:l.Il he written as

f(x)dx + g( _v )dy = 0 (4)

then we can simply integra1e both sides of Equation 4 to obtain a soluLion to rhc
differen1ial equation. In this case. we say that Equation 2 is separable. A separable
differential equation is !he eru:iest Lo solve because it readily reduces lo a problem
of in1egration. For example. the differential equal ion

dy X\"

dx y+ I

is scparJblc because i1 can he written as

(7,. +I)
X
xdx= dy

. . . x2 .
winch can be mt~gratcd 10 give -'} = .v + In .v + c as 1.he sulu1ion.

Figure 11.2 Example 3:


TI~ ,oordi11;11c :,,y,k·m ~cd in example J.
Consider thi.: \'Cnicnl motion of a body of mass 111 -.ubject to a grav,i tmional
The posi1iv~ x a.xis i.~ <.hrt:ctcd upward. :-o
1hat 11 = d.r /tll is negative for a falling force IIIR and a re.-.istive force proponional 10 its s,peed. !If we let t' = dx /dt
body. and lake: 1hi.: positive x a.-.:i~ to be directed upward as in Figure 11.2. Lhl!n the
519

equation of motion of the body is

dv
111 - =- yv - mg
dr

where l' is iu; velocity. Solve this equation with the initial condition u = v0 .
SOLUTION: This differential equal.ion is separable. so

mdv
---=-dr
yv + m.c:

or
JI/
- ln(yv + m~) = -1 + c
y
or

where A= r,rr/ni_ Le!ting v = t'i) at r = 0 gives A= y1.-10 +mg.or

Nore that this solution say~ that a body falling through a resistive medium
approaches a constant veloci1y. -mJ;:/y. called the terminal velocity. Note
also that we can obtain this result by letting dv/d1 = 0 in the original
cquat.ion.

Another rypc of lirsr-ordcr differential equation lhat is easy to solve is an


exact differe11tial eq11ario11. An exact differenLiaJ equation is the toial derivaLive of
F(x. y) = c. or

JF aF
dF = -dx ay · = M(x. ·v)dx + N(x. ·v)dv- = 0
ax + -dy (5)

in which case rhe equal i1y of the mi xcd '-C'COnd partial dcriva1ivcs of F (x. y) gives
the criterion

(6)
f.J yux dX
If Equation 6 is satisfied, then 1he solurion is given by

F(x. l')
• 'dF
. = ) -dx
ax + g(v) = ~
Joy . +
aF
-dv h(.x) (7)

where rhc integral ions arc "pan ii.II"' integral ions and where h (x) and g ( y) are
arbitrary •'functions of integration." In other words. y is held fixed in 1bc firs1
integral and x is held fixed in the second. For example. con~ider the differential

C gl
520 Ch,1p1t·r 11 / Ordinary Di if r nl ial Eq uations

equation

(2y .. : 2)dx + 2.r dy = 0


Equation 6 is '-alisficd because D(2y + 2)/Jy = <1(2t')/clx. and so

F(x. y) = f (2y + 2)dx + g(y)


Now y is a consranl in 1his integral. so we have

F(x, y) = 2.ry + 2.t + g(y)

We can dctennine g(y) by substiruung this result into uF Jay= N(x, y) = 2x 10


obtain
aF dg
-=2x+-=2.x
Jy dy

from which we have g(y) = c. where c is a cons1.an1. The solurion, Lhen, is


F (x. y) = 2.r y + 2x = A
where A is another consranr.

Example 4:
Solve the differential equation

d\' a 2 · -2xy-yZ
--'-::.;-----

SOLUTION: First write the equnt.ion in the fom1

Equation 6 is s:itisticd because

o(a 2 - 2.n - v2 )
----•;...._--'";...._ = - 2x - 2 \' = -
iJ(x + ·1') 2
ay . ilx

and ro we sec that the differential equation is cxacr. Therefore.

F(x, y) == f (a
2
- 2.xy - y 2 )dx + g(y)

Now use the fact that

aF ) dg 1 , ,
-
a_v = -.r- - 2n
- + -d_,· = N(x. r) = -(.r
.
+ •r) - = - x· - 2.n - v-
. .
11. I Oiii<•H·11tial Equ,11ions of Firs-I Order and Firs-t Dr·,_:n•t· 521
\']
to obtain g(y) =-=-+ c. Therefore,
3

ll1c toll:J.I derivative of F(x. y) = -c is exactly 1he original diffcrcmial


equation.

If an cqua1ion is not exacr. it can sometimes be turned into an exact equation


by multiplying ii by an appropriate function of x and y . Such a function. if it exi~t~.
is called an i11rc:grati11g factor. For example. if M(x. y) and N (x. y) are sums or
products of powers of x and y. then a possible integrating fac1or is x 0 yfl. Let's
consider the differential equation

(8)

where M(x, y) = _i + lry 3 and N(x. )')=I - xy. Equation 8 is not exact as it
s1.an<ls. but if we multiply it by xuyff and 1hen set ;JM /Jy = DN /ux, we find tnat
a= 0 and fj = -3 (Problem 22) . &1ua1ion 8 becomes

(9)

which is now exau. Using Lhe procedure 1ha1 we developed above for cxac1
equations. we lind 1hat 1he solution to Equation 8 or 9 is (Problem 2))

. Jx~ x I
j (X. r) =- ...1. - - -., +C
· 2 r 2y-
a.s you can verify by direct subs1i1ution.
Textbooks on differen1ial equations develop procedures for finding integrating
factors for cerwjn types of di fferenrial equations. bur even then finding an intcgrat-
ing factor can be a matter of good fortune. Nevertheless. in1egra1ing factors play
an important role in some theoreticaJ devclopincnrs of differenrial equations. as
we shall sec in the ncx1 section.
Recall from Section 6.4 that a homogeneous function of degree II is a function
that .saListie.s the relation

/(1..x. A._\')=)." J(x. y) ( 11)

A differential cquarion M(x, y)dx + N(x. y)dy = 0 is s.aid 10 be homogeneous if


M(x _y) and N(x. y) are homogeneous frunctions of 1he same degree . Problem 18
has you prove that the xubs1i1ution w ::::: y ,lx tran~fonns a homogeneous differential
equation into one whose variables are scparnble. For example, the equation

C
522 Ch.ap1cr 11 / O,c.linary OiHeren,ial Equ,1tioth

dy x~
)
+ y~ ')

--
dx 2xy

is homogeneous. The ~ubstit-ution y = 11 .r yields


1

dw I+ w 2
x-+w=---
dx 2w

which yields

or

y(x) = ±(x2 - Cz.t) 1/2

Example 5:
In Chapter 13. we shall encounter equmions of the fom1

dy 2y
- = --
dx x - y

Find the gcner.LI solution 10 this equa1ion.

so LUTIO N: Both 2y and x - y are homogeneous functions of degree


one. so we lc1 y =
uu 10 get

dw w{I + w)
x- = - -- -
dx I - 111
or

(I - w)dw dx

Upon integration. this gives

11 .1 Problems
1. Show lhat 1hc exprc$.'•:ion on Lhc left is a solution to the diffcren1ial equation on 1he right.
dv .,
(a) y=x 2 +cx x-=- =x - y
dx
d2,,
(b) J = c 1 c.:os .r + q sin x _ . +y=O
dx~
11.1 Diifr•rt•nti,11 fquati1m~ of First Ord(•r and Fir~! D1•~rt•1• 523
d 2y dr
- 2 +--=- - 6r = 0
dx dx ·

(d) y = c.l'. + c 2 \'=X -


Jy ' (dy
-
)2
· clx dx
2. Solve the following diffcrentiul e<Juations:

(a) ~Y
d.\
= __Y_
x - I
(b) a (xd y
d.x
+ 2")
.
=xrdy
. dx
( c)
ds
-
dr
= 15 - 16s ; s = 0 when r = 0
J. Solvt x 2 dy = r 2 dx
dx . dy
4. A copper pellet at a temperature of 200°( is dropped inro a l..irge bucket of warer at 20"C. (The buckel is
so large that the temperature of the warer doesn·1 change .) Af!er six minutes. the IcmpcrnIure of the pellet is
IUOQC. How much longer will ii take for the temperature of the pellet 10 reach 25°C? U e Newron'l- law of
cooling, which says in this case that d T = -k (T - 20). where T is the 1cmpcraturc in degrees Cclsi us and k
dr
is an empirical constanl.

5. A simple equation d<.:M:ribing population growth is d x =


ax - b:/2, where x(f) is the population size and 11
dt
and b :ire empirical con~tnn1s . Derive 1he solution 10 this equation. called the lo,:.istit: cun,e . Plot 1his curve and
explain its behavior. Show that x(t) ➔ a/ b for all x (0) Iexcept for x (01 = 0 J.
6. De1ennine which of the following equations is exact and solve 1he onl!S 1ha1 are:

(b) (2.r + y )dx + (y - x)dy =0


(d) (sin x + y)d x (x - 2 cos y)d y = 0

7. Sometimes a simple substitution 1ums an opaque differential equation inlo one that is easy 10 solve. Can you
find such substitutions for the following differential equations?
d\' - r dr x y - I
(a) xdy+ydx=xy:i.dx (b) -=-
dx
= xx--·
+y
(c) --=- = - - -
dx x y +I
(d)
dr
-:.. -::::
d.x
x\·
----;;--=------
x- + r
8. TI1c speed of a body faJling through air <.:xperienccs a resistance that is a function of its speed. Newton's
. . dv . .
equation can be wnucn as mdr = 111,: - /(11). In many cases. f(l►) 1s found 10 be propon1onal to the speed .
Derive an equation for 11(1) assuming 1hat f (u) = av and that the body is initially released from rc:-t. -.o 1..h::11
11(0) =0.
9. ~rive and solve a differential equation that describes 1he curve such that the intercept of its tangent line at
any point (x. y) with they axis is equal to 2xy 2.
IO. Dctcnnine 1.he equations of the curve~ for which the.! normal~ (line:- 1ha1 arc perpendicular to the tangent lines:)
at any poinl p.1ss through the origin.
11. Detennine the famjJy of curves that are orthogonal to 1.he family of rectangular hyperbola..:. described by xy = c.
12. De1em1ine a family of curves that i~ orthogonal ro the family of cardioids. r = c( I+ sin tJ) . Him: Use
Equation 8. 1.3.
13. Find the equmion of a curve that has the properry that every point (x. y) on 1hc curve is equidistunt from the
origin and the interseclion of the x axis with the normal to the curve at that point.
14. Consider the flow of a liquid through a hole in 1he bouom of a container. If /J(t) is 1hc height of the liq•
uid above the hole. then the velocity oft.he liquid emerging from 1he hole will be given by 1.1 = c(2gh) I/:?_ where g
524 Chapter 11 / Ordinary Differential Equ;ition,;

is the acceleration of gravity (9.81 m - s- 2 ) and c is an empirical construu, which is about 0.6 in many ca~cs.
The rale of change of rhe volume of liquid in the Lank is d V = -a v = -ac(2gh) 112• where a is the cross-
dt
sectional area of the hole. UJ.i ng rhe fact thar V = foh A (h )dh. where A (h) is I.he cross-secr.ional area of the

di, ac(2~Ji) 112


Lank al a height h, show that - = ~ . This rcsull is known as Torricelli's law.
dt A(/1)
15. This problem relies upon the prcviou:, problem. A hemispherical Lank of radius 1.00 meters is full of liquid.
How long will ir take all the fluid to flow through a hole of cross-sectional arc.a 1.00 cm 2 ?
16. A lank in1ially contains 1000 li1ers of an aqueous sall solution of concentration 100 gram.s per liter. If pure wa1er
enters the tank at 5 li1ers per minu1e and Lhc solulion flows out at the same ra1e. calculate the concentration
of salt in the solution in lhe tank after one hour. Assume that the solution in the tank is well stirred, ~o that
solution is kept uniform throughou1.

17. Repeat the previous problem if a salt solution of 5 grams per li1cr is used lo flush out the tank.
18. Show that the substitution u = y Jx t:ransfom1s a homogeneous differemiaJ equation into a differential equation
whose variables arc separable. Hint: Let A = I/x in the definition of a homogeneous function 10 write
__!_11 f(x. y) =f (:.. ~) = </J (::).
X X X X

19. Use the method oullined in the previous problem to solve the following differential equations:
(a) dy = x2y (b) dy = xy
dx x3 + y3 dx x 2 - y2
. . . r d\•
20. Show that a d1ffcrcn11nl equ:lllon of the 1orm ~ =- + by + -
ax ---'-'- c with
. a{:) - a b I:- 0 can be rt.-ducc: d to a
dx ax+{Jx+y
homogeneous differential equation by the substitution x = 11 + x 0 and y = u + Yo where x 0 and Yo arc given
by
ax0 + by0 + c =0
ax0 + /J.vo + y =0
d}' 2r + y - 4
21. Use the method of the previous problem 10 solve ---=-- =- ---
dx x -y +I
22. Evaluate et and /j such that Equniion 8 becomes exact if tl is mul1iplied by x" yfl.
23. Show 1ha1 Equation 10 is .n solution 10 Equal.ion 8.

11.2 Linear First-O rder Differen tial Equations

There is an imponant class of differential equations Ihat we didn't men1ion in


Section I. A <lifferential equation is said to be /ineur if every term conlaining the
dependent variable is raised lO the first power and if. in addition. no 1cnn contains
a product of the dependeni variable and any of ii.<; derivatives. Thus,
11.2 Linear Fir~t•Order Difierential Equations 525
is linear. whereas

dy
y-+x=O
dx
and
dv .,
+xv- =0
_.!...
dx -

are nonlinear. lt 1urns out tha1 linear differcnliaJ equations are much easier to solve
than nonlinear differential equations. Furthermore. many natural laws can be ex-
pressed by linear differential equarjons to a high degree of accuracy. Consequently,
linear differential equations occur frequently in applied problems.
The generaJ form of a first-order linear differential equation is

dv
. . . : . + p(x)y = q(x) a<x<b (I)
dx

where p(x) and q(x) are known functions. We can solve this equation in gcneraJ
by finding an integrc16ng factor, which in this case is fairly slraightforward. First
write Equation I as

[ p(x)y - q(x) Jdx + dy = 0


Now muhiply by µ(x). which we hope turns ou110 be an integ:ra1ing factor:

I µ(x)p(x)y - µ(.x)q(x) ]dx + µ(x)dy = 0 (2)

If Equaiion 2 is lo be exac1, ttien we must have 1ha1

-aµ. = -al µ,(x)p(x))' - µ(x)q(x) l = p(x)µ(x)


ax 8y
or
d In µ.(x)
- - - =p(x)
d:r

=
Solving for µ (x) gives µ (x) ef p(.r l ,1,.. If we multiply Equation I by this inre-
grnting factor, then we obtain

dy ef p(.r)dx + p(x)yef p(x) dx = q(x)ef p(.t) dx


dx

Notice th.it the left side of trus rcsuh is the derivative of yef p(x) dx wilh respect
to x, and so we have

-d [ ye- r p(,:) dx] -


-f/ J
( x ) e p(x) dx
dx
526 Chapter 11 / Or(I n, ry Dii'le ~Pri!i,il F.qu,)lion~

or

y(x)ef p{x>d...- = f tJ(X)ef p(.r)dxdx +c (3)

Solving Equal.ion 3 for y(x) gives

•( ) _ -J
) x - e p(x) d.r J ()Jq x e p(x) Jxd
x + ce - J p(.,·) dx (4)

The existence-uniqueness theorem for firs1-ordcr differenlial equal.ions presented


in the previous section tells us 1hal Equation 4 is the general solulion to Equation I
over any interval in which p (,,;) and q (x) are contjnuous.
Let's use 1his result to solve

First divide by x to put this equal.ion into the fom1 of Equat·ioo I;

dy 2 .,
- + -v=x-
d.r .x.

Thus. the integrating factor is I-' (x) = ef p<.r)dx = rf : dx = e 111 J = x2• Equa1ion 4
.1

gives

y(x) = -.r2I J x 4 dx + ,c
x~

as you can verify by direct substituLion.

Example 1:
Figure 11.J shows an R L circuit, which is an clcct.rical circuit containing
a resistance R and an induct.am:e /,. The voltage drop is Ri across the
resis1ance and Ldi jdt acro,s the induc1ance. where i is the current. If £(1)
is the driving volwgc. then Kirchoff's law ror clc<.:1rical circuits gives
R
di .
L-+R1=£(1)
rlt
Figure 11.3
An clcc1rical cin:uit co11~i,1ing of u
re~israncc Rand an ind11c1anc.:e L.
Solve this equalion if E(r) = £ 0 = constant.
SOLUTION: First write lhe equation in standard form :

di R. Eo
- 1=-
dt L l
527

Then ·-J p dx"' =Rt/ L. and Equation 3 give$


i(l)E,Rr/L = J ~o eRr/Ldr + ,:

/:'o Rt/l
=-e +c
R
Let i (0) = i 0 . so c = i 0 - £ 0 / R. and

Figure I 1.4 shows thal i (t) bee< Hlll'S stet1dy at £ 0 / R when 1 » L / R.


UR Rt/L

Figure 11.4
l11c current in Example I plo11cJ again~r
Rr/L.
Example 2:
Find the gcner..il solution of

(sin 20 - 2r cos O)t/1) = 21/r

SOLUTION: Fi~t write the equation a.-.

dr I .
- + r COS() = - SITI 20
d0 2

which is in the form or Equation I. Now ·-J p d:(" = f cos 0 dl-J = sin Ii and

= f sin H cos fJ
111
e'" dH +c

ln1egn1tc by pan:-. le11ing ·•11" = sin O and ··d l'.. 11 1


. t''i ' cos O dO. to gel

And so

There is a slight extension of Equation I. which is always discussed along


with Equation I. The equation is

dy
p(x)y=q(x)y" (5)
dx

C gl
528 Chapter 1 I / Ordinary Oifferenlial Equations

and is caJled Ben10ulli 's equation. When 11 = 0 or I, Equation 5 is just a first-order


linear differential equatjon . When n :/= 0 or I. even 1bough ii is non linear, it can be
reduced 10 the firs1-order linear equation

du
- + (I - n)p(.x)ii = (I - 11)q(x) (6)
dx

by the substitution " = y l-n (Problem I 0).

Example 3:
Solve

dy I 1 2
-+-y+x·y =0
dx x

with the condition y( I) = I.


SOLUTION: Comparing this equa1ion with Equation 5. we see 1hat I.his
is a Bernoulli equation with 11 = 2. so we let u = Y'-n = 1/y . The above
cquaLion then becomes

du I
- - -u=x 3
dx x
The solution to this first-order linear equation is

/I X]
-= - 1- c
X )

or

3
y(x) = 3cx+x 4

(Problem 11 asks you to verify that 1hjs is indeed a solution 10 the original
differenlia.l equation.) The condition y(I) = =
I gives c 2/3. so the particular
solution we arc ..ccking i:-

y(x) = - -3 -4
2.i + x

11 .2 Problems
J. Find the general solutions of
dy 2 , dy 2
(a) - = x - 3x-v
dx .
(b) - + -y = :c 2 + 2
dx x
11.2 Linear First-Order Diffenmti.:il Equations 529
2. Find the generaJ solutions of
dx 3x
(a) - =2y- -
dy y
3. Find I.he solutions of
dy dy ,
(a) x - + y = 2x y = 2. x = 2 (b) - + (tan x) y = cos- x y = -1, x = 0
dx dx
dy {x O$ x < 2
4. Find n continuou.<i solu1ion of dx + 2.xy = f(x). where J (x) =
0
and y(O) = 2.
x2'.:2
5. Find Lhe general solutions of
di . .
(a) (x + y 2.) -dy
dx
=I - = 31 - 5 sin t
(b)
d1
6. Derive an expression for i (I) in Example I for the square potential.

I
O r ..::: 0
£(I)= E0 O~ 1 $ I.
0 I> I

Lei io = 0 and assume that the current is a con1inuous func1ion of time.


r <0
7. Derive an expression for i(I) in Example I if E(t)
. .
? wt
= { £ o Sin t > 0
. Assume 1ha1 i (0) = 0.
8. Consider the two-step k.ine1ic process

This kinetic process might represent radioactive decay or a chemical reaction. The differential equations
describing this scheme are

dA
- =-k 1A
dr
dB
- =k A - 1 k2B
dt

dC = k'>B
dt ~

where k1 and k 2 are called rate constants. Solve the first equation for A and substitute the rcsull into the second
equation and then solve lhe resulting first-order linear di!Terent..ial equa1ion for B. Plot lhe re~ult for various
ratios of k 1 and k2 . Assume thai A(O) = Ao and B0 = 0.
9. This problem presents another method of solving Equation 1. If q(x) = 0 in Equation I. then Equation 1 is
said to be lwmo~e11eous. l.f Equation I were homogeneous, it would be easy 10 solve by separation of variables
10give y(x) = Ac·· f Pd.{. Assume then that the solution 10 Equation I (the inhomogeneous equation) ha!- I.he
same form. bm wi1.h A =. u(x ). and detennine 11(x) and hence the solution to Equation I,
IO. Verify that 1.he substitution 11 = _y l-n into Equation 5 yields Equation 6.
11. Verify that the solution found in Example 3 is indeed a solmion 10 the original equation.

al
530 Chapll'f J 1 .' Ordinary Diii1·rl'nli,1I I qu,111nrv,

12. Pind the gencr.al solutions of

+ 2x = 2r-,.\'"~
dv ., dx
(a) x ~
dx
+ .\' = lr 3.,,~ (b) ,._
. dy

13. Find 1he solution of dy - }' + .re-.!.( vJ


dx . .
= 0 wi1h .r(O) = I.
14. He.re are some tir ·t-order differential cqu:11ions. Find their gcncral ~olutions.
+ \' = x + ·,.
dv 3 3
(a) ~ = x---- (b) -
dy
dx 2
3xy dx Jx + 3_,. - 4
15. Hl.'.re are wmc fir~t-order diffrrcntiLII equa1ions. Find their general!solutions.
d II dv y ( I - 2t) v-'
(a) - = 4t - 2111 (b) ~ + =-- == ·
dt J.x 3 3
16. In chemical kinetic-. and other types of rale proce-;-;cs, you frcqucnl'ly cncoun1cr the scheme A -:;;::. B representing
the intcrcon\'cn,ion of two species. A and D. The ra\e equalion for this in1erconversion can be written as
-dA
dt
= -k 1A + .
k,,B, where J; 1anti k, :.irccallc<l rare constants. Bycon:-crvalH)ll of mass. A(t)
- -
+ B(r) =Ao+ 8 0.
whtre Ao= A(O) and B0 = 8(0). Solw the above equation for AU) and 8(1).
J: k,
17. Another kinetic scheme invoh•cs the sequential decay. A ~ B ~ C. The simultaneous eq11a1ions 1hat

des.cribc rhii, M.:ht:m~ arc d A = -k, A. dB = k IA - k, B. and dC = k, B, Sol vc the fi rsr equal ion with the
dt dt - dr -
initial condition A(O) = k 1A 0,,-k,, - k 2B.
= Ao and ::;ub~1i1utc 1hc rcsull into the second equation to obtain dB
dt
Solve this equation with the initi,tl condition 8(0) = 0. Plut your resul1 for various values of k 1 and k 2 and
interpret lhe result"ing curves.
18. Solve the cquarion y' = I + 2.xy in tenns of an error function.
19. De1erminc the equation of the curve which passes lhrough the point ( I. 2) whose slope at any point (x. y) is
2 - y/.x.
20. A container contains I00 liters of a salt solution whose concentralion is 200 grams per liter. A s.alt solution of
conccnlration 2 grams per liter is aJded to the container at a rate of 10 liters per minute and the efflux from the
container is 5 lilers per minute. Calcula1c the minimum amount of sol1 in the conrniner and when it will occur.
A~sumc 1ha1 the solu1ion is stirred vigorously so that ii is maintainc<.l at a uniform concentration.

21. Solve the equation x 2 dy + xy = sin x along with the condirion y = 2 whe-n x = 1. Hi11r: Leave your answer
dx
in terms of a definile inregraL

11.3 Homogeneous Linear Differential Eq uations


with Constant Coefficients

In the previous section. we found 1he general :-olu1ion of a firsl-ordcr linear <lif-
fcrcniial equation. This is not possible for general higher-order linear differential
equations. We'll sec in this sect.ion. however. that we can solve higher-order linear
diffcrcnt·ial equations if the coefficients are constants. Fonunatcly. a great many
of the differenlial equations that occur in physical applica1ions have constanl co-
11.3 Homo nC'Ou I in •ar Differ nli_., I Eq11_,_1tion~ wirh ConsrJ1n1 ocifil i •111, 531

efficiems. We'll discuss some properties of general higher-order linear differential


equations firsL and then spend lhc rest of this section discu.ssing lho$e with constant
coeffi c ien rs.
A general 111h order linear differential equation can be \\ ritten in lhe form
1

where we shall always a,sume that the a /x) are continuous functions of x on some
interval (a. fJ ). Nole that all I.he lem1s involving y or its derivatives occur only to
the fir,;;t power and that there are no cross terms. lf f (x) = 0, Equatjon I is said to
be hnmogeneous; otherwise it is T1(.111lw11wge11eous. It is some1imes convenient lo
write Eqm11ion I in lhe abbreviated fom1.

Ly= /(x) (2)

where L is the linear differential operntor

An important property of a homogeneous linear differential equation, Ly= 0. is


that if y (x) is a sol utjon. then so is cy(x) whcr~ c is a constant. Furthermore, if
y 1(x) and y 2 (x) are solu1.1ons to Ly= 0, Lhcn so is c 1y 1(x) + c: 2y 1 (..r). or

(4)

because £y 1(x) = 0 ,md L_r 2(.r) = 0. We can continue this process nnd say 1hal
if y 1(x), .r2(x) ..... Yn(x) (n can be any pos-ilive integer) are solutions 10 tltc
homogeneous equation, then ~u i.':i the linear combination c 1y 1(.r) + c 2y 2(x) +
···+ c 11 y,,(x).
We introduced the idea of a vector space in Section 9.5. Recall 1hat a vector
space is a scl of quantities that satisfy a ~et of rcquircmcn1s, laid down roward the
end of Section 9.5. It turns out that the set of solutions of an 111h order homoge-
neous linear differenLiaJ equation forms a vector space. ea.lied Lhe so/111ivn space.
This statement follows directly from Equation 4. for if y 1(,.r) and y 2 (x) are any
two solutions to Ly :::: 0. then soi~ the linear combination c 1y 1(x) + c 2y~(x ). Fur-
thermore. the dimension of the solutton spac..-e of an 11lh order homogeneous linear
differential equation is n. Thus. there must be II linearly 1nde::penden1 solut.ions to
Ly= 0. and a general solution is of the fonn
II

y(x) = L C;Y;(x) (5)


,-== 1

if Lhe y;(.r) are linearly independcnl.


Recall from Section 9.5 lhat a se1 of functions /;(x). = I. 2..... 11 is said to
532 Chaprer I 1 / Ordinary Differential Equa1ions

be linearly independent in an interval (a. /J) if


n
(6)

implies that all the c j = 0. We presented a convcn.icnt test for l.inear independence
in tenns of the Wronskian determinant. Differentiating Equation 6 n - I limes
gives us n simultaneous equations in the c j. which can be wrinen in matrix form as

h
[ J,
1; 1; (7)

!/LI) fi"-I)

If the JjCx) are linearly independenL. then c = (c 1, c 2 , •..• c,JT = 0 is the un.ique
solution to Equa1ion 7. But 1his will be so only if Lhe derenninanl

/1 h f,,

lV(/1, h, ···.Jn)=
1; 1; 1,;
(8)
ft(n-1) 1t-1)
for some value of x io the interval (a, /J). The determinant given by Equation 8 is
called the Wronskian determinant. and Equation 8 is a test for linear independence.

Example 1:
Three solutions to

y"' - 3y" + 3y' - y =0


are y 1(x) = e:r. Y2(x) = xe.r. and y3(x) = x 2r. Are they linearly
independent?

SOLUTION: The coefficients of 1he di.fferenlial equal.ion are continuous


for all values of x (they're consl:lDts) and the Wrooskian determinant is
equal to

e-1' .xt? x2e-1'


W = c ~+x£r 2xr +x 2~
ex 2~ + xe-t 2e.r + 4xe-'" + x 2e-1'
= 2e 3.r-::/= 0
Therefore. 1he 1hree solutions are linearly mdependent over Lhe entire x axis.
11.3 Homogeneous Linear Differen1ial Equations with Constanl Coeffici~nts 533
We'll now spend the resl of the section discussing homogeneous linear differ-
ential equations with consLant coefficients. These are best discussed by means of
examples. Let's srart with

y"(x) + y'(x) - 6y(x) =0 (9)

This equarjon will be saLisfied by a function whose derivarives are multiples of


itself. The function e°x (where a is a consr.anL) is such a functjon. If we substitute
y = eax inro Equation 9. we ger
(10)

The factor c i= 0. so Equal.ion I O tells us Lhat

a
2
+a - 6=0

or that a = 2 and - 3. 1\vo solutions to Equation 9 are e2 r and e- 3.t and the general
solution is

Equal.ion 10 is called the au.ri/ia,}' equarion of Equation 9.

Example 2:
Dc1ermine the solution of

y" + y' - 2y =0
subject lo the conditions y(O) = 0 and y' (0) = 6.

SOLUTION: The auxiliary equation is a 2 + et - 2 = 0. which gives a =I


and -2. The general solution is

Applying the condi1ions y(O) = 0 and /(0) = 6 gi\'cs c 1 + c 2 = 0 and


c 1 - 2c2 = 6. or c 1 = 2 and c 2 = -2. Thus. the particular solution is

If we auempt 10 solve

/'(x) - 2y'(x) + y(x) = 0 (11)

lhc aux..iliary tX!Uation, - 2a + I = 0. gives us only one distinct root a = I.


a2
Thus y(x) = er is one solution. but we need 10 find another linearly independent
C gl
534

sol u1ion in order to have a genern.J solution. It is not uncommon 10 have one solution
in hand and to search for another. This can be done very nicely by a mc1hod called
reduction of order. The solution Lhat we have in hand is y(x) = cex. To find a
second solution, we assume that

( 12)

where u (x) is to be detem1ined. SubsIitute Equal.ion 12 into Equa1ion I I to obtain

The factor e·r 'I- 0. so 11'1 (x) = 0. This gives us 11(..r) = c 1x + c 2• which subslilulet.l
into Equation 12 gives

(13)

The two functions t? and xex arc linearly independent (sec Example I). and so
Equal.ion 13 represents the general solution 10 Equation 12.
All.hough we have inlroduccd the me1hod of rcduc.1ion by a specific example.
I.he method is general.

Example 3:
Find lhc general solu1ion of

y'"(.r) - 3y'(x) + 2y(x) = 0

SOLUTION: =
The auxiliary equation is « 3 - 3a + 2 0. You can see by
inspection that a =
1 is a solution. Dividing a 3 - 3a + 2 by a - I gives
2
a + a - 2. and so 1he ot.her two values of a are a= I and et= -2. TI1c
=
root a= I is repeared. so subsLilulc y(x) u(x)e 1 into the above differential
equation to obtain

(11
111
+ 311")~ = 0
or 11
111
+ 311 11
= 0. We can solve this equation by first lclling 11
11
=;: to gel
:' + 3: = 0. Now we integrate to get
: = ,,,, = e - Jr
ln1cgra1c twice more 10 gel

The complete soluLion is


J 1.3 Hom<,gPfl(•ou, Linear Differential Equ,1tions with Const.ml Cndfi( ii•ril~ 535

where we have simply replaced c3 + 1/9 by another con$1anl, C.i-

Example 3 shows us thal our method of solving homogeneous linear differ-


ential equations with constant coefficienis is ccnainly no1 limi1cd 10 second order
equa1ions. Funhcrmore. we can use reduction of order 10 find the "companion"
solmion(s) of the one solution obiaincd from a repealed rooE of the auxiliary equa-
tion. If a roo1 a occurs n times. 1hcn the solu1ion associated wi1h that root will be
of the form

( 14)

&X
hample 4:
Find lhe general solution of

y'" - 3y" + Jy' - y =0


SOLUTION: The auxiliary equation is a 3 - 3a 2 + 3a - I= 0, or
3
(et - I) = 0, so that we have a uiple roo1 of o- = I. According to Equation 14.
the generaJ solution is
X

as you can verify by direct subs1i1u1ion.

So far. the rools of the auxiliary equation have been real Let's consider the
equation

x"(t) + .r(t) = 0 ( 15)

The aux.i liary equation is a 2 + I = 0. so a = ±i. The general solution in rhi s case is
( 16)

We can use Euler's f□ miula, e 11 =cost + i sin t. to write Equation 16 as


Figure 11.5
x(r) = C:t cos r + C.J sin t ( 17) The func1i crn x(t) = ,c ,1Gos 1 + c_ . in I
plo1tcd ugain.st I for variow. value/ of ,· 1
where c3 = c 1 + c 2 and c4 = i (c 1 - c2). Figure 11.5 shows Equation 17 plotted and c 2 . Note that the mot ion is harmonic
for various values of c 1 and c 2. The rea,;on that the plots are hannonic (sinusoidal in all ca,;c,~.

C gl
536 Chap1er 11 / Ordin.uy Differen1ial Equa1ions

or cosinusoidal) is because x(t) can be wriuen as A cos(wt + <fJ ). where A =


(ci + c]) 112 and <I>= tan- 1(-c.if c3) (Problem 18). Thus, we see that Equation I 5
has oscillatory solutions.
Let's consider the following case, where lhe roots of the auxiliary equation
are a complex: conjugate pair:

x"(t) + 2r'(t) + 2x(t) = 0

TI1e auxiliary equation is a 2 + 2a + 2 = O. so a = - I ±i. The general solution is

where c 3 = c 1 + c 2 and c: 4 = i (c 1 - c 2). Problem 18 helps you show this equation


X can be written as

x(t) = Ae-r cos(/+ ,P) (18)

where A = (Cj + cJ) 112 and <J> = tan - 1( -c-1/c 3). Thu.-.. in this case. the solution
displays damped hannonic behavior (Figure 11.6).

Figure 11.6
The func1ion :c (I) given by Equation 18
pl~tcd against r, Example 5:
Solve the equal.ion

subject to the initial conditions x(O) = A and x(O) = 0. What do these initial
conditions represent physically?

SOLUTION: The auxiliary equation yields a= ±i lJ), and so the generaJ


solution is

The initial condi1ions require 1hat c 1 = A and c1 = 0, so the particular


solution we are seeking is

x(r) = A cos w1
This solution oscillates cosinusoidally in time, with an amplitude A and
=
a frequency of w radians per second or v w/2,r cycles per second. The
drfferentiaJ equation is lhal of a harmonic oscillator and the initial conditions
depict displacing 1he oscillator 10 its amplitude and then just letting go.
11.3 Homogeneous linear Differential Equations with Cons1anl Coe(ficients 537

The equation in Example 5 occurs in a variety of physical applications. II


represents 1he mo1ion of a mass connected 10 a spring that obeys Hooke 's Jaw, a
pendulum swinging through small angles. the electric current in a circuit contain-
ing an inductance and capacitance, and numerous olhers. Let's use the case of a
pendulum swinging in a fix.cd plane for concre1cncs.s. We'll express the cqua1ion
of motion in terms of the arc length s(t) = 10(1), where 0 is the angle that the
pendulum makes with respect 10 1he venical (Figure 11.7).
The potentiaJ energy is given by (see aJso Sect.ion 3.5)
Figure 11.7
A pendulum oscillating in a single plane.
V(0) = mg(/ - I cos0)
The pendulum support is rigid 11J1d
ma..~s-less. Hnd suppons a mas.~ m .
and t.hc force is given by

av I av .
f = - - = - - - = - mgsm(;)
os I a0
which becomes J = -mg0 for small values of 0 . The momenrnm of 1he supported
mass is ms= m/0, so Newron·s equation is

or

( 19)

where w0 = (g / I) 112 is the natural frequency of lhe pendulum. The solution to


Equation 19 wilh lhe initial condition 0(0) = 00 and 0(0) = 0 is 0(r) = 00 cos Wo' ·
(See Example 5.) Physically. this solu1ion depicts lhe back and forth motion of the
pendulum.
We can introduce frictional resistance in a fairly simple way by saying lhat
the frictional force is proportional to but opposite the motion. ds /dt = /d(J /dt .
Equation 19 then becomes

(20)

where y is a fric1ionall coefficient. !Let's solve Equation 20 undc.r the initial con-
ditions 0(0) =
Bo and 0(0) = 0. The auxiliary equation i~ a 2 +ye,+ w5 =
0, and
yields

(21)

You can see lhal the motion of the pendulum depends upon the relative values
of y 2 and 4w5. 4w5
If > y 2• a is a complex conjugate pair and Lhe solution to
Equation 20 is

8(1) = e-;n/2 (c 1 cos wr + c1 sin wt)


538 Lh,1pler 11 / Ordinary Diffc•f!'.111:,11 FqL1,1I i1111,

where w = (4w5 - y 2 ) 112 /2. Applying 1he inilial conditions. 0(0) = 00 and
0(0) = 0, give.'- ( Problem 16)

0(t) = e-yi/l (00 cos wt+ ~~ sin wt) (22)

Using the result~ of Problem I7, Equation 22 can be written as (Problem 19)

-, ) 1/2
8(1) = 0Oe-Y 112 ( I + y-'> sin(wt + </,)
4w-

or

(23)

where 8 = y/2w 0 and <JJ = ,an- 1f(I - 8 2 ) 112 /8]. Equarion 23 reduces to 0(/) =
00 cos w 0 r, as y ➔ 0 ( Problem 19).
Equation 23 is plotted against Wo' in Figure 11.8 for several values of 8 =
y /2.wo < I. No1e that the motion represents damped oscillations and Lhat 0(t) ➔ 0
as I ➔ oo in all cases. which represents the pendulum having slopped and hanging
Figure 11.8 straight down. The motion in this case is ell.I led 1mderdamped.
fa111,uion 2.1 ploned again),! wor for
M:veral vnlui:.; of ,'i --. )' ;2,,,, 1 = 0.20 (long
A.s )' approaches 2t:uo, the oscillations given by Equation 23 become weaker
du.~hed), 0.40 1:-.tu,n ua),hcd), and 0.80 and weaker. un1iJ they disappear ultogelher when y = 2w0 . As y ----> 2uJ0 , Equa-
(solid). tion 23 becomes

(24)

You can see this by taking the limit, !, ➔ I, in Equation 23 (Problem 20) or by
going back to Equation 21, where CJ = -y /2. Being a repeated roOI, the general
solution 10 Equation 20 is

=
and Equation 24 is the particular solution for 0(0) 00 and 0(0) 0. The motion =
corresponding to the case where y =
2wo is called critically damped because it
represents the point where the oscillations no longer occur as y increases.
For y 2 > 4w5.
Equation 21 yields two real vaJues of a. and Lhe general solution
to Equation 20 is

or
11.3 l-lomog n ou Linear Di(fcrcntial Equations wilh Conslan1 Coci/i( i,•nr, 539
where CJ= c 1 + c2 and c4 = c 1 - c2 . The par1icular solutioo for 0(0) = 00 and 0/00
=
0(0) 0 is (Problem 21)

(26)

Equation 26 is plotted against "Jo' for several values of o > I in Figure 11.9. The
pendulum simply approaches its equilibrium position IJ = 0 mono1onically. There Figure 11.9
arc no oscillalions in lhis case, and the motion is called overdnmped. Using the Equation 26 ploucd agai nst Wo' for several
limits sinh x ➔
x and cash x ➔ I as .r ➔ 0. you can readily show that Equation 26 =
value_s of 8 = y /2wo I .1 (solid). 2.0
(long da,,hc.d). -4.0 (shon dashed). and 8.0
reduces to Equation 24 a" ,5 ➔ I. Furthem1orc. you can show that Equation 26 (da.~h-dot).
becomes Equation 23 if 8 < I (Problem 22).

11.3 Problems
I. Arc I. x. x 2 • and x.11 inearly independent? What about I. I + x. I + x 2 • I + x:i'!
2. Arc e'. sinh x. and cosh .x linc.arly independent?

3. Are I+ x. I - x. and x 1 linearly independenl'!


4. Find the general solurion of

(a) y"(x) - y'(x) - 2y(x) =0 (b) y"(x) - 6y't.r) + 9y(x) = 0 (c) /'(x) + 4y'(x) + y(x) = 0
5. Find the general solution of

(a) y"(x) - 4y(x) =0 (b) y"(x) + 2y'(x) + 4y(x) =0 (c) y"(x) + 9y(x) = 0
6. Find the general solu1ion of

(a) y''(x) + 6/(x) = 0 (b) y 11 (x) - 4y'(x) + 3y(x) = 0 (c) y"(x) + 3y(x) = 0
7. Find the panicular solutions to the equations in Problem 4 if y(O) = I and y'(O) = 0.
8. Find the particular solutions to the equations in Problem 4 if y(O) = 0 and /(0) = I.
9. Solve

(a) v"(x~ - 4v(x) = 0 Y(O) = 2. \ (0) = 4


01

(b) /'ex) -
5>•'{.n + 6y<x> o = );<°'> == - 1. x'(O) =o
(c) y'(x) - 2y(x)=0 y(0)=2

10. Find 1.he general solution of y"'(x) - 2y"(x) - y'(x) + 2y(x) = 0.


11. Find the general solurion of _r"' (x) - 6y" (x) + I 2y' (x) - 8y(x) = 0.
12. Given 1ha1 y = x 2 satisfies x 2/'(x) + xy'(x) - 4y(x) = 0, use reduction of order 10 find a second solution.
13. Given that y =x ~ati:dics x 2 y"(x) - xy'(x) + y(x) = 0. ui-c reduction of order to find a .-;ccond solution.
14. In this problem. we will derive a general result using I.he method of reduction of order. Consider the second-
order equation a 2 (x)y"(x) + a 1(x)y'(x) + a 0 (x)y(x) = 0. where a 2 (x). a 1(x). and o 0 (x) are continuous in lhe
540 Chapter 11 / Ordinary Differential Equations

open intervaJ (a. /3). Suppose we know one solution, y 1(x). Assume that a second solution will be of the form
y 2 (x) = u(x)y (x), where 11(x) is to be determined. Show 1bat u(x) sath,lic:~
1

Note t.ha1 1his result is a linear first-order differenLial equation in u'(x) . Let v(x) = u'(.r) ro write
,
v (x) + -·-
a(x)]
[2y;(x) + - - 1
v(x) = 0. Show Lhat
y 1(x) a 2 (.r)

where c 1 is a constanL. which we can take equal 10 I. (Why?) Now sho\v lha1

u(x) = f
exp [-
.,f a,(..r)
r1.:,(.rl
dx]
dx + c2 .
Yj(X)

where we can let c2 == 0 (why?). The second solution then is

f
f
0
exp [- 1(x) dx]
CJ 2 (.r)
Y2(x) = )'1(.r) dx.
Y12(x)

15. Use lhe genera.I resuh of Problem 14 to re-do Problems 12 and 13.
16. Derive Equation 22.
17. Show !hat A cos +B
sin/ can be wrillen as C sin(I
f + rf,).where C = (A 1 + 8 2) 112 and IP= tan- 1(A/ 8).
Hin/: Work backwards from sLn(a + /3) = sin a cos ft + CO!\ a sin fJ.
18. Show that A cost+ B sin r can be wriuen as C cos(t + 1/J ), where C = (A 2 + B 2 ) 112 and VJ = tan- 1(-B /A).
Hint: Work backwards from cos(a + /3) = cos a cos /3 - sin a sin {J.
19. Derive Equation 23 from Equation 22 and show that it reduces to 0(1) = 00 cos wt as y ➔ 0.
20. Show 1ha1 Equation 23 reduces to Equ.a1ion 24 as S ➔ I (y -► 2w).

21. Derive Equation 26 from Equation 25.

22. Show that Equation 26 becomes Equation 24 as 8 - I and Equal ion 23 if 8 < I.

23. In this problem, wewiU show that the Wronsk..iandeterminantofthetwosolu1ionsof a 2 (x)y"(x) + a 1(x)y'(x) +
a0 (x)y(x) = 0 can be wriuen as W = ce- f ,,dx. where p = v 1(.x)/a 2 (x) and c is :1 constant. This re.sull is
known as Ahe/'s form11/a. Start with a2.v;1 + a 1.v; + oOYt = 0 and a2y~ + a 1 + OoY! = 0. Multiply the first
1
y;
of lbese equations by y2 and the second by y 1• then subtract. and show that lhe result can be written as
dW
a2 - + a 1W = 0, where W = y 1y2 - I I
y 1y 2. Now obtain Abel's formula.
dx
=
24. Use the result of I.he previous problem to argue that W 0 either everywhere in the interval (a, fJ) where
a 1(x)/a 2(x) and a 0 (x)/a 2(x) are continuous or nowhere in the inlerval. Thus. if \1/ =/: 0 al some poinl in (a. /J),
then W -:/:- 0 everywhere in (a. {J).
11.4 Nonhomogeneous Linear Oitforen1ial Equations with Constant Coefficients 541

11 .4 N onhomogeneous Linear Differential Eq uations


with Con tant Co ffi cients
In this section, we shall consider nonhomogeneous linear differential equations,

Ly= f(x) (I)

where f (x) i= 0 and

The method of finding the general solution to Equatjon I is based upon the
following theorem:

If Yp (x) is a,ry solution to Equntion l. 1Jie11 Jhe general solution is given by

where y,.(x) is the general Jo/urion to the homoge11eous equwirm Ly= 0.

In Equation 3, y,..(x) is called the complementary solution and yp(x) is called a


particular so/11tio11.
The proof of the above sta1emen1 is fairly shon. Given Yp(x), let Ypi(x) be
any other solution of Equation I. Because L is a linear operator,

Thus, Yp - Ypl is a solution of the homogeneous equation (Equation with


j(x) = 0). and so Yp - Ypl = Yi·• or

Yp1=Yc+Yp

Because Yp 1(x) is any other solution 10 Equation 1. all solutions can be written as
Equation 3.
Equation 4 says that 1he difference between any 1wo particular solutions is a
solution to the homogenous equation. Consequently. two particular solutions 10
Equation I may be quite different. For example, both Yp(x) = 3x - sin 2t and
3x + 4 cos 2x are soluLions to

y"(x) + 4y(x) = 12.r (5)

They differ by 4 cos 2x + sin 2x. which is a solution to y'1 (x) + 4y(x) = 0. whose
general solu1ion is Ye = c I cos 2x + c2 sin 2x. The genera.J solu1ion to Equation 5
is

y(x) =c 1 cos 2.x + c2 sin 2x + 3x (6)

al
542 Chaph•r 1J / Ordm,1ry Oifferenli,11 Equations

It may be fine tha1 Yp(.r) = 3x is a particular solution or Equation 5. but how


do we find a particular solu1ion in 1he first place? There are two me1hods that
arc presented in all 1ex1s on differential equations. These method~ are called the
me/hod of undetennined coe_{Jicients and the method of variation of parameters.
The method of undetermined cocfficienrs is not only limited 10 linear differential
equal ions with consIan1 coefficients, bu1 the function f (x) in Equation I must
be of a ccriain form. which we will learn about later. lls advantage is lhal the
mc1hod is fairly easy to use. The method of variation of parameters is not limited
to linear differential equations with constant coefficients nor is f (.-..:) restricted to
any panicular form. but the method is fairly tedious to use. Consequently. we shall
discuss the method of variation of parameters only briefly. The references at the
end of 1hc chapter discuss both methods in detail.
We're going lo use an almost empirical approach and introduce the method of
undetermined coefficients with a few examples and then develop it into a formal
procedure. Let's consider the nonhomogcneous equation

y'' - 2y' + y = 2x (7)

We can sec by inspect.ion 1hot a particular solution is of Lhc fom1

Yp(x) = a + {Jx (8)

because if we subs1i1u1e Equation 8 into Equation 7, we obtain

-2/J +Cl+ {3:r: = 2:r:


Equa1ing coefficiems of like powers of x on rhe two sides of !his equation gives
a - 2/3 = 0 and /J = 2. and so we see tha1

Yp(x) = 4 + 2x
which you can verify by direct substitution into Equation 7. lf J(x) = x 3 instead
of 2x. then the difTerential equation is

y" - 2y' +y = x3
we would try

)'p(x) = a + fix + yx 2 + 8x 3
Subs1itut.ing 1his into Equa1ion 7 gives

2y + 68x - 2{3 - 4yx - 68x 2 +ex+ {Jx + yx 2 + i5x-l = .-..:.l


Equating coefficients of like powers of x on the two sides gives a = 24. f3 = 18,
y = 6. and 8 = I. and so
11.4 Nn:1l111111ng1•rw,1m Lint·,ir Differenria! rq11,1t11,11, wirh Cons1c1n1 Lc1t-1ru i«·n!, 543

Yp(x) = 24 + 18x + 6..- 2 + x 3


as you can veriry by direct suhst.itu1ion.
Generally. if /(;r) is a polynomial of degree 11, we assume that Yp(x) is a
polynomial of degree 11 and dc1ermine iIs par.une1ers by subs1iruting ii into 1hc
nonhomogeneous equalion and equat..ing lhe coefficients of like powers of x on
each $idc of the resulting equation .

Example 1:
Solve I.he differenrial equation

y" + 1y' + 2y =6 + x 2
SOLUTION: We assume that )'p(.r) =a+ {Jx + yx~. Substituting Yp(.r)
into Lhc diffcrenrial equation gives

2y + 3fi + 6yx + 2a + 1{:Jx + 2yx~ = 6 + x 2

from which we find a= 19/4. f3 = -3/2. and y = 1/2. or

·me complcmentnry solution is

and so tlic complete solurion is

Now lei's consider the equation

Again. by inspcc1ion. you can see Ihar Yp(x) = ae 2x. Substitu1ing this in10 the
differential equation gives

or that a= 1/4 . Thus. we !-CC that a particular solution is yp<x) = /!.J."/4. as you
can verify by direct substitution.

gh r, al
544 Chap1er 11 / Ordinary Oiffcrenrial Equations

Example 2:
Find a particular solution of

y" + 3y' + 2y = 12tell-


SOLUTION: You will find thal Yp(x) = cu2.,; will no1 work because we
need terms of the form xeu on the left side of the above equation. Lei's try
Yp(x) of the:: fonn

Substituting 1his into the differential equation gives

(4a + 4f3)e 2.x + 4/hel..x + (6a + 3f3)e2:r + 6/Jxe'l..r + 2ae2.x + 2{J.re2J:


= 12te!.r
Equating coefficients of e2.x and xelx on the rwo sides gives ex = - 7/ 12 and
/3 = I. or

as you can verify by ctircc1 wbs1itutlon.

Generally, if f (x) is a polynomial of degree n rimes an exponenLial. then Yp(x)


will be of the same form. (See Table 11.1.)
Let's look al another case, and find Lhe particular solulion of

y" + Jy' + 2y = 10 sin x (9)

Notice that if we assume that }'p(x) = sin x, t.hen we·u generate terms involving
cos x when we substitute it into Equation 9. So let's u-y

Yp{X) = (t cos X + fJ sin .l


Substituting this guess into Equation 9 gives

(a + 3/j) cos x + (/3 - 3a) sin x = IO sin x


Equating coefficients of like rerms gives

Yp(x) = sin x - 3 cos x

as you can verify by direct substitution.

rn
11 .4 Nonhomo~:C'1wous Linear DiH(•rt·nri,11 Equa1ions with Constant Lodiicients 545

Example 3:
Find a particular solution of

y" + 3y' + 2y = !Ox cosx


SOLUTION: Using Example 2 as a guide. we'll try

Yp(x) = (a + f3x) cos x + (y + 8x) sin :c


Substiruting this into the djffcrcn11al equation gives

(et + 3/3 + 3y + 28) cos x + (y - 3a + 3J - 2,S) sin x + (/3 + 38)x cos x


+ (8 - 3/3):r sin x = !Ox cos x
from which we find et = 6/ 15. /J = I, y = - I 7 / 15. and & = 3. Therefore.

= ( ~ + x) cos x + ( 3x
17
yp(:c) - ) sin x
5

Lei's look at one last case, and find Lhe particular solution of

y" + 3y' + 2y = tan .x (10)

Certainly assuming lh,H .Yp(x) = a tan x will not work. because I.his will yield
2a sec 2 x tan x + 3a scc 2 x + 2a tan x on the left side of Equation I0. lf we
1.ry Yp(x) = a scc 2 x tan x + /3 sec2 x + y tan x. then ii gets even worse. The
=
problem here is that f (x) 1an x genera1es more and more terms as it is repeatedly
differentiated. The Limiiations on /(x) that we alluded to in the beginning of this
section is rhat repealed differentiation off (x) must yield only a finite number of
terms. AlJ the successful cases lhat we tried above are of I.his nature. Table 11.1
summarizes the forms off (x) and of Yp (x) for which the method of underermined
coefficients is successful. Note that the last entry in Table 11.l includes all the
al.hers as speciaJ cases.
Before we apply the method of undetermined coefficienlS to some mechanical
and elecuicaJ sysrems. we musl discuss an important modification that is required if
I (x) happens 10 be a solution to I.he homogeneous equal.ion. Consider the equation
( 11)

If we assume thar Yp(.r) = ae-~. then we obtain

The problem here is that cu-.x is a solution to the homogeneous equa1ion. We need
Yp(x) 10 be of rhe form such that its derivatives yield e-.x along with some 01hcr
=
lerrns. Lei's try Yp(x) axe-.x. Substituting this guess inlo Equation 11 gives

al
546 Ch;:i pll•r i I l Ordi11,lr\' Oifi , r nti.11 [qu:1rion,

Table 11.1
Forms of Yr(x) to use for various forms of J (x).

f(x)
C11 + ' · · + <',,:rn
f· l'1,\ 0'11 + £11.t' + · · · + Ct 11 X 11
ko + ,· 1.r + · · · + 1·,,x")t'r., (c::ro + a ,x + ... + a,,x" )er,r
(1'0 + c 1x + · · · + t·,tx -.in <vx
0
) (ao + a 1 \" + · · · + a,,x") cos 1,i.\·
or + (fio + /hr + · · · + /3,,x") l-in wx
(r·o + t·1x + CnX COS WX
--i-- · • ·
11
)

<co+ £"1.\ + · · ·-:- l·n.r'1)e 11 sin wx (cto + et 1X + · · · + 0' .t")f"·r CU:... l,J.\'
11

or +(/Jo+ /31,t + · · · + /3,,x"kn sin wx


ko + c,.r + ... + CnX")erx cos (f)X

a = 2. so we see that 2.xe-x is a particular solution of Equat.ion 11. Generally.

_[~] C
if a tcnn of 1he a..":-umed fom1 for Yr(x) in Table I I. I is 1he same as a term in
the complcmenLary solution, lhen all the renns in the entry in Table 11.1 must
be mu lri pl icd by x. and sometimes even a higher power of x. We 're going to
use this result when we discuss an oscillaior in resonance with no damping (see
R Equation 24).
The method of undetermined coefficien1s is applicable 10 a numher of prob-
lems involving mechanical oscillators and eleclricul circuits. We'll use an R LC
Figure 11.10
circuit a" our primary example thgure 11.10). We can model an electrical circuit as
An ,e lccuical ci.rc:uit contuining u
rcs istanc-e. R. an ioducll1Jic•c. L. nnd a having components that resist an clcc1.ric current (a resistance. R). a change in 1he
capacitance, C. current (an inducwnce. L). or a change in vohage (a capacitance. C) (Figure 11. I 0).
The voltage drop acro~s each componenr is given by VR = i R. V1 = l di /dr.
and V =Ji (I) dr i C. where i (r) is I.be current. According to Kirchoff\, law. the
driving voltage of the circuit is equal 10 the .sum of the volwge drops across each
component, and so we write

L-di + Ri +-I / ' i(f)d1


, I
=V
dr

DifferenLiating with respect to lime gives

d 2i di 1 dV
L- + R-+ - i = - ( 12)
dr2 dr C dr

Before we find a panicular solut.ion of Equal.ion 12, let's investigate the


propcnies of the complementary solutions, or the solutions of
11 .4 Nonh1 lmn:-:1 ·11e•rn1, Linear Diff r ntia l Equ,11ions with Con tant Co ffi t i ~,w, 547
I
The auxiliary equation associated wilh Equal.ion I 3 is La 2 + Rex + - = 0. whose
C
solu1ions a.re

= --R-±--
2
(R - 4l/C) 1 12
(X ----- ( 14)
2L

If R 2 :::- 4 L / C, bo1h values of a arc real and the solution to Equal ion 13 i~ of the
form
Figure 11.11
PloL<: tlf i(I) given hy b 1,u:1tiu1 1 1.5 again ~!
( I 5) Rt /2,l for 4l/ R 2C = 1/2 for variou..<:
\'".Jlu.:c-; of L· 1 <1nd c 2 . The bch11vior in thi:;
where (.J = R/1L and b = (R2 - 4L/C) 1l 2/2L = R( I - 4l/ R 2C) 111 /2L. Fig- c;:asc ,~ callcrl ovcrdamping.
ure I I. I I shows i (r) ploned against Rt /2 L for 4 L / R 2C = I/2 for various values
of c 1 and c1 . The graphs cur the horizontal axis one 1imc ar mos1 and correspond
to overda111pi11g. Figure 11.12 shows i (r) for lhc crirically damped case. where.
R 1 = 4l/ Candi (I) = 1c 1 + c1 f)e-Rr/~J.. The curves a.re similar to 1hose of over-
damping.
If R2 < 4L/ C, then the values of a in Equa1ion 14 are complex conjugates,
which we write as --a± iw0 • where cv~ = (4L/C - R2)/2L. The solution to
Equation 13 in this c;1sc is given by

( 16) Figure 1 f.12


Plots of i U) gi\·cn by Equ;;i!irm 15 ag;iin~1
Rr / 21. for 4L / R~C = J.0 fo.r variow.,
Equation I 6 corresponds lo 1mderdamping. as shown in Figure 11. 13. Thus. we valu,_.., of I and r~. lihc bchavior in this
see that R LC circuits provide examples of overdamping. critical damping. and ca.-.e i, c:i lled l'fl I i1:-al tfampi ng.
underdamping. jusl as we saw in Section 3 for mechanical oscillalors.
Now let's go back to Equation 12. the nonhomogcneous equation. Assume
that V (1) .-.-=: F. 0 cos wl. so that F4uation 12 reads

(17)

Rr/2L
Wc"II considenhc case where R = 0 first and 1hcn include the cffccl of rc,i,aancc
later. &1ua1ion 17 with R = 0 can be wriucn as
Figure 11.13
Plots of i I 1) given by El.j11 t11ion 16 a1:!·;1111s1
( 18)
=
Rt /2 L for L tll11 / R 2 .·0 for , ariuu)(
rnlul", of c 1 :ind c~. TI1t: bchaviur in 1J1i.s
case is c::iJkd 1mdcrdampi·ng.
where UJ1~ = I/ LC. Equa1ion 18 cOITL'!'\punds 10 a harmonically driven harmonic
oscillator wilh no r~,i--tam.:e.
Lei's consider 1he case u1 -:J::. w0 firs1. rf w I= w 0 • Table I 1.1 tells us 10 use
ip(I) = CJ cos 1111 + fJ sin wt, in which case we ea.<sily find that et = 0 and /j =
-E0 w/ L (io~ - (o~ i. or

gl
548 Chap1er 11 / Ordinary Differential Equa1ions

The complete solution is the sum of ic(r) and ip(t), or

(19)

We can use Equation 19 to illusa-ate the phenomenon of beats. Suppose initially


thal the circuit is in a quiescent state. with i = 0 and di /dt = 0, and then the driving
't'Oltage is imposed. In this case

and

and so

Wo :j:. W (20)

Now if we let w ~ a>o and wrile w = Wo + €,where€ is small. then Equation 20


becomes

i(r) ~
Eo . wor - sm(a.io
--Ism . + €)t l (2 I)
2l~
We can use the trigonometric formula for sin a - sin {J.

.
sm . f3
a - sm a+/J.
= 2 cos - a-/3
- sm - -
2 2
to write Equation 21 as

Eo . '='
i (r) ~ - sin - - cos w 0 t (22)
U 2
Equation 22 is plotted in Figure 11.14. Because € is small. the period of
Figure 11.14 sin(€ t /2) is large. and x(t) oscillates with a frequency Wo (wavelength 2rr /WQ} with
E.qua1ion 22 plo11cd against, for l = 0 . lO an amplitude that oscillates with a frequency€ /2 (wavelength 4rr /€) . Equation 22
11nd we,= 1.0. illus1ra1ing what is called
illustrates an example of the phenomenon of heats, which occurs when the driving
amp{ irude-mod11lario11.
force is close to the natural frequency of the system, or when a system is driven by
two nearly equaJ frequencies. Figure 11.14 illustrates what is called an omplirude-
mod11lared wave. or amplitude mod11/atio11 (Lhe "AM'' on a radio dial).
If w.:i =win EquaLion 18, then we must assume Lhal ip(/) is of the fonn

(a + {JI) cos Wol + (y +DI) sin Wol (23)

When Equation 23 is substituted into Equation 18, we find that ip(r) =


-A (I cos Wol) /2(.,Jo, or that

1
.( r ) = c cos w 1 + c sin
. iu(ll + Eol, cm, w0 r (24)
1 0 2
2
11 .4 Nonhomogeneous Linear Differential Equation.~ with Constan1 Coefficients 549
When w = WO, the system is said 10 be in resonance. Jn this case. the displace-
ment increases with lime until lhe circuit undergoes failure (Figure I I. 15). The
frequency Wo = ./fTLC is called resonant angular frequency.
Lel's consider the more realistic case in which there is a resistance. The
differential e-quation to solve is

d 2i di I .
L- + R- + - i = - £ 0w sm wt (25)
d,2 dt C
Figure 11.15
The particular solution to this equation is (Problem 10) An illust.ralion of resonan~ in an LC
circuit. The frequency tui:1 is called Lhe
RE XE . resonance frequency.
ir(t) = --f,
2
cos wr + ---:/- sin wr
Z
(26)

where
1
X =wL- - (27)
wC

is called the reactan.ce of the circuit and

(28)

is called the impedance. We can write Equation 26 as (Problem 11)

(29)

where the phase angle <I> is given by

<I> = tan -1 ( - X) = - ran -t (X)


R R. (30)

The complete solution is the sum of ic(t) (Equation 16) and ip(t) (Equa-
tion 29). Because of the factor e-r,1 2 in i,(f), this part of the complete solution
dies our for large values of lime. and the solution is then given by Equation 29.
The solulion ic(t) is called the 1mnsien1 solution and ip(t) is caJled 1he steady-state
solution. Thus. after the sleady stale has been reached, lhe system oscillates with a
the frequency of the driving force, albeit with a phase factor</>. whose magrticude
depends upon lhe resistance. The amplitude or the oscillation is given by

(31)

figure 11.16
The ratio Ra/ £ 0 is ploned against w/wo, wherewo = (I/ LC) 112, for various values The amplirude of the s1eady-$tate currenl
of L 2w5/ R2 (Problem 12) in Figure 11.16. The value of w thal produces lhe greatcs1 in an RLC circuit plotted against w/«>o
for various values of L 21,>5/ R 2• where
magnitude is called the resonance frequency. Note that 1he amplitude remains
wJ=I/LC .
finite, unlike in Figure 11.15. where R is equal to zero. TypicaJly. the experimental
550 Ch,1plt'r I ) / Ordi11.1r~· Differer11i.1I (qu.11ion~

quantity thal is observed is Lhc absorption of energy from electromagnetic radial ion
by an atomic oscillator. 1hc resonance bchavior of a cavity in a laser. or the
amplification of a signal in an electric circui1. among many others.

Example 4:
lr's pmsible 10 dc1erminc 1hc ~1e;.idy-s1a1c solution 10 Equation 25 directly.
We know physically lhat at steady stale the circuit sustains oscillations with a
frcquency equal 10 the driving frequency but wilh u pha.,;c angle rp. Therefore,
~ubstitutc i(f) =
a co~(u>t + rj)) into Equation 25 and determine a and</).

SOL Ul ION: Subs1iru1ing i(t) = a cos(wt + cp) into Equarion 25 gives


., .
- Lw-a c:0,(,,11 + <J,) - Rrva sin (c,;t + q:,) + -a l:OS(r,Jt + rjJ) = - E0w sm
.
wt
C
Now U!-t' ,in(w, + r/>> ~- sin w1 cost/) + cos wr sin <J> and cos(wl + r/J) =
cos u)t cos q, - sin tvt sin tj, to get

+ ( ,.sm
Lw-a </) - RtJ.Jl1 cos {/J - a~111. ff> )·sm fJ>I = - £
C · l),o .sm rut

from which we tind

-owX cos ,p - Rwa sin rp =0

X
ruvX _,;:in <fJ - Rwu co~ rJ> = - E0 w sin wr
The lir~I cqualil)n gives</,= lan- 1(-X/R) =-
tan- 1(X/R). We can
determine cos ,:p and sin r/> from lhi~ expression for tan tf> by using
R Fisure I I .17. where we cons1ruc1 :i righ1 triangle such tha1 tan rJ, = XJ R.
The hypotenuse is equal 10 Z =(
R 2 + X 2) 111. and so
Figure 11 . 1 7
. .
A geometric aid to the dctcm1ina1ion of
Equn1io11 32.
and s1n(-,P) =- sm q, = -X (32)
z
Substituting in10 1hc second of the nhovc two cqua1ions gives,,= £ 0 / Z. in
::igreemenr with Equation 29.

There is actually an easier way to do Example 4. This me1hod uses complex


exponentials rather 1han sincs and cosine~. lr!:i efficacy results from the fact that all
derivatives of t' 1"'' produce eiw:. whereas derivatives of ~incs and cosines alternate
be1ween the two. Let V (r) = E0 ei,,,,. so thar Re IV I = £ 0 1..'.0!". wt. The response of
1he R L C circuit to V (t) can be expressed as i (f) = / eiw(l+tJ, ). where / and <P arc
11 .4 Nonhomc 1i~t•1wous Linear I )iffi•n•n!i.11 fqu.11iom wilh l• m~r.int Lm·ffo ic•111~ 551

to be: dctennincd. The response 10 1he actuaJ voltage. Re I V(t)\, is Re {i(t)I. We


write i (r) in rhc norarion

(33)

where I = /</<!>is called a plwsnr. Solving Equation 25 is equivalent to dctennin•


ing I. Subsli lute V (t) = £ 0/M and Equal.ion 33 inlo Equation 12 to obtain

1 . I .
-Lw-I + 1wRI + -I= 1w£0
C

Solving for I gives

I= i Ea = Eo = Eo2 ( R - i X)
iR - X R + iX Z

The magnitude of I is £ 0 / Z and its argument is cp = tan - I ( - X / R). so that the real
pan of I is

i(/) = Eo cos[wr - 1an- 1(X / R)J


z
in agreement with Equation 29 and Example 4. The use of phasors is readily ex-
lended to networks and essentially reduces calculations involving a•c networks to
lhe algebra of complex numbers. Electrical engineers have taken such calcula1ions
to a fine art.
Wc"ll conclude lhis sec1ion by briefly presenting an alternative met.hod for
solving nonhomogeneous linear differential equa1ions. This method. called the
method of variatirm of parameters. is nol Ii mited 10 linear different ia\ equations
with conslant coefficienLs nor must rhe repeated diffrrentiation of f (x) yield a
finite number of 1errns. We shall illusLratc the method of variation of parameters
wilh the second-order equation

(34)

Suppose we know that the gencrnl solution to the homogeneous equation is

(35)

The method of variation of parameters assumes that the solu1ion to Equation 34 is


of 1he form

(36)

Recall I.hat we made similar assumptions when we solved a linear first-order equa-
l ion in
Section 2 and when we imroduccd 1he method of reduc1ion of order in 1he
previous section. Substi1u1ion of Equation 36 into Equation 34 yields (Problem 25)

C gl
552 Chap1er 11 / Ordinary 0iHerenti,11 EqualiOn$

=.f(x) (37)

Because y 1 and y 2 are solutions to the homogeneous equation, the first two terms
in Equation 37 vanish. Equation 37 now becomes

(38)

Furthennore, because we have two functions. u 1(x) and u 2 (x), to be detennined.


we need to impose two conditions on them. The first is that Equation 36 must
satisfy Equation 34, which leads to Equation 37 in the first place. The second is
clearly suggested by looking at Equal.ion 38, which simplifies greatly if we say
that

(39)

because then the second and third tenns in Equation 38 vanish, and Equation 38
becomes simply

(40)

Equations 39 and 40 constitute two s.imulraneous equations for u'1 and u;. We
can solve these equations using Cramer's rule to gel

(41)

and

(42)

where W(yi, y 2) is the Wronskian determinant of y 1 and y 2 . Because y 1 and y 2


are linearly independent, W ¥ 0 in the interval in which a 2 (x ), a 1(x ), and a0 (x)
are continuous. Integrating Equations 41 and 42 gives us u 1(x) and 11 2(x), and
therefore a particular solution to Equation 34.
We'll illustrate the method of variation of parameters with two Examples.

Example 5:
Find the general solution of

y"(x) + 4y(x) = tan 2x 0~ X < 1t /4

SOLUTION: The solution 10 I.he homogeneous equation i~


11.4 Nonhomoi-teneous linear OiffeM1!i.-1I Equa1ions wilh Cons1an1 Coefficients 553

Yc(x) = c 1 sin 2.x + c2 cos 2x


and W(y 1• )'2) = -2. Using Equations 41 nnd 42 with y 1(x) = sin 2r and
Y2 = cos 2x gives us
,
" 1(x) = -I cos 2x tan 2x = -2I sm. "_
.w.
2
,
ll2(X) = -iI sin. "··
.w. tan 2x

So
11 1(.r) = - -4I cos 2x
11-,(x)
-
= ~4 sin 2.x - !4 ln(sec 2x + tan 2.r)
and

.
vP(x) = - -4I cos 2.r ln(sec 2x + tan 2\") 05:x<rr/4

The general solution is y(x) = Y.(x) + Yp(x) . Why do you think that there
is a restriction 1hat x < ,r / 4?

Example 6:
Find the general solution of

SOLUTION: The complementary solution is

So. using Equations 41 and 42 wi1h y 1 ::::: e-.r :md _r 2 = xc-.r. W(y 1• .Y2) =
e-2.r.and

The particular solution that we obtain from this is

The general solution is


554 C.h.iplt·r 11 ,. Ord111.11y l)i(r<·r1 •nti.tl fq11 ,1tion~

Notice that neither Example can be handled by the method of undctennined


coefficients.

11.4 Problems
. h 3
I . Exp Iam ow bolh -4 + -2I .\' and l'., + -43 + -2I .r can be particular
. . f ,,
solutions o \' -
.
3v
.
,
+ .,.. .\' =x .
2. Given tlrnl x - I is a solution of:./' + 3/ + 2y = 2.x + I, find I.he general solution.
3. Given that x -1- 2, x + sin x. and x + I - sin x are solutions to a certain nonhomogeneous linear .second-order
differential equation, find the general ~olu1ion.

4. Use 1hc mcr.hod of undetermined coefficient.~ 10 find u particular solution of

(a) y" - 2y' + 2y = xe-' (b) /' - y =x 2


5. Use the method of undetermined cocfficienls 10 tind a particular solution of

(a) y" + y = x + e - .r (b) y'' - y = 2r 3 - .\·

6. u~ the method of undetermined coefficients to find a particular solution of


(a) y" + y = ex (b) y" - 3y' + 2y = x 3
7. Use the me1hod of undetemiined coefficients 10 find a particular solution of

(a) y"(.r)+_v(.r)=sinx (b) y"(x) - y(x) = t:'

8. Find the general solution of y" .,- y' - 6y = x.


9. Find the generaJ solulion of v" + 4 .v = 6 cos lr + 20 sin ix.
to. Show that Equal.ion 26 is a particular solulion of Equation 25.
JI. Show that Equation 26 can be written as Equation 29.

12. Show 1hat Equation 31 can be written in lhe form


Ra_ I . _ 1
Eo - [ L 2w~ ( w u.>0)2] 1/~. where WCJ - I, LC.
I+ - - - - -
R2 w0 w

The ne.o tell problems deal wit!, mrious mechanical sysrems.

13. Consider a frictionless hannonic oscillator (wi1h = I) driven by an external force f (I)= A sin w1, so 1ha1
m
2
. wt. SI10w t ha, 1hc pan1cu
d .'< + w 2x = A 510
-~ ' Iar so Iu11on
. f or uJ ::/=, uJo 1s
. xp(r) = ., A ~ s,n
. wt.
0
d,~ w0 - <1.r '-

14. Suppose thnt 1he oscillator in the previous problem is iniLially in its quic~l:cnt stale ( x(O) 0 . .i(O) O) = =
and then rhe driving force /(1) = A sin wt i~ imposed. Show 1ha1 1hc resulting complete solution is
.r(I) = ,A ., . .
(wti sm wr - (JJ sin <,Jut).
wo(w5 - w-- )
IS. Use the resuh of the previous problem to i.llustratc the phenomenon of bcaL,; . Hint : Let w ~ <t'Q and u$e the
rrigonomet:ric relation sin a - sin fJ = 2(cos(a + ,8)j2]fsin(a - tl)/2).
I 1.4 Nonhomog nt>ow, Linear Diffcrenl ial l:qu,11:io1ls with Cons1an1 Cc ((j ients 555
16. Show thai the solution to Lhe equation in Problem 13 for w = wo is
A
x(t) = c 1coi; wt + r 2 sin w0t - - t co~ Wu'
2wt)

Use this result to illuslrJtc the phenomenon of resonance .


. . . . . . d 2x dx .., .
17. Consider a dnven hannomc oscillator w1Lh a damping lerm -~ + y - + WoX = A Sinn,. where we have
dt" dt
ser m = I for convenience. Show rhat the general ~olut.ion is

yQA cos Qr + (Q 2 - wJ)A sin wt


x(t) =e-Y'i:!(c 1 cos wt+ c 2 sin wt) - - - - - - - - - - - - - -
y2Q2 + m2 _ '"6,2
where w 2 ::- ,,,i - y 2 /4 > 0.
18. Show that the steady-state solution for the previous problem is
x~,(r);;
A sin(Qr + <I>) ,, • where q;, = 1an- 1[vQ/(Q-') - 2
w0 )1.
2
IY 2 n 2 +(Q--WciH 112
1

19. Use the resuh of the previous problem to illustrate rcsonam::e with :1 damping factor.

20. Start with the diffcrcnl.ial equation in Pmblcrn 17. multiply by d.r/dt. and derive

d
[ ,
I d X ,,- (t>ijX-
-dt -2 ( -dt ) + - 2 + y
'] dX
(-)
dt
•~ dX
= -dtA sin Q,

Give a physical interpretation 10 each of rhcsc terms. (Remember that we have set the ma,;s = I.) Show that

I 11:r/n (dx )2
= ---
(2rr / Q) o
y ~
dt
dt

y/\2Q2
= ---=---------
2 I (w5 - Qlf'. -r y 2 f2 2 I

in the steady state. Show that lhis qu3llt.ity has a rna:<imum ar n = Wo· Wh:11 does this mean phy1-il.'ally'!
21. Referring to the previous problem, the average ,Jte at which lhe driving force is doing work i1- ~ivcn by

[( dx"'
dt
) A sin n,] . :i,·e
Show that this is equal to the average rate at which energy is dissipated .

22. A simple yet useful model for the absorption of electromagnetic radiation by an electron in an atom is 10 a-;~umc
2
. boun d c Ias11ca
LI1at the e Iectron 1s . II y by thc nuc 1cus an d so obcys the equation
· 111-d x + my- dx + mw 1 x =
d1-1
d1 0
Ee cos wr. where m is the mass of the e[ectron, e is ir:; charge. l!Jo is Lhe ··natural" frequency of the elastically
bound clcc1ron, £ ,is the ampliLude· of Lhe radiation. and y is the radiurio11-damping tcm1. The nature of I.he
energy dissipated through y is a continual rerad.iation of some of the ahsorbed energy. Show that rhe averc1ge
el 1:.·2,t>2y
rate of absorption of energy is given by (see Lhe pre,•ious problem) / (w) =- , , ~ ., . Plot
2m(y~w- + ({JJ11 - w 2 )-)
I (w) against w for small values of y and discuss your result.
556 Chapler 11 / Ordinary Differ<"nri;il [qu,llions

23. Show I.hat n solution to Ly= / 1(.x) + fi(x) is Yp 1(x) + )'p2 (x). where Yp 1(.r) is a solution 10 Ly= J 1(x) 3nd
Ypz(x) is a solution to Ly= h_(x).

24. Show lhat the particular solution to y"(x) + y(x) = f (x) can be expressed as Yp(.r) = fox /(z) sin(x - z)dz..

25. Derive Equation 37.


26. Use the mc1hod of variation of parameters 10 solve y 11 (x) - 2y' (x) = tr sin x.
27. Use the me1hod of variation of parJ.meters 10 solve y' (x) + y(.t) = csc x .
1

-.r
28. Use the me1hod of variation of paramelers lo solve y" (x) + 2y' (x) + y(x) = :__.
X

29. Use the mc1hod of variation of paramct.ers 10 solve y"(x) + 2/(x) + y(x) = e_,. In x. x :/= 0.
30. Use a CAS to verify your solutions 10 Problems 26 through 29.

11.5 Some Other Types of Higher-Order


Diffe rential Equa tions

Up 10 this point, almost all the higher-order differential equations that we have
solved have had constant cocfficienls. There is no gcner.d procedure for solving
higher-order linear differenrial equations wirh variable coefficient<:- as !here is for
firs1-order equations (Section 2). bu1 there are n few special cases that occur
frequently enough in applications that it is wonh discussing them here, We'll
discuss Lhrcc types of higher-order equalions in this section:

I. The Euler, or Cauchy. equnlion which is of the form

where 1he "/· are conc;1an1s. Note that 1he power of x in each 1cm, is the same
as the order of the deriva1ivc.
2. Equa1ions. even non linear ones, in which lhe dependcnl variable. y. is missing.
An example of this case is

(2)

3. Equations. even nonlinear ones. in which Lhe indcpendcn1 variable, say x. is


missing. An example is

yy" - 2(/) 2 + 2y' = 0 (3)

We'll learn how 10 solve each of these types of equations in turn.


Let's look at the Euler-Cauchy equi..11..ion

2
x y"(x) + 4xy 1
Cx) + 2y(x) = 0 (4)

m
11.S Some Other Types ol Higher-Order Difl1•r◄ -nl1.1I ~quations 557

From the nature of each tenn. it is clear that the substilut 10n y (x) = xm migh1 yield
a solution. Substituting y(x) =
x 111 into Equation 4 gives

m(m - 1) + 4m + 2 = 0
or m = -2 and -1. So, the general solution to Equation 4 is y (x) = c,x- 2 + c 2x- 1•
Generally. if the second-order Euler-Cauchy equalion is of the fonn

(5)

then the equation form (which we'll call the auxiliary equation) is

(6)

Uthe two roots of Equation 6 are distinct.. Lhen we obtain two linearly indcpcndenl
so!ULions. as we did above .
The following example shows that Euler's equation can be tran~formed into
one with constant coefficients.

Example 1:
Show that the substitution x = e= transforms Equatjon 5 into an equation
with constant coefficienK

SOLUTION: We use I.he chain rule to write

dy dy d ~ 1 dy
-= - - =--
dx d z dx x d'-

and

i 2y _ d ( I dy)- _ I dy + I dz d y
2
= _ .' dy + I c/1 y
dx 1 dx x dz. x 2 dz x dx d, 2 x 2 dz x 2 dz 2

Substiluting these results into Equation .5 gives

(7)

hample 2:
Use 1he result of Example I to :-olve Equation 4.

SOLUTION: The transformed equation is

y"(z) + 3/(z) + 2y(z) = 0


558 Ch,1ph•r 11 / Ordin,uv D!((erent ial Equa1ions

The auxiliary equation is a 2 + 3a + 2 = 0. whic:h give~ et= -I and a= -2 .


The general solution is c 1L' - ' : + c2e-~. or c 1x- 2 ;. c~ .\ -t. in agreement with
our above result.

Problems 3 through 6 have you find solutions lo Equa1ion 5 when 1he roots of
Equation 5 are repealed or when they arc complex conjugates.
Now let's consider diffcrcntiul equations in wh.ich lhe dependent variable y is
missing, sut.:h as Equation 2:

In this case. we simply let _v' = p(x) and the equation reduces to a tirst-order
equation in p(x) . Equation 2 becomes

dp ,
- = p(p- + l>
dx

or

p2
In - - . ,
I+ p-
= 2x + c 1
Solving for p gives

y(x) = sin- 1(11 112 ) + c3


= sin- 1(cJe-') + c3

Example 3:
The differential cqualion for an ideal cable 1hal is suspended between two
horizontal poin1.s and hanging under its own weight is

d-y1 = et [I+ (dy)-


- -
1] 1/2
dx dx

where a is a i.:ons1an1 Ihn1 tlepcnds upon the tension in 1hc cable and its
weight. Solve 1his equation for_\ (0) = 1/a and dy/dx = 0 ar x = 0.
I 1.5 Sn1111• Other Typ• oi Hlgher-Ord -r Di i ren1ial [qu.1ti1111,
0
559

SOLUTION: The dependent variable is missing. so we let y' = p and write

The solution to rhis equation is

or

The condition r = 0 when .r = 0 gives us 11 = I. Now solve for r to obtain

eiu., - I
p= =sinhux
1e'"
y
Ano1her inrcgrJtion gives
-a a
y(x) = -1 cosh etx + c
a

If y(O) = I/rt. then c = 0 anc.l we have

.\'(X) = -aI cosh ax


The curve is calloo a ca1e11a1y and is ploucd in Figure 11.18.
figure 11.18
I .
A ca 1c.n:uy, y (x ) = cosh c,x, 1s the
CJ.
The next type of equations that we shall discuss are those in which 1hc :-hap(; of an idc:il c:iblc su ~pcnded al both
independent variable is missing . Equation 3 is an example, since it does not cnch.
contain x. We solve these by a kind of reducrion of order. where we lhink or y
as the independent variable. and p as a function of y. If we lei y' p and wri tc y" =
dp dp dy dp .
as - = - . . . :. . = p-. then (Equation 3)
dx dy dx dy

becomes

dp
p ( y--2p+2 ) =0
dv

Cert~inly. ,-, ::;;: 0. or y(x) = c. is a solution 10 Equation 3. Another solur1on is


given by

_!!}:_ = 2dy
p- 1 y

or ln(p - I)= In y 2 + c 1• or p = I+ c2 y 2 (where c: 1 = In cJ. Thus. we have


560 Chap1er 1I / Orcli'nJry Differential Equarions

dv
___:_ = I + C2Y 2
dx

which gives upon in1egration

I -I
- - tan Jciy = .\'. + C3
.Jci
or

y(x) = - I tan(J<;;x + c4 )
F2

Example 4:
The equa1ion

call'e d 1he Poisson-Boltzmann equation, occurs frequently in colloidal


science and biophys-ics. 11•~ a fairly good approximation for the electrosw1ic
potential <J, at a distance x from a charge surface that is in contact with an
electrol'y,tc solulion. Solve I.his equation under the condition lhm </J(x) - 0
(and drp/dx - 0 as x ➔ oo) and 1ha1 (J,(0) ct,0 . =
SOLUTION: The equa1ion is missing 1he independenr variable x. so we
let d 2q,/d:r 2 = p dp/d(/) to get

p
2
= 2 cosh <I> + c,
or

-d<t,
dx
= ±(2 cosh <t, + c 1) 112
Using the fact that both et, - 0 and d<J,/dx - 0 a.~ x ---1- oo. we find that
=
c 1 -2, or tha1

dtp = ±(2 cosh et> - 2) 112 = ±2 sinh P.


dx 2

Lntegrate again. using ,t>(O) = <t,0 , lo obtain

whe.rc we chose the negative !.ign because</) ➔ 0 a" :r ➔ oo. This cqua1ion
can be solved for f!Pl 2 :

m al
11.5 Some Or her Types of Higher-Order Di rt •rential Equa1ions 561

</)
\ \
\
\ \
\. '\

Figure 11.19 shows. rt> ploned against x for several values of ip 0 .


'' ',
....... ' .........
....... --- :::::...:-- -
X

Equations like the one in Example 4 occur often enough Lhal it's wonh seeing Figure 11 . 19
another way to solve them. Suppose we have the equation The polenLial calculated from the Poisson-
Boltzmann cqua1ion (Example 4) plotted
.T for t/>r,
11gain,1 =
0.20 (solid). 1.00 (Ions
(8) dashed). and 3.00 (short dashed).

Multiply by dy/dx. use the relalion

~~
2 dx
(dy) = (dy) (d y)
dx
2

dx
2

dx 2

and Lhcn integrate bolh sides of Equal ion 8 to obtain

-I ;· -
2
d (dy)
dx
-
dx
2
dx = f f(y)-d.x
d.x
dy
or

2I(dy)
dx =
2
I f(y)dy +c (9)

Of course. 1his is equivalent to wha1 we did in Example 4. but in a slightly different


procedure.

11 .5 Prob lems
1. Solve the equation x 2y"(x) - 2xy'(x) + 2y(x) = 0.
2. Solve lhc equation x 2y"(x) + 4.ry'(x) - 4y(x) = 0.
3. Suppos.c Lhc root~ of Equation 6 are repeated. In that case, we use rcduct.ion of order 10 find a second solution.
Use 1hi., met.hod 10 solve the equation x 2y"(x) - Jxy'(x) + 4y(x) = 0.

4. Show rhal if the two rooLc, of Equu1ion 6 are equal tom. then the two linearly independent solutions are x 111 and
xm lnx.
5. Suppose Lhc roots of Equation 6 :ire complex conjugates or each otJ1cr. as for the equarion x 1y"(x) + x/(.t) +
y(x) = 0. Show rh:'11 the gcnerjl solution to thi" equation is y(x) = c 1xi + c 2:r-i . Now use Euler· .. idcnlily to
=
show that x 1 cos(ln x) ± i sin(ln x) . .i :., O. and 1ha1 the ge11cra\ solu1ion 10 1he above differential equation
is c 1 cos(ln x) + c 2 sin(ln x).

6. Use lhe method of the previous problem to solve x 2y''(.r) + 3xy'lx) + 5y(x) = 0.

C
11 .6 Systems o( Linear Diff renti,il [qu,1liom 563
11.6 Systems of Linear Differential Equations

Syslems of II simultaneous firs1-ordcr linear differcnrial equations such as

( I)

where a _::s .x s /3. occur quite frequently in physict1l applictions. We can wri1e
Equa1ions I compactly in matrix notation:

y' = A(x)y Ci<X<{J (2)

where A(x) is an II x II ma1Tix. We encountered and solved several homogeneous


systems of differential equations in Section I 0.3. where we discussed eigenvalue
problem~ and Lhe methods thal we shall use in this section will be fairly similar.
This section contains linle new materiaJ~ bu1 will summarize the rcsulL, for systems
of linear equations in one place.
Because Equation 2 is linear. if y 1(x) and y 2 (.x) are solutions. then so is
(Problem I)

(3)

Thi$ result can he readily ex1endcd to any finite number of solutions. If A is an


11 x matrix. 1hen there arc II linearly independent solutions to Equation 1. If
II

y 1(..r). y 2 (x) .. . .. y11 (x) arc 11 linearly independent solu1ions. then the general
solution is given by

II

y(x) == I>JY j(x) (4)


j=l

Then vcclOr,, arc linearly independent if

(5)

which implies that all the Cf• arc equal lo zero. Otherwi~. they are linearly
depcndenl.
Equation 5 provides us with a convenient test for linl'ar independence. Let the
jth component of Y;(x) be denoted by .";j(.r). Then Equation 5 c.an be expressed
in matrix fom1 as

C1_\'11+c2Y21+···+r:,,y11I)
C1Y1~ + C2Y2~ . ·.. -l- C,,):112
(0)
= ?.
.- . . -
(6)
( . . - .
l'1Y1n + L'2Y2.,, + · · · + CnY11n 0
564 Chapter 11 / Ordinary Differen1ia I Equations

If we define a matrix Y with elements Yij and a column vector c by (c 1• c2 •••• 1


, en) .
then Equation 6 can be written in matrix form as

Yc=O (7)

We learned in Chap1er 9 that if IYI #- 0, Lhca y-l exists and the only solu-
tion to Equation 7 is c = 0. Furthermore, if IYI = O. Lhen there is a nontrivial
solution to Equation 7. Thus, we have the important result Lhat the solutions
y 1(x). y 2 (x) •... , y,, (x) are linearly independent if and only if IYI = 0. Note lhaL
IYI need be nonzero for any vaJue of x in the interval (a. /3) to obtain the result
=
c 0. and so the condition !YI =/= 0 for any value of x implies Lhat IYI f. 0 for all
values of x in (a, {3).

Example 1:
Show thttt the following three vector functions are linearly independent on
the en1irc real axis:

SOLUTION: We need only 10 show thal IYI "I= 0 for any value of .x:

e-" e'r e-.r


IYI = e-" -<? e-x = -4ex -::/:- 0
~ 2er 3e-.,

Up to this point, our statements apply even if the coefficient matrix in Equa-
tion 1 or Equation 2 depends upon x. For the remainder of this section. we shall
assume that all the coefficients in Equation I are constants. Equation 2 becomes

y' = Ay (8)

Being the solution to a set of first-order linear differential equarions with conslanl
coefficients. it is reasonable lo expect y(x) 10 be of (he form

y(x) = veu (9)

where 1 and v arc to be dcLermined. Substituting Equation 9 in Lo Equation 8 gives


1hc eigenvalue problem (afler cancelling e).__r from both sides):

Av =>..v ( 10)

The eigenvalues 1 are given by IA - .1.. 11 = 0 and eigenvectors ._, are solutions to
Equation 10.
11.6 Systems or Linear Dillerential Equarions 565

Example 2:
Solve 1he equations

)';=)'1+2)'2

Y; = 2y, + Y2
SOLUTION: The coefficient matrix of this syslem is

11,e eigcnvalut!s and corresponding eigenvectors arc A 1 = 3, >. 2 = -1,


= =
v 1 (I. I) T, and v2 (I. -1) T_ The general solution is

or

and

The eigenvalues in Example 2 are real and distinc1. which is the simples1 si1ua1ion.
Now lel·s consider the case of complex eigenvalues. If the elements of A
are real. any complex eigenvalues will occur as complex conjugate pairs, wilh
corresponding complex conjugate pairs of eigenvectors. To see that this is so. stan
with the eigenvalue equation

(A - ,\ I )v =0
and take its complex conjugate,

Thus, we see that,,, v and>..*. v"' are eigenvaJue--eigenvector pairs if the elements
of A are real. The generaJ solution 10 Equation 8 would include the terms

or. if,\= a+ ib, the two terms would be

We usuaJJy want to express our solutions in terms of reaJ funcLions. We can


do lhis using Eu.Jer's formula 10 write Lhis part of the solution in terms of sines

C gl
566 hapler 11 / Ordinary Oiff renli, I Equ.:itiom

and cosine~. bul ii is much easier algebraically to realize 1ha1 if ,.eJ...<- is a solution
to Equation 8. then so arc ils real and imaginary parts separately, giving us two
Iin early independent real-valued solutions ( Problem 12). The ad van1.agc to this is
that we need 10 consider only one of !.he pair of complex conjugates.
Consider lhe pair of equations

( I I)

with .)'(0) = (0. I? . T he eigenva lues arc A = 3 ± 5i and the corresponding eigen-
=
vectors arc v 1 (-i, l) if and \'1 =(i. l)T. lNotc 1ha1 ,· 1 = v~.)
- -
Therefore.

is a solution to Equa1ion 9. We use ei. = cos 5x + i sin 5x to write y(.r) ;is

y (x ) =( sin Sx) e·
1x
+ 1. ( - <.:OS

5x) e· ,.
+ cos5.r ~n5x

Now the reaJ and imaginary parts of y...._(x) are linearly independenl solutions. so
the general solution to Equation 11 t:xprc,sed in terms of real functions is

_ . ( ~in 5x ) ,l..- _ ( - cos 5.r ) ,11 ( 12)


y(.r)-l1 S t +1.2 ,· c. (
co~ .\ sm .,x

The initial conditions y (0) = (0. I) T gives us r 1 = I and c 2 = 0, so the particular


solution 10 Equation 9 is

v(x )
sin 5x ')
= ( cos h
<' .. ( 13)
· Sx

or

y 1(.x) = eJ.t sin 5x and

in component fonn. TI1Us. y 1(x) = e 1 r sin Sx and y 2 (x) = r: 3.x cos 5x are 1wo
linearly independent s.olutions to Equations 11.

Example 3:
Solve lhl! cqualions.

r'=4v- I -2\·,
- I . -

wi1h the initial conditions y(O) = (I. 2)T_


11 .6 Sy~ll' rnS of l i ne.1r DiffL•re1lli,1I [qu.1tio11~ 567

SOLUTION: The eigenvalues arc A =


I ± i and the corresponding
eigenvectors are (3 ±i . 5) T_ Choming ,i,, =
I + i and v = (3 + i, 5) 1• we
have

Y+(X) = ( ,3 +5 j) (I
e +11..-
·

x + 3 sin x ) .r
-- ( 3 cos x - sin x ) e-+1
r . ( cos
• . e
5cos.r )s1nx

The general so\ulioo is a linear combination of the real and imaginary parts
of Y+tx ). so

y (x) = 1•
J cos .r - sin .r )
e.r + c-.• ( cos x +. 1 sin x ) r r
I ( ) COS X 5 S111 X

The initial condi1ions give us 3c 1 + ,:2 = 1 and :5t·1 = 2. or 1 = 2/5 and


(:2 = -1/5. so the particular solution is


L"OS .r - sin .r ) _..
vx)=
(
(
. t'
· 2 cosx - sm x

or

and

in component fonn .

Example 4:
In S~cLion 3. we wrote 1..he equation describing a harmonic oscill::it.or as rhe
secomJ•order equation

In Chapter 13, it will be convenient 10 write this cqua1ion as 1wo coupled


first-order equations. Express this equation as coupled equations in matrix
fonn for 1hc momentum and rhe pm,i1ion of the o,;:cillator. Find the general
solution.

SOLUTION: Let p = m d:r: idt and x = x to gcr rhe coupled equations in


matrix form:

( .\'p ) = ( 0
l/111
-k ) ( p )
0 x

The i.:i!!i.:nvalucs and corresponding eigenvectors are :... = ±iWo and


=
( ±i J..: / c,>o, I) 1 . where to12i k / m. Therefore,
568 Chapter 11 / Ordinary Oifferen1ial fqu,11in11~

is a solution corresponding LO>..= 'Wo· Using Euler's formula. we oht.ain

- -k sin Wo1 ) ( k
U+ = (
WQ + I. -"-'o cos WQI )
cos Wei sin w 0 r
p
The. real and imaginary parts of u+ conslilute linearly independent solutions,
so the gcneraJ solution is given by

X
(
~
p ) -_ (1. ( - W() sin Wol ) + c,- ( .!!:..__
W()
cos WQl )
(
· cos 'ilo' sin w 0 r

Notice that p = -kx and lhal .i = p/m. If we plot p(t) against x(1)
Figure 11.21 parameirically. we obtain a fwnily of cll.i~cs, a..,; shown in Figure 11 .21 . The
A parametric plot of the momenrum curves (trajectories) in Figure 11.21 represent periodic motion .
p(t) against the displacement x(I) for 11
harmonic oscillator.

11.6 Probl ems

1. Show thal if y 1(x) and Y2(x) are solutions to Equation 2, then so is a linear combination of y 1(.x) and y 2 (.x).

2. Test the following three vector func1ions for linear independence over the .x axis:

3. Solve rhc system of equations

4. Solve the system of equations

.v; = }'J + )'2


Y; = 4y1 + Y2
S. Solve lhe system of equations
11.6 Systems oi Linear Oifferenlial [quaIions 569

6. Solve the syslem of equa1ions

7. Solve the system of equations

Y; = Yi - 2.Yl
y; = 2)'1 + )'2
8. Solve the sys1em of equations

y
,= (43 2)3 y

9. Solve the system of equations

10. Solve the system of equa.tions

11. Solve the system of equations

Y; = 2y1 + Y2 - )'3

y~ = -4y 1 - 3_\'2 - Y:t

.v_; = 4_v 1 + 4n + 2y:i


U. Sbow thal tbe real and imaginary parts of Equation IOare !-epanuely solutions to Equations 9.
J.:, J.:
13. TI1e rate equal.ions for I.he chcmicaJ lunet.ic scheme A ~ B ~ C arc
k.1

dA
-dr = -k 1A +k-,8 ~

dB
-;;; = k1A - (k2 + k3)8
dC
- = k3 B
dr

Solve thc,c equations for k 1 = kJ = 2 and k2 = I with the initial conditions A(O) =
A0 • 8(0) = C(O) = 0.
=
Notice that the sum of these equation.~ is d( A + B + C)/dr 0. Wl1a1 does this mean physically?
570 Chapter 11 / Ordinary Di({erential Equation~

14. Consider the two-mass. three-spring sy~tem shown in Figure 11.22.

Figure 11.22
The two-mass. three-spring system
referred ro in Problem 14.

Assuming 1hat the springs obey Hooke"s law. show that I.he equatjons of motion are

m1i1 = -(k1 + k2)x1 + k2:r2


m2.'i2 = k2:r1 - (kz + k3)xz

where x I and x 1 are the displacements of the masses from their equilibrium positions. Write these equations
as four coupled first-order linear equations.
15. Write the rhird-orc.lcr equation _v"' + a1 _-/' + a 1.'r,1 + a0 y = 0, where the a/' are constanL-;. us three simult.ancous
.
fi rsL-order cquatwns . y = x 1• y ' = x ' = x 2, y " = x r = x>. an d y "' = -a1x 3 - a 1x 2 - u 0x 1• N ow
by Ietung
1 2
show 1h::it the uu."<ilinry equation as. ociated with y'" a 2 y" a 1y' + a 0 y = 0 is the same as the characteristic
equation for the system of first-order equations. What does rhi · tell you?

11 . 7 Two Invaluable Re ources for Solutions


to Differential Equations

The book Ordinary Differe111iol Equations and Their Solutions by George M.


Murphy (see the references at rhe end of the chapter) 1s 10 differentiaJ equations
what a table of integrals is to integrals. The first half of the book reviews lhe
methods that can be used 10 solve a great variety of differential equations, and
the second half of the book lisL<; over 200 pages of differential equations along
with their solutions. A great fearure of this Iist is that it is organized in such a way
that any of the many differential equations included can be readily found. They
are ordered as follows:

I . First Order and First Degree (7 51 equations)


2. Fin.I Order and Higher Degree (423 equations)
3. Second Order and Linear (596 equations)
4. Second Order and Nonlinear ( 196 equations)
5. Order Grenier than Two and Linear (196 equations)
6. Order Greater than Two and Non linear (34 equations)
I I .7 Two Invaluable Re:-.ourr e, (or ~r,luti ◄ m, to Differential Equations 571

The tables include homogeneous as well as nonhomogeneous equations. Unfortu-


nately. this 1960 book is out of print: nevertheless. it is such a great resource that
it is worth checking to see if it is available in your library.
The second invaluable resource in 1he title of this shon section is a Computer
Algebra System such as Mathematica. Matlab. or Maple. Each or lhL·sc progr..uns
can solve differential equations analyticaJly (called symbolically). hx example.
the one-line command

DSolve [ y" [ x ] + 3 • y' [ x ] + 2 • y [ x ] == 12 • x • Exp [ 2 • x l, y [ x ]. x ]


// Simplify

in Mathematica gives the solution

Lo 1he nonhomogeneous differcnijal equation in Example 2 of Section 4.


These programs can also yield anaJy1ic solutions to systems of equa1ions. For
example, the Mathematica command

DSolve [ {x'[ t] == 4 • x( t] - y( t) + Exp [ -t ]. y'{ t} = 5 • x[ t)


- 2 • y[ t] + 2 • Exp[ -t J, x( 0] == 0, y[ 0] == 0), {x[ I], y[ I]}, I]

gives 1he solution

x(t) = - -3e -
16
1
-163 e-·" + -le
4
I _ 1

10 the two )",imultaneous differential equations

with x (0) = 0 and y(O) 0. =


The abili1y of these CAS to solve differential equations symbolicslly is some-
wha1 limi1ed. but you should be aware that these $ystems can also solve differen1ial
equmaons numerically. we·re not going to discuss numerical methods for solving
differen1ial equations here. but they are discussed in most texts on differentiaJ equa-
tions. (See the references 10 Edwards and Penney and to Boyce and Di Prima at 1he
end of the chapter.) For example. Mathematica is unable 10 provide a symbolic
solution for

2_r"tx) + y'(x) + 8/ = 0 ( I)
572 C.h,1plt•r 11 / Ordinary Diffcrcnrial EquJlions

y but the command

NDSolve [ { 2 • y'' [ x I+ y'( x] + 8 • y[ x ]"3 =·= o, y( o I = 1,


y'[ 0 ] == 0 }, y [ x ). ( x. 0, 20 } ]

x provides a numerical solution wilh lhe initial conditions y(O) = I. y'(O) = 0 over
the interval O ~ x ~ 20. which is ploued in Figure 11.23.
Neither of these resources is a substitute for an understanding of how solutions
Figure 11.23 are obtained, bur they are great supplements.
A plot of the numerical solution 10
Equation I with 1he iniriaJ condirions
y(O) = I and y'(O) = 0.
Refe rences

W.E. Boyce and R.C. DiPrima. 2000. Elementary Diffcre,uinl Equations and Boundary
Value Proble,nr, 7th ed .. Wiley
R. Bronson, 1994. Differenti.al Equatio11.S, 2nd ed .. Sc:haum·s Ouiline Series. McGraw•Hill
E.A. Coddington, I 961, lntrod11crio11 ro Ordinary Differemia/ Equations. Dover
Publications
C. H. Edwards and D.E. Penney. 1989. £/emenu1ry Differential £qua/ions wilh 801111dory
Value Pmble11Lr. 2nd ed .. Prentice-Hall
E.L. Ince, 1956, Ordinary Diff~remial Equario11s, DO\'Cr Publication~
D. Lomen and J. Mark, 1986. Ordinnry Differential &,11111ions with Unear Algebra.
Prent:icc•Hal I
D. Lomen and D. Lovelock, 1998, Differe111ial Eq11a1ions: Grophics, Models. Darn. Wiley
G. M, Murphy, 1960. Ordinary Dilferr.•111iol Eq11a1ions and Their Sol111ions. Vun No~lrund
M. Tenncnbaum and H. Pol.lard, 1985. Ordinary Differential Equmions. Dover Publications
The Weierstrass Function: A Function That Is Everywhere Continuous,
But Nowhere Differentiable.

There arc many functions that are continuous everywhere bu1 not differentiable at certain points. One of the
simplesl examples is /(x) =
lxl. which is continuous everywhere, but does not have a derivative at x 0. =
In 1872. however. Wcicrstrass showed that the function
co
f (x) =L hn cos(anJT x)
n=O

where O < b < I and a is an odd positive integer, is continuous everywhere. but differentiable nowhere. We
can use none 01her than the Weierstrass M test (Section 2.5) 10 show that / (x) is uniformly convergent by
=
taking M,. b". in which case we have lb 11 cos(a"rr x)I :::. h". Bccau"e /(x) is a unifonnly convergent series
of continuous functions, J (x) itself must be continuous.
The series obtained from terrn-by-tenn differentiation off (x) diverges for ah> I, which in itself does
not prove that J (x) is not differen1iable, but does suggest caution. A careful analysis of I J (x + h) - f (x)) / Ii
shows that i1s magnitude diverges for ah > I for any value of x. and so f (x) has a derivative nowhere.
What does such a function look like? We can '1 sum an infinite number of terms. but the figure below,
=
which includes 1000 terms in 1hc above summa1ion for a= 3 and b 4/5. gives you an idea.

573

al
CHAPTER 12
Series Solutions of Differential Equations

ln Ihc prcviom; chapter, we learned how to ~olve any linear differential equation
with consIant coefficients and a few !\pecial types of differential equations who~e
coefficients i.Jre functions of x. Generally. however. there is no method that allows
us to tind an analytic solution to a differential equation of rhc form

even if 1.he aj 's arc well-behaved funcIions or x. a1 le,1st in a finile number of steps.
There is a met.hod. however. that docs allow us lo solve the above equal.ion in tenns
of a power series in x. In it:-; simplest application. we assume thai y(x) is of the
fom1
,x,

y(x) = L cwr"
11=-n
then substirute i1 into l.hc diffcren1i::il equ:11ion. and dc1crmine all the coefficient~.
lc11 ). in the power scric~. We can then use and manipulate the power seril!s ..,o]urion
much as we would any simple analytic .solution.
Surely, the coet'licknts a Jin the differenLial equation dc1cm1ine the efficacy of
1his series method. For example. Lhc radius of convergence of 1h1.' .;nlution depends
upon the radii of convergence of the ~cri~s expansions of the cocfti.cicnts !11 jlx) 1-
Funherrnore. the behavior of the coefticienls abour the poi111 x = () determine if
such a power series ex..ists. In most ca~s. the series has 10 be modified somewhat
in order 10 serve a~ ::i snhnion. and wl' ,hall learn how to do this in 1J1is chaprcr.
You may think Ihat rcsoning 10 :series solutions is not a particularly convenient
approach. but ii turns out that many of rhe most important differential equations
of the physical sciences can be solved only in tcnns of infinite series . These
equations are very often sc~ond-ordcr equations with non-constant coefficients
and are named after the ma1hema1icians who introduced them and applied them
to signiticant problems . Thus. we hnve Bessel's equation with Bc~scl functions as
its solutions. Legendre·s equations with its Legendre polynomials and Legendre
funclions. and a host of othel'$. Even Lhough these "'name" functions are formally
defined only through power series. ii is possible to deduce many of Lheir properties
and relations between them. as we shull do for Bessel funcLions in Se.ction 6. 575
576 Chapt ·r 12 / Serie~ Solurions of Differcnri.il Equarion~

In time. you can be as comfortable with Bessel functions as you are with the
trigonometric functions. In fact. if we were 10 define sin x and cos x formally as
the odd and even power series solutions to I.he equation

y''(x) + y(x) = 0
we would still have our myriad of trigonometric identities.

12.1 The Power Series Method

We shaU illustrate the method of solving differentiaJ equations by rhc power series
method with a fairly simple example. Consider rhc equation

y''(x) + y(x) = 0 (I)

We solved this equation several times in the previous chapter to obrain y(x) =
a cosx + b sin x. bul let's assume here that we are unable 10 solve it using the
methods of the previous chapter. Now let's assume lhal y(x) is a power series i.n x:
.:,0

y(x) = L OnXn (2)


11::,:,cQ

Our task then is 10 determine the a,, in Equation 2 such 1ha1 Equation 2 is a solution
of Equation I. Substitute Equation 2 into Equal.ion I to obrain
00 00

L n(n - l)a,,x"- + 2
L anxn =0 (3)
n=O n=O

Notice that Lhe first summaLion here vanishes if n = 0 or 11 = I. so Equation 3 can


be wriuen as
00 00

L n(n - l)a,,xn- 2 + L a,,xn = 0 (4)


n:=:2 n==O

The first summation in Equation 4 written out explicitly is

Using this series as n guide. we can change the lower limit in the first summation
from 11 = 2 to n = 0 by writing lhc summand as (11 + 2)(11 + l)a,,+ 2x 11 • so that
Equation 4 now becomes
00

L[(n+2)(11+ l)o 11 + 2 +anJxn=O (5)


11--0

(Problem I provides practice in changing indices of summation if you need it.)


I 2.1 The Power Series Method 577
We now use the fact that if a power series vanishes identically over some
inrerval, lhen each of lhe coefficien1s of lhe series must equal zero. Therefore.
Equation 5 g·ives us

a,,
0 n =0. I, 2 .. .. (6)
"+ 2 = - (11 + 2)(n + I)
Equation 6 is a recursio11Jvrm11la for a,,+ 2 in terms of a"" As you letn 0, I, 2, ... =
in Equation 6, you get the even-subscripted a's in terms of a0 and the odd-
subscripted a·s in terms of a 1• Thus.. we get rwo separate sets of cocfficient.s.
Starting with n = 0, we have for even values of 11.

Cl~=---=
a2
=-
ao ao
4 ·3 4 ·3·2 · I 4!

The general resull is

11 = 0. I, 2.... (7)

Notice that we have determined only the even-subscripted coefficients (all in terms
of a 0 ). To determine the odd-subscripted coefficients, we start with " = I and we
use odd values of 11 in Equation 6:

a3 at
a,=---= = a1
-
· 5·4 5·4 ·3 ·2 5!

or
(-1)"
a ----a1
2n+I - (211+ I)! n = 0. I, 2 .... (8)

lf we subs1j1ute Equations 7 and 8 into Equation 2. we obtain


00 00

y(x) = no L --,
It ,
( -
.c" + L l11
(-1)" ,
,Xa1+1 (9)
"=O (2.n) . 11=0 (2n + I) ,

Notice that the solution has two arbitrary constants as you would expect for lhe
general soluiion for Equation I. Funhennore. notice that the radius of convergence
of each of the power series in Equation 9 is infinily. 1n fact. the 1wo power series
in Equation 9 are the Maclaurin series of cos x and sin x. respectively. so Equa~
lion 9 is
y (x) = a 0 cos x + a I sin x ( 10)

which we could have obtained easiJy since Equation I has constam coefficients.
Nevertheles.s, we used the power series method 10 illustrate the procedure. Usually

m
578 Chaprcr 12 / S ri o lurion oi Diifor nri.:i l Equation~

you will not be able to iden1ify the resulting power series solution with any known
function. but will have 10 deal with the power serie!. as such.
Suppose now that we did not know 1hat the two solutions in Equation 9 were
the familiar sine and cosine functions from 1rigonomc1ry. We might define 1wo
functjons

and c(x)= L -(- l)"x'.!."


N
-- {211)!
/I )

The first task might be to evaluate tbese series numericaJly a.-. a f unctjon of x and
plo1 them. (This is certainly a lot ea."ier nowadays than i1 was when most of the
special functions were first s!udied.) We then might no1ice thal .dx) =- c(x) and
that r' (x) = -s (x). With a lit1le inspirat.ion, you migh1 notice that

c(x) ±i .~(x) = e±i•.t. = I: -(±ix)"


-,- ll.
n= 0

and so on. If s(x) and c(x) occurred in a number or diffcrcn1 problems. then they
would evcnru.:illy be given names accepted by the m:nhematical communily and
become part of the matJ1ematical I itera11.1re.

Example 1:
Solve 1hc cc1ua1ion

y"(x) + 3xy'(x) + 3y(.r) = 0


by the power scric~ method.

SOL u TIO N: SubstilUlc Equation 2 l lllo the above Cllllatiun 10 ob1ain

~ ~ oc

L 11(11 - l)a11 x"- 2 + 3 L 110 11 .r,; ...... 3 L a x" = 0 11

,,~ n=O JJ:=(l

00

Rewrite the Ii r.-.t sumrnal ion a., L (n + 2) (11 + 1)11 ~ r" 11 : and col lcct tem1s
11::::.0
to ge1

=
Ll(n + 2)(11 + l)a,,. ~ !- 3(11 + l)a,,J.r" =O
n,::,Q

Sc1ting the coefficients or .r" equal 10 zero gives 1he recursion formula:

)
tin+'.!= ---a,, 11=0.1.2 ....
I!+ 2
580 Chapter 12 / Serit..., Sr,lutions ol Diflerenlial Equations

or
oc
ao + a 1x + 3arp + L [(n + 2)(11 + l)an+'.! + 30,,+1 + 0 11 + 2 Jxn+
2
=0
n=O

Thus. we see in th.is case that a0 = 0. a 1 = 0. and


3
a 1--
n+~ - 112 + Jr, + 3 an+ 1 n = D. I. 2 .... ( 13)

But Equation 13 says that a 2 = -a 1 = 0. o 3 = -0 2 /7 = 0. and so on. Every


cocfficienr in Equa1ion 2 is equal to zero. so 1hcrc is no power series solution
10 Equation I I.
Equation I yields two linearly independent power series solutions. while
Equation 11 yields none. The onJy difference between these two equations are
the coefficients of y''(.x) and y(x). so clearly 1here is some propeny of these
coefficients lhat dictates when a power series solution can or can nor be obtained.
This is the subject of the next section.

12.1 Problems
I. Rev,rite 1hc following summations ,-.o Lhat 1hey begin with an 11 = 0 1crrn:
00 .~ ::,0

(a) L nanx"- 1
(b) L n(n - l)anx"- 1 (c) L(n - 2)c11 _2.x"
n~I n~ '.! n=2
2. Determine a gencraJ ex pre sion for a,, in 1cnns of a 0 from 1hc following recursion formulas (n ::: 0 in aJI ca..scs):
2nn II+ 2 an
(a) "n+l =- n + I (b) a"+l = 2(n + \) o,. (c) a,.+1 = - (11 + 1)2
3. Evaluate 11 !! and IO!!.
(2ll+I)!
4. Show that (211 + I)!!= - - - and tha1 (211)!!
211 11 !
= 2",1!.
5. De1em1ine general expressions for n2r, in 1erms of a 0 and a2n+i in terms of a 1 for the following recursion
fonnul~:
a,, " ·• I
(b) a,1+' =- --a 11
(11 + \)(11 + 2) - 4(11,2)
6. Solve !.he equation y' (x) + y (x) = 0 abou1 x = 0 by 1he power series method.
7. If the differential equation is nonhomogcncous. we equate 1he coefficients of like powers of x on both sides of
I.he equa1ion. Solve the equation y' (.r) + y = I + x abou1 x = 0 by 1he power series method.

8. If the differential equntion is nonhomogcneous. we equate the cOt!ftidents of like powers of x on bc:ith sides of
the equation. Solve the equation xy'(x) - y = x 1 eX about x = 0 by 1hc power series met.hod.

9. Solve the equation ( I - x:\r'1 (.r) - 2.xy'(x) + 1y(x) = 0 about .r = 0 u$ing the power series method.

10. Solve the equation y" (x) - 2ry' (x) + 4 y(x) = 0 abou1 x = 0 using the power ~cric" method.
12.2 Ordi11.1ry Points and Singular Points of Oiffr•rPllliaf Equations 581
00 00

11. Staning with rhe two infinite series s(x) =L


n:O
(
- I)"+x I)!
(2,1
2n+l
and c(x) =L
,,=0
---.
(-l)nx2n

(2n)! ,
show 1ha1

s 2 (x) + c 2 (.r) = I. If you can't do ii in general. show that it's true for sequenriaJ powers of x. Similarly.
show thal s(2x) = 2s(x)c(x).
12. S1arti.ng wi1h s(x) and c(x) of the previous problem. show that

S(.X) X) 2 5 17 7
-c(x) = X + -3 + -15X + -315
X + · · ·.

13. Show tha1 the radius of convergence of each of the series solutions in Example I is infinity.
14. Determine the radius of convergence of each of 1hc following series:
cc oc ( I " 2'1
<a>
n
I::c-w+'::._ ->x (b) I:
ir=l ,i (211)! n-=O
15. Detemiinc the radius of convergence of the following series:
n
'°" :2n._ '°" ~.
0C II 00
(a) (b)
L.., L.., II
n=O ni;;;:I

16. Show that t11e equation x 2y"(x) + X)-''(x) + (.t 2 - ¾)y(x) = 0 ha«. no power series solution about x = 0.
17. In all our examples. we have found a so\u1ion about x = 0. Suppose we wish to find a solution about some other
'-
point instead. say x = I. We could subs1i1u1e y(x) = L a 11
2
(x - 1) D.J1d then de1ennine 1he a,, in the usual
,1=0
manner. but it is easier 10 let z. = ,r - I and convert the differential equation so 1ha1 z is the independent variable.
Then find the power series solurion about z = 0 and then substitute x - I for;;. Transform t.he independent
variable of the equal.ion xy"(x) + y'(x) + xy(x) = 0 10 z = 2.x - I.
18. The solutions lO 1.he equation y"(x) + y(x) = 0 are sin x and cos x. an odd function and an even function.
respectively. There must be some property of the differentjaJ equation lh.a1 gives this result. Show !hat if y 1(x)
is a solution. then so is y 1(-x), and y 1(-.x) is a cons1.an1 multiple of y1(x). Show that this consrant is± I, or
thal y 1(x) mu.'it be either an even or an odd function x.

12.2 Ord inary Point and Singular Points of


Differential Equatio ns
Consider the linear differential equation

A(x)y"(x) + P(x)y'(x) + Q(x)y(x) = 0 (I)

where A(x). P(x). and Q(x) are polynomials containing no common factors.
Suppose we wam to solve Equation I in s.ome interval containing the point x 0 .
1f A (x0) =/- 0, then the point x 0 is caJled an ordi11ary poin1. In this case. we can
divide Equation I by A(x) to obtain

y"(x) + p(x)y'(x) + q(x)y(x) =0 (2)


582 Chapt r 12 / olution o( Oific-rcn ti,1 1 Equations

where p (x) = P (x) / A (x) and q ( x) := Q (x) / A (x) are continuous in the ncighbor-
hood of _\' 0 . The functions p(x) and q(x) in Equation 2 will u~ually be r.:itio~ of
polynomials if A(x). P(x). and Q(x) arc polynomials. in which case Lhey will
have series expan$ions about the point x 0 . We can acrually relax rhe condition 1hat
A(x ). P(x ). and Q(x) in Equation I be polynomials to the condition that p(x) and
q(x) arc analytic al x 0 : thru is. they have convergent series expansions about x 0 . If
p(.x l and q (x) arc analytic at x 0 . lhcn x 0 is an ordinary point of Equation 2.
We're now ready 10 stale (bu1 not prove) an important theorem concerning
ordinary poinls.

If x 0 i.'< a11 ordinory point,~(


y''(x) + p(x)y'(x) + q(x)y(.r) = 0 (3)

the11 thl' g<'m'rol .wlution ,~( Eq1w1im1 J crm::.·isr.\· of lh'O linearly i11dt'p£·11dn11
1mwer .11 ·ri,· ~
:x, 00

( ) = C1 °'L..,,""' a,, (x
y x - \ 11 )" + c:1 °'L..,,""' b 11 ( .\. - . )"
.\o
11 ·O n- 0

where c 1 and c 2 an· arbirmry amsfmrfs 1111d rlu· a ·sand b 's are deren11i11ed
ox in Secrin11 I. Furthermorl'. the two po11·er .\erfr,s are 1.mulytic 1.11 .r x0 =
and rlu: mdi11s ofcorn•agem ·1· of each one i.1 ut least n1· /ar.~e t1s the 111i11in111n1
of the radii of ronverf.!e1u:e of the seril's expansions f~{ p(x) and q (x ).

Let's consider the solution of

( I - x\v,,(x) - 6xy'(x) - 4y(x) =0 (4)

in the ncighborhood of 1he poin1 x = 0. This point is :rn ordinary point and
6x 4
p(x) = ---~ and q(x) = - --1
I - x~
I - X"'

We can ob1nin the series expansion of p(.\) i.md q(.d by using 1hc geometric series
for ( I - x 2)- 1• in which ca-.e we have

p(.x)
<)C

= -6 L x?.11 + I and 4(.r) = - 4 L..,,;C '°' , .


IX

n=O n=0

The rndius of convergence of each of these series is (the same as that or the
genmetric ~l·ric:-), so we expect 1ha1 the two serie-. :,.olutions to Equation 4 converge
for at lcasl ~ x I < I. It 1ums out lhJl rhc power seric:,. ,olu1ions 10 Equation 4 arc
(Problem I J

y(.r) = a0 ~
L_)n
00

+ "'11
I).\"
'°'
0C

+ a 1 L..,, -211 -+
- J x-~n T·
3
I
(5)
,,~n 11=0 •

which fortuitously can be expre!-.'-cd in closed fonn as (Problem 2)


12.2 Orclm.11~• Poinr, .rnd ~i11,!.(ul.1r l'llml ~ oi Di ft r nrial quation 583

(6)

The ra1io test shows 1ha1 lhc two series in Equation 5 converge for lxl < I (Prob-
lem 3).

Example 1:
Predict the radius of convergence of each of I.he power series solution.~ to

(I+ 4.r 2),v ''(x) - 8y(x) =0


about the point x =0.
SOLUTION: The point x = 0 is an ordinary point and
oc
p(.x) =0 and q(x) = - - -8 - = -8 L...,(-1)"(4x~)"
" "I

I + 4x 2 n.-0

The radius of convergence of p(x) is infinity. bur that of q(x) is 14x 1 [ < I.
or Ix\ < 1/2. Thus. we ex.peel the two power series solulions 10 converge for
at least lxl < 1/2. It lum-. our 1ha1 Lhc two solutions arc given by ( Problem 4)

The first tcnn has an innnitc r<1dius of convergence (ar /eusr as l.tf£e as
lxl < 1/2) and th.:.:it of the ., l!cond tcml is In.

Equation I and lhe equal.ion in Example I of the previous sect.ion yielded 1wo
linearly independent power series solutions because x = 0 is an ordinary point of
both equations. FUJ1hem10re. the radii of convergence of p(x) and q(.x) in each
case is infinity. and the solutions converge for all x. Equation 11 of the previous
section.

4x 2_/'(x) +Ox+ l)y(x) =0 (7)

however. is a differenl s1ory. In this case. Lhe point x = 0 is Ml an ordinary point,


and conscquenlly q (x) is nol analytic about x = 0. Thus. we should.n"r be surprised
that we found no power series solution .
A point that is nol aa ordinary point of a differen1ial equal.ion is called a
singular point. For the case in which A(x), P(x). and Q(x) in Equation I are
polynomials with no common factors, a singular point is a point al which A(x) = 0.
Notice rhar x = 0 is a singular point of E.qua1ion 7. All other points of Equa1ion 7
are ordinary points. We expressed our statement about the radii of convergence of
the two power series about an ordinary point in 1erms of rhe radii of convergence
584 Chap1er 12 / Seiil', ~olutions ot OiilPu•ntial Equ,11ion~

of p(x) = P(x)/ A(x) and q(x) = Q(x)/ A(x). We may express it in tenns of
singuJar points as well.

lf x ::::: xo is an ordinary poinl of

A(.r)y"(x) + P(x)y'(x) + Q(x)y(x) = 0


1hen 1he general .wllltion cons fats of Jwo linearly independent power serit'S
~ 00

y(x) = c 1 L a,,(x - xot + c1 Lb 11 (x - xo)"


n=O

The radius of convergence of each series is at least as great as the distance


from x 0 to 1/ie 11eares1 singular poi111 (real or complex) of 1/Je differential
equation.

For example. consider the equation

(J - x 2)y''(x) - 2xy'(x) + y(x) = 0


The poi.nt x = 0 is an ordinary point, and Lhe nearest singular points are .x = ± I.
Consequently. the radii of convergence of the two power series solutjons are al
Ieast I . The so Iut ions are guaranteed to converge wi lh in rhe intervaJ Ix I < I. This
is the same as the interval of convergence of p(x) = P (x) / A (x) = -2x / ( I - x 2)
and q(x) = Q(x)/A(x) = 1/(1 - x 2 ).
Consider now I.he equation

(I+ x 2 )y (x) - 2ry'(x)


11
+ y(x) = 0 (8)

Once again. x = 0 is an ordinary point. The nearest singular points are x = ±;.
The disLance from x = 0 to x = ±i is I, and so the two power series solutions
10 Equation 8 converge (al least) for Ix I < I. Even Lhough the singular points
are complex, I.hey still govern the convergence of the solutions, and also the
convergence of
' 2x I
p(x)=--- and q(x)= - - ,
1 +x 2 I+ x~
for that mancr (see Section 2.7).

Example 2:
Ocie rm inc the radii of convergence of I he power scricl, so Iuti ons to Eq uarion 8
about the ordinary point .r = 2.

SOLUTION: The singular points occur al x = ±i. and as Figure 12.1


Figure 12.1 shows, the distance from the point x = 2 to the poi.nis x = ±i is Js, so we
A pictorial aid to dcrermining 1.hur lhc expect the two power series in (x - 2) to have radii of convergence equal to
dj:.iance from the poinr .r = 2 10 the points J5 (al least).
.r = ±i is Js.
12.2 Ordinary Poi111~ and Singular PoinlS of DiifC'rC'nti.11 Equations 585
The solutions lo differential equations often have dramatic behavior around
singular points: for example. Lhe solution may blow up or go Lhrough rapid
oscillations. On the other hand, there may be two linearly independent solutions
Lha1 are finite at a singular point. For ex.ample, Lhe Euler-Cauchy equation

.r 2 y''(x) - 2.xy'(x) + 2y(x) = 0

hns the 1wo linearly independent solutions y 1(x) = x and y2 (x) = x~. bolh of which
arc well behaved al the singular point x = 0, while the Euler-Cauchy equation
x 2 y"(x) + 2.xy'(x) - l2y(x) =0
has rhe solutions y 1(x) = x 3 and y 2 (x) = x- 4 • one of which diverges al x == 0.
The functions p(x) = P(x)/ A(x) and q(x) = Q(x)/ A(x) diverge at a sin-
= =
gular point x x0 because A(.r) 0 [provided A(x). P(x), and Q(x) have no
common factors involving x 0 ]. The nature of the solutions in the neighborhood of
a singular point depends critically upon how strongly p(x) and q(x) diverge !here.
In particular, if p(x) and q(x) diverge at x = x0• but

Lim (x - x 0 )p(x)
x---0
= finite and lim (x - x 0 ) 2q(x)
.r-0
= finite (9)

then we are able 10 tind lwo linearly independent solution:-;. Equations 9 mean
that p(x) nnd q(x) do nor diverge more strongly than 1/(x - x0 ) and 1/(x - x 0 )1,
respec1ively. The point .r = x 0 in Lhis case is called a regular s;n~ular poi111. 1f che
singular point x =
x 0 is not a regular singular point. it is called an irreg11lar singular
point. The srudy of lhe solulions to di fferentiaJ equations about irregular singular
poinls is a fairly advanced topic ::md will nor be discussed here. Fortunately, most
of Lhe equations of applied mathematics do nol involve irregular singular points.

hample 3:
Cla~sify the poinr.s x = I, 0. and -1 for the equation
x-\1 - x)y"(x) + (I - .r)y'(x) - 4x_v(x) =0
SOLUTION: The point x =-\is an ordinary poinL The point x == 0 is an
irregular singular point because

. ( ) . x( I - x) . _,
I1m xp x = 11m l
x->O .1-->0 x-(1 - X)
= .x-0
11m x - =

No1e that lim x 2 q (x) is finite. but all it Lakes is for one of Equations 9 to fail.
The poinl x =
I is a regular singular poinl because

. . x-1
hm(x- l)p(x)= hm - - = 0
x-1 x-1 X3
12.3 Series Solu1ions Near an Ordinary Point: Legendre's Equation 589
Starting with 11 = 0 and using even values of 11, we find that

a~= -
a(a + I) a
0
2·I
(a - 2)(a + 3) a(a - 2)(a + l)(a + 3)
t14 =- 3 4
. "2 = 4
! ao

(a - 4)(a + 5) a(a - 2)(a - 4)(a + l)(a + 3)(a + 5)


a6 =- 5.6 a4 = 6! uo

or generally

a(a - 2) · · · (a - 211 + 2)(a + l)(a + 3) · · · (a + 211 - I)


01.n = (-1) n - - - - - - - - - -- - - - - - - - - - a 0
(211)!
n ~ I (3)

Similarly. we find that

- l)(a - 3) • • • (a - 2n + l)(a + 2)(a 4) •••(a + 2n)


a2n+ I = (-})"(a
- - - - - - - - - -- - - - - - - -- - - - a l
(2n+I)!
11 ~ I (4)

The two linearly iodependenr solutions or Equation 1 then are


00

Y,(x) = L a2,r-r211 and (5)


r1=0

For arbitrary values of et. both series or Equation 5 diverge at x = ± I. Ye1 in


=
many applications. x cos 8, where B is the polar angle in spherical coordi nares.
Consequently, we of1en require a solution 1ha1 is finite at the poims x = ± I,
which corresponds 10 0 = 0 and 9 = 11 /2. It turns out !hat this is actually easy to
accomplish. If we set a equal 10 zero or a positive integer. then one of the two series
in Equation 5 will truncate, resulting in a polynomial. Thus. we will always have
a polynomiaJ solution to Equation I for inleger values of a. This may be easiest
10 sec if we write our the expressions for I.he first few coefficienl.S in Equa1ions 5
expl.icitly. Expressions for a 2 • n 4 , and a 6 appear above in Equation 3, and using
Equation 4 for n = l 11nd 11 =
2, wi: have

(a - l)(a - 3)(a + 2)(a + 4)


a5 =- 120 a,

So. if we let a= 0, we get a2 = 0. a4 = 0, or 02,r = 0 for n ~ land thus y 1(x) = a 0 .


If a= I, we get 03 = 0, a 5 = 0. ora:?J1 +1 = 0 for11:::. I and so Y2(x) = a 1.x. lf a= 2.
a 2 = -3a0 . and then a2i, = 0 for n ~ 2. and so y 1(x) = a 0 + a 2x 2 = a 0 ( 1 - 3x 1 ).
If we continue in this manner. we generate a se1 of polynomial solutions to
Equation I. If we denote these solutions by f,,(x), where 11 is the value of a, then
we have (setting the arbitrary constants a 0 and a I equal to unity)
591

Example 2:
Show 1har the first few Legendre polynomials arc onhogonal over 1he interval
(-I.I).

SOLUTION: Using Equations 7.

because of the parity of the P11 (.,· ). In add it ion.

J -I
I
P0 (x) P1(x) dx = -[
2
11
0
(3.r~ - I) dx =0

You should keep in rni nd I ha1 when a = 0. I. 2 .... in Equations 3 through


5, we ob1ain a polynomial s.olulion from one of Equations 3 and 4 plus an infinite
series that diverges al x = ± 1 from lhe al.her. for example. if a = 0. Equal ion 3
leads 10 ~ 1(.r) = I. while Equation 4 gives

( I 0)

This infinite series is equal lo ( I/2) 1n l( I + x) / ( 1 - x)] (Problem 6). It is mstomary


to denote this second solution by Q0 (x). so we write

I I+ X .\" ) .\""~
Qo(.r) =- In - - = x + - + - + --. t I I)
2 1-.r J 5

Thus. the complete solution tn Equution I with u =0 is o linear combination of


Po(x) and Q 0 (x):

( 12)

For a = I. the complete solution is (Problem 8)


( I 3)

where
( 14)

gl
592 Chaprer 12 / Serie Solutions o( Differential fqu.ition~

Generally.

Q,,(x) = P, (x)Q 0 (x) + a polynomial of degree 11


1 - I

X
The first few Q ,.(.x) are plotted in Figure 12.3. As the functional forms indicate.
they all diverge at x =±I.
The general solution to Legendre·s equal.ion whea a is zero or a positive
integer is
Figure 12.l
The first few second solutions. Oo(x) 11 = 0, I. 2, ... ( I 5)
(solid), Qi(.r) (long dash), Q 2(x) (shon
dn~h), QJ(x) (dash-dol), 10 Legendre's where Pn(x) is the 11th degree polynomial that is finite for all values ofx and Q,,(x)
equation ploued agninst x. Nore thar they
all diverge a1.r = ± 1.
1s a logarithmic function that diverges at x = ±I. Because we want y(x) to be finite
=
at .r ± l in the vast majority of applic.ations. we choose c 2 0 and ·work wilh =
the Legendre polynomials. This happens so frequently that you tend to forget that
!here are indeed non polynom.ial solutions to Legendre's equation even when n is
4

an integer.

12.3 Problems
t. Derive Equation 2.
2. Extend the polynomial solutions given in Equarion 6 up to J5(x).
3. Show that the polynomials / 4 (x) and f~(x) that you generated in 1he previous problem satisfy Legendre's
equation.
4. Use the polynomi3ls .f4 (x) and / 5(:r) 1hat you generated in Problem 210 dcri\'e expressions for P4 (x) and P5(x).
5. Use Equation 8 to gener.:1.te the tirst four Legendre polynomials .

6. Show that the Maclaurin series of In[ ( I + x)/( I - x) I is given by Equation 10.
7. Show that ln((I + :c)/(1 - x)I is n solution 10 Legendre·s equation for a= 0.
8. Show that Q 1(x) =x Q0 (x) - I is a solution to Legendre's equation for a= I.
9. Show tha1 I.he. coefficient of xn in P,, (x) is (2n) !/2'1 (11 !) 2.
10. Show thal Legendre's equation can be wrincn a-; !(I - x 2)y'(.K)j' + a(a + l)y(x) = 0.
11. In 1his problem, we'll show in genera.I that the Lcgl!.ndrc polynomials are onhogonal over 1he interval (-1. I).
St.art with Legendre's equation in the previous problem for a = n and a = m . Multiply the fi~t by Pm (x) and
1hc second by Pn(.r) and integrate both resuhs from - I 10 + I by parts. Now equale the rcsulL-.; nnd show the
onhogonality.
U. Show that the first few Legcn~ polynomials satisfy I.he recursion fonnula
(n + l)P,,+1(.r) - (2n + l)x P11 (x) +nP 11 _1(x) =0 n ~ I

13. Use the formula P,, (x) == (- l)n ~ ( I - x 2 )" to generate the tirst few Legendre polynomials. This formula is
2,.11 ! dx"
called Rodrigues· Jom1ula .
14. Argue that the solutions to Legendre's equation should be cilher even or odd func1ions of .x (sec Problem 18
of Section I). Are they?
12.4 Solutions Near Regular Singular Points 593
15. Often. in physicaJ problems. x in Equation I is equal to cos(). where 0 is the polar angle in sphericaJ coordinates
(0 S 0 S Tl /2). Express Equation I in lenns of 0.
1
16. Show that
(l-2xt+t) 2 112
=
P0 (x) + t P 1(.x) + ,2 P2 (x) + -• -. 1l1e left side of this equa1ion is called a

generating function of the Legendre polynomials.

12.4 Solutions Near Regular Singular Points


We've already seen that a power series method may not work for solutions about
regular singular points. In these cases we shall assume an expansion of the form
00

y(x) = L awx"+r (I)


n;:0

lf r happens to be an integer, then Equation I is just a power series, bul often r


will not be an integer. The general series given by Equation I is caJled a Frobenius
series, and the use of such a series to find solutions about regular singular poin1s
is called the method of Frobenius. The theory of differential equal.ions tells us that
there is always at least one Frobenius series solu1ion about a regular singular point.
To detennine !he coefficienrs in the Frobenius series, we subst..itute Equation I into
the diffcrenrial equation and equate the coefficients of the various powers of x 10
zero. much as we did in the previous sections.
Let's slart with the equation

2xy"(x) + 3y'(x) - y(x) =0 (2)

If we wri1e Equation 2 in the form

y''(x) + p(x)y'(x) + q(x)y(x) = 0 (3)

then we see that the point x ;: 0 is a regular singular point because xp(x) = 3/2
and x 2q (x) = -x /2 arc anaJytic about x = 0. Therefore. we use the Frobenius
method and substimtc Equation I into Equation 2 10 obtain (Problem I)
ex;

aor(2r + l)xr-l + L [(11 + r)(2n + 2r + l)a,, - a,,_i] xn+r-l =0 (4)


ne=I

The coefficient of Lhc lowest power of x gives

a 0 r(2r + I) :;:: 0 (5)

Th.is equation for r, which is always obtained by setting the coefficient of the lowcs1
power of x equal lo zero. is called the indicial equoiion. Equation 5 tells us lhal
r = 0 and -1/2. We reject the choice that a0 = 0 because we have taken a0 to be
the coefficient of the lowest power of x i.n Equation I. Therefore. we expect our
594 Cl1,1ptn 11 / Series Solutions of Oi(i♦ 'r(·11ti,1I Equations

lwo linearly independent solutions 10 be of the fonn


00 X,

Y1(x) = L b ,x 1
11
and Y2(X) = X-J/ 2 L CnXn
n=O 11=0

The first solution happens to be a power series. but the second is not.
If we set 1hc remaining coefficients in Equation 4 (with r = 0) cqua.1 to zero.
we obtain

II~ I (6)

or (Problem 2)

Similarly. for r = -1/2. we obtain (Problem 3)


2"c0
C =--
11 (211) !

and so the general solution 10 Equation 2 is

2" X"
= ho I:--- +crp-1/2 L---·_
00 XI ')II \ , II
y(x) (7)
,,=0 (2u + I)! o=O (2n)!

where h0 and c0 are arbitrary constant-;. [Do you recognize these series? (Prob-
lem 4)1

Example 1:
Solve the following equa1ion about x = 0:
8.r 2y''(x) + 10x/(x) - (I+ x)y(x) = 0

SOLUTION: The point x = 0 is a regular :,;i ngular point becaust


xp(x) = -45 and

arc bot.h analytic about x = 0. If we substitute Equation I in10 the above


differential cqua1ion. we obtain
~ oc
L[2(u + r)(4n + 4r + I) - 11 u,,x"+,. - L a,,x" +r 1
=0
11:0

The lowest power of x (which is xr) comes from Ihc n = 0 tcnn of 1hc first
sumrnmion, sot.he indicial equa1ion is

2r(4r + I) - I= 0
12.4 Solu1ion~ Near Rl'•gul,,r Singular rnltil\ 595
The rwo root, of this cqua1ion are r = 1/4 and -1/2. Thus, we expect two
Frobcnius series,
:0 00

Y1(X) = xl/.$ L b,,xn and _,·~(.r) = x- 1/~ L c,,x"


u::=.O 11=1)

The recursion formula for rhe coeflicicn1s in terms of r is

a,,
a ------------
"+
1
- u,, + I - r)(4,i + 4r + 5) - I
/I~()

Jf we let r = 1/4 and -1/2 in lurn. we find 1ha1 (Problem 6)


1/J :c'!,/J :c 9/ J
\'1(X)
.
=.r + - 14 + -616
and

So far everything has been fairly su-aigh1forward. Finding series solutions


often involves quite a bit of algebra. but the proc~durc itself is straightforward.
Let's look at 1he equation

x=\ (x)
11
+ Jx y'(x) + ( I - 2x)y(x) =0 (8)

The indicial equation associated with 1his equation is (r 2 + \)~ = 0. and so has a
repealed root of -1. Clearly. we a_rc not going 10 obta.in two linearly independent
solutions by the method that we have been using.
The general t.heory of differential equations tells us that there is at least one
Frobenius series solution about a regular singular point. 1-0 the Frobcnius met.hod
is always going lo give us one solution. Once we have one solu1ion. we can always
tind the other by the me1hod of reduction of order. Recall 1ha1 the method of
reduction of order says that if y 1 (x) is a solution 10 Equation 3. then we can find
anorhcrsolution by. ~suming Lhal y 2 (.r) = 11(x)_r 1(x). where 11(.x) is detcnnined by
subslitutin g J:! (x) in10 Equal.ion 3. Th~ final result for y 1 (x) is (Problem I U.14)

e -( fldx

f
~
)•,(x)
·-
= .v (.r)
1
y (x)
, dx (9)
1
Let's go back to Equation 8.

x 2 _r"(x) + 3x/(x) + (I - 2x)y(x) =0


which we saw above has the indicial equation (r + 1) 1 = 0, and so has a repeated
596 Chapter 12 / Series Solu1ions of Differen1ial Equations

root of r =- l. If we substitute Equation I with r == - I into Equation 7. we


eventually obtain the simple recursion relation (Problem 12),

2a11-l
an= - - n ~ I
n2

or

so the Frobenius series solution is

\'1(X)
-
== X
-1
'°'
L
oo
-- = -
I
(2x)"
+ 2+
(n!) 2 x
X
2t2
+ -9 + · ..
11:::0

To use Equation 9. we first note that/ p dx = 3 / d;( = 3 In x. so Equation 9


becomes

= y,(x) f ---.,-d_x_ _ __
x + 4r + 6x 3 + O(x 4 )

= V1(,t)J
! x[ I + 4x
dx
+ fu 2 + O(x3)]

Now expand I/[ I + 4x + 6x 2 + O(x 3)] u."ing the geometric series (Problem 13)

n(x) = y 1(x) f [~ - 4 +!Ox+ O(x


2
)] dx

= Y1(x)[ In x - 4x - 5x 2 + O(xJ)l
= y 1(x) In x - 4 - 13x + O(x:!) ( 12)

Thus, we see that a second solution to Equal.ion 8 has 1he fonn y 1(x) In x + a power
series in x. The y 1(x) In x tenn will always occur when lhe indicial cqua1ion has
two equal roots, as in 1his case.
The Froben.iu.s method sometimes runs into difficulty if the 1wo rooL'- of the
indiciaJ equation differ by an integer. ln this case, you may or may not obtain
a second solution of the form y 1(x) ln x. Rather than go through a number of
examples involving lots of algebra. we'll slate the general results for the three
cases in which (I) the two roots of the indicial equation do not differ by zero or
an integer, (2) the two roots are equal. and (3) the two roots differ by an inleger in
the form of a general 1heorem:

Suppose that x = 0 is a regular singular poi111 of the equation


y"(x) + p(.r)y'(.x) + q(x)y(x) = 0
12.4 Solutions Near Regular Singular Points 597
and 1h01 xp(x) and x 2 q(x) have series expansions abow x = 01hr11 converge
for Ix I < R. where R is 1/1e smaller of the radii of convergence of xp(x) and
x 2q(x). There ore two linearly independent sol111io11s rhal are valid for ar
least O < [.r I < R, who1;e Jom, d11pends upon the relative values nf r 1 and r 2.
Let r 1 and r2 be !lie two mo/s of lhe indicial equation. with r 1 ?:. r 2 •

I. If r 1 - r1. 'I- 0 11or mi integer. then


00

)'1(x) = 1xr 1
L anxn (13)
n=O

and
00

)'2(X) = lx1' 2
L b xn 11
( 14)
n,e:Q

00

)'1(.t) = lxlr, L anx11 ( 15)


,r=I)

and
OC•

J2(x) = y 1(x) ln x + l.r!' +1 L b, .x" 1


1
(16)
11=0

:,,.:i

Y1(x) = l.tl'l L a,,:rn ( 17)


,.,,,o
a11d
00

Y1(x) = C)"1(x) In lx l + l.rl' 2 L b,,.r: 11


( 18)
11:::::.0

ln each case, the a·s and the b"s may be delerrnined by substitu1ing y 1(x) or .n(x)
in10 the differen1ial equation and setting the coefficients of the various powers of x
equal 10 zero. In case 2. Lhere is no requirement 1ha1 b0 ::/:- 0 and, in fact, the en1ire
series may be absent. In case 3. I.he cons1ant c may equal zero and so 1he complete
solu1ion is the sum of two Frobenius series. as in case l. Finally. all the power
series in the three cases define funcrions Lhat are analytic at x = 0.
We ·11 illustrate lhe above 1heorem with a few examples to see how rne solutions
confonn to these rcsulL~. Consider l.he equal.ion

4x
2
y'1 (x) + (3:r + l)y(x) =0 ( I 9)

for :r > 0. Here we have


3 I
p(.x) =0 and q(x) =- + -
4x 4x 2
I 2.4 Solu1icms Near R,u 'L41Jr Si n •ul.ir Points 599

for x > (L In 1his case.

1 2 ') 00

p(x) = .'.: and q(x) = - - - =.: I:x"


X · x( I - x) x n=O

Substituting Equarion I into Equal.ion 25 yields the indicia.l equation r 2 + r = 0,


or r 1 = 0 and r 2 = -L The solution corresponding to r 1 = 0 is (Problem 19)

(26)

and 1he solution corrcspondi ng 10 r 2 = - I is (Problem 19)

(27)

No1e lha1 x 2q(x) converges for x < I. and allhough y 1(x) converges for :ill values
of x. y 2(.x) converges only for O < x < I.

12.4 Problems

I. Derive Equarion 4.
2. Show that the n.·1.:ur.;ion fommla b11 =b 11 _ 1/n(211 + I), 11 ~ I, lL'aJ, to b,, = 2" /(211 + I)!.
3. Show 1ha1 the recursion fonnulu fur 1he second solution of Equation 2 is c,, =c 11
_ 1/ 11(211 - I) . 11 :::: I, and that
c,, = 2"c0 /(2n)!.
4. Identify 1he $erics in Equa1ion 7.
5. Using the results of the previou~ problem. show that (2x)-lf2 sinh Jfx und x - 112 cosh ./2r an~ .'solutions 10
Equation 2.

6. Verify the solu1ion~ g iven in f\amplc 1.

7. Use the Frobenius method to di;:tcrminc the solution of


4x 2y"(x) - 2dx + 2)l(x) + (x + 3)y(x) = 0 about the point x = 0.
Determine the indicial eq11mio11 and its 11m mots for the ec111,11ions in Prubh·ms 8 thrr>ugh I 1.

8. 3x~y 11 (x) + xy'(x) - (I+ x)y(x) =0


9. 4x y"tx) + r I - 2.x)y(x)
2
=0
JO. xy''(x)+y(x)=O
11. x 2 y"(x) + xy'(x) + (x::! - 4)y(x) =0
12. Derive Equation 10.
13. Derive Equation 12.
14. Deri\'e the Frobcnius solution 10 Equation 19.

JS. Derive Equation 21.


16. Derive Equa1ion 23.

rn
600 Ch.aprer 12 / Seri •s Solutions of Diffcrenrial Equations

17. Derive Equation 24 by substitutjng Equation 18 into Equatjon 22.


18. Derive Lhe Frobenius series solution (Equation 26) to Equalion 25.
19. Use Equation 9 10 derive Lhe cx.pre~~iun for y 2(x) for Equa1ion 25.
20. Show 1hal Equation 24 can be wrinen as c 1x( I - 2.x + 2..-- 2) + c2xe - 2.~.

12.5 Be sel's Equation

ln Section 3. we studied Legendre's differential equation. whose solutions that


are finite at x = ± I are Legendre polynomials. We stated Lhere lha1 Legendre's
equation is one of the most important equations in lhe physicaJ sciences because
iL occurs in many problems involving spherical coordinates. a:. we shall see in
Chapter 16. Another equal Iy important di ffcren l ial equation is Besse I's cq uatj on.
which occurs in many problems involving cylindrical coordinates. Bessers equa-
tion nicely illustrates the various cases for the relorive va.lues of the roots of Lhe
indicial equation rhat we discussed in Lhe previous section. so we·u spend IJ1is
entire section discussing Bessef"s equation:
2
x y"(x) + xy'(x) + (x 2 - v\r(x) =0 (l)

where v ~ 0 is a parameter that will subsequently label the various solm1ons. called
Besselfimctions. Equation I is called Bessel's equatio11 of order v.
The variable x usually Lakes on Lhe values O:::: x < oo in physical problems.
so we will always take x to be equal 10 or greater Lhan zero. First note thil.l x 0 =
is a regular singular point and that all other vaJues of x are ordinary points.
The functions p{x) and q(x), when Equal.ion I is wrillen in Lhe form l'(x) +
p(x)y'(x) + q(x)y(x) =0, are
1
. I 1r
p(x) =- and q(.x) = l - -:;
X x~
Because xp(x) = I and x 2q(x) = x:! - v 2 are analytic everywhere. we expect the
solutions to Equation I (the Bes.';e) functions) to converge for O < x < oo. and
perhaps for O S .r < oo.
Jf we substilule y(x) = L
a,,xn+r into Equarion I. we obLa.in

(r2 - v2)aoxr + [(r + 1)2 - v2}01xr+I + l[(r + 2)2 - v2]02 + ao}xr+2


(2)
+ · · · + ll(r + 11) 2 - v2 Jan + On-2\xr+n + · · · = 0
The n = 0 term gives the indicial equation because a 0 # 0 (e.ssentiaJly by
definition) and so r = ±v: tl1e n = I term reads [(r + 1) 2 - v2 la 1 =
l(±v + 1)2 - v2 Ja 1 =(I± 2v)a 1 = 0: and the n ~ 2 terms give the recursion
formula for the other a ·s:

11 2: 2 (3)
I 2.5 Bl'~'>('l's Equation 601

Taking \J lo be zero or positive. we see that r 2 - r 1 2v, so depending upon rhe =


vaJue of v, Bessel's equation provides examples of aJI Lhree cases in the gener.t.l
theorem of the previous section.
Many appLica1ions occur in whkh v = 0 or a positive integer, wilb v = 0 being
particularly impon.anr . Thus, let's look at rhe case v = 0 first. If v = 0, we have
=
r 1 = r1 0. According to Equations 15 and 16 of lhe previous section, Lhe two
solutjons 10 Equal.ion I are of the form

00

Y1(x) = L a11x" (4)


11=0

and
00

Y2(x) = Y1(x) In x + x L b 11 x
11
(5)
n=O

We can det.ennine the a,, by substituting y 1(x) into the zero-order Bessel equation.
in which case we obtain a 1 = 0 and (Problem I)

,1 ~ I (6)

Thus, if we denote y 1(x) by the standard noration J0 (x), we have


Jo(x)
J0 (x) = ~ --- :. )2n
00 ( l)'1 (
(7)
~ (11 !) 2 2
n~

Equa1ion 7 defines a Besself11nctio11 of the first kind of order zero. II is easy 10 show
1hat J0 (x) converges for O ~ x < oo (Problem 2). The function J0 (x) is oscillatory.
as you can se~ in Figure 12.4.
We can obtain 1he second solution to Equa1ion I (with v = 0) by substi1u1ing
Figure 12.4
Equation 5 into Equation I. This gives (Problem 3) The i.cro-ordcr He~~ci function of tile fi~t
kind. J0(.( ). ploncd agains1 x.
2 4
y 2(x) = J0 (x) In x + ( -x - -3x + . ..) (8)
4 128

The generaJ form of Equation 8 is (Problem 23)

y,,(x)
- 2
2
= J0 (x) In x + [ x- 2 - -2x -2
4

2 ·4
( I+ - I) + - -x -
2 22 - 42 · 62
6
( I + -I) + .. ·]
I+ -
2 3
(9)
lf we define tbe quantity

I I I II I
H,, =I+-+-+·
2 3 .. +-=~-
L. ( 10)
II ]"" 1 J
12.5 1:1 I' Equa1ion 603
where
Yn(x)

y = lim (Hn -
11--00
In 11) = 0.5772215 · · · ( 14)

is called Euler's cons10111 (Section 3.4). This certainly uninruitive fonn or Y0 (x)
X
is chosen because of its convenient bchavior as x --. co, as we shall sec in 1hc
next section. Figure 12.5 shows Y0 (x_) ploued againsl x. Like J 0 (x ), Y0 (x) is an
oscillarory function of x. Noricc thar Y0 (x) _, - asx ➔ 0 due 10 the In x tenn in
Equation 13. In summary. the general solution to the zero-order Bessel equation is Figure 12.5
The zero-order Bcss:d function or the
( 15) second kind. Y0 (x). plotted ai::ainS'I .r.

Let's now look at the 1.:ase where v in Equation I is a positjve integer. so lhal
= 2.n. an inregcr. A1.:1.:ording to Equations l 7 and 18 of the previous ection.
r 1 - r:.
the two !>Olutions 10 Bessel's equation of integer order are
oc
Y1(x) = xn L (ljXj ( 16)
J=O
and
;:,c

Y2(X) = C)'1(x) In .r + x-n L brxj (ho-:/= 0) ( 17)


j-=d.)

Subs1i1uting. Equal.ion 16 inro Equarion I (with 11 =11) gives (Problem 4)


(-l)iao
n,.
~J
- -a2J-
-+2-2n)
= -(2))(2} - = ------------
22 ij!(n + 1)(11 + 2) - • • (n + j)

The facror (n + I) · · - (11 + j) in the denominator of a 2 ) can be wrincn as


f(n + j + 1)/ r(n + I). where f (;:) is a gamma function (Sec1ion 3.1 ). so a 2j
can be wriuen a,;;

( - l).i r (n + I)a 0
a,.
_,
= -22J - - -- --
J!r(j + I+ 11)
( 18)

It is customary 10 choose a0 to be I/I 211 r (n + I) l. and so Equa1ion I 8 becomes

and one solution 10 Equation 1 is

J,.<x> = L -
...._
- ---- (::2)2j+n
(-1))
( 19)
jc.a f(j + l)r(j + I + n)

Equation 19 defines a Besiel Jim et ion of the first kind of ordt'r n. It ·s easy 10 show
by the ra1io 1es11hat J,,(x) converges for O ~ x < oo (Problem 5).
604 Chapter 12 / Seri~ Solutions ol Oilferen1ial Equalions

If we let n = I and n = 2 in Equation 19. we obtain the first -order Bessel


functions of the first kind:

(20)

and
xl x4 x6
l2(X) = 22 · 2 - -4-
2 - 3!
+ - 7 - + ...
2 · 4!
(21)

The functions J0 (x}, J 1(x). and J 2 (x) are oscilla1ory and arc ploncd in Figure 12.6.
X Because they are power series. J0 (x). 1 1(.r), J 2 (x), and all their deriva1ivcs are
continuous functions of x .

Figure 12.6
The B~scl functions J 0 (:c) (solid), J 1(x)
(dashed), and J 2 (x) (dash-do1). planed Example 2:
against x. Show thal J~(x) = -J1(x).
SOLUTION: We can differentiate a power series tenn by term within its
radius of convergence. From Equal.ion 7.

J()
0 X
oo
=L (-1)" _z,,
---.>;
2211 n!n!
= l+Lo:i (-l)''x2n
2211 11!n!
n~O n~I

and

By comparing the finaJ result wilh Equation 19. we see that lf)x) = -J (x) .
1

Equation I 9 does not give us a second linearly independent solution if we


replace n by -,i. In fact, Problem 6 has you show that

n = I, 2 . . ..

Example 3:
Use Equation 19 10 show thal J_ 1(x) = -J 1(x) .
SOLUTION: Equation 19 with n = - I is

The j = 0 term has r (0) in the denominator. but f(O) = oo. Therefore. the
summation really slarts with the j = I term.
606 Ch,1p11•r 12 / SniE•~ Solution~ oi DiiinPnti,11 I qu,1:io11~

into Equation I with v = 1/2. we obtain (Problem 7)


2)1/'.! (-l)"x2" 1/2
J1;1(X)
-
= (-
,r
L - ----
oo

n-o (2n+l)!
(24)

If we substi1ute Equation 2J into Equa1jon I wilh v = -1/2. we obtain (Problem 8)

(25)

These two series are linearly independent since one starts with x I/'.! and lhe other
with x- I/"!.. Thus. we can wrile the general solution 10 Equation I a'i

(26)

where J 112 (x) and J_ 112 (:r:) are given by Equations 24 and 25. respectively.
h turns out that J 112 (x) and J 1,i(x) can be expres~cd in terms of sin x and
cos x. From Equal ions 24 and 25 we see that

2 ) 1/2 . 2 ) 1/2
J1/2(X) = ( 7T- X Sin X and j_ lf](X) = ( 7C X COS X (27)

Note that J 112 (x) converges for al I values of .r. but 1ha1 J _ 1/'!. (x) diverge-:: a1 x = 0.
r
The IWt> funcrions J 1 2 (x) and J .. 112 {x) are planed in Figure 12.8.
figure 12.8
TI,c Bessd fu nt.·t inm, ./ 1f2 ( x) hOlid) and
J 1.~rx) (d:isht.-d) plo11ed againsr x.

Example 4:
Show lhat 1 112 (.r) and J _ 11'J.(x) are linearly independent.

so LUTIo N : We' 11 form the Wronsk ian detenn inant. so we need rhe
dcrivative.s of J l/2 (x.) and J _ 112 (x ). Using Equations 27. we have

112
, ,(x)
1 11 = ( -2 ') ( C.OS X - sin-
- x)
- rrx 2.r
2
J_ 112 (x) = -
1 (
-2 ) l/ ( .
sin COS
x -i- -')- X)
rrx _r

and ~o 1he Wronskian determinant is

sin x
I('
\V - -
- 2 1COS X X _
7T
-
j fl X '-In X COS X

x l/2 2x 12 X IJ 2 2 r3/2

I
= - - yf:0
X
12.S Bcssl'l's Equation 609
equation is very convenient to know:

The general solution to this equarion is

(43)

We wiJl refer to this equation a number of times in later chapters.

Example 5:
Use Equation 42 to show lhat J..,(ix) and Yv(ix) arc solutions 10 lhc modified
Bessel's equation. Equation 35.

SOLUTION: By comparing Equations 35 and 42. we see rhat a =0.


y = I. v = v. and /3 =i. Therefore, according to Equation 43, the general
oolution to Equation 35 is
y(x) = c 1J,,(ix) + C2 Yv(ix)
which we acluHlly write as

Example 6:
Consider a mass supported by a string swinging in a plane. Let the lengt.h
of the string be played out at a constant rate b. so 1ha1 the length of 1he
string at any time is I = a + l>t. If 0 is the angle that the pendulum makes
with the vcnical. then Equation 8.2.12 shows that the transverse acceleration
aJong the arc of motion is /0 + 2i0. so that the equation of motion of this
pendulum is
m (10 + 'llB) = -m8 sin 0
or
(tJ + ht)O + 2b0 + g0 = 0
for smal I fJ. Find the general solution of this equation.

SOL U T rON: First let x = a + bt. so that the equation of motion becomes

If we compare this equation to Equa1ion 43. we see that I - 2a = 2. y = 1/2,


=
a 2 - v 2 y 2 0, and {J 2 y 2 = g/b 2 • The general solution is

C
610 Chc1plerr 12 ~..:r,ic•s 5:olu1 1ons of Differ ·nri.11 E_q 11.11rnns

We'll return 10 this problem in 1he nex t ,;;c1.:1ion. but lilOle here Lhal we will
rcrain the Y1[(2{gx) 1l 2/bl 1enn because .r =a+ br. and never equals 1.ero if
a > 0 and b > 0.

In the next. and last, scc1ion of this clrnp1cr. we shall study some more
propcnics of Bessel functions.

12.S Problems
I. Derive Equation 6.
2. Show that J0 (x) given by Equarion 7 converges for all val ucs of x.
3. Show that the ~c:ond solu1ion 10 Equarion I for,_.= 0 i~ given by Equation 8.

4. Derive Equalion 18.


5. Show that Jn (x) given by Equation 19 converges for al I values of x.
6. Show that J_,,(x) = (-1)" J, (x)
1
when II i:-: an infcgcr. Hi,u: Recall that r(.:) = when: is equal to zero or a
negative integer.
'X)

7. Derive Equu1ion 24 by subs1iw1ing y 1(.r) = x 111 L ti,,x" i nio Equation I.


11,::,(1

8. Derive Equation 25 by suli~lituting y 1(x) = x - 112 L a,,x 11


inI0 Equation I.
n=O
9. Show that J"(i.r) is a solution to E<1ua1ion 35.

· Equa11on
I 0. Dcnve • 24 1-rom
• Eq uat1on . r(11) r
. U:-;e the 1dcnlJIY
- .1, 0 . Hmr: .
2
1)
( n + - = :r 112., rc211) .
2.:.i,- 1
11. Derive Equation 25 from Equation 30. s~e the hint gjvcn in Lhe previous problem.
12. Show 1hat 1hc subs1i1ution x = i: Lransforms the modified Bc.,.scl cqua1ion. Equarion 3S. into Bc~,cl's lXJUation
given by Equation 36.
13. Find the general .'-olution or y"(x) + 4.x 2y(x) = 0.
14. Find Ihc general solution of x 1l 2 y"(x) + y(x) = 0.

15. Find Lhc gcn<.·ra.l ~lution of .r\"(x) + 5xy'(x) + x 2y(x) = 0.

16. The following equa1ion occur in a 1renIment of the stability of a flexible vertical rod. y"(x) + o 2xy(x) = 0.
\Vhcre a depends upon the Iinear ma.-.s density, I.he Young ·s modulus. and 1he radius of the rod. Find Lhc general
solution 10 this equation.

17. Consider the differential cquulion y"(x) + p(x)y'(x) + q(x)y(x) = 0. Suppose that y 1(x) and y 1 (x) are two
linarly independent solutions. MulLiply lhc cqua1ion for y 1(x) by J'.!(x) and the equalion for y 1 (x) by y 1(x).
61 2 Chapter 12 / Series Solu1ions of Oiffcren1ial Equations

12.6 Be sel Functions

Often in applications, the first few Bessel functions of the first ki.nd are the most
imponant. Recall that

( 1)

and

(2)

Generally. J,.(.x) is an even function of x when n is even and an odd function


of x when n is odd. Also. J0 (x) = I when x = 0 and J,.(x) = 0 when x = 0 and
n ~ I. Extensive tablesoflhe J11 (x) are available for many values of n and there are
built-in Bessel functions in many CAS. Figure 12.6 shows that J0 (x) and J 1(x) are
oscillatory. each having an infinite number of zeros; or in other words. solutions
to the equation Jn(x) = 0. These zeros are distinct and well tabu.lated, and often
play key roles in physical problems, as we shall see in Chapter 16.
You can see direcLly from Equations I and 2 that

and (3)

We can use Equations 3 to derive a number of integrals involving J0 (:c) and J 1(x).

Example 1:
Show that

SOLUTION: Integrate by parts. letting "u'" = x 2 and "dv" = x J0 (x )dx:


f .xJlo(x)dx = xJ J 1(x) - j 2x 2 J 1(x)dx

For the remaining inlcgral. let ..u•· = x 2 and "du"= J 1(x)dx:


f 2
x J 1(x)dx = -x 2J0 (x) + 2 j :cJ0 (x)dx

= -x 2J0 (x) + 2x J 1(x)


Putting nil this togelher gives

f x 3J0 (x)dx = x 3J 1(x) + 2x 2 J 0 (x) - 4xJ 1(x)

= x(x 2 - 4)1 1(:c) + 2x 2J0 (x)


12 .6 Bessel Func1ion$ 613

Bessel functions have an intcrcsi.jng rype of orthogonality condition. To see


wha1 it is, first write Equation I of Lhe previous section with v = 0 in 1erms of z
ratber than x:

,,d2y dy 2
z.--2 + z- + z
dz. dz
v(z)

=0

The solution to this equal.ion is J0 (z). Now let z. = ax 10 ob1ain

whose solution j5 J0 (a.x). Now let 11 = J0 (et;x) and u = J0 (aj.r). which satisfy the
equations

xu" + 11' + a.;xu = 0 (4)

Xl/
11
+ v' + a~xu
J
=0 (5)

Multiply Equaiion 4 by u and Equalion 5 by u and then subtract to get

x(u"u - uv") + (u'v - uu') = (a~J - a!).wv


I

The (WO tenns on the left can be wrinen as I X(L1 1 V - uv') r. and so we write
d , , "> 2
- ( x(u u - uv) I= (cr-1 - a.)xuv (6)
dx '

which upon imegralion be1ween O and I g-ives

where ]~(a;) and lo(Ci. j) denote Jix) evaluated at X = a; and X = et j. rcspecl..ively.


If a 1 and etj are two 1..eros of J0 (.r). Lhen the right sidt! of Equation 7 is equal 10
zero. Funhermore. a; a;
t= if i -/=- j because the roots of J0(.x) = 0 do no1 repeal,
and so we have

(8)

If i = j. we have

(9)

for any real constant (Problem 5).


614 Ch.1prer 12 / 5 ri ·v fu!ifms ,;( Difi renrial Eq uJtion

Example 2:
Show thaL if a, is n11y ;,.ero of 10 (.r). then

SOL LIT ION: Simply u.-:c Ihc firs I of 8:1uutions 3:

The second solu1ion 10 Bes~ers equation when v ::::: 0 is (Equa1ion I 1 or the


previou., section)

( 10)

Figure 12. I 1 shows J0 (.r) and Y0 (x) plolted against x. The key point is !hat J0 (.r) is
Figure 12.11 well behaved for all valuc.s of x ?.: 0. where;.is Y0 (x J diverges as x --> 0. In ccnain
1nc zero-order Bc,--cl funi:1iu11 s J0(:r l scattering problems we need to know the behavior of J0 (_r) and Y0 (x) for large
(solid) and Yo(x) 1J;1;.,hcd) ploue<J
values of x. Asymptotically.
agai nst x.

( 11 )

and

( 12)

where p(x) and q(x) ➔ 0 as x ➔ oo. Thi.s convenient relatjon between these
expressions is due 10 the "peculiar'" dcfi nit ion of Y0 t.r) in Equation I0. as discussed
in the previou~ scc1ion. The similarity of Lhe asymptotic fonns or J0 (x) ;_ind Y0 (x)
10 cosine u.nd sine functions and the relation t' · r·, = cos O ± i sin f:J has led to the
lklinition of 1hc following auxiliary Bcs$cl functions:

Ht(x) = J0 (x) + iY0 {.r)


( 13)
1-11\'2. 1(.r)::::: J0 (.rJ - iY0 {x)

Thcse auxiliary functions are called Hm,h:I _{1mcrio11s. Their primary use is in
problems involving the scattering of electromagnetic radiation because they h.ive
12.6 Bessel Func1ion_ 615

the following asymptotic fom1s:

( 14)

Just as there arc numerous relations bc1wccn Ihc trigonometric functions. there
arc numerous relations between Bessel functions. For example, we showed above
that J~(x) = -J 1(x). By s1aning with the definition of J,.(:r).

( - ))'1 \")2n ·
(:_
l,.(x) L-
X
= n=O ---
11!r(I +11 + 11) 2
1•
(15)

i1 is sLraightforward to show that (Problem 6)

-d [X -vJ ,,(X) 1= -x -•·1 ••+1(X ) ( I 6)


dx

and that (Problem 7)

( 17)

Equation 17 gives J~(x) = -J 1(x) when v = 0.


We can use Equations 16 and 17 to derive 01her rcla1ions between Bessel func-
tions of various orders . For example. if we carry out the derivative in Equation 16
and then multiply by x'" 1, we obtain the recursion fomlU!a

( 18)

Similarly, if we carry out the derivative in Equation 17 and Lhen divide by x•·- 1•
we gel

( I 9)

Equations 18 and 19 arc recursion formulas involving dcrivmives. We can


obtain a pure recursion formula (one wi1hout dcriva1.1ves) by equating Equa1ions 18
and 19:

(20)

EquaLion 20 gives J,,+ 1(.r) in tem1s of J,,(x) and J,._ 1(.r).

gl
616 Ch,1p1cr 12 / s~•rit"S Solutions of Differential Equations

Example 3:
We found in Lhc previous section thal

1112(.r) = ( -2 ) l/2
sin x
rrx

Use the above fomwlas to find h;2(.x.) and J5n(x).

SOL UT 10 N: Use Equation 16 to find h12(x):

Now use Equarion 20:

112
ls;2(.r)
2 )
= ( -,r.r ( 3 sin. . -x - -
-- 3 cos
-x - .
sin x
)
;r-- X
X

Figure 12.12 shows 1 1;i(x), h; 2(:r). and J512 (x) plotted again.c.t :r. Note
Figure 12.12 1ha1 all three functions arc equal to zero a1 x = 0 (Problem 8). The functions
The Jkssel func1ions. J 112 (.r)1(solid), defined by j 11Jx)= (rr /2x) 112 111 112(x) are calJed spherical Besse/fimctions.
lJn(i) (long dashed). and J 512 cn (shon The ,q uanrum-mechanicall problem of a particle in a spherical cavity involves
da..~hed). plotted ag:ii nst x. spherical Bcsscll funcrjoas (S-ec6 on 16.8).

Although lhe Y11 (x) do not play a role in mos1 applied problems. we point out
here that the Y11 (x) satisfy the same recursion fonnu las as the J,, (x) (Equations 16
through 20).
Using Lhe result of Problem 18 of the previous section and Equal.ion 18 for
J:,Cr) and Y~(x), it is an easy matter to derive the relation (Problem 25)
')
J)x) Y,,+ 1(x) - li,+J(x) Yv(.x);:::: - --=-- (21)
rrx

Equation 21 is oae of a family of such relations. These relations find frequent use
in physical problems.

Example 4:
In Example 6 of the previous section. we showed that I.he generaJ solution to
Lhe problem of a pendulum whose length increa~cs at a cons1an1 rate is
I 2.6 Bessel Funcrions 617

where x =a+ br. Find the particular solution if 0 = 00 and d() /di = 0 at
I=0.

SOL u TIO N: The first initial condition gives

(22)

where>.. = 2(ag) 112/b. To implement the second initial condition. we write


0 as

where u = 2(g.x) 112/band then use Equation 16 with v = I:


d8 d0 du
dt du di

= 2gl /2 [c,i._ (J•) . du+ c2 ..:!_ (1). du]


b du II dr du u dt

= 2gl/2 [-ci Ji - C2 Y2] gl/ 2


b " II X

Letting t = 0 and setting the resuJt equaJ to 1.ero gives


(23)

Solving Equarions 22 and 23 for c 1 and c 2 gives

;r ).a 11200Y~(),)
2

and

•0

where we have used Equation 21 to rewrite the denominacors. Finally, then,


we have

Figure 12.13
This result is plotted in Figure 12.13 for).= 600 and, in unir.s of b/4g The solution to Ex~mple 4 (multiplied
(Problem 28), by 2/rr).. ~00 ) ploucd against 4g1 /b for
). =600.

rn
618 Chapter 12 / S · ri • · Solution. of Diif re nti.11 Equalions

In Section 3.7. we encountered the idea of a generating function. and in


particular, we saw 1haL

lei:< 00 t"
j(x.1) = -- = ~ B 11
(1)-
e' - I L n!
n=O

where the B,,(x) are the Bernoulli polynomials. There is a generating function for
the Bessel functions of integral order that ha~ Lhe fom1

(24)

We can derive this generating function fairly easily from the recursion fomiula.
Equation 20. Since many of the "name·• functions of applied mathematics have
generating functions 1har can be derived from their recursion relations, we shaJI
derive Equa1ion 24 from Equation 20. Perhaps it's easic~t to write ou1 Equa1ion 24
for reference:

Now multiply Equation 20 (with 1.1 = n) by r" and then sum from 11 = -oo 10
n = oo. The 1enn on the left in Equa1ion 20 gives
00

L l 11 +1t" = · · · + ./_lt- 2 + 10 ,- 1 + ./ 1 + ht + ht 2 + • • -
G(x. t)

The first tcnn on the righl gives


00

L 111,,t" = ... - 21_~,- 2 - J_1,-I + 0 + 111 + 2J.i1· +.,.


II '="' -

=t-
oG
ar
The second 1enn on the right gives
oc
L Jn-Jin= ... + l_2f-l + J_l +lot+ 1112 + ...
11 =-

= 1G(x. r)
Pu1ting all this toge1her gives
619

Integration gives

G (x . 1) = ;\ (x) exp [ 1( t - ;) ]

where A (x) is the "cons1an1" of the panial in1egration with respecl 10, . Equarion 15
=
shows 1ha1 all Lhe Jn (x) = 0 at x 0 except J0(x ). which equals one al.\ = 0. Thus.
G (x = 0. r) = I. A (x) = I. and we obtain Equal ion 24.
We can use Equation 24 10 derive many relalions invol\'ing Bessel functions.
For c~ample. using I.he fact that J -,iCx) = (- W Jn(x ). Equation 24 can be writ-
1en as

Letting r = e;r; and using sin 0 = (e;o - e-lt))/2i and cos 0 = (ei/J + e-i0)/2, we
have

Equating real and imaginary pans gives

cos(x sin 0) = J0 (x) + 2h(.x) cos 28 + 2.1-t(.r) cos 48 + · · · (26)

sin(x sin 8) = 21 (.x) sin()+ 21J(x) sin 3fJ + · · ·


1 (27)

hample 5:
Use Equation 25 to show that

OC,

t1xco~ 0 = Jo(x) + 2 L i 11 J,,(x) cos IIX


rr=I

This formula is used in quanrum-mechanical sc.aueri.ng theory.

SOLUTION: Lel 0 = ip + T· Then sin(ip + rr/2) = cos q,.


. I .
sin n(ip + 1r 12 )l=sm111.pcos -
n ;r
+cos111{)sin-
. 11 TT

2 2

= (-1 ) ~ cos n,p for 11 odd


nrr . . nrr
cos I n(I{) + JT /2)1 = cos mp cos - - !-Ln 11,p sin -
2 2
= (- I) ~ cos ,up for II even
12.& Bessel Functions 621

Finally, just a,-. J 112 (x) and J_ 112 (x) can be expressed in terms or sin x and
cosx.

/ 112(x) = ( -rrx
2 ) 1/2
sinh x and '-1 1:(x) = ( -2 )
1TX
1/2
cosh x (33)

There is a huge textbook lirerature on Bessel runci-ions. Abramowitz and


Stegun devote four entire chapters to Bes~I foncr.ions (Bessel Functions of Integer
Order; Bessel Functions or Fractional Order; ln1egrals of Bessel Functions: and
Struve Functions and Related Functions). (Struve functions are closely related to
Bessel func1ions.) The classic reforence is Watson's 800-page "A Trearfa-e on rhe
Theory of Bessel Functio11:·

12.6 Problems

l. Show 1hu1 the general solution 10 / ' (x) + ..!. y' (.x) + o- 2y<x)
.'(
= 0 is y(x) = c I J0 (a x) + c 2Y0 (ox) .

2. Show tha1 t
lo
x J 0 (ax)dx = ..!.1 1(a).
a

3. Show rhal f x In x J0 (x)dx = J0 (x) + x In x J 1(x ).

4. Show thal / J0 (x)J 1(:r)dx =- ~JJ(xJ.

1
-, I l 1
S. Show that
o
xlij(ox)dx = -[J
20(a) + Jt((/)1.
6. Staning with Equation 15. show 1ha1 [x-''J,,(x)]' = -x - 1
' J1, •1(x).

7. Starting wilh Equation 15. show Lhal [.r''Jt.(x)]' =x''J,,_ 1(x).

8. Determine the behavior of J 112 (x) = (2/rrx) 11~ sin x. hn.(x). and J512 (:r) given in Example J for small values
or x.
9. Assuming that we can intcgra1e Equation 25 term by 1em1. show thal

oc
IO. Show1hnte; 1 ' 1110 = L /n J,,(x).
11=- >C

11. Assuming I.hat we can multiply Equation 26 by cos 110 and iniegrate 1erm by term, show Lhal
Jn(.r) = -1
;r
1n 0
cos(x sin i.J) co~ 110 t/0 for n even.

12. Assuming 1ha1 we can multiply Equation 27 by sin ,,Band integrate tenn by 1.em1, show that
Jn (x) = -11:r. . . s,n(x sin (I) !>In nfJ
t"
dfJ ,or II odd.
;r (l

C gl
CHAPTER 13
Qualitative Methods for Nonlinear
Differential Equations

We studied primarily linear differential equations in the previous 1wo chap1ers. We


saw that I.here are systematic methods for solving many of Lhesc equations. espe-
cially if we can be satisfied with series solurions. There are no general syslematic
methods for 1he analytic solutions of nonlinear differential equations. A method
that may prove useful for one equation may be useless for others. In a sense, each
non linear differential equation is a challenge unto itself. Nevertheless, even though
we may not be able Lo find any type of analytic solution, we often can determine
certain key properties of solutions, such as their behavior for large vaJues of the
independent variable or whether 1be solution is periodic.
Many of the equal.ions that we shall be discussing in Lhis chapter are motivated
from classical mechanics. and so consequently will be of the fonn

i + j(x, x) =0
Even though il is a fairly simple maltt!r to solve differential equations numerically,
it is slill useful to extracl properties of the solutions before attacking them corn•
pulationally for several very good rea~ons: I. You can gain physical insight into
1he nature of the solUlion: 2. You may glean rcsrrictions on the values of certain
paramelers and reduce t.he number of computer runs; and 3. You will have some
checks for rhc final numericaJ solut..ions.
Section I prcsen1s an overview of 1he approach that we shall use 1hroughout
the chapter. We'll show that we can learn a great deal about t.he solurions of the
equation that governs a pendulum of nrbilJary amplitude.
.. .,
0 +a-sin 0 =0
without ever solving it. The key concept that we inrroducc in this section is 1he
phase plllne. ln Sections 2 and 3. we introduce critical points in the phase plane
and their classification and stability. Then, in Section 4, we apply the:;c ideas 10
nonlincar oscillators. We'll learn about an oscilla1or with no external driving force
that will oscillate with an amplitude of 2 after transient tenns die our. for just about
any initial condi1ion.s. Finally. in Sec1ion 5, we'll study some models of popula1ion
dynamics thal involve coupled first-order nonli.ncar diffcrcntial equations. Al the
end of the chapter. we shall brieny discuss chao1ic system,. 625

C
626 C h.1plt•r 13 / QualirJliw ,'vtcrhod~ for iNotdinc,u Diff r nti,11 Equarioni.

13.1 The Phase Plane

Let's st.an off with a simple equarion,

i + u}x = 0 (I)

which is 1he equation of a simple hannonic oscillator. We know Lhat the solu1ion
to this equal ion, x (I) = x 0 cos wt + (v0 / w) sin wr. where x 0 and ,,0 are 1he init ia1
posi1ion and velocity. is periodic wi1h frequency bJ. II so happens thut we can
deduce this impo11an1 property or 1hc $Olution wi1hout ever ~olving Equation I.
Multiply Equation I by.,: and U)';C Lhe 1wo relations

I d .1 I d 1 ,
- - x- = XX
...
and - - x- = XX
2 dt 2 dt
10 get

d . ., , ~
- (.t ... + w-.c) =0
dt
.r
or

.i 2 + w 2x 2 = constant (2)

Equa1ion 2 simply expresses conservation of energy. Now plot Equati1)n 2 in an


x . .r coordinate system. which shows a family of ellipses cen1ered at the origin
(Figure 13.1 ). The arrows on the ellipses indicate the forward direc1jon of Lime.
Figure 13.1
The phase portrait o( :.1 simple harmonic This x ..i- coordinate system is called Lhe plwse plane. and e.a1.:h ellipse represents
oscillntor, The arro1VS indicate the forward a trajectory of the hannonic oscillalions. The phase plane, including rhe family of
direcrinn of time. trajectories, is called a phase portrait.
Figure 13.1 contains the csscmial physical propenics of 1hc s.olution of Equa-
tion I. Each ellipse depicts oscillatory motion. If we write Equatjon 2 as

m .
-x 2 + -x~
k ..
=£ (3)
2 2
and use the fact that .i- = 0 when x is at one of its extreme values. we sec that x
varil!~ between ±(2£ / k) 112, or that the amplitude is (2£ / k) 112. Of course, this
is the same result 1hat we would ob1ain from Lhe explicit solution of Equation I.
which we happen to know in this case (Problem I). Furthermore, you c.an show
tha11hc period or 1hc oscillatory motion is r = 2rr /w (Problems 2 and J).
Before leaving this simple example. we"ll look at i1 from another point of view.
II" we let .i- = y in Equation 1. we can write ii as the pair or first-order equations
(see Example 11.6-4)

x=y
. , (4)
y = -w-x
13. 1 The Ph;ise Plane 627
or

v=Av (5)

in matrix fonn, where,,= (x, y)T and

A=( -w-
O') 0I) (6)

Note 1ha1 x = y = 0 is a solution lo Equation 4. Any point for which x = y = 0 is


called a critical pnint. In lhis ea.~. the critical point corre~ponds to 1..he oscillator
at rest in its equilibrium position. For I.his reason, critical points are sometimes
called e411ilibri11m />oillls.
The eigenvalues nnd the corresponding eigenvectors of A in Equation 6 are
±itu and (1, ±iw)\ so according to Section I 1.6. the real-valued solution to
Equation 5 is (Problem 4)

cos {J)/ sin


V = (
X )
)'
= C1 (
.
-W !illl Wt
)
+ l''J- ( WI
W COS WI
)
(7)

Equation 7 rcprcscn1.s a fomily of cllipse!t ccntcrcd at the origin, as in Figure 13.1


(Problem 5). The point (0, 0) is culled a cemu. We can w,e Equations 4 to verify
the directions of the arrows in Figure 13.1. Notice from Equation 4 thatx > 0 when
y > 0, so x incrca ·es with time in lhe upper half of the phac;e plane. and t.hat x < 0
when y < 0, so x decreases wilh time in the lower half of the phase plane.
Now let's look at a damped harmonic oscilla1or and sec what its phase plane
behavior is like. The differential equation is

(8)

Let i = y and wrirc Equa1ion 8 as


.i- =y
. ) (9)
y=-yy-w-x
or

v. =A,·= ( -w0 2 ( 10)

in matrix fonn. Note t.hat lhe point (0, 0) is a critical point of Equation CJ, corre-
sponding to the final st..a1e of rest of the oscillator. The eigenvalues of the matrix
in Equarion IO arc

( 11)

and so rhe nature of the solutions depend!. upon the relative values of y and w. If
y 2 > 4w2 • then both eigenvalues are real and negative, so x(t) decays mono1oni-
cally to zero. Recall from Chapter I 1 that we called this behavior overdamped. As
628 Chc'.lpter 13 / Qu.:ilit,llive Methods for Nonlinear Oifferenrial Equations

=
a concrete example. let y 5 and w = 2, so that the eigenvalues and corresponding
eigenvectors of A are -4. - I and ( -1. 4) r. ( - I. I) T. Therefore. the solution 10
Equations I O in this case is given by

(
X )
y = c 1 ( _- 4I ) e --4r + c 2 ( - 1I ) e _, ( 12)

X The phase portrait for this system is shown in Figure 13.2. Note that all the
trnjectories approach the origin t.angentially to a single straight line as r ---+ oo. II
turns out that I.his straight line coincides with the eigenvector direction of (-I, I) 1 .
x
which is the line y = = -x in lhe phase plane (Problem 8). The reason for this
behavior is lhat as t increases. the first term in Equation 12 becomes negligible
compared to the second term, and so the direct.ions of the trajectories coincide
Figure 13.2 with the direction of (-1. I) T as / ---+ CX). The critical point (0. 0) in this ca~e is
The phase ix-,rtr.i..il of an ovcrdampcd called a node, and a stable node, in particular.
ham10nic oscillnror.
i

Example 1:
Investigate Lhe phase portrait of lhe damped harmonic oscillator for I.he case
where y 2 < 4w 2 .

SOL u TIO N: If y 2 < 4-u/, then the eigenvalues given by Equatjon 11 arc
complex numbers of lhc fonn

Y i 2 2 I/ "')
A±= - - ± -(4w - y ) ~
2 2
so the motion of the oscillator is underdamped and is described by a damped
harmonic function. Figure 13.3 shows the pha,c portrair of this motion. Note
that the trajectories spiral into the origin (the cquilbrium position) as, - oo.
as you should expect on physical grounds. The critical point in this case is
called a spiral point.

Figure 13.3
1bc phase pon.rait of ao underdampc<l
hannonic oscillator. So far we have discussed only ca.1,es where we could solve the differential
equatfon analytically. Now let ·s consider a case LhaL we cannot solve it analytically.
In Section 3.5. we imroduced the problem of a pendulum of arbitrary amplitude.
Figure l 3.4 shows a mass m suspended from a fixed poi.nl O by a light rod of
length/. which !'i.wings in a fixed plane. If() is the angle of the rod ftom vertical.
then the equation of motion is given by {Problem 16)

ii + o} sin fJ = 0 (13)

where w 2 = g /I. Note thal if the motion is restricted to smaJI angles. then sin 0 :::::: 0
Figure 13.4 and Equation 13 becomes that of a simple hamionic oscillator. Generally, however,
An illu.~u-ar.ion of a pendulum consisting
Equation 13 is nonlinear and can be solved analytically only in terms of fairly
of a mas..~ m suppor1'--d by a mussle ' rig.id
rod of length / swinging in II fixed plane. advanced functions (cllip1ic functions) .
1.1. I The Phas.e Plane 629
We can learn a great deal. however. about the solutions to Equal.ion 13 without
solving ii. First we'U write Equation 13 as two first-order equations by leuing
Q = 8. so that Equation 13 becomes

0=Q (14)
Q = -w 2 sin 0

The phase plane in tJ1is case will have coordinates Q and 0. The critical points are
given by the two equations

&=r2=0
(15)
Q = -w 2 sin (} =0
orby n = Oand8 = ±111r. forn =0. I. 2 .... The points wi1h" = 0, ±2, ±4, ...
correspond to the pendulum hanging straight down at rc~I. We expect these po-
sitions to be stable in the sense that small displacement.') about those points wilJ
remain near those points. The points with 11 = ±I, ±3 . . . . correspond 10 the pen-
dulum balanced straight upwards at rest. We expect these positions to be unsi.able
in the sense that small displacements from those poin1s will result in the pendulum
moving away from those points.
We can investigate the nature of the trnjectories near these critical poinr.s by
linearizing Equations 14 about these points. Let's rake the poinl (0. 0) as typical
of a stable critical point. If we linearize sin O about the point B = 0, Equations 14
become

( 16)

=
The eigenvalues of this system are .l.. ~ ±iw. and so we see that the motion is
oscillatory about the point (0, 0) and t:hat the trajectories arc ellipses centered about
the origin. as in Figure 13.1. 1l1e point (0, 0) is a center.
Now let's look at the trajectories near an unstable critical point, such as CO. rr ).
Ir we linc.1ri1..c sin (} about 0 = rr in Equation 14, we obrai n (Problem 17)

( 17)

where 0 1 = 0 - rr. The cigenv1:1lucs in this case arc>-.±= ±wand the solutions arc

(}

Figure 13 .5 shows the trajectories about the point (0, 1t) for various initial condi-
tions with u> = 2. Notice that the trajectories are hyperbola<; with the eigenvector
directions as asymptotes and 1hat 1hey are all repelled by the unstable crili~aJ point Figure 13.5
(Problem 20). Such a point is called a saddle point. The 1rnjcc1orics of the pendulum descrilx-d

We have shown that the point (0. 0) is a center and that the point (0, 1t) i.s a
=
by Equution I J about the point Q 0,
0 = rr for various initial conditions with
saddle point. !i's easy to show generally that a.lJ the critical poinls (0, ±mr) with w=2.
634 Chapter 1 J / Qu:1li1.1tive Method~ tor N cmlir.w,H D1fi0.1t-111i,1I EquatiQr\i.

13.2 Critical Points in lhe Phase Plane

In the previous sec1ion we used mechanical systems as examples. and so one of


the first-order differential equations wa.1, always of the fonTI .i' = y. That"s why the
equilibrium points all lay on rhe x axis. We'll be more general now and consider
the pair of equations

i- = P(.L y) (I)
.\' = Q(x. y)
with Lhe ini1ial conditions x = x 0 and )' = Yo a1 , = 10 . The f:irs1 thing 10 note
is Lha1 1hese equations do not contain time explici1ly. Such equations are caJled
awonomous. Physically. an autonomous system is one where i1s parameters arc
indepcndcnl of time. Mathematically. the '-Olutions to Equations I depend only
upon 1hc elapsed time r - ,0 , where 10 is ~me initial Lime (Problems I and 2).
Autonomous systems of equations have the property 1ha1 closed trajec1ories in 1hc
phase plane represent periodic solutions.
The solutions 10 Equations I. x(I) and y(I). parametrically describe the 1rnjec-
1ories in rhe phase plane. If rhc initial value problem associated wirh Equa1ions I
is unique, as ir will be if P(x. y) and Q(x. y) arc com.inuously differen1iablc. then
the trujec1ories will never intersect each other (Problem 3).
The point.sin the phase plane where P(x. y) and Q(x. y) equal 1.cro arc called
critical point,1·, or equilibrium points. We'll denote criticaJ poinls by (xc, Ye). so
tlrnl we have

and

Example 1:
Find Lhe cri1icaJ points of the system

.r=l5x-3x 2 -4xy
.Y = 9y - 3y 2 - 2ry
SOLUTION: We set each equation equal 10 zero:

15.x - 3x 2 - 4xy = x(\5 - 3x · · ..J.y) = 0


9y - 3y 2 - 2xy = y( 9- 3y - 2x) = 0
Cenainly (0. 0) is a critic.ii point. Let x = 0 and y =/:- 0. The second equation
gives y = 3. giving a crilicaJ point (0, 3). Now let y = 0 and x -/; 0. The first
equation gives x = 5. for a critical point (5. 0). Finally. let x cj:. 0 and y =/:- 0
and solve the two equations

I 5 - 3.r - 4 y =0
9- 3y - 2.r =0
simultaneously 10 give (9. -3) as 1.hc fourth crirjcal point.
638 Chaplet 13 / Qualit.llive Me1hods for Nonlinear DiHerential Equa1ions

I)
is illustrated by the system

The solution is given by u(1) = 11 0e"- 1 and v(I) = v 0eu 1. If a < 0. the trajec1ories
approach the origin and if a > 0. the trajectories recede from the origin as ,
increases. as shown in Figure 13. I I. This rype of critical poinl is called a proper
node.
The case in which c2 and c4 are not both equal to zero is illustrated by

(13)

Figure 13.11
An illustration of a proper node wirh rhc The eigenvalue).= 1 is repeated in this case and we obtain only one independent
1rajcc1orics approaching the cri1ical point. eigenvector. (0. I) r_ However. Problem 12 helps you show that if we substitute
Equations 11 imo Equations I 3. we obtain

and ( 14)

We can solve for the trajectories explicitly in this ea~ by solving the first of
Equations 14 for, and substitute the result in10 the second of Equations 14. This
gives us

,, ( 15)

The 1rajec1ories given by Equation 15 are shown in Figure 13.12. No1e tha1 al I rhe
trajectories recede from the critical point (the origin) tangentially to the vertica1
figure 13.12 axis, and so the critical point in this case is an improper node.
The trajectories given by Equation 15 for
various vnlues of c 1 and c 2. The: c-ri1ical
point is an improper node.

Example 4:
Investigate the criticaJ point of the system

( 16)

SOLUTION: The critical point occurs at (0. 0) and the repeated eigenvalue
is - I. To find a solution to Equations 16, substilU!e Equmion.s 1 I with). =- I
into 1hem 10 obtain

and

Equating the coefficients of like powers of r gives c2 = 0. c 1 = c-1, and CJ


13.2 C.:riti( al Points in the Phase Plane 641

Table 13.1
A summary of the type of critical poin1 associated wi1h lhe propcnics of Lhe
eigenvalues of I.he coefficien1 matrix. of the pair of firs1-order differential equal.ions.

type of stabiliiy of
eigenvalues critical point the critical point

I. real. uncquaJ. both negative improper node asymptot1cally stable


I. real. unequal. both positive improper node unstable
2. real, opposite signs saddle unstable
3. equal and positi\'e proper or improper node unstable
3. equal and negative proper or improper node asymptotically stable
4. complex conjugates. real part > 0 spiral poim unstable
4. complex conjugates. real part < 0 spiral point asymptotic.ally stable
5. pure imaginary center stable

13.2 Problems
I. Let r =, - ,0 in Equations I and sho~v tha1 lhcy 1ak.e on the same fom,.
2. Show that the solution to x = y ond j, ·-· - x for x = x 0 and y = 0 at r = r0 is a function oft - r0 .

3. Argue that lhe uniqueness of the solutions to Equations I prevents. trajectories in the phase plane from
intersecting.

4. Find the critical points of


(a) -~ = I - xy (b) .i- = I - y x. =x - 3x~J + xy x=.r-y
2 2 (c) . 4 2.x (d)
Y = xy - _\' .i· = x - .v y= y - r -
?
Y j, = x 2 - I
5. Modify Lh<: equa1ions in Problem 4 so tha1 the critical poinL'- occur at (0, 0).

6. Dctcnninc the cocfficicnl matrices of the lincari1.cd equation$ of Problem 5.

7. Show thal the general solution to Equations 7 ii- given by Equations 8.


8. Show 1ha1 the eigenvector directions (- I. l)T and ( I, 2)T thm pass through the origin correspond to lhc s1raight
lines v = -u and v = 2u, respectively. in the 1rn-plane.

9. Show 1hot the s1raight Ii ne in the eigenvector direcrion (- I + J2. I) T that passc through the origin is described
=
by v 11/(Ji - I).

10. Show tha1 all the 1rajec1oric~ a~~ociated with Equations 8 (except for 1wo) approach 1hc origin 1angen1ially to
1hc eigenvector (-1 + h. I) r as t -), oc.

11. Show that Equation 10 is the general wlution of Equation 9.

12. We' II derive Equa11ons 14 in this problem. Substitute Equal.ions 11 into Equations 13 to obtain
c2 + (c 1 + c1t) = c 1 + c11 and c-i + (c3 + c-11) = k1 + c1l) + (c3 + c-11). Now equate ~imilar powers of
, on the two sides of each equation 10 show that c1 = 0. c 1= c4, and c3 is arbitrary. This gives us Equations 14.

13. Classify the singular point of 1hc sys1em ( 1) = (-~ 1


-~) ( 1~ )-
1.l .J S1.,,bility of ritical l'oints 643
SOL u TION : The c i.gcnval Ul' S of 1hc syi-tc m arc ±i. so the trajcc t orics
are circles centered al the critical point (a ccnler). Any circle that stan.s al
a distance equal to the radius of the circle from the center remain~ at 1ha1
disiancc. so a ccn1cr is necessarily a stable critical poin1.

Example 2:
Delcnuinc the nature of the stability of the critical f.l()inl (0. 0) for the system

.i- = .r + 2y
.\' = 2x + y
SOLUTION: The t·igcnvalues and corresponding eigenvector:- are - I and
3 and ( - I. I) r and (I. 1)1. so the cri1b1..I point is a saddle poinl and the
trnjc-clorics are given by X

There arc many trajecrorie:.~ thal start close 10 the origin and tend to inl'inily
as , increases. ('See Figure 13.16.) Becaus~ a saddle point occurs when
lhe eigenvalues are rcaJ and of opposite sign. a saddle point is necessarily Figure 13.16
The lr;ijel·toric~, around the s.addk poin1
unstable.
o.ssocia1cd wiih 1hc system In Example 2.
,illus1rnting tha:1 a sad.die poinl i.s un~ta.blc.

Example 3:
Determine 1he nature of the stability of the critical point (0. 0) for the system

(i) (-1 I) (·.r)


.,. - I -2 y

SOLUTION: The cigcnval ues and corrcspondi ng cigcnvccrors arc -3 and


-I and (-1, 1) 1 and (I. l)r:

The 1wu cigcn\'alue~ have the same ~i!;n. so the cri1ical point is an improper
no<lc. und since they are both nc~ativc. the critical point is an asymptolically Ftgure 13.17
stable improper node. Furthcnnorc, as r - co. Lhc second tcnn in the The 1trajcc11.,ril·~ d..-~l'fil~d in fa.ample 3.
showing l:ttnl the: critic:.il poial (0. 0) i~
e<1ua1ion becomes negligible compared 10 Lhe first term and so the trnjectories an as}'mp101ic.aJly , 1ahle improper node.
appronch Lhe node tangent to the eigenvector direction (I. I) T. or 1angcnl 10 Nnte thal all 1~ 1rujc.:tnries appro,;u.:h
the ~•raight Ii ne y = x (~Cl' f-igurc 11.17). the orrgi11 t:ingen1ially 10 thl.'. ~1r:a.igJ1t line
y = x (!'>lack) as r incrca....es.

C gl
13.3 S1ablli1y of Cririt .11 Points 645
We can summarize all of our results regarding rhe rype of cri1icaJ poinl and
the nature of its stability in Lenns of two quantities, p =a+ d and q = ad - be,
where

The eigenvalues of A a.re given by

Let's look al 1hc 1hree cases. p = 0. p > 0. and p < 0, in 1um.

I. If p= 0 and q > 0. then the cigcn\'alucs arc pure imaginary and the criticaJ
point is a cemer, which is necessarily stable.

2. If p > 0 and q > 0. then the eigenvalues arc real. unequal. and positive if
6 = p 2 - 4q > 0. and ~o the critical point is an unswble improper node . If
6 < O. on the other hand. Lhcn the cigenvalul'" are a comple.'< conjugate pai.r
wilh a positive real pn.n. so the critical poin1 i!> an unstable spiral point.

3. Jf p < 0 and q > 0, then !he eigenvalues are real. unequal. and negative if
6 = p?. - 4q > 0, and so 1he critical point is a s1ablc improper node. lf n < 0.
on !he other hand. 1hen the eigenvalues are a complex conjugu1c pair with a
negarive real part, so the critical poin1 is an a."ymp101ically stable spiral point.

4. FinaJly. if q < 0. 1hcn the eigenvalues will be real and with opposi1e signs for any
value of p. and so the cri1icaJ poinl will be a saddle point. which is necessarily
unstable.

We can summarize these resuhs in a plot of q against p, as we show in


Figure 13.18. Note that the critical points in the upper half plane are unstable if
=
p > 0. stable if p < 0, and stable if p 0. The critical points in 1he lower half plane
=
arc always saddle points. Also note Lhat the curve I),,. 0. which is the parabola
described by q = p 2 /4, separates various types of crit.ic::il points in the figure .
The eigenvalues ~iven by Equation I are real and equal along the parabola. The
nature of the critical poin1 changes as we cross the parnbola. but the s1ahility of the
critical point is nor .itTected. This bchavior illustrates the fact that small changes
in 1be parameters of A can affect the type of node, bu1 not its s1abili1y when the
eigenvnlues are real and equaJ. Another fe.a!Ure of Figure 13.18 to notice is that Lhe
positive vertical axis. where the eigenvalues a.re pure imaginary. separates a region
des.cribing an asymptotically iHable spiral point from an unstable spirJI point. The
figure emphasizes just how sensitive the ca.,;:e of pure imaginary eigenvalues is.
Even a small displacement from Lhe verticaJ axis can lead 10 either a stable or an
uns1able cr1icaJ point. At this poin1, you should be able 10 relate Figure 13.18 lo
Table 13.1.

C
13.3 ~1;ihili1y of Critical Points 647
(b) If the eigem alue.'i are pure imnginary (i.±
1
=
±iv), 1lze11 the critical
point is eilher a ce11ter or a spiral poi111, and tire spiral point may be
asymptotically stable or UII.St.able.
(c) In all other cases, rite type of critical poi111 a11d i1s s1abiliry is the same
as that of the corresponding linear system.

Note lhal rhe two cases where the resuhs for the non linear system can differ from
those of I.he linear system are the so-called sensitive cases that we discussed above
and are illustrated in Figure 13.18 by the vertical positive axis and the parabola
~ = 0. Thus. in a sense. the effect of the nonlinearity is equivalent 10 introducing
a small uncenainty in 1he values of a. b. c. and d.

Example 4:
Find all lhe critical poin1s and invcsrigate !.he type and si.abilily of each for
the system

.i· =X - y

y =J. ~ }'

SOLUTION: There are two critical points, (0, 0) and ( I. I). The linearized
cqualions abou1 (0. 0) arc

The eigenvalues are A±= ±1. and so the origin is a saddle point. which is
necessarily un~tablc.
The linearized equa1ions about (I. l) are given by suhstituting 11 x - I =
=
and v y - I into the above diffcrent.ial equal.ions and lhcn linca.rizing them
10 ob1ain

The eigenvalues arc ±i and so the crirical point ( I. I) is either a center or a


spiral poinl, which may be eirher asymplotically stable or unst.:ible.

Before we leave this section, there is one more topic to discuss. Consider the
sys1em

i=x+y-x(x 2 +y 2 )
(4)
~\· = -x + y - y(x2 + y2)
The only critical point is at the origin. The presence of the (x 2 + y 2) term in both
equations suggests Lhal we use polar coordinates in Equations 4. Letting x = r cos 0

C gl
13.4 Nonlir>C,H u~l ill,11iu, 651
13.4 Nonlinear Oscillators

In this section, we're going 10 apply 1hc ideas that we have presented in the previous
sections of this chap1er to systems of equations that describe oscillating systems,
whether 1hey arc mechanical or elccuical sys1ems. Let's s1.ar1 wi1h a nonlinear
oscilla1or described by
1 lx > '
3
.t +X - x =0 ( I) \
I
f \

I
Like most any differential equation. we can solve Equation I numerically using a
CAS. For example. if we solve Equa1ion I with the in i1ial conditions x (0) = 0. 20 I
=
and i(O) 0. we obtain Figure 13.20a. You migh1 have expe.cted whal appears LO \ I
-1
be essenlinlly harmonic bchavior bee au~ the initial conditions restrict x (r) to smal I
oscillations. where Equa1ion I will be almos1 linear. Figure 13.20b shows .r(r) for Figure 13.20
1he initial conditions x(O) = =
0.99 and .i-(0) 0. The behavior is still o.-.cillatory. A numerical .~lu1ion 10 1he equation
bu1 does not look ham1onic. Notice also 1hat 1he frequency is abou1 one-half of the x+ x =
- .r 1 0 for the ini1i11I condi1io11s

Frequency in Figure 13.20a. a good ex.imple where the frequency of a nonlinear


= =
iai .r!O) 0.20 ::i.n<l .i"(O) 0 (solid color):
(b) ..rf0) =0.99 and .i-(0) = 0 (dashed
system can depend upon the initial conditions. If we increa'ie x(O) IO equal I. color): (c) x (O) = I and :c(O) = 0 (black).
we obtain Figure 13.20c. which is decidedly non-oscillatory. If we now increase Noti: 1h;11 ct.lf"\'C c is simply a h(lrinmtal
Jin\;. .
x (0) beyond I. we get a deluge of error messages from our CAS. The same 1hing
happens if we let x (0) = 0. 8. .i- (0) = 0 .8 or x (0) = 0. ,i- (0) = I. and so on. Thus.
even Lhough we could simply solve Equation I numerically. i1 will not give us
much insighl in10 why the :,;olutions vary so much with the initial condiLions. The
methods 1ha1 we have developed earlier. however. arc easy 10 apply and will explain
the numerical results 1hal we mentioned above.
Firs!. we'll write Equation I as two first-order equations and then f'ind the
critical points. Letting i =
y. Equation I becomes

X =y
(2)
_\' = -x + x 3
The critical points are given by y = 0 and -x + x 3 = 0. or by (0, 0) and (± I. 0).
The lineruized equations about the origin are

( -~y ) = ( -]0 0
I ) ( .r )
.y

The eigenvalues are i.± ;:;; ±i. so according to Poincare's theorem. the critical point
is eirher a center or a spiral point. We can distinguish be1ween these two choices
by realizing that 1here is no damping tenn in Equa1ion I, so we expect the critical
poinl lo be a ccnter. (We'll verify 1his below.)
The linearized equations about the critical points (±I. 0) arc (Problem 2)

(3)
652 Chaprer 13 / Qualir.:iiive Methods for Nonline.:ir Differ ritial Equa1ions

The eigenvalues are A± = ±./2 and the corresponding eigenvectors are (I . ./2) T
and ( I, -./2) r. and Lhe solution to Equa1ions 3 is

The critical points at (± I. 0) are saddle point.s and the eigenvec1ors serve as
asymptotic directions of the trajectories. The sLraigh1 lines corresponding 10 the
eigenvector directions are

lJ = ./2." and I.I= -h.11


Therefore, the asymptotic direcLions of the trajectories around the critical point
( I. 0) are

y = -✓2(.x - I) and y = -h(x - I)

and 1hose around the cri1ical point (-1, 0) are

~
);: X
y = -.1'2(.x + I) and

(Problem 3). Figure 13.21 shows the phase pon.mit of Equations 2 that we have
deduced so far. We'll now fill in the rest of it. Divide the equation for y by the
Figure 13.21
equation for .r in Equations 2 10 obtain
The phase portrnit of Equations 2 abour il!-
thrce cTirical points al (0, 0) and (±I. 0). y dy x-' - x
= =
i dx y

An integration gives

'I XJ 'I
r= - - .c+c (4)
2

where c is a constant. Looking at Figure I 3.21. we see (guess?) that the largest
dosed trajectory passes through the points (± I. 0). so we set x = ± 1 and y = 0 in
Equation 4 to obtain c = 1/2. Thus. the largest closed trajectory (actually. it's the
separJtrix) is given by

I 4
v-
,
l
+ x- 1
- -x
2
=2 (5)
X

and the smaller trajectories are given by a family of curves described by

Figure 13.22 )'~ + .cl -


1 ] 4
-x =C (6)
The family of closL-d Lrnjcciorics gi\'cn by 2
Equation 6 for various value , of C. The
separ.mix. which is shown in black. is with C < I/2. Figure 13.22 shows trajectoriei; for various values of C. both
givc:n by Equation 5. greater than and less than I/2. The closed curve that separates the region of closed
lJ.4 Nonlinl'.'lr Oscilla1ors 653

trajeclories from Lhe region of asymptotically unstable trajectories is a seporarri.r.


The separatrix is given by Equation 5.
We can now explain the behavior that we ob1ained numerically in Figure 13.20.
No1ice thut when the initial values lie within the closed curve described by Equa-
tion 5. the motion is oscillatory. If 1hc initial conditions are x (0) = ± I. ,r (0) = 0.
then x = y = 0 (the poin1 is a critical point) and the solution is x = ±I.as shown
in Figure 13.20c.

Example 1:
Include a damping term of I.he form .i- in Equation I and discuss its phase
portrait.

SOLUTION: We stal1 with

x + .i· + x - :r
3
=0
which we write as

.i' = \'
.\' = -X - )' + X) ' \I

The critical points are at (0. 0) and(± I. 0). The eigenvalues associated with
the equation linearized about the origin are (- I ± i vJ)/2. which s.ays that
the origin is an a.<-ymptorically :-table spiral point.
The equations linearized about the critical points (± l. 0) are I.he same:

( 1iiJ ) = ( 02 -I
l ) ( 11
v
)
Figure 13.23
n,c pha~e ponrail for 1hc system described
in Example I ::ibour irs three critical poinls.
The eigenvalues and corresponding cigcnvcmlrs arc -2 and I. and (-1. 2/
and ( I. I) 1 . respectively, and so these two critical points are saddle points.
Furthennore, the trajectories abour the saddle poi nl (+I. 0) are asymptolic
to the two straight lines _\' = x - I and y = -2(x - 1). and 1hose about
the critical point ( - I. 0) arc asymptotic to y = x + I and y = -2(x + I).
Figure 13.23 shows the phase pomait about the three critjcal point,; .
You can now fill 1n the other 1r.1jcctorics using numerical methods and
judicious initial value~ to get rhc result shown in Figure 13 .24.

In Section I. we us.ed the physical problem of an undamped p~ndulum of Figure 13.24


arbitrary amplitude 10 introduce the idea of a pha._e plane. critical points. and a The complete phase po11rai1 fur the system
pha<;e portrait. which is shown in Figure 13. 7. With the experience we have, it described in Example 1.

~hould be fairly straightforward to sketch the phase portrait of a damped pendulum


of arbilr<lJY ampliLude. described by

0 + y iJ + w1 sin 0 = 0 (7)
654 Chapter 13 / Qualitative Methods for Nnnlimw Differential Equations

We rewrite Equation 7 as

f) =Q
(8)
Q = -y Si - w 2 sin 0

The crilical poin1s occur al (n,r. 0), with n = 0. ±I. ±2 ..... just as in the un-
dampcd case. Expanding sin 0 about 0 = nrr. we ob1.ain sin 0 = (- W (0 - nrr)
+ · · ·. and the linearized equations are

(9)

= !(
where 0" = f) - "JT. If n is odd. the eigenvalues are). ± -y /2 ± y 2 + 4u}) l/2.
so the cri1ical points are saddle points. If 11 is even. the eigenvalues are >..± =
-y /2 ± J. (y 2 - 4-ai) 112 • and the critical points are stable nodes if y 2 > 4w2
(overdamped) and asymptolicaJly stable spiraJ points if y 2 < 4u>2 (underdamped).
Figure 13.25 shmvs the phase portrait for the underdamped case. No1jcc 1ha1 every
Figure 13.25
The phru;c ponr:ait of an underdamped
realized trajectory approaches a spiral point as r - oo.
or
pen du Ium nrbi lrnry amp Ii Iude described
by Ec)uations 8.

Example 2:
Sketch the phase portrait for the overdamped case for the system described
by Equation 8.

SOLUTION: In the overdamped case. the critical points at (11rr. 0) with


11 = 0. 2. 4 .... are stable nodes like in Figure I 3.2. rather than spiral poinLS
as in Figure 13.25. Therefore. the phase ponr.iil look1, like the one shown in
Figure 13.26.

Figure 13.26 We'll study one more equation in this section. The equation
The phase ponr.iit of an 011~rdamp,:!d
pcnd u1um of arbi 1r.1ry amp Ii t udc dcscri bed
by Equation., 8. e:::-0 (10)

has some iniercsting properties. If .r < I. then the damping term is negative.
which means that it causes x(t) to increase with time. As x exceeds unity. the
damping term becomes positive, meaning that it causes x(f) to decrease. We might
expect, 1hen. that a balance will be reached. the solution will become stable and
periodic. and that Equation 10 will admit a limit cycle. Funhennore. even with
.. initial conditions such as x(O) = 0.0010 and .i'(O) = 0, the system will approach
50 t its limit cycle and become periodic with its amplitude being independent of the
I
-1 vaJue of x ( 0). This behavior is shown in Figure 13.27.
Realize that the amplitude grows in10 i1s final value (determined by 1he limit
cycle) without any input of energy. This type of behavior is called .i self-excited
Figure 13.27 o.rcillntion. and can occur only in non Ii near systems. Equation IO is ea] led the van
T11c displacemcn1 of ll van der Pol
oscilhuor plo11c<l ugain I rime ror der Pol equation. and is a classic equation of non Ii near mechanics. It was originally
x(O) = 0.0010 und .i-(0) =0. derived to describe certain electrical circuits with feedback. but ii (and its relatives)
656 Ch,111tl'r 13 / Quali1,11Jvl.' Methods (or Nonlintw Differen1i.d Equ.Jlions

13.4 Problems

I. Do you see why Lhe point x = 1 in Equation I might be a special point? Plot 1..he potemial energy.
2. Verify Equations 3.
3. Show that the eigenvector directions of ( I. ± J2) T passing through the poi nLS (±I. 0) are described by
y = ±J2(x - I) and y = ±Jl(x + I). respectively.
4. A spring governed by a potential energy of the fonn V (x) = fkx 1 - { #.r 4 i, ,aid to be hard if f:J < 0 and .wfr
if f:J > 0. For example, the spring a!-sociated wi1h Equation I is soft. and is the more interesting case. Sketch
the phase portrai1 for a nonlinear oscil.lalion governed hy a potential V(x) = {.x 1 - ~x-1 and ~e why we say
that rhe sof1 case is more in1eres1ing physically. (Take 111 = \.)
S. Sketch I.he phase portrait of t..he nonlinear oscillator in 1he previous problem. but lei there be a damping 1.enn
+i.
6. Consider the nonlint!.lif oscilla1or described by .r + x - {x 2 = 0. If this equation i~ solved numerically with
=
the initial conditions x(O) 4 •.r(O) = 0. the result is shown in Figure 13.29. Why do we obtain this result?

4--------

Figure 13.29
The numerical solution to
+ x - {x 2
."i = 0 for the iniLi:11
condi1ions x(O) = 4..i-(0) = 0.

7. De1ennine I.he phase portrait of 1he oscillator described in t..he previous problem.
8. Which of 1.hc curves in Problem 7 is the separat.rix'!
9. Show that it takes an infinite lime for the sy~1em 10 reach the poin1 (4. 0) along the separatrix of the previous
problem.
10. Show that the t.rajcc1oric... of mechanical $ysrcms crof:s the x axis at right angle!> except al critical poinL<:.
I 1. Dc1.ennine and classify the cri1ical points for the nonlinenr oscillator described by j.= + 9x - x 3 = 0.
12. Whal is I.he rea$On for I.he critical points 1.11 (±3. 0) in the previous problem? Him: Plot the potentia.l energy.
13. The differential equation for an oscillator whose linear part of the force is repulsive rnther lhan attractive is
x 1x
.r - + 3 = 0. Detem1ine the nature of the critical points for this system. Sketch it!- phase ponrait.
14. An oscll. Iu1or moves .In 1he poten11a
. I V(x) = -I - I .,
-x- + I
-x .
-1 PI 01 th1~
. po1cn11a.
. l Detcnn1ne
. the nature ol. lhe
4 2 -I
critical points and sketch the pha.-.c portrait.
3 6
15. An osc1·11 oror mow.s ·10 the potential
· V
(.r) = -1 - -3.x::! - -x + -:lr~ + -.
x PI ot tl us
· po1cnt1aJ.
· L'sc this
· plot 10
4 10 8 40 10
determine the nai-urc of the critical points. Ske1ch the phase portrait. (See the previous problems.)
. I Rr 2 3 5 6
16. Plo1 I.he po1en11al V(x) = -I - -- - -7x - -3Jt"-l- - -2 ix- \·
:...... (Note 1hat there is no left-hand minimum
4 I 15 23 460 115 10
here.) Sketch 1he phase portrait for rhis . . yqcrn and compare it to the pha!.e portrait of the Iwo previous
problems.
13.5 Population Dyn,1mi1 ~ 657

17. This problem continues the lhrec previous problems. Show lhat the crilical point~ of i + f(x) = 0. where
f (x) i!- a polynomial. must either be ccnters or saddle points. Can you show that ccnters and saddle poinL..;
must occur nl1.emntely along the x axi.,? Hint: fapand J (x) about the cri1icaJ points.
18. Show that cnch of the following equations has a limit cycle:

(a) .t + dx 2 - l).r + x 3 = 0 € > 0


(b) i + d2x:! - l).i + x + XJ = 0 t: > 0

19. In Example 3, we showed rhar the van der Pol equa1ion has a limi1 cycle about 1he origin if t: > 0. Show that the
eigenvalues corresponding to the equation linearizcd about the origin are).± = € /2 ± (e 2 - 4) I/'! /2. Show
that the origin is an unst.able spiral point if O < 1; < 2. How do you reconcile this result with the existence of
the limit cycle?
20. Another classic non linear oscillator equation is Rayleigh's equnlion, x - € (x - ~i 3) + x = 0. Lord Rayleigh,
certainly one the greatest physicists of the 19th centW)'. first derived Lhis equation to describe the vibrations
of a violin siring caused by moving the bow across it, but it has since been applied to the screech when cha.lk
is dragged across D blackboard. the squeaking of an unoiled hinge. 1he waving of a tlag in 1he wind, and a
number of other systems. Use any CAS to !-how that Rayleigh"s equation displays self-excited oscillations.
Take x(O) = 0.010. i(O) = 0. ::md € = 0.0100.
21. Use any CAS to show that Rayleigh's equation (see the previous problem) has a limil cycle of radius 2 for
smaJl values of,;.

'13.S Population Dynamics

The malhematical modelling of population growth is quite sophisticated and em-


braces a number of malhema1ical methods. but here we shall present only the
fundamentaJ ideas. Neven.heless. even al thi~ ~imple level the basic equations are
nonlinear and require the techniques lha.t we have developed in 1his chapter.
We'll start off with the simplest system. describing a single populalion. We
assume thal the population grows al a rate 1hal is proportional to the size of the
popula1ion:

P =kP ( I)

Fork> 0. Equation I leads to an exponential growth. which might be valid during


the initial stage of growth, but cannot persist indefinitely due to limited resources
or overcrowding. We can include 1he~c effec1s by letting k be a function of P
which decreases as P increases. so 1.hat 1he ,Jle of growth decreases with increasing
popula1ion. A ~imple functional dependence 1ha1 reflec1s 1his effec1 is k = a - bP.
in which case Equation I becomes

P = aP - bP 2 (2}
801h a and b arc positive in Equation 2. You can see that the P 2 1crm leads to a
decrease in Pas P increases. Equation 2 is readily solved hy separation of variables

C gl
658 Chapter 11 / Quc1 li1a1ive Methods lor Nonline.:i r D iffcrc ntiJI Equ.:it,on~

and iLs solution is (Problem I)

(a/b)P
P(I) = - - -- -0 - -
50 P0 + (a/ b - P0 )r' 11

where Po = P (0). Note 1hat P (I) -. a/ b as t - oo, so that Equation 3 predicL,


a stable popula1ion. You can sec 1his same resull by setting P equal to zero in
1/a X
Equation 2. Equation 3 is plotted against, in Figure I 3.30 fora /b = 50 and several
Figure 13.30 values of P0. Equa1ion 1. or iL-. solution. Equation 3, i:i. called lhe l01:istic equ01ion.
Equution 3 plo11cd ugainst f for various and is one of the early equations of population drnamics.
Ylllues of P0.

Example 1:
Modify Equation I 10 include 1he effect of the addition of individual" 10 the
popular ion due to immigr..irion. for cx:.irnplc. i.11 a c.:ons1ant r..11.c /.

SOLUTION: Equation I becomes

This equation can be solved by separation of variables and yields (Problem 2)

(4)
p
where

and
LOO A = _Pi_o_-_a_/_::?_b_-_ K_/_2_h
1/K Po - a/ 2b + 1< /2b

Figure 13.31 Nore !hat Equa1ion 4 ~ays Iha1 P (I) - (11 + K) /2h as , - co. i ndependen1ly
Equation 4 ploI1cd agaiml t for a= 0.020, of the value of P0 . Figure 13.31 shows P (1) plo1ted agi.linsl r for a = 0.020.
=
b 0.00020. and I = I 0 for various b = 0.00020. and / = I 0 for various values of P0 . The I imit ing value of P ( t)
values of P0 • The limiting value of Po is
is (o + K)/2b = 279.
[a + (a 2 + ;Jb/) I/~ ]/2/J = 279.

A more interesting population study is that of 1wo species that interact with
each other. One species (the prey) lives on an abundant supply of food in its
environment. while the other species (the prednwr) lives off the prey. Let x(I)
be the population of the prey am.I y(t) be the popula1ion of the predator. Then .i-(1)
increases proportionally to x (r) (as in Equation 1) and decreases proportionally to
its encounten. with the predator. If we assume 1ha1 the encounters are proportional
to x(r )y(r), then :i"(r) is given by

;r(r) = ax(I) - bx(r)y(I) (5)


13.5 Poµularion Dynamics 659
where a and h are positive constants. Similarly. if we assume that the only food
sourc~ of the pred:.11ors is the prey. then .,·(r) is given by

-~·(I)= -cy(r) + dx(r)y(r) (6)

where c and d are positive constants.


Equations 5 and 6 arc classic equations of population dynamics and arc called
the preJarur-prey equotim1s. or the lotka- Vollt:rra <'q1w1io11s. They were intro-
duced independently in the 1920s by the American biophysicist A.J. Lotka and
the ltalian mathematician Vito Voherr.i. There are extensive data of population
records and the Lotka-Volterr.i equal ions have been applied 10 shark/fish popula-
tions. lynx/hare populations. bass/sunfish populations, ladybug/aphid populations.
and many others. l11e constants a. h, c. and t1 :.ire empirical cons1an1s that are used
to fit 1he population data.
Equarions 5 and 6 have two critical points. (0. 0) ,md (c/d. a/b).The origin
is a saddle point and can he reached a-.ymptotically only if x(f) = 0. in which
ca,;e y(t) decay~ exponentially to zero according to Equation 6. The other critical
point is much more interesting . Equations 5 and 6 lincarizcd about (c/d. a/h) are
(Problem 1)
1
11
,i ) _ ( 0 - :;· ) ( ) (7)
( 1; - od v
O
/J
The eigenvalues arc>.. = 2
±i (ac) 112 ,
indicating 1ha1 the critical point is a (stable)
ccntcr for the linearized system. The trajectories are ellipses ccntcrcd al the critical
point ( Problem 4 ).
Rcca.11, however. that if the eigenvalues of the linearized system arc ±i µ
200 X
(indicating a center). then the critical point of the parent nonlinc.:ir equations may
be either a center or a spiral point. Equations 5 and 6 present no problem. however. Figure 13.32
because ir we divide one by the other. we obtain Equation 8 wirh n = 1.0. b = 0.040.
c = 4.0. and d = 0.020 plo11ed in the
x dx a x - bx y x(a - by) pha.!ie plane.
-=-=
v dy -cy + dx_r = y(dx - l")

which can be readily integrated to get

(../ In y - hy + c In x - dx = constant (8)

li"s not obvious (nor I.hat easy to prove). hut Equation 8 represents a family
of closed curves in the phase plane. Equation 8 is shown in Figure U.32 for
a = 1.0. b = 0.040. c = 4.0. and d = 0.020. Note that Lhe curves are centered at
x =1:/d = 200 and y = a/b = 25. Figure 13.33 shows Lhal both .r(t) and y(/) are 2,r
periodic func1ions. bu1 we have 10 solve Equations 5 and 6 numerically lO obtain
x(I) and y(1). Figure 13.33
Plot.~ or x(I) (culur) and y (t) (bl:ick) from
We can learn about the solutions to Equation 5 qualilatively by restricting
Et~uations 5 :tnd 6 with a = =
1.0, b 0.040.
ourselves to regions near I.he critical point. where we can use the linearizcd version. = =
c 4.0, ::ind d 0 .020 plo11ctl against 1.
660 Cha pier 13 / Qualitarive Mc-lhc Kb tor Nonlinea, Dif<erenri,11 [qu,11ions

Equa1ion 7, to determine x(t) = 11(1) + c/d and y(t);;;: u(r) + b/a. The solutions
to Equation 7 arc (Problem 5)

x(t) == ~ + C cosj(ac) 1l2, + q,]


d
(9)

2JC

Figure 13.34 These equations say Lhat x(I) and y(t) are periodic wilh a period 2rr /(ac) l/"J.
The numerical rolu1ion of Equa1ion, S that does 001 depend upon the initial conditions. Figure I 3.34 shows x(t) and
llJld 6 for x(f) (oolor) and y(I) (black) for
y(t) obi.ained numerically from Equations 5 and 6 with a= b = <: = d = I and
u = b=c d= = I and x(O) = 2 .0 11.nd
x(O) = 2.0 and y(O) = 0.50.
y(O) =0.50.
Nore that lhe rwo curves have a period of about 2JT. The preda1or curve lags
behind lhat of the prey, as you might expect physically. As the population of Lhe
prey builds up. the predator has an ample food supply. and so its population grows
at the expense of lhe prey. Then. as the density of the prey decreases. the density of
the predator decreases. thus allowing the density of the prey to increase. and this
cycle continues to play out. The following Example shows 1ha1 phase difference
bc1wcen the two curves in Figure 13.34 is one quarter of I.he period, or rr /2(ac) 112.

Example 2:
u~c Equations 9 to show that the pha.sc difference between .\' (I) and y(t) is
rt /2(ac) 112 •

SOLUTION: + ,J>] = I
Lei's find the difference in times when cost (ac) 112,
and sin[ (ac) 1 , 12
+ rf, 1= I. Lett = re when the cosine equals I and t = Is when
the sine equals I. This occurs when (al") 1·' 2, + ,p = 2111r (11 = 0. I. 2 .... )
=
for the cosine and when (ac) 1121 + </J 2.,m + rr /2 (n =
0. I. 2 . . .. ) fort.he
~i ne. The difference between ,, and '~ is rr /2(ac) 112 • or [2;r /(oc) l/2 J/4.

Notice that neither Equation 5 nor 6 reduces to Equation 2 when b = d = O.


when the 1wo populalions do not in1crac1. In lhc absence of any interaction, the
population of I.he prey increases indefinitely. while that of the predator dcc~scs
indefinitely. We can, therefore, improve the simple Lotka-Volterra model by in-
cluding negative quadratic terms in Equations 5 and 6. Let's consider the equations

.i = 3x - x y - 2r 2 ( I 0)
.}' = - y + 2x y - y2
These equations have critical points at (0. 0). (0. -1). (3/2. 0). and (I. I). We
can ignore the one at (0. - I) becaus.e x (I) and y(t) must be greater than zero.
The critical poini at the origin is a saddle poini, which is approached only along
1he line x = 0. The criticaJ point at (3/2. 0) 1s a saddle point with eigenvalues
(-3. 2) and corresponding eigenvectors (I. 0) T and (-3/ I0. I) r_ The asymptotes
662 Ch<1p1cr 1.l / Quali1,1rivc Methods for onlin ,:ir Differential Equation

We have considered only pairs of coupled non linear differential equations. but
one of the requiremenL, that a system of non linear first-order differential cqualions
must fulfill to show chaotic behavior is that there he at lea,t three coupled non linear
equations. It may not be obvious at first sight that Equation I I can be wrinen as
=
Lhree first-order equations. but if we let y .i- and.:=,. then Equation 11 becomes

.i- =y
_,. = x-1 - 0 .050_, + 7 .50 cos ~ ( I 2)

z=l

and these three equations lead 10 chaotic bchavior. The parameu-ic plot of y(1)
X
against x(1) in Figure 13.38 suggests a chaotic behavior.
We don·t mean to imply that all seL-; of three or more coupled non linear first-
order equations lead to chaotic hehavior. Both the forms of the equations and rhe
Figure 13.38 numerical coefficients must be just so. Unfortunately, rhere are no simple general
A parnmerric plot of y(t) ag-1in.,1 x(I) for criteria that tell whether or not a set of etjUi.tlions will di!:iplay chaotic behavior. but
&juations 12. many systems have been studied both experimentally and computationally and
there is an extensive literature on the subject. Problem 20 discusses the Lorenz
equations, which are the classical equations of chaos theory

13.5 Problems
I. Solve Equation 2 10 ob1ain Equation 3.
2. Derive Equation 4.
3. Linearize Equations 5 and 6 about the critical point (c/d, a/h).
4. Show that lhe solutions ro Equations 7 are ellipses centered at k/d. a/b) .
5. Derive Equations 9.

The ji)l/owing six problems require rhe uJe of a comp111er.

6. Divide_\· by .r to construct a phase portrait for the predator-prey system

.t = -2r - 0.0050.ry
_i,• = 4y - 0.015xy

7. Compute x(t) and y(I) for the system in Problem 6 and compare the period to 2,r /(ad 112•
8. Plot x(t) and y(t) parnmcrric:ally for the system in Problem 6 and compnre your result to the appropriale
member of the family plotted in Problem 6.

9. Figure 13.39 shows the parJmetrit: plor of y(r) against x (I) for 1hc sys1em

x = 5x - 0.40xy
j· = - IOy + 0.20xy
with x (0) = 200 and y(O) = 10. Interpret this result.
664 Ch.iph•r I 3 I Q11,11i1.i1iv1· ,\\t'thod~ tor Nonli1w.1r Oiiil'rt·n!i.11 (q11.11iun,

16. Analyzc the competing species model


i: = I Ox - 2.x ~ - 5x y

_,. = 14y - 4_v2 - li:y

What is Lhe ultimate fate of each specie),?


17. Analyz.e the compcling species model
.\" = 6x - 2x 2 ~ X)'

-~'=Sy - 3y~ - 2.xy

Show that I.he two species coexist in this case.


18. Equations similar 10 those in this section have been u!-ed lo model chemical reactions involving (imcrmediare)
species whose concentralions oscillate in time. Consider the kinetic scheme

i:~
X + Y ---=-+ 2 Y
y~p

Add these reactions lo show that the over.ill reaction is A--+ P. (Thai"s why X and Y are called
"intermediates.") Show Lhat rhc rate e<1uations corrc~ponding 10 thi~ reaction scheme arc

X = k1XA - k2XY
Y =k2XY - kJY

where A. X. and Y represent concentrations. Take A= Ao= 1.00 mol • L- 1 (this can be readily achieved by
=
adding A 10 I.he rcacr ion sys1cm as the reat:tion proceeds). k. 1 1.00 L • mo1- 1 • s- 1. k 2 = 0 .500 L • mo1- I - s- t.
=
k 3 0.100 s- 1• and X(O) = Y(O) = 1.00 mol · L -t ond show that X(f) and Y(t) arc periodic. Him; De1em,ine
1he phase portrait for this system.
19. E.qua1ion1> si.rn.ilnr 10 Lho~ in this section have been used to model epidemics. Suppose that out of a 101.al
population of II individuaJs . .r of them arc considered lo be su5ccpt.iblc. y of I.hem arc infected and can transmit
the di~ca-.c 10 a ~uscep1ible individual. and :.:. of them are recovered and immune. Argue 1ha1 1he equations

.r = -fhy
-~- = /fry - yy
t = yy
can model the. spread of the dil>eas.e. Show tha1 x + y +~= constant. Tokc fi = 0.20 and y = 0.040. x(O) = I
=
(in some uniL~) and y(O) 0.030 and solve the equations numerically for .r(t) and y(f) and dii.cuss the rcsull.
20. The conccpl of chaotic systems came to the anention of physical scientists and engineers in 1963 when the
MIT atmospheric l,cienti~t E.N. Lorenz proposed a simple model for 1.hemtally induced fluid convcx:1.ion in !he
atmosphere. His model consiste() of three coupled nonlinear equations:

.i- = a(y - x)

_\' = fix - )' - X;::

: = .ry - y::
665

There are hundreds of papers discussing 1hese equations. and many of them choose CL= 10. y = 8/3, and allow
/J 10 vary.
Using these parnmc1ers. show that I.here are critical points at (0. 0. 0). (f; (/3 - I) )112 • { ~ (/3 - I)] 112. f3 - I),
and (-1~(,'J - 1)] 1' 2 • -[~(,8 - l)J 1/.2, p - \). Use any CAS 10 show that the eigenvalue~ associated with
(0. 0, 0) are real and differ in sign (1wo nega1ive and one posili\le) when /J :> I. and the eigenvalues a.~sociated
wi1h the other two cri1ical poi nh arc the same and that one is ncgaiive and the other two are of !he form a ± i b.
Now show 1hat a < 0 ir /J < 24.74 ... and a > 0 ir /J > 24.74 .... How would you inicrprct Lhis result?
Now ,olve the Lorenz equations for {J = 15 and x(0) = 15. y(O) = 10. and .:(0) =
15 and then for f3 = 15
and x(O) = 15. y(O) = I 0.1. and z(O)= I 5. and show that the results arc es.,cn1ially idcn1ical. Now do the same
thing with fJ =
30 and show 1hat the 1wo solutions differ markedly.

Rl'ferences
NONLINEAR DIFFERENTIAL EQUATIONS:
D. Acheson, 1997, From Calrnlm to Chao.f. O.,ford
W.E. Boyce and R.C. DiPrima. 2000. Elementary Diffi•re11tial Equuticms and Boundary
Value Problems. 7th ed .• Wil~y
I.R. Epstein and J.A. Pojman. 1998. An ln1rod11c1io11 to Nonlinear Chemical Dynamics.
Oxford
C. Hayashi, 1964. Nonli11eor OscillcJ1io11s in Physical S_,·stC'm.s. Princeton
LD. Huntley and R.M. Johnson. 1983. linear ancJ No11/inear Differemial Equations,
Hals1e::1d
D. \V. Jordan and P. Smith. 1999. No11/i11ear Ordinary D(l]en,ntilll Equations. Ox ford
R.A. Struble, 1962. No11li11eor /)ifl1.·IT1uinl Equatirm.s. McGraw-Hill
F. Verhuls1. 1990. Nonlinn,r Di.fferelllial £q1wtio11.1· and Dy1111mirnl Systems, Springer-
Verlag
CHAOS:
G.L Baker und J.P. Gollub. 1996, Chaotic DvnomicJ·. 2nd ed .. Cambridge
R.C. Hilborn, :WOO, Clwm and Nonline"r Dy1uu11ics. 2nd ed., Q:,,;ford
G.B. M.indlin. 1996, Non/i,u:ar Dynamics: A Twn-"0y Trip from Physio to Moth, Institute
of Physic-s
S.N. Rasband, I 997. Chaotic Dynw11ics of Nrmlinear Sy,Ytems. Wiley
S.H. Strogatz. 1994. Non linear Dy11nmi• •1· and Cluw.c iri//, applications to physics, biology.
chemistry. tmd e11J:ineeri11g. Pc~eus
J.M.T. Thompson, 2002, No11/i11ear Dynomics and Ch1.Jos. 2nd ed., Wiley

C gl
668 Chapter 14 / Orthogonal Polynomials and Sturm -liouville Problems

14.1 Legendre Polynomials


We saw in ChapLer 12 lhal !he Legendre polynomials are solutions 10 the differen-
tial equarion

(1 - x 2 )y"(_t) - 2.x/(x) + n(n + l)y(x) =0 (I)

where n is an integer. The firsl few Legendre polynomials are

P0 (x) =I
(2)

and are plotted in Figure 14.1. Nole that P,1 (x) has exactly 11 - 1 distinct zeros in
figure 14.1 the open interval (-1. I). A general formula for Pn(x) is
The Legendre polynomials. P0 (x) (solid).
ln/21 .
P1(x) (long dashed), P~(x) (~hortdashetl).
P3 (x) (dash-dol), :ind P4 (.1 I (t.loncd). P11 (X ) -- ~ E<- I)i 2 2
- - (-n-- - -
1
J).- X n-2) (3)
2" j=O j!(n-j)!(n-2))!

where [n/2] = n/2 when n is even and (11 - 1)/2 when n is odd. Bccausex appears
as x 0 - 2J in Equation 3. P,, (x) is an even function if 11 is even and an odd function
if ,1 is odd (Problem l ).
ll shouldn't be apparent at this point. but 1he Legendre polynomials form an
orthogonal se1 of funct.ions over the interval [-1. I!. For example,

Generally,
1

1 -l
P,1(x)P,,,(:r)dx =0 if m '#- n (4)

The orthogonality propeny of the Legendre polynomiaJs follows from the


differential equation that defines 1.bem. Let's go back to Equation I and write it in
the form

(5)

with a similar e<1ua1ion for Pm(.x)

(6)

Multiply Equarion 5 by Pm(x) and in1egra1e from - I to+ I:

-
11 -I
-, I I
l(I -x~)P,Jx)] P,,,(x) dx = 11(11 + I)
fl-I
P,,(x)Pm<x)dx
I 4. 1 Legcnd,e Polyrn ,r1, i.1 I, 669
ln1egra1e the lef1 side by parts 10 obrnin

Now multiply Equation 6 by P,, (.r i and integrate by parts from - I to+ I to get

(8)

Subtract Equation 8 from Equarjon 7 10 get

111(11 + I) - m(m + 1)11 P11 (x)Pm(x) dx =0


-1

lfn f. 111. lhcn we get Equa1ion 4, the on.hogonality condilion of the Legendre
polynomials.
The Legendre polynomiaJs also satisfy a number of recursion fonnulali. For
example.

11 2: I (9)

Example 1:
Use Equation 9 to derive expressions for P2 (x) and P3 (.r) from P0 (x) =I
and P 1(x) = .r.

SOLUTION: Let 11 = I in Equal.ion 9:

or
I
P.,(x)
-
= -(3x
2
2
- I)

For n = 2:
or

Equation 9 is particularly well suited for numericaJ rourjnes.


The function
00

G(x. r) =L P11 (x)1'


1

n=O

is called a generating f11ncrion for the Legendre polynomials. If you know the
funcLion G(x, r), then the coefficient of 1" in the Maclaurin expansion of G(x, r)

C gl
670 Ch,1pll•r 14 / Onhogonal Polynomial$ and Slurm-Liouville Problem~

is P,,(x). Thus. G(x. t) is said to generate the Legendre polynomials. Problem 6


helps you derive an explicit ex.pression for G (x. t) from Equation 9. The result is

I
G(x.t)=------ ( 11)
(1 - 2.xt + , 2 ) 112

Example 2:
Us.c Equation 11 to generate l11e first three Legendre polynomials.

SOLUTION: Use rhe expansion

-If' Z. J 1 5 1
(I - ;:) - =I+ - + -z.- + -z· + ···
2 8 16

with 4 = 2.xt - ,~ .

~ ' ~
2
, 3 1 5
G(x, t) =I+ .rt - -
2
+ -14x-r
8
- 4xr· + O(t 4 .)I+ 16
3 3
-[8x t + O(t )]
y
3x 2 - I 2 5x 3 - 3x l _.
= I +xt - -I + ---r -1- O (I )
2 2

Equa1ion 11 may not look familiar. but you actually know the equation from
electrostatics. Suppose an electric charge q is located at a point specified by a
vec1or I. The electrostatic po1en1ial at a point specified by r (Figure 14.2) is given by
X

V= q ( 12)
4rrfol r - 11
Figure 14.2
The gcomc1ry used 10 calcula1e the
clcctroslatic potcnrial at r due ro II charge
where € 0 is the penniuivity of free space. Using the law of cosines. ! r - 11 =
locu1cd al J. (r 2 + ( 2 - 2rl cos 0) 112, Equation 12 becomes

V = _____ ...;..q_ __ _ _

I (' ) t/ 2
4m: 0r ( I - 2- cos 0 + -
r r1

Comparing Equation 13 to Equation 11, with x =cos O and t = I/ r. and using


Equa1ion IO gives

( 14)
14. 1 l.cgenclre Polynomials 671

Suppose now we have a set of charges qi at posilions specified by I;. The


potential is given by

Equation 15 has an important physical interpretation. Let the sum mat ion over j in y
Equation 15 be denoted hy

M 11 = L, q/J P, (cos 0;) 1


( 16)
j

Then V can be wrillen a,;


+
\I _ _ I_L :x,
M" ( I 7)
4rr1; 0 r 11 1
11=0
X

Let's apply Equation 17 to the two-charge distribution shown in Figure 14.3.


Using Equation 16, 1he first few M,, in Equation 17 are
Figure 14.3
The two-charge distribution used derive
M0 = lJ - q =0 Equ:.ition 18.
10

M1 = -qi2 cos0 - qi
-
2
cos(rr -0)
.
= qi cos 0 = µ cos 0
where,, is the magnitude of the dipole moment of the two-drnrge distribution, and

q/2 q/1
M,
-
= -A(cus0)
2 -
- - P., I ws(:r - H)I
2 -
=0

Equation 17 becomes

Jl cos O
V= -- + O ( - I ) (18)
41T1; 0r 2 r.J

Equation I R is 1he clec1ro~1a1ic potential due to a dipole located at the origin of a


coordinate system. Jf r » I. then the first term dominates all the others.
Figure 14.4 shows equipotential lines and the corresponding eleclric lieh.l
for the dipole shown in Figure 14.3. RccaJI that the electric field is given by
E = -grad V. which in this case i~

nV I JV Figure 14.4
E = - er + - -·- e/} The equipotential lirws (color) 3Ild the
ar r an corre~pondi ng electric field (black) due to
the dipole shown in Figure 1-t ..l.
µ cos 0 µ sin fJ
= - - - - 3 er - - - - 1 e11
4rr 1; 0 r 4rr e0 r·
672 Chap!l!r 14 / Or1ho8onal Polynomic I · nd Slurm-Liouville Problems

)'
Example 3:
Apply Equation 17 10 the poinl charge distribution shown in Figure 14.5.

SOLUTION: Using Equation 16, the lirsl few 1Hn are Mo = 0.


qi qi
M, = - cos0 +- COS(JT -0) =0
2 2
q(!- 4(!.
M-, = - P2 (cos 0)
- 4
+ -P,(cos(,r
4 -
- 0))

X 2 2 2
q/ q/ 3 cos 0 - I
=- P2 (cos0) = - - - - -
2 2 2
Figure 14.5 qi]
The linear quadrupole Lhal is used to M3 = -8 P1(cos
. 0)
de1em1ine the ele-ctrosratic potenrial in
Example: 3. To calculate M 2 and M 3• we have used 1hc fact rhat P2 (- cos 0) = + P2(cos 0)
and th:H P3(- cos 0) = - PJ(cos 0).
Equation 17 becomes

where Q is the magnitude of the quadrupole moment of the charge


distribution. If r > I. 1hen the potential due to 1be quadrupole arr.he origin
is given by r.he first 1em1 above. The equipotential lines and electric field
(arrows) due to a quadrupole moment located at the origin are shown in
Figure 14.6.

Figure 14.6 The quantities Mn in Equation 16 are called m11ltipole momellls. with M 1
The equiporcnti::i.l lint'S (color) and rhc being the magnitude or the dipole moment. µ,. with 1W2 being the magnitude of
corresponding electric field (black) the quadrupole moment. Q. and so on, and Equa1ion 17 is called a multipole
due 10 1he linear quadrupole shown in
Figure 14.5. expansion. Mult:ipole expansions play a key role in the theOI)' of the interactions
between molecules.
Equation 11 is awkward to use to generate Legendre polynomials. but il is very
useful for developing general properties of Legendre polynomials. For example.
Problems 7 and 8 have you use Equation 11 to show generally lhal

1
2
1 -1
P 2 (x)dx
II
== - -
211 +I
( 19)

Combining Lhis result with Equation 4 gives


1

! -1
P,,(x)P,.,(x) dx
2
= ---cSnm
211 + I
(20)
14. I Lcgend1e Polynomials 673

Example 4:
Show 1hat the first few Legendre polynomials obey Equation 19.

SOLUTION:
1

f-1
PJ(x)dx =2

J I
-I
Pi(x)d.( = -]
4
JI -l
(3x 2 - 1)2dx
':>
= .::
5

A useful propeny of Legendre polynomials, as well as other orthogonal


polynomials that we shall encounter in Lhis chapter. is that it is possible 10 expand
a suitably behaved funcLion J(x) as an infinite series of Legendre polynomials
00

J(x) = I::a,,Pn(x) (21)


n=O

We shall explain below what we mean by this equality, bu1 first let's de1em,jne
the a11 by multiplying both sides of Equation 21 by P111 (x). integrnting over x. and
using EquaLion 20 to obLain

=La,, ll P,/r)Pm(x) dx = n=0211


00

1-I
1
j(x)P,,,(x) dx
c.o

"=-0 -l
L ~ 2 + l snm
2aJI/
= 2m+ I
or

a 111 2m +
= --1
-
-
11 -I
1
J(x)P111 (x) dx (22)

EquaLion 22. along with Equation 20. is reminiscent of an expansion of a


vector in terms of a basis of orthogonal vectors. In fact, suppose that J (x) is
continuous on the intervaJ (-1. 1). It is easy to show that lhe set of all con1inuous
functions on (-1. I) forms a vector space. It is an infinite ctimensionaJ veclor space.
and so requires an infinite number of basis vectors. as in Equa1ion 21. The inner
product in Lhis vector space is defined by

(f. g) = 1 1

-I
J(x)g(x)dx

where j(x) and g(x) are any 1wo vectors in 1he vector space. The 0 111 given by
Equation 22 can be 1hough1 of as the components off (x) in the basis I Pm(x) l.
m =0. I. 2.....
676 Chapter 14 / Orthogonal Polynomi,11~ arnJ Sturm-Liouville Problem\

This inequality is called Bessel's inequalif)', which is valid for any vaJue of
N. i.ncluding N ---+ oo,

SOLUTION: Start wil.h

Then.

f/ 2La 1
I N l
2
(:r::)dx - 11 J(x)P,,(x) dx
-I 11=0 -I

L L a am f
N N I

+ 11 P,,(x) Pn,(x) d,x :::: 0


n=Om=O -I

or

(25)

You can use Bessel's inequality to show that hm l2/(2n


11
+ l)J 112 a,, = 0 (Prob-
lem 18).
00

The Legendre ~ries L a 11 P, 1 (.r) of any square integrable function f(x.)


11=0
converges to j(x) in 1he mean. We say lhal the Legendre polynomials fom1 a
co111plere set; they span the vector space of square integrable functions. Because
lhe Legendre polynomials form n complete set. Equation 25 implies that

1(
I oo

.
2(x)dx ~ ___2_a
= L.q I,,
2
(26)
-J n=O _n +

Equation 26 is called Parseval 's equaliry. You can think of it as an infinite vector
space version of Pythagoras's theorem.
Equation 26 is a statement of tht! cornple1cne~~ of the Legendre polynomials.
You might 1hink that 1he Legendre polynomials are complete because there is an
in ii n.ite number or Ihem. bu1 that is nol so. If we snip our one of Ihem. say P18 (x).
west iII have an infi ni1e number of them, hu1 they are no longer complete. Although
Equation 26 gives us a definition of <.:omple1eness. ii is difficult in practice to show
1ha1 any set of functions is complete.
14. I Legendre Polynomials 677

14.1 Problems
J. Use Equation 3 10 generate the tirst few Legendre polynomials.
•I
2. Show lhat P,,(x) is ort:hogonaJ ro every power of x less than n. In other words. show Lhar j_, r' Pn(x)dx =0
s
ifs < 11 . Hi 111 : Use the fact lhat we can express .r·' as a linear combina1ion x ·' = L a1 P/x).
J==O
3. Use Equation 3 to show that P,, ( -x) = l -1 ) 11
P11 (.r).

4. Show explicitly th::it P 1(x) is orthogonal to P'!.(x) and to P.1 (x).

5. U::-:e Equation 9 to derive an expression for PJ(x) from P2 (x) and P.3(.r).

6. We'll sec how 10 derive a genera1ing funcrion G(.r, r) from Equation 9 in 1his problem. Mulliply Equation 9
by 111 and sum from 11 = 0 ro oo to obtain G' (/) - 2.'(/ G' - xG + t (1 G )' = 0 or ( I - 2.n + ,'!) G' = (x - 1 )G.
Integrate 1.his expression and use lhe condi1ion Po= I to obtain Equation 11 .
7. In this problem and the next problem. we're going 10 use 1he generating function (Equa1ion 11) to show that
the Legendre polynomials arc onhogonol. S1an with

OC 00

G(.r. l)G(x, 11) =L L P,,(x)P111 (x)t"11"'


n=Om=O

Now argue that lhe Legendre polynomials arc onhogonal if j'


-I
G(x. 1)G (x. 11 )d.r = func1ion of 111 only.
8. Carry our lhc integra1ion in the previous problem and show explicitly that the result is a function of,,, only.
9. Lei x = I in the gencrnr:ing func1ion in Equation I I to show 1ha.1 Pn( I) = I.

10. Use 1hc gcncra1ing function in Equa1ion 11 10 show that P11 (- 1) = (-1)".
11. The gcncra1.ing function in Equation 11 is a function of 2.xr - , 2. which we cm write as G(x. t) = f (2.xt - 12).
.
F1rs1 show tha1 -
ac = 2, F , and -ac = (2.r - ,
21)F. where
,
F means d F 1d(2x1 -
,
1·). Now show that
1
dX
(.r - 1) ~G - r ;~C
ux ol
JI
= 0. Given lhat G(x. 1) = t
n=O
P11 (.r)t 0 • show 1ha11he Legendre polynomial),, ,atisfy the:

differential recursion formula x P,/r) = n P,.(x) + P~_ 1(x).


12. We"ll derive another <liffcrcnrial rccuri-ion fomlLila (see the pre\•ious problem) in lh.is problem . First show
ac
1ha1 - = t(I - 2xt + r ) - - =
1 'l .,
00
,
L
P (x)1 11 and that - = x - , ac
"> r =
oc
nP11 (x),n-r _ Now L
ox n=I ,, Ar (I-L'(/+1-) 3 ~ ,i=I

multiply 1he fin-I equation by ( I - 12)/,. the second by 21. and sub1rac1 to show that

~ ~ oc X

L P~(.r)I"-' - L P~(x)1"+' - L 2.nP,.(x)t" = L P,,(x)1".


678 Ch.iplt•r 14 / Ortho~onal IJolynomi.11, .ind ~lurm-Liou\'ille Prob/i•m~

13. We can derive Legendre•~ differential cqualion from the two diffcrcn1ial recursion fom1ulas in Problems 11 and
I 2. U,c these two results 10 show thal x P,;(x) = P,;-1)x) - (11 + 1) Pn(.t) or x P~_ 1(.r) = P,:C.r) - 11 P11 _ 1(x).
Di ffcrcntiate 1his with respect to x to obtain x P,;'_ 1(:r) = P~' (x) - (11 + I) P,;_ (.r). Now use this result
1

and the result of Problem 11 10 eliminate P,1


, ...
1
(x) and P'' .
11 1
,,_ 1(x) to obtain ( I - .r~ J P,, (x) - 2x P" (.d +
11(n + l)P11 (.t') = 0 .

l 2n~t(n!)2 .
f
n
14. Show thal x P,,(x)dx = -----. /-111,r: Use rhe fact that x" = f:2"(11!) 2/(211)!]P,,(x)+ a lower
-l (211 + 1)(2n)!
dl"gn:c polynomial.

15. The integraJ f I

-I
xP11 (.x)Pm(x)dx occurs in atomic spectroscopy. Show that/=
2 11
(
(2'1+1)(211+3)
+ I) cS,,, ,n+ t+
2n
8
(211 + 1)(2n - I) "' ·" I·

16. Consider lhc sequence of func1ions f,.(x) =


x" for U.:::. x s I. Show th::il this $cqL11.·11u· converges to
1
Jim J,,(.x.)
,,-,co
={
O O .'.:: x < . Now show th::it it converges in 1hc mean 10 f(x) 0 for O :5 x:::. I.
I X = I
=
17. Show that D~ given by Equation 23 is a minimum with respect 10 et,, if they are equal lO the a,, given by
Equation 22.
18. Use Bessel's inequali1y to show that lim (2/(211 + 1)] 1nll, = 0. (Sec Equu1io11 25 .)
,, 1

J9. Expand I.he function f (x) = sin rr x in a series of Legendre polynomials. Plot the Ii rsl few partial sums .
20. Expand j(x) = I -.r~. -I -- x :S l. in terms of Legendre polynomials. Verify Parscval's equality for rhi:- case.
21. Suppose we wish 10 expand ::a fum:tiun defined on lhc interval (a. fJ J in terms <>f Legendre polynomials. Show
1ha1 the rr..in~fonn;-ition" = (2-l - a-• /J)/(fJ - a,) map.~ the func1ion onto 1he interval (-1 . l).

22. Expand /(x) =I- x 2 /4, -2 ::: x :::. 2. in lenns of Legendre polynomials.

14.2 Orthogonal Polynomials

As we implied in the previous section. the Legendre polynomials are just one of a
number of ··name" polynomials that arise in applied mathematics. In this sec1ion
we will presen1 a general lhcory of on.hogonal polynomials that ~m;ompas'.'-e~ any
one of them as a special case.
Consider a SCI of functions i;t>0 (x). 4> 1(x), .... This sel is said to be orthogonal
over an interval a ~ x ~ b with weight function r(x) ~ 0 if

J II
t, r(x)<t,,-(x)<J>lx)dx =0 (I)

Note the presence of the weight function r(x ). which is equal to one for the
Legendre polynomials. If in addition 10 Equa1ion I. we have

1,tl
r(x)</J;(x)dx =I (2)
680
Table 14.1
Some commonly-used orthogonal polynomiaJs.

nnme symbol intervaJ weight function

Legendre P"(x) -1 ~x ~ I
Chcbyshev(Tchebychef) T,,(x) -11
~x ,;::, I ( ii _ x1)-112

Laguerre L,; (X ) O~x < oo e-.t


A~~ociated Laguerre L~a'>(x) Xa (:, - J
0 !: X
Henni1e 1-/n(X ) <X < ,, - .r '

Hermite Hen_(X) < ,l' < e - ., 12

The defining intervals and weight funclions of some other commonly occur-
ring sets of orthogonaJ polynomials are given in Table 14. 1.
We devoted the entire first section of this chap1er 10 1he Legendre polyno-
mials. The a~ociated Laguerre polynomials reduce to the Laguerre polynomials
(Example I) when a = 0 and also occur in Lhe quamum-mcchanical treatment
of a hydrogen atom. The Hem1ite polynomials. H,,(x). occur in the quantum-
mechanical trea1.mcnl' of a harmonic oscillator and those designated by I-/ e"(x) in
Table 14. I are used in mat.hemarical s1a1is1ics. The first few Hem1i1e polynomials,
Hn (.r), arc (Problem I)

Ho(x) = I H 1(x) = 2.r f-1 2(.\·) = 4x 2 - 2


(4)
H3(x) = 8x 3 - 12.x H:!(x) = 16x4 - 48-r~ + 12
f T,.(x)
Note t.hat H 11 (-x) = (-l)"H,,{x) and Lha1 the coefficient of x" is r (by conven-
tion). The Chebyshev polynominls ure used in numerical analysis; 1he first few are
given by

T0 (x) =I
(5)
figure 14.9
The Chcbyshcv ,polynomials. 7i) !.l")
(solid). T1(.x) (IQng d:ished), T~ h l h hon By convcnLion, Tn(l) = I. Note that Tn(-x) = (-l) 11 T,,(x). Figure 14.9 shows the
dashed). T3(x) (cfar-da.-.h), and 7~(.\)
first few Chebyshcv polynomjals plo!lcd against x.
(<.loncd). plouc<l agoinsr :r.

Example 2:
Show thal' T0(.r) is onhogonal 10 T:!(x) with respect lo the weight funclion
( I - x~) I .'Z over the inierval (-1. I).

SOLUTION:

al
14.2 Orthogonal Polrnomial 683

where we have used T 1(x) = x. Purring all lhis together. we have

=L
,:.:
1- :rl
G(x. r) = T, 1 (x)1 n
I - 2xt + r- n=O

Table 14.3 lis1s the genernling functions associated wilh the recursion fonnula.s in
Table 14.2.
We can use a genera1ing function 10 de1ennine rhe value of

just as we did in the previous secLion for Lhe Legendre polynomials. Let's do lhi~
for 1he Henn ire polynomials. We mul1i ply the square of G (x, 1) by r (x) = e- .1 ~
::ind in1cgrntc over x:

(8)

l'~i.ng 1..hc orthogonality of the Hn(x). the integral on the righl becomes 811 "''1,,,.
and we have

~ ·X· !"Tm ,211


L '°' '°'
'.X.•

'°' L -8 1 !
11-0 111-0 II .Ill.
nm h m = 11L: 0 - h
( I)'
II . -
11

Table 14.3
The gcnera1ing functions of rhe orthogonal polynomials
listed in Table- 14.1.

g~ncrating function

P,.(.r) ( I - 2:rt + r 2) i.~= L P,,(.r)l


11
ltl < l
,: , ,-()

T,,(.r)

ltl < I
686 Chapter 14 / Orthoi,.:011.11 Pol1,1lllmi,1I~ .11111 S1u1rn-Liouville Prohl('f11~

14.2 Problems
1
I. Generate rhc first few polynomials 1hat arc onhogonal and normalized wirh respccr 10 e-.1: over 1he interval
(-oo. oo).

2. Show lhat H0 (x) and H 1(x) arc on.hogono.J to 1-12 (.x) with rc~pt.,-cl 10 lhc weight function e--.r? over the interval
(-oo. oo}.

3. Dc1ivc an expression for H4 (x) from H2 (x) and H3(x) using I.he recursion fonnu la in Table 14.2.

4. Use tJ1e rccur:-.ion formula for lhe Laguerre polynomials (a = 0) 10 verify the formula for L:i(x) from 1hc
expressions in Example I.
5. Show !hat lhe Laguerre polynomials that we derived in Example l are onhogono.1.
6. Srart.ing wiLh the recursion fonnu la for Hermite polynomials in Table 14. 2. derive the generating function

= L I Hn(,r)t". Lei
., 2 ::-.. I
G(x. t) = ,,-tr-, Ho(x) = I.
1'=0 11.

7. Use the recursion formula in Table 14.2 10 derive I.he generating func1ion of lhe Lag_uerre polynomials (wi1h
a =0).
8. Use tllc generating function in Table 14.3 10 derive formula.-. for the first few Hennire polynomials.
9. U:..c 1hc gcncra1ing func1ion for 1hc Laguerre polynomial~ (,virh C'i = 0) in Table 14.3 10 ~hnw that
lor~ t! - r L,,(.r)L,,,(.r)dx = 8,m,·
10. TI1c inti;grnl / = j"°
-:xi
1
e- " /·ln(x)x H 111 (x)d.x occurs in a discus,-ion of rhc vibrational spectrum of a d1a1omic
molecule i.vhen il is modelled a~ a hannonic oscillator. Show that 1his integral is equal 10 1..cro unless m = n ± I.
11. The avemgc potcn!iaJ energy or a quantum-mechanical harmonic oscillator is directly related to 1hc integral
I= f_ -.:J 1
e- ·•' H11 (.x)x H,1(x)dx. Show 1ha1 / = (11 + ~)ft 2"n!.
12. Derive Equat·ion I0.
13. We shall derive Equa1ion 7 in this problem. We choose un so th:H IP,,+ 1(.r) - a,,.r<t,,,(x) is at most of degree 11
(the x 11 +l 1cm1s cancel). Then

f/

t/>n+1(X) - a 11 X</>n(X) = L a,f/>1(x) (I)


/=0

Now multiply both 5idc~ by ,- (x )et,; (x) (i = I. 2, ... , 11) nnd inlcgr..ite over (a. b) and ~how rhat

(2)

Now. argue thar because x<t,1(.,) is a polynomial of degree xi+I _l!he numerator in equation 2 will equal lCro
=
unles i 11 - I or 11. Therefore. only a 11 1 and a,, in c.qua1ion I arc non zcro. and so cqua1ion I reduces 10
Equation 7.

14. Derive Besser~ inequality. Equation 12.


15. ll-..c a CAS to expand /(x) = sin 2:rrx in tem1s of Chebyshev polynomials.
14 _3 Slurm-Liouville l hrory 687
16. Use a CAS to expand e-.,~/ 2 cos 2t· in rerms of e- ·1 l 2 H,r<,x). Verify Parscvafs equality for I.his case.
2
17. Use a CAS to expand ,-.( cos 2x in tcnns of ,,- x / 2 Hn (x) and verify Bessel's inequality and Parseval·s equality
for this ca.se.
18. Use a CAS lo expand e-x! in Lcrms of,. t Ln(x). Verify Bessel'.-. inequality and Parseval's equality for this
case.
19. U. ~ a CAS to expand 1, -l fl in terms of c ~: -~ Hn(x). Verily Bessel"s inequaliry and Parscva.J's equaJity for this
case.

14.3 Sturm-Liouville Theory

In Section I. we showed 1hat the Legendre polynomials arc onhogonal by s1aning


with 1hc diffcrc111ial equarion that they s.i1isfy. Also in Scc1ion 12.6. we showed
1ha1 the Bessel functions, Jn (x). obeyed a ccnain 1ypc of onhogonali 1y condition.

where et; and aj are (dislinct) 7.eros of Jn(x). We derived Equa1ion I by s1arting
with 1he diffcren1iul equation for l,,(x ). In Lhis section, we shaJl pre~nt a general
theory tJ1a1 encompasses all 1he orthogonaJ polynomials that we have discussed
up to now and also Equation I, an onhogonali1y rel.ition t.ha1 does not involve
polynomials. This theory is due to S1unn and Liouvil\e and is called the St11m1-
Liou11ille theory. S1urm-Liouville theory plays a central role in 1he mathematical
formulation of quamurn mechanics.
Consider the differential equation

I p(x)/(x)]' + [q(x) + >..r(x)ly(x) =0 (2)

where >.. is an un.!-pecitied parameter at this poi nl. The functions p (x ). p' (x ).
q(x). and r(x) are cominuous on the interval [a. bi and p(x) 2:. 0 and r(x) 2: 0
everywhere in Ia. b l Equation 2 togelher with the boundary condilions

(3a)

and

(3b)

cons1itutc what we call a Si-urm-Liouvillc problem. In EquaLions 3 we a.,;sume 1hat


1he coefficicn!s in &jua1ions 3 are !ieul and tha1 al leas1 one a and one /J -:/=- 0.
Equations 2 and 3 describe a greal variety of physical problem~. Equation 2
may look rc~1ric1 ivc. bu1 even the general second-order cquat ion a (x )y'' (x) +
b(x)_,.,(x) + c(x)y(x) + >..d(x)y(x) = 0 can be put into the form of Equation I
if a(x) ;t O (Problem I).
14 .3 Srurm-Liouville Theory 689

We tacitly assumed that the eigenvalues in Example I are real. We can use
Equations 2 and 3 or Equations 3 and 5 to not only show that Lhe eigenvalues
of a Stunn-Liouville problem are real. bul also that the eigenfunctions fom1 an
orthogonal se1. Even though p(x). q(x), and r(x) in Equatjon 2 are real. we'll
allow for the pos.1.ibility that A and y(.x) are complex. S1a11 with Equation 5 for
Yn(x) and y111 (x) and take the complex: conjugate of the equation for Ym(x) to
obtain

and

Mu.ltiply the first of Lhese equations by y ;, (x) and the second by y,,(x) and integrate
both from a 10 b to obtain

1 n
h
r (.x)L -m
•"
r'" (x)dx = ).,"m j
u
·h
r(x) Jm
'-.• (X)) 7n (x)dx

Now subtract these two equations:

(6)
Using the definition of L given by Equation 4, the left side of Equation 6 is
(Problem 2)

Equation 7 is a key equation of Sturm-Liouvillc theory. lt"s e.a$y to show 1h01


1he boundary condirions in Equation 3 make the right side of Equation 7 equal
10 z.ero (Problem 3). If p(a) "!- 0 and p(h) -=I=- 0, we have what is called a regular
Sr11rm-Lio111•illt: problem . In this Ctl!-it, p(x) > 0 for a ~ x ~ b . On the other hand.
suppose that p(a) = 0. Then we really don't need the boundary condition 3a; all we
require is that the solution and its derivative be finite at x = (J. Similar ~ituaiions
occur if p(b) = 0, or if both p(t1) = 0 and p(I>) = 0. If p(x) == 0 al either (or
both) boundary, Lhen we have what is called a singular S111m1-Lio111•ille pmhlem.
In this case, p(x) ::: 0 for a ~ x ~ b. We shall see below that some of Lhc general
propert.ies of S11.1nn-Liouvil\e systems depend upon whether it is a regular or a
singular Stunn•Liouville problem.
14. 3 Siu rm -Uouvi II Th ry 691

Now if,.,,, and An are different, then A,11 f. A. 11 and the inLegral in Equation 11 must
equal zero. Thus. we )\ec that

( 12)

The eixr11fl111cricms correspo11di11p. ro d!l],·re1·1r eigt>11wlf11es of a Srur111-


Lio1ll'illc .r_,·srcm arc ortltogo11af.

We learned about a number of onhogonal polynomials in Section 2. All the


polynomials discussed there a.rise from singular Stum,-Liouvillc problems. The
differential equations that these polynomials satisfy are listed in Table 14.5 . We can
ca.!;I these equations into a Sturm-Liouville form by using !he procedure (lUllincd
in Problem 1. We show I.here that lhe equation

,1(.x)y"(x l + b(x)y'(.r) + c(x)y(x) + )..d(x)y(x) =0 ( 13)

can be wrillen in the Stunn-Liouville fo1m:

[p(x)y'(x)J' + (q(x) + h(x)]y(x) = 0 ( 14)

where

p(x) = exp
.I b(.r)
--dx
a(x)
( 15)

and q(x) = p(x)c(.r)ja(x) and r(x) = p(x)d(x)/a(x). so long as a(x) t- 0. You


can readily verify this result by sutistituting Equation 15 inI0 Equation 14 and then
comparing your result to Equation 13.
Table 14.7 li)\1s the ditferentiul equal.ions in Table 14.5 in Stunn-Liouville
fom1 . Note that thcy are nit singular for one reason or another. The function
p(x) = I - x 2 for Legcndre·s equation is e.qual to zero at x = ± \,so the boundary
conditions at .r = ± I arc simply 1hat the solution (and its derivatives) is finiIc ar
these points. Recall from Chapter 12 that this is exactly the condition thal leads 10
the solution being polynomials. The form of r(x) in each differential equation in

Table 14.7
The Stum1-Liouville fonn of the defining differential equation for a few
orthogonal functions .

range of x differential cquntion

P,,(x) - I .:5 x 1( ,1 - .r:\v'(x)]' + A(A + lh(x) = 0


t~01 (x) 0 .\' < [_ro+le-.ry'(x))' + AXcr e - .r _\'(X) = 0
f-1,,(X) -OO < X < rc-.1 \' (x) I' + 2>-.e- · \<x > = o
T,,(xl - I~ x ~ I [( 1 _ .r2)1f2_r'(x)J1 + )._(J _ x2)-1/2y(.r) =0
J0 (.r) 0::: x::: l l.ry'(x)I' + )~ry(x) = 0
692 Chaplt•r 14 / Onhogonal Polynomials and Sturm-Liouvdle Prohl(•rm

Table 14.7 is the weighting factor for the orthogonality property of 1he functions
listed. RecaJI that lhey were given wilh no jus1ification in Table 14.1.

Example 2:
Express Lagucrre's diffcrcntinl cquntjon (Table 14.5) in Lhe form of a
Stunn-Liouvil.le equation and use Equation 12 to deduce the on.hogonality
condition.

SOLUTION: Lagucrrc·s differential equation is

xy"(x) +(a+ I - x)y'(x) + ny(x) = 0

To convert to a St:unn-Liouville form, use the re..;ult of Problem I. which


say!; 1hat

p(x) = exp
(! b(x)
a(x) dx
)
= exp (! a + xI - X
dx
)
= x + Ie - >.
0

q(x) = p(xk(.r)/a(x) = 0 and r(x) = p(x)d(x)/a(x) = xc,e-.r. Thus. the


Sturrn-Liouville form is

in agreement with Table 14.7. TI1e orthogonality condition (Equation 12) is

"' 'I- n

in ogrecment with the previous section.

Let's go back to Example I in view of what we have learned about Sturm-


Liouville problems. The eigenvalues are A,,= n 2 rr 2 . 11 = I, 2.... and I.he eigen-
functions arc q,,,(x) = c,, sin nrr x. Note that these eigenfunctions are orthogonal
over (0. I) lx.~ausc

£
•0
1
sin mrrx sin mrx dx =0 111 f. II

We can normnlize them by wriring

"11 ..,
c~
"
0
sm~ 117TX .,
dx = c- • -I = I
n ?
-

Thus. the or1honormal eigenfuncl ions are tp,, (x) = 2 112 sin ,rn x. = l. 2, ....
11
695
may or may not yield discrete eigenvalues. Th.is is one of lhe primary differences
be1wecn regular and singular Sturm-Liouville problems.

14.3 Problems
1. Show thal Lhe equntion a(x)l'(x) + b(,x)y'(x) + c(x)y(x) + i.d(.x)_r(x) = 0 can be put inro a Si-unn-Liouville
form by dividing through by a(x) and 1hcn multiplying by p(x) =exp(! b(x)
a(x)
dx).
2. Derive Equation 7.
3. Use the boundary condil.ions in Equal.ions 3 to show that 1he righr side of Equation 7 equals zero.

4. Show rhat if p(u) = p(b). 1hen 1he right side of Equation 7 will equal zero if y(a) = y(h) and y'(a) = _r'(h)
(periodic boundary conditions).

5. Show that the eigenfunctions in Example 3 are onhogonal.

6. Show explicitly lhat 1hc cigcnfune1ions of Example 4 arc orthogonal.


7. Determine 1he eigenvalue~ nnd eigenvec10~ of /'(r) + ). 2y(x) = 0 wi1h the boundary conditions y(O) = 0
and y'(/) = 0.
8. Determine 1.he eigenvaJues and eigenvectors of y"(x) + i.\(x):.: 0 with 1he boundary conditions y'(O):;:: 0
and y(rr) = 0.
9. Detennine the eigenvalues and eigenvectors of y''(x) + }/y(x) = 0 with the boundary condition."- y'(O) =0
and y'( I) 0. =
lO. Consider the Sturm-Liouvillc problem y"(x) + .l.y(x) = 0 with y(O) = 0 and a_\'(/_) - y'(I) = 0. Dc1cm1inc 1he
conditions for which rhere is a nega1ivc eigenvalue.

11. Write Hennite's differential equation in rhe fom1 of a Sturm-Liouvillc cqualion. lden1ify p(x), q(x), and r(x J.
12. Write Chebyshev·s equation in the fom1 of a Sturm-Liouville equal.ion. Identify p(x). q (x). and r(x).

13. Use the rcsull of Problem 11 10 write the orthogonality condi1ion for Hcrmi1e polynomials.
J4. Use the result of Problem 12 to write the orthogonaJity c.:ondition for Chcbyshev polynomials.

IS. Classify the following Sturm-Liouville problem, a, regular. singular. or periodic:

(a) (xy'(x)]' + ).xy(x) = 0 10. 00)


(b) [( 1 - x 2)y'(.r)]' +A)'(.\·)= 0 l-1. l]
(c) (I - x 2 )y''(x) - xy'(x) + ).y(x) = 0 1-1. I]

16. Determine rhe cigenvaJues and eigenfunclions of y"(x) + .i-. 2y(r) :: 0 wilh the boundary c.ondilions
y(-o) = y(a) and y'(-a) = _y'(a).

17. Show that 1he nonnalii.a1 ion i.:onslant of 1hc eigenfunction~ in Example 4 is 2r,y-/ (2/3,, - sin 2fin) I/ -.
18. Show that 1he eigenvalues and eigenfunctions of 1..hc cqu:ition y"'t r) + 1-..y(.r) = 0 wilh the boundary condi1ions
y(O) + y'(O) = 0 and y( I)= 0 are 1.0 = 0 with y0 (x) = x - I and >-,1 ;::;: fi~ wilh y,,(x) = fin cos /3,,x - sin fj,,x
for 11 ?_ I, where the /Jn art! the positive roots of tan x x,=
698 C·,,,1p1t·r 14 / Orthogo11,1I Polynomi,il, ,ind ~turm-Uouville Prol1lp111,

Example 3:
Expand f(x) = x over Ihc inIerval (0. I) in renns of the eigenfunclions i.n
Example 4 of the previous section.

SOLUTION: The eigenfunctions are y11 (x) = ~in {J,,x, where the /3,. are
the solution~ of tan /J,, = - f:1,,/5. Thus.
:x,

f(xJ =La,, sin /3,i-"


11==:I

Multiply by sin /3,w"< and intcgralc 10 obtain

a,,, f
lo
I
1
sin {3,,,x dx = 1
u
1

x sin /3,,,x dx

2f3m - in 2/Jm ) _ in /Jm - /Jm COS /3m _ _ 6 COS /3,,,


a,,, ( - .., -
4~ ~ 5~

where we have used tan 13m = - /Jm/5 in 1he last line. We huve. then.

!( ..,.) ~
' = _ 24 L co ~"' ~
.m,-,111:,
5 n=I 2/J,,. - Stn 2/Jm

You can de1cnni111e the ./3,, numericaJly. bur they arc also wcll-1abulated (set!,
for cxarnple. Table -4 .19 of Abramowir.z and Stcgun ). Figure 14. I3 ~hows
1

X partial sums of the ser,ies for j(x) consi'-ting of up 10 50 tenns.

Figure 14.13
The punial sums of Example J consisting
of 5 (dotted). 10 (~hort da!.hed)·. and 50
(solid) terms.

One important difference between a regular Sturm-Liouvillc problem and a


singular SLUrm-liouville problem is that the eigenvalues of a singular problem
may not be discrete. That is. there may be a continuous range of eigenvalues for
which there arc non-trivial solutions. When rhis occurs. we say that the problem
has a co11ti111wm spectmm of eigenvalues. l.n some cases, Lhe eigenvalues rnny
be discrete over some interval and continuous over another. There is no general
theory lirnt you can use to detenni ne the nature of the eigenvalue:- for any pan icular
case. ln those cases where the ,eigenvalue~ are discrete. you can show that the
eigenvalues arc real and that the eigenfunctions are orthogonal Furthermore. there
is a generalization of the theorem that we gave earlier in this section that guarantees
lhe convergence of the series expansion of a func1ion in terms of eigenfunctions.
The Legendre polynomials arc a good example of this case.
14 .5 Green's Function, 703

well-behaved given funclion, and where rhe boundary condi1ions arc

and (3)

We assume 1hat bo1h the solution ,r(.r) and the nonhomogcneous 1erm g(x) can
be expanded in terms of the cigenfunc1ions of L. which satisfy the cigcnvaJue
problem
(4)

where the </>,,(x) satisfy 1he boundary conditions in Equation 3. For notational con-
venience only, we assume thar lhe <l>n (.r) arc nom1alizcd and also write Equa1.1on I
as:;

.Cy(x) - 11r(x)y(x) = r(x)J(x) (5)

where r(x)f(.r) ,·., t:(x). We shall actually expand J(.r) = g(.r)/r(x) inslead of
g (x ). and wri1e
00

_r(x) = L an<f>,,(.r) (6)


" I

•:C

f(.r) = I: fntl>,,(.r) (7)


11=1

We kTiuw the J,. because they arc given by

( 8)

and we wish Lo determine the o,,. Substi1u1c Equal.ions 6 and 7 into Equu1ion 5 to
obt::iin
~ ~ ~

r(x) L A a,,<Pn(x) - µr(x) L an'J, (.r) ==


11 11 r(x) L JA>,,(x) (9)
11::] n=I

Multiply both sides of Equation 9 by cp1(x) and integrate over [a. h I using 1he
or1hogonality of 1hc </J,,(x) wi1h respect to the weighting function r(x) to tind 1har

I = I. 2 .... (10)

Ifµ :/= 1.. 1 for/== I. 2 •.. . . Lhen a formal solu1ion to Equation 5 is given by

00 OC f.
y(x) =L a,,cf>n(x) =L -,-"-<j),,(x) ( I I)
n=I """I 1.,, - /.L

If/ (x) is cominuous. then 1he series in Equation 11 converges point-wise to the
solu1ion of Equation 5.
If /J. is not equal to any of the eigenvalues. then 1he solu1ion lo the boundary
value pmhlern given by Equations I and 3 will be unique for any conlinuous
14.5 Green's Functions 705

appropriate eigenvalue problem is (Equation 4)

The normalized solution that sa1isli4.'s 1hc boundary condil'ions is

</>,,(x) = 2 112 sin n,r x


and the eigenvalues are given by >..,1 =11 2rr 2• Equation 13 gives
_ ,, ='">'°' sin mr:i:: sin 11 n z.
G(.\ ' ~) - ~ 2 .,
II ,r- - I
11=1

Example 2:
Use the Green ·s function in Example I 10 solve the equation
11
y (X) + y(.x) = X
with the boundary condit.ions y(O) = .r( I) = 0.
SOLUTION: According to Equation 15

y(x) = fo I G(.x. z.)(-z)d;:

= 2 I:----
cc sin11:rrx
2
,, ..., (nrr) - I •o
l 1

(-z) sin 11,rz dz.

=2 L
ix: .
sin 11rrx
(11;r) 2 - I mr
11=1

= 3_ f
iT n=l II 11 -
1
( - 1)"+ sin 11rrx
(1m)-J

This is perhaps a good time 10 point ouL 1hat the definition of a Green's
function varies from au1hor to author. the difference being a fac1or of± I. Not.ice
in Example 2 that x. the nonhomogencous term of the differential cqua1ion. is
represented in Equation 15 by -z. The difference can be !raced back 10 tJ1e
definition of£., in Equalion 2. Some authors define f., as (p(x)y (x)J' + q(x)y(x).
1

but then require that p(x) < 0 and r (x) < 0 for o regular Srurm-Liouvil\c problem.
There is no consensus on this is.,;ue. and if your work leads 10 working wirh
Green's functions, then you' II simply refer to your favorite source book on Green's
functions. Whichever conven1.ion you use, your final answers will always be
independen1 of that choice.

C gl
706 Ch.iplcr 14 / Onho •ona l Polynomial and ~lurm-Li uville Pr<11lle11l',

We can derive a differen1ial cqualion for G(.r. :) by fom1ally operating on the


x dependence of both sides of Equa1ion 15 wi1h 1he operator L - 1.u (x). Using
Equation 14 for the left side. we obtain

1>
[L - µr(x)]y(x) =g(x) =
1/I
{[L - µr(.r}]C(.r, .:))g(:)d.: ( 16)

Equation 16 says. however. rhat the 1cm1 in curly bracket~ in the integrand is a
Dirc:1c delta function (Section 3.6) because

[h
g(x) = I,, 8(x - z)g(z)dz
• ,1

Thus, we have

LG(x . .:) - µr(x)G(x . .:) =8(x - ;:) ( 17)

as a differential equation for the Green's function. C (x . .:).


Lei's see how lo use Equation I 7 by deriving the Green's function for the
equation

y"(x) + y(x) = 0
with boundary conditions y(O) = y( 1) = 0. (Sc.c Example I.) We obtuin Lhc equa-
tion for the Green·~ function by letting L =-d'! /dx 1 . µ. = I. and r(.r) = I. in
which case Equation I 7 becomes

G"(x, z) + G(x. z) = -ci(x - ::) ( 18)

where 1he primes denote differentiation with respecl to x. The boundary conditions
on G(x . .:) in this case are G(O, z) = 0 and G( 1. z) = 0. We work with 1he regions
0.:::: x < z and z < x .::: I separately. In each case. G"(x. ::) + G(x, ::) = 0. and 1he
solutions arc

_ _ { C1
G(.r ... ) -
sin X
.
+ C2 cos X 0~ x <,
CJ sm X + C4 cos X ;. - X S; I

=
We can apply 1he G(O, z) 0 boundary condition 10 the firsl solution and
C (I, z) = 0 to 1he second to obtain (Problem 20)

a sin .r O.sx<;:
G(x . .:)= { b. ( I) ( 19)
Sin X - ;: ,x :=: I

We sLill have two conslants, a and b, to determine. Wt: do this by integrating


Equation 18 from:: - e co z + f. where f ➔ 0.

(20)
14.5 Green' Fu n lions 707
Equation 20 implies thar one or both of the integrands on the left is discunrinuou/-i,
for olherwise borh integrab would equal zero as f """? 0. Since differentiation
produces discontinuiries whereas in1egra1ion smooths 1hem out. we shall assume
that G(x. ;:) is a continuous function of x .and that dG /dx is discontinuous al x = ::.
Therefore.

lim
,-0
j ·· G(x. :)dx = 0
•"-f

:-<
(21)

and

(22) :: r

Figure 14.15
Although G(x. :: ) is con1inuous at x =::.its first derivative has a jump discontinuity An illuwarion 1ha1 a Gr\..--en ·s function is u
there (Figure 14.15). r1m1inuous fum.-1ion of .r an<l 1hat i1; tirsl
<.knv:i\t\'i: h:i:- .:i tli,~·nnlinuiLy of - I 01
We can now u<;e Equations 21 and 22 to detennine tJ and I> in Equation 20.
X ::- :.
The continuity of C (x . .:) at x = :: gives

a :-..in:::= h sin(~ - I)

anu Equ.ilion .22 gives

I> cos(:: - I) - t1 cos -:. =- I


Solving 1hese equations for a and /J gives

sin(~ - I)
a=-----
sin t
and
sm -
b= - -
sin l

so finally we have

. _ _ _I_ { - ~in .\· ~in(:: - I ) O,:::x <:


G (.\. ~) - . . (23)
.'-in 1 - sin: sm(x - I) :<x.:::I

Nole that G (.r. :: ) is a symmetric: function of x and ::: that is. G (x. ::) -= C (::. x ).
(ff we allow for a complex. vector sp<1ce piclure in which the eigenfunctions
arc complex.. 1hen rhc symmetry condition is G(x. :) = C (;: . .r) (Problem 21).)
Figure 14.16 shows G (x. ~.) in Equation 23 ploued against .r for / = I. You can
see that G (x. ::) is continuous at .,· = :..
but that its slope is dis.continuous there
(Problem 22).
Because of the .symmetry property of G(x. :). we can also write Equation 23
'.\'
as
Figure 14.16
I - sin::: sin(x - I) 0:-=::.<x
G (.r. :::) =- - sin x sin(:: - I)
(24) The Green·., function given by b1u:11ion 23
sin I X <;: ::'.:: I plouc<l against .l' and ;:.

JI
Transfinite Numbers

What would you say if someone told you that 1hcrc is 1hc same number of even numbers a" there are integers?
Or 1ha1 there is the Sc,tme number of rat,i onal numbers as there are integers? As you might expect, it depends
upon whal we mean wtricn we say "!he same number as." In lhe lale 1800s. Georg Cantor investigated the
properties of infinite set,;, and he discovered that there,~ more than one type of infinity. He introduced the
concept of a transfinite number 10 enumernte what is called the cardinality of an infinite set. He. assigned
1he cardinality ~o (aJeph naught) to the se1 of integers ., and then he assigned cardinality N0 to the even
integers by the decep1ivcly simple proccs.s of matching 1he even numbers one-to-one wi1h -the in1egcliS hy the
arrangement

2 3 4 5 6 7
t :t t :t t i
2 4 6 8 10 12 14

Similarly. you can -.how 1ha1 1he se1 of sqmires of int.egcrs. Lhe se1 of odd numbers, or any infinite sequence
has the same cardinality as the sc1 of integers.
We said above 1hat the number of m1ional numbers. is 1be same as 1hc number of integers. Cantor was
able lo show by a:11 ingenious counting process tha1 the se1 of a1'1 ration.al numbers can be matched up one-
10-onc wiLh Lhc intcgen;. or Lhal the ~et of aU rational number-- has a cardinality of ~ 0 • We say thaJ the set of
aJI rational numbers is countable, or denumernble.
Even Ihough the ralional numbers arc dense. in 1he sense that Lherc i.~ always at least one ra1ional number
between any 1wo (lhe average of 1he two). there is sril\ lhe set of irrational numbers (both algebraic numbers
and transccndcn1al numbers) in any line segment. Numbers lha1 arc !.0lu1ions 10 polynomial equations with
integer coefficient<. are called algebraic numbers: if not, they are called 1ransccndental numbers (;r and e are
1ran5cendental numbers). Cantor was able 10 show !hat lhe cardinality of lhe set of all algebraic numbers
is ~ 0 • even though they arc much more general than rationnl numbers. He furthermore showed that the
cardinaliry of the scl of all transcendental numbers is not denumerable. and he assigned it a cardinality of ~l-
Thc cardinality of Lhe cont.inuum of real number-., which is also ~ 1• is due to the trans1Xndcn1al numbers.
In a sense. it is 1he t.ranscendental number~ that till up the continuum of reaJ numbers, Camor also showed
1hat the set of the continuum of 1Huplcs in any finit.e dimension is still ~ 1. and thnr !here arc sets with
cardinalities greater tha □ ~ 1• thus developing a hierarchy of cardinalir ies. One of the great unsolved problems
in m.111!l!111a1ks is to detcnnine if there is a l:ardinaliry between ~o and ~ 1.

711

al
Joseph Fourier (1768-1830). who gave us Fourier seri~. was born on March 21. 1768. in Aux.erre. France.
where his father was a tailor. He was orphaned when he was IO years old. Because of I.he calen1 that he
displayed in his early school years. he received financial support 10 finish his educa1ion . In 1780, he cnccn~d
the local Ecole Royalc Militairc, run by the Benedicrine order. where he soon discovered mnlhematics. In
1787. he entered rhe Benedictine abbey of Sl. Benoil-!\Ur-Loire wi1h 1he in1ention of becoming a priest. He
renliz.ed 1hat his true calling was mathematics. and he lert the abbey in 1789. returning 10 reach at his fonner
school. In 1793. he became involved in the French Revolution. joining the local Revolutionary Committee.
At tirst. Fourier was enamorcd with 1hc goals of the revolution but became disillusioned by the Reign of
Terror thnt followed. In July 1794. he was arrested as a result of a pwtest peech that he gave in Orleans.
bur he was freed when Napoleon came lo power. In 1795. he began teaching al rhe Ecole Polytechnique and
succeeded Lagrange as Chair of Analysis and Mechanics in 1797. Fourier was a gifted orator and was an
outstanding lecturer. In l 798. he. along with several other scien1is1s. joined Napoleon's army in che invasion
of Egypt as a scien1ific advisor. At the request of Napoleon. he served as. Prefect in Grenoble. Among his
du1jcs and accomplishment<; in Grenoble were the draining of the swamps in the are.i and the construction of
a highway between Grenoble and Turin. In 1822. he published his work. TJu:orie a11aly1ique de la cha/cur.
in which he intnxluced Fourier seric:-. In his later years. he became an "insufferable bore'' with his stories
aboul rhe glrnies of working with Napoleon and the wonderful work he was going to do. His years in Egypt
led him 10 believe 1hn1 de, crt heat was the ideal condition for health, "o he always won: hca,,y clo1he. and
lived in very hot rooms. He died of heart di sea e on Mny 16. 1830. in Paris.

di
CHAPTER 15
Fourier Series

This chap1er is devoted to a single topic. Fourier series. one of the most useful and
impor1ant tools of applied mathematics. At the 1um of1he 19th ccn!Ury. the French
mathematician and physicist Joseph Fourier analyzed the flow and 1he distribution
of energy as heat in solid bodies. Th.is work was summarized by Fourier in his
book, The Analytical Theory of Heat, one of 1he mos1 famous science books ever
published. Newton had previously proposed that the rate of the 1empcrarurc change
of a body is proponional to the difference in I.he 1emperature of the body and
tha1 of iL<; surroundings (New1on\ law of cooling), bul this observation appljed
only 10 temporal behavior. Fourier·s work addrc~sed the spatial distribution of the
temperature wilhin ::1 solid body and gave us Fourier's law of heat conduction,
which says that the flux of energy as heat lhroughoul a body is propon.ional to
lhe gradient of 1he temperalure. Because Fourier was interested in both lhe spa•
rial and 1he temporal distribulion of lhe temperature 1hroughou1 a solid body. he
considered functions of 1emperature of the form T;:::; T(x, y, z.. t), which leads
Lo partial differential equations, as we shall see in the next chapter. Upon solving
lhese equa1ions. 1--"ourier found it necessary to express temperature dislribut.ions as
infinite .">cries of sines and cosines of the fom1
00

j(.r) =~o + I)a,, cos11rrx + b 11 sin nrrx)


,,=I

where 1hc a 11 and bn depend upon f (x ). This type of series is now called a Fourier
seri,•s. It migh1 no1 be unexpected that :i Fourier series would converge to a func1ion
f (x) if f (x) is continuous over some interval. but the amazing thing about Fourier
series is that J (x) does nor even have to be continuous. We .sha11 see in this chapler
I.hat a Fourier series will converge to J (x) even i r J (.x) is disco11ti1111011s, such as
might occur initially across the boundary of a hot solid quenched in a cold liquid .
At the time, il was incredible that a scrie!. of conLinuous func1iom, could converge 10
a discontinuous function and Fouria', work was severely cri1icizcd. Ncvcnhe\css,
Fourier's work not only survived almo..,t two cenrurics of mathematical scrutiny.
bul has fostered seveml areas of modem mathematical re.search.
We introduce several variations of Fourier series in the tirsl two sections of
this chap1er and then discuss the nature of the convergence of Fourier series in
Section 3. In the last section, we show how to use Fourier series to solve linear 713
714 Chapter I / Fourier Series

nonhomogeneous ordinary dilTcrcnlial cqua1ions. Then. in the ncx.t chapter. we


shall show how Fourier series are used to solve pan.iaJ differential equations.

15 .1 Fourier Series as Eigenfunction Expansions


We saw in 1hc previous chapter that the normalized eigenfunctions of the regular
Stunn-Liouville problem

4>,,(x)
I/
+ J..,,<J),.(x) = 0 (I)

with 'Pn (0) = </J,/1) = 0 are

(2)
2
l/ . f/iTX
</>,,(x) = 1 sm - , - 11 = I. 2... . (2)

These eigenfunctions form a complete onhonom,al set over the interval [O, /]:

2 [' . nrrx . mrrxd t-


l losin - - sm - - x
1 1
= 0,, 111 (3)

Similarly. the nomialized eigenfunc1ions of the regular Stumi-Liouville prob-


lem

(4)

wilh <t>~(O) = cp~(J) = 0 are

(2)
2
l/ ltil' X
"' (x)
'l'n
= -I . cos - -
I
n = I. 2 . . .. (5)

and these also fonn a complele scl over the in1crval [0. /].
Now let's consider Lhe periodic S1urrn-Liouville problem

(6)

with the periodic boundary conditions

and (7)

The solution to Equation 6 is

(8)

Problem 9 ha_, you show that the periodic boundary condjLions given by Equa-
tions 7 give ).. 11 = 11 2rr 2 / / 2• where 11 = 0. I. 2 ..... The boundary conditions in this
case allow both a 11 and {J,, to be nonzero. so Lhcre are two set, of cigcnfunctionc:.
{sin ,m x / /} and (cos nrr x /I). corresponding to l,, = 11 2 rr 2 / / 2 . Furthennore. the

al
15. 1 Fourier S ri · d Ei 1 eniunc liu n E ·1 ;:i nsio ns 715

= 0, ,is allowed in !his case because the corresponding eigen-


zero eige nvalue, ), 0
function given by Equation 8 is 1he nontri,vial function <J, 0 (x) = a constant.
Although Equations 6 and 7 do nol constitute a regular S1unn-Liouville prob-
lem. we· II see that t he powerful cxparisio n rheorem of Sect ion 14.4 stil I applies in
this case. Thus. the expansion

J(x) = ?a0 + L.., '°'"'' (an cos - -


11,r x .
+ bn sm 11rrx)
-- (9)
- n=-:I I I

converge~ for a generous cla-. ... of functions. J (x). By ·'converge..,_-· we mean either
point-wise convergence if j(x) is conlinuous. or convergence in Ihe mean. We'll
sec below that the factor 2 in the a 0 term is included for convenience only.
We showed in Section 14.3 1ha1 the eigenvalues or a periodic Sturm-Liouville
problem are real and Lhnl the eigenfunctions are onhogonal. In particular. we have
(sec Problems I through 8)

£
1

• -I
_.
sin
11;rx .· m;rx d _
I
SIil
I
X -
f 1

-I
cos
11rrx
I
_ mrrx d _
co~
I
X -
i::
U11111
.f(.r)

f
r . 111u· mrrx d
Sin - - C0.4- - - X =0
-I I I

where II and m = l. 2 ..... It's ea!-y to show that the special case tf,0 (x) = cons rant
is onhogonal 10 all the sin nrrx// and cosnrrx//.
All !he terms in Equation 9 are periodic funct.ions of period 2/ (Figure I 5.1 ). Figure t 5.1
(Recall 1ha1 a function has period 2/ if f(x + 11) j (x) for all values of x.) = A pcri !tlil" funl-" ri ,11 with periml 2/.
Therefore the series in Equation 9 is pardcularly suited for the expansion of
functions of period '21. In fact. ir J (x) is defined on rhe inter\- al [ -/. /) and has
period 2/. then the series in Equation 9 is called the Fourier .\t'l"it•.,- or f (x) if the
coefficients are given hy

an= -I
/
1 1

-I
f(x) cos - -
11rrx
I
11 =0, I. 2 .... ( I I)

I>,. -1
/
J'
-1
j(X) . n;rx
SIil - -
I
n = I, 2.... (12)

1f(x)
These coefficients. l'.alled Fourier coefficients, are obtained using 1he 011hogonality
conditions in Equa1ion I0. The inclusion of the "2"' in the denominator or the a 0
term in Equation 9 allows us to use Equation 11 10 calculate a 0 • Otherwise. 1hc
integral for a 0 would have to be listed separately.

-/ X

Figure 15.2
Example 1: The periodic fon ct 011 Jd111,,;d by
Determine the Fourier ~erics of f(x) = 12 - x 2 for(-/. I) and f(x) = / Lr) = /2 - x 2 on the intcrv-JI 1-l. /Jund
.f (x + 21) outside rh is interval (Fig,,urc 15.2). /( x )= /(.\ . 2/) 11u1side 1.hi~ i111crval.
716 Chapter I / Fourier Seri

SOLUTION:

a
II
= -II 1 1

_,
., .,
(I- - x-) cos - -
nrr x
I
= -2I

'I J(x) The b11 :.i.re equal to zero bccau.<,c (1 2 - x 2 ) ~in(nrr x i I) is an odd function
of .\' . Thus.

00
2/ 2 4/ 2 (-1) 11 + 1 mr x
f(x)= -+ - I : : -- co
3 ;r2 n=l n2

-I X

Figure 15.3
The function /(xl =12 - x 1 t:,.nlid) for Figure 15.3 shows J (x) = (2 - x 2 and 1he fi rs1 few partial sums
t)f 1hc Fourier
(-I. I) together with l (solid). 2 (dashed). series rcprcSent.alion of J (x) over the interval -/ 10 /. Real iz.e that the Fourier
and 3 (dotted) panial sums of the series in
series represents not only f (x) in 1he interval -/ to/. bur its periodic extcn~inn as
Example I.
well (Figure 15.4).

f J(x)
Example 2:
Detem1ine the Fourier scrie:- of f(x) =x for[-/,/) and /(x + 2/) = j(x)
ourside this inrerval <Figure 15.5 ).

So LU TIO N : The Fourier coefficients ,ire


-/ X
a,,= -
I
11 1

-I
n;rx
f(x) cos -
I
- =0
Figure 15.4
The periodic ci1.1cn.,;ion of the func1ions
2 [sio (n:r. / /)
ploned in Figurt: 15.3.
bn = -/If'-1 f (.r) sin
.· 11rrx
-- = -
I I
~ .,
wrr - j /~
1 -
x cosi.nrrx//)]'
- -- --
11rc / l 0

(-iyr+12,
= n = I. 2 ....
11:r

The a11 == 0 because the integrand is an odd function of x. TI1c Fourier series:
or j(J: ) i~

Figure 15.S /(x)


= -(- It
= -21 I::
1
. n rr x
- - sm - -
The pcrioruc function defined by ;r n= I 11 /
/ (x) - .r on the interval I - / ./) and by
=
f (x) /(.r • 2/) outs ide 1hi" interval.
720 Chap1er 15 / Fourier Series

j(x)
All our examples so far have been on a symmetric interval -/ to/, Suppose
we want to expand f(x) = x 2 in (0. 21) with f(x + 21) = j(x) (sec Figure 15.14).
Problem 20 has you show that since f (x) is periodic with period 2/. the integrals
for the Fourier coefficients (Equations 11 and 12) can be written as

a,,= -I
I
J'
-I
nrrx
f(x) cos - dx
I
= -'1/+c
I -l+c
nrrx
j(x) cos - dx
I
( 13)

and
2/ X

Figure 15. 14 b,, = -I 1 1


j(x) sin
. nrr:c
-dx = -I 1/+c f .
nrrx
(x) sm - d x ( 14)
The function defined on the interval 10, 21)
I -I I I - l+r I
by /(x) = .r 2 and f(x • 21) = /(x).
where c is an arbitrary constant.
Therefore. the Fouriercoefficents of the expansion of _f(x) = x 2 in (0, 21) and
f(x) = /(x + 2/) are given by Equations 13 and 14 wilh c I, or by =

a,1 = -I
/ o
1 21
nrr x
f (x) cos --dx
l
= -II 1o
21
nrr x
x-., cos --d:r.
/
4/
=~
n-rr
2

J(x)
I {21 81 2
ao= t ]o .x2dx = 3
and

21 ,,.,
2/ X
~
Figure 15.15
b,, =
J 1 O
x 2 sin nrrx dx = - ~
J Trll
JJ = I. 2, ...
The function j(x) = x 2 over the inrerv.il Thus.
JO. 21) (black) ::ind the par1ial sums
consi, ting of 5 1cm1.~ (dotted color).

L -,.,I cos -nrrI-.Y - x)


2 2 00
10 1erms (long da.~hed color). and 50 4/ 4/ I . 11rr
1enns ( ·olid color) of che Fourier series f(x) =- +- (
- sm - -
representation of /(x) . 3 'JT JTW n I
11=el

Some partial sums of f (x) arc shown in Figure 15 . 15 for the interval (0. 21). and
in Figure 15.16 for the periodic extension of J(x) . The l/11 terms in the Fourier
/(:c) series representation off (x) are due 10 the discontinuity of the periodic extension
off (x) at x =
±nl with n = 0, I, 2.....

Example 5:
2/ X Expand the function

Figure 15.16
f _ { I - 1/2 ~ x < 1/2
The Fourier scri1..-s representation of/ (.r)
(x) - 0 1/2 S: .x < 3/2
in Figure 15. I5. showing lhar tbe Fourier
:1<:rii:s rcµrcscn1n1ion is a periodic function
of period 2J . with j(x) = f(x + 2). (See Figure 15.17.)
I 5. I Fourier Seri as Eigcn(unc1ion Expansions 721

SOLUTION: The period 21 = 2, so we use Equations 13 and 14 with I= I /(x}


and c = 1/2:
J 12
= -2
{.Jn =
l J/2

- I/2
f (x) cos 11rr X d.x = 1
- f /2
cos 11'1T X tl:c
II rr
1117
sin -
2
II# 0

"o = I
-I X
I/~
h,,=J -1/2
sin11rrxtlx::::O Figure 15.17
The func1ion /(x) from Example 5.
Thus.

~ sin(nrr /2)
f(x) = -I + -2 L.., ----cosn;rx
2 TT n=I II

I 2 ( - 1)"+1
oo
=- + -
2 rr
L - -- cos(2n -
2,i - I
n=I
l)nx

The partial sums of /(x) over the in1erval (-1/2. 3/2) arc planed in
Figure 15.18.

For simplicity only. most of the examples of Fourier series that we will discuss
in this chapter will involve functions defined over a symmetric interval (-I , /). but Figure 15.18
The partial sum~ of the Fourier serie ·
1he above Example shows that it i~ not at all necessary that f (x) be defined over rL-prei.cntat.i on of /(x) given in Ex ample 5
a symmetric interval. rnnsi sting of 5 (dotted). I O (sol id black).
Another fonn of a Fourier series is the complex Fourier series and I 00 ( o lid color) ttm\S.

,x

j(x) = L c,1/"·'T.l/l
11; -

Notice that the summation runs from -oo to oo. We can determine the c,i in
Equation 15 by realizing that {e 1n,--, .,"/ 11 is an orthogonal set over the interval -/
to I (Problem 24). Multiply Equation 15 by e-ikrr r/ l and integrate from-/ 10 / to
obtain

( 16}

Let's determine 1he complex Fourier series representation of f (:<) =x in


[-/,/]with f(x + 2/) f(x): =
Eig nun Ii n 723

9. Show thin the pcricxlic boundary condi1ions gi\'cn by Equation 7 give A,1 = n 2 ,r'1-; /"! for n = 0. I. 2 . . . . , :i.nd
tlrnt both cx 11 and~" in Equ~Lion 8 may be nonzero.
10. Show 1ha1 lhe a,, = 0 in Equation 11 if J(x) is an odd function and thal the /J., :-: 0 in Equation 12 if j(:r) is
an even fum:tiun.

It. Find the Fourier scric$ of J (.x) = ~ l -;r .r < 0


O ~ x<rr·

= x 2 for -rr · ; x < ;r.


12. Find the Fourier !-IC:l'it·s of J (.r)
. t I1c Founcr
IJ • Finu,I . ot· 1· (.r) = { .. n -rr ,:: X <. I)
. scncs
!.111 X O X < 7(

14. F;nd 1he F,,uner scr;cs of /(x) = f ! - 1r


-;r/2 _ x < 11/2
iT I'!.
X <

~ X -· iT
1r /2

15. F;nd 1he Four;er series of J (.<) = I X

2/ - .(
I ~ .r < 21

16. Find the Fourier ,°'t·ri~s or f (x) = .r..i for()::: .r < 2rr .
17. Find the Fourier series of _f(x) = {~ -2 ~x
O~x < 2 .
0

J8. Find the Fourier series off (x) c.:: I cos .rl for -,r ~ x < ;r.

19. Find the Fourier s.eries of j(x) = cos 2 x for -J'f ~x < ;r.

20. Verify Equations 13 and 14.


21. In this problem, wc·11 dcrivi:- the Fourier series for a funclion defined in lht:: gcner..11 intl!rval \a. I>).
·· ti h b +a
,Sh 1)W thur ,t we de nt! 11 .y x = - - + - - 11, then I c interva a -< .\· <
b - ll h . I b be
. come s -rr < 11 < rr.
2 2rr - -
and lhat the function f (x) = ff (h + o)/2 + (b - a)u/2rr I= F(u), h:.L'- a pc,riod 2.i"T. Now show that
"'-
F(u) = Oo2 "\'
~(a 11 cos 11u + b,, sin 1111) bcc<>mes
11:ccl

a
+ L0v [ a cos -
11 rr ( 2.\ - a · •b) . ,m(2x - a - b)]
.r
(x)= - 0 - - - -- b,, !-m - - -- -- , where
2 11::- I
11
b-n b- a

o,4 = - -
2 !,h _ n;rl~x - " - b)
j (x) cos - - - - - - d x n = 0. I. 2 . ..
b-a u b-n

2 ib f . ll!d2x - a - b)d
b,,= - - (x) sin -- -- - -- x 11 = I, 1 . . .
b-1.1 a - b-11

22. Show rhat 1hc gcncr.:.d fonnulas in the pl'l!vious problem rt!ducc to Equa1ions 11 and 12.
23. Use the general formulas in Problem 21 to derive 1he Fourier .serie.i- of

(a) /(x) = ,'T - x 0<X<7T (b) f(.r:)=cosx 0 _::: X<TT

24. Show that {ein t/


1\ is an orthogonal sel over the interval (-I. I;.
I 5.2 Sine and Cosint• Series 725
and J,(x)

bn = -I 11 J . nrr:cd
(x) sin - - x (2)
I -I I

show 1ha1 an= 0 if f(x) is an odd func1ion of x nnd bn = 0 if j(x) is an even


function. Thus. f (x) = /2 - x 2 in Ex.ample I of Section I ha.~ a cosine series nnd
-I (a) X
J(x) = x in Example 2 of Section I has a sine series.
Suppose now we have a function f(x) defined only on the interval (0.1). We
may ex.tend this function in two ways. One way is to ex1end f (x) so 1.ha1 ii is an
even function of x over I.he interval (-I. I). For example. if f (x) x in (0, /), =
I.hen the even cx1ension, fc, is defined by

X
This even extension is shown in Figure 15. 20a. We can also cx1end f (x) as an odd
func1ion of x, ] 0 (x). by defining

,.
f. (x) =
o {.X (b)

Figure 15.20
which is shown in Figure 15.20b.
(ii)The even c,ctcnsion or f(x) = x in 1hc
Generally, if a function f (x) defined on the interval (0, /) i.,; nei1her even nor inlcrval I O. /) plouoo :.igain,I s. (b) The
odd, then its even and odd extensions are given by o<ld extension of f(x) = x in 1he inll!rval
I 0 . /) plotted ili:?ain~t .r.
f (-x) -I.:'.:: x < 0
fe(x) ={ f (x) 0 .:'.:: x < I
(3)

and

- <0 Jc
= { - Jf(-x)
- / ,:'.:: X
/o(X) (4)
(x) 0~ x <I _,

For example. if /(x) = x2 - x on [O, /J, then

x2 +X
fc(x) == { x2 _ .t

a.nd
:c
-.x2 - ,\"
fo(X)= { .r2-x

Figure 15.21
These even and odd extensions arc plotted against .x in Figure 15.21. The (11) even and (b) odd t·.x1c11 ions or
/(x) dcfinoo by x 2 - ,t' on [0, /) plotted
ugninst x for 1-/.1).
728 Chapter l 5 / Four,er SE•rie\

of the sine :;cries arc given by

1 ( - l)nl2
bn =2
1 0
3
(x - .x) sin 11Ttx d.x = - -'.1- -
n· 1r

and the sine series is


00

f(x) =1
12
;r
L --,-
(-1)" .
,,-,
sin n;rx
11·
(9)

The coefficicnls of the co,irn.· llCrie.s arc given by

(a) a, 1 = 2 fo\.x 3
- x) cosmrx dx

S,,(x)
2 "2 12
112
rr 2 l(-I) + IJ+ ,, 4 JT..,[l - (-1) I
11
11 =foO

JI= 0
2
X
and the cosine series is

(b) f (x) = - -:4I- + 2


--:;
:r-
'L°" I'+ (-1)"2
•:-,0
, + - 6- [ 11- rr-11-1
I - ( - I)
II
I
}
cos n ,T x
n=I
Figure 15.27
(a) The Fourier scrie of the: odd extt·nsion = - -4I + ---:;-
2
"]f-
[( --:;
12
J(~
- I)

COS J1 X + -3 CO$ 2,r X
4
of the function defined 'in E:rnmple 2 arid
the pwtfo.l sums of Equation 9 wnr:iining
4
just two terms. {b) The Fourier series of
·the even e.x1e~, 111 of I.he function ddincd
+ ( -- 1
27rr
- ~) cos Jrrx
9
J63 COS 4rr X + · · ·] ( 10)

in Ex amp.le 2 and th<: partial swim I)(


Equation 10 consisting of 4 tenns tblackJ. The conrinui1y of fn{X) and its first dcriva1ives leads. 10 Fourier coefficicnl.~
!I tenns (dotted). and 16 terms (~1lid that dcc:ay a.~ l/11 3, \1/hcrca~ 1he continuity of /,Jr:) but the di$con1inuity in
color).
f~(x) leads to Fourier coeflicienLs that decay ru. I/ ,, 2. Some partial sums of
Equations 9 and 10 are plotted in Figure 15.27. It takes almost twenty terms
using the cosine serie..c; 10 achieve the .,amc accuracy a.<: just two tenns of 1hc
sine l>cric~. as. you may have ex peeled.

Example 3:
E)Cprcss 1he function

j(:c) = 7r - X O~X<JT

as a Fourier sine ~ries.


Figure 15.28
The odd extension of I.he fu □ction defined SOLUTION: To express j(x) as a Fourier sine series. we use the odd
in Example 3 plotted :igainst x. extension of J (x) (see Figure 15 .28):
730 ChJpt r IS i Fo uri r Ser.if'~

8. Express the func1ion f (x) = xJ - .r. 0 .:5 .r < 2. as both a Fourier sine series and cosine series.
9. Express the function f(x) = x 2• 0:: x < I, as a Fourier sine series.

10. Expn: ... s Lhe function J(x) = sin x. 0 ~ x < rr. as a Fourier cosine !->Cril'S.
11. Ex pre..,~ the func1ion J (x) =cos x. 0 ::: x < :;r. a.s a Fourier sine serie.s.
12. Express the function in Problem 10 as a Fourier sine series.
. f
13. Express Lhe runction (x) = { .,_ _.\x O<x
- < l as a Founer
. sine
. series.
.
l::5x < 2
14. Express Lhc fonc1ion in Problem 13 as a Fourier cosine series.
15. Express the function in E:<ample 3 as a Fourier cosine series.
16. Consider 1he funcrion J(.x) = sin x defined on the interval (0. Jr ). Would you ex peel 1he Fourier series of i1s
even or odd extension to converge more mpidly?

15.3 Convergence of Fourier Series

In the previous chapter. we learned abou1 the convergence of eigenfunc1ion expan-


sions a.:;sociated with Sturm-Liouville problems. Because the Fourier series thal
we have been discussing in Lhis chapter can be viewed as eigenfunction expnn-
sions, it shouldn't be surprising that lhe convergence propenies of Fourier series
are similar to 1hosc in 1he previous chapter. Fourier series are so imponanr in their
own righ1. however. that we shall resta1e Lhe convergence lheorcms here.
First. there's the property of conver!!cnce in the mean. Lei SN(x) be the nth
panial sum of Lhe Fourier expansion of j(.r):

S,v(x) =
ao
2
+ L N ( a,, cos - -
1
nrrx
+ bn sm -
. nrrx
1
- ) (l)
n=l

TI1e mean square error in approximming J (x) by S,v(x) is

(2)

But

f-1
I f(:c)SN(x)dx = f-I
I f(x) [ ~
-
2
+L
N
n=I
( a,, cos -
nrrx
I
+ b, 1 sm -
. nrrx ) ]
I
dx

Using Lhe definition of the a,1 and bn (Equations 11 and 12 of Section I). we have
1 I 2 N

1 -1
J (x)SN(x)dx = ~
?
+ I ~(a
L
2
+ b2)
- n=l
II II
(3)
734 Chap!er 15 / Fourier Se,ies

or

rr I I I I
-=I--+---+- +···
4 3 5 7 9

This was the firs! series involving rr ever discovered. It converges much
100 slowly to be useful for calculating the value of ,r. however, bur ii was a
wonderfully curious result when it was first discovered.

We frequently want 10 differentiate and inregnHe Fourier series rerm by rcrm.


This leads us to the following theorem:

Let f (:r:) be piecewise conti'nuous in [-l. f) and periodic wirh period 21.
then the Fo11rier serie.t off (x) may be imegrllled ren11 hy tcnn.

lntcrcsringly. Lhc above t.heorem holds even if the Fourier series for f(x) does
nor converge poinrwise 10 f(.r). The result of the term by term integration of
the Fourier series is 10 introduce a factor of I/ n into the resulting series. hence
enhancing its convergence . This is due to the fact Lhal integration is a smoorhing
process.
RccaJI from Chapter 2 1h111 if the function~ fun(x)) are continuous in (a, b).
00

then we can integrate the series S(x) = L un(x) term by tem1 if 1he series is
n=l
unifonnly convergent 10 S(x ) . The fact Lhal we can in1egrate n Fou.rier series term
by term makes you wonder about the uniform convergence of Fourier series. We
can slate the folJowing theorem:

If f(x) is piecewi.H' ,1moo1/J in rite i111en,al [a. b) and is periodic. rhen the
Fourier series off (x) co11verges 1mifonnly to f(x) in e1'ery closed imerval
conwininR 110 discontinuity.

As the above theorem says. 1he partial sums of a Fourier series cannot approach
j(x) unifonnly over any interval containing a point where f(x) is discontinuous .
This is njcely illustrared with the Fourier series for 1he step function

-l:5 x <0
f (x) = {- I
O~ x<

whose Fourier series is

4 ~ sin(211 - 1)71'x
f (x) = -rr L
2n - I
( 15)
n=I
1 5.3 Conve,gence o( Fourier Series 735

Figure 15.31 a shows the sum of the first ten terms of Equation 15 plotted aga..inst x. S 10 (x)
As more and more tenns are taken, the small oscillations along each horizontal
portion get smaller and smaller and eventually disappear. excepl for the two outer
ones of each portion closest to the discontinuities. (See Figure 15.31 b.) Even in
the limit of an infinite number of te1ms. there persists a small --overshoot." This
overshoot is called the Gihl>.\· phenomenon, and is due to the fact that you cannot
have uniform convergence at a point of disrnntinuity.
It so happens that the Gibbs phenomenon was not discovered by Gibbs. nor .r
by a mathematician, nor by a theorist. It was actually discovered by the Amer-
ican experimental physicist Albert Michelson, of Michelson-Morley experiment
fame. Michelson was a pioneer in interferometry and developed a harmonic ana-
lyzer, an opticaJ-mechanical device that rakes a periodic input and breaks it down
into iL-. individual Fourier componenL.::. He was able 10 calculate almos1 I00 terms
of Fourier series. and noticed that 1here is always a smaJI discrepancy around S~ 0 (x)
points of discontinuities. He brought this to the atLention of J. Willard Gibbs
(of thermodynamic and statistical mechanics fame). who analy1..ed the overshoot
mathematically and showed 1ha1 it approaches about I 0% of the height of the dis-
continuity (Problem 17). Thus. the Gibbs phenomenon was discovered empirically
by an experimentalist ex.traordinaire.
X

Example 3:
Use the Fourier series in Exarnplc 2 of Section I,
(b)

2/ ~ (-1)'1+[ . IIJTX Figure 15.31


X =- L - - - Sin -- ( I 6)
(a) The sum of the lir..t 10 1cm1s in
" n=I II /
Equation 15 plorted against x. (b) The
sum of the firs( 50 lcnn:- in Equation 15
to derive a Fourier seric~ for x 2 in the interval 10. /). plo11ed :1gain ·1 .r.

SOLUTION: ln1egr.i1e the abo\•e series rcnn by term from Oto .r.

2
(-w+ 1.r. nrru 1

-=-I:---
x
2
2(
rr o
oc
~m--d11
/ II
,t=I

.n1T.\")
=-'2rr2
_/2
L
( - l)nll (
~ ---
,:xi

2 11
l-rns--
f
11 • 1

The first summation here is of lhc type that we studied in Section 3.7 and i~
equal to 1r 2/ 12. TI1us. we have

(-1,n+1 . . mrx]
.,.2 = ---:;-
4/2 ["2
,r-
-12 - L --,-
oc•


cos - -
I
n=l

C
736 Chapter 15 / Four11:r · r1

Figure I 5.32 shows some pania.l sums of this serie,-.. Being a co,-.ine series.
it uctually represcnL-. J (x) = x 1 in[-/./). as well a~ it, pcrio<lic extension.

Let"s differentiate Lhe .series for x in Equation 16 instead of integrating it.


Differentiation gives

00
1.1. 2
-
""'(-nn+
~
1 cos nrrx
I
n=I
.\'.

This series doesn't even converge! Notice that differentiation of cos ,irr x / I or
Figure 15.32 sin n rr :r / I introduces a factor of 11 into the resulting series. so we can say that
=
The function /(xl x~ (black) and differentiation of a Fourier series is a threat to convergence. The relevant theorem
1he pan.i:u sums or the Fourier ~cric~ in regarding term by term differentiation of a Fourier series is
Example 3 corL~ist.ing of 4 (don~ -:olor). 8
(da..,hcd color), and 16 (solid color) terms.
Lt!t J(x) be a co111in11ousfimctio11 in 11,e interval[-/, ll with f(-1) = /(/).
and /er J'(x) be piecewise .!WIOtJlh in f-/, ii Then 1he Fnurier series of
J' (x) can be obtained by differenria1i11g rite Fourier series off (x) /em,
by um11. F1mhen11ore. 1/ie resul1ing !ieries for f' (x) converges ro f' (x)
1
where it is cominuous and to ( / (x+ >+ f' (x-) 1/2 lit /he poillfs 1\·here ii is
disco111i1111011s.

No1ice that lhe Lheorcm governing 1em1 by term differentiation of a Fourier


series is more demanding than forterm by tcnn intcgraLion. Recall from Chapter 2
that you can differentiate a series 1errn by term only if I.he resulting series is
uniformly convergent.
The reason 1hat we couldn't differenL.i.:1te the Fourier series for J (x) = .r in
Equation 16 is that the function f(x) = x represen1ed by lhe Fourier series is
1101 a conL.inuous function when iLS periodic ex1cnsion (which the Fourier series
represents) is considered. It is continuous in its fundamentnl interval [-/. I ), but
not as a periodic function.
We've observed several times that the rale of decay of Fourier coefficients
depends upon whether or not the periodic extension of J (x) is continuous. We
found lhat if the period le extension of f (x) is disconrinuous. then it.s Fourier
coeftlcients decay as 1/n. anc..l that they decay at least as fast a, 1/n 2 if the
periodic extension off (x) is continuous. We can formaJize these observations
in the following theorem :

If f(.x) and i,s firs/ k derivu1ives .w1isfy the Dirirhlet conditions nn the
interval [ -/./]and if the periodic e.rtensions off (x). J' (x) .... ' r~ ·
11(x)
are all contim,oux. 1hcn rhe Fourier coefficients of J (x) decay £11 leas/ a.r
rapidly as I/ n-"+ 1•
15.J Com·er~t•nce of Fourier Serie5 737

J
Example 4:
De1ermine lhc order of the Fourier coefficie-nls of /(x) = .r 2 on the interval
[ -/, /).

SOLUTION: Figure 15.33 shows f(x) =


x 2 in [-l. /1 and its periodic
extension. The funclion f (.r) is continuous, but its first derivative is not.
Therefore, k = I in the above theorem. and so we cxpccl the cocllicitnls in X

the Fourier series for f (x) to decay as l/11 2 (at leas!).


Figure 15.33
TI1c function /(.r) =
X~. -J :'.:: X < [,
and iL~ periodic extension defined by
f (:c + 21) /(.r).=
15.3 Pl'oblems

I. Derive Equation 4.
DO ..j

2. Use Pa.rseval"s equality and the Fourier series in Example I of Section I to show t.hal L _!_ = ~.
n:-J 11 90
4

:x. -I
3. Use Parseval's equalitv and I.he Fourier series in Example 3 of Section I to show that
,
L 1
,,..,, (211 - I) 4
= ~.
96

Hint : You need to use 1hc rcsul1 of Example I ( f, 11-


2 = rr 2/6).
n=l
oc ')

4. Use Parseval's equality and the Fourier series in Example 5 of Scc1ion I 10 show that L -_ 1
, = .:.=_
(2n - 1)-
n=I 8

S. Use Parseval's equality and the Fourier .~eries in Example 4 of Section to show that L (4n--. . I- I)·.,
oo

n=I

=; ~ I
(2,r - 1) 2 (211 + 1) 2
rr 2 -8
= -16. .

6• Th e Founer . o f j (x) = .\...l


. si.nc scnes - 4x. O ::S x < 2. 1s.
, ( (x) = 396 ~ ( - -I)"
L.., -
3
- - - . sc p arseva 1•s
. 111( X LJ
sin
.i 1t=--l 11 2
oc 6
. t""' I
cquahty lo show I.hat L
6 = -1T .
11=1 II 9J5
00
7. The Fourier sine ~erie~ of f(.r) = x(;rr ,
- x). 0::; x < ;r. 1s J(x) =-
8
L sin(.211
1T n=I
,
·• l)x
(2n - \) ..
. .
Use Parseval s
oo I (;
equality 10 show that L (211 - I) 6
= .!!_
960
n-:=I

8. The Fourier series of f (x) = x for (-/. /) and f (x + 21) = f (x) outside that intcrvaJ is (Example 2 of
00
(-1)"+ 1 . nrrx
. 21
SectJon I) /(x) = -
1T
L-
n= I
-·-sin--. Why does the senes equal zero when x
n I
.
= O?

9. Evaluate the series in E:,;amplc I of Secrion I at x = 0 and x =I. Do these answers make sense?
15 .4 r uri r Seri and Ordin,l · Oiff ..renri.i l Equation 739

15.4 Fouri r Seri es and Ordinary Differential Equations

Fouria series are frequently used lo solve linear nonhomogeneous differen1ial


cquarions with constam rnefficients. For example. consider the equation

y"(.t l - y<.x) = I Ix> 0:::.r~/ (I)

wi1h 1hc hounclary condirions y(O) = _d/) = 0. To :solve Equution by means of


Fourier ~criL·,. first ~x pre~~ f (x) a-: a Fourier series.

j(.r) = ?a0 + L~ (
(In co -,-
11 n x
+ l'>11 Sill
. 11 :r x
- ,-
) (2)
- n=I

Now write y(x) as a Fourier series.

+ L (A" cos - n,r.r)


'-I

\'(x)
. .
= -Ao
2
IIJT .\·
/
.
- + B" sin --
I
(3)
11=1

whl're the A,, and B,, are to be de1cnnined. We s\Jbslitute Equations 1 and 2 into
Equation I to obtain

-Ao + L"X. ;\ 11
(i - -, ~;r~)
- cos.- 1
lllT.r
- +8
(
I - --
,,~,.r-') . nrrx
SITI - -
2 , I " I~ I
it=I
( 4}
~
=? + L Ill)

- 11=1
( (/Jf cos - -
IIJT.X

I
+ l>TI
. /liT.\'
!(111 - -
I
)
Equation 4 gives 11s (Problem Ii

{I 1'!.
A,,= I-' - n,, , and 11=0, I •.•. (5)
,r-

with/ f; nrr. Rcali7.c that Equ:i1ions 5 :ire valid only for Equa1ion I. (Sec Problem 7
for a ... light gcncrnliza1ion.)
We ._,ill have the boundary conditions 10 consider. The coefficients a,, and
h,, arc dct~nnincd hy .f(x). and particularly how we c.:hoo~c 10 extend f (.n tii
the interval [-/. 0). Theo"= 0 if we extend Jex) such that ii is an cxld runction
over[-/./ I and the /J11 =i Q it' we extend f (x) such that ii is an even func1ion. The
=
boun<.Jary conditions. y(O) = y(I) 0. will he satisfied by y(.r) in Equation 3 if
all the A,, = 0: this wilrl be 11:!e case if all the 0 11 = 0. and so we choo!;e the odd
extension of /(x) 10 dl!,·clop its Fourier series in Equation 2. Thus.

(6)

satisfic::- Equation I and the boundary conditions y(O) = y(/) = 0.


744 Chaplt•r I ~ / Fourier Series

8. Use Fourier series ro solve the equn1ion y"(x) + {J 2_\·(x) = x with boundary condition~ y(O) = y( I)= 0. Him:
You need to use 1he rcsulr of Problem 7.
9. Solve 1hc equation in Problem 8 by the method of undetermined cocfficienl'-.
. to Prob Iem 9 1s
I 0 . The solu11on . y(x) x
= ---:; - sin
, _{Jx . Expand y(x) .in a Founer
. sine
. sencs
. an d compare your
{3- p- . m /3
resuh to the one obLaincd in Problem 8.
11. Derive Equation 8.
12. Show 1hat Equation 12 is a solution to Equation 7, when the x,. (/) salisfy Equation I I.
13. Show I.hat Eq uaLion 14 is a pan icu lnr sol u1i on of Equation I I .
14. Derive Equmion 15.
IS. Show thar the lhird term in Equarion 17 dominares if m = 4. y = \/ I0. k = 98. and w = I. Derive lhc equation
I.hat corresponds 10 Equarion 18.

16. Determine the steady periodic solution to y"(t) + 2y'(I) + 3y(r) = f(t), where j(I) = 1. -JT ~ r < 1r, and
j(l + 2JT) = f(I).
J7. Consider the general linear second-order nonhomogcneous differential equation wi1h con.~1an1 codficien1s
.x''(I) + a 1.r'(r) + ao-r(r) ::::: J (r). Show that if f(r) = Ae 11v', then the reaJ part of the steady periodic response
corresponds to the resultin~ .x 5 P(t) if/ (1) = A cos wt and the imaginary pan corresponds 10 x"P(t) if
f(l)=Bsinwt.

References

H .S. Carslaw. 1952. l111roduc1ion 10 1he Theor_v nf Fourier Serin and /111egrals. Dover
Publicarions
Harry F. Davis, 1963. Fourier Series and Onhogmwl F11nctio11s, Dover Public.at-ion-';
N.W. Gower and J.E. Baker. 1974, Fourier Serit:s. Chano and Windos, Lrd.
Fritz Oberhcninger. 1973, Fo11rier Expa11.1io111. Ac.ndemic Press
Murroy R. Spiegel, 1974. Fourier Ar,a/y.1i.\'. Schnum·s Ou1line Series. McGrnw-Hill
Georgi Tolstov. 1976, Fourier Series, Do\'cr Publications
CHAPTER 16
Partial Differential Equations

Many of the equalions of the sciences and engi necring describe how !.Ome physical
quantity such as 1cmpcrn1urc or an electricaJ potential vary with pm,ition and
time . This means thaL one or more spatial variables and time serve as independent
variables. If we let T = T (x. y, ~. r) be the temperature. an equal.ion that governs
how T(x. y. :, 1) varies with x. y, :.. and, is

or

where o- 2 is a consrant called the thcnnal diffusivity. This cqua1ion. called 1he
heat equal ion. is a partial differentiol equorion because the dependent variable. T.
occurs as partial deriva1ives. A study of partial differential equations is one of the
most imponant areas of applied ma1hcma1ics because so many physical processes
(such as heat llow) are fonnulated as partial differential equations.
The fidd of partial differential equations is very large. We are going lo discuss
only a few special pariial differential equations in this chap1er. but ones 1hat occur
in a wide range of applied problems. We shall concentrate on Laplace·s equation in
Section 2. lhe wave equation in Sections 3 and 4. the heat equation (also called the
diffusion equation) in Section 5. and 1hc Schrbdingcr equation in Sec lion 6. II lums
ou1 1ha1 these equations (and closely related versions of 1hcsc cqua1ions) comprise
a siz.ablc fraction of the panial differcn1ial equations that many scientists and
engineers encounter in practice. Funhermore. we shall see that they illustrate all
the various types of linear partial differential equations according to one important
clcissi iication of such equal ions that we shall discuss in Section 7. In this chapter.
we shall use the me1hod of separalion of variables to solve these equa1ions. and
in the next chapter. we shall solve these equations by wha1 are called integral
transform methods. 747
7 48 Ch.1pll·r 16 / Partial Dif{e-renllal Equations

16.1 Some E ·ample of Partial Differe ntial Equation


We derived lhe heat equation in Section 7.4 using the divergence theorem. As we
stated in the introduction. if we lcl T = T (x. y. z:. r) be the temperature al the point
(x. y, .::) at time t. then the he.at equation is

(I)

where a 2 is the lhermaJ diffusivity. Equation I is a second-order partiaJ differential


equal.ion because the order of the highest derivative is two. Fun.hermore. ii is a
linear partial differential equation because T and its various part.iaJ derivatives
appear only Lo lhe first power and there are no products of rerms involving T or
il.S derivatives. Many of the cl as.sic partiaJ differential equations that occur in the
physicaJ sciences and engineering are linear and second-order.

Example 1:
Show that

T(x. y. z. r) = Ae_3ai, sin x sin y sin z.


sati~fics the three-dimensional heat equation. Equation I.

SOLUTION:

",T
v - , -3a~r
= -.,Ae . x sin_\'
sin . .
sm;: = - 3A T
aT ,
-=-Ja·AT
a,
so

Let's consider I.he one-dimensionaJ version of Equation I:

J a2 r
a---=-
aT (2)
ax2 a,
Equation 2 might describe a long. thin bar of length/ where its temperature varies
only along its length x. Equation 2 is second-order with respect to the spatial
coordina1e and first order with respect to time. This suggesL.:; thar some sort of
general solution must be made 10 satisfy two boundary condition.-. and one initial
1b. I Som amplt.._ 01 Partial DitiPr ·niri,,I [quJ1iun 749

condition. For example, we migl11 specify the 1cmpcrarurc at I.he ends of the bar.
x = 0 and x = I.

and

and the initial 1emperat-urc profile of 1he bar

T(x, 0) = j(x)

ll turns out that 1hesc conditions are sufficient 10 speciry Lhe solu1ion T(x. I)
completely (and uniquely). We' II learn how to find such .soluLions in 1his and Lhe
next chapter.
Equa1ion I (with differeni symbols) describes the diffusion of one substance
through another. If c(x. y. z. t) deno1e.,; the concentration of a given substance
around the poinl (x. y. z). then c(x, y. :::. 1) is given (approximately) by (Prob-
lem I)

(3)

where D is 1he diffusion consranr. Equa1ion 3 is called Lhe diffusion equation. We


also derived the diffusion equation in Section 7.1.
Another irnportan1 partial differential equation is the wave equwirm. For
simplicity only. we shall derive rhe wave equation for the case of just one spa1ial
variable. The physical system reprcsenicd is 1hat of a vibrating sLring. Consider
a perfectly flexible homogeneous string s1retchcd 10 a uniform tension r bc1wcen
1wo point-.. Lcl 11 (x. t) be the displacement of lhc string from its horizon1al po~ition
Figure 16.1
(see Figure 16.1 ). The quantities r 1 and r 2 in Figure 16.1 are the tensions at the A vibrating ~lriog at an instant of time.
points P and Q on Lhc sLring. Both r 1 and r 2 are tangential to the curve of the The quan1i1ic..s shown in 1hc ligure nrc
siring. Assuming that there is only vertical motion of lhe siring. the horizontal used in 1hc dci:iv;ujon of lhc cli~~ical
one-dimensional w.ivc equ;_ni«>n.
components of Lhe tensions al all points along Lhc string rnwa be equal. Using the
notation in Figure 16.1. we have 1hat

r 1 cos a = r 2 cos fJ = r = conslanl (4)

There is a ncl force in the vertical direction thal causes the vcrtict.11 motion of
lhe string. Again, using the notation in Figure 16.1. we- find that the net vert.ical
force is

nc1 vertical force= r 2 sin /3 - r 1 sin a

By Newton's second law. this ne1 force is equal to the ma."s p6.x along i:he segmenL
P Q times 1he accelera1ion of the string. a2u/or 2 • Thus. we wri1e

(5)

gl
750 ChJpler 16 / Parti;il Oltteri·nt1.,I Equations

Dividing Equa1ion 5 by Equation 4 give$

a 11 2
tan /j - l.a.J1 a = -p lH
- --
r a, 2
(6)

But ran /3 and ran a arc 1hc slopes or the wrve of the string at x + 6x and at x,
respectively, and so

ifo ' 8/J


Ian a= -
Jx
= IJ .rCt) and tan fJ = -
ux =
. + 6x)
llr(X (7)

where u, denotes the panial derivative of u with respect to x. Substituting Equa-


tions 7 into Equation 6 gives

(8)

Expanding 11_Jr + 6x) in a Taylor series about 6x = 0 gives

11 v (x
a,, r
+ 6.x) = 11 + -ax· !>,.x + 0 I( 6.r t l
1
}
J •

and so Equa1ion 8 becomes

(9)

in the limi1 .6.x ·-• 0. where v = (r/p) 1-" 2 has uni1s of speed.
Equation 9 i, known a~ the cla~sical one-dimensional wave equation. Its
extension 10 more ~patial variables is given by

( I 0)

The two-dimensional equa1ion describes the vibrations of a membrane and 1he


three-dimensional equation describes the vibrational motion of an cla,1ic solid.

Example 2:
Show 1hal

11(.r. 1) = sinx sin tit


i~ a ~olution lo 1.hc one-dim~n~ional wave t!quation.

SOLUTION:

IIT,=-/1 nnu
16.1 Som amp!.._ of Part ial Diff r nlla l fq11 .J tio1 ·~ 751

.s.o 11(.r. r) =~in.\ ,in vl is a solution to Equ:irion 9. Nore 1hat the so lurion to
1he heat equariun in Example I decayed exponentially wi1h time. while the
above solution to 1he wave equ.11ion oscillates in 1ime.

Being ,cLond-order in both x and , . it may not be surprising 10 learn 1ha1


Equation 9 requires two boundary condi1ions and 1wo initial conditions to yield a
unique solution. For example. 1bese conditions might be

11(0.,)=0 11(/.1)=0
11(x.O)=f(x) 11,(x.O)=.i:(x)

The two buundal)' condiIions say that 1he displacement is zero at Ihe two ends
of the string and the initial conditions give the initial uisplacement and 1he ini1ial
velocity of 1he ~rring.
As. Example 2 .-.uggcsL'-, the 1ime dependence of the wave equation is oscilla-
tory. If we substiIu1e

11 (.r. I) = X (., )11'°''''


into Equation 10. we obtain

V~ X (x) <,/ X (x) =0 (I I)

Equation 11 occurs in a number of physical problems and is called rhc He/111/10/1:


eq11otio11.
Another panial di ffcrcntial cq11,1Lion 1ha1 we sluill discuss is Lll(llan· \ ('q11a-
1io11.

If T in Equa1ion I is independenr of time. then Equation reduces 10 Lapl:.1cc's


equation. which 111 thi~ case dc~cribe~ the equilibrium or the steady-sr:itc tcmper-
a1ure dis1rib111ion in a hody.
Lupbcc·s equation aJso governs the clcctrostaric potential in a charge-free
region of ,pace, rr there is a charge den~i ry pAx. y. :) within the region, 1hcn the
elcctros1atic poLcnti~I ii;; !!iven by P11iH 011 ·s eqw.uirm.

(13)

where '=ii is the pem1ittivity of free space. Poisson·s equa1iun is a nunhomogencous


version of Laplace·s equal.ion. We derived Poisson·s equation in Scc1ion 7.4 from
Gauss·.-; IJ\v of clcc1rosta1ics.
752

bample 3:
Show that

(X. _\'. Z) =/= (0, 0. 0)

satisfies Laplacc's equation.

SOLUTION:

with analogous equat..ioils for <Pyy and <P::· Thus,

= (2x-- -
1 1 1
)'"' - ;:-) + (2y- - 1 1 'I
~ - .: ~) + (2 .:-1 - x-1 - J
y-) =0
(x ! + y:? + z2 f•/..

hample 4:
Show thal ir </> is i.-phericalJy symmetric (in other words. depends only upon
Lhc radial coordinate r ). then Laplacc's equation becomes

Solve l11is equation 10 show that

C
<f>(r)=- r-:JO
r

where c is a constant.

SOLUTION: Referring to Table 8.3. we see lhal

If,:/> depends only upon r. then


1b. 1 Some b:.,mples of Partial Differential Equation-. 753

and 'v 2q, = 0 can be written as


!!__ (r2dtf>) =0
dr dr

Integrating once gives dtJ,/dr = a/ r 2• and integrating once again gives


a
1/,(r)=--+b
r

where n and b arc const.anL,;. If we require lhat <t>(r) -. 0 as r -. oo. then


we find that h = 0. Replacing the canst.am -a by another constant c gives
,:/) (r) = c/ r (Coulomb's law).

Before we conclude this section, Lhere is one other partial differential equation
that we must discuss. The central equation of non-relativistic quantum mechanics is
1he (time-independent) Schrodinger equation. which for a single particle of mass m
in a potential V(x, y. z) is

ft2 2
--V
2m
t/1 + Vt/J = Et/I ( 14)

where Ii;:: J,/2Jt and h is lhe Planck constant. The dependent variable in Equa-
tion 14, 1/1 (x, y. z), is the wave function of 1he pan-icle. The wave function has the
physical interpretation that 1/1.,.(x. y. z.)'t/l(x, y. z.)dxdydz. is the probability that
rhe pan-icle will be observed to be in the volume element dxdydz. at the point
(x. y. z:). According to quantum mechanics. this probabilistic interpretation is the
most complete description possible.
The Schrodinger equation cannot be derived from more fundamentaJ prin-
ciple!-., any more than Newton's classical equations of motion can. Each can be
deduced from other basically equivaJent approaches. but each one is essenLiaJly a
postulate that has withstood intensive and long scrutjny.
If we define the so-called Hamiltonian operator by

fj 2 -,
'.)-{ = - - V - + V(x. y. z)
2111

we can write Equation 14 as an eigenvalue problem

(16)

The eigenvalues. I£,,}. have the physical interpretation that they are the allowed
energies 1.ha1 the par1icle can have. Equation 16 is generally a partial differential
equation. bul it is ju~l an ordinary differential equation in one dimension. Prob--
lcm 14 works through the mos1 basic non-trivial quantum-mechanical problem. a
particle in a one-dimensional box .
754 Ch.1p:1·r 16 / PJr1ia l Di ( renliJI fq1u!i111h

16.1 Problems
I. Use the continuity equation (Equation 7.1.11) and Fic-k's law (.J =- D gnid c) 10 derive Equa1iun :I.

2. Verify that 11(.r. y. 1) = sin(x /2 1


~)~in(_\/ 2 112 ) sin vr is u solution 10 1he two-dimem,ional wave equation.

3. Vcrify 1.ha1 11(x. 1) = f (_\ 1- 1.11 ). where [(:) is a diffen:ntiablc function. is a solution 10 the one-dimen~io11al
wave equation.

4. Show that c·(x. 1) = ,-,


2 exp( - .r 2/ 4 D1) is a solution to 1he one-dimcn"ionol diffusion equal ion. Wh:it init.ial

condition dm;s 1hi'i solu1ion corrc~pond to?

5. Verify thal c(r. r) = ,-HJ. exp(-r~ / ➔ Dr) is a solution 10 the diffu~ion equa1ion in spherical conn.Jinates.
1 . ik
. w,I. h t he contrnuI1y
6. S 1a111ng . . equa110n . ( Equa11on
. 7. I . I 1). show 1h,11 ik
-;- = /) :----,
i:1 c µ-:--- represents a sy.i.tcm
i-Jt nxp dx
that i~ under~oi ng di ffus ion ant.I moving wi1h a coustant drift vtloci Iy ft. This equatinn cnu Id rcprc.,cnI I he
diffu.,io11 of a large charged pa.nick such a, a protein in a uniform electric tield.

7. Verify that c(x. I)= 1 1i 2 c.:xpl -(x - x 0 + f)I)~ /4 Dr I is a solution l(1 the cqua1ion in the prcviou.-. prohh:m,
What ini1ial condition docs 1hi.~ solu1ion correspond to?

8. Max.wdl's equations for a nonconJ11c1inf! medium with no free char~c.:~ c.·in be wrinc.:n as JivE = 0.
aB . I ilE . .
curl E + - = 0. d1vB = 0. and curl B - , -:-- = 0. where E anJ n arc the ch~c1m: field and Ihc macne11c
cl/ c· or ~
induction. respectively. Show that if C rcpre.'-cnts any of the components of E and R, then ( satsifies
~
2
( = --:c-I;- :.=,.
~
-1~

a,-
Hi111: u~ 1he vector identiry curl curl ,, = gr.id div v - •;i:!.v.

aT
9. Show that I.he subst..itution T(x, I) = if) (x. f)e- 111
• where h is a constant. reduces the equation
o, =
,J~T . . .
a·--, - hT 10 the one-d1men-.1nnal heal equation.
£1:c·
10. Show 1hat the kineric energy and the potential energy of a unifonn vibratill!! ,1rint! ar'l~ given by K =
!!.
2 Jo
((0_~)
a,
2
dxandV=!.
2
f'( ~
Jo cJx
1
')
2
dx.

11. If all the points of a uniform s1J1ng experience a frictional force proportional to the velocity. show 1hat 1hc
I ;i2,, au c)!u
wave cquu1ion becomes , -;---,
tJ- i.1,-
}' -:-- - ::---::;- = 0. where y is the proportional frictional coefficient.
rl t rJx-
12. Show that the substitution 11 (x. r) = ((.r, -.in t11 reduces the one-dimL·n,ion.il wave equation 10 ,m ordin::try
di lli:rcnt ial equation in x. Docs thi, sub._1 i 101 ion mah· ~L·nse phy, kally?

13. Use the divergence theorem to derive the hear equation if a hear source is present within the ~!!i1111 of interest.
Let F (.r. y. : . r) be the rule of energy a.--. ht·at generated rx·r unit vo 1umc.

14. The lime-independent SchriiJin,;er equa1ion for a free: particle constrnincd 10 I.he one-dimensional region Oto
2
a is - fi"2 " ~ = E 1/J. Because ,t; •1/r dx represents the probability that 1he partjcle will be locnted within the
2111 dx-
intervul (x. x + dx ). we require 1hn1 t/J (x)
, =0 for x .::: 0 and x ~ ". Show that the energy of the part ick is

1111t111liZt'd. uud in pan icu1lnr. 1ha1 'E11 = :i-h., , where


8m a -
11 = I. 2 .... and f, = h/2r...
756 Chapter 16 / Panial Oi({erenlial E:qu,11ions

boundary, then the potential within the region enclosed by the boundary is uniquely
detem1ined.
The other commonly occurring boundary condition is the specification of the
normal derivative of" on the boundary, or

with ou = f on B (4)
on
where rfo/011 is lhe normal derivative of u, taken in the outward direction. Thi~
type of boundary condition is called a NeumaJJn boundary condition and Equa-
1ion 4 cons1itutes what is called a Ne11maim problem. You might remember from
electrost.aLics tha1 if you specify the charge densi1y (which is dfreclly related to
nu/ih,) on a boundary, Lhen I.he polenria.l is determined to within an additive con-
stant.
The proof of the uniqueness of the solutions to 1he two-dimensional Di rich le!
problem and the two-<limensional Neumann problem (to within an additive con-
s1ant) are g1ven in Problems 24 through 27. The Di rich let problem will always have
a solution (existence theorem). but the Neumann problem may not. Problem 28
helps you show tha1 Lhe Neumann problem has a solution only if

J ~u ds = J: J ds =0 (5)
rB on rB
where the integr.il is nlong the boundary. Equation 5 is called u compatibility
co11ditior1. We can give a simple physical inrerpreta1ion of 1his condi1ion . Suppose
that II represents I.he steady-state temperarure in .some region. Then a" /011 f =
represenls Lhe flux of energy a, heat across Lhe boundary. Equation 5 says Lhat
the net flux across I.he boundary musl be zero in order 10 maintain a s1eady-1aa1e
tempera1ure.
Now tha1 we've discussed some general propenies, let's solve Laplacc·s
equation for a few special cases. Consider 1he Dirichle1 problem for a rectangle:

0< X < 0
(6)
O<y<b
)'
T(O. y) =0 T(o.y)=O
(7)
T = j(x) T(x, 0) == 0 Tfx. b) = J (x)
b
This problem might correspond 10 a determination of I.he s1eady-s1ate temperarure
T=O T=O disLribution in a rectangular plate wiLh the temperuture prescribed on each of its
edges (Figure 16.2).
Laplace's equation, a-. well as many other partjal differen1inl equations that
T=O a X
arise in physics, can be solved by a method called separation of variable.,·. The
key step in the method of separation of variables is 10 assume that T(x. y) in this
Figure 16.2 case factors into a function of x only times a function of _v only, or lhc.1l
A summary of 1he boundary condition:-
for Equations 6 and 7.
T (x. y) = X (x) Y (y) (8)
16.2 Lapl;i ' Equation 757
If we subslitutc Equation 8 into Equalion 6. we obt ..,in

Now divide by T(x. y) = X(x)Y(y) and obtain


d2X dY 2
(9)
X(x) d:c 1 =- Y(y) dy 2

The left ~ide of Equation 9 is a function of x only anJ the right side is a func1ion
of y only. Because x and y are independent variables. each side of Equation 9
can be varied independen1ly. The only way for 1hc equality of the two sides of
Equation 9 10 be preserved under any variation of x and y rs that each side be
equal 10 a con scant. If we let 1h[s con slant be k. then we can write

X"(x)-kX(x)=O and Y"(y) +kY(y) =0

where k is called the separation conswm and will be determined by the boundary
conditions, Equations 7. The boundary condi1ions can be expressed in terms of
X(x) and Y(y) by

X(0)=X(a)=0. Y(O)=O. T(x.b)=f<x) ( 11)

Notice that the me1hod of separation of variables has given us rwo ordinary
differenrial equations. one for each independent variable.
We do not know at this point if the value of k is positive. zero. or negative.
Let's first assume 1ha1 k = 0. rn this case. Equa1ion~ 10 give X (.x) = a 1x + fii and
Y (y) = a 2y + {32,. and we find that 1the only wa,y to sa1isfy X (0) = X (a) = 0 is for
a 1 = /3 1 = 0, which gives us a trivial solution. Problem I asks you to show that the
same situation occurs if k is positive. This leaves only the possibility that k < 0.
To emphasi:ze that we consider/.: to be negative. we write it as -).. 2 (with ). real).
so thar Equations IO become

and (l 2)

Solving rhe tirsc of Equations 12 for X (x) and applying the boundary conditions
give ).. 11 -= nrr /a and

. Tlrt.\"
X,i(.l"")=sm -·· n = I, 2, ... ( 13)
a

Solving the second of Equal ions 12 for Y (y) gives

nrry nny
Y,,(_r) = c 1 cosh - - + c 2 sinh - - (14)
a a
758 Chap1e, I r, I P,1r1i,,I Di ffer ntial quarion'.-

The houndary condition Y (0) = 0 forces c 1 10 be equal to z.ero. So far. then. we


have
JIJTX l'IJT\'
T,.(x. y) = sin - - sinh --·
{/ {l
11 = I. 2.. ..
Because Laplace ·s equation is linear. the complete solution 10 Equation 6 is given
by a superposilion of 1he T11 (x. \').

'XI
II JT \'
T(x. y) = "~ c,, sin. -
11 X .
- srnh - - ·
}T
(15)
""" I o <I

Equal ion I5 satisfies Equalion 6 and the three homogeneous boundary conJit ions.
We can determine all the c,, in Equation 15 by using the founh boundary condition.
=
Letting y h in Equation 15, we have

nrrb . 11JTX
= L c,, smh - - srn - -
,x, .
T(x, I>)= f (x) O<x<a
n"' t a {I

Using the onhogonality of the {sin 117rx./a) ewer the interval (()_ a) gives

.
en srnh - -
111Tb
= -21[/ .f(x) SIii
. --d:r
1/lT.'f
( 16)
(/ £1() £1

If J(x) = T0 x(n - x). for example. then

c11
.
sanh - -
mrl>
(I
= -2T0
(I
lu
.o
.r(a - .r) sin
. nrrx
--dx
{l

amJ so the solur..ion is

. II JT \'

.
<>7· 'I
) o oa -
7 (x. v = - -
L -I --(--
N
- 1·)n l -smh- -__-. sin - -
[
a . . 11 rr x
( 17)
. rr 3 11-" . 11rrb a
11=1 smh - -
a
y
Figure 16.J shows the stea<..ly-s1a1e tempernture given by Equation 17 ploucd
Figure 16.3 against x and y.
Equation 17 plotted a_gainst x and y.

Example 1:
Determine the s1eady-state temperature dis1ribution in a rectangular plate
that is insulated J long the edges x = 0 and x = o. so thar

T, (0. yl = T,.((I. Yi= 0


760 Chapter 16 / P;ir1ial Oiffen~n1ia I Equations

lt"s easy 10 verify 1hat T(.r. y) sati fies Laplacc·s equation and all lhc
boundary conditio □ s. Figure 16.5 shows T(x. y) plotted over the rectangular
region.

y Example 2:
The velociry profile 11 (x. y) for the steady flow of a viscous fluid through a
Figure 16.5 rectangular conduit is governed by the equation
lne. teady-statc temperature di~tribution
for the system dc~ribcd in Example I.

where r, is the coefficient of viscosity and y is a constant pressure head.


y Solve this equation with rhe "non-slip" boundary conditions

11(-a. y) = u(a. y) = u(x. b) = u(x. -b) =0


f
b The gcomt!lr,, is shown in Figure 16.6.
I - SOLUTION: As with ordinary differential equations, th~ solution to 1he
t X
ubove equation is given by
b
I

where u,Jr. y) is the complementary solution (that is. the solution to 1he
homogeneous equation). and "P(x. y) is any panicular .<-olution 10 the
Figure 16.6 homogeneous cquati()n. Certainly
The coordinates for a ret:tangular cro ·s-
section describing unjdin.'Ctional flow y ,
in lhe n:crangular conduit described in lln(X. \')
•. .
= -211.\'~ + rl .\' + C1-
Example 2.
is a panicular solution.
The boundary conditions thal we shall use are the so-called non-slip
boundary condi1ions. which say 1hat the velocity of the fluid at a boundary
is zero. Applying the non-slip boundary condit..ions 10 "r/r. y) al _I' = ±bin
Figure 16.6 gives us

We use separation of variables 10 find the complemenrary solution . Leuing


u(x. y)= X (x) Y(_r), we find that
and

where >.. 1 t.'- the separation con~1an1. The solul ion to the equation for Y (y) is

Y(y) = Cr cos >.._r + C2 sin AY


16.2 LaplacE•'s Equa1ion 761

We've taken -a~ .r ~ a and -b ~ y ::; b. Physically. we expect the vclocicy


profile 10 be symmetric in x and y. so we set c2 = 0 in Y(y). The no-slip
boundary condicion. u(x, -b) = u(x. b) = 0. gives us c 1 cos >.b = 0. or
). 11 = (211 - l)tr J2b. with 11 = I. 2 . .. . . Thus. we have

(2n - l),rv
Y,1 ( v)
.
= cos 2b
· n = I. 2 ....
The solution for X (x) is given by c.1 cosh .l..x + r·,1 sinh J..x. We'll sc1 <'~ =0
so thal X (x) is an even func1ion of x. and so we have

(2n - I)rrx
X11 (x) = cosh - - - - n = I. 2 . .. .
2b
The complcmcn1ary solurion is a supcfT)o-;i1ion of X11 (x) Y11 (y).

~ (2n - l);r x (2n - l)JT \'


uc(x. \")
.
= L,
'"",·,, cosh - -2b- - coo; 2b
·
II== I

and the 101a\ s.olution is given hy

11(.r. v)
y
= -(y- ..,
-
.,
b- ) +
Lc
:x.,
(2.,1 -
cosh - - - - cos
l)rrx (2n - l)JT\'
·
. 'J .
_,, 11 = 1 " "b
- 2h

We can determine the c11 by requiring that u(x, y) = 0 at x =±a . or from


00
y , (2n - l)JTa (211 - l)rr\'
u(±u. v)
.
= -(Y
" .
2
-b-) + '""c cosh - - - - cos
L, " )J 2b
· =0
-Y/ 11=! _,

so that
(211 -
c cosh - - - -
1),rn
= b-I f.,, v -, ,
-(b- - v-) cos - - - - d \ '
(2n - l)rrr
" 21, _,, 2,, . 2b .

The fin.ii sulurion is

(2n - l)rrx
16yh2 C)...• (-l)n+I cosh 2b (211 - l)rrr
cos .
,pr 3 L
11 -' 1
(2 11 - 1) 3 (2n - l)na
cash - - - - -
2b
21, .\

The velocity contours are shown in Figure 16.7.

Figure 16.7
Wlocity contou~ for the steady
To keep things fairly simple. we chose three of the boundary condiLions to unidirectional flow through a rccrangular
rnnduit for u =2.h.
be equal to zero in all the above examples. Problems 6 Lhrough IO show how to
handle the more general c.:ise where
16.2 Laplace's Equalion 763

have

1/]TQ) = -
c,,lo ( - -
1
21
/ 0
1
T(a. ::) sin
. 11rr-
- - rl:.
/

2Jo
= --
1
lot .
:.(I - :) sin - -d:
IIJT:

47i1 /2 n
=- , - 11-(-1) I II = [. 2....
w ' rr.1 Figure 16.8
The steady-stale tcmpcr,11urc disLriburion
The final solu11on is T(r. z) given in Ex:unple J for r =" am.l
r=o/2.
4Toi2~fl-( - 1)"]/0 (wrrr/a). nrr:
T(r. :) = -- ~ - - - - -- - - ~1n - -
,rJ ,1=l 11 3 /o(iur//a) I

TI1is re~uh is ploued in Figure 16.8.

A standard appltcation of Laplace ·s equation in spherical coordinates is the E


problem of a conduc1ing sphere in a uniform electric field (Figure 16.9). The sphere
is centercd at rhe origin of a spherical coordinate system and we take the ;: axis lo
be in the direc1ion of 1he external field. One of rhe boundary conditions is tha1 the
poten1ial is constanL over the surface of the sphere. so it is naturnl 10 use spherical V

coordinates. Laplacc·s equation in spherical coordinates is (Table 8.3)

, I o ( ,')-OIi) I o ( . 011 .) )
2
0 11
'v-11=--:-- - + - - - - s1n0---:- + - - - =0(18)
r 2 ifr nr 2 r sin 1 2
f) iW i:lH r'.! sin 0 J,t,

Since we have taken the : axis of our spherical coordinate system to lie ;.ilong
the direction of the electric field in Figure 16.9. 11 will be independent of 1/J. and
Equation 18 becomes
Figure 16.9

( r 2 -i}u) + - I- - -
TI1c geometry ~socia1cd wi1h a
a
I -
- ii (..
sm/:J- a11) = O ( 19) conduc1ing ~phere in a unifonn cll!clric
,- 2 i.}r iJr 2 r sin 8 (10 ae field.

Two bourn.Jary conditions are

11(0.0)=0 and 11(r. 0)-,,. -£0 : = -E0 r co!-.0 as r ~ oo (20)

e . ays that the surface of the conducting sphere is an equipotential


The first of 1he
surface (we take the potential to be zero) and the second simply gives 1he fom1 of
u(r. 0) far away from the disturhing influence of 1he sphere centered at the origin
of 1he coordina1e system. (Recall 1ha1 E = -n11/oz.)
We ~eek a solution of the fonn u(r. 8) = R(r)(~(0). This substitution leads to

-d ( ,')dR(r)) .
- - - - 11.R(r) =0 (2 I)
dr dr .
764 Chapler 16 / Panial Diht•rl'nti.il Equations

and

-d ( Sin
. 0d<➔
-(0))
- . r,n ,-,
+ A sin n ( () ) =O (22)
d0 d0

where>. is the scpara1ion cons1anL The equation in R(r) is

r 2 R"(r) + 2r R' (r) - ).R(r) =0 (23)

This is an Euler equation (Section I 1.5). We substitute R(r) =r 111


and find 1.ha1
m(m - I)+ 2m - ). = 0. so tbat

(24)

where m I and m 2 arc the roots of m 2 + m - A= 0. where>- is ye1 to be determined.


The equation for 8(0) may no1 look at all recognizable. bu1 if we lei x = cos 0
and 0(0) = P(x). then Equa1ion 22 becomes (Problem 20)

(25)

This equal.ion is Legendre'!- equation. which is the sole topic of Section 12.3. Le-
gendre ·s equation often arises as lhe separated equat'ion in 0 in problems involving
spherical coordinaLe<;. We learned in Section 12.3 that Equation 25 has a finite
solution at .r = I (al fJ = 0) only if>..= 11 (11 + I) where 11 = 0. I, 2 .... and that the
solutions in this case are the Legendre polynomials. Thus. we write Equation 25 as

11 = 0, I, 2, . . . (26)

Therefore. the solution to Equation 22 becomes

(27)

where

I .,
0-,(8)
-
= -2 (3 cos- 0 - I)

Given that A= n(n + I), the two roots of Lhe equulion ,,,~ + 111 - )._ = 0 are
111 1 = 11 and m 2 = -(n + I). and so Equation 24 becomes
11=0.l.2 .... (28)

The general solu1ion to Equation 19 is a superposition of the R,,(r)(-),,(0)

:c ,:,:;;
u(r, 0) = L a irn P,,(cos 0) + L b,,r-(n+ n P,,(cos 0)
1
(29)
11-:::0
I u.2 Ldµl a . •· Equation 767
13. Substitute 0 11 and b11 of Problem 11 into u(r, 0) and interchange the order of summation and integrJtion to gt:t
Poisson 's i11revot for11111/a for a disk:

11(r. 0)
<I: - r~
= -..,
__ ,r
f_., f (x)dx
(25)
-.'!
o 2 - 2ar tOS(x - 0 ) + r2
14. Let r = 0 in rhe solut ion 10 the previou., problem and show 1ha1
11(0. 0) = -I
2rr , il
£·' ll(ll , O)d0 (26)

Interpret 1his result physically. This property of rhc solutions 10 Laplace\ c4u,Hion is culled 1he mran l"(J/11e
f'rrl/lt!rf_\'.

I 5. u~~ Poisson'), integral formula for a disk (Problem 13) tn show 1h..11 if the c IL-ctrostatic potential on a c i rclc is
a con::aan1. !hen the potential within the circle will be ::i constant .
16. Use Poisson's integral formula for J. disk (Problem 13) to show that the maximum value of the electrostatic
polenti.il with in a circular region mus! occ.:ur nn the houndary. This result is known a..:; 1hc 111ari11111m principle.
/-lin1: Show 1ha1 if j(B) .:'.:: M (a constant). then 11(r. 0) ~ M.

11
, '
17 • Ven·1·y t l1c genera l so I ut1011
. ol. r-' R 11 (r) + r R (r)
• '
- - :. -
--,-r- ' R (r) = O given
. .111 Examp Ie ..
3
/-
18. DL'.lenninc the: po1en1ial within a circular annulus of radii equal to 1 ;ind 2 wi1h 11( l. 0) = sin 2 I) and 11(2. 0) =0
( Fif ure 16.1 I ).

19. 171c rnrved !-Urfacc of a right -circular (;ylinder of radiu~ ll ( Figure 16.11) is maintained al zero temperatme .
and its two plane end · m :: = / are fixed al T (r. f:J. /) = To( I - r 2 /a 2 ) at::= I and al zero at:-. = 0. Dcrcrminc
the s1c:.u.ly- s1.a1c ternpcmturc in lhc cylindl!r. Hi111 : You need the integml
1
f ( I - .c )x J0 (awd d.r = 2J_(a,,)/ a,~ .
lo
20. lei .r = cos 0 in Equal ion 21 to der ive Leger11Jre·s (!{1u;ition. Equal ion 25.

21. Determine the ck_;tric porential in.<.ide a hollow sphere of radius a given 1hat rhe potential nn its surface is
110 cos 10 .

I \"

{I

X
2/

-----------

Figure 16.11 Figure 16.12


A circul:.ir :.innulus uf inner rac.liu~ I and The righ1 cylinder dc~crih.:d in
outer r:.idiu., 2. {SCl' Problem I~-) Problem 11>.
770 Ch µ r 1b I Partia l Dill r nt1.:i l Equal ion

SOLUTION: We can :O-l'.:: this ma1hema1ically by con.,iueri11g only the lirsl


hannonic in Equation t,.

where. for convenience. we have set (j) = 0 . !;,inµ the 11i~1i11omctric idcmi1y
. I .
sin Ci cos /3 = -2I srn(a
.
ff) - Sln(O' -
2
/3)

The wavelength of the firs1 harmoniL· ·i~ )._ = 21 (see Figure I6.I-lu). and so
11 1(x . r) can he wri11cn a)\

Each of tht: term." in 11 1(x. 1) represent" a rrave/ing 11·m·" · If we look al some


posi1ion x &nd let 1 vary. the tin-1 lcnn i-n 11 1(:r. ,, ) would appear 10 bell w.ivc
of wavL•li:ngth i . :111d frequency 1> =
,,J>-.. tr;1vdi11g !o 1h · 'lcd1. The ~econd
tenn in 11 1(.r. f) would be a similali \,a,c 1lravdin~ lo the ri1glh1. Tl1us. we
i.ce that a sranJing wave is rhe superp<1,ition of two simiilar rruveling ,w1,c~.
travrl ing opposite direc1 ions.

II is inslruc1ivc to rnnsider a simple c·.J.se in which 1t(.\. , ) l.'.Onsis1s of only Lhe


first two ham1l)nics and is. of the form ( Equal ion 6)

11(.r. r)
. -JTX + -I
= L'OS w 1t sm cos
(
w -,t + IT) .
- sin 2.rrx
-- (8)
/ 2 - 2 I

Equa1ion 8 is illu.-;tr:'.ltcd in Figure 16.15. ThL· left side of Figure 16.15 shows 1hc
lime dependence of each mode separately. Notice thut u~(x. r) has gone through
one complete oscillacion in the lime depicted while 11 1(.r. r) has gone: through only
one-half cycle. nicely illus1rn1ing thJL cv 2 = 2,v 1. The righl side of Fif!un.: 16.15
shows the sum of the 1,vo hamwnics. or 1hc accual motion of the string. a., ;i funccion
or time. II is inceresting to s.ee how a superpn ·it ion of the .standing wave)' in 1he
lefr ,ille of Lhe figure yields the tr.iveling, ave in 1he right side.
We're nor finished solving Equation I. El1ua1ion 5 (or equivalently, Equation 6)
s1ill ha-. 1wo infinite sets of constants 10 be de1crmincd. No1c rhat Equation 5 i · of
Lhc fonn of <1 Fourier sine series. where 1hc Fourier coefficients are a11 cos w,,,
1>11 sin w, 11. We can evaluate all rhc a,,_ and h., from Ihc two initi::il conllirions .
772 Chapre, I b I Partial Difi1•rc•n1i.1I Equarions

where w,1 = n ,r u/I. The in.itia.l condition 0) = 0 implies !hat the bn = 0. The
11 1 (x.

a 11 are given by the initiaJ condition u(x. 0) = u0 sin 2rr x I J,

211 0 ( . 2nx . mrxd .


On = J }o Sin' ~
1
- SIO - , - X = 110 ~"'1
where 8n 2 is a Kronecker delta. The complele solution is

.
u(x. t) = u0 sm, -
2JT
-
.l'
cos - -
2,r IJI
(13)
/ J

It's easy 10 see that this soluLion satisfies 1hc wave equation along with rhc given
boundary conditions and initial conditions (Problem 4).

Example 2:
Express Equation 13 as the sum of two 1raveling waves.

SOLUTION: We simply use the trigonometric relation

. a
sin cos
/3 = -2I sm(a
. " + -I sm(O'
+ ~) . - fJ)
2
to write

11(x.1) = :2I sin. [2rr ] I . [2rr


1 (x + tJI) + zsm 1 (x - tJ/) ]
The first 1cnn on 1he right is a sine wave trnveling from right 10 left and the
second 1enn is a sine wave 1rnvclling from left 10 righ1.

Ex.ample 3:
Suppose a string i, initially displaced a small dis ranee b at the middle of I.he
string and then released (in at.her words. plucked in the middle). Determine
the subscquen1 mo1ion of the sr.ring.

SOL u TIO N: The initial conditions lnin.slale into


2h.x I

l
O<x<-
- - 2
u(x.0)=
2b J
- ( I - x) - < X < I
J 2 - -

and 11, (x, 0) = 0. Equation 12 gives b,, = 0 and Equal ion I Ogives (Problem 5)

a,,= -2
,
[1'u
12
2hx
-
/
. nrrx
!,.In --dx
/
+ 1, 1
2h
-(/-
• / /~ /
. --dx
x) sin 11,rx
/
]

m al
776 (h,1pl(•r 16 / Par1i,1I LJitkr,•111i,1I Equ,11ions

1he string is 4' (x) and that its in i1ial velociry is if., (.x ). so tha1

tt(X, 0) = </J (.t) and u,(x. 0) = v1(x) (23)

Then.

<l>(x) = f(x) + g(x) (24)

and

,fr(x) = -vf'(x) + vg'(x) (25)

where the primes here mean differen1iating with re.spect to the arguments of f
and g. Jntegrating Equation 25 from O lox gives

f(x) - g(.x) = - -()11:r


0
1/J(u)du + c

where c = /(0) - .~(0) . Adding and subtracting this result from Equarion 24 gives

/(x)
l
= -<t,(x) - -
I 1.r ,/J (11)d11 + -c (26)
2 2v o 2

and

g(x) = 1
-</>(x) + -I 1·Y 1/J(u)du - c- (27)
· 2 2u o 2

Therefore, the solution to Equa1ion 14 is (Problem 17)

-1 X
<J>(.x - ul)
11(x.r)=--------+-
+ (j)(.r + vl) I 1·r ,., if.,(11)du (28)
2 2u .1"-ll/

Figure 16.1 B
A plot of the initial dis1urb.111cc Equal.ion 28 is known as d'Alemberr ·s ."iolurio11 to 1he one-dimensional" an: cqua-
,J,(x)= I - lxl when -I:_ x:: I and 1ion. It givc.s us !he displacemcn1 "(x, , ) in terms of 1he given initial displacement
q,(x) = 0 otherwise. u(x. 0) = <J>(x) and !he ini1iaJ velocity 11 1 (:r, 0) = i/l(x).
Suppose that lj)(x) = u(x, 0) is given by
u
-1,:S:x,:S:l
(29)
otherwise

and that V' (.x) = 11,(x. 0) =0. Figure 16.18 shows that cp(x) is a triangular wave-
form centered al x = 0. Now. as time increases, !he displacement is given by

I I
11(.x. I) = -(j) (.r - vl) + -<p(x + ul) (30)
2 2
Figure 16.19 This resull sugges1s that as time increases. the original triangular wavefonn splits
An ini1inl uiangular wavefom1 breaking
into two. with one half moving 10 the right and the other half moving 10 rhe lefl.
in10 two and moving in opposite
directions. as in Figure 16.19,
163 The One-Dimensional Wave Equarion 777

Example 5:
Consider an infini1cly long string Lhal is released from rest with a
displacement

q,(X) = (' - X
l

Dc1cnnine Lhc subsequent mo1ion of I.he string.

SOLUTION: Using Equation 28 with ,J,(x) = 0.

The displacemeni of I.he slring a1 vr = 0. 1.5. 2, and 8 is shown in


Figure 16.20. Note Lhat one haJf of the initial disturbance propagates to Figure 16.20
the right and the other haJf propagates to the lct't. ll1c di5placemenl in Example 5 at
=
vr 0.25. 0.50, 0.75. and 1.0.

When d"Alemben discovered bis general solution 10 the one-dimensional


wave equation. he initially thought that he had discovered a method that is ap-
plicable to other partial differential equ.at'iort-;. bur it was not to be.

16.3 Problems
1. Verify Equation 4.

2. Show Iha1 F cos ilJf + G sin (J)f can be written as A cos(wr + </J).
3. Show Lhat I.he number of nodes of sin nJT x / I between O and I is n - I.

4. Show that Equa1ion 13 saristies I.he wave equation and the boundary conditions 11(0, r) = 11(1. I)= 0 and 1he
initial conditions u (x. 0) = 11 0 sin 2rr x / I and 11 1(x. 0) = 0.
.
5. Venfy thnt b,1 ;:::: 0 and that a11 = -n 8b . nrr.
, sm -
2;r- 2
in Example 3.

6. Solve 1he one-dimen~ionol wave equa1ion subject 10 the conditions 11 (0. t) ="(I. t) = O. 11 (x. 0) = sin(Jrr x /I).
and 11,(x. 0) =0.
7. If 11 (x. 0) = 11 11 sin 3 rr x / I in the previous problem. can you predict which nonnal modes will be excited?
8. Solve 1he one-dimensional wave equation subject 10 the condition~ 11(0, t) = 11(1, t) = 0, u(x, 0) = 0. and
",(x. OJ = 11 10 .
9. Solve the one-dimensional wave equatjon subject to the conditions 11 (0. t) = 11(1. 1) = 0. u (x. 0) = u 0 x(l - x).
and 11 (.x, 0) = 11,0,
1

10. Show lhat (.x. I) = f (.x ± u1 ). where / (z) is a differeni-iable function. is a (~enernl) solution to the
11

one-dimensional wave equation.


778 C.:h,1.pll'r 16 / Pania.I Differen tial Equa1 iu11~

11. If a vibra1ing string has a rcsisiancc 10 motion 1ha1 is proponional to lhe velocity of the string. Lhcn the
82u 0211 Ju .
wave equation takes the fonn (sec Problem 11 of Section I) - ., = 1I -") + y-. where y 1s a constant
or v- a,- at
that is a measure of the resistance. Solve this equacion subject 10 the conditions u(O. f) = u(I. I) = 0.
11(x.0)=110 sin1rxi/.and11,(x,O)=O. AssumeLhatJ1 < 1//u.

12. Problem 15 of Section I shows that if the linear mnss density of a string is nor uniform. then the wove equation
;i2"
takes the form r - = p(x JiJ211 . _ . . . .
- . Show that separauon ol vanables leads to the Sturm-L1ouv1lle C(1ua11on
iJx 1 iJl 1
rX"(.d + ).p(x)X(x) = 0.
13. The kinetic energy ond potcn1ial energy of a vibrating string are (see Problem 10 of Section I) K =
; [
_ • ,,
1

Di
(~)
2
dx and V =; f' (~'o.\~)
_ ,,
2
d.r. Show I.hat the total energy of Lhc string in Problem 4 i., n
constant.

14. Using the equations in the previous problem. show that the total energy of e1.1ch normal mode in Equarion 6 is
a constanl.

15. Use a ("AS 10 show thl.lt 11 1(x. r) = e- I61 l - tJ 1 and 11'.!(.r, /) = 1.Se- <.( - 1)? arc Iwo Gaussian waveforms 1raveling
in different directions.

16. Show that Equation 22 is a solution lo Equation 14.

17. Derive Equation 28 from Equa1ions 16 and 27.

18. Use d' Alcmbcrt's formula 10 find the ,olu1ion to the wave equation rhat ,ati,fil'S the initial conditions
u(x. 0) = =
sin :rx/ I and 11,(x. 0) 0 fur -:x, < r -:c- -:io .

19. Repent the previous problem with the initial conditions 11(x. 0) = sin(;rx //) cos(;r x / /) and 11Jr, 0) =
cos(rrx//).

20. Coosidcr an infiru1ely long string thill is released from rest with a di~pla~c111cn1 <J>(x) = 1/( I -- -ix 2). De1erminc
the subsec1uen1 motion of the ).Iring.

21. Show that if 'v?. depends upon only rhc radial coordinate r in spherical coordinalcs, then 'v 211 = f!.11 can be
2
.
wnnen d (ru) =/(·(r11).
as --.,- '·" N ow~ how t hat 'f
I ,....-,
v- depcn d s upon on I yr. then the wave equation
. bcconll!S
dr -
., a2(ru} I a2(rn)
'v-11 = --.,-=---; - -,-.Show
. . .
that Lhc gc11c.rnl solution 10 1h1s cquanon 1s 11(r.
.
t) = -I j(r + vt)+
ar- v· a,- ,.
I . .
- f.: (r - vf). rnterpret this resu It physically.
r

=
22. Consider the wave equation in the previous problem. First show Lhat rn (r, 1) r R(r )e ± i ' reduces the partial
differential equation to an onlinary differential equarion. Now show that IJ1e solution 10 that equation is of the
fonn rR(r) = e · iwr /1•. Finally. show th::.it 11(r. I) is of the ~l·neral form given in the previous problem.
11,, .4 The Tw o-Oimen io na l \Na ve q uation 783

(23)

where k i~ a ,;cpa1111ion constant. Problem I J has you show I.hat k must be ncg;:.11ivc.
so we'll write ii ask= -.A.~. which assures l.hat k. is negative so long as A is real.
The solution to the equation for T(t) is

(24)

We could have ,m1icipa1ed that the scparntion constant k must be negative hecause
we expec.:\ physically that T(() should be t)scillatory.
The equation for R(r) is

2
d R
- + -l -dR + ,,._ 2 R-O
- (25)
dr2 r dr

Do you recognize thi:- equation? This is Bessel's equation of order zero. Refening
to Equation 12.5.42. we find that the: solution to Equation 2.5 is (Problem 14)

(26)

where J0().r) and Y0 0.r) arc zero-order Bessel funcl ions of the first and second
k.ind. rc~pccLively. and where A is yet to be determined .
A ~olution to Equation 20 is the product of Equations 24 and 26:

Recall fmm Section 12.5 that Jn(.:) is finite for all values of ; . bur that Y0 (z)
diverge as :, ➔ 0 (Problem 15 ). Jn order to assure rhat the di!.placcmcn1 of the
membrane remains finite as r---? 0, we must ~d a 2 = 0 in Equation 27. Therefore.
we have

(28)

where b 1 = a 1c 1 and h~ == o 1r· 2•


We can nmv determine). by applying the boundary condition. 11(ri. t) = 0.
This boundary con<li1ion implic.., Iha1

J 0 ()..o) =0 (29)

Recall 1ha1 J0 (:) ii. an oscillatory func1ion !~l'.C Figure 16.23) and has an inlini1e
number of discrete zeros (that is, value~ or z. where 10 (:) = 0 ). Let these zeros be
denoted by a 1• o- 2 • a-' . . . . . Their numerical values are a 1 :-::: 2.4048. a.!= 5.520 I. Figure 16.23
ui :--,: 8.6537. aJ. = 11.79 U. and ,.;;o on. Equation 29 lhen says that The ,.cm1h order lk~sd fu 11d i, m of 1hc
fi . 1 kind . 111(:: l, ploncd ;1gain,1 ~-
II = I. 2. ], .. , (30)
786 Ch.:iµl<'r 16 / l'ani;il DiifercnJ ial l:qualions

8. Express f (.\·. y) = x 2 y~ for -a :::: .r < a and -b ~ y < b as a double Fourier cosine )-Cric~.

1 21r \' f
2JI X
9. Express f(x. •\')=cos -(1- cos· --·
,,
or -a.::=: x < a and -
b
~ r <bas a double Founer
.
. cosine
. senes.
.

10. Solve the two-dimensional wave equation for the case where a rectangular membrane is given the initial shape
. 2rrx . 3,r r . d f
u (x. v)
.
= u 0. sin - - sin --· and 1s rck:a.i;e rom rc~t.
a b
11. Solve Equation I wilh !he conditions 11(0. y. I)= 11(0.. _\'. /) = 0: uJr. 0.1) = 111,(X. b. I)= 0: ll(X. _\". 0) =
f (.r. y); and 11 1 (.r, y. 0) = 0.
12. Find 1he solution 10 the previous prohlem if j(x. y) = (sin 3 rrx/u)(co. 2 '2.rry/1>).
13. Why mwa k be negative in Equation 23?
14. U e Equation J:2.5.42 to \'erify Equation 26.

15. . e the definition of Y0 (.::) in Section 12.5 to show that Y0 (;: ) - - a~ .::-. 0.
16. Use any C AS to determine numerically the values of the first few zeros of J0 (x ).

17. Show that the 11 = J mode a.~sociatcd wi1h Equation 12 has a nodaJ circle at r = 0.2779 a and one at r = 0.6179 u
and that i1 vibrates with a fre<"1ucncy 1.598 w 1.
18. Show 1hat the initial condition 11 1 (a. 0) = 0 requires that all the b 2n in Equal ion 11 must vanish.
19. Discuss how the solution to the vibrations of an annulus wou Id di ffer from rhat of a circular membrane.

16.5 The Heat Equation


/ Consider a slab of uniform material of thicknes~ I in the x direction as shown in
Figure 16.27. Let 1he y and ::. di mcnsions of the slab be much gTCatcr than /. so
r that we can ignore any edge effecrs al the _,. and : t>oundaries. The temperature
I /
I ; distribution in the slab will vary only in the x direction. so the governing equation

----(
/1
I /

----- V
is the one-dimen.--ional heat equation

0 :S x :SI
( I)
0 < I
.\
To find a unique solution, we must have two boundary condi1ions and one initial
condition (unlike the wave equation. which requires rwo initial conditions). Let's
assume tha1 the two faces of the slab are m;:iintained a1 T = 0 and 1ha! the in11ial
temperature profile in the iilab is given by .f(x). In tenns of equations. we have
Figure 16.27
A slab of uniform ma1erial of thicknl'"-" '·
7(0. t) =0 T(l.1)=0 T(x. 0) = /(x) (2)
I £>.5 The He.:it Equarion 787
Equations I and 2 could also describe a lhin homogeneous rod or wire of :-
length / 1hal is i n.su lated along its lateral surface. as shown in Figure 16.28. so lha1 . X

its 1emper,11ure varies only in the x direct ion. Figure 16. 29 summarizes EquaLions 1
and 2. Figure 16.28
To solve Equa1ion I by separation of variables. substitute T (x, , ) = X (x)8(1) A 1.hin homogeneous wire of length/ 1.h:i1
is insul::itcd along iL~ lateral surface so
into Equation 1 and divide by T(x. 1) = X(x)B(r) to get 1h:ll the temper.11-vr,e varies only in !he .I'
direction.
X"(x) (0'(t ) .~
- - = --
2
- =-),· (3)
X (x) a 0U)

or

X" (x) + A~ X (x) = 0 (4)

amJ

8
1
(1) + ,/>. 2 8((J = 0 (5) T=O T=O
We wrote the separalion constant as-). 2 because the boundary conditions require /(:r)

that it be negative (Problem I). The two initial conditions are equivalent to X (0) = 0 X

X (I) = 0. The solution to Equal ion 4 with these boundary conditions is


Figure 16.29
. IITTX A ~ummo.ry of the boundary condilion~
X(x) = sin - - II=\, 2 .... (6) ;ind initial condition~ for Equations I
/
and 2.
The solution 10 Equ<:1tion 5 gives

(7)

T(t" t) =cII sin--e-~


111T X r,~,,- ' 11 - 1 1 / )
' = I. 2 .... (8)
II • • • I 11

Equation 8 with 11 =
I. 2, ... sali:sties Equation I and the boundary conditions.
The general solution is a surerposirion of the .solution,; in Equal.ion ~:
t,o N
"\""""' "\""""' . IIT(X -0=11 ~;r i 1J J~
T(x. t) = L T,,(x. I)= Len sin - -e ' (9)
u=l n=I
1

We have yet to impose lhc initial condition. If we set, = 0 in Equation 9. then


we ohinin
00
~ . l'.liTX
T(x, 0) = f(x) = L., en sin - - ( I 0)
11=1 I

Equation 10 is a Fourier sine series and the en arc given by

c,1 = -2
I .o
ll ). . (x) ~m --d:r:
/
1/JCX
( 11 )
788 Chaprer l 6 / P,rniat DHtNPnlial EquarioM

Example 1:
Solve Equal.ion I subjecr to the boundary conditions T(0.1) = T(I, t) =0
and the inilial condition T (x. 0) = 70 = cons1ant for O < x < I.

SOLUTION: The .'-Olution is given by Equal.ion 9 wilh

en= -2To 1' . sin


IITr X
--dx =-
2'10
[I - (-1)
n
I II= I, 2, ...
/ 0 / ""

= 4T0 n = I. 3. 5....
nrr

and 1..cro orhcrwise. Thus,

. (2n - l )JU
sm - - --
T T(x. I)= 4To L I
rr ,r;;;l 211 - I

Figure 16.30 shows T(x. t) ploned againsl.r for several values of, (acrnally
a 2r I(!.).
x/1

~ Figure 16.30 illusLrate.-. a difference between the wave equation and the heal
1-- - - ~ -
/ equation (or the diffusion equation). Notice 1ha1 the bo:t.•like initial temperature
profile smooths ou1 as time increases. This is typical of solu1ions 10 the heat equa•
/a 2
, I 12 1jon. Compare this ro Figure 16.19. which shows an iniliaJ triangular waveform
splitting into two triangular W8\'Cforms moving in opposite direction . The under-
Figure 16.30 lying reason for this behavior is 1ha1 the wave equation is Lime-reversible. in 1hc
The solution 10 the equations in E.-c:ample 1
sense !hat changing t to -/ lea\"cs the equation unah.ered. If you run time back•
plou.cdl agairu.r x for ~vcral \':tlu cs of
a2, I ,2_ ward. the solutions simply retrncc Lhemselves, as you can see by changing, Lo - ,
i.n I.he wave equation. Newton ·s equations of motion are also time•reversible, nnd
lhus we expecl mechanicaJ sysrems in general to be rime-reversible. This is not
so for the heal equation; it is not time-reversible. Unlike mechanical systems. the
hear equation admiL'i a certain directionahl-y in time. which leads 10 the smoothing
process iIlustrated in Figure 16.30. A molecular (slatisticaJ mechanical) derivation
of 1he heal equa1ion would show that i1 rests upon several statistical assumptions.
which lead to its unidirectionality.
You may have wondered how the initial temperature throughout the material
could be 70 and yet the boundary conditions say that T(O. I) = T(J. 1) = 0. In
practice there really isn ·1 a sharp discontinuity at the boundary: the temperal'urc
might fall from To to O over a very short dist.ance. Whal we have fonnulnted in
Example I (and most other problems that we consider) is an idealization of the
actual situation. Unless we are specifica/Jy in1erested in a small boundary layer
al I.he edges of the material. the discrepancy is of no physical importance. Part
of 1hc chollcnge in representing a physical sys1cm by a mathemo1ical fomiaJism
is to model the syst.em such rhat I.he impon.anr physical features are faithfully
represented and yet keep the ma1hematical description as simple as possible.
16.S The Heal Equation 789
So far. the only boundary conditions that we have considered are T(O, I) =
T (I. t) = 0. In other words. the 1cmperature is prescribed on the boundary. This
type of boundary condition. prescribing a value of T (not necessarily zero) is caJ led
a Dirichlel boundary condirion or a boundary condition of the first kind. Another
common boundary condition is 10 prescribe the Hux of energy as heat across a
boundary. Mathematically, this amou.nls to prescribing the normal gradient of T
at the boundary. For example, if the surface is insulated, then lhe flux across the
surface is zero. Thi$ type of boundary condition is called a Ne11ma1111 boundary
co11di1ion or a boundary condi1io11 of the second kind. There is also a boundary
condirion of 1he third kind. Let the surface be in contact with the surrounding
medium. which we model as an infinite heat bath at temperature 7<1. According to
Newton ·s law of cooling, the flux of energy as heat across the surface is given by

flux = a( r (0.1) - ToJ

The constant a is called the coeffcie111 of surface heal transfer. This flux. must be
equal to the flux from the body into the surface of the material due to conduction,
or

tlux = -A arl
-
ax .r=O

where>.. is the thermal conductivity. Equating these two fluxes gives the boundary
condition of the third kind:

(al a boundary) (12)

Equation 12 is also called a Robin boundary condition or a ratliatio11 boundary


wndi1io11. In practice. it is often difficult 10 prescribe a fixed surfnee temperature
(as in a Dirichlct boundary condition) and a radiation boundary condition may be
more appropriate. Usually. however. a Dirichlct boundary condition is simpler to
implement
We'll now solve the one-dimensional heat equation for boundary conditions
of the second and third k.ind.

0 0
Example 2: II II
A thin wire of length / is totaHy insulated a11d its in.itiaJ temperature profile ~ f-.."'
is given by T(x. 0) = 70 sin 2 1r.r//. Determine the subsequent Lcmpcrature 2
To ·in ,rx/1
profiles. (Figure 16.31 summarizes lhese conditions.)
X

SOLUTION: The ma1hema1ical fonnulation of this problem is 10 solve


Figure 16.31
A summary of the boundnry condition~
and the initial condition for Exnmplc 2.

C
790 Ch;:iprcr I t> / l-'ar1ial Difforenlial Equation~

=
wirh rhe houndary condilion~ T, (0, t) Tr:U • 1 I : 0 and rhc inirial condition
T (x. 0) = T0 sin 2 ,r / J. Separ.ilion or vari;bles givt:s 1he 1wo equations

X"(x)+>-. 1 X(x)=O

and

=
with X' (0) = X ·U) 0. The separation constanr h,L'- been Iaken to · · i. 2
bec.:ause ii is nOI poss ible 10 s.11.isfy rhc boundary condit..ions if i1 is po~itive.
We now determine the allowed values of the 1-epan1Lion constanr >... Nore
IhuI ). = 0 yields X (x) = nx + h. The boundary condi1ions require 1hut
u = 0, bur b is arbi1r:1ry. So far. 1hen, we huve X0(x) = b nnd ). = 0. For
), =j:. 0. we have Xnl.1) = cosnrrx/1 whcrcn = I. 2 . ... , and .so we have

X (x)
n
= I COS - -
!,;rx
/
11=0
II= 1. 2 . . ..

where we set Ihe value of b ec1ual 10 I arbirrarily. The cqua1ion for (-),,(r)
gives

c:onst.ant 1/=0
(-),,(/) = { e - 0
2 l l / /2
;r II I
n = I. 2 . .. .

TI1c supcrpnsi1.ion of X,,(x)f>,,(t) gives


cc
T( .r. , ) = constant + "L a,, e rrl;r211 ; 1/ / l COr
.. 1/TfX
...
1
ir = I

This resulr would be or 1hc form of a Fourier co~inc series if we lei cons1ant
= ao/2. ,md so we wrile

We now use 1he initial condition to write


)0
_. . 2 nx o0 ~ nrr x
f (x. 0) = T0 sm - = -;- + L a 11 cos - -
/ ... n= I /

and so the a,, arc given by

1
2T0 [ . ., rr.r 11;:rx
"" = - -
1
lo sm- -,- cos - -t1x
1

I -cos -
2rr-
/
X) 1/!r X
cos - -dx.
( 2 1
800 Chdp! ·r I(., / Panial Difr •ri~nti,.11 E:qu,Jlions

regular singular points at x == ±I (Problem 5). so we expect that lhcre is at least


one Froben1us series solution about x =±I. We won·t go through that process
here. bu1 it rums ou1 that /3 must cqunl /(/ + I) wi1J1 I = 0. I. 2 .... (as in the ca!,.e
of Legendre's equation) for Equation 15 10 have solutions that are finite at x =±I
(0 = 0 and rr). Furthermore. it turns out that m must be :5 /. Thus, we see that
a rigid rotator is restricted 10 have only energies I.hat !,.ati~fy f3 = /(I + I), where
I= 0. I. 2 ..... Using Equation 8. we sec 1ha1 the energy of a rigid rotator is given
by

,,2
E = -1(1 + I) 1=0.1,2 .... (16)
21

These allowed em:rgii:s account for the experimentally observed microwave spec-
tra or diatomic molecules.
The solutions to Equation 15 that are well behaved at x = ± I are called
associa1ed Legendre ftmctirm.'i. which are customarily denoted by Pt 1<x). The
superscript is wri lten as Im I because m appears only as m 2 in Equation 15. TI1c
associated Legendre functions can be defined in terms of Legendre polynomia.ls
by tJ1c relation

( 17)

Note 1hat Pt' 1(x) = 0 if m > I because P1(x) is an Ith degree polynomial. Note
also tha1 the P) i(x) are polynomials only if m is even. The first few associated
111

Legendre functions are given in Table 16.1.


Just a.:. Lhe Legendre polynomials satisfy the onhogonality relation

= r d0 sin 0 P1(cos 0)Pn(cos 0) = ~


1

1 -1
P1(x)Pn(x)dx
lo 21 + I

Table 16.1
ll1e first few associated Legendre
functions. P/'
(.r).

Pg(x) = I

P~(x) = x = cos 0
Pi'(x) = (I - x 2 ) 1l 2 = $in O
Pf= ~(3x 2 - I)= !Ocos2 0- I)
P,J(x) = 3x( I - x 2 ) 112 = J cos O sin 0
P}(x) = J( I - x 1 ) = 3 sin 2 0
802 C!1 clJ) h·r 16 / Pa11i.1l Di(i r nti I [qu ,ili 11--.

Remember now Lhat we·re in the process of so lving the SchJodinger equation
for a rigid rotaior. Equation 7. Putting everyLbfog 1ogc1hcr, the rigid-rotator wave
func1ion~ arc P1·"':(w,0) 8m(</.i). By referring to 1:.quul.i,ons 14 and 19. we sec that
the func1ions

are normal i1ed solutions to Equal ion 7.


The Y/ 11 (II. (/)) form an orthonomial set.

(21)

Nole that the Y,n' (0. rp) arc orthononnal wi Ih respect to sin fJ d0d</) and nut just
d0t1</J. The fac1or ~in fJ d0dfj) has a simple phy$ical interpretation. The dilTercntial
volume element in spherical coordinates is ,.~ ~in 0 drdfJdc/>. If r is a constant.
as it is in the case of a rigid rol!lLOr. and ~, equal lo unity for conveniem:e. 1hen
!he sphericaJ coordinate volume element becomes a sphciical surface dernent.
d A = sin d(Mtj,. If this surface clement is integrated over 0 and ,t,. we obtain
4n. the surface area of a sphere of unit radius. According 10 Equation 21, the
Y/n(0. </J) are orthononnal over a spherical surface and so nrc called .~phericol
harmonic.{;. Spherical harmonics occur in a great varie1y of physical problems
involving spherical coordinates. The lir!-t few spherical harmonics are given in
Table 16.2.

Example 3:
Show 1hat Yi'(fJ. <J,) is nom1ali1,ed and onhognnal 10 r:\o, </J).
SOLUTION:
I r,.6 Th S hrodi ng<fr Equation 803

In summary, 1hc Schr&linger equation for a rigid ro1ator is

'J-fYj1'(0. r/>) = 1/t(l + I) 't'"f"(0. <J,) (22)


21
wi1h '.}{ given by

C. THE ELECTRON IN A HV0ROGEN ATOM


The spherical harmonics occur whcneva the Schrodinger equation is snlved for
a poten1ial of the form V = V(r). 11·s s1ra.igh1forward 10 show that if V V(r). =
then (Problem 13)

if1 (r. 0. tp) :;:; R(r) Y/" (0. rt,) (23)

where R(r) is the solution to 1hc ordi nary differen1ial equation

. li-
--
2

2mr · dr
-d ( r-, JR)
-
dr
+ [li 2
W
2mr~
+, I) + V(r) - £ ] R(r) =0 (24)

The hnal problem 1ha1 we shoU discuss in this section is the Schrodinger
equa1ion for u hydrogen atom. one of Lhc grca1 triumphs of quantum mechanics.
As our model. we shall picture 1hc hydliogcn atom as a proton fixed at rhe origin
and an electron of reduced mass J.L inh:tac1ing with t.he proton through a coulomhic
potential,

l
e-
V(r) = --- (25)
4rr€11r

In Equation 25. e is the charge on 1hc proton. e 0 is the pcrmi11ivity of free space.
and r is the distance between the electron and the proton. The model sug_gests that
we use a spherical coordinate sy~tcm with the proton al 1he origin. Therefore. the
Schrodinger equal.ion for a hydrogtn atom can be wrillcn as

or

(26)

Note that this equal ion is of the form 'J{ 1/.t =-= E iJ, . regardless of i1.s apparent
806 _,h 1pi .. , I r, / l',111ici l Diii ren1 ial [J] U, LiOl'h

The orthogonality condition i-. given by

= ---" JT 1·"1 dO ~in O cos fJ l"C dp p-' 2 - p)e_,, =0


32:r O •0

been use of I.he integral over 0.

16.6 Problems
I. Derive Equal.ion 3.
2. Show rhat the nomwhz-ition constant I)( ~,•er. y. z.)1/1(x. y . ;::) '!Ii Equal-ion 1 is A.rA_,.A: = (8/ohc) 1I?. .
J. Determine the allowed energies of a particle in a two-dimensional rectangular box of sides a aIJd b.
4. Show Iha! lhc allowed energies of a particle constr..tincd to the circular potential-free region arc given by

En,,, = :,:a /J,;111. where finm is lhe mlh zero or lhe f1r,1-11rder Bessel function .J,,(x).
5. Show that Equ::11ion 15 with m -:f:= 0 has regubr singular points al x = ±I .
6. Show 1ha1 rhc firs! few associated Legendre functions in Table 16.1 satisfy Equ:ition 18.
7. Shnw that the fiN few associated Legendre functions in Table 16.1 satisfy the recursion formula

8. . c u CAS 10 show Lhar 1hc firi-:1 few Legendre func1io11s '.-atisfy the recun.ion formula

p !m 2 (\') - :?(m + l)x p lm , l (xl+(11-m)(11+m+l)P 11111 (x)=0 .


11 • I _ .r 2) l/ 2 " - 11

Be sure rhal your CAS defines P,t' 1C-r) cxac1ly as we have done. A fe\v authors include n foe tor of (-1 )"'.

9. Show rhal the spheric.ii harninnic· satisfy I.he e<1ua1ion

a ( <J Ym) · 1 ym
sin 0 - sin B-1- + --1- + 1(1 + I) sin~ H Y1"' = 0.
uH a0 a~~

10. Show explici1ly 1ha1 rhc fir.-1 few spherical hannonics in Table 16_2 sa1isfy the equation in Prohlcm 9.
11. Show 1ha1 rhc tina few spherical hannonic:s in Table 16.2 i.lrc onhogonal 10 each 01h~r.
12. U~ing Table 16.2. show 1ha11Yi (tJ,~)l:?+1r~•(ll.,f,)l l IY1- 1(H.r,(>)J 2 =J/4:-r . Thi~ i, ;1 spcci;:il c:i:-:e ofa
I

gencr..11 theorem_ L IY/"W. <Ml~= con.~ lanl. km1wn as l 111sii/d\ tht'OH'IIL


m=-1
13. Shnw 1hu1 if V = V(r) in the- St:hri.'idingcr cqu;11in11. then v,(r. /J. lj)) = R(rH'/"(H. </>). w liierc R(r) ~ali '!- lics
l:.quation 24.
16. 7 The Cl a ification of Pc1r1ial Diii r nli,1 1 Eq1MliOns
1

807
14. The equation 'iJ 211 + k 211 = 0 is called the 1-/elmlwltz. equario11 . Show tha111(r) = j(r)Y/"(B.1>) is a solu1ion
lo Lhc Hclmhollz equation in spherical coordioare.~.

15. The LL.iguerre pnlynnmiols can be defined by the Rodrigue1. formula L,,(x) = e:c ..!!:._(t• - .x x" J. Use this fom1ula
dx"
to gcnc..i1e the fi~I few Laguerre polynomials.
16. The associ:itcd Laguerre polynomi.il~ ..:an be defined in terms of the Laguerre polynomials (s.ee the pre\•ious

problem) by L ct(x)
n
= !}::_
d:r:«
L,,(x). Use rhis formula 10 generate the first few entries in Tnble 16.3.

17. Show explicitly 1hat V'ilO(r. fJ. rp) ~ari~lies Equation 27 with E given by Equation 28.
18. In this problem . we shall determine rhe allowed energies of a particle constrnincd to lie within a potential-free
sphere. \.'.sing fa1ua1.ion 24 a<; a start, show 1ha1 the radial pan of the Schrodinger equation for this system i:-.
2 , [ 2m E I (I + l) ] . ·- 0
R (ri + -R (r) + - 1- - - - 1- R(r) = 0 w11h 1he boundary cond1t10n R(a) = . Show that the
11

r h- r-
solu1ion Lo rhis equation is R1tJ(r) = c 1, - l / l 11 ! (/fr) c 2,-ll 2 y1 f (fir) where f.;= (2111£) 112;/i. Why musr
c~ = O? Now show that /fo must be :.i 7.Cro of, - •/ ~11 ! (r). If f3tm is the mlh zero of , - 11211+ ! (r) , show that
. . , hJ
me allowed energies arc given by £,n = /31~ .
1
2ma-1

16. 7 The Classification of Partial Differential Equations


la Section 2. we leamet.l that the wave equation ha" a general solution

11(.r. /) = f(x + tlf) +R(X - VI) (I)

where J and g are suitably weU -behaved functions. You might have wondered if
the other panial differential equal ions tha1 we have discussed have similar geneml
solutions. Recall 1ha1 we derived Equation I by t.ransfonning the wave equation
into I.he form

(2)

under the linear lransfonnation

~ = ..r + VI and ,, = x - or (3)

Consider rhc second-order equa1ion in two independent var1ables,

(4)

where a. IJ, and c life cons1a111s (for simplicity. tha1·s all we'll discuss in this
section). and the linear transformation

and I)= X + fit (5)


16.7 The Cl.1,~iikation of Partial UitfercntiJI Equations 809
a = c = I and b = 0. so
2
b - 4ac = -4 < 0
Thus, uiplace's equation is an elliptic partial differential equation.

=
We sec from lh.is Example that a= -v and f3 v for the wave equation, so
that the l.inear transformation in Equation 3 is ~ = x - vt and 11 = x + ut. as we
found in Section 2.
We didn't include the heat equation in Example I because ii is nol of the form
of Equation 4. A more general second-order panial differential equation is

(10)

The above classi fie.at ion scheme in Equation 9 applies to Equation 10 also; the
classific.arion scheme depends only upon the coefficients of 1.he second derivatives
in Equation I0. Therefore, if we consider the heat equation.

o2 T I oT
ax 2 cr 2 a,
we see that a= I and b = c = 0. so that b2 = 4ac = 0 and the heat equation is
parabolic.

Ex.ample 2:
Classify the Helmhollz equation.

SO L U TI ON : The c Iass ifie al.ion scheme depends on Iy upon the coef1ic ient:s
ofLhe second derivative!-, so the Helmholtz equation is cllip1ic. like Laplace'!.
equation.

We can derive a general solution for Laplace's equation. much like we did for
= =
the wave equation. According lo Example I. a c 1 and b 0 in Equations 7 =
and 8, so a= i and fJ = -i. If we subst.itule these values in10 Equation 6. we obtain

(l 1)
810 Ch.:ip1er 16 / r.ir1i.1I DiilL·r<·nli,tl Equarions

where

~ = X + iy and ,, = ~- - i_\' ( 12)

Integration of Equation 11 gives (Problem 11)

11(~. 1]) = _((~) + g(11)


where j and g are twice-differentiable functions. Using Equations 12. we have
Lhe following as the general solution to Laplc1ce·s equation:

11(x . .,•) = j(x + iy) + g(x - iy) ( 13)

Example 3:
In Example I of Section 2. we found that a certain solution to Laplace's
equation i~
IIJL'( . 1111' V
11 11 (.r. )')=cos - - smh - -·
11 a

where 11 = I. 2 ..... Show lha1 this solution is of 1.he form of Equa1ion 13.

SOLUTION: First use Lhe relatjon

sinh;: =- i sin i::.

lO Wrtle

. mrv . i,,rr,•
sinh - -
· = -i sm - -·
(I a
Now use Lhe relarion

cos a sm
. /J = -I s1n(a
.
+ /:J) - -I s1n(a
.
- /:J)
2 2
to write

u 11 (x, y) = --j [ .',Jn. -117T( x + iy) - • 111T


sin - ( x - iy) ]
2 a o
= j(.'C + iy) + g(x - iy)

Usually the general solutions to partial di fferentiaJ equal.ions arc not I.hat usl!ful
due to I.he difficully or determining 1he unknown functions in terms of rhe boundary
conditions. lr docs rum our.. however. lhat 1.he different types of partial differential
equations require different types of boundary conditions in order 10 be well posed.
By a well-posed equation, we mean an equation whose solution that obeys the
boundary conditions is unique and sioble. By stable. we mean 1hat the solution
812 Chaprer 16 / Par1ial Differential Equaiions

S• Sct b , = 0 m. Probl em 4 10 e 1·1mma1e


. t he cross 1em1 .ma ' x r2 + b':r ' y ' + c ' y ,,,- + d' x ' + e ' y ' + f = O an d argue
that the resulting equal.ion is that of an cllip.">C if a' c' > 0, that of a hyperbola if a' c' < 0. and that of a parabola
if a'c' = 0.
6. Show that ax 2 + b.xy + cy 2 = d is (a) the equation of a hyperbola if b2- - 4ac > O: (b) I.he equation of an
ellipse if b 2 - 42c < 0; and (c) the equal.ion of a parabola if b 2 = 4ac.
7. Detennine whether the following partial diffcrentiaJ equations arc hyperbolic. elliptic. or parabolic:

{b) U.r.r - 2u.ry + 2u_i·y = 0

8. Determine whether the following partial differcn1ial equations are hyperbolic. elliptic, or parabolic:

(a) ll_r:r - 2u:r_v + ":r +II= 0 {b) U.ry-Uy-U.r+J11=0

(c) u.u+u_l'y-11r+4u=0

9. Show Lha1 if v' 2 depends upon only the radiaJ coordjnalc r in spherical coordinates. then v' 2u = k 211
2
can be written as d (r u) = k 2 (ru). Now show thal if v' 2 depends upon only r. then the wave equation
dr 2
a2 (ru) I a2 (ru) . .
becomes V 2u = - -- = - 2 --,-. Show that the general .-;olut1on to I.his equa1.1on 1s u (r. 1) = - J (r
. . I
+ v1)
ar 2 V ar~ r
I . h .
+ - g(r - vt ). lnterprcl I.his result p ys1cally.
r
=
10. Consider the wave equation in the previous problem. Firs! show that ru(r. t) r R(r) e±iw, reduces lhe partial
differenlial equation to an ordinary differential equation. Now show that the solution 10 that equation is of the
fom1 rR(r) = e ±io>r/u_ Finally, show that 11(r, t) is of the general form given in the previous problem.
11. Show that the solulioo 10 Equal.ion 11 is u (~. '1) = J (~) + g(ri) where t and 17 arc gl ven in Equation 12.
. I 5 ot. Sect1on
12. Eq uat1on . 2 shows Lhnt "" (x. y) = sm. -
r1Jr x . h nn y . .
- sin - - 1s a solution .
to Laplace·s equa11on. Show
a a
that this solution i~ of the form of Equa1ion 13.
. (2n - l)JTx (2n - l)7ry
13. Example 2 of Section 2 shows that u 11 (.r. y) = cosh - - - - c o s - - - - is a solution to Laplacc·s
a a
equation. Show that this soluLion is of the fonn of Equation 13.

Referenc s

L.C. Andrews, 1986. Elemen1ary Partial Differential Eq11arinns with Boundary• Value
Problems, Academic Press
SJ. Farlow, 1993. Parria/ Differe11tia/ Eq,wrioru for Scit·mi.w (lJld Engineers, Do\'er
Publications
H. Levine. 1991. Partial Diffemuial Equatio,L\·. American Mathema1ical Socic1y
P. Moon and D.E. Spencer. 1969. Parrial Differential £qumirms. Heath
P. Moon and D.E. Spencer. 1961. Field Theory for £11 i11eers. Van Nostrand
Tyn Myial U. 1987, Partin/ Differential £.tj1wtio1 for Sr in11i.r1s and Ensi11cerl·, Elsevier
Ian Sneddon. 1957, Eleme111s nf Partial Diffe1·,m1ia/ Equations. McGraw-Hill
H.F. Weinbt!rger. 1995, ,\ First Co1Jrse in Pnrtit1l Differential Equations, Dover Publicn1ions
CHAPTER 17
Integral Transforms

We spent 1hc cmirc previou" chapler learning how 10 solve partial differential
equations. We learned mat lhri:c of the most commonly used part1al diffcreniial
equations. (the h~at c.qu:.ition. 1hc wave equal ion. and Laplacc's equation) arc lin<!::ir
partial differcnrial equations with constant cocfbcienL,, and can be readily solved
by !he method of SL'parat.ion of variahlcs. There is anoIher meIhod for solving 1hr.sc
equations. which is often more convenient, using integral transforms. which is the
subject of this chapter. An imcgraJ transform is a rcla1ion of 1hc fonn

1,
f(p)=
1 a K(p.x)f(x)dx

Given a fun<.:tion K ( p. x), ea[ led the kernel. this re lar ion IrJnsfonn)'; ft r l into
F(p ). There are a number of kernels 1ha1 find use in applied malhematic.,, but the
two most comrnonl)' used ones arc a Laplace tTJnsfonn.

and a Fourier transform.

Wc"\l lcarn how to use these tran~fom,s 10 ~olve pa11ial differential cqualions, and
we'll see th.it 1he. type of boundary condi1ion~ detennines which type of 1ransfoml
to use.
A characteristic propeny of these integral transforms is that they can be used
to redul:C the number of independent variabk~ in :.i partial differL'ntial equa1ion by
OHL". Thus. the: one-dimensional heat equation or wave equation can be transfom1cd

i1110 an ordinary di rfrrcntial equation in the transform runc1 ion F( p). If you apply
1he transform method to an ordinary diffi::r~nrial equation (only one independent 815

C gl
816 Ch.1prcr 17 / ln!t·gr,11 lr.111~forms

variable), 1hcn you get just an algebraic equation for the IIansform function. In
either case. i1's usuaJly much easier to solve the resulLant equa1ion for Lhe 1r:msfom1
function than it is 10 solve the original equation. The finaJ step in the method is
to undo the transform and deduce the function f(x) from F(p). This process is
culled inversion.
We' II learn some of the properties of Laplace transforms in Section I. and then
in Section 2. we'll learn a number of methods to invert them. Then in the nexl two
sections. we'll learn how to use Laplace tranforms to solve ordinary djfferential
equations and partial differential equations. We'll learn about Fourier Lransforms
in Section 5 and then use tbem to solve partial diffcrenlial equations in Section 6.

17 .1 The Laplace Transform

The Laplace transform of u function f (t) is defined by

.C/f(I)) = F(s) = fo"' e-Jr j(l)dt (I)

where .'i is a parameler that may be complex, although we' II consider it to be real in
most of Lhis chapter. For Lhe integrn.1 in Equal.ion I to converge, s must be greater
than zero ifs is reaJ, or the real part of s must be grealer than zero ifs is complex.
Nme 1hat the Laplace transform converts a function oft to a func1ion of s. For
example.

r -,
.._f,-1 = 100
o
t 2e -H
- dt = -s22
and

(2)

The evaluation of .C {f (r)) consists of no more Lhan perfonning 1he integration in


Equation I.

Example 1:
Determine .C { ,n), where 11 is an imeger.

SOLUTION:

00 I
,_,,
r{ ") =
1o
" -,td t =- --
re· II .
s" I
{3)

where we have used 1he fact that /n


rx: .\" 11 e-x d.'C = 11 !.
• I)
17. I The Laplace Translorm 817

Example 2:
Determine t.he Laplace transfonn of

f (t)
2
={ t ,~,
0:::1<1

SOLUTION:

Example 2 shows that /(r) does nor have 10 be continuous in order for its
Laplace transform to exist. Jn fact. if /(I) is only piecewise continuous over every
finite interval fort ~ 0 and of exponential order as t ➔ oo, then .GU(t) f exists.
By exponential order as r - oo. we mean Lhar there exisL-; a constant et such that

,n
is finite. For example. 111 (n an integer) is of cxponenliaJ order because e-b, ➔ 0
as t ➔ oo for any b > 0. An example of a function that is not of exp()nemial order
2
is e' • which diverges much faster Lhan t/' 1 for any value of b (Problem 5).
The above conditions for ..C..{f(t)} to exist are sufficient. but not necessary. A
classic example of a function that is nor continuous fort ?. 0 but whose Laplace
lransform exists is /(t) = 1- 112 . In t..h.is case,

Table 17. I lists a few Laplace transforms for easy reference. There are
many extensive tables of Laplace transforms. The CRC Mathematical Tables and
AbrarnowiLZ and S1egun list over a hundred. The most extensive readily available
table is Volume I of Tables of lnregra.l Transfonns by Erdelyi er al., which lislS
over a hundred pages of Laplace transforms. Furthermore. most CAS have built-in
Laplace transfonn rou1ines. For ex.ample.

Laplace Transform { f ( t ), t. s I

in Mathematica gives Lhc Laplace transform of f (t) in terms of s.


Even with extensive tables and computer programs, it's still important to be
aware of some general properties of Laplace transforms. 1\vo of these are called

al
818 Ch pt r 17 I lnt~ral ran ·forms

Table 17.1
A short table of I .aplacc transforms .

J(I) .C(J(I)) = F(s:) J(r) L [f(r)) = hs)


,, ~ r(k I)
t" II= 0, I. 2. . . . k > -I
sn - I

"I)/ X > (I 1''e" 1 ,f > {I


s- a (s-a)11+1
(I s
sin tlf s> 0 cos (If s>O
s~ + a2 sl - a 2
a
sinh llf -.s~.-
,-- .,
a~
S>O cosh a1
s 2 - a2
s>a

2as s2 - a2
r .sin at , cos I)(
(s~ + n~) 2 (x2+a2)2

rranslarion properlies. The first says 1ha1 if .C{f(I)} = F(s), then


.C{ea 1 f(r)J = F(s - n) s > (I (4)

Note in Table 17. I, for example, Iha! .q 111 ) = 11 !/s" + 1 whereas LI t "e"'} =
n!/(s - a)n+i, in agreement wi1h Equation 4 .

Example 3:
Show that

SOLUTION: Fir~! use

.C{coswt} = 1 e- ~, cos <.ut dt - - ~


- -.,
s- + r,r

nnd then replace .1· hy s + 2 10 obtain


f L k -~,
- C()Slt)f
)= ., s +2 .,
s- + 4s + 4 + o>-

There is a second translal ion property of Lapl;icc 1rnnsfon11s. Lcr \; consider


the func1ion defined by

Figure 17.1
0,St<a
(5)
A func1io11 /(r) ::inJ /(I - a) pll111<!d (I .:::: '
agaim1 , . ~otc 1hn1 f(t - a) i, ,imply
/(1) .~hifrcd 10 I.he right by a units. Nole rhat ~(r) is simply j(r) shifted a units to the right (Figure 17.1 ). lt"s easy to
822 Chap! r 17 / lot ra l Tran fo rms

17 .1 Problems
r . W r S
I. Show t1lat "'lsin wt I= - ..,- -., a.nd ..__(t:os 011 I = - .,- -., .
s- + ur .s- w-

2. Show 1hat .Cjr 11 e-u 1 I = " I


· where s +a > 0 and II is an integer.
(s + o)" 1

n f
3. Show that Llsinh ar) =-.,--.,and L/cosh ar I=-.,-·- 1
.
s- -a- .f~ -11-

4. Show lhal the following functions are of exponcn1ial order as t - oo:


(a) f 1J COS I (b) r11 eu 1 (c) cash 01

5. Show I.hate': is not of cxponcnt.ial order as, - '>-

=
6. Show that lim F(s) 0 if J(f) is piecewise continuous over every finite interval for , ':'.:. 0 and is of
' ">C>
cxponcnliaJ order. The significance of this result is that there are no Laplat·c lransfonns that arc polynomials
ins or trigonometric foncrions oi s.

7. Derive Equal.ion 10.


8. Suppose lhal / (r) is 1101 conti nuou.s. as required by Equation I 0. Show th::it Equation IO is modi lied to read
L{J'(l)I = sF(s) - f(O) - e- · 11 lf(f 1 1-J - /(f 1_)] if f(O has a (hni1c) di~cominuity al,= 1 1.

9. Show 1ha1 ( I - e-x) 2 /(I - e- 2x) = tanh(x /2).


10. If j(r) i.'- piecewi.~e continuous :md of cxpont.·nrial ordC'r as I - • then it is valid \0 differentiate Equation I

= Lf (-r)n f(I)\. u~ rhii: re!>ult to show th:.11 .C.{, ~in <J)I I = u- l <usw-)2 ·
with respect to .r 10 obr::iin fM(s)

11. Use the result of Problem 10 to show that Lit costul} = (s 1 - f,/.)/(.,· 2 + w - ) 1 .

12. UsetheresuhofProblem lOtocvaluate /(cr)=1 •· ,'lc- v'sinrdr.


0
JT
~in lVI 0 .:51 :::: -
IJ. Show 1ha1 ;f /(I + 2.• /wl = /(1) an<l ;f / (t) = 1 JT
{l)

2;r
0 - <f <-
w /1.)

rhcn i'(s) = L.{J(t)I = ~ ( I - e- m fwJ-1_


s- + (t )-
14. Show that r.he Laplace tran ·fom, of S(I) is cquaJ 10 I. Why Jocsn ·1 f"(.,) - , 0 ass-,. oo in this ca~L'·.,
.I · · tJ
15. Show that ..C./ ear f (r) l =
4

f ( - o) . Use this n:sult It) show that .C le"' rn~ II)/\ = ,\-,-------
·· 2ns -1- a '! ·:· w~ ·

16. Show thm if f (1 + 2) = J ( t) and if/ (1) = { 2 ~ 1 0::: f ~ I h~ r{ .


. t en ..._ _I ( t ) l = ranh(s/2)
, .
I < t 2 s-
0<r< k
17. Show lhat L{S1;(f )} = --k.<
l'

s
. .
,f ,S(. (I) = { 0I -
k _ t
.
18. Dell!nnine L 1,- 11 :.e-,,ii, 1. Hi111 : Use the integral that you evaluared in Problem 1.9. 14.

19. Show Lhal i:1, - 31 -e - a 2 / 1 } = C~,) I/ ~ e- :!u_..,.1!_ Him: Use the resuh of the previous problem .
823

20. Recall the defini1ion of the complcmenrary error func1ion given in Section 3.3. namely crfc(x) =
_3___
ft
fr
-x e_ 11 .d11. h turns out 1hat ({crfr(a / Ji> I for a > 0 occurs in a number or physical problems.

We will evaluate L(erfc(o / ✓I)} in this problem. First lei

erfc(a/ J,) = -a 1•1 e- a:h . d,,


,fir o 3
·., .
y I-
a > 0. Now write F(s)
~
11 =a/ ~

= L /erfc(a / J,) I = 1
in erfc(a / J,J and show that

o
e- , , erfc(a / Ji)dr as a

double iniegrn.1 and then inierchangc the order of integration to show tha1 F(s) = v;r,,·
: Jn[.x. -n-tr .v dy
y3/2.
Now let .r == x 2 and show I.hat

21. Show rhat Lllo(t)) = ~ • where J 0 (r) is a zero-order Bessel funclion or the first kind . Hi111: Use the
\ ,I + I
series detinhion of J0(r) and also use the resu It of Problem 2. 7. I 3. (Sec also Problem 23.)

!J(t l I. rhcn L [/(ar) I = ~ F (:.). Use Lhis rcsull to show that£. 1}0 (ul) I = ,
1
22. Show that if F(s) = L
a a (s · + a-), lf., .

23. Here is another way to evaluate ..C.{J0 {1)}. Stan with ,(Section 12.6) J0 (t) = _!_ (1 cos(/ sin ())dO. Now let
11 lo
= _!_ (1 I:! cos(t cos n )d0 = ~ r'
12
</) =0 - rr /2 and show that Jo(t) cos(t cos ())dB . Now lake the Laplace
n J-.., r1 rr lo

lr:rnsform of J0(r) and interchange the order.- of integration 10 gel


1,
. e- H J0 (t)rlr = -2s 1:r/_ d0
.
~- • + cos-1 0
1
n rr o
Now USC 1hc fnL't thal r.
Jo
.' 1, / l .
.~
d{I
+ cos·1 0
= 2s( 2
l1
+ I)
1/2 10 show that ..Cllo(t)J =~
s2 + I
-
24. Dcrcrmine the Laplace 1.nmsfom, of H (I) - H (r - I /2).

25. Show thnt if j(r) is picce\vise conlinuous and of exponentiaJ order and, in addition, if lim f (1)/ t exis1s. 1hen
t 0

( {/:I)} = [ F(/)ds' .

26. Show that lim /(1) = lim ~· f'(s) provided thc:~c limiL" cxi-'-L /-li111: Stan with ..C. (f 1 (r)J.
:-.x.. ,-o
27. Show that lim
, .... o+
J(I) = .,-,
Jim s F(s) providcll 1hei.e limits exis1. Hint: S1an with .CIJ'(,)I.
826 Chapter I 7 I lnh:!gr.:i l Transforms

or ex.= f:l = lj2. y = -1, and 8 = 0. Therefore, we have

- ----
s4 - a-i
- - +2(s
2(s + a)
-- ---
- a)
--
s2 + o 2

We can re.idily use the resuh of Ex.ample 2 to invert F(,·). Referring to


Table 17. I . we see that

f(t)=I,e-1{_
+2
}+IL-1[-' }-,e-1{~}
s
1
+a 2 s- a s- a~

= cosh at - cos at

If we try to use the result of Example I 10 invert F(s) in !hat ca\e, we don't
seem 10 be able to u.,:;e Table 17. I . We can use Equation 4 of the previous section,
hO\vever, which says that

Liem j(l)J = F(s - a) s a (2)

Lei's use lhis result to find the inver.se of

F(s) = -.-
. , - --
S"' 4s + IJ

Add and subtract 4 in the denominator to write it as (s + 2) 2 + 9. so that


• I
F(s)=----
(s + 2)'.! + 9

Table 17. I shows lhat L {sin a 1) = a/ (s 2 + a 2 ). so thal we use Equation 2 10 write

L- 1 {,.,
s-
1
+ 4s + 13
}=~e- 3
2
'sin]r

Example 3:
Find 1.hc inverse or


F(s)= - - - -
s+ I
2 - 4s 5 +
SOL u TIO N: We first write the denomi na1or as (.) - 2 )2 + I 10 gel

s +l
F(sj = - - --
( - 2) 2 + I

rr
17.2 The Inversion of Lapl;ice Trans<om,s 827
The form or the denominator suggests lhat we are going 10 have a result such
as F(x - 2). and so we write F(s) a~

~ (s - 2) + 3 s - 2 J
F(s) =- --- = -
(s - 2) 2 + I
--- + ---
(s - 2) 2 + I
-
( .r - 2) 2 + I

Referring 10 Table l 7.1 and using Equation 2, we have

... + I
£ -I {
- -- - } = t·-,,, cos 1 + 3e-,,, sin. r
s2 4s + 5

We can aJso use the second 1ransla1ion propeny to inven Laplace 1ransfonns
that cont.a.in factors of e-uJ . Recall 1ha1 this property says thal if (see Equations 5
through 9 of 1hc previous seer ion)

0.51 <a
(3)
a S. r

then

.C{g(t)j = J°" dr e-SI j(t - a)= rx dr e-rl H(t - a)f(r - a)= e-(1,t F(s) (4)
" k
Let's use Lhis resuh to dc1em1ine L- 11e- 2s/s 2 l. Equation 4 Lelis us that F(s) =
l/s 2 • so that (sec Table 17.1) f(t) = t, and

or g(t) = (1 - 2) H (t - 2). where H (:i:) is a unit step function. li"s easy 10 show
thal L{g(t)J:::: e-a,;.,.2-.

Example 4:
De1ermine

.(,-111 -eJ.• 1
s2

SOLUTION:

The first term gives ( - 1


{ ~ } = r. Using Equa1ion 4 we see that the second
gl r-
828 Chapter 17 / lnregral Transforms

rcrrn gives
f
3 -3.f) =(t-3)H(l-3)
L- 1 es
I 2

Therefore.

3
f (t) =I - (t - 3) H (t - 3)
Figure 17.4
1nc solulion 10 Example 4. which is plotlCd in Figure 17.4.
f(t) = t - (I - 3)H(t - )). plortcd
against,.

Let"s invert

A I e-.f
F(s) = -s2 - - --
s(I - rs )
(5)

The first t.enn gives,. but what abour Lhc second? Lei's c,cpand 1/( I - e-s) as a
gcomeuic series to obtain

I e- 1 l,-:h· e-lf
- ,--------+···
s· s s s

lnvening term by tcnn using Equation 4 g-ives

f(t) =t - H(I - I) - H(t - 2) - H(t - 3) + ···


)'
, O<t<I
I - I l<t<2 (6)
-
{ I -

and so on
2 2<1<3

This result is plotted in Figure 17.5, which shows that j(t) is a periodic sawtooth
X
function of period I.
Figure 17.S Jt may not be surprising 1ha1 the inverse of F(s) in Equation 5 is a periodic
The function /(r) = t -- H(I - I) - function if you remember Equation 18 of the previous sec1ion. Recall thal if
H(I - 2) - H(t - 3) + · · · ploued
against,.
JU)= f (t + T), lhen its La.place transform is given by

(7)

and so 1he denominator of Equarion 5 suggests that f (r) might be periodic. (Sec
Problem 26. however.)
17 .] Laplace Trdn~forms and Ordinary OiH1•r,•n1ial Equa1ions 833

each term and use Equation 2 10 obt.ain

s 2 Y(s) - sy0 - y 1 + 3sY(s) - 3yo + 2Y(s) = 0

Solving for Y(s) gives


Y(s) = syo + 3yo + Yi
s 2 + 3s
+2
syo + 3) 0 + J'1 1

=-----
(s + l)(s + 2)

We use partial fractions to write

SYo + 3yo + Yt = 2yo + Yi _ Yo+ Yi


(s + l)(s + 2) s + I s +2
Referring to Table 17.1, we sec that

This is the generaJ solu1ion 10 Equation 3. which could easily have been
obtained from Section I 1.3 (Problem I). Note Lhal we obtain the solu1jon with
the initial conditions built in. This is because of the nature of Equation 2, which
includes the initiul conditions. Note also thar the method works so smoothly
because Equation 3 has constant coefficients. It's not necessary that this be so.
but applications to ilifferentiaJ equations with non-constant coefficients mos1 often
rely on fortuitous cancellations of terms or special results. The real power and
convenience of the Laplace transform method is for non homogeneous differential
equations (usually with constant coefficients).

Example 1:
Solve

y''(t) + 3/(t) + 2y(r) = ,/


with y(O) = I and y'(O) = 0.
SOL u TI ON: Tiling the Laplace transfonn of both sides yields

2 ~ ,.. , I
s Y(s) - .~ + 3sY(s) - 3 + 2Y(s) = --
s - 1

Solve for Y(s):

~() I 3+~·
Y s =- --- -- - + -
(,,; - l)(s 2 + 3s + 2)
---
s2 +3s+2
I 7.J Lapl.lce Tr.m sform and Ordina,, Oifierc•11ti.il Equations 835
where we use<l 1he result of' Problem 10 of the previous ~1.:1ion. (Sec also
Problem 22.) No1c that in this case the displaccmcn1 incrca.~cs with time due
10 1he resonance condition ro = 1i~ 1•

The next Example shows how we can solve a nonhomogeneous equa1·ion in


tem1s of a convolution integral.

Example 3:
Solve

with y(O) = Yo and y'(O) = t'o•


SOLUTION: Take the Laplo.ce transform of bo1h sides and solve for Y(s):

The i nversc of Y(s) is

This is the general solution to the abo\'e differential equ.11ion. which you
could obtain using variation of parameters.

One tinaJ sel of problems that we shall consider before we finish this section
involves the bending of a beam or bar under an applied load. These sysrcms are
different from the others that we have dis.cussed because they are lks<.:tibed by
:.i founh-urdcr <liffcrc.:ntial equation. !he independent variahlc is a di~ta11c~. and
the boundary is not necessarily specified ar x = 0. Consider a uniform beam that Figure 17.8
A uniform beam suprxincd al its ends and
is slightly bent due to some applied load. a.,; shown in Figure 17.8. The theory of under an applil'.XI load.
elasticity tells us that if y(x) denotes the deflection of the beam from the horizontal.
then y (x) sat isfics

(4)

where y is a constant 1ha1 depends upon the material of rhe beam and i 1s shape and
w(.r) represents the distribution of the load along the beam. The typical boundary
836 Chapter 17 I Integral Transforms

figure 17.9
An illustra1ion of the typical boundary condi lions associ ate<l with the calculation of the de flccti on of a uniform beam.
(a) The ends arc supported: (b) the ends arc clamped. as imbeddcd in a suppon: and (c) one end is clamped and the
other end i..: free.

conditions arc (Figure 17 .9):


y

I. Simply supported, as resting on a support:


y = 0 and y" = 0 al the point.
2. Clamped end, as imbedded in a support:
X
y = =
0 and y' 0 at the point.
3. Free end:
y" = 0 and y 111 = 0 at the point.
Figure 17.10
The deflection of a uniform beam
For example, let's detennine the shape of a beam of length I 1hat is supported
supported at its two ends under a un1fonn at bothends and is under a uniform load -w0 acting downward in Figure 17.8.
load -w0 . Equation 4 is very simple in this case and yields

There are four boundary conditions to determine the four constants of integration:

y(O) = y(I) = 0 and y'1 (0) = y"(I) = 0

Figure 17.11 The resulting curve is


A beam of leng1h 2/ that is clamped al its
left end and free al its right end.
y(x) = - -Wo( - x J + 2c1 - x)
24y

Figure 17. IO shows this detleclion. This example is prelly simple and doesn · t
require the use or Laplace transfonns.
The following Example is a little more demanding.
2/ X

-I ,___ _ __
Example 4:
Consider the beam of length 2/ in Figure 17.11. The beam is clamped at iLs
Figure 1 7. 12 left end and its right end is free. Determine the resulting deflection of the
A plo1 of the load w(x) = beam under the load (see Figure 17 .12).
-w0 [H(x) - H(x - /)] thal
occurs in Ex.ample 4. w(.x) = -wo[H(x) - H(x -1))
17.J L1pl,11 e lr.m,fm111, .md Ord inary D1iiert·nli,1I tquations 837

SOLUTION: The boundary conditions are y(O) = 0, y'(O) = 0, /'(2/) = 0.


and y"'(2/) == 0. Take I.he Laplace transfom1 of Equ.Ltion 4 to obl:lin

-I - l ~ / ,, "' I-
-JJ_'
.,· Y(s) - s· y(O) - s-y (0) - .~y (0) - y (0) = __o - --
C _,.,

i' X

Now y(O) = r'(O) = O. but we are not given y"(O) or y"'(O) . Denote these
by a and /J. respectively.
Solving for Y(s) give!-

so tha1

r(x)
.
= -O'X:!
2
· -/j.x6 3 - -
24 y
Uo 4 -I
[x - (x -I) H(x -/)I

We can now dctennine a and {J by using _v"(2/) = y"'(2/) = 0. For/ < x :S 21.

1 3
\'(x)
.
= ax
- 2 ' -{:J.x6 - wo -t
- 1.r -
24 y
(x - I)
.i
I

This gives y''(2/) =a+ 2{31 - 3w0 11 /2y and y"'(2/) = /3 - u•0 1/y. s.o 1ha1

Wo/ 2
a = --- and
2y

Finally, then. lhe dctlection of the beam is given by


yy/,vo
wol2:r~ uiol.rJ wo " -I
y(x) = - - - + - - - -1.r - (x -/) H(:r -/)]
4y 6y 24y
X
This result is plotted in Figure 17 .13. A beam with one end clamped and the
other free i.-. calkd a ca111ilc1•cr.

This Example illustrates 1hal it is nor necessary for aJl the boundary condi- Figure 17.13
tions 10 be specified at x = 0 (or r = 0). Problems 16 through 20 involve other The deflection of the beam de:.cribed in
E.x:implc: 4.
calculations of the dcncction of beams.

1 7 .3 Problems

l. Solve Equation J using the method given in Section 11.3.

Use L.np/ace trrm4on11.'i to solve tht' following diff'ermtial equations:

2. y''(n + 4_v'(I) + 4y(I) = 9t·' with y(O) = y'(O) = 0.


3. y''(tj - 4y(t) = 5 sin 21 with y(O) = 0 and _r'(O) = I.
839

17 .4 Laplace Transforms and Partial Differential Equations

Laplace tr.rnsforms can be used to solve partial differential equations. As usual. the
partial differential equa1ions mus1 be linear wi1h cons1an1 coefficicnLS. but as we
saw in the previous chapter. 1his still provides us with a large number of physical
applica1ions.
As our first example, consider the onc-dimen:-ional heat equ::nion.

Osx
( I)
0 < I

over the semi-infinite re!!ion x ~ 0. This equation describe$ 1he conduc1ion of


he.:11 in a long rod whol>e larernl surface i~ insula1ed . Let 1hc rod be initially al
zero temperature and let the left end face of the rod (x ::::: 0) be kept .it a li:~.ed
temperature 11 0 . These two conditions translate in10

11(.r.0)=0 11(0. 1) = 11 0 (2)


We also impose the physical condition that 11(x. r) = 0 as x ➔
Let's 1ake the Laplace trnnsfonn of Equation I wi1h respect to the time
variable. Denoting .Cl 11 (x, 1) \ by {; (x, s). Lhe left side becomes

(3)

provided that it is le!!ilimate to interchange the orders of in1cgration and differ-


entiation. We know the conditions for doing Lhis from Section 1.8, but since we
don't know 11(x. 1) yet. it's difficult 10 .iccess it. Ncvcr1hcless. we would expect
any function thal describes the php,ical ~irua1ion of thi; temperature in a rod to
be suitably well behaved. In all ca~cs involving integral 1ransfom1s to solve partial
differential equations. we ,hould verify that our final result satisfie, both the partial
differential equation and its associ(lted boundary conditions and initial conditions.

Example 1:
Show explicitly 1ha1

where L is taken with respect to I.

SOLUTION:

a } 2sx
£.., { -i:l.r cos.r, . = (j-1 sinxtl = - ( 2 + x:! ) 2
840 C h,1prcr 17 / lntegr::i l Tt,rn~ffl mh

The Laplace transfonn of the righr side of Equation I gives

au } , ,
£. { a, =sU(s.x)-11(x.0)=sU(s.,t) (4)

Finally. the Laplace trclllsfonn of the boundary condition u (D. 1) = 11 0 gives


, llo
.C{u(O. 1)) = U(O. s) = - (5)
,\'

Equarions 3 and 4 give us

2
d {J (x. s)
---- - -
s u' <X S)= 0 (6)
dx"!. a2 •

We have written the derivative of U(x. s) as an ordinary derivat..ive because we


considers to be a fixed constant at this point.
Notice that tJ1c time variable has been eliminated (transfonned oul} in Equa-
tion I. leaving an equation in just one independent variable. Generally, applying
a uansform to a differential equation will reduce the number of independent vari-
ables by one. In the case of an ordinary differential equation. the result is an
algebraic equation.
The solution to Equation 6 is

(7)

where we recognize that the two "constants of integration .. may depend upon s.
The condition that 11(x. t) - 0 as x ➔ oo forces us to choose c 1(,~) = 0. Setting
.r = 0 and using Equation 5 yields

Therefore, we have

U(s.x) = ttoe- s1 12/ a (8)


s

3.'- lhe Laplace transform of our solution. The inverse or e-x.,.ir:/ c, is


1 4
xe-x / oi, /2(rra 2 t 3 ) 112 (see Problem 2-22), and so (Problem 2-20)

We can ex.press thi$ result in terms of a complementary error func1ion by letting


:2= x1 i4a2y:
17.4 Laplace Transforms and Partial Differen1ial Equations 841

(9) u/110

= u0 { I - erff(x /(4<X 2 t) 112 1}

= u 0 erfc[(x /(4a 2t) 112 ] (10)

Equation IO is ploned against x for various values oft in Figure 17 .14. No1e Lha1 X
the temperncure approaches "o for any finite value of x but that u (x, t) ➔ 0 as Figure 17.14
X-+ 00. The plot of u(x. r) = u0 erfcl.r/(4a 2t) 1f2]
Let"s now verify th.a1 Equarion IO is indeed a solution 10 Equation I. Using against x for various val uc.s of t .
Leibnitz's rule (Problem 2) with Equation 9, we see that

and 1haI

ou = Uo-X e-xi ;4a1,


a, (4rra213)1/2

=
so that Equation I is satisfied. Furthermore. 11(.x. 0) erfc(oo) = 0. u(O, r) =
=
u 0 erfc(O) u 0 , and 1'(.r. t) ➔ 0 as x ➔ 0. Thus. we sec 1ha1 Equation 10 satisfies
Equation I and its associated boundary conditions and initial conditions. Having
done this once. the verification of the subsequent soluIions will be lert as exercises.
Before we do some other examples. le1·.s consider raking 1he Laplace IJ"ansform
with respect LO the x variable in Equation 1. No1e that if we were to do that, we
would need 10 have both u(O, t) and llx(O. 1) specified. So even 1hough O ~ x < oo
in this case, we cannot transform with respect to x.
There are really no new principles involved here. and Ihe best way to illustrate
the application of Laplace tranfonm 10 solving partial differential equations is by
example. Consequently, the rest of this section will consist of various Examples.
The next Example specifies I.he flux of heat rather than I.he Iempcraturc at x = 0.

Example 2:
Solve the he.al equation for a semi-infinite rod whose l:11eral surface is
insulated.. whose initial temperature is zero. and wilh a cons1an1 heal flux
mainIaincd at its left-end face.

SOLUTION: The problem to be solved is

- 1= - - X > 0, f > 0
ax 2 a a,
with u(x. 0) = 0. -K11_r(O. t) = f3o. and u (x. t) =0 a,(; x - oo. Take the
842 Ch,,p!L'r 1 7 / ln!<'i:r,ll Tr.m~fmms

Laplnce transform with respect to lime 10 obtain


2 ~ . .
dU:c2, ) - ~ 2 U-(.\,S)-
· -0
dx a

with -,dl.c<O. s) = {30 /s and U(:r. s) = 0 as x - oo. Solving for U(x. s)


gives

•(
U x.s
)
= -KS/3oa3/ '>(!
-
- x 1121.,

Now. according to Problem 2-24,

and so (Problem 2-20)

rrud

where we let z = x/(4a 2 y) 1l 2 . We can ex.press this result in tenns of the


error function by integrating by parts. letting ·•11" = e- -~ and ''du"= d-/:. 2•
10 get (see alw Problem :U.16)

X
This solution is plotted in Figure 17.15. Problem 3 asks you 10 show lh.ll this
Figure 17.1 S resull satisfies the above partial differential equation and all irs conditions.
The solution gi\Jcn in Eitample 2 plollLxl
again~•.'< for various vnlue.~ of 4a 2r.
848 Chapter 17 / lnh ~~rJI Tr, m~f,~rms

SOLUTION: Using k a" the transform variable. we have

The inverse of F(k} is

f(x) = -I loc -
dk e - a k,2 I -, cos kx
· 2rr -oo

= I e-.c-1/ 211!
(2rra 2) 1 -

Problems I through 6 involve finding the Fourier transform of some elementary


functions.
-2 2 x or k Example I !ells us that the Fourier transform of a Gaussian distribution is
a Gaussian distribution. In addi1ion. Lhe widths of the 1wo normalized distribu-
figure 17.19 lions are reciprocals; f (x) gets narrower and F(k) gels wider as a decrea~s
The Fourier tr..in._form pair (a)
f{x) = (.:!.rro~)- I /~ cxp(- x 2 / 2o 2 ) :md (Figure 17.19). This reciprocal relritionship is an illu~tration of the uncertainty
(b) F(J.:) =-= (n 2/2,r) l/2 cxp(-u 2J.- 2/2).

Table 17.2
Some Fourier transform pairs. Ln aJI cases. a > 0.

/(x) F(k) /(x) F(kJ

(2;r) 112~ (k) 1<s:gn(x) - (~ y/2 ~


1T J.:

(2n) 1128(k - kn) e - iko r (2n) 1f!8(k + ko)

(;~ )''2 k2: a2


xe - al.r l _ ( ~)
:r
111
2tiik
(k2 a2)2

1/2
I k. 2/~a-
~ I/. '>- e 1/x -i ( ~) sgn (k)
(2a-)

( ..!!_ ) 1/2 t. - alk l


I
2"2 .
•· H(x) (~)l/28(k) {2.Jr) 1/ 2k

'" i-i.,num x = - I for x < 0 und I for x > 0


n: unit step fonccion = 0 for x O,U1d I for .r ;,- 0
Nore 1ha1 H( rl--= }1 1 + :;gn (. ) J.
850 C I I,1p11•r I 7 / lnr grill r;insfo,ms

Example 3:
Find uhe Fourier l'ran sfonn of J(:r:) = S(x - T"0 ).

SOil u TIO N: Us,ing Equa1jon 9 formally. we have

We say '°fonnallr}'°. because S(.r - x 0) is not piecewise smooth, but


nc'-'c-nhel • s the resu1h is still valid tand useful).
If we im·crt F(k) using Equal.ion 9. we have

(I I)

which is one of the most u~cful dcfini1ions of a ddta function.

There arc some general propcnies of Fourier 1ransforms that are useful in
applications. Two of these are the so-called sh1:fri11R properties:

( 12)

.'.Tk'''\"f(.r)}:::: F(k - a) ( 13)

The proofs of Lhese are fairly s1raightforward (Problems 8 and 9).

Example 4:
Find the inverse of

SOLUTION: Table 17.2 shows thnl

Using 1hi.c: resull and Equation 12 give.'-

An important property for rhe npplication of Fourier transforms 10 the solution


of boundary value problems is 1hc derivative pmperry. It's easy to ~ee using

m al
17.5 rouriPr Tr.m,in:111~ 851

integraiion by parts thnt

( 14)

if j(x) and aJI its derivatives vanish at infinity. Note tha1 this fonnula differs from
1he corresponding relation for Laplace transforms in that no initial conditions arc
required. We'll see in the next section 1ha1 Fourier transforms are particularly
appropriate if the independent variable bci ng ~Ii mi natcd ranges from -oo 10 +oo.
There is also a fonnula involving the derivative of a Fourier transform . If j(x)
is piecewise smooth and absolutely integrable. th~n

( 15)

This formula follows easily from Equation 8.

Example 5:
Use Equation 15 10 verify 1he entry for x e-a l rl in Table 17.2.

SOLUTION:

J =t ·(2)''~(
-
;r
- .,
'liJk)
(k-+a - )-
' } = - -rr (2) 1
/!. ,2.tiik ) ,
(k-+a -) -

There is a convolution theorem involving Fourier transforms. We define the


convolution of f(.r) and g(x) by

f * f? = J_'o.; f(11)g(x -
X
11)d11 = 1_:J,J f (x - u)g(u)du ( 16)

The Fouril!r lr.tnsfonn off 1· ~ is

' -., J".\:)


:Tlf *.id=--,_ .
dx e - ,b f'X du f(x - u)g(u)
(2.tr) - - oc . C\..

= - -I - 1·-x. d11 ,c:(u) 1C\:i dx e -i.,.


~- f(x - 1.1)
(2,r) 1/2 _ -00

Now lei:=.,· - 11 10 write


852 Ch,ipt(•r 17 / Integral Transforms

or

(I 8)

You may have noticed that the convolu1ion integral in Equatjon 16 differs from
the one lhal we defined in Section 2 in that 1hc limits in Equa1ion 16 nrc -oo 10 oo
instead of O to t as in Equation 17 .2.12. l.f f (x) and g (x) are equal to zero when
x < 0, however, the two integrals are the same.
We frequently use Fourier !Tansforms in three dimensions:

-00

where k · r = k_,x + kyY + k,J•· The inverse transfom1, in a more common noration.
is

.f(r) = _1_1_2 [{[ F(k)eik-rdk (20)


(2,r)-: 111
where f(r) denotes f(x. y. z). f'(k) denotes F(kx- ky, k.). and dk = dkxdk\.dkz..
Suppose now Lha1 f (r) depends on.ly upon lr I = (x 2 +· y 2 + z2 ) 112 • as is co~only
the case. To evaluate F(k) when f (r) = /(lrl). introduce spherical coordinates
into Equal.ion 19 and wri1e

F(k) =- I ,,
(271')·31~
10
00
dr r 2 1" d0
0
sin 0 12..'I' d<J>
0
f(r)e-lk-r (21)

Choose the z axis (the polar axis) of our spherical coordinate system to point along
k.. so 1ha1 k - r = kr cos 0. We can then integrate over 0 and <I> in Equation 21 lo
gel (Problem 15)

, = (2)
F(k) - 1 rr
1
'
2

u
00

j ( rrsinkr
) - - dr
k
(22)

The inverse formula is

J(r) =(~
.,) 1/2
Jot'° F(k) k s1;
.· k
r dk (23)

Notice that F(k) = F(lkl) = F(k) when J (r) = /(lrl) = j(r).

Example 6:
De1em1ine the Fourier transform of

f(r)
z3
= -e-2Zr
!T
853

given that r is I.he rndial coordinate of a spherical coordinate system.

SOL u TIO N: We use Equation 22 :

A
F(k) = ( -2 ) 111 z3
- loo re--'2 r sin kr Jr
rr rrk <>

As a tinal topic in this section, we shall derive Parscva!"s theorem for Fourier
transforms (see Section 15.3 for Parseval's theorem for Fourier serie:-.). Start with

where we huve allowed f (I) 10 be complex for generality. ll!sc Equation 9 to wrire

= 1-: .r:
dw du/ fr • (w)F(w )8(w
1 1
- w)

= 1_: dwF .. (w)F(w) = 1_: dw 1F(w)l


2
(24)

We used Equation 11 in going from the second line to the thi.rd line.
Equation 24 is known as Parseva/'s theorem. We could have derived Equa-
tion 24 by s1ar1ing with the convolution theorem (Problem 20). but the above
derivation illus1ra1es the manipulative utility of t.hc delta function. The following
Example is a nice application of Parseval's theorem.

Example 7:
Let the electric field in a radiarcd wave be described by

0
E(r) = { e _t/ r sm. 0WO' I<
I> 0

U!;e Purscval's theorem to find lhe power radiart=d in the frequency interval
(u>, uJ + dw) .
854 h,1p1er I 7 I 1111 sra l r, n fom,s

SOLUTION: The Fourier transfonn of £ (f) is

The to1al power rn<lia1etl is given by

Using I.he fact that the second lcrrn in IE'( w)I will dominate for 1,J .> 0, we
ha\'e

... ., I dw
IE(w)l-dw~ - - - - - - -
8,r ., I
(w - wo)· + -
r :?

This frequency spectrum is ., huwn in Figure 17.21 .


When o) = (/)0 ± 1/r. the height of the spectnim falls off by a fr1ctor of~.
as shown in the figure. Therefore. the width at half power is £iven by 2/r .
figure 17.21
This result is reminiscent of Ihc unccrta.inIy principle. In this ca.sc, we have
The frcque!ll')' spectrum a...soci:iti:d with
lhe rJdialed WLl\'C d~M.:ribed in, example 7 that the width of a frequency ,pcctrum varies im·l·r~cly :1~ the <.luration of the
=
for wi> 20 um.I r =
l. signal. The finite wave train Ji,cussed in Exsmple 2 is anolh~•r l.·xample of
this version of the unccnainry principle.

l 7.5 Problems
J. Find Ihe Fourier 1ransfonn of f(r) = lj(,2. + 02.).
. 1rans fom1 o ff (f) =
2• F.1n d l I1e Founer { I -a -< 1. ---~ 11 . D'1.,cu.~s l he rec1proca
. . t)C[W<.!cn t he wic' II h o/.
I re Ianon
0 othcrwr,~:
f(r) .Jnd F(w).
3. Find I.he Fourier 1ran.sfom1 of u,- 1 111. a > 0.
4. Find the Fouril!r lr.1nsfom1 of 1/x. Hint: Be sure 10 use 1he fact 1hat 1/x is an odd function and proceed formally.
112
5. Use 1he result of 1he previous problem to show that .7/sgn (xJI = -i (~) ~-
rr k

6. u.~e the resuh of1hi: previous problem 10 show that :J{H(.r)I = ( ~)


112
[,5(k) - ;kl where. H(x) is 1he unit
srcp function. Hin1: l'.xprcss H (x) in terms of "-:,?n (x) .
17.6 Fourier Tr.1rhl11rms and Partial Diff r -mial Equations 857

ilie Fourier transfonn of Equation 2 yields

dC(k, I) - - DJ.2C(k. t) (3)


dr
with the initial condition
00
C(k, 0) = (2rr) I,'~ / e-ihc(x. O)dx = (2rr)- 112 c0 e-ik.ro (4)
-oo

The solution to Equa1ion 3 is

(5)
C
We can determine c(x, r) from Lhe formula

c(x, I)= (h)- 112 1-: eikxC(k, l)dk

-co
= 2rr Joo e ik x - ole -J:~r>,d'·" X
-N

= Co e-lx-.r( 1)! /4D1


(6)
( 4rr DI) 112

Figure 17.22 shows plo1s of c(x, I) for various vaJues of/. Notice how Lhe diffusing
species spreads out as time iocreiL,es.
II ·s aJso easy to solve Equation 2 for a general initial condition c(.r. 0) = J (x). Figure 17 .22
The fundomcnlnl solu1.ion of lhc one-
dimensional diffu ion cquatinn plL'ltcd
against x for increasing \'alue, of Dr.
hample 1:
Solve Equal.ion 2 wi1h 1hc initial condition c(x. 0) = /(x).
SOLUTION: Lei C(k. 0) = F(k). Then Equation 5 changes 10

We use the Fourier transfonn convolution theorem 10 write

(,·(x . I) = I 1/1
(4rrDr) -
f j (11)e-1 .,-u1= .uJ, du (7)

Nole 1ha1 Equation 7 is the convolu1ion of c(x. 0) with Equation 6. We can


give a nice physical interpretation of Equation 7. Picture f (11) a.s consisting of a
sum of delta functions spread along Lhe x axis . Then Equation 7 represents the sum
of 1hc rcsuhs of each de Ila function at the point x . Because of this. Equation 6 i~
caJled thcfundamenral solution of the one-dimensionaJ diffusion equation over an
infinite interval. It is also the Green's function for this system.
860 Chapter I i I Integra l Tr.an!5forrn.s

= -I Joo dk ih
e cos kut Joe dx ,e - ik 1
,
f(x)
2,r -00 -oo

=~
2
f
-oc
00 dx' f(.r')[8(x - :r' + ut) + 8(x - :r' - vt)]

I
= -lf(.x - ut) + J(x + ut)I
2

We used Equal.ion 11 of the previous secrion in going from the third line
10the founh line. Thus we obtain t:he D'Alemben solution of the wave
equation. Problem 13 has you derive the more general re.--ult tha! you get
when u,(x. 0) #: 0.

Recall that a Dirichlet problem consists of solving Laplacc·s equation wi1h


Lhe potential function specified on the boundary. The next Example illustrates the
use of Fourier transfonns to solve a Dirichlet problem.

Example 4:
Determine the po1en1ial function 11(:c. y) that sa1isflcs

-'-<X<OC
0 .- _\"

with 11(.r. 0) = .f (.r) for -oo < x < oo and 11 (r) - 0 as r -i- oo, where
r = (x 2 + y 2) l/2. Give a physical interpretaLion of I.his problem.
SOLUTION: This problem might rcpre~en1 I.he steady temperature
c.Jis1ribu1ion in a large lhin rectangular sheet. wi1h i1s lateral surfaces insulated
and the temperature spec.:itied by Jlx) along one edge.
We take the Fourier transform wilh respect to x to obtain

wilh {; (k. 0) =
F(k) and{; (k. y) ➔ 0 as y ➔ oo. The solution that s.ali~fics
these condjtions is

0::y<OO

The inverse of e-lkly is

O~y<oo
17.6 Fourier Tr,1minrn,~ ,md P.irti;,I D1ff(•rl'nli.1t l:qu,1tions 861

and so

This equation is well known in elecLrostatics and is called PoiJ·so,1 'x i11teKral
fomwla for the half-plane.

As a final Example in this section, we shal I dc1crrninc the fundamental solution


of I.he 1wo-dimcnsional isotropic diffusion equation.

Example 5:
We wish to sol Ye

(14)

with c(x . y . 0) = c0i5 (x - x 0 )S ( y - y0 ) and c(r. r) - 0 as r - oo.

SOLUTION: Firs! nolc that

=- I
2:rr u
1~ drrc(r,t) 12.-r ,,- ilrro,o dl)
0

The integral over B is 2,r J0 (k r) (see Problem 12.6. 9). and so

C(k. I)= 1;.v dr r J 0 (kr)c(r. I)

The inwrse of C(k. 1) i~ (Problem 15)

c(r. I)= fo.x- dk klo(kr)C(k, I) ( 16)

Tak..ing the Fourier Lninsfonn of Equation 14 givci;

dC = -k"!.DC
dt

with C(r, 0) = r0._t,- /k-ro_ The solution 10 this equation is


2rr

C(k . {) = .:Q. ,-ik•roe-k!D,


1
2rr
17.7 The Inversion Fom1ula fo, Laplace Transforms 865

17 .7 The Inversion Formula for Laplace Transforms

Up to this point we haven't presented an explicit fonnula for the inversion of


Laplace transforms, although we were able Lo inven many of them using tables and
special techniques such as panial fractions. When we derived the Fourier integral
theorem, on 1he other hand, we were led directly lo a Fourier transfom1 pair, which
gave us explicit integral fonnulas to go from J (x) and F(k) and back. l.n this brief
section. we shall derive an integral formula for the inverse of a Laplace transform.
We'll see that ii involves integration in the complex plane. a topic 1hn1 we have not
considered before.
Recal I Lhc Fourier integral theorem, which says I.hat if / (/) is everywhere
piecewise smooth and absolutely integrable. then

11(1) = -I Joo dw Joo dz ."<r-::)11(z)


e11 (I)
2rr -oo -

(2)

Equation 2 leads directly to the Fourier transform palr

(3)

and

(4)

Now let's consider the function

g(r) = e-cr j(r)H(r) (5)

where c is a positive real constant. j(t) is piecewise smooth for, ::: 0, and H(I)
is Lhe unit step funclioa. We assume that g (I) is absolutely in1cgrah!t.:, and so f (1)
must be such I.hat

(6)

The function g(t) satisfies lhc condi1ions of the Fourier integral theorem. so we
can write (Equation 2)

g(t) = -I
2rr
loo . foe
-OC,
dp e'P1
-0<.•
.
dx e-,p.r g(x) (7)

or. using Equation 5 in both sides of Equation 7.


CHAPTER 18
Functions of a Complex Variable: Theory

We discussed complex numbers and runctions of a complex va.iiable in Chaplcr 4.


and we hav,c seen sever.ii examples where functions defined in tem,s of the complex
var,iab'le z. = x .• iy somehow underlie the properties of fonctions f (x) of a real
variable. For e;xample. the power series

l , ~ n
f(x) = --, = 1-.x-+x -x + · -- lxl < I
I+ x-
ha.~ an interva1t of convergence - I < x < I even though Lhe left side appears lo be
a perfectly well-behuvcd function of x for all values of x. As we pointed out in
Chapter 4. the reason t.ha1 x is restricted 10 the interval -1 < x < I is that J (x).
considered as a function of a complex variable

I
j(z.)= -, - .,
+ --
has singularities at::= ±i. and .so z. is restricLed to lie within a unit circle centercd
at the origin. Thus, x. the real pan of z.. is restricted to the interval -1 < x < I.
Another hint that functions of a complex variable underlie functions of a real
variable occurred in Seclion 12.2 where we saw 1hal the radius of convergence of
a power series solution 10 a diffcrcnrial equarion about t.hc point x 0 is at least as
great a.s Lhe distance from .r0 to the nearest singular point (real or complex) of the
differential equa1ion .
It turns out that the analysi~ of functions of a complex variable is one of the
riches! areas of upplied mathematics. In this chapter, we shall learn about the
general properties of functions of a complex vuriable. Some of these propertjes
arc really remarkable. We'll see thal if/(:) has a first derivative in some region of
the complex plane, then all iL<; de1ivati cs exis1 there. Furthermore. we·11 see that
if j(z) is dillerentiable on a simple closed curve in the complex plane. lhen Lhe
values of J(z) throughout lhe region enclosed by Lhe curve nre de1ermincd by Lhe
vaJues of f{:.J on the curve.
In Section I. we'll define limits and continuity of fwictions of a complex
variable. which will lead to the definition of Lhe derivative of J(;:.) in Sect.ion 2.
Then we'll learn two important (and easy 10 use) in1egrnl t..heorems in Sections 3
and 4. In Section 5. we'll .study Taylor series of functions of a complex variable 869
870
and then Lauren! series. which are series of the fonn

Notic:e that a Laurent series is a series in bolh ascending and de.-;cending powers
of;: - a. Finally. in Sec:1ion 6. we·11 learn what i-. callt·d Lhe residue theorem,
which lead .... lO a powerful method for evaluating not only inLegrals of functions
of a complex variable. but for evalu..1ting integmls of functions of real variabk·-. ;1s
well.

18.1 Functions, Limits, and Continuity


Let;:= x + iy. where.randy urc real. represent a complex variable. If we have
a prcscrip1ion for assigning one or more valui:s of ano1hcr complex variable w to
each va 1ue of:::. then \\"L' ,ay I hnt JJJ is n fu nc1 i0n of:: and wri1c w = J (: \. Becuusc
ui is compkx nnd depends upon x and y through :. we often write t,_, = J (:) as
U1 = f(,.) = ll(X. y) +i v(x . .}') (I)

where 11(x. y) and 11(.r. y) arc rcill functions of x and y. If only one value of w
corresponds to each value of.::. 1hcn 11 1 = j(':.) is called a single-valued func1ion
of :. If rnore than one value of 111 = J (:) corrc.,;;ponds 10 ellch val uc of : . then
u· = /(:)is called a muhiple-valucd or multivalued function of:. A inuhi\'alued
func1ion can be considered to be a colleclion of single-valued funclions. For
example. consider Lhe 1wo-valucd function J(: ) =
-'!-_ We c<.1n see the nature
of 1hc 1wo values off(;.) by expressing ii in polar fonn.

where r 1I - 0. Now. when H = O. u = , 112 • As fJ increases from tJ = O in a


counlcrclockwi~ direction. 11• w1rics ;:is rt /'!. e' 12. After a complete circuit fJ = 2rr
and 1t 1 = , 1 J.e,·' = -, 111 .
Wi.: i.:an make f (::.) = ~ t(!. single-valued hy in1roducing a hrnnch cut into the:
plunc as we did in Figure 4 . 26. and as shown af!ain in Figure I 8.1 . The mu l1ivalued
X
function J(:) = -112 is separated into two single-valued func1ions by the branch
cur along the positive x axis. When cru1-ses the br.rnch cul. we go from one
f}

(single-valued) l,mnch of J(::.) to ano1hcr. For one branch. 0 :5 0 2rr. and for
the other. 2n O < 4rr. Note that /·) < 2;r on 1hc lir,;;1 branch and that 0 < 4rr on
the second brnnch. Cmssing the brnnch cul as 0 goes 1hrough 2;r is like going from
Figure 18.1
An illustrarinn of a b,,rndt wt along
u• = l:Y ~I = r 11~ to w= -1 - 1121 =
- r 11 . As (I goes through 4n. ,w :-imply go
the po •irivc .t ::n:is h•r 1hc f1nu;;1ian from the second branch bad 10 lhe lirst bmnch.
(:) = -'1'' . We saw in Sccr ,i oo 4.5 1hat ln .- is a multivalued function (an infintly of ~ingle-
1

valued functions in this case). The following Example reviews the multivalued
nature of In:.
18. I Fun lions, Limits, iHi d Curtlinuit , 871

Example 1:
DeIennine Lhe branches of the logarithmic function. w = f(· ) - In: .

SOL u TI ON: Express i: in polar form . .:= nrl1• Then ln:: can be wrinen as

In::.= In r + if!

But ,/ = ,,; o in ,,i. where 00 • which lie$ in the interval [0, 2rr ), is coiled the
principal argumcnl of 2:. Arg z.. and 11 = 0. ± I. ±2 . ... . Thus.

In.:::= In r + i(00 + 2rr11) II= 0. ±1. ±2 . . ..

and we sec that In:: ha!>'. an innnire number or branches. If we restrict 0 10


the interval I0. 2rr ), then we write

Ln ~ = In r + i00 = In r + i Arg ::
where Arg :: = 00 denotes the pri11cipol arg11111t·111 of :: and Ln ::. is the
principal bro11d1 of In:: . Thu~. we cun also write
)'
In::= Ln::: + 2rrni 11=0,±1,±2... .

Figure 18.2 shows a brnnch cul along the positive x axis for f(:) = ln : . As X
in Ihe case of f (z.) = .:Y 2, rhc angle 0 is equal 10 zero on the upper pan of the
branch cur. and 0 increa..~s in a countcrclockwisc direction until ii reaches Ihe
lower pan of the branch cur. where it approachc$ but never equals 2;r . Each time 0
cms~cs the branch cu1. we move from one branch orln;: to anoIhcr brJnch . Unlike
for f(:) = : 1 ·~. however. Ihere is an infiniIe number of branches for/(:)= In;:
because we go from one branch (wi1h n increasing by one unit) 10 another as 0 Figure 18.2
A brunch cut along the pos itive .\ ::i.,i.-.
increase. The origin in Figure I 8.2 is also excluded because z. can never equal for ( • J = In - and a branch poi m al Lhl·
zero. The origin in Ihi~ case is called a branc/r point. With the idea of branches of origin.
funcrions at our dispo~al. we shall always consider a function LO be single-valued.
We ci.111 defi n~ the I imit of a funct..ion of a complex variable much us we Jcfim:d
the limit of a function of a real variable in Chapter I. First, we must define a 8
neigli/Jorhood in Lhe complex plane. The neighborhood of a point :c: 0 i~ the set
or all points such that lz - :ol $. where S is a positive cons1an1. A deleted 8
neighborhood of:: is the set of all points such t.ha1 0 < I=: - : 0 1 < 8. Ln other words.
it's a 8 neighborhood of : 0. ex cl1udi1ng the poi nt : 0 i1sdf. We now say 1hal Lhe limir
or a func1ion /(:) defined in a J ncighborhood of ::0 is equal 10 I as: approaches
:o if I/ L::) - ·/I < f. whenever O I:: - : ul .,._ "· We write this as

lim / (:)
~- · ·o
=/ (2)
872 Chapter I 8 / Functions of a Complex Variable: Theory

• y V

Figure 18.3
An illustration of the limit of a functi on / (:) in the complex plane. (a) The point ;: lies within a
a ncighborhood of •0 . (b) The point w = /(:.)
li<..-s within an c 111:ighborhoo<l of/.

Figure 18.3 illLL5trate.'i the idea of a limit in lhe complex plane. As the figure
implies. the limit in Equation 2 must be independent of the direction in which z
=
approaches z0 . Consider the limit of j(z) z~ /z. a<;;:-+ 0. We can let z.---+- 0 along
the x axis by setting _v equal 10 zero and then letting x -+ 0. in which case we get

lim :• = lim ::, =I


:-·O ;: x-+() X

We can aJso take the limit along they axis by setting x equal to zero first:

lim ::_
~· = lim ---
- i,· = - I
:-u z y-0 iy

We obtain different values for the limit depending upon how we calculate it. so we
say Jim z"" /z. as z-+ 0 does not exist. (See also Problem 19.)
Irwe write /(z) = u(x. y) + iv(x. y) and ::0 = x 0 + i_,·0 • then lim f(z) exists
;: ·o
if and only if u (x, y) and v(x. y) have limirs as (x. y_) ➔ (x 0 • y 0 ). In this case.

Jim j(z) == lim 11(.x. y) +i lim v(x. y) (3)


~- • 4i (.,·. y)--. (xo, )'c,) (x .y)-· (xo ,Yo )

Once again. the two limit$ in Equation 3 must be independent of the direction in
which (x. y) approaches (x0 • Yo).

Example 2:
Does the following limh cxi "t?

• ( --
hm I -
z-0 l+ el / x
+ "··
- ')
.
18. 1 h111 -lions, Umil.s, and Continuity 873

SO H JT ION: Certainly the limit of v(x. y) = y-' exish as (x. y) - (0, 0).
Woat about the limit of u(x, y) = 1/(1 + e 1f.')? Well.

Hm el / r = oo n.nd lim el/x =0


x-0+ x-0-

so J,i m u(x, y) depends upon Lhc direc1ion in which x - 0. Therefore. 1he


.t ~90
limit does no1 exist.

li's easy 10 slilow that if / (z) -• o and g(t.) ➔ b as z----,. z: 0 , then /(z) ±
g(z) ➔ a± b, f (z)g(z) ➔ ab, and J(z.)/g(z) ➔ a/bas z ➔ z0 • provided 1hat
b r= 0.
Now rhat we have defined a limit in rhe complex plane. we can define conti-
nuity. Let f (l) be defined in some S neighborhood of ;:0. Then f (z.) is continuous
al z0 provided that / (z) ....,. J (z.0) ns z - z0. This statement is equivalent to

I. llim J(z)
~-~o
= I must ex.ist.
2.. /(: 0) must be defined at z0 .
3. J(zo) = I.
A function is continuous in a region R if it is continuous at each point of R.
As with limirs. if f (z.) = u(x. y) + i u(x. y) and z0 = x 0 + iy0 • 1hen f (z) is
continuous at z0 if and only if u(x. y) and u(x. y) are continuous at (x 0 , y 0). Fur-
rhennore. if f (z) and g(;::) are continuous at z0 • then so is f(z) ± g(z.), f (z)g(z).
and f(z)/g(z.), provided that g(z 0 )-:/= 0.

Example 3:
Show th:u e 1·: is continuous everywhere.

SOLUTION: Write e;~ as

=e- >(cosx + i sin .r)


801h ,:-.,- cos x ond i: - -" s in x are continuous everywhere, nnd so is ei~.

Example 4:
Determine the points where /(:) = co1 z: is discontinuous.
SOLUTION: First \Vrite
cos ;::
J(:?.) = cot z = -.-
.m z

C gl
874 Chap1er 11:1 / Fun lion of a ompl ' · Variable: Th ry

Example 3 shows that ei: (and e-i~) are continuous everywhere. Conse-
quently cosz. = (e'~ + e-i~)/2 o.nd sin;:= ( e1l - e- i~ )/2i arc continuous
everywhere. Therefore. cot ;: is conLinuous everywhere excepl where
sin z = 0. or at the poinrs 0. ±JT. ±2rr, ... (Problem 14).

18.1 Problems
The first nine problems consriru/t' a review of some nf the topic.~ rhal we diRussed in Chapter .J.

1. Show that I~: -+'~ I I when Im:> 0.

2. Ex press eio ~ in I he form " + i t•. Assume that a is real.

3. Describe Ihe upper half of the disk (x - x 0 J2 + (y - y0) 2 _ R1 in terms of;:.


4. Detenninc all the possible argumems of : 1• z2 • :: 1.:2, and :. 1/:~ if : 1 = I+ i and z2 =I- i.

5. Show that .:IJ1? :- 1:-:~ = arg Z1 + a.rg Z2, bu11ha1 Arg :-1:2 :f. Arg Z1 + Arg :::2.
6. Show that sin i z = i sinh z..
7. Show that sin;:= sin x co~h y + i cos x sinh y.
8. Show that sin 2 z. + cos2 : = I.
9. Detcnnine the values of In(! + i).
10. Let w = j ( z.) :; ;: 3. Determine the value off (2 + i).
1J. Show 1hut / (;:.) = z. 112 is mul!ivalucd by dtLCrmining 11(x. y) and v(x. y) explicitly.

.
12• D e1ennmc 1·1m ( x-, + y·1 +,sin-·
. . 1T \') .
:-1+2l 2

13. Determine lim - "- . Him: Let:= 1/IJ and then let 11 -·• 0.
• - :,o I + iz.

14. Show that the zeros of sin;: are all real and equal 10 1111, where n = 0. ±I. ±2, ....
15. Ex.press /(z) = z. 2 + (z") 2 in the form w = + i11.11

16. Is Arg;: single-valued? ls arg z.?

17. Determine lim (xe·n- + i e'-'" ) .


l-,.i X +I
Re (:)
18. Where is the function f (z)


=
I ~ con1inuous?

19. Show Iha1 lim .:::.... does not exis1 by Laking Ihc limiI along the ray y = mx. where m is a conslanl.
:-0 l

z J"Jrrd
20. Show tha1
.
ilm
~-lrrJ2
.I
: cash -
3
=- -.
32
18.2 Oi fferenri._11ion,: Th" a11r hy-Riem;rn11 Equationi; 875
18.2 Differentiation: The Cauchy-Riemann Equations
The definition of the deriva1ive of a runction or a complex variable is similar Lo
t.hc one we u$e for functions of a real variable. We define the derivative of J (z) at
some poinr z. 0 by

(I)

if the limit exist~- Clearly./(;:) mus1 be defined in a neighborhood of z = z0 . For


the limi1 in Equation I to exist, it must be independent of the manner in which ;:
approaches :o-
For example. if f(z.) = 1/z. then
f'(z.o) = lim l/z. - l/zo = lim Zo - ;.: = --
: ;:o :: - zo =""" .:n z;::o(z - zo)

Therefore. f'(z) exis1s for all values of z cxccpl for:= 0.

Example 1:
For what values of::= to does/'(::) exist if J (~ l = 2x + 3iy?
SOLUTION:

' .. [2(.r - x0 )
f (z) = 11m .
+ 3i. (y - y0 )]
= ~• (x - xo) + , (y - Yo)
= lim [2(x -xo)2 + 3(y - to 2+ i(.r-,xoHY-Yo)]
:-;.:o (x - xo)" + (y - Yo>-

Let's look ul the limit of the real pan of the express.ion i,n brackets .

If we [cl x x0 first. then the limit is equal to 3. If we kl y - y0 lirs1, then


the limit i~ equal to 2. Therefore. the limit does not exist and /(z) 2r + 3i y =
is differenlinble nowhere.

It turns out thai u(x. y) and v(x. y) must satisfy cenain relation$ for J(z.) =
11 (x.y) + i v(x, y) 10 be differentiable. These relal.ions are known as 1he Ca11cl,y-
Rieman11 eq11nrio11s, and are ca-.y 10 dt!rivc. Let's evaluate j'(: 11 ) by letting y ➔ Yo
fiNt and then le11.ing x ➔ x 0 . In this case
878 Ch.J IJI ·r 18 / f- u11 tions u J Complex V,rni,1bl : Th ory

Points al which _f (z.) is analytic are called regular points. Poims at which
.f (:) is not analytic are called sinR11for poims. Singular points arc often, but not
necc ... -;nrily. points al which f (:) becomes unbounded. For example. !he point
=
z 2 is a singular point of J(::J = 1/(z - 2). Let :: 0 be a singular point of f(z).
If there is a deleted a neighborhood about zo con1a.iniog no singularities. then zo
is said to be an isolated si11g11/ar point. For example, the point z = 2 is ao isolated
singular point off (z.) =
1/(z - 2).
The function I/ sin( I/;:) is a classic example of a func1ion wilh a singu-
lar point that is not isolated. Note that sin( 1/z) = 0 when ( I/;:) = 11,r, with
11= =
0. ± I. ±2 .... or when z. = I/ nrr. The point .:: 0 is a singular point. bu1
it is nol isolated because no mutter how small you take~ 10 be. there is an infinite
number of singular points wilhin any dclercd ,5 neighborhood of z = 0. We shall
deal only with isolated singular poinLS.
If lim (.: - z. 0 ,n f <..)=a -yf O for .some pos11ive integer 11, then zo is called
• --1 •
a pole of n.rder ,,. For example. I.he point .: = I is a pole of order 2 for f (z) =
1/(: - 1) 2 . A pole of order I. as is the point.:= 2 for f(z.) = 1/(:. - 2). iscallcd
a Jimple pole.

Example 4:
Determine all 1he poles of

SOLUTION: Any CAS will give the four ~cros of the denominator to be
;: = i, -i. I. I. and so we wri1e I(:) us

y
Thus. j(z.) has three poles: simple poles at i and -i and a pole of order 2
31 I.

Branch points arc also considered to be singularities. We have seen seve ral
limes tha1 In .: has a branch point a1 : = 0. Branch points arc associated with
multivalued functions and are examples of non-isolated singularities. A branch
X
point z: 0 has the property that 1here is some small enough circle around zr:i such
I.hat f(:) varies continuously with z. bUl does not return to its orig1nal values when
z goes around the circle. We can use Lhis definition to show 1ha1 z = 0 is the only
branch poinl of In:. Figure 18.4 shows a point :.o =I- 0 with a circle around ii in the
complex plane. As z. travels around the circle. the path does not cross the branch
Figure 16.4
cut. Therefore. both zl and fJ return to their original values and In .;: undergoes
An illustr:ition 1ha1 :o O is noI :i branch
=
point of In :. but ::.o 0 i~. no change, For -0 = 0. however. the path ml!lsl cro,.'< the br-.1I1ch cul and so the
imaginary pan of In ;: changes by 2rr as : travel." around the circle. Therefore.
: 0 = 0 is a branch poinl. and 1he only branch point
884 h.Jpl ·r 18 / Functions-of a Complt>lC Variable: Theory

0
= { 2,ri
This tums out to be a very well usc<l resull la1cron and should be remembered .
Note that we indicated the closed contour by writing£ / (z.) dz.. We shall

always evaluate contour integrals around closed paths in a counlerclockwise


direc1ion unless we slate otherwise.

Anorher useful method lo evaluate contour integraJs is 10 express l f(z) dz.

in 1enm, of real line integrals by writing f(:.) = 11(:<. y) + iv(x, y) and dz. =
d.x + idy:

{ /(z) d: = [ (11 dx - u dy) +i { (u dx + u dy) (6)


le le .le

Example 3:
Evolu111c fc f(z) dz from z: = (0, 0) 10: = (2, 4). where f(z.) = .r 2 - iy 2

and C is the parabola y = x 2.


SOLUTION: Use Equation 6 to write

2
y = fo (x 1dx + 2'< 5dx) + i fo\-x~dx + 2'<:-d.x)
J!o, +o) 8i
A.
Jr,----r-----,_... =24+-
I
5
B I
I
I
I
I X
I If we evaluale the integral in Example 3 along the straight line connecting the
I
r =
points (0. 0) and (2, 4) ()I 2r ), then the value of I.he integral comes out to be
(-a. -a)
------- ----~ A
24 - I 6i /3 (Problem 13). so lhe value of the integral depends upon the pa1h from
Lhe poinr (0. 0) 10 the point (2. 4) . This isn't alway~ the case. however. Let's look
al the integral of J (z) = cos z along Lhe 1wo path!. shown in Figure I 8.8. We firsl
Figure 18.8 write
Tu·o diffi:rcnl paths along which to
intcgmte /(:.)::::: c:o ;,; . cos:. = cos x cosh y - i sin .r sinh y

=u(x.y)+iv(x.y)
I 8.3 Complex In! · •ra tion: Cm h ·s Theorem 889
We can use general formulas for f J (z) dz. integrals s.o long a, the an1iderivative of
f(z.) is analytic in the region conraining Lhe integration contour. This Lheorem also
explains why the integral of f(z) = 1/z depends upon the 1wo pa1hs in Figure 18.8.
The anrideriva1ivc, F(:.) = In z. is nor single-valued and consequemly not analytic,
having a branch point at z = 0.
We can integrate j(-, ) =
1/.: if we keep track of which branch of In z. we are on
at various values of z (or 8 if the contour is a circle or an arc of a circle). Consider

I= J. dz.
1'c· :.
where C is 1he unit circle. Let's integrate from B = 00 (0:::: 00 < 2JT) to 8 =Bo+ 2JT,
and wri1e
0u +2:1
I= [ In: ]
/Jn

Recall lha1 we have defined In z. with a branch cut along the positive x axis
(Figure 18.4 ). so that
In z = In r + (8 + 2rr 11 )i n = 0. I. 2 ....
The value of n = 0 gives us the principal branch of In z;. wit.h O::: 0 < 2rr. hu1 nor
equaJ 10 2rr. If f:) = 2rr. we move lo the second branch of In z.. wilh 11 = I. and
2rr :s 0 < 4rr. Therefore.
ilt, :c.,,
I= [ Jlnz On =lnr+W0 +2rr)i-lnr-00 i=2rri

Note that this resull is in agreement wirh the result that we obtained on page 886
for the palh shown in Figure 18.8.
There is one other consequence of the Cauchy-Goursat theorem that we shaJI
discuss hefore concluding this section. Consider the sirua1ion in Figure I8.15.
where Co and C 1 are Lwo simple clo~ed curves, lhe point z0 may or may not be
a singularity of f(z.), and where there are no .singularities of j(:) in the region
d
bounded by and including C0 and C 1. Then. a direc1 consequence of 1he Cauchy- Figure 18.15
Tu·o simple closed curves. C 0 and C 1, in
Goursat theorem is that
lhc complex plan.! surrounding a ptJin1 t 0 .
( 12)

where both integrals are taken in a counterclockwise direction. as indicated in


Figure 18. 15.
we·re going to prove 1his resul1 below not only because i1 is fairly straightfor-
ward. but also because the proof involves a procedure Lhat we will use a number
of limes later on. Before doing I.hat. however. we want 10 emphasize 1he utility of
Equation 12. Suppose we are asked to evaluate
I B.4 C ,Hit h}·"- lritq:r,11 Formul.1 895
Now, given any '=. there is a 8 such !hat

I/(:) - f (a) I < <: when I : - o I < fJ

Therefore. the integral on the right side of Equation 3 satisfies

Ii ,-
If(.:) - j(a)l
: - a
d-1 -
(, <t: Ii~,
~E: - tl
-< _.]Tf

and Equation 3 becomes

1 .((:)
re ;: - d:.
{1 d:. - 21T1f(t1)
. I::: 21H (4)
1

Bccausl,! ,f cm, be made urbirn iril.y small. Equation 4 leads 10

i j(~ )
- - d:.
c :..-a
= 2,ri/(o)
Equation I is an invaluable tool for evaluating contour integrals.

)'

Example 1:
Eva! ua1c i - - -'
(
~
. ...
tl:.. where C is the rectangle shown in Figure 18,23.
2 X
SOLUTION: Using Equation 1 with/(:) , , e~ and a :: 2. we have
',

Figure 18.23
A rt.· ·1.mgll c ~urroundl ng 1hc po int ;: = 2.

'y
Example 2:
Evaluate

I =
reJ : + co rr::
2 (I - i): - i
d::.

for the two different contours shown in Figure 18. 24. X

SOLUTION: The denominator of the integr:ind foctorx into - - i)(~ + I).


so there are simple pole, nt z. i and .: = = ·-
I. Contour I surm unds bo1h
poles. and we cm dcfom1 the contour into two small circles surrounding
each pole. Forthc pole at:.= i. we have j(z.) cos ,rzil.: + I) and a= i in =
Equ.i1ion I. so we have
Figure t 8.24
2rr i cos i 1r 2ir i co h 1r The 1wo con toor lo he used lo cvalu:.ilc
/(nt:.. = i) = 2rrij(i) = . = - --- lhc in1ce,,m'I i n E,ampk 2.
1+1 I+ i
896 Ch;:ip1er 18 / Fu11 !'ion" of a Complex V.:iri,1bl : TI-ieory

For the pole a1 z = -1. we have .f(zl = cos rrz/(z - i).so

/(al z = -1) = 2rrif(-\) = - - - - = -2rri-


2,ri cos(-rr)
-1- i l+i

Therefore,

ic1 72
cos rr z.
+ (I - i) z - i
d 2,r i cosh ;r
z= ----
I+i
2,r i
+- 2Jr i
- = --(cos
I
h
JT + I)
+i I+ i

Con1our 2 surrounds only lhe pole at .: =i. so j(z.) = cos rr z/ ('<. + I) al


a= i:

'f. -, .
f.Jr1COS17f
I
I =2 T(/ (1) =- ---
1+ i

Both of 1hc above Examples have simple poles. What if we want 10 evaluate
something like

I= i j(;.)
c (z - a)·
1
d 7
~

where f (.:) satisfies Lhe conditions of the Cauchy integrul fonnula and C encloses
the point .: = a? We can't use Equation I be,caus~ f(z)/(z. - a) 2 may not be
analylit: a1;: = a. ll so hnppcns Lhat we can use Equa1ion I to derive a result that
does allow us 10 evaluate integraJs like the one above. Lei's form the derivat.ive
=
off (z) al z a using f (a) from 1he righr side of EquaIion I. Firs1 write

f(a + h) -
/r
/(a)= _l_
2rr i h reJ. ( ;: - al - _z._-I_) h (1
/(1..) dz

t J f d.:(z)
= 2rr i re ( a -z: - II)(: - a)
Using the identity

- I- - -I ( l + -
h -)
x-lt x x-J,

we can let x = z: - a 10 write


f(o + h) - J(n) I 1 j(z.) dz Ii J f(:) dz
h = 2rri re (z. - a) 2 + 2rri re (z - a) 2 (z - a - h)

I
=-
21ri
i f(:)
---·dz+t!i.l
C (z - n) 2
(5)

The last tem1 here, 6 /, is proponional 10 h. so we may hope Lhal this Ierm vanishes
asI, ➔ 0. If Lhat's so, Lhen Equation 5 becomes

= - 1- J. /(:) dz
j'(n)
2rri re (z - a) 2
(6)
898 ~h,) pl r 18 / Fun tions of .1 omp l x Vari, bi : Theory

For C1 • 1ake f (:)=cos :/: 2• a= i. and 11 = 2:

I2 = JT I. [ dd~~-, cos~- : ]
-'l- = . 17 COS I. + 4.I Sin
Fr I
. I. I :::; .i I.( 7 COS h I + ➔_, Sin.
. h I)
- .... ;==:.i

The 101al inregr.:il is

I = 11 + 12 = -6;r i + j'( i (7 cosh I + ~ sinh 1)

There arc a number of import an l consequences or lhe Cauchy ini cgrn I fonn u I a.
Many or Lhcse are what you might call theoretical. and are discussed in lexL~
on complex variables. but some arc direc1ly applicable to physical problems.
Two that we shall use in Sect.ion 19.5 are Poisson·s integral formula for a half
plane and Poi~"on's integral formula for a circle. We derived these formulas in
Seel ion~ 17 .6 and 16.2. but they can be derived directly from the Cauchy integral
formula (Problems 15 through 20).

18.4 Problems
l. Show 1hut /'(O) = 0 but 1h.:it .("(OJ does no1 exist for j(.t) = x 3 sin( 1/:r).
r
2. Evaluate/ =
t e;:- 1
- -d: where C 1s given by
C - - a
. .
1:1= I or all values of u.

3. Evalua1c / =J ~ d.:: where C is given by ( a) 12-1= I and (b) 1::1= I.


fc· 4- - rr

4. EvaluaLC / = J ,, :: l d=. where C i~ given by 1:1 = 1.


.re- .:- - 4

.f,r "' -
5. Evaluate/= --.-d: wht:rc C is given by (a) lz - II= 4 and (b) 1.:-i = 3.
i ;;r

6. Evaluate/=
iC
.,
z- -
in - . .
, d:_ where C 1s given by (a) j.:- - II= 2 and (b)
1r- /4
I: - II= 3.

7. Evaluale / = fc(-
j ,,"i · d: where C i!. gi \'t.·111by -1= 2 by (a) using partial fractions and (b) dcforma1ion of
-+ I)
coniour.

8. Evalua1c / =
i sin.i::
- - - - d : where C is g11\'en bv I I = 2.
C ( :; - tr/3),l
. .

9. Evaluale / = i ~:~ ~ d:. where 11 ~ 0 and C irs gi ven by 1-1=2.


900 Ch,l1Hcr 18 j FtHll 11011~ of a Comp'lt"x Variable: Theory

Show tha1 this will be so if ( 2 = (1. and !hen show thar 1his leads to

Now let /(z) = j(x. y) = u(x. _\') + iu(x. y) and cqua1c real paru 10 obtain

which is Poisson's integral fonnula for a half plane.


16. Use !he resul! of the previous problem to delcnnine the electro~tatic potential in the upper ha.If plane subjccL
to 1he Dirichlet boundary conditions u = ,, 1 for x < x 0 ond 11 = 11 2 for x > x 0 .
=
17. Plol your result in the previous problem for x 0 0. 11 1 = 100. ,, 2 = 0, and y = I. ReaJizc that the o.rctangcn1s
in the solutjon musl lie between O and ,r because of the geometry of I.he problem. Hinr: The fonnula
tan- 1(-x) =-
ran- 1 x is nor valid in tJ1is case because O _:: ,an- 1(y/x) := ;r.
18.
X < -I
- 1 <X < I
X I

19. Plo1 your result in the previous problem for 11 0 = I. 11 1 = I 0. 11 2 = 5. and y = I. Real i;,..c 1hat Lhc arc1angents
in tJ1e solu1ion mus1 lie between O and rr because of 1he geometry of the problem. Hi.111: The formula
ran- 1(-x) = - tan- 1 .r is not valid in thi!-. ca.,e bccnuse O < 1an- 1(_r/x) _ T(.

20. Derive Pois~on·, integral formula for a circle from Equation 1. Hint: Use the same trick that we u~cd 10 derive
Poisson·s in1egral for a half plane in Problem 15. Take lhe point outside the circle 10 be the poinr R 2 /(~. which
is the renec1ion of 1he poin1 { through I.he origin (i11vcr,1·e of ( ).
21. In this problem. we derive Couch_,. 's ir1eq11ality. Show 1ha1 if f (::) is analytic inside and on a circle C of rJdius
< ) 11 !M
R ccntcrc<l at~= a. then I f " (a) I :5 - - 11 = O. I. 2 ..... where If(:)!_ M.
. R"

22. Show that if j(';:) i-; analytic i.nside and on n circle C with ccnler at.~="· then /(a)= _2._ (:1 f(a + reiO)dB.
2rr lo
Interpret this rcs11l1.
23. Rl:call that an entire function is a function that is analytic 1hroughouI the (finite) complex plane. For example.
1:~. CO!-.;:. sin:. :md any polynomial in : are entire funclions. Prove Uo11Pi/le ·s theorem. which says that any
bounded entire function must be a constant. Hint; Srart with Cauchy's inequnli1y with n = 1. Problem 21.

24. Evaluate J 2__!!__;;_


J;_. + ...
where C i given by (a) l;:I = l. and (b) l·I = 3.
25. We' II prove 1ha1 t:.I In Equation 5 vani~hes a!-. Ii --,. 0 in 1his prohlen1. FiiPa deform C 10 c·,.. a circle of radius
p cenrcrcd and lying t:111i,re ly within C. The point o + h wilU .fiie wii,h i m C,, i r we clwo-.e p > 21 /, /.
a1, i1.
and since we anticipate 1h11t h -~ 0. the circle CfJ wiill lie entirely wi1h C. Now use 1he triangle- inequality,
1:. 1 - ·21 ?:. l:: d - !;:21, lo show tJ1al 1hc magnitude of the denominator oft:./ sa1i~tic, C: - a)~(~ a - lr )I :::_
, 3 1h IM 2MI 1, I .
p·(p - lh l) ~ p· /2. Now lei M _ 1/(:)1 and show that !~ I I _ - -1- · 2;r p = --,-. which goes to zero
11 p · p·
as I, - 0.
18.5 Taylor Serie5 and Lauren! Series 901

18.5 Taylor Series and Laurent Series

We spent al I of Chapter 2 discussing infinite series of real quanrit ies. Many of Lhc
resulls of Chapter 2 carry over to infinite series in the complex plane with little
change. For example, if
I/

Sn(:::)= L Jjlz.)
j=O

then the series converges 10 S(z) if there exis1s a value of N such that

IS(z) - s,1(z)I < E when n > N(E, .:)

where we emphasize that N depends upon the vaJue of E and possibly.:. lf N docs
not depend upon the value of z. then 1hc series converges unijon11/y to S(;:). We
can use 1he ratio tes1 for convergence. The ratio test says that

The series f
n.::::.O
J~, (z) converges if lim
n-
I/,,f,,+ I= L < I and diverges if L > I.
1

The test is inco11cl11sin.! if L -= I.

Example 1:
Use the ratio test 10 determine for what values of z. I.he geometric .~cries
00

SN= L z." converges.


noO

SOLUTION:

.11+11= lzl
Jim -'-
1 z."
n-"):i

and so s,.,(z) converge~ (a~olu1ely) for 1:1 < I and diverges if 1::1> I. You
can ~e that it diverge~ for l.zl == I becau~ the nlh term doei. nor go to 1.cro
a-.11 - oo.

Recall that the geometric series can be summed in10 closed form by multiply-
ing S11 by z and 1hen sub1rac1ing S,, 10 get

Solving for S,, gives

I - l"+I
s,1(;:) = --- (I)
1- z
902 C h.ipl('r 11:1 / Fu11c rii 111, or a Complex Vari.1ble : 1 h"ory

For lzl < I. : 11 + 1 -> 0 as 11 - oo and we ob1ain


.:x,
I ~.
S(z) = --_ = L :1 (2)
I - .:. . o
j'C":

Equation 2 is a power series about lhe point : 0. Power series play u cenlral =
role in complex variable theory. We have I.he following theorem fur power ~eries
in a complex vari~ble:

Consider rhe power Hries about 1/re poi11r:: = tI:


'.X)

Jc- _t ~---= L ell(.: - 11)" (3)


,r=fl

where tlw seriex c,mvery:esfor z. - aj < Rand Ji1·erg1'.1for I~ - al ;... R. The


=
circle :;: - ll I R ( on 11,11,icl, rhc series may or may 1w1 cmwcrge) is coiled
tllt' cin:ll! of convergence. TIil' rodiu:. of 1hi.,- did, is given by

-RI = 11
.
hm IC11+l
-- 1
• :i.. r,,
(4)

If R = 0. then 1/ie xeries ,·nm,t'r;l!t'.1' 011/y 111 tin· 11oin1 ;: = r1. {/ R = "'-.. tlir11
1/11· .Ini£,.1· co,werges in llw l!lllirt! complex plane.

Example 2:
Derive Equntion 4.

so LU TIO N: Simply apply the ratio ICSI 10 Equal ion 4:

lim
II - •

The series converges if 1- - ol < Rand diwrges if:.- - al> R.

We learned in Chup1er 2 thal power series converge unifom1ly and that uni-
fonnly convergent series arc !-.pccial in the sense 1ha1 t.hey can be manipula1cd much
like polynomials. This is also true for power series of complex variables.

Let 1- =
o I R he rhe ci1d1• vjl'om•1·rgcm·e o/tht· po1n•r Sf'ries in Eqiwtio11 3.
Th II this sait·,· c(mv1.·I·r.:1:.~ unifom1ly {IJI(/ abs:olurely wi1/,i,11111d 1.1111111y circle
imerior ro tltt' circlt· of cvm·erxence. Nor only do pm,·er series com·t-r 'L'
;\.

11111Jnrmly ond ob.w/111ely, h111 also the p,m·er .\"t'rie · L c·,, ( - - a )n i/egrihc.1·
r, ;=0
all analytic function ,,·irhi11 the cirrle of com·t·rgence.
18.5 T,1ylor Seri -, and Laurent Serie!\- 903
The above properties of power series explain why power series play such an
importanr role in complex vari:ible theory.
The Weiertrass M-1est for series of a real variable carries over essen1iall y

unchanged for a series in a complex variable. Consider the series L~ fn (z.). The
11:0
\Veicrstrass M-lest says that if there is a set of posi1ive constantc; {Al,,} such 1ha1
"'.)C

lf,,(z)I ::s M,, for all II and for all : in some region R, and if the series L M"
n=O
00

converges, then the .series L J~,(z.) converges unifonnJy.


n=fl

Example 3:
DC

Show thut 1hc series L l/(11 1


+ z) converges unifom1ly for Re: ::: 0.
n=l

SOLUTION:

I= [-
I- - ----
I I ] I/:! -,:· -[
'I ., ') ., - .,
11 -+- (n- :·:r)-+y- n-

co
for Re z = x 2:. 0 . The series l/11 2 converges, and so the series L l/(11 2 + ;:)
n=I
converges uniformly for Re;::::_ 0.

We'll now prove !hat every analyt.ic function can be rcpre~~ntcd by a power
series. Start wi1h the Cauchy integral formula.,

J (:> == _1__ J f(O d( (5)


2n-,rc {-Z.

and 1hc geometry shown in Figure I 8.27. The quantity R in Figure 18.27 is the
radius of the largc~l t:ircle about !he point : = a within which J (:) is analytic.
In other words. R i~ the distance from a to : 0. the singularity off(::.) nearcsl a.
The circle C in Equation 5 is a circle ccntcred at a whose radius p is greater than
i:'. - al (but less than R). so 1ha1 : lies within C. Write 1/(~ - :) as figure 18.27
The gcomcrry lL'-t-d 10 de-rive E4ua1ion 9
from Equation 5.
I 1 [ I ]
t; - :: == ( - a I - (z - a)/(( - a)

where (z: - n)/(s - a) < I because ( lies on C and: is within C. Use the algebraic
identily

l xn
- - = l +x +x 2 + · · · +xn-l + - - (x -fa I) (6)
1-x 1- x
906 <...h.1plN 18 / Functions oi a Complex v .u i,, b le: Th ry

where C is any simple. closed. piecewise smooth curve lying in 1he annular region
between C 1 and C~. (See Figure I 8.29.)
Referring to Figure 18.29, we see 1hat .f (z.) is analytic within the region
bounded by the closed palh C 1 + C 2 + C 1 + C4 • and ~o we can use lhc Cauchy
integral fonnulu 10 write

Both C 1 and C2 are taken in the counterclockwise direct.ion. and we have recog-
Figure 18.29
The geomc:1ry u~d 10 dcn\c [qu;11i1111' 12 nized that the contributions along C:i and C 4 cancel. l'n the integral le, on the right
lhrough 14. The poin1 ~ lie-. in the region side of Equation 15. we take
between C 1 and C2 .

(16)

with(: - a)/({ - a) < I on C 1• because z. lies in the region between C 1 and C 2


s
and lies on C I in Figure I 8.29. Substituting this expansion into lc 1 leads lo
(Problem 16)

/ -
1
=L
11::00 1
-2m1 . i I
f(t;)
(( - a) 11 + 1
d( l OO
(.:: - a)'1:::: """'a,,(:
L..,,
n==O
- ut ( 17)

where the a,, are the same as in Equation 13 .


For 1he integral lc1 on the right side of Equarion I 5. we ta.kc

_J
{-z
=--] (
z.-a
1 -
I
{-a
- -
) = - -1
z.-o ,,,,,,,(J
f: (~)11 z.-a
(18)

z. - a

with ({ - a)/(z. - a) < I on C 2 because { lies in C2 in Figure \ 8_29. Subs1itu1ing


this expansion into le~ leads to (Problem 27)

le
2
=- t,
,,:::O
_I
2rri
1 f<O
h i ({ - a ) n
d{ l
(z - a)-,,-1

00

=- Lh,,(z-a)- 11 (19)
14=1
18.5 fa ylor Seri . and Laurent S ri 907
where the bn are the same as in Equation 14. Subs1 i1mi ng Equa1ions 18 and 19 back
into Equmion I 5 gives Equarion 12 with the coefficients given by Equations 13
and 14. Because (z - o)/(t; - a) < I on C 1, we sec tha11.he first series in Equa-
tion 12 converges for lzl < C 1• Similarly. because ({ - a)/ (z - a) < I on C2, we
see lhal the second series in Equation 12 converges for lzl > C 2 . TI1e common
region of convergence of Equation 12. Lhen, is C 1 < Izt < C 2•
Note that the uurenl seric~ assumes that f (:) is analytic only in the annulus
detennined by C 1 and C'!.. lf j(:) happens to be analytic within C 1, then

b,, = _1_ J f (O d{
2rri rc 1
({ - a)l-1t

for 11 =::_ I

according to Cauchy's integral formula because (( - 0)' 1 - 1 f(i:) is analytic within


C 1• Thus, all the coefficients of Lhe negative powers of:: - a in Equation 12 vanish
and we have a Taylor series. We can consider a Taylor series to be il special ca"ie
of a Laurent series.
Let's consider the Lmrent series for

I
JU.>=---- (20)
( I - z){2 - .:)

There arc singularities at .:= I and· === 2. Therefore, we have 1hree regions (lzl < I.
I < l·I < 2. and 2 < 1.:1) IO consider. For lzl < I. we can jusl expand both 1/( I - :)
and 1/(2 - z:) in geometric series and obtain
1
= ~ (I + z + : 2 + · · ,) ( I + ~ + ~ + ~ + · · ·)
2
f ( z.)

I 3;: 7-:- 2
=-+-+
2 4
-+
8
·" l.:I < I (21)

This is just a Taylor series because there are no singularities w11hin 1.:1= I.
Now let 's look at the annular region. I < 1::1 < 2. We expand the firs1 factor in
f(z) as

which converges for I < 17 1. We expand the second factor in f(z) as


2 3
- I - = -I ( z z :
!+-+-+-+·" )
2- z 2 2 4 8 .

which converges for lzl < 2. The producr of these rwo expansions is (Problem 17)
909
The series for e~- 1 converges for all values of lz - I I, so the Laurent ~ries
converges for O < I - II < oo.

We'll sec in L.he next section lhat it turns out in practice that the most important
tenn in a Laurent expansion is the term b 1/(z. - o).

Example 6:
Find the coefficient of 1he first few 1enns of ;1,cending powers of z. of

I
f(:)= -_-
e'- - I

about:::= 0, which is valid in the annulus O -:: l~I < 2JT (Problem 19).

SOLUTION:

- - =-------
,z - I ' ~)
;:+.:::....-=--~+·· ·
2 6

"'
:: I + :. + z.~ + 0(:3 )
2 6

= -:::I - -2I + -12;: + 0(::: 3)

18.5 Problems
:,0

Leu~ converges for which vaJucs of z.?


1. The series

~ (' .)//
00 oc

2. Sum the senes L - - L


(l+i)"
3. Sum the series L ;
n_==O 1 11
4. Sum 1he series 11 ::: •
4
n=O n:.ail n=O
,0

5. For whrch values of l docs L e"1~ converge? Sum tJ1e series.


11:0

6. De1ennine the radii of convergence of the following power series:


QC

I::,,
~211 00 (: - i)"
00

I)n +2")z" I: ,,!z"


00

(a)
11=0
(b)
I:~11=1
(c)
n==O
(d)
n=l
11"
910 Ch.1ptc-r I 8 / Fune ti l'111s of a Complex Variabl : Th ory

.
7. For which values of;: does 1he scnes L
. ~ - -I - converge un11ormly?
·r
1
11 -+;:
11=0

8. ObLain a Taylor series expans·ion of e= about the point : = 2.


9. Obi.a in a Taylor ~ries expansion of cosh z about the point z. = 0.
10. Obtain a Taylor series el(pansion of cosh ;: 2 about lhe point z = 0. (See 1he previous problem.)

II. Ob1ain a Taylor series expansion of Ln ( I + z) about 1he point ;: = 0.

12. Obtain a Taylor series expansion of Ln;: about the point.:= 1.

IJ. ObLain a Taylor series expansion of/ l;:) = L: e-~"'dw abou1 lhe poin1.:: = 0.

(-l' ),n 72'1 + 1


14. The answer 10 the previous problem is
oo
L
,, _ (11
.
+ O!
.Can you express this in closed form? Evaluate the
0
integral in the previous problem and compare your an.swcr.

15. Show thnr the remainder in Equation 9 goes to zero us 11 ➔ oo.

16. Verify Equarion 17.


3
_ 1(
I+ ~I I
+ ~I + · · ·') and I ( ;: .:: ) .
2 I+ 2 + 4 + 8 + · · ·
i:
17. Show that Lhe product of-; + ~ is

I I I z ,, • . 4

···- - --;; - - - 1 - - - -- - •••.(Be sure to keep trnck ot lhe v:mous powen- ol .::.)
7z.· z.- - 2 4
18. Verify Equaiion 2J.

19. Why is the Laurent s.erics of 1/(t•~ - I) given in Example 6 valid in 1hc annulus 0 < lzl < 2.Jr.
20. How many Laurent series about.:::= 0 arc there for/(.:) = 1/(1 - z)? For which values of 1.:1 arc these series
\'a.lid? Find 1hr,L· series.

21. Find 1he Lauren! series of f(z.) = 1/( I + z.2) for 1-1;;,- I.
22. Find 1he Lturent series off (z.) = e~/( I :-) abou1 the point z = -1.

23. Find all 1he terms involving negative power.. of:: - I and the first few tenns wilh positive powers for the
Laurent series of/(.:)= •··"j(:- 2 - I) about the point.:= I.

24. Find 1he Laurent serie.s for f (z) = ;: 3 .~in( 1/z) about 1hc poin1 z = 0.
25. Find 1he firs1 few terms with positive and negative powers of z (if any) for all the Laurent series of
/(z) = 1/(.: + 1)( :- 2) about 1he point z = 0.

26. When we derived the Laurent series for f (:::.) in the 1ex 1. we substitured lhe serie.s given by Equalion 16 i.nro / c 1
in Equa1ion 15 and integrated 1erm by term. This is a valid procedure if 1he sc:ries in Equu1ion 16 is uniformly
convergent. Show that the series is uniformly convcrgen1 in (.

27. This problem is related to the previous problem. Show 1hat the ~l·ric-; in Equation 18 1s uniformly convergent
in {.
912 Chaprer 18 / Fun< lion~ ui a C1>rnplt•x V,1ri.1hll': Theory

y ( (z.)
·
=z2 (. I + -zI + -,
I
2z-
+ - I 3 + - I 4 + -· -)
6z: 24z.
, I I I
= z.- + z. + - + - + - -2 + ...
2 6: 24:::

so we see rhat the residue off U.) at z = 0 is 1/6. Therefore, &juation 2 tells us
X
Lhal

i
c
1 t/-d
z.e 2rr i
~ z=-=-
7T i
6 3

Example 1:
Figure 18.30 Use the residue 1heorem to evaluate
The contour used in Exu.mplt! I and the

reJ.
singular points of the i nt.cgrand.
I= cos .: dz
z3( ..: 2 + 9)
.v
where C is the circle given by lzl =2 (Figure 18.30).
3 SOLUTION: The singularities of the inregrand occur at z =0 and
t = ±3i. so C enclose.'> only the singularity al z = 0. The L1uren1 series
for f(z.) = cos zJz3(z 2 + 9) in the annulus O < 1: 1 3 is

l 11
= - 3· - - + 0(.:)
9;,: 162.z
The residue of f(z) at::.= 0 is -11/162. and I.he residue theorem tclb us
Figure 18.31
The conrour dcscribed by I.: I = 4 and
that
the singular points of lh~ integrand of the
integral in fas.mple I.

What if the contour in Example I enclosed all three singular poin1s of 1he in1e-
grand? Let's suppose that C is the circle given by IzI = 4, as shown in Figure 18.31 .
Because there are only Lhree isolated singularities. we can deform C as shown in
figure 18.32 so that
.r

and then apply 1he residue 1hcorem to each integral separately. This leads to the
general result that if f (z.) has a number of isolated singularities within C. then

figure 18.32
The defonm1tion of 1he con1our in
Figure 18 .31.
i /(z) dz.. = hi I sum of the residues of J(':.) inside CJ (4)
914

The lwo mosr important types of singularities for our purpose.c- are best di,,;-
cussed in lem1s of Laurent series. Consider the uurenl series

(5)

where t.he expansion in negat.ive powers tenninates at lhe Nth 1erm (assuming that
hN i- 0). In rhis case, 1he singulariry of/(;:) at :: ::;:: a is said to be a pole of order
N. For example,

=
has a pole of order 2 al ;: 0. If N ::;:: I. the pole is called a simple pole. The
function J (:.) = e·'. / z. h,1.s a simple pole at I.he origi 11. fr N ::;:: oo ( in other words.
if 1he negative powers of the Laurent series coniinuc indefinitely), 1hc singularity
is called an e.uenrial .~i11R11lariry. The function 1· 1 ·.: has an essential singularity at
.:=0.
We L,m detem1ine the order of a pole of j(:) fairly easily by using 1he fac1
that n is u pole of order N if and only if

(6)

where g(;:) is analytic at : =a. To prove this. start with Equation 6 and e.xpand
g(:) in a Taylor series abou1 : - a and gel

wbich is essentially rhe definition of a pole of order N, according 10 Equ.ution 6.

Example 3:
Cla."sify Lhc singular points tif

I
/(:)= - - - - -
(:: - 1) 3 ~ + 2i)

SOLUTION: There ar~ ,i ngulnr points at : = I and at , = - 2i. u~


Equal.ion 610 write f(:) a."

f (:. ) = ~
' 2:- J)]

where)?(:)= 1/(:: .. 2i). Because g(z) is analytic al::. = I. the pole at::=· I
915

i:- a pole of order 3. Similarly. for the pole at z = - '2i. we write

f (-::.) = g ( z)
z 2i

where now g(::) = 1/(: - l)J_ Bec.1use ,c:(::) is analytic at:= -2i. the pole
at z = -2i i~ a simple pole.

Example 4:
Cl:.1..-.:-ify rhc singular point at z. = 0 of


f(:..)=---
1 - cosh::

SOL u TI ON: It i~ not convenient to use Equation 6 here. Using the Taylor
series of cash :.

cosh: =I 2 24
0(:6)

we ~cc that lht! denominator cun be written as


~ ..,
-~ + 0(:4 ) = - :· 1, + 0(/)]
2 2
Therefore,

and so wl: ~cl: that the point : = 0 is a pole of order 2.

Wi1h our dis.cussion of singular points beh.ind us. we·re ready 10 show how 10

find the residue of f ( z.) at a pole of order N. Suppose f (z.) h~ a simple pole al
z. = a . Then. according to Equa1ion 5.

If we multiply borh sides by - - a and then let.: - a. we have

b 1 = Jim(:: - a)f(:) (simple pole) (7)


,: 11

Suppose now 1hat / ( z) has a pole of order N . as i 11 Equation 5. If we muhiply


by (z - a)N. we have

(8)
916 Chapter 18 / Funcrions of a Complex Vari.ible: Theory

This equation represents the Taylor series expansion of (z. - a_)N f (z.). If we let
G(z) = (;z: - a)N f (z), then Equation 8 can be written a_(,

G()
z = G (a)+ G'(a) (;_ - a) + - · · + - -
I - --- (dN-lG) (z - a)
- N-1
+ ---
(N - I)! dz.N-I :.=

(9)
By comparing Equation 9 with Equation 8. we see that

Note that Equation I O reduces to Equation 7 when N = I.


The following two Examples illustrate the use of Equations 7 and 10.

Example 5:
Find all 1he residues off (z.) gi\'en in Example 3.

SOL u TIO N: The point z = I is a pole of order 3. In this case.

Res(:= I) = ~I Jim
2!=-•1
[i 2
I
- 2 - --
dz (~ + 2i)
] I
= --"'""'
(1+2if~
= - -11 + -2i
125 125

The point:= -2i is a simple pole. so

. I
1·1m - - 11 2i
Rcs(z=-21)= - =-
3
=- - -
:.--li (z - 1) (I+ 2i) 3 125 125

E"ample 6:
Find the residue al z = 0 of J (;,) given in Example 4.

SOLUTION: The poin1 z = 0 is a pole of order 2. Using Equation 10.

2
Res(:= 0) = lim [!!_ - - e-~- ]
7
-
~---o d~( l - coshz) z~
= -2

In I.his ea~, it is just as easy to determine the residue from the final result in
Example 4:
919

28. Evaluate
reJ s1nh_d: z. , where C is the ellipse described by 4x:! + y 1 = 16. (See Problem 18.)

29. Evalume J
re +
Problem 19.)
(z
:~d\
2)~(z~ - 4)
dz, where C is the square with venices ( 1. I). (I. -1). (-1. \). (-1. -1). (Sec

30. Use any CAS to dc1cnninc the residues in Problems I through 10.

References

James Brown and Ruel Churchill. 1998. Complex Variable.1· with Applirntiom·. 6th ed ..
McGraw~Hill
Steven Krantz. l 999. Handbook of Complex Variabh·s. Birkauser
Jerrold Marsden and Micbacl Hoffman, 1998. Basic Co111pll':r A1wly.1·is. 3rd ed .. Freeman
Reinhold Remmen. 1990, Theory of Cumple.x Funcrions. Springer-Yerlag
Lester Rubenfeld. 1985, A First Course in Applied Complt>.\ Variables. Wiley
Richard Silverman. 1974, Complex Analysis with Applirnrions, Dover Publil:ali1,n~
Murray Spiegel. 1964. Comple.r Vnrinhfc.f, Schaum's Ou1linc Series. McGn:iw-Hill
David Wunsch. 1993, Compf,·x Variohh•.f wi1h Applicmio11s. Addison-Wesley
GENERAL:
Paul Nahin. 1998. A11 Imaginary Tall': The Story of ../="i". PrinL"cton
CHAPTER 19
Functions of a Complex Variable: Applications

This chapter coniinues our study of functions of a complex variable. Complex


variable theory, or analylic function theory as il is sometimes called, has numer-
ous applica1ions in the physical sciences and engineering. We saw in Section 17.7
that the inversion formula for Laplace Lrnnsfonns involves integration in I.he com-
plex plane. and we sbaJI learn how lo use the residue theorem to inven Laplace
transforms analytically in Sect.ion I. The inversion formula for Laplace lfansforms
is cxprc.s.sed in 1crrns of a contour iategraJ, so it's nor surprising that we can use
the residue theorem to evaluale the integrals. Whal might be surprising. however.
is 1ha1 we can use the residue theorem to cvalualc real defini1c in1egrals such as

1
t'o dx
= lo I +x 4

and
o::, sin 2 x
I=
l0
--dx
x1

In fact, we shall see in Section 2 thal the residue theorem can be used to evaluare
a great variety of renl definite integrals.
Not only can the residue l.heorem be used to evaluate real definite inregrals.
bur it can also be used to derive closed expressions for summa1ions such as

and
00
(- !)"112
S= L (TI + a) 4
We'll develop a general mcchod for evaluating such summations in Section 3.
Complex variable theory can also be used lo determine 1he number of roots 1here
are to an equation in a given region of the complex plane. For example. in Seel.ion 4,
we'll learn how to determine that there are two solutions to the equation e' = 3.:: 2
I.hat lie within t.he unit circle. We"ll do this using a theorem that says I.hat if f (z) 921
922 ( h.1pl(·1 19 / Fun( tu rn s of a Complex V._1riabh.:: Appli <.:,Hio n.~

is anaJytic inside and on a closed curve C except for possibly a finite number of
poles and if f (z.) -:/= 0 on C. then

1
- -
2rri rJ J'(:)
f(:)
d: = Z - P

where Z is 1he to1al number of zeros and P is the rota I number of poles within C.
A prac1ical applica1ion of this Lheorem and iLS rela1ed ma1erial in Section 4 is
Lhe determination of the stability of mechanicaJ and eleclrical systems. Basically,
I.his theory rests upon a determination of the number of zeros of a polynomiaJ
equation such as Q (s) = a,,s 11 + a 11 _ 1sn-l + · · · + a 1s + a 0 thal lie in the righl ha.If
of the complex plane. We shall develop a rather simple and convenienl graphical
procedure called 1.he Nyquist s1ability criterion for doing rhis.
The last two sections deal wi1h solving Laplace's equation in two dimensions
by means of a procedure called conformal mapping. We briefly discussed the
t.r:rnsfom1a1ion or mappi.ng of curves from 1he z plane to Lhe 1J.1 plane described
=
by analyl..ic runctions w f (z.) in Chap1cr 4. We've learned that Lhe two functions
11(x, y) and u(x, y) in the rcln1ion w = J(z.) = 11(x. _y) + iu(x. )') arc harmonic; in
other words. 1hey each sa1isry Laplace ·s equation in two di mcnsions. We can utilize
this properly of u(x. y) and u(x, y) i..n the following way. Suppose that we want
10 solve Laplace's equation in a somewhat complica1cd region. such as 1he region
between rwo parallel non-coaxial cylinders. If we can find a mapping w ::=: .f (z)
that 1ransforms 1he region in question (in 1he ;:-plime) in10 the region between rwo
parallel coaxial cylinders (in the w-plane). then we can solve Laplace·s equation
in I.he w-plane. where Lhe geometry is simple. and then transform the resu!Ling
solution back into the z-plane 10 produce the solu1ion to lhe orig1nal problem.
The procedure is rem_jniscent of integral transform methods, where we rransfonn
a given problem into a simpler problem in transfonn space. solve it and then find
the inverse 1ransforma1ion to ob1a.in lhe solution to the original problem.

19.1 The Inversion Formula for Laplace Transforms

In the final section of Chap1er 17. we prcscmcd a formula for the inverse of a
Laplace 1ransform. We showed 1hal if
00
f'(s) = f e-S/ J(t) dt (I)
.lo
then

(2)

where c lies to the righ1 or all 1hc singular points or f'(s). At Lhe time. we didn'r
know how lo hand.le the integral in Equation 2. but now we recognize ii as an
inlegraJ i.n Lhe complex plane.
19.1 Thi.· Im' r ion Formul.1 IC>r l ,1pl.:ice Tr,m\lorms 923

A srandard way 10 evaluare 1he integral in Equation 2 is by using a closed


contour C like the one shown in Figure 19.1. The contour shown in Figure l 9.1 is
c.allcd rhe Bromwich contour. Using this con1our. Equntion 2 becomes /3

JU)= -
2m
J . t
c
r'' F(s) ds - -
A )

2rr,
. i
r
e-'"1 F(s) ds
A
(3) r
C
If we can show IhaI Ihe inregraJ along r vanishes as R, the rndius of Lhe arc, goes X

to infinity, Lhen

f(I) = -.
I
2rr1
i .
C
11
e· · F(s) ds (4)

and a.,,;;suming tha1 the si ngu lariIies of F(s) arc isolated, we can determine f (,) in Figure 19.1
A Bromwich contour for imccning Laplace
Ierms of 1he residues lying w iIhin the closed curve C. mmi.f11rms. The clo,cd con1our C con,i,I\
Now lct"s show that the inIegral along r vanishes as R - oo. We firsr lets= of Lhe \Cnical line (CY. /3) :ind the t·ircular
Rf/ 0 along r. Now, assuming that the real parts of all the singular points of F(s) !lfC f.

arc less than some finite number c. Ihen the i ntegra I over r (in a counrerclock wise
dirccIion) approaches

lr I 1:r/2 eR,,.'' F( Re'")i R/ 0d0


= -. 1

2JT I 311 / 2

as R ~ oc. Bui

Now assume that the magnitude of F(s) along r is.::: M / R1.:, where k > I and M
is a constanL Under 1his condition

M
llr l :S 2rr Rk-1 (5)

Now, cos 0 ~ 0 when rr /2 '5:. 0 ~ 3rr /2, and if you plot the integrand of Equation 5
for increasing values of Rt. you'll sec that it goes 10 zero everywhere. except at
the end points (0 = rr /2 and Jn /2). which contribute less and le"" 10 the in1cgrnl
as Rt increa~el--. Thus, we see that llrl - 0 as R - oo. and that j(t) is given by
Equation 4. (Problem 22 takes you through a more rigorous demons1ration that
this is so.)
Now let's use Equation 4 to find the inverse of F(s) = a J(s 2 + a 2). Using
Equa1ion 4.

J(I)=-
2rri
I ic
ae~·,
--ds
s 2 +a 2

The singular points of the integr.1nd occur al ±ia and the residues there ure
924 C h,1pter 19 / Fum ti11ns ol a Complex Variable: Apµlications

ae5, ,/ar
Res (s =ia) = s-ia
Jim - - = -
+ ia 2i J

and
a(!·, e-iar
Res (s = -ia) = s--ia
lim
s- ia
=
-2i

Thus, Equal.ion 4 gives

j(r) = -2rrI i [ 2;ri (


ei,:,r

2
,. -
e - iat )]
li = sin ar
This is one of the entries in Table 17.1.

Example 1:
Use Equa1ion 4 to find the inverse of
• s
F(s) = -,--2
s· +a
SOLUTION: There arc 1,imple po)c!1, :lt s = ±ia. The residues at Lhw,c
poles are

s t iaeia, /at
1
Res (s = ia) = s-,.iti
lim
s + ia
= 2ia
= 2

:ind Res (s = -ia) = ,r 1


m/2. Therefore. Equation 4 gives

j(t) = -2rrI I. [ 2rri (ei"1


-
e- ia1
+- - )] = cos at
2 2

The next Example is a littJe more involved.

hample 2:
Use Equation 4 to fin<! the inverse of

SOL u TI ON: The singular poinL<: of F(s) are poles of order 2 ul s = ±2i.
The residue of e-r' F( ~) at 1hnse poles are

d s2es' e2;, ( i)
Res (s = 1i) = s-2;
lim - . _ .,
ds (S + 2,)-
=-
4
,- -
2
19 .1 The lnwr,ion Formula (or 1,1pl,1n· lr,in~f,u 111\ 925
and

Res (s = -2i) = e'llr (


r+~
j)
4
Therefore.

:-in 21 I CO. 2t
f ( f)
, =- 4- 2
You can verify I.bis resull by showing that c{J(l)) = s 2 /((s 1 + 4) 2
(Problem I).

Example 3:
Find th~ inverse of
, I
F( s) = - -
s4 +4
SOLUTION: Thtrc are four ~ingular poin1.s. al. =I+ i. -1 + i,
- I - i, and i - i (Problem 2), The. dcnomin:nor of F(s) factors in10
(s - I - i)(s + I - i)(.~ ' I + i)(s - +i ).The 11esidwes a( these point$ arc
11

Res(s=l+i)= lim [ t•·'' ]


I (s+l-i)( + l + i)( - l+i)

=---
Si(l + i)
- , it
Res (s = - 1 + i)
.
= _8ie(_ _
I - i)

Res ( s = - I - i) = 8i ( I + i)

Res (s =I- i) =
8i (I - i)

The sum of the rcsiducis ;,..

e;' (e 1 + e- 1 ) - e- 11 (e' + e- 1 ) - iei' (t-1 - t'- 1 ) - ie_;, (e' - e- 1 )


-
16i
(ei 1 - e- )(e'
11 + e- 1) - i(e;, + e- i1 )(e' - e- 1 )
=----------------------
16i
sin r cosh t cost sinh,
=----
4 4
19.2 Evaluarion of Real, Udinil • 1!1tet_lf.1ls 929
19.2 Evaluation of Real, Definite Integrals

The inversion formuJa for Laplace Lransforms is a contour integral. so 1he ap-
plication of the residue 1hcorem 10 the dctermina1ion of lhc inverses of Laplace
transfonns is obvious. Less obvious. however. is that we can use the residue Lhco-
rcm to evalua1e a wide varie1y of integrals of a re.al variable. For starters. let's
consider the integraJ
2·'1' d0
I=
1 l 5 - 3 cos 0
(I)

Integrals of lTigonometric functions like Equa1ion I can be converted to contour


integ.rnls by Lhe subsl.itutions
eio +e iu z + 1/.: z2 +I
cos8 = = =
2 2 2z.
(2)
n = dz. e' - e-iO z. - 1/z z2 - I
d 1, sin0 = - - - - = =
iZ 2i 2i 2i l
and then in1egrnling around the unit circle lzl = I. Using Equn1ions 2. Equation I
becomes
I -
- f 2id,- -
3.::2 - JOz + 3 -
f 2id-:-~
(3z - I)(: - 3)
(3)

The integrand has simple poks at:= 1/3 and z = 3. Only the pole at z. = 1/3 lies
within the contour intcgr-dl ll=-1 = I). and so

I =2rri Res (z = 1/3)

? . r [ 2; cz - 1/3) ] rr
= -rr, -~_!.11}3 (32 - l)(z - 3) = 2

Example 1:
Evaluate
2.-r sin 2 ()
I=
1o
---dfJ
5 - 4 cos f)
(4)

SOLUTION: Use Equations 2 lo write

The in1cgrnnd has a pole of order 2 at z. = 0 and simple poles at z = 1/2 and
z = 2. The residues al 1hc 1wo poles lying within Jzl = I arc

2 2
Res(.::= 0)
. [-
= ~Lim d -- (;: -
- 1)
--
]
= 4-5
o dz. (2z - I)(;: - 2)

Res(;:= 1/2)
.
= lim
z-111
[<~- 2
l /i)(: - 1) ]
;::2(2.;: - l)(;: - 2)
2
=- ~
4
19.2 Evalua1ion of Real. Deiini!P lnh,gr.1ls 933
residue at : = I +i. the pole that lies in the upper half plane. is

Res (l = I + 1). = .---1-JJ-;_(z


.
11m -
d
-
I
1+1)-
. ") = 4

Thus.

Another general type of real in1cgrnl that can be evaJ uated by contour integra-
tion is of 1he form

J
C\:

-cc
F(x) cos mx dx or l:. F(:r) sin mx dx

where f(x) is a ralional function of x. In this case. we replace F(x) by F(:.).


replace cos mx by (i"'= + e-im:)/2 and sin mx by (eim:. - e-im~)/2i and use the
contour shown in Figure 19.5. As usual, F(z.) must be such 1ha1 the in1egral along
r in Figure 19.5 vanishes as R - oo, which means that I F(z.)I ~ M / Rk for large
values of R. where k > I and M is a constant.

Example 4:
Eva.Juatc
00
cos w.x
1 o
--d:c
I+ x 2

where w > 0.

SOLUTION: SI.art with

where C is rhe contour shown in Figure 19.5. We musl be! sure lo show thal
the integral along r vanishes ar R - oo. The magnil'ude of the integrand
along r is equal to

Th.is quantity vanishes as R .- oo because w > O and sin 8 > 0 all along r.
The iniegrand has rwo simple poles. al ;:: = ±i. Only lhc pole at : i lies =
within C, and lhe residue there i~

Rcs(::=i)=lim
:-i [/w:]
--.
Z + I
= -.
21
e- w
934 l h.1p!,·r l'J / Funt li11n o ,1 lo111pl;•); Vari.1h!1•: Appl11 ,tlirnh

There.fore.

,. _ ei(' !~
f
I COS WX -W
- - -dx
2
= - --dx = rr e
. -oo I +x -oo I +x 2
or. using the fac1 that 1he imegrand is an even function or x.

w>O

Example S:
Evaluate
x, .

i
- ::,.; X
.r.rnrr:c dr
2 + . ..\ + -
.., - "' •

SOLUTION: Stan with

where C is the conwur shuwn in Figure 19.5. The argument thal lhc inll"!,!rnl
a.long r vanishc.s as R - • u· is csimi1lar IO 1ha1 outlin~d in the previo~
Example. There arc simple poles nt • = - I ±i. am.I only the poiirnr - I + i
lies within the cnnlt)Ur. The residue or 1hc above in1cgrand at : = -1 + i is

::ei.--r ::: I- i
Res(::== -I+ i ) = lim - - - - = - - l'-rr
-l+i ;: + 1 + i 2i

and so

By using Euler's r~la1.ion on the left. we get the two rcsu! L"

-x sin ;rx dx
l f
oc, X COS :'l' X tf_r - n
- - - - - =rr e and
(X.
-- -- = -;r(•-·rr
-~ x 2 + 2x ·' 2 -,x; x~ 2.r + 2

We'll finish the section wi1h Lhc integral of a function with a brdnch cul. Lei's
evaluate
,,- )
I =1 0
_x_ _ tlx
l+ x
19.2 Evaluation o( Wc<1I, Oefinilt· lnh•gr,,ls 935

where O < p < I. Start with y

where C is the contour shown in Figure 19. 7. The integral along r vanishc~ m,
R ➔ x because O < p < I. Similarly. the integral around C, vanishes as € - 0
(Problem 13 ). Using , = x along the upper part of rhe branch cut and z = xe2:ri
X

aJong Lhe lower part. we can write

-x rP-ldr 10 - - - - - - =
e Jrr i(p-lJxp-Jdx

1
o
-·--· +
l+x oo l+x
2,r i (.sum of the residues)

The only si_ngular point of the integrnnd within the contour shown in Figure 19. 7 Figure 19.7
is at .: = -1. and the residue there is (-1 )P- 1 = ei.-,(t•- 1>. Therefore, The contour u~d 10 cvalua1c 1hc in1cgml
in Equation 10.
x p-1
rl -e 21ri(p-l)]
l
•o
X
- - d .\.
I +x
=-,. . n1e• (p-llni
or

rr
sin prr

Example 6:
Evaluote

xl l-'dx
l
oc
I=
•0 (1 + X )2
SOLUTION: u~e the contour in Figure 19.7 to evaluate

A<:. before. 1hc integrnls ::ilong CH and C, vnni,;.h in 1hc limits. l 1,e ::. = .r
aJong lhe upper par! of the branch cut and : :=- :re2.:-r; along lhe lower par! 10
wri1e

The singularity at ;:, = - I is a pole of order 2. and 1he residue there is given
by

d -1/ 3 el:r/J
Res (::= -1) = ;: lim- 1 -d:..- = - --
3
936 ( h.1p1er 19 / Funttions oi a (.nmple, V.ui.1hll': Applic,llions

Therefore.

2rr iei:r/J ;r 2rr


3( I - e2:u' ·') 3 ,in(JT / 3) 3 J3

It simply takes practice 10 become facile wilh contour integration. The prob-
lems not only rellecl the methods Lhar we have used in this section. bu1 illustrate
other methods as well.

19.2 Problems

I. Show thar 12ft


0
d0
3 + cos 0
1T

Ji.'
2. Show that 12.Jr ----d0
o
~in0
= 0.
3 + cos 0

3. Show thal 11n a+ b sinO =


0
d0 2rr
(a2 -b2)1f2·
4. Show Lhat L7o cos 2 0
d0
, =
+ 4 sin· 0 2
,"T

5. Show that t' ., d() ., = !!..... 6. Show Lhat LV d0 i2FI dfl


Jo a 2 sin- 0 + b- co~ 2 0 ab 0 11 + IJ cos 0 - :r a+ h cos 0
7. Calculate alJ the residues of /(z.) = 1/(1 + z4 ).
8. Evaluate 1= -:x.
dx
l+x 4
using the contour shown in Figure 19.5. Use Lhe result of the previous problem.

oo \.:? rr r2 rr
J 1
00
9. Show that · ,, ,,dx = -. 10. Show that - ·- 4 dx = r;:;·
-oo(l + x-)- 2 o l+:c 2-.,2

1
00 \" 7T
11. Show that • dx = -.
-X' (x 2 - 2x + 2)2 2

12. Show that the iniegral along r in Figure 19.5 vanishes as R - oo if F(x) is a ratio of rwo polynomials witJ1
the degree of Lhc dcnomina1or at least two greater than the degree of rhe numera1or.
13. Show I.hat the integrals on CR and C( in Equation 10 vanish as R--+ oo and E--+ 0, respecli\•ely.
.x ,in wx
J
oo
14. Show I.hat
2
dx = rre-wa where w > 0.
-C\; X + a2
I 5 . Evaluate
/
00

- "'). X ~ -
cos rrx dx
,,
2x
and
+2
f '"° sin rr:c dx
,
-00 X ~ - 2x
.
+2
sin 2 ltH rr
f
,:x:
16. Show th:11 -·- - ; dx - -( I - e- 2w) for w ~ 0.
-co I+ x- 2
sin 2 wx ,.u., . . .
f
00 ,r
17. C an you show I.hat - --,, dx = :- (I - e- 2 ) w 1thoul re-doing tht prev 10u.c; prob ltm?
-oo a 2 + x- la

I~ An integral thot has occurred a number of times in the 1e,-1 is / = f


-x,
c:-.. .,in x dx. We will C\'aluate this integral
X
1fJ.J Summation of ri s 939

determine how many solutions lo the equation }'

there are within I.he unit circle 1:1 = I and we'll !cam how 10 dctcrrninc if a
polynomiaJ such as
X

has any z.cros whose real pa.rt is positive. This second problem occurs in the /,
-~v 21)
assessment of 1hc stability of mechanical and electrical systems, panicularly in
regard to feedback and conrrol sys1ems.
The method of Lhe summat..ion of series res1s upon the fact that cot 1t: has Figure 19.12
simple poles at z = 0. ±I, ±2, ... (Problem l). Considerthe contour, CN, shown The co111our u,cul to l'Vafua.tc 1hc intcgval
in Figure 19.12. Firsl we' II show lhar I cot rr zl is bounded on the contour. Along the in Equati1un ] . fau:h .!-ide of the square i" at
::i dhta.lilcc N + ~ (where N is un integer)
right(vcrtical)side,x = N + l/2andso.: = (N + 1/2) + iy, where-(N + 1/2) ~ from the origin.
y :5 N + 1/2. Therefore.

col 7r r = cos lrr(N + ~)- + irry]


s1n [Tt ( N + *) + i
~
JT\'
~
I tanh ny

col\ [( N + i )11 I cosh rr y -


= - - - - - - - - - - - - - - - -- - -
i sin( (N + ~ )rr] sinh JT y

sin[ (N + !)rr .I cosh rr y + i cos( (N + f )rr] sinh ,r y


J:r 2tr y
Using 1he fact lhat cos(N + l/2)n = 0 and sin(N + l/2)rr = ± I. we can see lhal
lsinhrrvl
I cot rr : I = ~ = I ianh rr v I
I cosh rr YI · Figure 19.13
A pic1orial rcpP ·cn1a1ion of 1hc
inequalities -1 _ lmlh !T)'.:: I.
But -1 S tanh rr .r ::: I (see Figure 19.13). so I cot rr ;:! ~ I on the 1igh1 venical pan
of 1.he con1our CN.
A similar argument shows !hat this is true of Lhe left vertical part of the contour.
also. Along the top side of CN. y = N + 1/2 and -(N + 1/2).::: x::: (N + 1/2).
and so

I cos rr x cosh JT y - i sin ;r.r s1nh rryl


I COi ]T:. I = - -------------
1sin JT x cosh rr y + i cos rr x sinh rr JI
Anticipating 1ha1 we arc going to let the value of N approach infinity, we
recognize that cosh rr y and sinh n y are essentially equal for large values of y =
N + I /2 (see Figure 19.14. which shows that cosh ,r y and sinh rr .r are almost equal
even for N = l). Therefore. \"

Figure 19.14
I cos rr x - i sin rr x I
I cot rrzl - ------- =1 An illu,1r;1ti11n of 1hc fact thlll cosh .'7 y and
I sin rr x + i cos i7 ,r I ~inh ;r y arc CM,cn1iully l'qLI:11 for y c..: I.

along the top side of CN with a similar result along the bouom side of CN.
944 Chapter 19 / Fun( tii ms of a Complex Variable: Applications

00 2
4. Show Lha1 L
n= -:.., (n- - 1
a 2 )-
., = ., ;r_.,
2o- sin- ,ra
+
rr cot rr a
2a 3
• where a
.
1s re.al and not equal 10 0. ± I. ±2, . . ..

oo I -I
S. Show that your answer to lhe previous problem reduce..,; lo'°' - 4
L.,, 11
= !!__
90
as a - 0.
11=1

00 2cosech 2 ;ra
6. Show 1.ha1 L (11-., + 11 )- = rr coth
/'J=-00 2a 2
.,
3
rra
+
1r
"I _
£U
2
where a > 0.

oo I JT coth JT a rr 2cosech 2 ,r a I
7. Show that L -, , = + , - - whc.rc a> 0.
n--=1
(r,- + a-) 2 4a 3 4a· 2a 4

8. Starting with the defining equation for the Bernoulli numbe~. _x_ =
eJ-1
L Bnx" . show thal.
11!
n=O
~ (-1) 22,, 11
,,_ 1 I x x·' +
cot X = L.,, (' ) I 8 211X 2 = - - -3 - 4 - . ..
,r={) .f.11 • X )

oo I
9. Evaluate S =L 6 .
11:.I n

~ 2
10. Show that L.,, ,, 4
11
_
04
= rr colh
2o
,r a
-
rr col '!Ta
ill . where a
.
1s real and not equal 10
J
0. ± I, ±-. _. __
n·....c "'-

11. Lei a - 0 in the previous problem to show that L 1n-I = !!.....6 _


oo

11"'-I
2

12. Derive Equation 7.

13. Derive Equation 8.


00

14. Show that L


,, __ '- (11
( - I)"
a)-
., = rrsm·,'1(1
_cos
., rr a . where a 1s. real nnd not equa.l to 0. ± I. ±2 .....
2

oo B ,,
15. Use the defining equation of 1hc Bernoulli numbe~. _x_
e-r-1
= '°' _..!!!.._. 10 ~how 1hat
L..,, n!
11=0
3
I x 7:c.
cosec x = -X +- +
6
-
360
+ -· ·.
00 1
16. Show thar I:--
(-1)"+
,,=I n
4
- = -.
7rr.J
720

(-1)"+1 rr2
17. Let a ➔ 0 in the resull of Example 3 10 show th:ll Loc

n2
= -12
11=1

I
18. Use lhc resuh of Problem 3 to show 1hat L oo

n>=O (2n
, = -8 .
+ I)~
1r1
19.4 Locatio n ol Zeros 947

Example 2:
Re-dn Example I with C given by 1:1 = 3.
SOLUTION: In 1his case. all Lhe pole:~ of J'(:)/j(:) lie within C, and the
expression for .f'(:)/f (") in Example I shows thal the sum of I.he residues
is -3. in agreement wilh Eq11:i1jon 4.

Let"s sketch the proof of Equa1ion 4. Suppose thal / (z) has a zero of muhi-
plici1y ,1 al:= o 1• Then the Taylor expansion of J(z) has the fom1

Therefore.

J '(-) 11c,, (- - a,)" - J + (n + lkn....-1(: - a;)" + ···


- - =---- - -- - -- - - - - --
/( ::: ) C, Oj) + C +1(::: - 0;) +1 + , --
1
(::: -
11
11
11

II
= - - + 1 + 0(;: - ll;)
:::. - tl;

and

{'(:) }
Res
{ -{(:::.) DI :=a, =11;

Using 1he same approach. if/(.:) has a pole of order mj at .:: =hi, then ( Problem 4)

Equation 4 follows by evaluating the in1egraJ by 1he me1hod of residues using Lhe
above results,
We evaluated the integral in Equation 4 explicitly for che case f ( :) =
z/(2z 2 + I). but there is another. geometric, way to evaluate it that is often more
convenient. Fir~!. we recognize that/'(:)//(:)= d In f(z.)/dz.. and write

J J'U.) dz = J. d In /(z.) d:
k f(z.) fc d::.

=id In f (z) = l:::.c In f (:.)

where b.c In f U:,) is Lhe change in In f (z.) as z makes one circu il around C in u
coun1erc\odwise (positive) direction. Now wri1c In f (z.) as

In f(~.l = Jn !f (zll + i arg f(:)


948 Chapler 19 / Fun er ions of a Complex Va riab ll': Appl it -.i Iit 1ns

y so that

tl.c In J(z) = 6.c In IJ(z.)I + i6.c arg j(z)


There is no change in If (z)] as z makes one circuit around C. but arg J(z) may
very well change (as we'll soon see). Therefore, we may write Equation 4 as
.t'

-
2,rj
I i C
/'(z)
--dz
f(z.)
=Z - P
I
= -6.c
2;r
arg f(z)
.
(5)

This result is known as the argument principle. or the principle of the argument.
As we implied above, Equal.ion 5 has a nice geometric interpretation. Let C,u
V be the image or C in Lhe w-planc under the mapping w = J (z). ln other words.
=
as z traces our a path C in the z-plane, w J (z.) traces our a path Cu, (the image
curve) in the w-plane. Tbe image curve will also be a closed contour. but it need
not be one-to-one with C. as in the case of w = z. 2• where w makes 1wo circuits
around the origin in rhc w-planc as z makes one circuit (Figure 19.15).
Lei's use Equa1ion 5 10 sec how many zeros of

f(z) = z2 + 2z + 2 (6)

lie within the circle i.:I = I. (The zeros of j(z) are clearly z =-I± i. but we'll
consider more chuUenging cases soon.) Let z = x + iy and wri1e

Figure 19.15 w = f(z) = u(x, 11) + i v(x. y)


An illus1mtion of the fuel that
w == /(z) == z2 makes a complete circui1 = x 2 - y2 + 2.x + 2 + i (2.ry + 2y)
in the w-planc when ;:: mak~ one half a
circuit in !he :-plane. We wish to plot was z make..~ one circuit around lzl =
I. We can do this by writing
x and y as x= cos 0 and y = sin 0. 0 .:5 0 < 2:rr. in which case we have
u (0)= cos 2 fJ - sin 2 0 + 2 cos 0 + 2
(7)
v(0) = 2 cos 8 sin tJ + 2 sin 0

We now plo1 v parametrically against 11. (This can be done easily using any
computer algebra system.) The result is shown in Figure 19. I 6a. Note carefully
that 1hc change in arg w is zero because the curve C 111 does no1 enclose the origin
as you can see in Figure 19.16a. where a few of the values of arg w are shown.
Thus, there are no zeros of f (z) lying within the unit circle.
Now let C be the circle lzl = 3. l.n rh..is case. x(0) = 3 cos fJ and y(8) == 3 sin 0,
and Lhe curve Cw is shown in Figure 19.16b. Note that Cw encircles the origin two
=
t.imes, so !).c arg w 2 in !his case, having picked up the two :z.eros off (z) al
z=-l±i.
Thus, we can express Equation 5 as

Z-P=N (8)
19.4 Locarion of Zeros 949

\J V

Figure 19.16
(a) A plot of 11(.r. _v) againsr u(x. y) in
0 0 =
lhc w-plane for w f(z) = z2 2z + 2
for one circui1 of:. around II unil circle
u
" in lhc :.-plane. Tihc numbers O and rr
rcprt:-')C[ll ,the values of() in the parametric
reprc~ntation of 11(0) and i,(f)) in
Equ.arions 7'. (b) Similar to {a). bu1 in this
(a) (b)
case z traverses rhe ci'rde I ;: I = 3 once .

where N is the number of Limes thal C 1, .. the image curve of C. encircles the origin
al w = 0. The quantily N is somel.imes c:aJled lhe winding number of w = f (z.).

Example 3:
How many solu1·1ons does 1he equa1ion

have within the unit circle?

SOL u TIO N: We wish to determine how many zeros the function


f (::.) = e~ - 3: 2 has within the unit circle. There arc no poles. so Equation 5
will give us the number of zeros. Write

w=f(z)=e=-3z 2

==- e.r cosy - 3(x 1 - / ) +i (ex sin y - 6xy)

= 11(.r, y) + i u(xy)

Let x = cos 0 and y = sin 0. 0.:: 0 < 2rr, and write


u(0) = tP"'' cos(sin 0) - 3(cos2 0 - sin 2 B)

u(0) =ec'" 11
sin(sin 0) - 6 co~ 0 sin 0
u
A parametric plot of v(8) against 11(8) is shown in Figure 19.17. We
sec that C w encircles the origin 1wo times, and so there arc two zeros of
f(z.) = =
e~ - 3z2 • or two solutions to lhe equation e! 3z 2 • rhat lie within
I.he unir circle Iz/ =I. Figure 1 9.1 7
A plot of l'(0) ugaim.1 u(0) in the w·planc
as z. trJ,·crsc5 the unit circle once for lhc
function given in Example 3.
There is a companion theorem to the argument principle. called Roucl,e 's
theorem. that is a great aid to 1hc location of lhe zeros of a fonction.

Rouche's theorem: let j(z.) and g(·) be analytic inside and on n closed
cun•e C. and s11ppou that lf(z)j > lg(z)I all along C. Tlre11 f (z) and
f( z) + g(z) have the same number of zeros within C.
950 c h.,ptt·r 19 / Func11n11" ui a nmplf•, V.1ri.1bl ; Applil ,, tions

TI1e proofof Rouche·s theorem is developed in Problem 9. The following Example


illustrates the utility of Rouche's theorem.

Example 4:
How many roors to the equal ion

arc there within 1hc unit circle'!

SOL U T ION : Let


JU.)+ g(:l = :8 - 4:5-!.. __ :? - I

We want If (:)I'> lg(z.)I on the unit circle. so choose f(':.) = -4.: 5 and
.11(.:) = : + .:
8 2- I. This choice assures us 1hat jf(:)I > g(:)I on the unit
circle bec:JUse

on Ihc unil c.:irclc. According to Rouche's theorem , J (: ) + g (:) = : 8 -


4 ~ + -2 - I has rhc same number of zeros within 1-1= I as J (:. ) = -4::5 .
But f(:) has a zero of mutIiplicity 5 at:= 0. so .:K - 4.,.S ~1 - I= 0 has
five roots within 1-1- I.

Example 5:
Show that all the roots of
:
1
- 2.:-1 + 5.: - 9 =0
lie between ,the circles l~I = I and 1::1 = 2.

SOLUTION: Conj der lhc circle lz l = I. :rnd choose f(:) = -9 and


g(:) 7
=:
- 2z + 5;:. On 1he circle
4
1~1 = I. we have

According to Rouche's theorem, f (:) + K(Z) has rhc ~me number of zeros
=
inside 1-1 I .is J ('::.) =
-9. wbich is zero.
=
Now consider 1the circle 1-1 2. and choose /(:) 7 and gL:) = =-
-22:,1 + 5.: - 9. On. ~he circle 1-1 2. we have =

In Ihis case, f(z.) has 7 zeros within l;::1 = 2. so all 7 roo1s nf - 7 - 2;:°' + 5: -
9 = 0 lie between 1hc circles 1-1= I and 1:1= 2.
952 Chapter 19 / Fum 1ion~ of a Complex Variable: Applic.alions

extremes (w ➔ ±oo). Notice that u(w) and v(w) are both negative as w ➔ +oo
and that u(w) < 0 and v(w) > 0 as w ➔ -oo. as indicated in the figure.
Now let's investigate the behavior of arg was we travel aJong the curve from
w = oo to w = -oo in Figure 19.19. which corresponds 10 the contributjon from
the integral along the vertical axis in Figure 19. l 8. The arg w is shown as Lhe
=
angle</> in Figure 19.20. If we start at arg w 0 and follow the curve to w oo, =
arg w increases from O to +3rr /2. and if we SI.aft al arg w and follow the curve
to w =: -oo. arg w goes from O10 -3;r /2 (Problem 21 ). Therefore, if we start at
figure 19.20 w = +oo. where arg w = 37T /2 and move along 1hecurvc 1oward w = -oo. where
An enlargcmem of the curve in
arg w = -Jrr/2, weseetbatlhechangeofarg w iscquaJ to-J;r/2- Jrr/2 = -3rr.
Fi gurc 19 .19. showing </J = urg w and how
it varies a!-: you move along the curve This resu\L together with 3rr from the semicircular arc, gives a net change of 0.
toward ii~ 1wo extremes. The arrows poinl Thus, we conclude that there are no zeros of Q(s) = s 3 + 6s 2 + !Os + 6 in 1he
in the direction of dccrc.using w a.long the
right half plane.
path.

Example 6:
Determine if

has any zeros lhat lie in the right half plane.

SO l U TIO N: Ac.:cording 10 E::qua1ion I 0. the con1ribution 10 :1rg 1r from


the semicircular arc is 3.rr. The contribution lo arg w from Lhe imaginary axis
is found from
W --+- 00
V
Q(iw) = -iw 3 - 2w
2
+ iw + 4

which gives

and v(w) =w - wJ

IV -lo 00 Figure 19.21 show~ Lhc imnge curve in Lhc w-planc. As w --+ ±oo.
dv/du - 3w/4--+ ±oo. which says !hat the curvt: in Figure 19.21 i~
Figure 19.21
vertical at i1s two extremes ( w = ± ). If we :-tan wilh arg w = 0 and follow
The ima.ge curve for Example 6. No1c that
this curve docs no! enclose the origin. the curve roward w = +oo. then we see that arg w--+ - rr /2. Notice that
because I.he curve in Figure 19.21 does nol enclose lhe origin. arg w starts at
0. build~ up to a small po.-.itivc value, and then dccrease-s lo zero and continues
10 decrease toward -rr /2 (Problem 22). Similarly. if we s tart at arg w =
0 and
follow the curve toward w = -oo. 1hen arg w goes through small negalive
values. passes through zero, and then increases toward rr /2 (Problem 22).
Th us. if we sLan nt w = oo. where arg w = - rr /2. and move along I he curve
1oward w = - oo . where arg w = :;r /2. we see that the change of arg w is
equal to +JT. This result 1oge1her with the 3rr from the semicircular arc gives
a to1al of 4rr. Thus, there arc two zeros of Q(s) = s 3 + 2.\- 2 + s + 4 in the
right half plane.
954 h.ipt •r 1'9 / Fun( rio:is oi a ComplelC \l,1ri.:ibl1•: Applic ,11ions

14. Use Rouche's the.orem lo determine how many zeros the fonc,tion ( 1 - 5.~ I I ha.<i in the ::lnnulus I < lzl < 2.
Sec the hint to rhe previous prroblcrn.
15. Use Rouche's theorem 10 prove that rhcrc are II rools of the equarion e~ = o:" within the circle 1:1= 1 if o > c.
Compare thi.s rcsul! to the rcsuh of Example 3.

16. A classk applic;arion of Rouche ·s theorem involves the proof of thefw1damm1al theorem ofolgebm. th:11 every
polynominl of nth degree \Vilh complex coefficieflL<i. p(:) = z" + u11 _ I :'1 ·· 1 + a,,_ 2z."- 2 + · · •+ a I ~ + a 0 0. =
has exactly II comple.'\ rroots. Use Rouc-hc's theorem to prO\'C this rc-:uh. Him: Take /(z) =
z" and
g(;:) == a,1-1:: ' - 1 + · · · + 0 1z + ao.
17. Detcnnine if Q(s) = s. + 9s 4 + 24r\ + l 2s 2 - 60s - 60 has any zeros thal lie in the right half plane.
18. Determine if Q(s) = s 5 + 3.s + 5,l'J + 5s 2 + 1r + I has any zeros in tht! ri£hl half plane.
4

19. Detcm,inc if Q(s) = s.i + s3 + s~ + Ws + 10 has any zeros 1hat lie in 1he righ1 haJf plane. Caution: Be sure
10 in\"cslic.ale the image curve as 11 ~ ±oo.
20. Determine if Q(.t) = _._(., + 2s ~ · 3s 3 + s;! + 6s + 2 has any zeros thal lie in 1he right half plane. Co111io11: Be
sun: to invcst,igatc the imal!e curve .l! w ~ ±co.
21. Use the rclat·ion arg w == tan- 1Lu(111)/11(w)"ll10 plot arg w against w for the curve in Figures. 19.19 and 19.20 .
Be sure to use the appropriate branch ,of dte arccangent.
22. Use the rela1ion arg w = 1ar1- 1(u(w)/11{w)) lo plot arg w against w for the cur. e in Figure
1
19.21. Be sure to
use the appropriate branch of I.he arctangen1.

19 .5 Conformal Ma pping

We know lhal if /(z) = u(x. _v) + i v(x. y) is: analytic in some region R. then
11(x. _y) and u(x. y) satisfy 1he Cauchy-Riemann equations

<Jl 011 a
-=--
a,v oy ox
in R. If the second derivatives of u(x. y) and u(x. y) exist and are continuous in R,
rhen ii follows from the Cauchy-Riemann equations that both i,(.r, y) and v(x. y)
satisfy Laplace·s equation

in R. II may not be surprising, then. thal the lheol)' of annlyLic functions can be
applied to two-dimensional boundary value problems involving Laplace's equa-
1ion. By 1wo-dimensional. we actually mean a lhree-djmensional system that is
symmetric in one of its dimensions so that its cross section is invariant in that di-
mension. Before we can discus.-. this type of application. we must review the idea
of a mapping from 1.bc .:::-plane to lhe w-planc from Chap1cr 4.
Lei's start off with some simple mappings and t.hcn build them into more
complicaled mappings. Consider some region in the z-plane and see how it is
I 9.S Conformal Mapping 955

Figure 19.22
The mapping w = /(:) = : + a 1runsla1cs
X ll a region in the z-plane a unit,; 10 the right
in the w-pl.anc.

rransformcd into the w-plane under the mapping w =f (:). For example. the
mapping

w = J(z) =:+a (I)

simply translates the region a units to the right because each point z in R is
incmised by a units, as shown in Figure 19.22. The point in the w-plane cor-
responding 10 : in the z-planc under the mapping w = f (z) is c.illed the image
of;.::.
The mapping

(2)

does two things. If we write this mapping in polar fonn by wri1ing w = pc'rfJ.
b = lhlei.8. and:= re 10 • then

We see that the magnitude of ;e: is scaled by a factor of lbl and :: is also ro-
late<l through an angle fj. This mapping is called a .ffali11g-m1arioTJ mapping.
Figure 19.23 shows the mapping of Ollr test region in Figure I 9.22 by u, =
(I + J'3 i ): = 2ei:r/ J:.

'\'

w = 2 ei.rr/3 z

Figure 19.23
The mappinl! of the tc-st region in
X /,/
Figure l 9.22 by lhc T:lling-ro1ation
=
mapping w (I+ J3 i)· = 2i ~/J. _
19.5 Conformal Mapping 957

y y

C C

X X X

(a) (b) (c)

Figure 19.2S
TI1e lltree c~t'4!s coasidc.rcd in Example I. (u) tr R < I c I- then the point ;:; '" 0 ljc - outside the
= =
disk I z. - c I ..:: R; (b) if R I c I. then 1hc point z 0 lie=<. on 11ilc clrcll! I z. -- (·I = R: and
1

(i;: ) if R > I c I. 1twn 1hc poin1;: = 0 lies inside the disk .: - c I < R.

Now let z = 1/w 10 gel


(5)

The three ~a.::cs to be discw,scd depend upon the value of R 2 - lcl 2 • as $hown
in Figure 19.25.
(a) When R < I c I. divide Inequality 5 by I c 12 - R 2 to gel

* cw c,. w• I
-WW + ., ., > , , ::> 0
I c 1- - R- I c 1- - R-

which can be transformed into

,. cw+ c•w•
U'JJ" - ---- < ---- (6)
1C 1
2- R2 2
I C 1 - R
2

by multiplying both .'-ides by -1. Now add I c 12 /(] t· i2 - R 2) 2 to both sides


of Inequality 6 and Uike the square root to obtain

(7)

TI1tt'-, the in1erior of a disk in the z-plane is mapped into the i nt.erior of a disk
in the 11;-plane if R < I,· I. (See Figure 19.26.a.)
(b) When R = I c I. the point z = 0 lies on the circle I;:: - ,.·I= R. In 1ha1
ca-.c. lncquali1y 5 become~ simply

l.f we let c = a + i h and w = 11 + i u, this inequality becomes


1
au - bv > -
- 2
958 Chaprcr 19 / Fun lio ns of a Comp! "x Vari able: Appli atio n

y t"

I (.'

\V = 1/z II

\ •
X

(ii)

_\'

w = 1/z

(b)

' y V

w = lh:

(c)

Figure 19.26
An illustration of the mapping di sc ussed in Example I. (a I R I c I and the interior of a disk
=
in the --plane is m:ipped inro the inrerior of a di, kin the u•-plane. (b) R I c I and rhe disk in
the ;: -plane is mapped inro the region below a strnighr line in I.he w-plane. (c) R I c:: I and the
interior of a di$k in rhc ::-plane is mapped inro the cxrcritlr of a di. k in the w-plunc.
960 Chap1er 19 / FunU ion'.) ()( a Complex Vari..1blc: Applications

z 1(t) and z2 (t) a.re mapped in10 1he curves

and

The slopes of w 1(t) and w 2 (t) at to are given by

and

1
Expressing (dwif dt),-,0 •• / (10 ),. and ~dz.if dt ),-,11 in Equation 9 in polar fonn gives

wi1h a similar result for (dw 2 /dr),=rc,· Provided that J'(l 0 ) -:/- 0. we can equate the
arguments on the two sides of Equatjon 11 to wri1e

( 12)

The same procedure for Equation I O yields

(13)

Sub1.racting Equations 12 and 13 yields

( 14)

Thus. we see 1ha1 1he angle between two smooth curves al Lheir point of intersec-
tion. t0 • in the z-plane is carried over 10 the w-plaoe by a mapping w = f (z) thal
is analytic. provided Lha1 J'(r0 ) -:j:. 0. A mapping with this propeny is said to be
confonnal, or a,,g/e preservi11g.

Example 2:
Where is the mapping defined by

3z + I
w= - -
2- - I
conformal?

SOLUTION: =
The func1ion w f(z) is anaJyric everywhere except at
;: =
1/2. Furthermore. dw/d~ =
-5/(2z. - 1)2 never equals zero, so the
mapping is conformal everywhere except al z 1/2. =
964 ChJpter 1'3 / Funclions oi a Complex Variable: Applic.itions

I 0'
J'
I • 11 I e D' N' •
G

.,. = II:

l
) ' £ E

...,.
_,
. .
J
JI -I 8'

7. The region exterior to the unit disk Ii. - ~ I~ ~ in the positive x half pl.:ine into the infinite venical strip
0 _::; u _::; I by w = 1/z..

w=...£.2 (-+.l)
.. ;:

D E A'
-I
- ;c • (j
• 11

8. The upper half plane with I.he half disk I·I ::: I, 0 ::: 8 ~ ;r remowd onto thi: upper half plane

by w = la ( z. I)
+~ .

• 1·

C'

--•(- ---t----- - ·- /I

• A'
.A B
• - I
• - - _r
8'
-I

. . . i - ~
9. The upper half plane into th~ unit tl1!>k ccntcrcd at 11 = 0. u = 0 hy w = .-_- .
I -
19.5 C nforma l Mc1ppin8 967

Example 4:
Verify the mapping shown in Entry 14 in Table 19.1.

SOLUTION: Let;:= reiO. so t.ha1 w = In r + i0, or


u = lnr and I I = 1:1

The two arcs are mapped into vertical lines 11 =


In r 1 and 11 = In r'2 and the
= =
angles 8 1 and 02 are mapped inro v 8 1 and v 02 • yielding the rectangle
=
shown in EnL.ry 14. Note thm if r 1 0 and r 2 ~ = . 1hcn rhc entire angular
arc 0 1 ~ 0:: 02 i~ mapped into the infinj1e strip 0 1 _:: : 11 .'.::: 02 .

The rno~t important property of conforrnal 1ransfom1a1ions and the one thal
we shall exploit in the next section is the following:

Com·ider some region R: in rhe :::.-p/a11e rhut iJ mapped imn a region R 11 , in


the w-plane by a conformal rro11.tforma1ion w = J(z.) =
u(x. y) + 1.1(.r, y).
ur 1his moppi11;: be 011e-1o-011e. so thar the im·er.1t• off(:). : f - 1(w).=
exists.
If tf, (x, y) .w Ii.efies

Ill D.

then <t>(u. ti) ,\ atisjieJ

where <l>(x. y) = =
et> (11. u) <t> ! 11(.r, y). u(x. y) ). F11rthemwre. 1/ </J(.r. y) =
g(x. _r) along some boundary curw C:. in tlte ::.-plane, then <l>(u, v) =
g(x(u. v). _,·(11. 11)) alo111-: the inwf:e curve Cw in rhe w-pla11e.

Problem 20 lakes you through the straightforward, but lengihy. proof of this result.
Before going on to lhe next section, we'll present a short problem to illusrrate
the above result . Consider Figure I 9.28. which shows the cross-section of a long
cylindrical conducting sheet touching. but insulated from. a planar conducting
sheet perpendicular to the page. Let the planar sheet be held at a potentiaJ \/0 and
the cylindrical sheet be held at a potential V1• We wish 10 solve Laplacc·s equation.

Figure 19.28
The cross-section of a long cylindrical
for the potential in the shaded region in Figure 19.:28. con<lucling sheet touc.:hing, but insulated
None of the met.hods that we have learned to solve partial differential equations from. a planar conduc1ing sheet
will work here. However. if you scan through Table I 9.1. you'll see that Entry 7 pcrpendicul:ir to the page.
19.5 Conformal M.1pping 969
12. Verify 1he mapping shown in Entry 7 in Table 19.1.
13. Verify rhe mapping shown in Entry 8 in Table 19. I.
14. Verify the mapping shown in Entry 9 in Table 19.1.

15. Verify the mapping shown in Entry I O in Table 19. I.

16. Verify the mapping shown in Entry 13 in Table I9.1.


17. Consider the two parrunclri.£ed curves. C 1: z 1 = t + it 1 and C~: z2 = 1 + 2it (t ~ 0), in the z-plane. Show
explicitly that the angle between these 1wo curve~ al their point of in1crsection at, = 2 is preserved under the
mapping w = z. 2.
18. Consider the two parametrized curves. C 1: ;: 1 = t and C2 : ;: 2 = ii (0 ~ t ~ I), in the ;:-plane, Show that the
angle between these two curves al !heir point of incersec1ion at z = 0 is rr /2 radians . Now consider the mapping
w = z2 . Show that the intersection angle between the image cwves in lhe w-plane is rr radjans. Why isn·1 the
intersection angle preserved?
19. ln the next problem. you need to use lhe re\a1ion

I)·'(-..:.) 12
2
= (a-Bx11 ) + (au)
-
ax
2
- --(a") + (au)
fJy
-
2

av
.
2
Derive thi .:; result.

20. In this problem we'll show lhat Laplace':; equa1ion remains invariant under a conformal transformation.
=
Let w = f(z) u(x. )') + i 1 tx. y). Because f (;:.) is invertible, we can wri1e u = 11(x. y), u u(x. )') =
=
and x = x C11. u), y y(tJ. t ). Now lei </, (x. y) sa1isfy Laplace· s equation in some region R::. und let
<1>(11. 11) = lf,(x(u. v). y(u. v)) be me function that results from rp(x. y) under the conformal transformation.
. a2tf, a2.r/J . . a24> a2 <1> .
We wish 10 show that - - + - ,, = 0 implies I hat - - + -:::--:,- = 0. as.sunung lhat w = f (;:.) is
,h 2 av- a11 2 ou-
. l App l y le
contorma. h c hatn
. rue
I .10 <t> (u, u) = tj>(x(u. u). 1"{11, u)). to sow
h I a(/J = -
t 1at -
au + -
J<l> - au
a4> -
. ax 011 clx au cJx
#>
= -cl<%) -011 + -cl<!> nu . .
and -
ay 011 oy au -iJy . Now apply the chain rule a second 11mc to get

with a similar equation for o1 ,f>/ay2. Now show 1hat

2
J <t> [au clu
+ 2- - - - + - -
011 c)u]
auau dX ox ay i:)y

<1>
Now show Lhat this resuh reduces to -:---:;-a2 + -a2<1> = l/'(z)l 2 (o--,
2
a2<1>)
<t> + -- .
ax- ay 2 011- a2
974 haprer l 9 / Function$ o( a Complex Vari.:ible: 1~pplic liMS

We could also have solved the problem in Example 3 using Poisson's integral
fonnula for the unil circle (Problem 6).
Let's do one last Example.

y Example 4:
Suppose we have two parJllel conduc1ing cylin<lrical sheers as shown in
Figure 19.35. Calculate the clecrrostatic po1en1jal in the space surrounding
these sheets if Lhe larger one is held at a potential V1 and the smaller at a
X
potential V~-
SO LU TIO N: Entry I I of Table I 9. 1 maps the region surrounding
the tylinders into the annular region between two conceniric cylinders.
According to Entry I l. the mapping i~ given by
figure 19.35
;:: - {I
The geometry referred 10 in fa:ample 4. w= --
a;:: - I

where

a= I+ 14 - ~ =3
7+2
The outer radius of the annular region is I and the inner radius is

14- I - 12 I
R= - - - - = -
7- 2 5
Furthermore, the smaller circle in the ;:;.plane is m~pped onto the oulercirck
in 1he w-plane and the larger circle is mapped onto the inner circle. so the
potential is V2 on the large circle an<l V1 on the smaller circle.
Letting p be the rn<lial coor<.lina.lc in the w-plane. rhe elec1ros1atic poremial
in the annular region is given by (Problem 2)

ct>(p) = A In p + B

The potential is V~ at p = I and V1 at p = 1/5. so


v, - v,
<f>(p) = ----
Jn 5
Jnµ+ V)
-

Burp= 11'1. so
0

From w = (;: - 3J/(3z - I). we find that


19.h Confom1.1I Ma1>ping ,rnd BoundJry \l.1luc Problem~ 975

and so

V) - V1 (x - 3) 2 + r2
rp(x. r) = ----
2 In 5
In
( x - 1)-
., ·
+ 9y 2
+ V--,
-

This solution is ploned in Figure 19.'.\6 for V1 = 0 and V2 = 100.

Figure 19.36
Tiu:: solu11on given in Example 4 for
19.6 Problems V1 = 0 and Vi= 100.

The first nrn problems prese111 f\rn simple Diricl,/e, boundary va/111' problems ,,·hose rt'sults ll"ill be used 11
1111111/Jer of times.

I. Solve Laplace ·s equal.ion in 1he strip -oo < 11 < oo. 0 ~ u _::. a. a">~uming tha1 '1> (11. 0) = <l> 0 and <ll (11. t1) = <t> 1.
2. Solve Laplace·s equation in the annulus of radii R 1 and R~, with R~ > R 1• a-'suming that <l>( R 1• 0 ) = ct> 1 and
<t>(R2. 8) = <l>2,
3. Verify the mapping th;:it is used in Ex::imple I.
4. Right aflcr Example I. we solveJ Example I by using in = :: 2 to m.ip the firs! qua<lrant into the upper half
plane and then using Poisson\, inleEral formula for 1he upper half plane. We can by-pass the use of Poisson 's
integr..il formula by using the inven-c lr,msfonnation in Entry 2 of Table 19.1. Use the se411cntial mappings
u 1 = : 2 and 11· 2 = ln w 1 to solve Example I.
1

S. In the text. we used Poisson·s integral formula for Lhe upper half plane to derive Equation 3. the temperature
· ·b u11on
d1s1n · m · t hc upper hall· 11,1-p Ianc w hen ""(11.
,+, o) = ~ To " <
0
. Dcnvc · J (Jy so I vmg
· Eq u::i11on i.. · 1....<lp I..u.:c ·s
1 ,

T1 11 - 0
e4uation in polar c:oordinatcs.
6. Use Pois:-on·s integral formula for the uni1 circ:lc to derive Equation 7.
7. Detem1ine 1l1t: steady-swte temperature dis1ribution in the wcJgc shown in Figure 19.37.
8. Dc1cn11inc the steady-state tcmpcrmurc- dis1rih111ion in the annular segment ,;;hown in Figure l 9.J8. The arcs
arc insula1ed.

. \' \'

X
To .\'

Figure 19.37 Figure 19.38


The wc:<.lgc: rcfcrrt·d 10 in Prohlcrn 7. TI1c rc1;ion referred 10 in Prohlem ~-
976 Chapter 19 / Functions of a Complel( Vari.able: ApplicJtions

y
lY j

I
Ti I

Figure 19.39
To X
()

Figure 19.40
Vi
r X

Figure 19.41
The n.-gion referred 10 in Problem 9. The region refem!d to in Prohlcm 10. The region referred to in Problem I I .

9. Determine the steady-slate temperature distribution in the region shown in Figure I 9.39. The arc is insulated.

to. How would you modify E.xample 2 if the region in the ;:-plane were like 1hat shown in Figure 19.40 rather than
the one shown in Entry 4 in Table 19.1.

11. Determine the electrostatic potential in the region shown in Figure 19.41.

12. Verify the mapping used in Example 2.

I
Vo
13. Generalize Equation 6 to the case <ll({. 0) = \/ 1

V2

14. Show that Equation 6 can be written as


Vo - V1 V1 - V.,
4>(u. v) =- - - arg(w~ +a)+ - --"" arg(w2 - a)+ V2.
rr rr

15. Show that the resulting potential <f,(x. y) in Example 4 satisfies the two boundary conditions.

16. De1em1ine the electrostatic potential in the region surrounding two parallel cylindrical shee1s of uni1 radius
whose centcrs are 4 units aparl. Lei one sheet be held at a potenrial tJ, 0 and the other at -r/Jo. Show that your
result satisfies 1he boundary conditions. Use a CAS to plot your result.

l 7. Consider rwo parallel cylindrical sheets whose cross-sections are shown in Figure I9.42. Let the radius of the
inner sheet be 1/2 and 1hat of the outer sheet be I. and let the surface of the inner sheet be held at a potenLial V0
and t.h.u of I.he outer .c;hect be V1. Calculate I.he potential between the cylinders. Show that your resuh salisfies
I.he boundary conditions. Use a CAS to plot your result.

Figure 19.42
The gcomc,ry associatt.xl with Problem 17.
19.7 Conformal Mapping and Fluid Flow 977
I)'

Figure 19.43 Figure 19.44


The region referred 10 in Problem 18. The region referred 10 in Problem 19.

18. Dctcnnine the temperature dislribution in the region shown in Figure 19.41 Take the inner cylinder 10 be
ccntercd at x = \/4 and iL.:: radius 10 be 1/4 and the radius of the outer cylinder 10 be I. Show thal your result
satisfies the boundary conditions. Use a CAS to plot your result.

19. Find the clcctroslatic potemial within the region shown in Figure 19.44. Take the rndius of the arc to be I.

19.7 Conformal Mapping and Fluid Flow

One of I.he nicest applications of confonnal mapping to physical problems involves


(two-dimensional) lluid flow. The partial differential equations that describe the
flow of fluids are derived from mass balance and momentum balance considera-
tions and are fairly complicated in general. However. if we can ignore viscosity
and if the fluid is incompressible. then the velocity of the lluid under steady-state
or stationary conditions is derivable from a 1•elocity poremiol. </) (x. y ), such that

a<1>
LI=- and (I)
.r ox
where vx and v_1• are the x- and _v-components of the velocity of a small element
of the fluid. The mathematical condition that the lluid be incompressible is that
div ,, = 0 (Problem I), which in rwo dimensions is

(2)

If we combine Equations 1 and 2, we see that the velocily potential satisfies

(3)

C ,a
978 Chapter 19 ,' ru11( 1i1111 · o a Complex V,tri,1hli•: :\pplii ,1tinn~

II turns out 10 be convenient to consider </J(x. y) to be 1he real part of J crimp/ex


potellfial

~Hz)= fl(x, y) = <t>(x, y) + i ijJ(x, y) (4)

If Q (,r. _r) is an analytic function of::= x + iy, then not only does </J(x. y) salisfy
Laplace's equalion, bu1 so does rf,(x, y):

(5)

Now we know from Section 18.2 t.ha1 the two families of curves

and

are orthogonal. (See Problem 2, also.) Therefore. because of Equation I, the


velociLy vector v = i u, - j u_1. = grad </). and thus is nom,al 10 the </J(x. y) = c 1
curves. and consequently, 1angcn1 to the iJr(x. y) = c 2 curves. Therefore, the fluid
flow follows along I.he i/J(x. y) = c1 curves, which are called strem11/ir,es. The
funcl ion iJ, (x. y) i Lself is called the strewn f1111ctio11.

Example 1:
Interpret the flow dc~cribc<.I by the complex po!cnlial

Q(::) = t,•oz
SOLUTION: The velocity potenria.l and the stream function are

nnd y1(.r. _\')=Vo_\'

The velocity is simply v., = v0. which represents unifom1 motjon in the .r
=
direction. The s1.rcamlinc-. ,uc 1•0 y cons\Unt, which arc horizon1al lines.
Thus, the complex potcnli nil Q l:-) =
v0z represents unifonn now in the x
direction. The complex potential Q (z) =
1·0 ,· " ~ repn:~enl'- flow al an angle
1

a with rcspccl to the x U..'(is (Problem 3).

Example 2:
rnterprct the flow described by

SOLUTION: The slre:m1lines in 1.hi!. ea~~ are given by 1he family of


rectangular hyperbolas

i/1 (x. y) = 2xy = c


1q _7 Conformal Mapping ,md Fluid Flow 981

becomes the complex pott:ntial,

in the :.-plane. We wanr Q(z) ➔ u0 ;: as r - oo. so wt! choose a= 2. The


streamlines are given by

i/t(r. B) = ,,0 (r - ~) sin 0 = cons1an1


and are shown in Figure 19.46. Even 1hough we mapped from one upper half
plane 10 another. rhc problem is symmetric about the x axis and so we show
streamlines both above and below the .x axis in Figure 19.46.

Problem 6 has you show lha11he speed L' of a fluid element at any point in the
fluid is given by

v=l ~~ I (6)

which for the complex potential in Example 4 gives

Thus. we see lhal u = 0 at the points ~ = ± I. Points at which v = 0 arc caJled


stagnation poims. Problem 11 asks you 10 show that 1hc stagnation points of the
flow depicted in Figure 19.46 occur al the front and back end-. of the cylinder. The
comer of the right-angle region shown in Figure I9.45a is also a stagnation poinl.
The map

(7)

has played a venerable role in aerodynamics. By lerting z. == ae;o, we see that

w =-
2
I ( + -I)
a
0
cos f) + -i ( a
2
I) .
- -
{J
sin 0

Usi.ng cos2 0 + sin 2 0 = I. we obtain


u2 v2

[I ( l\] +[12 (a-;1)] =l


2 2
2 a+-;;)
=
Thus we see that Equation 7 maps circles lzl a onto ellipses in the w-plane.
Now. if rhe circle in the z-plane is not centered at the origin. passes through 1hc
point z.. = I, and contains Lhe point z = -1 within it. you get quite ::i different result
982 ha pwr 19 / fun lions o( a C mp l x Va riabl -: Appli ,a tio n ·

)'

II' = .L(~
2
+ .L)
~ z
1
•,• I'


- - 1- - -----1--------- X II

Figure 19.50
The mapping of 1he c ircle described by
I· + ! - ~I= L1f,: into the w-planc by
the mapping in Equ.a1ion 7.

In this case. the ligure in the w-plane begins 10 resemble an airfoil (Figure 19.50).
The resultant figures in the w-pla.ne are called Jo11kowski profiles and Equal ion 7
is called a Joukows/.:.i map after Nikolai Joukowski. who is known as t.hc fa1hcr of
Russian aviation . By ~t.arting with figures in the ;:-plane that arc nearly circles. it"~
possible to produce figures 1n the w-plane that describe a great variety of airfoils.
Before we leave 1his sec1ion. we should point out that the concepl of a complex
potential is not limited 10 a treatment of Ouic.1 flow. In electrostatics. </)(x. y)
is the electrostatic potential from which we calculate the electric field intensity
uccording 10
. iJ</, . i:l<p
E =-1--J-
ilx ay
The curves <jJ (x. y) = c 1 and it, (x. y) = c 2 arc called the equipotential curves
nnd the flux. lines, respectively. For srcady heat flow. tp(x, y) corresponds 10 the
temperature and tj, (x. y) corresponds to heal flow lines.

19.7 Problems
I. Show lhat Equation 2 governs an incompressible nuid. Him : S1art with rhe l:OnIin11i1y equation .
2. Show that if J (:) = 11(.x. y) + i t1(x. y) i:- an anuly1ic function. then the 1wo families of curves 11(.r ..1) :-..: c 1
and v(x. y) = c~ are onhogonal.
3. Show that the complex potential Q (.:) = v0 e-iu: represents uni fom, flow that makes an angle a with the x
axis.
4. Derive a complex potential that corresponds 10 unifom1 flow in I.hex direction.
5. Show that dQ/d : = t\ - i u.\.. Verify this result for unifonn flow making an angle a wi1h the x axi s.
6. Show th:::it lv(.:)I = I dQ/d-.: 1-
7. Deduce 1hc type of flujd flow govern~d by the complex po1entia.l = ik In;:.
Q(;:)

8. Interpret thl.! 1ypc of fluid nmv a..\oo:s0C i,11cd wit.h the comple,c potential Q( z) = k In:. (See the next pmhlem
also.)
Referen es 983
9. Show that rhe electrostatic po1en1ial due to a line of uniform continuous charge density J.. is given by
()./2rre 0) In (a/r). where r is thcdi~ta.nce from the wire and a is an arbitrary constant.
10. Re-do Example 4 for a cylinder of radius h.

I I. Show thar r.he s1agna1ion points of the flow depicted in Figure 19.46 occur al the front and back ends of the
cylinder.
12. Find lhe srreamlines for the flow of an incompressible. non-viscous fluid within the comer shown in
Figure 19.45::i.

13. Find t.he streamlines for lhe now of an incompre!.Sible. non-viscous fluid over a wedge, as shown in Figure 19.51 .

Figure 19.5 t X
TI1c wedge refcm:d 10 in Problem 13.

14. Use a CAS 10 verify the mapping in Figure 19.50.

15. Let's look at rhc circle I.: + ~ I= ~ and iLS resulting mapping in the w-planc under w = l (;: + i). The

= I is rr. bur the angle at w


outer angle 10 1he circle in the ;:-plane m : = I. the image poinr or z. = I. is 2JT in
1hc w-planc. Why isn·r the angle preserved at this point'!

Referen ces

Jame:; Brown and Ruel Churchill. 1998. Complt>x Voriobles with Applications. 6th ed ..
McGraw-Hill
V. I. lvanov and M.K. Trubetskov. 1995. Handbook of Co11formal Mappings with Computer·
Aidt'd Visualiw1io11. CRC Pre~s
Srevcn Krantz, 1999, Handbook of Complex Vnrwb/c.~. Birkauscr
Jerrold Marsden and Michael Hoffman, 1998. Bt1,, it Co11ip.lt'x .f\Jl(1/ysis, 3rd ed., Freeman
Reinhold Rcmmcrt . 1990, Theory of Complex Fim, ·tirm.1·. Srprin g.t=r-Vcrlag
Lester Rubenfeld . 1985. A First Course in Applied Complex Variables. Wiky
Richard Silvcnnan. 1974. Comp/a Al1a/_\'.~is 11•irh Applicalions, Dover Publications
Murray Spitgel. 1964. Complex Variable.{, Schaurn's Ou!J]ine Series. McGr-..1w-Hill
David Wun~h. 1993. Complex Van·ables with Applicnrio11s. Addison-Wesley
GENERAL:
Paul Nahin. 1998. J\11 fow,~inary Tale: Tire S1ory of ,1-1, Princcton
CONFORMAL MAPS:
CRC S1a11dcml Marhemmic:al Tab/ex. 30th ed., CRC Prcs.~
CONFORMAL MAPPING SOFTWARE:
Lascaux Gr.tphics: www.primenet.com/-lascaw:
Johann Bernoulli (1667-1748). who, along with his brother Jacob. was a great promulgator of the Lcibni1z
(differential) fonnula1ion of calculus. was born on July 27. 1667, in Basel. Switzerland. Johann also had
li1tle interest or talent for the family business, and so his father reluctan1ly allowed him to srndy medicine at
the University of Basel. While at Basel. he studied malhcmatics under his brother Jacob. becoming Jacob's
equal in just two ycan.. In 1691. he traveled to Paris. where he met the Marquis de l'H6pital, who hired
Bernoulli to teach him the new calculus of Lcibnitz. In 1696, l'Hopital published the first cakulus book,
which was based mainly on Bernoulli's lessons. Bernoulli later claimed that J'Hopital's rule was actually his
work. Bernoulli wa" largely responsible for the spread of Leibnitz's formulation of the calculus. Upon his
return 10 Basel, he continued with his medical studies. as well as with mathematics. At this time. he married
Dorothea Falkner. Three of their sons becnme mathematicians. which was to become a family tradition.
During this lime. Johann and Jacob had a serious f aJling-ou1 over mathematics. Both brothers had difficult
personalit·ies, but it seems that the greater portion of blame lies with Johann. Johann has been described
as intolerant. mean. and nasty to anyone who disagreed with him, includi □ g his own son. Daniel. In 1695.
he accepted lhe Chair of Mathemat..ics at the University of Groningen in HoUand over the objections of
his wife and father-in-law. During his ten years at Groningen, he wns involved in a number of disputes,
including religious ones. In 1705, the family returned 10 Basel. While en route. his brother Jacob died. and
Johann succeeded his brother at Ba.~el. In 1718. he laid lhe foundation for the calculus of variations when
he revisited the brachistochronc problem. the cause of the bin.er ho. tility between the brothers . Bernoulli
died on January I. 1748. in Ba.-.cl. Johann Bernoulli was known as the "Archimedes of his age," which is
inscribed on his tombstone.
CHAPTER 20
Calculus of Variations

A standard problem in calculus is 10 find ex1:rema of functions. whe1her they are


functions of a single variable or ful]ctions of many variables. A standard problem in
the calculus of variations is the following: \\'hat curve lying in a plane connecting
two given points has the shortest arc length? In other words, wha1 is 1he shonest
distance between two points in a plane? Of course. the answer is a straight line. bur
lhe caJculus of variations provides us with a systematic procedure for proving it.
We can fonnulate this problem by specifying Lhe two points as (x 1, y 1) and (x 2 , Y2)
and then expressing the arc length in 1enns of an integral involving y(x) as

The value of/ depends upon lhc pa1h that we take from (x 1• _v 1) to (x 2 • y 2). and
we want to minimize / with respect to aJI possible func1ions y(x), or at leas! wilh
re...,pect to all functions within a certain class, such a-; all smoo1h functions. When
we learn how to do Lhis in Section I, we·u see that y'(x) = constant doe.s indeed
minimize /, above.
The detcrm1na1ion of the shonest disiance between rwo points in a plane is
a fairly simple problem, but what about the shortest distance between two point.,;
on some other surface, such a."i a sphere or a right circular cone? Such curves are
called geodesics. and geodesics play an important role in the theory of relativity.
As we shall see. the calculus of variations provides us with a method to determine
geodesics on most any surface.
Another standard problem of the calculus of variations. the one that actually
initin1ed the development of the field. is the so~called brachis1ochronc problem,
proposed by Johann Bernoulli in 1696. Consider t\VO points a and bin a venical
plane wi1h the x axi~ being horizontal and the y axis being directed downward.
(See Figure 20.2.) If a panicle starts from rest at point a, then along which path
will the particle reach poin1 bin I.he shones1 time? We'll see that in 1his case. we
need to minimize the functional

985
986 Chapter 20 .I Cal ulus of V,1ri,11 i11n~

wi1h respect 10 _v(x). The name brachisrochm11e derives from 1he Greek for shonesl
(brachislos) time (chronos).
Nor only does 1hc calculus of variations allow us to solve problems like those
above. bul many others. such as the determination of the closed curve of a given
length that encloses I.he larges1 area and the shape of a uniform flex.ible cable of
given length :-uspended al its ends by two given poililLS. As imponanr as these
problems might be mathematically. however. ii turns out that many of lhe laws of
physics can be expressed in variational fom1. For example, the laws of classical
mechanics c.an be expressed succinclly by Hamihon's principle, whkh says that
a (conservative) mechanical system will evolve along a trajectory such that the
integral
h
I=
l'1
(K-V)dt

where K is the kinetic energy and V is the poLen1iaJ energy, is an extr~mum


with re$pect to all trajectories. The laws of optics can be formulated by Fermat's
principle. which says tha1 the lime it 1ales light to travel from one fixed point
to another is an extremum with respect to 1ime. Furthermore, the Schrodinger
equa1ion can be formulated a" a variational solution to the problem of minimizing
the energy of a quanlum-mechanical system wi1h respect lo all possible wave
functions. This has produced a huge industry involving 1he calculation of atomic
and molecular properties from first principlc.s. We ~hall :-cc examples of aJI the
calculations I.hat we have discm,scd here 1hroughout this chapter.

20.1 The Euler Equation

All 1hc integrals I.hat we mentioned in Lhe introduction are special forms of Lhe
integral

I = 1h F(y, y', x) dx (I)

The value of I depends upon y(x) in the sense that different functions y(x) will
y yield different values of/. Equation I represenLS a mapping of some gjven class
of f unclions in10 a sc1 of numbers. We say that / is a Ji1ncrio11al of y (x) and
write / = / ly(x)J. The problem that we wish to solve is 1he determination of 1he
pa.nicular functjon y(x) that makes / an cxtremum. The simplest (and common)
case assumes that y(x) is prescribed at the end points a and b. Therefore. we use
a trinl f11nctio11 of the form
a b X

Y(x. E") = y(x) + e:11(.x) (2)


figure 20.1
1;;.-c::implcs of trial 1um.:1iom 1h:11 can he
where 11 (x) is chosen such 1ha1 ,, (a) == ,, (b) = 0. so that Y (x) and y (x) coincide
u,cd 10 minimiZC' lhc funl·tional given by
Equation I. at the end poinls (Figure 20.1 ). As Lhe notation SUg_i,!l'~b. we plan to let E --+ 0
20. 1 lh1· l:uh ·r Equat ion 987
eventually. We will always assume that the partial derivatives of F(_r, y', x) exist
and are difTerenliable and Lhat y(x) is twice differentiable.
If we substitute Equ:'Jl"ion 2 into Equal ion I. we obtain

/(e) == 1h (I
FfY(x. E'). nx. €. x)Jdx (3)

Because y(x) is the function Lhnt cx1.remizcs the value of/, /(E) takes on its
extreme value when€= 0. and so we can write

when E =0 (4)

Jf we di ffercntiate / (E) in Equation 3 with respect 10 f. we ob1ain

=
1,. [aF
u
-17(.r)
aY
;JF
+ -ry'(x)
oY'
] dx

Assuming that oF/'rJY and aFJoY' are continuous functions of c oF/rlY ➔


oF/ay and oF/oY' ➔ oF/ily' aH ➔ 0. and so we have

1b[ u
;J F
-17(X
ay
) +i) F-d11]
- d X-- 0
Jy'dx
(5)

Equation 5 is a necessary condition for / to be an cx1.reme with respect to aJI


possible trial funcr.ions Y (x) in Equal ion ~. It cenai nly isn · 1 a sufficient condi1 ion.
which depends upon properties of the ~econd derivative of / with respect to<:. It
rnms out thm an examination of this second derivative is fairly involved. so we'll
simply accept the fact that EquaLion 5 gives. us an exrremum. or that / is srarionOJ)',
and appeal to physical arguments that it gives a maximum or a minimum.
We can Ca.'-1 Equation 5 into a more convenient fom1 by integrating the second
term by pans to obtain

[aF dc
ff ]11(.x)dx + [r,(x)-iJF]b
l 0
b
-
cl_y
- --.•
tlx u)
. -.. =0 u)' c,
(6)

Assuming Lhat IJ(O) = TJ(b) = 0 (fi'Ced end points). Equation 6 becomes


[rlF d aF] 11(x)dx =0
1 a
h
-
ily
- --
d.x iJy'
(7)

Because Equation 7 must hold for arbitrary TJ(X) (other than IJ(a) = rJ(b) = 0), we
sec Lha1 the condition Lha1 / be an c.xtremum is that
d aF oF
- - -- = 0 (8)
dx 8y' oy
988 Chap1er 20 / Calculus of V,ui,11ions

Equation 8, called the Euler equation, is a central equation of the calculus


of variations. ReaJize that it is an ordinary, nonlinear, second-order differential
equation in y (x) because F (y. y', x) is a given function of y and y'.

hample 1:
Use Equ~nion 8 10 sbow I.hat the shortest curve connecting two point-. in a
plane is a straight llne.

SOLUTION: The inlegrnl in question is

I= 1b = 1h dx (11.t:2 + d_v2)1/2

= J\ (/
I + _v'
1 12
) 1 dx

where y' = dy Idx. The integrand is independent of y. so Equation 8 becomes


simply

or y'(x) = cons;tant. Thus. we see lhal the curve is a straigh1 line.

The result of Example I comes as no great surprise. You probably aJso know
Lhat the shonest dis1.ance between two points on the su.tiace of a sphere lies on a
grca1 circle. We'll prove this result in Ex.ample 2, but before doing so we'll simplify
Equation 8 for the special case that F docs not depend explicitly on the independent
variable x. We SI.art with the relation

, aF) ::::::y--+r
-d ( v-.- ' d aF ,,'rJ F
-
dx . oy' . dx ay' . fJy'

and then use Equation 8 for d(oF J'dy')Jdx on the right side of 1his equal.ion 10
write

-d ( \,aF) ,JF
' - =y-+v ,,aF
-
dx . oy' ay - ay'
f.JF dy JF dy'
= -- + -- (9)
ay dx oy' dx
We can express the righ1 side of Equation 9 1n a compact form by firs1 recalling
that the total derivative of F(y. y'. x) with respect to .x is given by

dF aF d y aF d y' aF
-=--+--+-
dx oy dx rJy' dx ox
994 ( h,1pt1·r ..!U / Calrn!u, of \'.1ri,11i11r1,

20.1 Problems
l. we·11 e\•aluale I.he inte~ral for <J, in Example 2 in this problem. First factor er sin-1 0 from lhe denominator nnd
use the
.d
I
. ., ,.,
= I + co1~, 0 10 gel <I>=
en111y csc- n
f ,csc 0 d0 ,,
2
1
, . Now notice 1hat this integrand is
[(a I c I J• - I - col - 0 I1/ -
• •

of the form d sin- I 11 = d11/(l - 11 2) l/1 if you lei 11 = tot 0/l(a/c 1) 2 - I JI12. Show 1hat this !-.uh,1i1.ution katb
to <fJ given in Example 2.

2. Determine 1he Euler equation for/ = lor" (_v:? - y


12
)dx.

3. Determine the Euler equation for/= {.\r/~ -


. {)
qy~)dx. Does 1hc result look familiar?

f,

4. Find 1hc: general solution of the Euler equation associated with / =


1
a
x( 1 - y'!) 1l 2dx.

5. Detcm1inc 1he equation of I.he curve that passes through 1hc points (0. 0) and (I, I) and for which
1 ( I + ~-'2) 1/2 _
I =
1IJ
· d.r 1s an extrcmum.
_\"

6. lxtermine the Euler equation for/ = 1" (xy' 2 - yy' + y)dx.

7. Re-do Example I in plane polar coordinates. Take 0 10 be 1he independeni variable.

8. Determine 1.he geodesic on 1he surface of a right circular cylinder of radius a. I-lint: Take cis 2 = {/·d0 2 + d:. 2 .

9. Detcm1inc the geodesics on lhc surface of a right circular cone. Him: Use d.~ 2 = dr 2 --- r 1 sin 2 et dtp 1 hecause
lhe surface of a cone is described by sin f) = sin a= constanl.

10. Set up the equations to determine c, Oiniti(JJ• and 01intl.l for lhe solution to the brachistochrone problem given by
Equations 14.
1(!.
11. Show lhal the lime of dl·,ce1H of the mass in 1he brnchi.<:tochronc problem il- given hy T = ( ?c_g ) '1tioal·

J-/i111: Use Equa1ion 13.

12. Evaluate lhc integral lhal leads 10 Equation 15 by making the subs1i1u1ion y =c 1 cash 11.

13. Show thal , 2 = 0 in Equation 15. thus yicldi ng Equation 16.

14. Show 1hat the roots of I - c co:-h( l/2c) = 0 arc c = 0.2351 and,= 0.8483.

IS. We'll explore rhc sonp film problem numerically in this problem. Us£~ a CAS 10 show t.hat tJ1c area given by
Equa1ion 18 is equal 10 '2;;ra 2 when b/o = 0.527696 and c is <:orre ,x,ndingly equal to 0.825519a. Show.
however. 1ha1 there are still two root, 10 Equation 17 for this valu~ of h/a. Show thal the area corresponding
to this other root is greater than 2rra~. Now show l.hal the value of h/a = 0.662744 corresponds Lo the point
where 1hcrc is no real !-olu1ion to Equation 17. This c.aJculation illustrates thot we obtain only a local minimum
for 0.527696 < h/a < 0.662744.

16. Derive Equation 16 using Equal ion 8 instead of Equa1ion I0.


996 Chaprer 20 / Calculus o( V,1riations

If Lhe force Fis conservative, 1hcn it can be wrinen as rhe gradient of a po1en1jaJ
function V(x. y. z):

av av av
F= -grad V = - i - - j - - k -
ax oy az
and we can wrire F • <Sr in Equarion 3 as

-8V
av
= --8x
av
- -ih - -8z
av
ox ay · az
wh.ich essentially defines 8 V. Therefore. Equation 3 can be wrinen as

or

8
1"
'1
(K - V) dr =0 (4)

Equation 4 is Hamillon's principle for a conservative system. We derived il


for a single particle. but the derivation can be extended to a system of particles
by summing and even Lo continuous systems by integrating. The quantity K - V
in Equation 4 is called the Lagra11gim1, l, and so we can write Hamilto □ 's princi-
ple us
1,

c5
1'1
L dt =0 (5)

We said above that the advantage of Hamilton's principle is that ii can be


applied equally convenienlly 10 any coordinate system. The Lagrangian can be
expressed in terms of any set of coordinates, whether 1hey are distances. angles,
or whatever. For example, we might use the three spherical coordi naLcs r. I}, and
</J inslead of .x. y, and z_ These new coordinates are called ge11eralized coordinates
and are customarily denoted by Qj• If ii takes 3N coordinates to speciJy a system
(as it would for N point ma-'ses). the generalized coordinates arc q 1• q 1 , ...• (J]N·
The time derivatives of the generalized coordinates. q1• q2 • .. . , lJ]N· are called
generoliz.ed velocir;es. Hamilton's principle. Equation 5. can be wrinen as

(6)

Equation 6 consritules a variational problem involving 3N dependent vari -


ables. ralher than just one as we treated in the previous section. For simplicity. leL's
assume that we have jusr two dependent variables, and derive Lhe Euler equation
that cxlfemizes

(7)
1000 Ch.1ptl'r 20 / Calculus of V.ariJlions

y
Example 3:
A B Use the geometry in Figure 20. 10 to derive the law of refraction from
Fem1a1's principle 1har the angle of incidence is equal 10 1he angle of
rcncc1ion. Assume that II is a con~ta.nt.

SOLUTION: Con::;idcr the geometry in Figure 20.10. We know from


Equation 13 1hat 1hc light will travel in a maight line because n is a constant.
so we want to fix P to be on rhe x axis and find Lhe \'alue of P({. 0) t.hm
minimizes t.he distance A PB. The rot.al distance is
X

Minimize this expression with respec1 lo~ 10 obtain


Figure 20.10
The geomclJ)' as~oci:11ed with Exampk 3.

which is midway between a and h. Thus. we ~e th::11 the angle of incidence


is equal to the angle of renection.

20.2 Problems
I. Derive Equation 8.

2. Find an extremum of/ = f ':


11
(.i'::! + /-) 112 dr. where x(rj) = x1 and y(ri) = .\'j for j = I and 2 are given.
Describe the result.
3. Extend Problem 2 to three dimensions.
4. Derive and solve Lagrangc's equation of motion for a ma,~ falling venically under the influence of gravity.
5. Derive Lagrange·s equation of motion for a particle in a gravi1alionaJ field constrained to lie on a circle of
radius a in a fixed vertical plane.

6. Derive Lagrangc·s cqua1ion or motion for a simple pendulum or length I in rcrms or 0, the angle that 1he
pendulum makes with the vertical.

7. Derive Lagrange's equations of motion of the double pendulum shown in Figure 20.11.
8. What form do Lagrangc·s cquatjons or motion in the previous problem take on for small oscillations?
9. Derive Lagrangc·s equation of mo1ion for a ma<;s hanging from a spring whose force constant is k. given that
the point of suspension moves verticalJy according to x = ll sin wl. Neglec1 gravity.
20 .3 Vari ational Problems with ConstrcJints 1001

Figure 20.11
TI1e double pendulum referred 10 in
Problem 7.

10. Derive Lagrange'::. equations of motion for n particle moving in two dimensions under a central po1ential V (r ) .
Which of these equations illustrates the law of conservation of momentum? Is angular momenrum conserved
if the potential depends upon 0 well? a.,
11. Derive Equation 12.
12. Calculate the path lhat a ray of light will travel if tht: index of refract ion varies as a/ y. where a is a constant.
13. Calculate the path thnt a ray of light will travel if the index of refr-..icrion varies as ay, where a is a constant.
14. Sho~v that a geodesic on a /u[rface described b]\~rthogonal curvilinear c~rdinates '\ and f3 is ;iven

by an extremum of I= 1 ~x) -+ ( ~y )- + (oz.)-


2

a
1, 2
Cl
+ 11A (dfJ)
det
dct. where 1,
2
a
= ( acr Ja OCl
and

11
1= (:;Y G;Y+ + (!;)
2

15. Use the result of Lhe previous problem to derive an expression for the integral 10 minirruze for the gco<lcsks on
the surface of li righ1 cylinder.

20.3 Variational Problem with Con traint

Just as we minimized functions of several variables with constraints in Section 6.9.


we can find extrema of functionals subject 10 cenain constraints. For example.
suppose we wish to find an extreme value of

b
I=
l 11
F(y, y', x) dx (I)

subject to the constraint condition

J = lb G(y, y'. x) dx (2)


20.3 V.ui,1ti11nt1I Problems with Con~tr.ilnts 1003

Obviously. 2J ~ 2a.
Because F + ).G does not depend explicitly on x. we can us:c the first
in1egral gfven by Equal.ion 6 to write

or

Solving for x. as we did for I.he soap film problem in Sec1ion I. giv~

(7)

Following Section I. let pgy - A = ,·1 cosh.: and get x = , 1;:./ pg + , 2• or


(Problem 3)

(8)

There are three consIants (c I , c2 • and),,) in Equation 7 and three conditions


(y(±a) = 0 and J = 2/) 10 specify t.hem. Problem 4 has you :,;how 1hat y(x)
can be wriuen as

y(x) = -er ( cosh -


pgx
- - cosh -pga) (9)
pg a et

where a i,s g,i\'cn by (J sinh(pga/a) = pgl. Thus. we see 1hat 1hc hanging -a a X
cable is described by a catenary (Figure 20. 12).
Figure 20. 12
TI,c so lution IO Example I with pg = 11 = I
,md I= l.".!.5. (a rum~ our robe 0.8455.)

hample 2:
De1em1inc the curve of length/ which passes through lhe points (0. 0) and
( I. 0) and for which the area between the curve and the x axis is a maximum.

SOLUTION: The area is given by

I= (' y d.r
lo
and the (fixed) kng1h is given by

where/ > I. In this case,


1004 Chapll•r 20 / Cal( uh 1, 01· Variations

does nol depend explicitly on x, so we can use the first inrcgral given by
Equation 6. which gives

or

where c 1 = -c. Solving for dy/dx and then intcgraring gives

Let .: = c 1 - y. in1egrnte. and then reillTilnge 10 get

Thus. our resul! is n circle of radius). centcred al (€- 2 • c 1). We can determine
c 1. c2• and ).. using the fact that 1hc circle passes l!hrough 1he poin1s (0. 0)
and ( I, 0) and using the constraint that Lhc arc length is /. The result is
c 1 = 0. c 2 = 1/2. and,\= 1/2. which yields a maximum been.use lhc !iolmight
line y = 0 produces a minimum.

Another type of problem that can be solved using the methods of this sec1ion
is caJled an i.wperi111etric problem: Of aH the closed plane curves with perimeter
/. which one encloses the largest area?

Example 3:
Detem1ine the curve of length/ that encloses the largest areo.

SOLUTION: We use 1he equation (Problem i.2.18)

= -2I i
C
(xy 1 -· \') dx
.

with the constrain!

We U$e Equa1ion 4 with

I , , . 1;1
F+i.C=-(xv
2 . -v)+).(l
. , + •v -) -
1005
10 ob1ain

-d [x-+ ,l.y'. ] +--


I O
d.x 2 (I+ r'2)1/2 2 -

or

Integrate once to obiain

Solve for y' to gel

dy X - c, ( 10)
-=±
d.x [), 2 - (x -
" I/?-
c1J-I

Let:;::::..:.\ - c 1 and in1egrate agnin (Problem 11 J

or
(l l l

This result can be wrincn as

which is I.he equation of a circle cen1cred al (c 1• c 2) and of radiu..'- ),_ I /2 rr. =


In olher words. a circle is 1he curve of a given length thai encloses the largest
area.

20.3 Problems

1. Derivt: Equari·on 4.
2. Derive Equation 6.

3. Derive Equa,1ion 8 from Equal.ion 7.


4. Show tha1 Equation 8 can be written w Equation 9.
5. Show tha.1 a function which ex1remizes / with t.be constrain! J = cornaanr also extremizes J wilh 1he conslraint
I = constant.
6. Find the i:qua1ion of the i-hortcsl curve 1ha1 pussc through the points (a. 0) and (b. 0) and encloses a g·iven
areu A between 1hc curve and the x axis. Him: U_e the result of the previous problem.

7. Suppose we wish to find an exuemum of the integral 1h F (11. v. 11 x• V.x. x )dx subject to the
20..J VJri.itional Formulation oi rig('tW.llue Pmhlc•r1h 1011

Example 2:
u~e the rrial funcLion given by Equation 12 to cal<..:ulalc the ~malles1
eigenvalue of 1he problem in Example I.

SOLUTION: We mus1 firsl calculate the elcmenls of the secular


determinant. The integrals are conveniently C\':ilm11cd in terms of the bcto
function. in which case we have

L11 = 28(2. 2) = 2f(2)f(2) = ~


f(4) J
_, 2r(J)f (3) I
L 1., = L., 1 = _B(J. )) = - - - - = -
- ~ f(6) 15

L21 = -28(3. 3) + 128(4. 3) - 128(5, 3)

2r(3)r(3) 12r(4)rC\J 12r(5)f(J} 2


= ----- + ---- - ----
r{6) f(7) f(8)
= --
105

S11 = 8(4. 3) = f( 4 )fC 3) = ~


rm 60

s., = 8(5. 4) = f(5)f(4) =


l_ f(9) 280

. f(6)f(5) I
S12
-
= 8(6. 5) = - --
f(II I)
= --
126-0

The secular determinant (Equation 23) becomes.


;.., I Ai;,
- --
3 60 15 280
=0
I A,:, 2
15 280
- - --
105 1260
}, ~

which givC!-

2
--- 4\1\4, + 403 ..1 -- 0
. - ? 1
I.If,

The 1wo rooL'i 10 this equation are 19.7395 ... and 204.26 . ... We lake the
lower root and obtain, A = 19 .7395 . .. compared with,\.= 20 from lLsiog
just one term .

So far in this section we have discussed how to find approximate eigenvalues


ai-~ociaicd wi1h Snirm-Liouville problems u~ing 1he Rayleigh-Ri12 method. II turns
out tha1 lhere is a very close connection between Stum1-Liouvillc problems and
the calculus of variations. Consider the functional

1> ,., .,
I=
1 ll
lp(x)y ·ex) - q(x)y·(x))dx (24)
1012 Ch;iprer 20 / C.ill ulus o( Vari.ai-ions

where p and q are given functions ofx. The extremum of/ subject 10 the constraint

J = 1b r(x)/(x) dx = constant (25)

where ,. (x) is a given function of x, is given by (Problem 6)

1• I :x [ p(x)y'(x)] + [q(x) + Ar(.,) I y(.,)) •1(x) dx - [p(x)y'(x)ry(x) I =O


(26)

Note that this result is equivalent LO the Srurm-Liouville differential equation,

d
dx [p(x)y
I
(:di+ [q(x) + J..r(x)] y(x) = 0
along with the boundary conditions lhat p(x)y'(x) vanish at an end where y(.~)
is not prescribed. In particular. you can see that the homogeneous boundary
conditions

y(a)=O or /(a)=O and y(b) =0 or y'(b) =0


easily satisfy these conditions. Thus. determining an cxtremum of Equation 24
with Lhe constraint given by Equation 25 is equivalent to solving a Srurm-Liouville
problem.
We can carry this development one step further. It is straightforward to show
I.hat finding an extremum of Equation 24 subject to Equation 25 is equivaJent LO
determining an extremum of the ratio

1\p(x)/ 2
(.r:) - q(x)y2(x)Jdx
). = - - - - - - - - - - - - (27)
[b r(.x )y2(x) dx
.fa
The condition that A be an ext.rem um with respect to y(x) leads directly to Equa-
tion 26 (Problem 7). Of course, Equations 26 and 5 lead 10 the same result. and,
in fact, are equivalent (Problem 8).

Example 3:
Use Equation 27 along with the triaJ function y(x) = c 1x(I - x) to calculate
A in Equation I. Compare your result to;..= 10, the one obtained earlier in
this secrion. The exact value is rr 2 = 9.8696 ....

SOL UT 10 N: Comparing Equation I with Equation 5. we see that p(x) = I.


q(x) = 0. and r(x) = I. Therefore.
1014 Ct1,1p11•r 10 .' L,il< ulus of V.1 1i,11i1m~

11. Fol lowing Problems 9 and I0, u.~c a triaJ f uncl ic 111 of the form </> (:r) = c 1.1"( 1 - x) + c~x ~ ( 1 - x) lo cst..i mare
the first 1..ero of J 1( :.) •

12. Use a trial function of the form q;,(x) = (I - x)(c 1x + c2x 2) lo estimate the smallest eigenva.lue of 11 11
+A.II= 0
= 11( I). Compare your rcsulr 10 !he one that you ob1..aincd in Problem 5.
wilh 11(0)
13. Describe whal happens if you use a trial function of the form <fJ (x) = c sin ;r x + , 2 sin 2rr x I in Prohlcm 12.

14. The ground state elecIronic energy £ 0 of a hydrogen atom is given by £ 0 =


If J'J{ .
i/1 Vt.n d tl
where
\''1; 1/10 d ll
i/lo is the ground state wave function, dv is the volume element, and '.}( is 1he Hamdtunian operator
2 2
'.}{ = - -h-1 - , , - d ( ,,-d- ') -
e . . •
- - . where J, 1s the Planck t:on-.1::mt, Jt 1s rhc rcdul:cd ma<,~ of rhc electron.
Srr-1-u- dr dr 4rr£or
2
c is t.he proronic charge. and e0 is 1he permi11ivi1y of free space. Using a Gau"ian trial function tJ>(x) =e-ar •

3ti'2o e 2a. 112 4 µ/'


show that £(er)= - - - I/" V an<l thnl Emin = - - - 2- , . Compare your result to 1he exact t.·ni:rgy,
2µ 2 ~eoJr· - 3rr 4t h- 0
£0 = (- l/2)(µe,J /4iih 2).
JS. Re-do the previous problem using a trial func1ion of 1he form <J,(r) = <·- (I' . Compare your rc~ult 10 the exact
energy. £ 0 =
(-l/2)(µ e-' / 4eJh 2). Why do you 1hink that the agreement is :-o good?
16. Show that t/,(r) = e - r/ao with a 0 = 11 2t 0 /rr /te 2 is an cig.cnfunction of rhc Hamiltonian operator given in
Problem 14. Nmv suppose 1hat one were 10 use a trial funl"lion of I.he form <J>(x) :;:: c 11• · f¥' + c 2,/1' · 10 carry out
a variation al calculation for the ground swrc of the hydrogen arom. Can you guess without doing any culculalions
j

what c 1• c 2• ex. and Eu wiU come our to be? Whul abour u trial func.:tion of 1J1c form <f>(x) = L c1:c - rj r fJ ' ~?
I. I

17. The ground s1a1e energy of a quantum-mechanical ham,onic o~illaror is given by £ 0 = -----:~,.,,,---.---
{" iJl(/KVJo dx

1 -~
V'o t/to dx

w I1ere I/Jo .1s 1.hc groun d stale wave luncuon


• . an d '1(
J
. h H
1s I c
·1 .
:1m1 tonran operaror
d2
:,t = ---.,- --,
I • "i
Rrr-JL dr
k ~
+ -x~.
2
where I, is lhe Planck constam, µ is the reduced mass. and k is 1he force cons1am. Use a 1rial function of I.he
form </)(x) = cos A.x with -rr /ll.::: .r ~ JT /2J... whcr~ .\. i~ a variational paramcler, 10 c:alcuta1e an upper hound
1
10 £ 0 . Compare your result to rhe exacl value. £ 0 = .!!.._ ( ~) -
4,r J.L

18. Re-do the previous problem using a trial function of the form <f,(x) ::::: 1/( I + f3x ~). where /j is a variational
parameter.

19. Re-do Problem 17 using a trial function of the fom, <P (x) = e-lhi. where f3 is a v:.uiat·ional paramctl'r. Why do
you think the agreement is so good?
20. Show that 1/Jo(x) = e- a.. f :. with a = (4JT 2k µ.,/ Ji 2 ) 112 is an eigenfunction or I.he quantum-mcchani~I harmonic
oscillator Hamiltonian operator given in Problem 17.
21. Show 1ha11he normali-z.ccl trial func:tion corresponding to Problem 5 is 4.404.r( I - x) + 4.990.r 2 ( I - x) 1 .
22. Sbow that the norn1a.li:1.crl trial funclion in Example I is (60) 112x ( I - x) and the nom,ali;£cd cxacl solution is
3.6851 x 1' 2 J 113 ( jl: 1!x J/ 2 ).
1016 Chapt,•r 20 I Calculus of Variations

Given that r,(x. y) is sufficiently arbitrary, we conclude that the tem1 in


brackets in Equation 5 must equal zero and write the Euler equarion i11 rwo-
dime11sio11s a.'- follows:

(6)

The cx1cnsion of Equation 6 to higher dJmcnsions is straightforward (Problem 2).

Example 1:
Show thal the func1ional

leads to Laplace's equal ion.

SOLUTION:

aF iJ11 iJF a,., iJF 011

und so Equmion 6 gives

We can derive the wave equation u.-.ing the calculus of variations, and using Hamil-
t.on's principle in panicular. Consider a flexible string fixed at its two endpoints.
For simplicity, assume that the tension, r. in the string and lhe linear mass density,
p, of the suing are constants. In I.be course of a vibration, an element of length dx
X x+ax X
occomcs an clement of length ds (sec Figure 20. 16). For small displacements.
Figure 20.16
ou - .,] 1/2 2
~ dx + 1 ( ax)
A vibrnling string at an inslanl of lime. I Ju
ds = [ I+ ( ax) d:r dx + ··-

where II is the displacement and where we have used Lhe binomial theorem. The
extension of the element is approximarely

2
dl==ds-dx:::::-1 (a )
2 ax
...!:!.._ dx

and the work done against the tension r, which is the potential energy. is
20.5 Mufri dimensional Variati onal Problem~ 1017
2
dV;::::; -T ( au )
- dx
2 ax
The potential energy of rhe siring is given by (small displacement)

v =½lot (aa: ) dx
2

The kinetic energy of 1he string is given by

K = !!.1, (a")2
2 u a,
dx

and so the Lagrangian of the string is

L = !!.. ['
2 lo
(a")2
a,
dx - ;
-
1' (a'~)2
0 eh
dx (7)

According lo Hamilton·s principle. rhe inregraJ

(8)

musr be siationa.ry. If we apply Equation 6 to this equation (with y -+ t and


u _\· -+ 11 1 ). we obtain

-a ( - rdll)
- +- a ( pOU)
- -0
ax OX QI Of

or

(9) a r----------.

where u = (r / p) 112. Thus. we see 1ha1 the wave equation can be thought of as a
Euler-Lagrange equation .
We can use lhe Rayleigh-Ritz variational principle 10 calculate upper bounds 10
the eigenvalues of multidimensional problems. For example. let's calculate an
upper bound to the lowest vibrational frequency of Lhe square membrane shown
in Figure 20. 17. The wave equation for this sys,em is a X

(10)
Figure 20.17
A ~uare mcmbrune cl11mpcd along its
Realizing that the time dependence of 11 (x. y . r) is harmonic. substitute perimeter.
u(x, y. t)= </)(x. y)e'w1 into Equation 10 to gel

(11)
1024 Chapter 21 / Probabilit\• Theory and Stochasric Processes

effecr. which consis1s of a train of pulses that occur ar random times. The shot effect
describes 1be noise in electronic devices. or the noise generated by the dumping
of sho1 on10 a mc1allic surface.

21.1 Discrete Random Variable


There arc a number of deep and philosophical questions regarding the definition
of probability. but here we address only the most pragmatic aspects. We all would
1/2 • •
agree 1ha1 when a fair coin is tosSt..>d N times. the ratio of the number of heads
H to 1he number of tosses i!. around 1/2. and approaches 1/2 more closely as N
105 N increases. Figure 21.1 shows a computer genernted simulation of the tossing of a
fair coin. Other examples of this &Ort of limiting behavior are the rolling of dice,
Figure 21.1 the drawing of card'i, and so on. We can use this type of observation to define
A computer simulation of the tos:s-ing of a
probability. ln general terms, suppose that an experiment is performed N times
fair coin. The graph shows 1he ra.1io of the
number of hcnds to 1.he number of toss:c.~ and 1hat a particular outcome occurs S times. If 1he ratio S/ N approaches a limit
a.~ a function of N. the number of 10..-.c~. as N ➔ oo. we say Lha1 Lhls limi1 is the probability of Lhat particular outcome.
Suppose now 1hat an experiment hac; n possible outcomes £ 1• £ 2 ••.•• En.
where one and only one outcome can occur in a given experiment. The set of all
possible outcomes, {Ej, j = I. 2 .... , ,r) in this case, is called the sample space
of Lhe experiment. As described above, the probability of any particular outcome
IS

)=1,2 ..... 11 (I)

where Nj is Lhe number of Limes Lhe event j occurs in N triaJs. We can see from
Equation I that if some event. say the atb. is certain to occur. Lben Na = N
and p(Eu) = I. On the other hand. if ii is certain no! to occur, then Nu= 0 and
p( E(i) = 0. Becnuse these 1wo cases repre.sent the possible extremes, we have that
0 :-::: p(E) ~ I as a condition tha1 a probability must satisfy. Furthermore, because
N 1 + N 2 + · · · + Nn = N. we have
II

(2)

This result says 1ha1 it is cenain that one of the II events will occur when the exper-
iment is perfom,ed (in other words. that something will happen). This condition
is called 11onnaJizmio11.
Now consider Lhe more complicated case of an experiment in which two eve.nls
A and B can or cannot occur. We can set up Table 21.1. Since the four cases given
in Table 21.1 account for all Lhe possibilities. we have

(3)
21.1 Discrete Random Variables 1025

Table 21. li
An experiment in which A and B arc Lhc lwo
possib'le outcomes of interest.

Result Number of occurrences

I. A and B Ni
2. A. but not 8 N2
3. B. but not A N3
4. Neither A nor B N~

If we let N be suitably large. the probability that A occurs is

p(A) = N l +N2 (4)


N

and that B occurs is

p(B) = N1 + N3 (5)
N

The probabi Iity that both A and B occur. p( A. B). is given by

p(A, B) = -Ni (6)


N

This is known as thejoinr probabiliry of A and B. The probability that either A or


B occurs is given by

p(A + B ) = N 1 + N,- + N1. (7)


N

We can also define conditional probabilities. The probability that A occurs,


given that B has occurred. is

p(AIB) = Ni (8)
N 1 +NJ

Similarly the probability that B occurs, given that A has occurred, is

(9)

We can use the formulas given above 10 deduce the following relations:

Ni N2 N-:.i,
p(A + B) = - + - + -
N N N
= p(A) + p(B) - p(A. B) ( 10)
1026 Chapter 21 / Prnb.:ibility Thewy ,md S10 ha lie l'roces

N1
p(BIA)p(A) =
N1+N2

(11)

( 12)
, 8
B

Equations I O 1hrough 12 can be seen pic1ori ally in Figure 21. 2, where the
Nol A nor B rectangular area E represents the space of all possible ou1comes of an experiment
and 1he areas A and B reprcsen1 the space of ou1comes of A and B. respectively. If
Figure 21.2 £ is a unit area. then the area A is equal 10 p(A) and the area B is equal to p(B).
A Venn diagram. which illus1ra1~ the
The region of overlap belween the A and B areas represenl~ the simultaneous
probability space of the ou1comcs of A
and B. occurrence of A and B. or p(A. B). Equation IO then follows immediately since
p(A + 8) is the total shaded area in Figure 21.2. Equation 11 follows by noting
that p(BIA) = p(A. 8)/p(A).
We can also use Equations 10 and 12 and Figure 21.2 to fonnularc some
definitions . If the occurrence of A precludes the occurrence of B and vice versa.
1here is nu overlap region in Figure 21.2 and p(A. B) = 0. Therefore. p(A + B) =
p(t\) + p(B) and p(BIA) = p(AIB) = 0. When this is so, we say Lhat the 1wo
events A and B are mu/ually cxc/11.sive. If the occurrence of A has no effect on
=
the occurrence of B. we have p( BI A) p( B) and say 1hat the Iwo evenrs are
independent. An important consequence of I he independence oft wo events is thut
their joint probability factors. so that p(A. B) == p(A) p( B).
Many evenL~ are descriptive in nature. such as the evcnl that the next card
drnwn from a deck of cards will be red. In many other ca.5es. events have a natural
numerical vaJue. Suppose thal all the possible evt!nls £ 1. £ 2..... £,, from an
experiment have a numerical value. Then we can represent the outcomes by a
random variable. say X. A randvm variable is a rule or a formula. that assigns
numerical values to each of the simple evenl5 in an experiment. We denote a
p m.ndom variable by a capital letter and a particular observation of the random
variable by a lowercase letter. The probability that we observe the value .r from
an experiment is wriuen as Prob IX= x} or simply p(x). It is often convenient to
interpret a set of probabilities {p(.x1) I as a unit mass dis1ributed along the x axis
XJ x such thatm 1 is the quantity of ma-.s located at the point.,f Because L p(xj) = I.
.i
we must have L mi = I. The probability distribution. then, can be pictured as the
j
Figure 21.3 distribution of a unit ma..-:s along the x axis (Figure 21 .3).
The discrete prob.ibility frequency We call 1.he variable X a random variable if it takes on I.he values x 1• x 2.... , x11
function. or probability dcn.~ity.
wilh respective probabi Ii ties p (x 1). p (x 2) ••..• p (.\'n) . The set {x 1 ) over which X
p(x 1> = Prob IX = .ri l.
varies is called the .wmple space. The expected va/11e of X or the mean mlut! of X
21. I Discre1c Random Varir1bl s 1027

is given by
II

/1 = E[Xl= LXJJJ(Xj) ( 13)


j=I

By referring 10 our analogy of mns.s. distribution. Equation 13 says that rhc ex.pcc-
lalion value is rhc cen1er of mass. More gencrnlly. if I (X) is a function of X. we
define the expectation value off (X) by

E[J(X)I= L f(x1)p(xj) (14)


j::: I

If f ( X) = X' 1• then E[ X 11 ] is called the 111h mommt of 1he probability disLrihution


{p(x 1) f and is often denoted by m,,. Note that the second moment correspond.;: to
1he moment of inertia of a mass distribmion.
You should be aware of the fact t.hn1 £1 X~ I does not necessarily equal £[Xf1
as the following Example shows.

Example 1:
Given the following probability data. calculate £1 X J. E[X 2 J. und El X f.

X 2 4 5

p(x) 0.10 0.15 0.05 0.50 0.20

SOLUTION:

£[X] = (1)(0.10) + (2)(0.15) + (3)(0.05) + (4)(0.50) + (5)(0.20)


= 3.55
EJXI: = 12.60

El X 2 1= (1)(0.10) + (4)(0. I 5) + (9)(0.05) + ( 16)(0.50) + (25)(0.20)

= 14.15
Nole: !hat E f X 2 t > £'[ X ]2. Wc sho 11 prove llelow Ihm this is a £Cncral ~~uh.

Example 2:
An important discrete probability distribution is the Poisson disrrihwirm,
where XJ = j and

. ll j _
p(j) = - e "
p
j = o. I. 2....

al
1030 ( h,1ph!r :! I / Pr, ,h,1hilih I !won ,111d !:>Inc h,hti, Pr

The quantity Cov [ X. YI;:::- El ( X - /1xJ( Y - µ y) J is culled the cnl'ariance of


X und Y. Problem 7 has you show thut £[ (X - JLx)( Y - µ,)I;:::- 0 if X and Y are in-
dependent. In such a case. X and Y are said 10 be uncorrelated. If Cov [ X. YI I- 0.
1hen the magnitude of E[(X - J.lx)(Y - µ_\)Ii~ G measure of the lack of inde-
pendence (degree of correlation) of X and Y. However. Cov [ X. YI= 0 docs not
imply independence.
We shaJI now discuss lwo well-k.nown and imporwnL discrete probability
distributions. First let us consider 11 successive losses of a coin and ask. wha11is the
probability that heads comes up exactly m times. The rcsuUs of any 11 successive
tosses can be represented by a sequence of h ·sand , 's :
Jil1hlh11t · · · 11h
There are II posi1ions in any sequence and Lherc are 1wo choices(/, or,) for each
position. Thus there arc 2" possible sequences. and since 1hcy arc all equally likely,
1he probability of any particular one will be 2-". Tbc number of such sequences
with heads exactly m limes is equal 10 1he number of ways we can arrang~ 111 heads
among II positions. or

m!(n -m)!

The probability of exac1ly m heads, p (m). then. is


1

p(m) - - -11!- - ( -I )'


m!(n - m)! 2
More generally. if a "sucl·c:-..:-.fur· ou1come occurs wi1h probabiliry p and an ··unsuc-
cessful'· outcome occurs with probability q. where p + q = I. then 1he probability
of 111 ··succe~sfur· ou1comes is given by

n' n'
p(m) = . pmqn-m = . p"'( I - p)"-m (23)
m!(11 - m)! m !(n - m)!

Thi$ distrib111ion is known as the binomial di.~1rib111io11 and is applicnhlc to the case
of repeated indcpendenl trials .such a,; the dr.iwing of cards from a deck. replacing
the drawn card after c.ach draw. Equation 2J is called lhc binomial distribution
because 1he binomial theorem of algebra says 1ha1
II I

(\• . ..i.I )')"-_ L


"' II.
.\.111 \.11-m (24)
Ill !(11 - Ill)~ .
(
m=J

Figure 21.5 shows I.he binomial dis1Tibu1ion plot1ed against III for ·everJI values
10 20 40 ,,, of 11. Nore I ha1 the dis1 ri bu1 ion hccomes more and more bcl I-shaped as n increases.
figure 21 .5
The hinomial Lli,11ibu1i11n ploued 1gain1-1
111 for scvl.'ral ,.iluc~ or 11 (6. 12., 24, ond
48). N111e that rhc di~1ritiuri,m lx:comc. Example 4:
increasin£1Y bell-shaped :i::- n incrc isc ·. Show thal lhc binomial distribulinn I'- normali1.cd and 1ha1 £1 M] = np and
= =
Var I M I n ~ 11 fU/.
21. 1 Di r 1 Random Variables
0
1031

SOLUTION: Equation 24 with x = p and _I'= q =I- p shows lhat


Equal.ion 23 is nom1alizcd. The mean is given by
II 1r I

ElM]:::::Lmp(m)=L m11. p,,,(1-p)'i-m


m==IJ m=O 111 !(n - n,) !

We can evalualc I.hi), sum by differentiating both sides of Equa1ion 24 wilh


respect to x and then multiplying the result by x to obtain

Le1ting .r = p and y = I - p gives

E[MJ=np

Another such differentiation gives

Letting x = p and _v = I - p gives £(M(M - I)]= 11(11 - l)p 2• or

The binomial distribution is used 10 describe a one-dimensional random walk


(Problem 12).
Allhough the binomial distribution is important and useful for its own sake. it
also serves .is the basis for two other perhaps belier known distributions. namely
the Gaussian disrrihution, which we discuss in 1hc next sc:l:tion. and the Poisson
distribution. which we derive now. Consider the following problem: We randomly
distribule II points over some interval (0, I). and then ask what is 1he probability
lhat exactly m of these points wi.11 lie in some subinterval !),.r (figure 21.6). We
can Lhink of this problem as consisting of,, repeated.. independent trials or placing
a single panjclc in the interval (0. t) with ··sucl:cs),." being 1he probability that
the particle will lie in the subinterval !),.r. which is equal to p = !),.1 / r. llrns, the Figure 21.6
probability I.hat III of lhe n particles will lie in !),.r is g-iven by 11poinls <lis1riburc-<l r;mdomly over rhc
intcn,.,1 (0. I) .
11 l
p(m) = . pm(I - p)''-rn
m!(n-m)~
(25)
= II.I A )nr
(~ ( 1 __A )n-111
u_l
m!(n-m)! t t

The average vaJue of m isµ = up = 11 at/ t, which we denote by ).. tit. We are often
in1eres1ed in the ca<,e where n i~ 'large and {),./ is small (with µ fixed at .\ !11 ). in
1032 Chap1er 2 I / Prob;ibiliry Theory and 510 ha Ii Proce

which case Equation 25 can be cast in a much more convenient fonn (Problem 14 ):

(26)

Equation 26 is the probability that there wi 11 be m poi n1s in an interval b.r if n poin1s
are randomly distributed over the interval (0. , ). n is large. t!i.t is small, and >.. = n / r
is the number of particles per unit oft. Equation 26 is a Poisson distribution. and
we showed in Example 2 thatµ.= a 2 = ).. 6.1.
Equal ion 26 applies to radioactive decay. among many other physical phenom-
ena. ln this case, ). is the mean rate of decay and Equation 26 tells us I.he chance
that we will observe m decays in a time interval Di.I. No1icc 1ha1 the probability of
observing no decay in an interval 6.1 is given by p(O) = e->.i:J.i .

Example 5:
A radioactive sample is observed to emit alpha particles at a rnte of 1.5 per
minute. Determine the average or expected numher of alpha particles that
you would observe in two minules. Calculate the probability that you would
observe 0, I, 2, 3, 4, and equal 10 or greater than 5 counts.

SOLUTION: According 10 Equation 26. 1he expected number of counts in


a two-minute in1ervaJ is )..f..1 = 3.0 counts. The probabili1y of observing m
counts i~ given by Equation 26. or

m 0 2 3 4

Prob IM= 111} 0.0498 0.149 0.224 0.224 0. I68

The probability that m ~ 5 is given by


.i

?rob !M ~ 5) = 1- L Prob IM = 111} = 0.185


m-0

The Poisson distribution hos an amazingly broad rnnge of physical appllca-


(a) 1ions. including radioactive decay. nerial search, the arrival of electrons sLrik.ing a
cathode. the transmission of a nervous impulse across a synapse, the distribution
(b) of galaxies. and so fonh. We"ll write Equation 26 in the general case as
0.2

(27)

2 3 4 5 6 7 8 9 where a is equal to £IM J. Figure 21 .7 shows p( 111) ploncd against m for several
values of a. For example, suppose it is known that there are 300 errors in a book
figure 21.7
The Pois on disiriburion plolled agains1 m
containing 500 pages. What is the probability 1ha1 a page contains no errors? Three
for (.i) a= 1.0 (bluck). (b) ll = 4.0 (color), or more errors? Here the "rime interval" is the area of a page. and we wish to
and (c) a= 6.0 (grdy). calculate the frequency of errors on a page given that on the avernge 1here are
1034 ChJp!er 21 / l'robabilily Theor,• and S10 ha Ii Processes

and

p(+ ID)+ p(-1 D) =et+ p(-1 D) =I


What does a positive test resull indicate? What about a negative result? These
questions are answered using Bayes·s fonnula.
Let's consider a rare disease where p( D) =
1.00 x 10- 4 ( I in I0,000) and
where Lhe lest has a= 0.0100 and /J = 0.0500. Now suppose thal you test positive
for the disease. The probability that you have 1.he disease given lhat you Lestcd
positive is given by Equation 29:

P(D I+)= p(D)p(+ ID)


p(D)p(+ ID)+ p(D)p(+ I D)
p(D)( I - fi)
=--------
p(D)( I - {J) + p( D)a

( 1.00 x I 0--.i) (0.950)


=--------------
(1.00 X tQ- 4 )(0.950) + (0.9999)(0.0\00)
= 0 .00941
or about I in 100. This result may not seem 100 informative. However, the chance
of having Lhe disease bas increased I 00-fold.
What about a negative test result? In other words. what is the value of
p( D I - ). In this case.

p(D1-)= p(D)p(-1_!!)
p(D)p(- I D) + p(D) p(-· I D)

p(D)/3
=--------
p(D)/3 + p(D)(I - a)

c1.00 x I o- 4 )(0.050<>)
=( 1.00
- -X-I -- ----------
O )(0.0500) + (0.999())(0.9900)

= 5.05 X 1()-ti

Thus. there is a very small chance 1hal you ' ll have the disease given 1hal you 1es1
negative .
Let's caJculaLe the chance 1ha1 you don"t have the disease and yet test positive.
In other words, ler's calculate p(D I+). This is given by

p(D I+)= I - p(D l +)


= 0.9906
Thus. Lhere is a 99% chance Lhal you don't have 1hc disease given that you test
positive. Finally. the probability that you don't have the disease given Lhat you test
1036 Chap! •r 2 I / Probabil ity Th •ory and S10 hasti Pro

21. 1 Problem

Probabi/iry problems can be .wmewlwr tricky 10 soh·e and somerimes the re.mlrs are fairly cmmterinruirive.
TJu, first rwo problems are examples whose rl'\'11/ts mony people find surpri.ring (or even 1mcom•i11ci11g).

t. If heads comes up ten limes in a row. many people feel s1rongly that the next toss is more likely to be tails than
heads. How do you explain ro them that they are dead wrong•.>

2. Consider a group of II persons. Calculate the probability that at least two of them have a birthday on the same
day of Lhc year for n = 50 (exclude leap year). What i.,; 1he ~--mallc.st value for which lhc probability is g.rc:ucr
than 1/2'! Him: First calculate 1he total number of ou1comcs and then cakulate the number of ways tha1 no
1wo persons have the .".>ame birthday. Now calculate their ratio to get the probability 1.hat no two have the same
binhday and then subtract this resull from I to get the probabi.lity that ai least 1wo do have the same birthday.

3. Let p(A) = 0.150 and p( B) = 0.420 be indcpendenl events. Calculate the probability that either A or B occurs
and 1.he probability that both occur. What if A and B are mutually exclusive?

4. Show that £IX - µ] = 0.


5. Show that Var [Xl = a 2 = £[X 2 ] - £[ X]2.

6. Show that E[aX + b)'"] = aE[X] + bE[Y].

7. Show 1hat El(X - µ.1:)(Y - µ_1._)] = 0 if X and Y are independent.


8. Show that Var [X + YI= Var [X I+ Var [ Y] if X and Y are independent.
9, Show that £1( (X - µ_ + (Y - µy) )3] = El(X - µ, ,·1 1+ £[ ( Y - µ 1 /J if X
1 ) and Y arc indcpendcm. In other
words, 1he lhird central moment of 1he sum of independent random variables is addi1ive.

10. Show that El /(X - µ.r) + (Y - µ 1.)}-11 ::/= E[(X - µx)''I + E[(Y - µy)' 1 ] if X and Y are independent. In other
words. lhe fourth central moment of the sum of independent random variables is not additive.

II. Show 1ha1 Var [ t, a,x,] t, t.= •,a1Cov [X;. X 1J.

12. We shall im1estigate a one-dimcn <:ional random walk in 1hi" problem. Let u particle s1::tr1 al the origin and lake
one step 10 the right if a toss of a coin yields heads and one --rep to I.he ldt if the toss yield.-; tai Is. Show that the
probabi liry of being at lhc point m aflcr n tosses of 1hc coin is

I 11!

l
n and m bo1h even or both odd
P(11. m) = 2" ( ~ ) ·· ( ~ ) ··
2 2
0 orJ1crwise

Show that the average position after 11 tos,c~ i~ zero. Plot P (11. 111) for al I ,,alucs of 111 for n = 3 and 4.
13. We show in this problem tha1 P(n. m) in the previous problem becomes a bell-shaped curve as 11 and m become
large. Take the lo~arithm of P(11. 111) and use Stirling's appm.xirna1ion 10 oh1ain

ln P = 11 In 11 - ( 11~
+111) In (" +m)
~ - (11~
- 111), In ("- -2- "') - 11 In 2
1038 lh,1pter 21 / Probabiliry Theory and \1ochasric Prm t...,~t-s

The cumulative distribution function P(x) is given by

P(x) = Prob {X ~ x) = 1·r -00


p(x) dx (4)

If P(x) is continuous. Equation 4 gives

dP
p(x) = - (5)
dx
The expected value of a func1ion of a continuous random variable is

£lf(X)I = 1_: f(x)p(x) dx (6)

The central momenL-. are simply E[(X - µ)"I.

Example 1:
A uniform prob:::ibiliry density is given by

otherwi.-.c

Firsl show lhat p(x) is nonn.:1\izcd and I.hen calculalc i~ l'irsl two ccnu---.i.l
moments.

SOLUTION:
1
b -a
1
'

a
p(x)dx= -
b- a
- =I
I b2 - a 2 b- a
f
b
µ=£IX)= .xp(x)dx = - - - = - -
11 2b - ll 1

We can extend Lhese ideas LO more than one rnndom variable. Let X and Y be
two cont..inuous random variables. The joint probability density is

p(x. y) dxdy = Prob {x :S X ~ x + dx and y .:s Y :S y + dy) (7)

and the a.:;sociated cumulative di ..;:1ribution function is

P(x, y) = Prob {X ~ x and Y ~ y I=


1\" 1·\
-,x,
_d_v
- v
dx p(x. _r) (8)
1042 Ch,1plL"r 21 / Prnhabiliry l heory Jnd 10 hasri Proc

where X = (x 1• X2 • . . . • xn) and l = (11. 12 •...• r,,).


Another useful properry of an n-dimcnsionsal Gaussian distribu1ion. which
is proven in Problem I0. is Lhc following: Lei X 1• X 2•..•• X n be II i ndepcndcnt
normally distributed random variables with means equal to zero and variances o}-
Le1 Y1• Y2..... Y11 be linear combinat:ions of 1he X,-: i.e ..

Y=MX (26)

where M is nonsingular. Then 1he ~i have an m-dimensional joint normal distri-


bution given by

(27)

where

D=MAM1 (28)

21 .2 Problems

I. Verify Equa1ions 11 and 12 if X and Y arc independent random variables.


2. Show Lhal £[ X Y J2 ~ £[ X~ / £[ Y2 J. Hi111: Follow the proof of the Schwartz inequality in Section 9.6.
3. Show that the l:Orrclation coeffkicn1 P.~J' sa1is l'i~~ -1 :::: p_,_,. ~ I. Hi111: Use:: the inequality in the prcviou-'
problem with X replaced by X - JL.r and Y by Y - /l_, ..

4. Show Lhat p(x) given by Eq111:cnion 17 is normalized.

S. Equation 24 shows thal the Fourier transform j, (s) f without the factor of (2;T) I/'.'. l of a Gau,sian distribution

1s
,:,,
e "·' -. Show
-
l}iat P(O) = I and tliat ->)
(
-
dP(s)
ds
= 0 ond (2")
d P(s)
--
d,-1
- = -n-." Docs
.
1h1s M1~gl',1
r=O .•-=-0
anything 10 you? See the ne~t problem.
6. Show Lhat rhe derivatives of l11c Fourier tnrnsfonn of any continuous probability density

P-(s )= f .x, 1· in

- "'-
- p(x)
dx arc re Iutcd· · to 11e . by (di/
I moments at. t he probab1·1·11y J cnslly P(.1·))
- _-,,-
d.5 .-:=0
=i JI
E[_X n ].

7. Use Equation 22 10 write out the Gaussian distribution for two random variubles, X and Y.
8. Show that your resuh for p(x. y) in the previous problem reduces to p(.r} p(_\') when t> n
1
= 0.
9. In 1his problem, we show tha1

( I)
21.2 C..1111tinuous R,indom Va,iabk~ 1043

Let S be the normalized modaJ matrix of A- 1• so I.hat sT A- 1 S = D and xT A- 1 x ..:· x ro x', where x = S x'.
( The quadr.:uic form is now diagonalized.) Now let u = s- 11 = S I I so that t 7 x = (S u) 1x ·"' u rSr x = uTx'.
Thus. we have

where the ),i are 1hc ei,gcnwilneli of A- 1• Now use

and u 10- 1u = trS o-i5r t = trAt to verify equalion I.

lO. We shall derive Equation 27 in this problem. Substitute x = M- 1y into the exponent of Equation 22 10
obtain xTA- 1x :::: yf (M- 1JTA- 1M- 1y. Now use the relation (SR) 1 R- 1s- 1 to show 1.h□t x1 A- 1x = =
)'T (MAM1')- 1y = yf o- 1y. which defines D. Lastly, show 1hn1 det O = det A(del M) 2.

The distribution:, of rhe speed.~ of molecule.t in the gas phase £Jre gow,med by Gaussian di.rrrih11ti01L1'. The
folloll'i11g prohlems irrvoh>e thr kim·ri, theory of 6 f1Jes.

11. The di.o;trihu1ion of the comp()ncnL-. of the vcloci1y of the molecules of a ga.-. i$ given by f (u r)d11.1 =
1/ 2
111
- - -
( 2ITk T )
e- "111_;/2k~Td11x, where III is Lhe mass of the molecule. kB is 1he Bohzm::mn constant and T is
11
the kelvin lemper.nure. Detennine E[Uxl- EIU_;J. and £[1111U_;l-

12. U,ing 1hc disr:ribu1ion in the previotL, problem. derive an expression forihe probability 1ha1 -11 ,n ~ Ux :S "xO·
bpn:~, your result in 1enns of the error func1ion of w 11 = (m/2k 8 T) 1l 211x0. Calculate Lhc prob,1bili1y !hi.It
(-2kt1T/m)::: U_,:::: (2k 8 T/m) 1l 2 .
13. Use the result of the previous problem to :.how lhat Prob II V.,12:: ".rol = I- crf (11 0 )- Cnlcu]ate
Prob {IU.rl:::. (kuT/111) 112) and Prob IIU.rl ~ (2ksT/m) 112 !.
14. Use the result of Problem 12 10 plot rhc probabi I ity 1hu1 -11_r1) ~ U_, :5 u .rO against ''xo/ (2k 0 TI 111 ) 1

15. We an: often more in1erc~1cd in the dis1rihu1ion of molecular :-pccds rnthcr than just componcnt.s or velocity.
To detem1ine the distribu1jon or
molecular spec-ds. hrsl write

Now. using the r::icl 1ha1 the speed " is given by 11


2 = u~ + 11~. + 11:. conven the :.ibove equation to
3/2 . ~
111
f'(11)d11::;: 4rr - - -
( 2nk T )
,/e-"1112 f 2kAT du. Now determine £[VJ. E[U 2]. and£[ {mU 2 1.
8 -
16. Another dis1ribu1ion that is fl'C(1ucn1ly used in 1hc kinetic t.hcory of g;N:, is the distribution of energy.
or kine1ic energy in panicular. Let <c = mu 1/2 and use lhe result of the previous problem !o show that
F(<: )d<: = 2;,, r '= I/'-e- / J.. 0T d<:. Now calculalc £[<'I and compare your rcsu.ll 10
I ..,
£1 ~111U-J 1hal you
(irkaT> -
obtained in the previous problem.
1044

17. In 1his problem, w~ shall prove :m interesting 1heorem called Clwbyslu· 1·'s ineq1111L/1y, whi,ch says 1h01
Prob UX - cl 2: ~ I .::: ~ £( ( X - c) 2 J. where c and i: are any real numbers. If p(x) is lhe density function of
€~
X. then
Prob [IX - l'I::: EJ = 11.r - cl _ i
p(x) dx. Now argue that

Prob [IX - cl:: '=I = f p(x) dx 5. f


18. Use Chcbyshcv's inequality (previous problem) to show lhaJ. Prob [IX~µ, 1 ~ a] 5.
1
02
. Interpret this re$ulL

21 .3 Char cleristic Func tions

Given a probability density function p(x). we can uniquely define a new function
f/>(s) by

(I)

The function tJ> (s ). called 1he characteristic function of p (x ). plays a central role
in probabili1y theory and iL, applicntions. Notice tluat 1hc churuc1cris1ic function is
essent.ially the Fourier transform of p(x) (without 1he prefac1or of l/(2rr) 112). It
is also equal 10 the expectation value of eiu·. as Equation I indicates.
We shall see Lhat the charac1eristic f unctjon has a number of important and
u.seful properties. The first that we demonstrate 1s the following. If we repeatedly
differcntiale </) (s) with respect to s and then set s equal to zero. we obtain

,, = 0. I. 2 .... (2)

We see then thal if the characteristic function is known, then it is a simple maltcr
to compute the 11th moment of p(:r).
If all I.he momcnLS a.re known. then the Maclaurin expansion of tj)(s), and
hence <J,(s) i1self. is known since

I II S2
1/J(s) = ,t,(O) + ,p (O)s + <t> (0)- + ••• (3)
2!
Now if tf>(s) is known. then p(x) is known uniquely through the inversion theorem
of Fourier transforms, namely (Problem 6)

p(x) = -I Joo e-'JX</>(s)


•. ds (4)
211 -00

Thus. a knowledge of all the momenL, of a probability density is equivalent 10


knowing the probabili1y density itself.
1046 ChJpl •f 2 I / Probabili ty Thi•nry c1 r1d Sto c IMslic Prnc

The first few derivative!. of t/>(.d yicltl

1
(/> (0) = i /t
and 1he fin:1 few derivative~ of <l>(s) yield

<1>'(0) =O <!>" (0) = -a 2 111


4> (0) =0

Although one does not often know aJI rhe moments wirhout knowing p(x),
it often occurs Lhat one knows the first few moments. The practical quest.ion Lhen
arise.<: as to just how much we can say about p(.x) if we know its first few moments.
Obviously. p(x) cannot be specified completely. but nevertheless. the first few
moments do describe the main features of p(x ). For example. all dis1ribu1ions have
♦ p a unit area under the curve. The mean is a measure of where the curve is "locared"
and the variance t.el ls the spread of the curve about the mean. More precisely, it is
the magnitude of the square root of the variance relarii·e 10 the mean. a/µ (lhal
is. a measure of the spread). This ratio is calJed the c11efficiem of variation and
i!'- usuaJly denoted by y. The reason that u must be compared to µ (.·an be seen
from the following example. In statis1ical mechanics. one finds lhat fluc1uations
of the order of 10 10 molecules might exist in an open macroscopic system. This
-I X
(a) might seem like an enormous number un1il ii is realized that there are 102° or so
molecules in Lhc ~ys1cm . Thus. we see that 10 10 represents a percentage fluctuation
p of 10-s ! Pulling it another way. picture a Gaus-.ian curve centcrcd al x = 1020
wi1h a spread of a few multiple.-.. i.ay 11. of 10 101. Most of the curve appears between
1020 + (11 x 10 10 ) and 1020 - (11 x IOIO). which fur all prnclical purpose. i,, adelLa
func1ion. Although I.he standard deviation a = I0 10 • 1he coefficienl of variation
y = I o- lll and is a more realistic indicat.ion of I.he spread.
The relative third cen1r.il momenr~ 11. 3/ a 3. called the coefficient (fskewnnis. is
-I X a measure of the asymmetry oflhc curve abou11he mean. If lhis moment is zero. the
(b) curve is symmetric about the mean. If ii is positive, the curve is skewed 10 the high
side of the mean. and vice versa. The relative fourtn central momenl, µ~/rr 4 - 3,
called the kurtosis. is a measure of the peakedness of the curve about 1hc mean
rela1ive to a Gaussian distribution (Figure 21.11 ). The physical in1erpre1ation of the
highcrcentrnl moments becomes progressively more obscure. Thus. we , ee rha1 the
first few momen1s characterize the general features of Lhe probabi Iity distribution .
In Problem 16, we show how a knowledge of I.he first few momt.~nls Ci.ln be used
to approximalc 1hcir associated probability distribution.
- I X
(c) Another approach that ho.s found wide applicalion to physical problems is the
use of lhc first few moments 10 determine rigorous upper and lower bounds on
Figure 21. 11 !he cumulative dis1.ri but ion function P (x). This approach is called the method of
An illu.111ra1ion oi (,a), a dis1rib1.11ion
1-k<.-wcd to ,the high side of ,t:he mean. (b)
moments and has been developed and applied 10 a wide r.inge of physical problems.
a di:-tribution sh-wcd to 1hc low ,idc. and This is a particularly beautiful 1heory. hut ii is beyond 1hc $COJ)C of this ch.1p1cr.
(c) u d.il!tribution with tl high degree of (See the reference 10 Shohat and Tamarlcin al lhe end of the chap1er.)
peakedness.
A particularly important property of the chamctcrislic function is illustrated
1049
and so we can drop .the O(A.s: 3) tenn and write p

(13)

But if we compare this result to either Example 2 or Example 3, we see that </Jz (s)
is the characteristic function of a normal distribution with mean J\ and variance A
(Figure 21 .12). (Problem I O offers a more rigorous derivation of Equation 13.)
We have shown lhac if Z is 1he sum of n independent Poisson distribu1ed 20 X

mndom variables. then Z becomes normnlly distributed as II becomes large. Prob- Figure 21.12
lem 13 offers a simple proof that the sum of II indepcndenr binomially distributed The envelope of a Poisson distribution
random variables becomes nonnaJly distributed as II becomes large. Thus. it might with u = 20 compared tu a normal

appear that the behavior is more general than it may seem. In fact. rhere is a fa- di stribution wirh µ = 20 ond a 2 = 20.
mous theorem in probability theory tha.1 says that the sum of II random variables
becomes Gaussiun a,; n becomes large regardless of the probability distribution of
the individual X; so long as they arc indcpcndenr. identically distributed . and that
their mean and variance exist. This theorem is called the cemral limir rltenrem and
states lhat

If X 1, X 2 , ...• X arc ind~p£'11denr. idemically distrihured rwulom


II
l'llriahle .~
1vitl, mean µ and l'arianc;e O"
2, 1Jie11 as 11 - co.

y = x, +x - + .. -+ x,, ( 14)
JI

is 11ormally disrrilmteJ wi1h mean µ and \'arilJ11ce o 2 / 11.

Note tha1 Y can be thought of as an average of n measurements of X. This theorem


is the basis for many statistical method~. a') we shall sec in the next chapter.
The proof of this theorem provides a good example of the power of using
characteristic functions. The characteristic function of Y is

(15)

where rj,(s) is the characceris1ic funccion of p(x), namely,

tf>(s) = f_
Using this expansion in Equation I 5 gives
1050 Ch,1p1t·r 21 / Probability Theo ry and St h;i Ii Pro

Using the definition

this expression becomes

(16)

for large 11. Equation 16 is the characlCrislic function of a Gaussian dislribu1 ion
with mean /.l and variance a 2/ 11. Note that it is independent of I.he distribution
of x. This resuh says that the average of repeated measuremenis will be approx-
{a)
imately normal, and hence explains why the expression "normal curve of error··
occurs so frequently in 1he anaJysis of cxperimcniaJ data and why the central limit
theorem plays a fundamcmal role in malhematical s1atis1·ics . In addition. because
many physical phenomena (such as Brownian motion) nre the result of a large
number of repeated smaJI effect,:;, the Gaussian distribution occurs often in physi-
cal applications.
Figure 21.13a and b show the ex.act density function and ils Gaussian approx-
3/2 imation for the sum of two and 1hrce independent un.iformly distributed random
(b) variables. whose densi1y function is

Figure 21.13 0<x < I


1l1c exact l.lcn.~i•y function (coJor) and iLS
,o.,u-.~ianapproximation (blnck) f'oF !he
olherwise
sum of {a) two indc:pendeni uniformly
di. uibu1.cd rundom variable--~ and (lb) lhrec Notice that Lhc Gaus ... ian distribution in th.is case is a fairly good approumation
independent unifonnly distributed random even for 11 = 2 and 11 = 3. Problems 14 and 15 have you obtain 1.hc results in
variables.
Figure 2 I .13.
There are several variations of the characteristic function that are used. For
example. if the range of X is not (-oo. oo) buI (0, oo), I.hen I.he Laplace tr,.msform
is oflen more convenien1 and we wri1e

(17)

Thi~ type of characteristic function has properties similar 10 Lhosc of the Fourier
transform characteristic function (Problem 7).

21 .3 Problem
I. Determine 1.he probabili1y density funct ion of Y = a X+ b (a > 0) if X is uniformly di~IribuIcd in I 0. I J.
2. Derermine the probabiliry density funct.ion of Y = aX + b (a> 0) if X is normally distributed with meanµ
and variance a 2 .
3. Use Equation 9 to deLcnninc the probability density function of Z = X + Y if X and Y are independent
normally dislribured random variables wilh mean O and variance I .
1052 Chaprer 21 / ProbJbi lity Th ;0,y and r hasri Pr

14. Lel X) be independent random variables unifom,ly distrihu1ed in the interval (0. T). Show 1hat E[X1 I= T /2
and Var [ X1-] = T ,~ / l 2. Show 1ha11he density
. f unction
· • X = )(· + X ·1s p x(x) = {
1or x__.. O < .r < 7
1 2 27 -• T < x < 27.
Compare this exact densiry to a Gau,,ian approxima1ion for Px(x).

15. Extend the calculation in the previous problem to lhc :-um of three independenr random variables.
16. Many distributions. aJthough no1 exac1ly normaJ, do approximate a nonnal distribution in some sense. There
is a systematic expansion of an arbitrary distribution about a nomial distribution. Such an expansion ha..,;; found
a number of physical applica1ions and is caJled a Gram-Chorlier scries. Let f (x) be <;Orne probability density
that looks somewha.1 Gaussian and lei tp(x) be a normal distribution. Without lo~s of generality. let the variable
be raken 10 be a s1andardi1..ed random variable X = (X - µ)/er: in orher words. one with zero mean and unit
variance. so thal '{J(:r) = (2rr J- I/ :'.<'-.r!12 . Then we can wrirc

j(x) = tp(x) + c 1<p ,(x) + -=-t,0


c, 11
<.t)
C3
+ ....:... <p
Ill
(x) + · ·· (I)
2! 3!

Now show that the nrh derivative of e_:r:i;z is equal to the Hcrmite polynomial H i"n(x) in Table 14. l. Therefore,
equation I becomes
l
j(x) = e - :r· ~laoHeo(x) + a 1He1(.r) + a2He'2(:c) +···I (2)

which is simply nn expansion off (x) in a s:et of orthogonal polynomials. since Lhe Hen(:<) satjsfy the
on.hogonality relation (Table 14.4 ):

(3)

Show I.hat the a j in equation 2 are given by a j = . \I' .


1
loci He1(x) J (x )dx. Now show rhat a0 = (2.1r ;- I/1 •
(2rr ) - J . -oo
a 1 = 0. a 2 = 0. a 3 = £lXJ]/3!(2rr ) 112 • and a 4 = (£1 X 4 ] - 3)/4!(2rr) 112 . Equation 2 become!:

(4)

which is the form usually presented as the Gram-Chartier series. The leading term is a nom1al distribution
and 1he remaining tcm1s represcn t dcv iations or J (x) from nom,al bch a v ior. It shou Id be poi n red out that Ihe
expansion in equation 4 is valid for only a fairly $mall cla.-.s of functions. but ofLCn physical grounds can be
argued for its use.

21 .4 Stochastic Processes-General
In the first part of this chapter, we introduced the idea of a random variable and
its associated probability functions. In a sense. we could say tha1 we discussed
static probability functions since the concept of rime did not enter the discussion.
In 1he remainder of this chapter. we discuss rnndom variables and probability
distributions 1ha1 depend upon time. and hence. we discuss 1hc dynamics of
probability fonctions. or stochastic processes.
1053
Lei's start by considcriing a counting process. A random process is said 10 be
a counting proc:esJ· if Lhe random variable X (1) denotes the 101aJ number of even ls
(the dc1ec1ion of a rndioac1ive decay particle, Lhe arrival of customers in a store)
tha1 have occurred in the time interval (0, , ). Clearly, X (0) = 0, X (r 2) ~ X (1 1) if
12 ~ r 1. and X (t 2) - X (1 1) is equal to the number of even1s thal occur in 1he interval
(r 1• 12 ) wilh , 2 2: t 1• The figure in the introduction to this chapter (reproduced as
Figure 21.14) is a compu1er-generated illustrnlion of a counting process. Note that
Lhe figure is a -.taircasl! shape with unit steps occurring al Lhe random time points tj.
A coun1ing process is said 10 be a Poisson process if X (0) = 0, if the number
of cven1.-. that occur in non-overlapping time intervals are independent. and if rhe
X
number of events that occur in a time interval 12 - 11 sat.is ties a Poisson distribution,
so that

I/ =0. I. 2, ...

Figure 21 . 14 is ac1ual ly that of a Poisson counting process.


Figure 21. 14 is Lhe outcome of one particular computer run. If we run the Figure 21.14
A computer-gencr::ul!d illustration of n
experiment a number of times. we will generate :i number of stairc:isc forms
counting proce :-.
wilh different values of lhe ,,.. We can label Lhe various outcomes by some index,
say 11. so lhat each curve is given by x (1: 11). The mathernarical abstraclion is 10
consider a set of functions x (t: n). 11 = I. 2 ..... N. where N ➔ oo. Such a set of
functions is called an e11semble and any one member is called a rea/i:atio11 of rhe
process. The family of rnndom variables X (t. 11) is called a random proci•.,s or a
stocha.'itic process. A (discrete) stochastic process can be thougb1 of a.,; a funcrion
of two variables, t and 11. Roughly speaking. 1he random variable X (t) does not
depend upon time 1n a completely definite (deterministic) way. but only in some
probabilistic sense.
A rdndom process X (1) can be described by a set of probabilily distributions.
Consider an ensemble of curves described by x(t: 11). At any given time. find
the fraction of Lhe lolaJ number of curves where X (r) has Lhe value x 1• This
fraction, w 1(x 1• 1). is called 1hefirst probahility dist ributio11. Now define the second
probability disrrib11tio11. w 2(x 1 • t 1 : x 2. 12), as the joinl probability of X having a
value x 1 at time 11 mid a value x 2 al time r2 . This proceS$ can be continued on
through 1hc third. fourth. and all subsequent prohahillty di$lributions. This sc1 of
functions completely charac1erizes the random process in a slatistical sense. lr we
know the funct·ions wJ for all j. we know all thar can be known about the random
process.
In general. the complete determination of the set of probability distributions
given above is not feasible. For example. the determination of just w 1(x 1, t)
would require Lhat we de1ermine 1he number of observatjons 1hat X equal x
for all values of , . Thus. we would have 10 observe lhe lime cvo\u1ion of a
large number of rcaliwlions. Fommmely, however. there are several reasonable
assumptions that can be made that greatly simplify matters. The first and leas!
restric1ive assumption we discuss is 10 assume Lhat 1he rnndom process is srat:ionary.
1054
By srarionmy, we mean lhat 1hc form of the probability dis1ribution functions
w,,(x 1• 11; x 2 • 11 : . .. ; x," t,,) do nor depend upon a shift of the origin of time. In a
sense, we assume that the underlying probabilistic mechanism of the process does
nol change with time. More precisely, we say that 1he random process is stationary
when the probability distributions of (X (I, 11) I and {X (t + -r. n)} are the same for
any value of r. Thus, w,,(.x 1, r 1: ••• : x,,, 1,,) = wn(x 1• 11 + r; . . . : x,, . t,, + r).

Example 1:
Consider lhc random process

Y(I) = (-]}""(/)
where X (f) is a Pois.,on process wi lh mean J..r, so thal ),_ is the mean rate
at whjch Poisson events occur. Note thnt Y (r) starts 111 Y (0) = I and then
switches between I and -1 at random Poisson times T1t T2 , .... as shown
in Figure 21.15. This process is known a~ a semirandom telegraph signal
Figure 21.15
A semi.random lelegraph sisnal. ~ otice
=
because 1he initial value is Y (0) I. instead of being random. Determine
I.Ml Y(O) = I. E[Y(l)I forth.is process.

SOL UT 10 N: The equation Y(r) = (- l)X<r l says that


if X(I) is c\'en
Y(t) = {- : ifXU)isudd

But X(1) is a Poisson process. so

0 .1)4

Prob f X(f) 1s even}= e-
j
J
[
I+ - -
(>. /)2
+- - + .. ·]
2~ 4!

and


Prob {X (t) 1s odd}= t·-
:>.1 [Al- + -(,l._/)3- + • .. ]
p 3!

The cxpecta1ion value of Y(f) 1s given by

£1 Y(I) I= ( I) Prob IX (t) is even! + (-1) Prob IX (n is odd!

Thus. we see that this proce~, is no1 stationary because £[ Y (r)] depends
upon,.

al
1055

The process described in Example I is a semirandom leleg.rnph signal because


Y (0) = I. If Y (0) takes on the values ± 1 wi1h equal probability. then the process
is ea.lied a rrmdom telegraph signal.

hample 2:
Show that a random 1elcgr..1ph ~ignal ha.-. a zero mean.

SOLUTION: Lcl Y(O) = y11 , where Prob l.ro = I)= 1/2 and
Prob U·o = -1 I = 1(2.. Then

Y(I) = I -I
if X (I) is. e\'en and ..ro = I
if X (1) is odd and Yo = -1
if X(t) is odd and Jo= l
if X {I) is even and Yo = - I

These four events are rnutuaJly exclusive, so

£1Y(t)]= (l)Proh (X(I) ii-even! Prob l.\•o= ll

+ ( I) Prob IX (1) is odd I Prob IYo = - 11


+ (- I ) Prob I X (I ) is odd l Prob 1_,·0 = II
+ (- I) Prob I X (I ) is even l Prob IYo = - II

For a su.11ionary random process. we may. in principle at lca.sL dctcnnine the


various probabilily distributions from 1.he ex.perimenwl observn1ion of .x (I) for one
sy:-.lcm over a long period of lime. This long time record can he cut up imo pieces
of length T (where T is much longer than any .. periodicities" occurring in the
process). and 1he:-c pieces may be 1rea1ed a.c; observations of different sys1ems in
an ensemble. The underlying assump1ion here is thl· !'-.0-called ergodic l1ypothesis.
which slates Lha1 for a s1a1ionary random process, a large number of observations
made on a single ~ystem al N arbitrary instants of time have lhe same stati.,tical
properties as observing N arbitrarily chosen systems at the ~ame time from an
ensemble of similar systems. The subject of ergodicity is extremely involved. but in
almost an physical appl ie-a1ions of stoch,.1s1ic process\.'!'-.. we as,umc that a sr.at ionary
process is ergodic.
The ergodic hypo1hcsis and ,t he assumpt.i,on of s1,.uionarity have an important
consequence. In deaJring with general random processes. there are two 1ype.s of
mean va,lucs that we. encoumcr. One is obtajncd by obf>CrvaL.ions made on many
systems al some fixed lime r. If we denote this ensemble avero~e by (f (xHcnscmble •
then
(/(.x))co~ntble =L f (x)w 1(x) (I)
.1·
21.4 Stocha..ric Pro es General 1057

and consequenlly, has generated a great deal or research, much of it arcane and
abstruse (a check of the University of California library holdings show 188 titles
involving Lhe key words "ergodic Lheory'')_
The probability distributions of stationary random proce~scs a.re indeed sim-
pler 1han 1hose of lhe general case. bur ii is necessary. nevertheless. to make rurther
restrictions as well. This leads us to another impor1ant classification of random
processes. A random process is said to be purely random when values of x at dif-
ferent ii.mes are completely uncorrelated. The probabili1y distributions in lhc case
become

w2(x 1• t 1 ;x 2 ,= w 1(x 1• r 1)w 1(,r2• 12 )


12 )
(4)
W3(X1, l1;X2, t2;XJ, f_i) = W1(X1, f1)W1(X2, l2)W1(X3, 11)

and so on. The random process is completely specified by w 1• Purely random


processes do no1 occur often in physicnl applications since in most reaJ situations
x 1(1 1) and x2 (1 2) will be correla1ed al leas! for small values of , 2 - 11.
The next more complicated case. and one that turns out Lo be a reasonable
abstraction for a large number of physical processes is to assume that the proccs
is a Markov process. In order to define a Markov process. first divide the time a.1tis
into srnaJI intervals of length iS. and lei , i = JS. Now inLroducc the conditional
probability Pn(X11, t,,l.ro. lo: ,1:1, ti: · · ·: Xn-1• ,,,-1) (to< '1 < ···<Ill) !hat X = Xn
at time 111 given lhat X (10 ) = x 0 • X (1 1) = x 1•.•. , X (r 11 _ 1) = x,,_ 1. We define a
Markov proceS-'!i by the requirement thaL

Ln other words. the probability 1hat Lhc system "is in the state x 11 " at lime 111 depends
only on its state direclly preceding t,, and not on the entire previous history of
the process. Thus. all the condi1·ional probabilities p 11 (x,,. 111 1:.\· 1• 11; ••• ; x,,_ I• 111 _ ,)
reduce 10 p 2 (x, 1 • 111 !x,1 _ 1, ,,,_ 1). Funhermorc. because of 1hc general rcla1ion

(6) 3
'~ j '
u
1/4
a Markov process rs c:omple1ely specified by lhc first two distributions w 1(x 1, t 1)
and w 2 (x 1• 11; x2 , 12 ). Since we shall consider only stationary processes, Equation 6
reads 1/4 1/2 ~, I /2
2
, I.
(7)

Markov procc~~cs allow us to introduce transition probabilities. To see what 1/2 1/2 1/4
we mean by Lh.is, we ·11 consider a Markov chain, which is a Markov process in 1/4
n ,,
wh.ich time as well as X Lakes on discrete values. Thus, a Markov cba.in evolves
=
in discrete steps. n 0. I, 2..... If X (n) =i. then the Markov chain is said to be
"
in the slate i af1er n steps. As a concrete example. consider 1he threc-s1a1c sys1em Figure 21. 17
shown in Figure 21.17. We define a probability transition matrix P having Pij as its A thn:t:-~ratc Marlo\' chain with a
matrix elcmems where Pij is the probability that the system undergoes a transition rran.~ition matrix given by Equu1ion 8.

C gl
1059
probability dis1ribution. w 1(x, f)dx, is the fraction of the members of the ensemble
for which X (1) lies belween x and x + dx at the time 1. We now define 1he second
probability disLribu1jon. w2 (x 1 • r1: x 2 • t 1 )dx 1dx 1 as the joint probability of finding
X between x 1 and .r 1 + dx 1 at time 11 and between x 2 and x 2 + dx 1 at time 11. As in
1hc discrete case. this process can be continued on through the lhird. fourth, and all
subsequent prohabi Iity disuibut ions. This set offunctions completely characterizes
the rnndom process in a s1atis1i1.:al ~ensc.
The probability dislributions for a ~Ia1ionary rdndom process become w 1(x).
w 1 Lri, x 1 : r2 - 11). w 3 (x 1 • x 2 , x_,: r1 - 11• l_1 - r 1). nnd so on. Equations I and 2
become

(I I)

and

(f(.r))1in1.·
1
= TLimoc, -2T f-T
T
/(.r(f )) dr ( I 2)

which arc equal according to the ergodic hypothesis.


There are two lunc1ions associated with stationary rnndom procc~.Sl'S that
are central to the theory of stocha."tic processes. These two functions are the
a111ncorrela1i,m Jime1io11 and 1hc .fpectral density. We shall sec that stationary
random processes can be well characterized by either of these two functions. The
crnTelation function of a continuous stationary random process is defined by

Rx(r):::: lim - I
T-~ 2T
!T-T
x(/ + r).r(r) dr (13)

Accon.ling to the ergodic hypothcsi,. we could also write

{14)

Note lhat if x 1 and x 2 are uncorrelated. !hen w 2 (.\· 1• x 1 ; r) = w 1(x 1)u· 1(x2 ) and
=
R..I' ir)2
\ l.'I I 'l'llllll,_.-
O n phy,il·,il g-rounds. x I and x 2 usually wi 11 be more corrda1cd for smal I r than
for larg(" values of r. and hence R(r) should be a bounded. continuous function
of r. In fact we can prove that R(O) ~ IR(r) I (Problem 11 ). In addition. R_t(r) is
an even function of r (Problem 12).

Example 3:
Dc1cm1inc Ihc au1ocorrclaIion function of the random telegraph ~ignal
described in Example 2.

SOL U T I ON : We need to cval u aie £ l Y (I) Y (I + r) I ti rs I. When r = 0,


= +
Y 2( r) I, so Ih..: value of Y (1) Y (r r) depend~ only upon Ihc number or

C gl
1060 Ch.:iplcr 21 / Prob.:ibili1y l h1•11ry and Srrn "h.c1stic Pro esses

C\'enl$ in Lhc interval (r, I + r ). The product Y (I) Y( I + r) = I if there is an


even number of evcnrs and Y (t) Y (I + r) = - I if 1.here is an odd number uf
events. Therefore. following Example I.

Rr(r) = E[Y(l)Y(I + T)] = (l)e-i.r cash >.r + (-l)e->.r s1nh }.r


( 15)

which we can write as e-!>. lrl because Ry(r) is an even function of r


(Problem 12). Exponentially decaying autocorrelation functions occur
frequently in physi,al problems.

The other of the two central functions in the Lheory of stochastic processes is
the spectral density of x(t). Let

-T-s_r-s_T
( 16)
otherwise

Let the Fourier lransfonn of xr(t) be A(w).

= JT
f
,,o
A(cv) = X;(t)e-iwr dr x(t )e- u'" di ( 17)
-oo -T

so 1ha1 by Fourier inversion

( 18)

The spectral dl'llxiry S(Lo) of :c(/) is defined by

I ~ '>
S(w) = lim -IA(w)I- ( 19)
T-oo 2T

Problem 13 has you show 1har S(w) is an even func1ion of w if x(r) is real.

Example 4:
Show that

SOLUTION: Using Parseval':i theorem (Section 17.5). we can write

1. ,
f
;:x.. ., I
_ , Xj-(1) dt = 2;r _ _ JA (w l""drJ)
2 1.5 S10 h,1stic Pro :_e" ,, - Exanipl ,, 1065

The vaJuc of 1:heo;;c .sums can be c.nsily dctennincd using a CAS. and we
obiain Prob !X(r) > 751 = 0.000372. The probability that X exceeds 50
calls is 0.462. which is about what you might expect.

We can also derive the probabiliry dcnsi1y for arrival limes in Figures 21.21
and 21.22. Lc11 1be the lime al wh.ich the first event occurs. Then. the events [T 1 > tl
and [X (t) = 01 arc equivalent. and so

(4a)

Thus, the cumulative di.stribulion function of 7 1 is given by

Prob IT1 ~ t) =I- e->.i (5a)

and the density function is the derivative of Equation 5a,

r> 0 (6a}

Equation 5a says Lhal I.he time 10 the first occurrence in a Poisson process is
governed by an exponential probability density. The expectation value of T1 is

(7a)

What is the probability law for the occurrence of the mlh event in a Poisson
process? The events rr,,, > t l and IX (t) < Ill - 11 are equivalent, aad so we have

o.,l .
=L
m-1
Prob {T,,, > rf = Proh IX(t) < m - 11 --r:-,.1
k=O kl
or
m-1 (1-.1)~ .
Prob {T,11 ~ r} =I - L -_,-e-
k=O k.
11
· (8a)

Therefore. the density function of r,,, is the derivative of EquuLion 8a and gives
(Problem 3)

t>O (9a)

Equation 9a is called the Erlang de11si1yfimc1io11. after the Danish mathematician


A. K. Erlang, who so succcssfully applied stochaslic prcx·c,.-,cs 10 the anaJysi s of
telephone trallic. The cx.pcctation value of T,,, is given by

ElT,ul--100o
-i.m
- 1"'
- e -i.l dt -- -Ill
(m - I)! 'J..

which is the expected time lO the 111th call.


1066

There is one other property of Poisson processes that we should discuss. Let's
consider lhe t·ime intervals between successive arrival Limes. Let Zn = T, 1 - Tn- I
be the nlh interanival rime. (See Figure 21.22.) The evenL-; [Z 1 > tland [X (I)= 01
are equivalcn1. so

Prob (2 1 > tJ = Prob {X(l) =OJ= e-)J


and
Figure 21.22 ( 10a)
The arrival limes T 1. T1 . ... and the
inrcrnrrival rimes Z 1• 2 2•.•. of a Poi~son
proces.s.
and the probability densi1y of Z 1(/) is

( 11 a)

Now

If you look al Figure 21.22, you can see Lhal Prob 12 2 > t I Z 1 = r) =
=
Prob IX (r + r) - X(r) 0). But Prob (X (1 + r) - X(r) = OJ is Lhe probabiJil-y
that no event occurs in Lhc interval ( r. r + r). which according to Equal.ion 4a is

Prob IX(!+ r) - X(r) = 0) = Prob (7 1 > t] = e_,._,


Substituling these resul1.,; into Equation 12a gives

( 13a)

or

( 14a)

Thus both Z 1 and 2 2 are exponentially distributed random variables. Repealing


the 5a.me argument shows that the interarrival 1imes of a Poisson process are
independent, identically distributed random variables.

Example 2:
Use rhe fac1 1ha1

and that the Zj are independent. exponentially disLributed random variables


10 derive the Erlang density function . Equation 9a.

SOLUTION: Because r ~ O. we can u.~ a La.place IT.lnsform as the


characte.ristic function of Zi and write

al
1068 Chap!l'f "1.1 I Pruh:ibili1y Theory and S1och, ~lie Pro

The anival times are independeni and occur at random. so the probability of an
arrival occurring between t i and r1 + dt1 is simply the unifonn distribution di 1; T.
Thus.

= -k
T
ix -x
Jz j(1 - z) = -k
T
1:x,
-x
j(u) d11 (3b)

This la-;l line follows because f (u) is nonzero only for some small time inicrval.
and so the integral from O 10 T can be wriucn aq an integral over all values of r - t1.
Now. the probability that exactly k evcnt.s occur in the lime interval (0. T) is
given by a Poisson probability

. ( )._T ) k -!..T
P~I, ( I.) = -k!
-(! (4b)

and so
......
£[/(r)] = L E(h(l)Jpi.U) (5b)
k=O

Substitute Equation 3b in10 5b and use the fact that (k) = ,\ T 10 obtain

Ell (r)l = (l (r)) =A 1-: J(u) d11 = >-.q (6b)

This result is known as Campbe/J"s theorem. Note that (/ (r)) is independent oft
as it should be for a s1a1ionary process.
We can use the same approach to derive the characteristic function of / (1 ),
which is. given by
00

(/J(s) = £fe' 111 n.1 = L P1;(r)(e 1


•"1.U)) (7b)
~=0

But

= n
k
(ei.1/(1-1;)) = (i f (/-1) •)'-' (8b)
j=l

where , 1 is any arrival I ime. Now


1069

= I+ -I
T
1 00

-oc
(/·1/(u) - l)du

Let this. las1 line be denoted by I + a, so that Equations 7b and 8b become


(Problem 5)

(9b)

Example 3:
Use Equation 9b to derive the firs! few momenls of/ (t ),

SOLUTION: Firs! note that ip(O) = I. as it should. Differentiating once


with respect 10 s gives

-d ,p = icp(.~)).
d .1·
1~
-:io
f (t1)du

which give~ Equal ion 6b when:;= 0. Ano1hcr diffcren1ia1ion give~

or
x
'
~
n 1- = (fl(r) - (/)]-) =i.
l )
--:-:• .r-(,.1)d11 ( JOb!

If we substitute Equation 9b into


1070 Cha pier 21 / Prub,1hil iry l11(•ory and S10< ha,1 ic Proc

Lhcn we find that the probabili1y density function for I (t) is given by

Joe exp (-i Is+). 1 [e


00

p(/) = -I 1
. sflin - 1J.du } ds ( I lb)
2,r -00 -00

Equa1ion 11 b may look prelly hopeless at first sight. but we should suspecl that
p ( /) shou Id approach a normal d islri bu Lion because of the cen traJ I imi t Lheorem.
Realize that / (I) is the sum of a large number of random variables. Problem 6,
which is not difficult. ha.c; you show 1ha1 p(/) d I becomes

(12b)

where (/) is given by Equation 6b and a; in Example I. Thus, we see 1ha1 p( I) is


indeed a nonnal distribution.
We can derive an expression for the correlation function using the same
method 1hal we used to calcula1c (/) and ,p(s). The final rcsull is (Problem 7)

Note that R depends only upon r. as we expecl for a stationary process. Also note
that the second renn on the righr side of Equal.ion 13b is equal to (/) 2 . This is due
to the fact thal R ( r) __., ( /) 2 a." r --+ oo. If we lei R0 ( r) be 1he correlation function
of I (t) - (/), then R0 (r) ~ 0 as r --,. oo, and we have

Ro(r) = >.. l: /(u)/(11 + r)du ( 14b)

Note Lhar if we le! r = 0 in Equation 14b, we get Equation I Ob for a J.

Example 4:
Determine R0 (r) if

!L - u/r~

I
II 2:_ Q
j(u)= TcQ
u<O

SOLUTION: Ul-ing Equation 14b gives


R1.deren es 1073

Refer nee

H. Ommcr. 1973. Elcmenls of Probability Tlu:ory. 2nd. ed .. Kric£a


W.A. Gardner. 1990. /111rod11crio11 ro Ra11dom PmC1.'..1·. e · wilh Applications 10 Sisnals and
Sy.tte11u. Macmillan
R. V. Hogg and A.T. Cr.iig. 1994. lnrmducrion IO Matl1ema1ieal Statistics. 6th ed .. Prentice-
Hall
H. Hsu. 1997. Prohahility. Random Variable.'i, and Random Procc.t.w.r. Schaum·~ Outline
Series. McGraw-Hill
S. LipschuLZ and M. Lipson. 2000. Prvbahility. Schaum's Out tinc Serie.s. McGmw-Hill
W. Mendenhall, R.L. Schaffer. and D.D. Wackerly. 200f. Matlu·matica/ Statistics with
Application.1·. 61h ed .. Duxbury
M. Pugano and K. Gauvreau. 1992. Principles of B.ios1mis1ics. Duxbury
A. P1Jpou lis. 2001, P1-ol>ability. Random Vnriables, and Storlw.wic Processes. 4rh ed ..
McGraw-Hill
S.M. Ros.-.. 2001. A First C11111·_1e in Prohahility, 61h ed .. Prentice-Hall
J.A. Shohat and J.D. Tamarkin, 1963. n,e Prohfem ofMomcmx. American Malhernarical
Sociery
M. Spiegel. J. Schiller, und R.A. Srini,a),an. 2000. Probobiliry and Statistics. 2nd ed ..
Sch;ium 's OutJjne Series. McGraw-Hill
J. V. . peosky, 1937, llltmd11c1iun tu Mmlrema1irnl Probabihy. McGraw-Hill
GENERAL:
John D. McGervey, 1986. Probabilities in /:,1·r1"!,·da_r Ufe, Ivy Books
Warren Wea,·er. 1963. lody Lurk. Doubleday
CHAPTER 22
Mathematical Statistics

Suppose we have a large group of objects or individuals and we wish 10 know the
average mass of the objects or the averJge age of the individuals or some 01her
parameter of the pop11/ativn (1hc entire group). We could. of course. measure and
record 1hc appropriate propcny for every member of the population, bur this is
usually impractical for reasons of 1ime. expense. effort, or whatever. Instead. we
examine a small portion of the population (called a sample) and then infer 1he
properties of the entire population from this sample. This process is called statis-
tical i11ference and is one of the primary undertakings of statistics. For example.
suppose that we know on theoreticaJ grounds or through extensive experience that
the properties of a certain population can be described by a normal distribution.
but that we do not know either the mean or the variance. We could select a sample
and then calculate its mean and variance. hut how do these values reflect the mean
and variance of the entire population? This problem is called esrimotinn ofparam-
eters. and in Section I we shall learn how to choose functions of the sample data
that give good estimates of the population parameters. Of course, these will not be
exact, but in Section 3 we shall also learn how to assign ranges for these s1a1istical
estimates such 1ha1 we can say that there is a cenain probability (say 99%) !hat
these estimates will lie within a cenain distance from the true population value. In
other words. we shall detem1ine confidence intervals for these estimates.
rn the final sec1ion we shall discuss regressim, annlysis, where we use curve-
fining procedures in order to es1 imate one variable in tenns of another. The standard
curve-fitting procedure is a least-squares procedure. originally due to Gauss but
developed into a powerful predictive tool by statisticians. We"ll not only determine
the optimum straight line through .i set of data. but we'll also derive confidence
intervals for 1his line. We conclude the chapter with a brief discussion of the theory
of errors of measurement

22.1 Estimation of Parameters

Suppo~e we have a population that is described by a probability distribution


f(x: 0). where 0 is a parameter. For example, 0 might he the probability p in
a hinomial distribution or the mean ). in a Poisson distribution. We choose a 1075
1076

random sample x 1• x 2 •.•.• xn of the population and use these data to es1ima1e
the parameter 0. More precisely. we use a function of lhese data:

( I)

Such a function is called a sample sIaris1ic. or simply a sIa1isric. Each population


parameter will have an associated sta1jsLic ,that, we use to es1imate the parameter.
Because these estimates are singk values, 1hcy are calted poi111 eMim01es.
It is cenainly natural 10 use the sample mean (de no1ed by x).

x = -"1 + X2 + , . . + x" (2)


"
as an estimate of 1he population mean µ. Jf we denote estimates by a carat. 1hen
we write
µ. = X1 + X1 + .. , + Xn (3)
n

Similarly, we might use the sample variance,

(4)
n

as an estimate for lhe popula1ion variance a 2 . h turns out rhat we obLain bener
estimates of a 2 by having 11 - I rather than II in 1he denominator of Equation 4. To
see why lhis is so. we have to consider the nature of estimates a li11le more fully.
When we sample from a population. we obtain n data points, x 1• x 2 • .. , • xn. IF
the sample data are 1aken randomly, Lhen each observa1ion. xj, may he considered
to be 1he value of a corresponding random variable Xj. As we learned in the
previous chapter, ii is convenient to work wi1h independent random variables
because their joint probability distribution factors in10 a product of individual
probability distributions. For then random variables IX i: j = I. 2.... , 11} 10 be
independent. we must not disrurb the populatjon in any way when we sample it. lf
the population is very large. the sampling has a negligible effect on the population.
but this is not the case if the population is "small enough.'' Consequently. we shall
always assume that we sample wi1h replacement. There are sta1jslical method.'> for
dealtng wilh small populations or for sampling without replacement. but we shall
nol consider Lhem here.
We may regard 0 = 0(x 1, x:i, .... .\'n) as a single observation of Lhc random
variable

(5)

This random variable is called an eslimarvr of the population parameter 0. For


example. Lhe sample mean

(6)
n
22. I Es1im;ition of Parc:imeters 1077

is a random variable 1ha1 is an esLima1or of rhe population mean. The expectation


value of Mis

~ I
E(MJ= - £(X 1 + X 2 + · · · + X,,J
II

= EIXi] + EIX2] + · · · + EIXnl (7.l


II

Because E[X ij is the ex.pecra1:ion value for the population, £[ X JI=µ and

~ n11
£[MJ= - =J,t (8)
n

Thus. 1hc expectation value of kf isµ. the population mean. Generally, if

£[0)=0 (9)

the estimator{~ is said to be unbiased. Equal.ion 8 says that the sample mean is an
u nbi ascd est.i mator of /'- .
The following Example invt:!stigarcs whether or not the random variable cor-
responding to Equation 4 is an unbiased estimator of the population variance.

Example 1:
The random variable expression corresponding 10 Equation 4 is

-,
s- = (X.1 - X).2 + (X -, - X) 2 + · · · + (X n - X) 2 = -I L" (X · - -Xtl
I/ II J
j=I

where

-
X = Xi+X,+
- -·- +X II
n

Show 1h01 S2 is not an unbiased c.stimator of the population \•arianc~ r1 2.


SOLUTION: According 10 Equa1ion 9, we want 10 show 1ha1 ElS~] # a 1 .

( 10)
1078 Ch,1ptt·r 22 / M,1ttwm,1t1t .11 Suli li es

Lei's look al cuch of 1hese terms in turn. The first 1erm is just

I
-
L El(X--µt]=--=<r
n

1
1 ,,,,,-
'
?
(II)
n j=I
n

Use the relation L Xi = ,, X in the second 1em1 10 write


"
L(Xi - µ) =nX -11µ =n(X- µ)
j=I .

so 1ha1 the ,ccond term becomes -2£[(X - µ) 2 J. The third lcrm con1ributcs
£f(X - µ)~]. so thal

Now lei's consider the t.trm E[(X - 11) 1 1. The Xi arc independent rnndorn
variables. so

£[(x°-µ)'[-£ [ ( X 1 +x,: . ·+ X,. -µ ) ' ]

I
= ----:;- £ [ ( X) - Jl + X2 - µ + ·'· Xn - J.l ):! ]
11-

The second 1cm, vanishes because Lhc Xj arc indepcndcn1 random variublL"'l-i,
so Equation 11 tells us that
,
- ., n-
El(X - µt]:..:: - ( 12)
II

Therefore, Equation IO becomes

c-21 - I 2
£1 ., =a 2 - (.12
- = -II - a
II n

Thus. we ~cc that £ [ 521,:/:- r, 2. and so 52 is not an unbiased esrimator of u 2.


Nole. however. that

is an unbia~cd estimator. The ob~rvation of Sf is

,
x- -· - -
I Ln (Xj -:r)-
- "> ( 13)
·' n - I
J=I
22 .1 Problem
I. C onsider a popula1]on consi ·1iug of 1he four numbe~ 1. 5. 8. and 10. each of equal probabiliry. Now consider
all the possible samples of size two that can be 1akcn (wilh replaceme.nt) from this population. Verify Equa1ion 7
by showing that 1he mean of all !he pos.,;;ihle sample mean~ h equal to the mean of the population. Also, verify
Equation 12 by showing 1hnt the va.ria11cc of 1nc sample means is t'.qual 10 the variance of lhc popula1ion divided
by the sample size ,, .

2. Show lhat E.qualions 14 a.re equivaleJi\t 10 the statemenl Lim Prob {10n -
,1
1

~
01 < E) = l. Hint : Use Chebyshev·s
inequality.
3. Dcrcnninc the nflaximum .likelihood estimate of,\., in ,the exponential di!-tribut.ion p(.r) = ;..e-:u. x ~ 0.
4. Determine the maximum likelihood es1imatc of p ln a bi11omial distribut.ion.

5. TI1e dis1ribu1ion xe 11 n(' dx /a . x 2: 0. ex > 0, is ,called a Rayleigh disrrib111inn (Figure 22 . 1). Firsl show
!hat a Rayld gb disiribu Lion is normaJiz.cd. Then show 1.ha1 1he mWt.imum likelihood cs1ima1or of r, is
a= 01211> I>J-
j

0.5

figure 22.1
The Rayleigh disrrribution x e- xlt ia Ja .
x ::'.: O. ci > 0. for scvi:mJ values of a. 5 X

6. Let 01 and~ be two independently de1ermi ned -es1ima1es of some pop1.1lation paramc1er O. Show that any linl!M
=
combination of 01 and {h B c 101 + c/lz wiLhi c 1 + c 2 = t is an, unbiased estim.ue if 01 and G-z are unbiased .
7. Ler 0 be the maximum likelihood estimate of 0 and let g(0) be a monotonically incre11.,;ing function of 8 . Show
that g(0) is the maximum likelihood eslima1c of g(0 ).
8. The fourth central moment of a nonnal distribu1·ion is Ja"'. Use lhe result of the previous problem to find the
maxim um Iikelihood estimate of 3 a 4 .

9. Show that a:i In l/rlp 2 < 0 at p = p for the likelihood function is given in Ex.ample J.
IO. Show 1hu1 a2 ln l/Ja 2 < 0 a1 a = &. for a Rayleigh distribu1ion. (Sec Problem 5.)
I I. Find the maximum likelihood e$1ima1e of 1he mean of a normal dislribution if u 2 is known . Show that
a2 In L/iJJt 2 < 0 atµ= ii.
12. Find the maximum likelihood estimare of the variance of a nom1al distribution if Jt is known. Show that
o2 In L/aa 2 aa 2 < 0 at a 2 2•=a
13. Verify rhe ma1rix elements given in E:<ample 4.
14. Verify Equations 26.

IS. Show 1h::i1 L(µ. a 2 ) for a normal dis1ribu1ion is n maximum at iL~ maximum likelihood cs1imates.

r,
22 .2 Thr e Key O i tri bution · U I in 51 a1is1i al Tc-5ts 1085

22.2 Three Key Di tribution U ed in Stati tical Tests


1n the previous section. we learned how 10 detennine point estimates of population
parameters. In the next section, we·11 learn how to assess a point estimate, in
the sense of providing an interval where 1here is a given probability (say 0.95
or 0.99) that the population parameter will lie within that interval. Such imervals
are called confidence intervals. but before we can discuss confidence intervals,
we must discuss three distributions that are used extensively in the determination
of confidence intervals. These three distributions are 1he normal dis1ribution. the
chi-square distribution. and the /-distribution.

A. THE NORMAL DISTRIBUTION

The nom1al distribution occurs in a wide variety of statistical applications. and


we'll use it numerically in the next sec1ion when we calculate confidence intervals
or the mean of a normal dis1ribut ion when the variance is known. We have discussed
1hc normal. or Gaussian. distribution throughout Chapter 21. but we shall present
its relevan1 propenies here for completeness. The normal distribution with me.an
µ and variance a 2 is given by

-OO<X<OO (I)

lt'seasytoshow that p(x) is normaJizedand Lhat E[XJ:::::. 11 and E[(X - JL) 2J = a 2 .


The effective width of a normal distribution is governed by the value of a. Small
values of rr produce narrow distributions. and in fact. p(x) becomes a Dirac delta (]" 2<T 3u X
function. iS(x - µ.), as n - 0. Figure 22.2 shows the area under a nom,al curve
that lies wilhin Jt ± (J. µ ± 2cr. andµ ± 3a. The percentage areas corresponding Figure 22.2
The area of a nl>rmal dis1ribu1ion bctwe!Cn
to 1hesc intervals arc 68.26~' . 95 .44 o/ . and 99.74%, respectively (Problem l ). µ - 11u andµ + 11a is 0.6826. 0.9544. and
In theoretical discussions, it is convenient 10 express Equation I in 1crms of a 0.9974 for n =I. 2, and 3, respecrively.
slandardized random variable, with zero mean and unit variance. If we let

(2)

then Equation I become..,;

(3)

Equation 3 represents what is called a sumdardized nonnal disrriburion. The


variable Z in Equation 2 is called the srandardized 110,mal \'Oriable.
A more general version of Equations 2 and 3 says that if the random variable
X is nonnally distributed with mean 11.( and variru1ce a}. then the random variable

(4)

is normally distributed with mean µ._1• = c1µ.x + c2 and variance a_;.= c~o; (Prob-
lem 2).
1088 Ch.,1p1er 22 / M:ithcm.itic.:il S1.:i1,~1,r~

and Lhe integral of p(x) from Oto .x gives the (cumula1ive) distribu1ion function
of x2:

P( X) = - --
2
I - 1.r It
In .,., ,
--. -I!
"''d
·- II ( 14)
211 1 r(n/2j o

The posilive integer n in Equations 13 and 14 is called the number of degrees of


5 X f reedr>m of 1he x 2 distribution.
Figure 22 .4 shows p(x) plo11ed agai nsr x for scveraJ values of 11. Nore I.hat p(x)
Figure 22.4 decreases monotonically for n = I and 2. and ha." a maximum value at x = n - 2
The chi-square distriibution for 11 = I. 3.
and 5 dcg:reei- of freedom. for 11 > 2 (Problem 14 ).

bample 2:
Detcrrnine rhe mC.!ln and variance of the chi-square distribution.

SOLUTION : We ' ll first sbow rhat p(x) given by Equation 13 i~


nomlaJilCd.

I
= ---
r1,,/2J o
loo -i1;" -ud
u hr- -. ·e u =l
Then

Loo x"/ 2:e-


E[XI = µ =
1
o
00
xp(x) dx = , I
2 11 H r tn./2) o
12t1x

2
=--
1· Xl
11
11 / :? '- ud
t' 11=
2r((II + 2)/2) =n
r(n/2) o r(n/2)

and

£rx 1 l = 4
--
1:-;;i u (o +l 12e _,,d11 = -
4f ((11 + 4)/2) ., ...
- - - - =11· + .t.n
f(n/2) o f(11/2)

o2 = E[X 2] - £[Xf =2n

We might expec! from Lhe definition of x2 (Equa1ion 11) and the central Umil
theorem that the x2 distribution becomes asympto1ically nonnal for large n. Jn
facL we have

P(x)~F
X--II) ( 15)
(-
,.ff;,
22.J onfi<Jt·11 ·" lnr ·n ,1l,s
1
1097

Table 22.4
Some values O'f a that s.alisfy
F(a) =(I+ q)/2 for several values
of the confidence le\le] 1J for a
1-distribution.

11\11 0.90 0.95 0.99 0.999


5 2.02 2.57 4.0J 6.87
6 1.94 2.45 3.71 5.96
8 1.86 2.31 3.36 5.04
9 1.8'.l 2.26 ).25 4.78
10 1.81 2.23 3.17 4.59
14 1.76 2.15 2.98 4.14
15 1.75 2.13 2.95 4.07
19 1.73 2.09 2.86 3.88
20 l.73 2.09 2.85 J.85
25 1.71 2.06 2.79 3.73
50 1.68 2.01 2.68 3.50
100 1.66 1.98 2.63 3.39
co 1.65 l.96 2.58 3.30

C. CONFIDENCE INURVALS FOR THE VARIANCE OF A NORMAL DISTRIBUTION


We learned in Section 2 thal if X 1• X2 • ...• X,, are independent nom1aJly dis-
tributed random variables with zero mean and unit variance. thc.n

{3)

is governed by a chi-square distribution with II degrees of freedom. It turns our


that if X 1• X 2 •••.• X 11 are independent normally distributed random variables
with meanµ and variance o- 2 . then the random variable

I II

Y=-'°'(X--Xi
n2 L..., J
(4)
)=I

is governed by a chi-square di ..::trihution with 11 - I degree~ of freedom. The rcnson


Lhat Lhe dis1ribution has n - I degrees or freedom instead of 11. as for Equation 3.
is that the II tem1s in Equal.ion 4 are r,or independent because X is given by

The proof that Y is govem~d by a chi-square distribution wi1h n - I degrees


of freedom is a linle lengthy and will not be given here. Never1heless, the fact
1098 Chaprer 22 / ,\.lathcma1ical Stati stic.

Table 22.5
A srep--wise procedure for determining the confidence intervals for 1hc variance
of a nomial disrribu1ion.

II

S1ep I: Calculate the sample mean :X and L<x J - x/.


J=I
S1ep 2: Choose a confidence level. iJ.
Step 3: Use a 1able of the chi-square distribution with n - I degrees of
freedom (see also Table 22.6) to determine values of o I and o 2 such that
fh

Pmb la 1 :5 .r :S n:d =
J 01
p(.r) d.x = TJ

p In particular. we choose u I and a1 such that the area 10 the lefr of a 1


is equal 10 the arc.a to the right of a 2 in Figure 22.10. In rcrms of rhe
cumulative disLribu1ion function P(x). o 1 and a2 are chosen such that

I + '1
and P(a-,)
-
= --2
Step 4: Calculat.e c: 1 = (11 - \)_(~ /a~ and c2 = (11 - \)J 2 /a 1. The confidence
X
in1erval for the variance is
Figure 22.10
TI1e two vnh1cl- a 1 end a 2 in lhc lc~t gi~·cn
in Table 22.5 arc chosen • uch 1ha1 the area
10 rhc left of a I is equal to the are~1 10 the
(The theoretical justification for this procedure is given in the referrnces at
righr of u 2 . the end of the chapter.)

Lhat Equation 3 yields a statistic for the variance makes it plausible that the
dc1em1ina1ion of confidence intervals for the variance involves the chi-square
distribu1ion. The step-wise procedure for doing so is described in Table 22.5.

Example 4:
Detcnninc a 95% confidence in1crval for rhe variance: of u normal distribution
for the following data.

41.60 37.74 36.82 .:12.36 :n.26 39.26


42.31 46.35 41.88 39.78 37.82 42.80
40.70 38.99 41.33 40.69 32.09 40.58
37.58 34.66 43.18

SOLUTION:

Step L x = 39.61 and s 2 = 11.96


Step 2: 'I = 0.95
22.3 Confidence !nt.erv.:ils 1099

Table 22.6
A few values of a 1 and a 2 that satisfy P(a 1) = (I - ri)/2
and P(o 2) = (I + 'f/)/2 for several values of 1hc
confidence level ,, for a chi-square distribution.

11 \ r/ 0.90 0.95 0.99


9 3. 33. 16.9 2.70, 19.0 1.73, 23.6
10 3.94. 18.3 3.25. 20.5 2.16. 25.2
14 6.57. 23.7 5.63. 26.1 4.07. 11.3
15 7.26. 25.0 6.26. 27.5 4-60. 32.8
19 10.1. 30.1 8.91. 32.9 6.84. 38.6
20 10.9. 34.1 9.59, 34.2 7.43, 40.0
30 18.5. 43.8 16.8. 47.0 13.8. 51.7
50 34.8. 67.5 32.4. 71.4 28.0. 79.5
100 77.9.124.3 74.2, 129.6 67.3. 140.2

= 9.56
Step 3: These are 21 data poinls. so using Table 22.6. we find that o 1
and a2 = 34.2 for 11 = 20.
Step 4: c 1 = (20)(11.96)/34.2 = 7.00 and c~ = (20)( 11.96)/9.56 = 24.9.
Therefore.

Conf {7.00 ~ a 1 ~ 24.91

Problem 15 has you show that 1hc 99 confidence inrcrval is I 5.98, 32.2 I.
In all 1hc cases that we have discus cd in this sel:tion, we have assumed lhat
the distribution i.,; nonnal. Whal if the distribulion is not nonnal. or perhaps even
unknown? We ~aw above 1hal bolh the 1-dis1ribution and 1.he chi-square dis1ribu1ion
become asymp101ically normal as 1hc number of degrees of freedom become
large. This result is a consequence of the cen1ral limit 1hcorem. which says 1.hat
if X 1• X~ . .... X,, are i ndcpendent. identically dis1ributed random variables. 1hen
rhe dis1ribu1ion funct·ion of the random variable,

Y=X1 + X2 +·· · +Xn


I/

is asymplotically normal with meanµ and variance 11C1 2 as 11 - oo , Often a vaJue


11 ?, 30 is sufficienl to approximate lhe sample dis1ribu1 ion by a nomwl dislri bution.

The central limit theorem is the primary reason I.hat rhe nonnal dis1·ribu1ion is of
such importance in s1.atis1ics.

gl
1100 Ch.,pler 22 / M,1llll'mat 11 al ~t.1ti,1ics

0. CONFIDENCE INTERVALS OF THE PROBABILITY OF SUCCESS IN


REPEATED TRIALS

An experiment that has exactly two outcomes. often referred to as succcs:). and
failure. is caJled a trial. If we define a random variable Y Lhat is equaJ 10 I for a
=
success and equal lo O for a failure. then X Y1 + Y2 + · · · + Y11 is equal to the
number of success.es in 11 (independent) trials. We know from 1he previous chapter
that the probability density of X is the binomial distribution

.r = 0. I. . ... n (5)

where p + q = I. Recall that che mean and variance of 1he binomial dis1ribu1ion
are np and 11pq. respectively.
Suppose now we have data such as

0 0 I 0 0 0 0 0 I
0 0 0 0 0 I 0 0
0 () 0 0 0 0

for 30 repeated trials. The number of sucC:\.'~SL'S is I 0, wh.ich leads to I 0/30 = 0.33
as an estimate of the probabi.lity of a success. We would like to assign a confidence
interval to Lhis estimate of the parameter p in Equalion 5. This turns out to be
fairly easy if II is sufficienlly large. because the binomial distribution is well
approximated by a normal dislribuLion for large vaJues of n. Ln panicular. the
dislribution function of the random variable

z - _x_-_µ - _x_-_,_,r>_ (6)


- 17 - (11pq) 1f2

is approximately nonnal with zero mean and unit variance for large values of n .
We now choose a confidence limil 11 and U£e table:- of rhe normal dis1ribu1ion 10
find the value of a that satisfies the relation

I
Prob {-a< Z <a } = - --
- - (2 ir ) 1/ 2 •
£"
-r,
e- 11 212 du = 1J (7)

U-.ing the expression for Zin Equation 6. 1he inequality in Equation 7 becomes

X-11p
< [l (8)
(npq) 1/2 -

=
If we let p x / 11 (the ratio of the number of succes ..es to the number of trials)
be our cstima1e of p. then we have al the Lwo extremes of Inequality 8 thal

p- p = ±a
[p(I - II
p)]l/2
.. (9)

Squaring boLh sides yields a quadratic equation in p. whose 1wo solutions are
(Problem 18)
2.. . Confiden lnl rvals 1101

~ c/J· [p(I - p) a2
p+-±o - - - + -
]1/2
2n II 4112
P± = a-
, ( 10)
l+-
11

The corresponding confidence interval is

Conf {p_ ~ p S P+J ( 11)

For sufficiently large values or n, we can ncglecl the 1crrns in a 2/ n and a 2/ 11


2 in
Equalion I O and write the confidence inren·al as

Conr
I- [~(1 - -)]''2
p - " p
"
p , [~(l -p)]'/2]
;:s p ::S p + a _P_ _ _
n
( 12)

Example 5:
Suppose an um con1ains red balls and white balls. A random drawing (with
replacement) of 50 halls showed that 22 were red and 28 were while. Use
bo1h Equations I O and 11 to ca\culale confidence inierv::ils for I.he ac1uol
frnction of red balls in the um.

SOLUTION: A 95~~ confidence level means I.hat 1he area in the 1wo
ex1rcme wings of 1hc normal dis1ribution arc ~at:h 0.025. and so we find Lhar
a= 1.96 for

I + ,,
F(a) = - 2 - = 0 .975
Equation 12 wi1h jj = 22/ 50 and ,r = 50 gives
Conf /0 .30 ~ p ~ 0 .58)

Equa1ion IO gives

Conf !O.~ 1 ~ p S 0.581

Example 6:
How many balls should we draw from the um in Example 5 if we want a
confidi:11cc level of 95.t.r tJ1a1 p and p differ by no more than 0.050?

SOLUTION: We st.-U1 wi1h Equa1ion 11 :

~
• ( I - JJJ
, . ] 1/2
p - l' = ±a [ JJ 11
1104 Ch,1p1t•r 22 / Mathem,1:i< ,11 S1ai-is1ics

Table 22.7
The step-wise procedure for Lhe ch.i-~quarc goodne~s-of-fit te,t..

St.ep I: Divide the x axis into n intervaJs (nm nece~sarily of equal length).
11• 12 • ... , I.,. such 1ha1 the number of observed -.ample values o1 in
each interval is al Jea....,t 5.
S1ep 2: Using 1he hypothesized popul::nion dis1ribu1ion F(x). calcula1e the
expected number of sample values eJ = 11p j in each interval. wlwre 11
is 1.hc total numbl'r of sample values and p J is the probabili1y that x lies
in the imcrvaJ I;,
Step 3: Calculate lhe quantity

( I)

S1cp 4: Choose :i significance level a (say 0.010 or 0.050) and then ulte a
1able of the chi-square distribution (see Table 22.8) with n - I degrees
of freedom to detennine ea such that

Prob l x. 2 > er, l = a (2)

Step 5: If x 2 > c".we reject the a. sumplion that F(.x) is the population
distribu1-ion. If x 2 .::: c0 • we m.:ccpt ithe assumption. or at lca..;t we don't
reject it.
This s1cp-wi$e process is called the chi-square rest .for rhe J?Ootl11ess of.fir.

chi-square dislrihution that allows us to do this. We'll present this method as a


step-wise proccs~ (Table 22.7), as we did for the various tesrs in the preceding
section.
You can see from Equation I that x 2 wi II be small if the sample disuiburion
and the population distribution are similar. Tables of the chi-square distribution
give values of the cumulative distribur ion function. Prob [ x 2 < x I. so Equation 2
can be expressed .is

Prob /x 2 :5 Cu)= I- et

An ex1e11sion of the above procedure says lhat if the population dis1nbution


has r unknown parameters. then we use the corresponding maximum likelihood
estimates and then use 1.he chi-square dist.ribution with 11 - r - I degrees of free-
dom.
The chi-square test is best illustrated by Examples.

Example 1:
A coin is tos..,ed 50 1jmes nnd heads comes up 31 times. Is it a fair coin?
22.4 Goudne$s of F-i1 1105

Table 22.8
A sho11 table of the chi-square distribution for 11 degrees of
frcedom. The table lisL~ the values of c0 for Lhe corresponding
values of n and et.

et

,, 0.10 0.050 0.025 0.010 0.0050 0.0010


2.71 3.84 5.02 6.63 7.88 10.83
2 4.61 5.99 7.38 9.21 10.60 13.82
3 6.25 7.81 9.35 11.34 12.84 16.27
4 7.78 9.49 I 1.14 13.28 14.86 18.47
5 9.24 11.07 12.83 15.09 16.75 20.52
6 10.64 12.59 14.45 16.81 18.55 22.46
7 12.02 14.07 16.01 18.48 20.28 24.32
8 13.36 15 ..51 17.53 20.09 21.96 26.13
9 14.68 16.92 19.02 21.67 23.59 27.88
10 15.99 18.31 20.48 23.21 25.19 29.59

SOLUTION: We partition !he resulls imo two intervals. heads and 1ai\s.
=
In Equation I, o 1 == 31 and o 2 19. lihe expected values, based on the
!CSSumption thal the coi n is fair.. arc e 1 = e2 =25. The value of x 2 is

~ _ (3 1 - 25) 2 2
X - - -- +(19-- -25-
) _.., 88
-~.
25 25

Choosing a value of ex. = 0 ..050, and u ·ing one degree freedom. we find or
from Table 22.8 that ca = 3.84. Because x 1 < 3.84, we accept 1he assumption
1hn1 the coin is fair. or at least we do nor reject i1.

Example 2:
Suppose now we 1oss the coin in Example 1 50 more times and get 6::! hcuds
and 38 tails (same ratio of heads to tails as in Ex.ample I). What do you
conclude now about the coin?

SOLUTION: In this case

2
_ (62 - 50) (38 - 50) 2 _ 76
X2 - - - - - + - - - - - . 5
50 50

which is gre.a1er 1han 3.84. Thus. we reject the a.--sumption thai 1he coin i~
fair.
1106 Chap1.er 2~ I ~·lath •m a ti a l Stati ti s

Why do you suppose we concluded that lhe coin was fair in Example I and unfair
in Example 2. based on the same ratio of heads to tails?

Example 3:
Someone give~ you an algorithm to gener.ite r.indorn digits 0. I. 2, .... 9
and i1 gives lhe following result when you use it to generate 200 random
digits.

0 2 3 4 5 6 7 8
observed 27 9 14 19 12 18 24 20 30 27
frequency

What do you think about 1his algorithm?

SOLUTION: The expected number of occurences in each case is 20,


and so
, 2 , . ,
, (27 20)- + (9 - 20) + (14 - 20)- + •· • + (12 - 20)- _ .,,
;c= 20 - --

Choo~ing a \'Jlue of a= 0.010. we lind from Table 22.8 with 9 degrees of


freedom that er:, = 21.67, Because x 2 > 21.67. we should be suspicious of
Lhis algorithm.

Le! 's do an Example wi1h a continuous dis1ribution.

Example 4:
We suspect 1ha1 a population is described by a normal distribution with
µ = 35 and rr = 4 .0. The population i~ sampled I00 1i mes wi1h Ihe fol lowing
results. (See Figure 22.12 for a bar graph of lhe data.)

30 34 38 X
x interval 30 JQ < X ~ 32 )2 < X::; )4 34 < X :S J6
Figure 22.12
frequency 11 12 19 19
A bar gruph or 1hc J:itH u~ in Example 4.
x interval 36 < X ,::5 38 38 < X:::: 40 .r ::> 40
frequency 12 14 11

Use these data to assess the assumption thar the popular.ion is normally
distributed withµ= 35 and a = 4.0.

SOL u TIO N: Using t.ablcs of the normal <listribut ion. we lind that 1he
corresponding expec1e<l values for 1he intervals in the above I able are I0.56.
12.10. 17.47. 19.74. 17.47. 12.10. and 10.56. The corresponding value of
1108 Chap1e-r 22 / Mathcma1ical S1a1i~tics

8. A pair of dice are rolled 180 limes and a ro1al of seven comes up 40 rimes and a 101al of ten comes up 20 times.
Tes1 the a$sumption that 1hc dice arc fair at a 5.0% significance level.
9. Six1y families wilh four children were sampled and the number of families with x daughters turned out to be
Lhe following:

X (l 2 3 4

frequency 5 12 24 12 7

= 1/2.
Tes11he assumption that 1hese da1a satisfy a binomial distribu1ion with p
10. Tes11hc assumption that the following data obey a binomial distribution with n = 4 and p = 1/2:

X 0 2 4

frequency 19 34 34 11 2

11. Test 1he assumption tha1 1he data in the previous problem obey a binomial distribution with ri = 4? The mean
of Lhc data is 1.43.
12. The grades on 150 exams are distributed according to the following:

interval < 50 50--oO 60- 70 70-80 80-90 90-100


number 34 33 41 27 9 6

Do you think tha1 these grades confonn to a nonnal distribution wilh µ = 62 and a = 15?
13. Te.-.1 the ossump1ion t.hal 1he following dota obey a normal dislribution with 7..c:ro mean and unit variance:

interval < -1.5 (-1.5. -l.0) (- 1.0. -0.50) (-0.50. 0)

frequency 6 9 24 22
in1erval (0. 0.50) (0.50. 1.0) ( 1.0. 1.5) (> 1.5)

frequency 18 9 6 6

14. TesL lhe assumption that the following data obey a norm.aJ dislribution witb zero mean and unit variance:

interval <-2 (-2 , -1) (-1,0) (0,1) (1,2) (> 2)

frequency 14 IO 25 24 13 14

IS. Test the assumption that lhe following da1a obey a normal distribution with zero mean and unit variance:

in1erval <-2 (-2.-1) (-1.0) (0.1) (1.2) >2

frequency 18 37 83 98 46 I8

16. Tesl the assumption 1ha1 the da1a in lhc prcviolL'- problem conform to a norn1al distribution? Take x = 0.0863
and s 2 = 1.75.
1109

22.5 Regres io n and Correla tion p

A common and practical experiment in aJI sciences i.nvolves the repeated mea-
or two different physical quantities for the purpose of detennining a
surement
numerical relationship belween Lhem. We often seek a linear relationship. even
transforming variables 10 obtain a linear relationship. For example, Figure 22.13a
shows the vapor pressure P of water plotted against the celsius temperature. which
50 1()0
is hardly linear. Thennodynamics tells us. however. that we should obtain a linear
(a)
(or at least almost linear) plor if we plot In P agains1 the reciprocal of rhe kelvin
temperature. as shown in Figure 22.13b. Ln P
Thi,s sec1ion involves regression and correlation. In regression analysis. we
assume thnt Olile of the two variables. call it x. can be measured withour appreciable
error. and so we regard it as an ordinary variable. The other variable. Y. which
i$ a function of x. is subject to some imprecision or uncenainty and is regarded
as a rando111 variable. We call x the independent vnriable and Y the dependent
variable. In regression analysis, the mean value of Y is assumed to be a function 1/T
of.r. µ = 11(.x) . The curve y = µ.(x) i~ called 1hc regression cun·e ofYr.mx. In the (b)

linear ca~e. J.L(x) =a+ bx. which is called the reg re.\ rion line of r on x and the Figure 22.13
slope b is called the regression coefficient. [n correlation analysis. bOLh variables (n) Thi! vnror pressure of water pluth:J
are regarded as random variables. and we wish to detcnninc if they are. in fact, agoins1 1hc crlsius tcmpera1urc. (b) The
logariLhm of 1.hc V-JJ)l)r prcs.."Cure of \varc..-r
related.
ploned against the reciprocal of rhe kdvin
Table 22.9 lists some typical data for the resis1ivi1y p of nichrome ns a 1.empera1ure.
function of the celsius temper,Hure , . These data are ploued in Figure 22. 14. Both
Figure 22.14 and theory suggest that 1he relation between p and I is linear. We
want to draw a straigh1 line through the daiu in Figure 22.14 in the most objective

Table 22.9
Ty pica.I d.11..1 for the resis1ivi1y p of nichrome ~ a function of 1he celsius
1cmpera1urc. ll1e rel>istivity is expressed in uni.Ls, of meter• ohm x Io- 7.

t f° C 20 25 30 35 40 45 50

p 9.137 8.913 8.665 8.528 8.242 8.203 7.972

Figure 22.14
The re~i~ti viry p oi nichrome as a
function of 1he cclsiu.s temperature.
The l'C$is1.ivi1y is cxprc~:.cd in uniL~ of
met.crs · ohm~ x 10 7.
25 50
22.5 Regr€-ssion and Corre!a1ion 1111

and so ♦ p

J, = - 4 -449 = -0.03812
116.7

and

o = 8.523 + (0.03812)(35) = 9.858


Figure 22.16
Therefore, the regres...-.ion line is p = 9.858 - 0.03812 r (figure 22.16). The daL:1. in Table 22.9 aJong with the
rcgrc.ssioo line p = 9.858 - O.OJ812,.

Example 1:
Table 22.10 lis1s some data for 1hc molar heat capaci1y of carbon monoxide
as a function of the kelvin temperature. Detennine lhe regression line for
these datu.

SOLUTION: The necessary quantities from these data are

"f =800 Cp = 32.34


Jf = 18750 src = 133.78
and so b = sTc /s} =0.007135 and CJ = 26.64. ll1c least-squares straight
line is

Cp ::; 26.64 + 0.007135 T


T

Thc,I! data and rc~ults arc plotted in r


◄ igurc 22.17. Figure 22.17
The dnlu in Table 22.10 along wi1h the
=
regrc-~~ion line Cp 26.64 +0.007135 T.

Table 22.10
The molar heal capa~ity (joule, per mole per kelvin) of carbon monoxide as a
function of the kelvin 1cmpera1un.:.

T 600 650 700 750 800 850 900 950 1000


c,, 30,93 31.54 31.32 32.18 32.25 32.27 33.41 33.21 33.97

Al one lime 1hc calculations of a and b were fairly 1edious and older s1.atis1.ics
books offer various !-.hon cuts for handling lo1s of data. but nowadays even 1he mos1
modest CAS calculates nor only linear regression lines. but regression curves for
polynomials and sums of essentially arbitrary functions (Problem 6).
We said al the beginning of this section that we consider 1he Yj 10 be random
variables. We'll now assume funher 1ha1 each one is normally dislribu1ed with
1112 Ch,1ptt•r 22 I Ma!ht>m,11i< ,11 ~1.11i,ri1 ~

mean

µ(x)=a+fJx (7)
Yj
and variance a 2 , which is independent of .r (Figure 22 . 18). No1ice I.hat we use 1hc
notation a+ /fr in Equal.ion 7. Equation 7 is the rer;:rexsio11 line of the pop11lario11
I and fi is the regression coefficie,11 of the pop11/a1irm. The values of a and h depend
upon the panicular sample, but a and /J are population parameters. We'll now
show that a and b given by Equations 3 through 6 arc the maximum likelihood
estima1ors of er and /3, respectively.
Figure 22.18
An illus1rmion of the assumption that each Because we arc assuming lha1 the Yj are independem and normally dis1ribu1ed.
random variable Yj is nonnally di~rribur1.'d the likelihood funcLion is given by
with mean µ = a + /jx :md variance 2
<T •
which is iadcpendcn t Of X.
(8)

(Notice that maximizing L(a. {3. o- 2) is equivalent to minimizing S(a, b) in Equa-


tion 2.) Taking the logarithm of L(cx. f]. a 2 ) and then maximizing In L(a. {3. a 2)
with re.~pecr 10 a and fJ gives (Problem 7)

a= a= y- b.t (9)
n
L .t 1 _r 1 - n xy
~ = h = _j==_I_ _ _ __ ( 10)

as the maximum likelihood estimat0rs of the population regression parameters ex


and fJ.
We learned in Section I lhat maximum likelihood estimators are nonnally
distribu1ed for large values of 11. In that case. we can de1ennine confidence intervals
for the population regression parameters a and /J. As we did in Section 3 for 01her
types of confidence intervals. we give a step-by-step procedure for dctcnnining
confidence in1ervals for a and f!, in Table 22.11. (Sec the references at the end of
the chapter for the theoretical justification of this procedure.)

E,cample 2:
Use the data in Table 22. JO to determine 95% conlidc:ncc intervals for ex
andfl.

SOLUTION: Example I gives LI', r = 800. Cp = 32.34. s} = 18750.


s7 c = 133.78. 11 = 9. t1 = 26.64. and /J= 0.007135. Fur1hem10re. the data
in Table 22.10 y icld .f~ = 1.0299. We now fol low lhc s1cp-by-.51ep procedure
in Table 22.1 I.
2 2 .5 R 'i,l • , ion ,md Correlation 111 3

Table 22.11
A s1ep-wi.se procedure for determining the confidence intervals for the
popula1ion regression paramcien- a and /3.

Step I: Calculate I.he two quan1i1ics


, _, ... 2 ,
., 1(11 - l)s_; + n:cl(s_; - b s;)
a-=----------
" 11(n - 2)s_~

and

Step 2: Choose a confidence interval. '7 -


Step 3: Use a table of the standardized normal dis1Tibution. p(z.). (see also
Table 22.2) to determine a value of y such Lhat
)-'
Prob / - y ::: : ::: y l=
!-;,
JI:.) d: = 11

or. in 1crn1s of Ihe i.:umulative dis1ribution func.:tjon. choose y such that

I +'1
F(y) =- -
2

Step 4: Calculate you and yob. The confidence interval~ of et :.ind /3 are
given by

and

The justifit:alion for I.his procedure is given in I.he references al the end
of the chapter.

S1cp I:

[(8)( 18750) + (10)(800) 21[1.0299- (0.007135) 2 (18750)1


= (9)(7)( 18750)

= 0.4179

= 5.743 X )0- 7
1114 Cl1,1pln 22 / ,\-\athemalical Stalislics

or
<J0 = 0.6464 and
S1ep 2: r, = 0.95
Step 3: Using Table 22.2. we find that y = 1.960.
Step 4:

Conf {25.37 < a < 27.911

and

Conf {0.00565 < /3 < 0.008621

Up 10 Lhis poinL we have assumed that the independent variables (the x 1 ) are
detennined with little or no uncertainty, so that we may regard them a,; ordinary
variables. From here on. we'll treat both X and Y as random variables. We define
a correfmion coefficient of rhe sample by

.~.n-
r = -·- ( 11)
s_rs_1•

Since s .ry may be either positive ornegative and s .r > 0 and s Y > 0, r may be eilher
positive or negative. Problem 4 has you show that

(12)

r
and so we see that r 2 ::: I, or that - I .::: ~ I. Funhennore, if 2 = I, thenr =0 aJ
and all the sample pairs (x 1• y 1). (x2 • Y:!) ..... (xw y,,) wi II lie on a straight line.
The converse is also true: I.hat is, if all the sample pairs lie on a srraight line.
then r 2 = I (Problem 11). Suppose, on the other hand, thaL there is no relationship
between the x j and the Yj · Then lhe Lenns in the sum

,,
2:)xj - x)(yi - y)
j=I

will be equally likely to be positive and negaLive. and so s.r_v• and consequently r.
will be zero. In lhis case, Lhe x j and 1he YJ are said to be uncorrelored. This aJI
suggests that Lhe value of r is a measure of the linear correlario11 between the x is
and lhc Yj-'').

Example 3:
Delermine the value of r 2 for the dai.a given in Table 22. 10.

rr
1116

Table 22.12
A step-wise procedure for dc.rermining confidence intervals for the popula1ion
correl nIi on coe ffic ien 1.

Step I: Calcula1c 1he quan1i1y

I I +r
:. = -2 ln - -
1-r
Step 2: Choose a confidence level. 11 (for example. 95' f>, 99'1 .. . .)
Step 3: u~ing tables of the standardized normal dis1ribu1ion (sec also
Table 22.2). detern1ine a value of~ such that

F(~) = I+ ri
2
S1ep 4: Calculntc

Pt= tanh (z. - Jn~- 3) and P2 = tanh (:: + -~-)


~
The confidence interval for p is given by

Conf IP1 ~ /J ~ P1)

Example 4:
Delermine 95% confidence in1ervaJs for the popula1ion correlation coefficent
p using 1hc da1a in Table 22.10.

so LU TI o N: According to Example 3, r = (0.927) 112 = 0.963,


I l +r I 1.963
Step I: z. = - In - - = - In - - = 1.98
2 I -- r 2 0.037
S1ep 2: YJ = 0.950
Step 3: Using Table 22.2. we find tfa1t ~ = 1.960
S1ep 4: p 1 = L'.1.nh (1.98 - 1.960)
J6 = tanh( 1.18) = 0.827
Slep 5: fJ'2. = lanh ( 1.98 + l~O) = ta.nh(2.78) == 0.992
Therefore.

Conf {0.827 .'.'.: p ~ 0.992J


1117

Before we finish 1his section, we should discuss the imponam 1opic of error
propagation in measuremen1s. Let f (x, y) be some quantity whose value we can
dctennine by measuring x and y ~parately. For example, f (x. y) might be an
area of a rectangle. which we determine by measuring its width, x, and leng1h,
y, 10 gjve A == xy. Suppose now we mea-sure x and y. so 1hal we have the pairs
(x 1, y 1). (x 2 • y 2 ). .•.• (x,,. y,,). We can caJcu late the means and sample varianc~s
of x and y according to

I " I n

-r=-'°'r-
L·1 and T-=-'°'r-
. L-1 ( 15)
1l } = I ll j==l

and ( 16)

We l:an al sll calculate II values of f (x, y) from the II pairs (x j, ."j) according to
f,= j(x1. Yj>• from which we can cakula1e
., I
and J-
f
= -11 - I
LU·- n~
n
1
- ,
( 17)
j=I

Just as s_-r and s,.. are measures of the imprecision of the values of lhe x j'f. and y_;s,
we considers f to be a measure of the imprecision of the f (xi. y1).
Assuming, a, usual, that the imprecisions are small. we can expand Jj =
f (x 1• Y) about xJ = x and YJ = "f lo oblain

J-=f(x
J
r)=-f(x.v)+(iJJ)
J'. _, . . d.r (x--x)+(nf)
J i:) . (\'- .,· -T')+-··(18)
.
:...r .> :r.,
If we divide bolh sides of Equation 18 by II and Lhen ~um over j. we obt.ain

(I 9)

=
Thus. if .f (x. y) x y represcnrs 1hc area of 1hc rectangle. then A = x y.
We can also detcnn.ine x} in terms of the imprecisions of the (x 1. Y;) data
pairs. Subsritute Equa1ions I 8- and I 9 into :r} (Equntion 17) to get (Problem 21)

2
iJ.f )
( -ay ;,y s _\' +.t..
2 "' ( -aI ) ( -a_r ) .~ 2 (20)
ax :C.y c>y ;,y .X_\'

where

I Ln - -
S.l , = --I (x J· - X ) ( \' · - ,J ,V )
(21)
II - j=I

If the values of the xj and the y 1 arc independent. rhen sl_1. = 0 and Equation 20
22. R- r ion ,l!ld Crmd.1tiu:1 1119

5. Use the result of the previous problem to show that all the poinls of a sample lie on the corresponding regression
2 = ~21· 2
I inc if and onlv.} if s.ry '- t' y·

6. Derive forrnula-. for a, b, and c in 1hc regression curvt· y = u + bx + cx 1 for a parabolic fiL
7. Show that 1hc maximum likelihood estimates of a and fl in Equation 8 arc given by Equations 9 and 10.

8. Use Lhe following data 10 de1ennine lhe least-squares regression line:

X 0.00 0.25 0.50 0.75 1.00 l .25 I.SO 1.75

y -0.2765 0.0605 1.132 1.854 2.300 2.925 4.422 J.248

X 2.00 2.25 2.50 2. 75 3.00 3.25 3.50 3. 75 4.00

r 4.120 4.453 5 .631 5 .125 5.412 6.684 7.307 7. 726 8.400

9. Use the data in the previous problem to de1cnnine confidence intervals for a and {J. assuming thal rz and fJ arc
nonnal ly dis1ribu1e<l.

JO. Use lhe data in 1hc previous problem 10 determine the correlation cncflicienl r.
11. Show that 1he correlation coeflicicn1 r = ± I if the sample pairs lie on a straight Ii nc.
12. Show tha I if I he l wo random variables X and Y are join I Iy normally dis I ri bu led. then I hey :ire i ndcpendent if
1hey are uncorrelated.

13. Let 11' be I.he visco.,;ity of a polymer solution and ,, be the viscosity of the solvent. Then (1l ·- TJ)!rJ = '1.~p is
callcl.l l11e inirinsic viscosi1y of the solution. Theory says that 'ls,,/c should ,·ary linearly wilh L', where c is the
concentration of the polymer in the solution. Table 22.13 gives some typical data. Use these data 10 determine
the lc.as1-squan: . . rcgn:~sion line.

Table 22.1 J
The intrinsic v, ... ~osity of cellulose nitrnlc in alcohol. The units of concentration are
grams per litcr.

C 0.00 0. IO O.:W 0.30 0.40 0.50 0.60 0. 70 0.80 0. 90 1.00

9.04 10.2 11.7 13.6 14.8 16.4 17.7 19.9 20.5 22.3 23.9

14. Use the do1a in the previous problem to dclcrrnine a 99 n confidence intervnl for <Y nnd /1. assuming thnt Cl ::md
/3 are nomrnlly di."1ribu1ed.
15. Use the data in the previous two problems 10 determine a 99% confidence interval for the population correlation
coerficicm p.

16. In the photoelectric effect. a rnc1allic ..,u_rface is i1TI1di:ne<l with electromagnetic r.idiation and electrons are
ejected from the ~urface. The energit·s of 1.hc ejected electrons are detennined by measuring 'll1c potenl.iaJ :ll
which the photock-ctric current drop~ to zero. According to 1he theory of the photoc·lectric cff~t·l. tlhc .-,topping
=
polenlial <P~- is linearly related 10 1hc frequency of the radia1ion by </J,- a v + b. Use the da1a in Table 22.14 10
determine l11c lcas1-squarcs values of a and band the value of 1.he correlation coefficient r.
1120 Chaprer 2- / Malhcma1i al S1a1is1i s

Table 22.14
The stopping potemiaJ for the photoelectric effect on sodium metal. The
frequency vis given as 10- 1:i Hz and the stopping po1cn1ial </J., is given in vohs.

II 0.46 0.48 0.50 0.52 0.54 0.56 0.58 0.60

0.0834 0.315 0.182 0.340 0. 786 0.352 o. 731 0.430

I 7. Use lhe data in 1he previous problem lo determine a 90% confidence in1erval for a and /3. assuming that a and
fJ arc nomially distributed.
18. Use the data in the previous two problems ro determine a 90% confidence interval for 1he popular ion corrcla1ion
coefficient p.
19. Table 22.15 give.s the vapor pressure of water P (in rorr) and the corresponding temperature , (in °C).
TI1ennodyn:i.mics tells us that a plo1 of In P agains1 I/ T. where T is the kelvin temperarure T = 1 + 27:.l.15.
should approximate a straight line. Detcnnine lht! leas1-squares fit 10 1hese data.

Table 22.15
The vapor pressure of wa1er. P. in torr and lhe corresponding celsius
temperature. , .

0 5 10 l:'i 20 25 30
p 4.6 6.5 9.2 12.8 17.4 2J.8 31.6

J 35 40 45 50 55 60 65
p 42.2 55.3 71.9 92.5 118.0 149.4 187.5

70 75 80 85 90 95 I 00
p 233.7 289.1 355.1 433.6 525.8 633.9 760

20. Use the dai-a in 1hc previous problem 10 determine a 99.5% confidence inierval for et and /J. assuming that er
and /J are normally distributed.

21. Substitute Equal.ions I 8 and 19 into Equation 17 10 obtain Equarion 20.


22. Show 1ha1 S.ry defined by Equa1jon 21 satJsfie,4- the inequality Is I I ~ sxs .. /-li111: S1a.r1 with
JI

A(I) = I)(xJ - x) + I (.\'j - y)f :!. 0 and use the facl 1hnt Amin= s; - s;)s;. ~ 0.
j=I

23. Suppose we ha\'e 1wo indl!pendenl mca.',uremems on a quantity y and that Lhc rl!sulls arc reponed as y 1 ± a 1
and y 2 ± a 2 . How should we combine these results 10 find the average'? Surely y = ~-(y 1 + y 2 ) is unsuitable;
the value with the smaller vaJue of a should somehow be weighted more, Use lhe maximum likelihood method
W1)'1 + w-,y,.,
-
lo show 1ha1 y = - - where w} = 1/aj.,
W] + W2
Answers to Selected Problems

Chapter 1
SH,lON 1.1
r- \---1
I. Ca) -4 :._ .1 :__ 4. (hl -'-. - , - "-· le) x > 0. (dJ x -f:. I: 9. An ellipse. :....,
II"
+ ,,,_ = I.
(x - 2)! ( \' , I): . ,
where h 2 = a 2 - c 2: IO. --,--:-- - · -1- : . . . : I: II. A par.1.bola.: 15. (u) Pcnod1c.
,r b-
JT /2. (b) Periodic. rr. (c) Nn1 periodic: 16. (:l) Odd. (b) Neither. (cl facn. (dl ~either.
19. (x! + I): + I= .r" --- ::!.r~ --1- 2: 20. y°2-/ ( I + y 2 ): 22. (a) l < x < 2. (b) - _1 - x < 1/5;
1

23. A ).f;1i rcfL..e : 24. A sequcni.:I! of square WU\ es. 1

SF.nmN 1.2
4. {a\ 3. (b) 0. (cl I. (d) 213: S. (a) 2. ( Ii) 112. (cl -6. (d) I /2: 6. (a) 2.r. (b) -1 /x~. k) co:u.
(u) - ~in x: 7. ( ai) 0. (,b) 1/2: 8. I.(:!~: 9. li m = 1 and lim a:: -1: I.O. -1: 11. -l:
< ~II < ,(l
14.llr.l: 15.2: 1·6 .1: 17.21'.I: 18.oo: 19. : 20. - 1.

SEC"TION 1..1
2. No. but 1hc point x = I ~s a remoHibk diswa1in11ity: J. u~c the in1cm1cdi:11c V'Jluc
1hc11rt·m and the fru.:1 lb.it ft l l - 5 and /( - 1) = - 1: 4. Let f c r) - rn). r - x and apply
the inic.rmcdiarc vulue 1hcore'm: 5. {a) - 1. (bi I. (c) doc~ nol ~xi1-t: 6. .\' = 11tr whl·rc
11 = 0. ·U. ±2 .... ; 9. We :-ay 1lu1t i: • ) di\crgc_, mon: s1rongly 1h.1n /(.1) al 1he point = 2.
Funhcnnorc. 1he on •· idcd limil.~ of J< x) di l'il!'r in .-.ign. bur 1ho~c: of g(, J do not: I 0. Ye:.,: 312:
=
11. lei /(.r) x3( - 2 and use 1hc intcnnr.diale \'aluc theorem ondl 1J1c ftK I thal J (0) -2 and =
/ll)=l: 12. a = /J- - 1/2.

, : co~.r s in., ~ , . J - </ '


l.(a)( I -4x - '::!..r e-• . (b) - - - - ,- .1 ~)::!.x tan 2x + 2r sec .r: 2.(a)- -(.r - I) •.
X .r· 1

I -1' - 3 . _ . J . - e ·' sec .\ Ian x sec x 1


(b) - ~ ,. (c.:) u 1 In a. . la) ---..,- . (b) - - - - - - - = ~el· .r .
2 (x - - x 1) 1/ - I + . - .:.i ...c'-· ·" t.m .r
(c) .i''" 1 ( In .r cos x •l si: .r ): 5. No: 8..r = I is n m:u;iJ11um :rnd x = 2 11nd
-2 o.rc minima. There arc iafkc1irn1 poin ~ ar .t = {(I ± J 13): 9. x = 0: IO. Lcr
~ = x - i and
13. r (I lnx):
note Iha\ F(O = (~ +
16. lt 1110 ~ =25/ 2:
~r ( -n •

18.(2- 11-'. ± 2 116 ):


. l
is symmetric aboul
19.1:20.
= 0:
Jr \/3/ 4;
16. I 8.1 t' l' co. 1 1 2) sin' (lr l + 2) + 2.rc - t . i,r 1(l , ., 2) - X (! - in 'n.r 2):
18co,;;5,in2 5 sin·' s '>
27. - - - -
('
-- = 1.-10-8.:.1: 29 ..U.4082.

Su: 1111N 1,5


J. Use ~x = - 5 and ob1ain 4.933: 2. I':!,. V =- 3.8.:J cm~. or 0 .75 , : 3. i'(X =
(3x -1- 1)6.r + (i.\x) 2 .... 0 a.~ i.\.r-> 0: 4. f (X ) =
+ a)ti.x (x
-• 0 a~ Ll.r -• 0:
(X + a)( x + a ti.x)
5. ti 1· =:: 1•(10 . 10) - ·(10.00l = 1.91 , d1· = 1
(x ) 6. .1· = (h - l)u..r = (19.0J(O.IO) = 1.90:
6. A-1 = l~,.J5) 1I- - (4.00) 1/ : == 0 .086, . 1123
1124

il.f 0.35
dy = J (.rla.r = ----i-
I

12
= 2.r
2(4.00) •
I/' == 0.087S; 7, d cos sin 0)d0 = (-
(-0.4226)(0.20° ) = -0.0&5: 8. (3) dx /Sx I/!. (b) -dx /x!!,. (c) 2 1ru1 x d.x / co~~ x.
(d) -dx/4x 312: 9. (al 2.r,Jx / 3(.r~ -2)~· 1, lh) co~ .r 112d.r/2.r 1 -. (c) -el'O!< .r l-in.rt/,c
10. Yes: 11. (a) -2 1an y/.t. (b) (J,~ - 6r)/(6x - 2r). tcl .. b 2x /a'!.y.

SECTION 1.6
I. 0.707 I: 2. 2.71828; 4. (a) 0. (b) O. (c) O. (d> 0: 5. (a) I. (bt 1/2. (cl -1. (d} I; 6. M 0.
(b) I, ld V. (d} I: 9. I; IO. I: 11. "fhc ncxl to the la.-.1 displuyt."d limit is 1101 an indcrcrminuLC
fom1 .

SECTION Li
I. ln1egrn1e the fonnula for the dcnv:uive of a prodm;t; 2. (aJ -( l ' x )c•-.{, (b) sin x - .r cosx.
(c) (In .r)! /2.. (cl) .r Jn x - x; 3. (a) - cos 1{x / a) . (b) In urn(lrr /8) = 0.8814.
(c) rr /8 • 1/ 4. (d) a 2rr /4: 4. (a) In 11 x + J.r 2 - a 2 ]. (b) ½( vll + ~inh- 1 1). (c) sinh- 1 I.
(d) lllnh- 1( 1/2) = 0.5493; S. 411 : 6. '"'~ /2: 7. -/17 • ln(4 + -/17) = 4.6468:
"
21. 1/3.

S ECTION I.R
I..--r/2: 2. (2.Jra- 3/J)[ I - (). 3 - l)( r l lnT - I)]: 3. JT: 4. ;c/ 2: 5. 2: 6. The
mregral di\crgc-!.; 20. h docs n111 con\crge: 23. 0.40J6: 24. (l.3857; 25. In 2:
26. (.7 l, 2 /:.u \,• -fr /,la~; 27• (1'f 1n / 2.il )£". ~h.

Src:T1nN 1.9
3. Diffcn.•n1i;i1c ,1;," ;, a,d, = 1/Ci with re.~~-cl lo ct II tinu.·,: 5. Dificrcn1ia1c
Jt e ,.. , 'dr = (n/4a.) 1l l wi1h n.'1ipec1 10 a 11 time~: 9. Ye,. The second inrcgr.il is
1
unifonnly convergent; 13. cor- a.

Chapter 2
SUTION 2. 1
9. (a) Increasing. (b) Decreasing if 11 > 2. (c) ll)c,crcasing. (J) Decrea~ing: 12. (n) s,, --,. I,
(b) sn ....., I. (c) .s,. ....., e. (d) ,, -~; 13. 0 .

Sr:..C110N 2.2

I. 3/2: 2. 1/J; 4. 3/1 J; 6. I: 7. ~ (1 - -


1
- ): 8. 3/'2:
2" - I
10. - - -, la$
9. 1/4:
2 2n +I 2"
n-+ oo: 11. 1/2:l; 12. (a) lxl < 1/2. (bi O < x < 2. (c) - I < .r < 2. (d) .r < 0: 14. No.

SECTION 2.3
I. No: 4. About 25: 8. No. S a"" written is un in<leterminaJc fonn. oo -
2. Yes:
IO. (a) Com•erge>, (b) Cuoverg ·~. (L") Con\crg ·~. (d) Di\crg~ : 11. Yts: 12. No. ye :
13. Yes. se the fact thnt I -- 2 + · · •+ 11 =1!(11 -t 1)/2.

S1: noN2A
I. N 1 .V+I SN RN IR,,,I
I 0.0625 1.0000 - 0 .0529 0.0529
2 0.012'.\ 0.9375 0.0095 0.0095
1 0.0039 0.9498 -0.0028 0.002~
4 0.0016 0.9459 0.0011 0 .0011
5 0.0008 0.9475 --0.0005 0.000S
Anw,·1.• r.-i ro ~t'let:led Problem, 1125

l. N UN I S, RN IRN I
l 0.2500 1.0000 - 0.1775 0.1775
2 0.11 1 I tl7500 0.0125 0.0725
3 0.062.S 0.8611 -0.0386 0.0386
4 0.0400 0.79R6 0.0239 0.0239
5 0.027li 0.8386 -0.0161 0.0161
3. About 9; 4. About I4: 5. Ahou1 21: 9. D1\·cr1:c,: IO. Converges; 12. (a) Conditionally
convergent. (h) Condirional ly com·crgenr. (c) Absolutely convergent. (d) Absolurcly convergent.
(c) C11ndi1io11ally comc~..:c111. ff) Diverge,: IJ. Cnnditionally conve1·!!cn1: 14. Diverges
(Raabe'~ 1e~1): 15. Converge..~ absolutely for lxl < I: 16. Comcrgcs:
19. N l!,v;-1 SN R,..- IR,..I
0.5000 1.0000 -0.4167 0.4617
2 0.3333 0.5000 0.0833 0.0833
3 0.2500 0.8333 -0.2500 0.2500
4 0 0.5833 0 0
5 0
20. Converges (Raubc's lcsl).

SECTION 2.5
1. (a) All values of x, (h) 0 < .\" < 4 and conditionally lltx = 0. (c) - I :.__ x ~ I. (d) -2 < x •·: 0 and
condirionall~ al.,·= -2: 2. (a) x = 0 only, (b) -1 < .r 3: 3. All value~ of .r: 6. la) Take
M =-..: J/11\ (b) Take ,\,f-= l/ 11 2: 10. No, because 1.ht: tern,, of the scric~ ;ire con1inuuus and
.l _l _x5 .I 7
S(.1 1 is di~c.:nn1inuous; 12. I+ 2.r + 3x 2 + ···: 13. x - - +- - - + •• •: 14, Yes .
3 5 7
Us1: the M 1esl with M = l/11'.!: 15. S,,(.r) di,cs noc converge unifom1ly 10 S(x ).

SECTION 2,6

2. (n) - I ,::: x :S 1, (b\ - oo < .r -::: oc : 3. (a) -o.. -= .1 -= -.:. (b) 1 5 .r :S 3: 4. lal - I _ x < I,
.r J x~
(b) -2 -: .r < 0; S. x + - + - + · · 6. 0.7357: 7. Difforcntiatc me gcomclric series
3 5
twice; 9. 5(.r) = 5in x and CL, ·1 = co~ x: 10. Lc::1 x = ./)JJ and 1an- 1 fi/3 = rr /6: use 11
terms. 3.14159J.

SECTlON 2.7
2. Each number is the sum of the two above ii : ]. Us.e the binomi:il expun-
sion wi1h x = I; 6. Use ~qua1i,,n 3; 8. Chno~c 11 ~, I0. 2. 718282: 9. Mui-
(-x~)" (- ll"x2n+l
.
11ply rhe exp~nsion of
.
{1 - r
2
hy .l': .r Loc ---
n'
- Loc ----:11!
IO. u~c
,,.::.n ,, o
X X~ x-1 °''J +, ••;
Equation 7 with Cl= 1/2. I+ - - - - - :..;_ 14. Use Equu-
2 8 16 128
2
tion 7 with Cl= - 1/2 and "x~ = -x 2 ; 19. I - xlrZ + (X + ~) tJ + O(r 6):
2 6 2-1
4 \" 3 17 ~ " 5 , 11 1 I J
20.-\' + - ·- - - , : 011 ) : 21.(x-l)-! - (x - 1)·+-t.r-lJ·+-(x-1) -
3 15 2 6 4
I
-(.t - 1)·\ I
+ -(x - 1) b 01(:r - 1) 1 ); 22. - .1' ' - 3rr·(x
' - JT) - ( 3rr ;rJ)
- (x - _,. ,- ,
20 60 2

( Jn~) (J,,. 5,-r·') Ix - " ( + 5:r- rr )·~


2
I+ -
(x -
1
,r ) · - - -!. - ;r) - -J )
(x - + O((x - rr) 6 ).
2 2 2~ 2 8
1126 Answers ro Sd led 1-'rohli·m~

SECTION 2.CJ
I. ln1.cgr;,i1c by purt~. !cuing ·•11" = l/11 and "tfr .. :.... ,u,-"" du: 4. - f(r) sin x •(xl l·us .c
'X. ( -1)"(2n-1) 1
7. Write Equa1ion 14 as I + " , · : 9. F( lO) = O.09901U using
~ '.? 11 x• 11 2n- l(11 - I)!
I I
1/.,· - l/ s ·1 1/s\ IO. F(.r)
s 1(·1
=- - -
• • · To two 1cm1s, F(.1) - 0 .99666. The

"exact .. r1:·sul1 is uin - 1<0.10) = 0.996687.

Chapter 3
S Ff"TIO X J.I

1. 1r(~
a • 2
)=-'
a
_3./rr. 5l 2 4
. 2. J/8:
3
, (')
3. - r ~- _ = , _"
,. ,
4. 'rr/ $: 5.(-ll''r(n+ ll:

6. (2,i) 112: LJ. 8.8553 ; 15. (a) 10!! = JS40, (b) 7!! = 105: 1.7. ! r(·m+
If
I)· /_I I

SEMlUN 3.2
I. (..h e Equ. uion 2: 5. :;rr / 8: 6. R{-1/J. 4/~) = re 1/31~ / 10 r 2/3) == 0.52.999:
7• re 113ir( 513> -
----- - ,,
~ -➔ s-•· s. ..
., 1 ➔• ..., 9 , =rr .1 • 10• r( t/6 ) · " /011
,, / •3vJ",. ,. ~r·(_-,/~..)).·
f(2 }
11. 2 1/JR(2/ 3. 2/ )/ 2 = l.29J55: 12. '.1n/ 8.

S F ri nN '.'.3
r !I
2. fi crf (2 /2)/2 =0.88617: 6. erf ( ./2)
,,
= 0.95.5; 9. - -- .r - :..... + ·
( \' J

./ii 3 10
10. Six terms give 0.8-126: 15. Let u = 1/4. 2h = o. D.Ild c = 0 in Problem 11.

SF.CTIO!'I JA
I. Jn1cgrn1c £,,<x) by part.,. lcLling __,. be "u" :ind ,,- '-d::. be ..,Ji,":
11
4. f.:_ [ I - -- • · ·]: 5. Expand sin 11/11 :.ind in1L·~r;1tc ll'nn by term .
.l • II (X n )2

$f("Tlrl',. 3.5
6. 4 ✓ 1 £(1 / 2 = 7.MO: 7. l6£ ( .3/2 ) = 19 .38; 9. First let cos(/ = 2cos~{ 0/2) - I
and 1hcn let fJ / 2 = ;r /1 - t/>. K ( I/ v'2l - F ( 1/ 2. :r / 4 }: 10. 1.8.5-11 - 0.~260 = 1.028 I:
16. K 1.H-1)/ 4 = 0.4777: 17. K ( I/ ")/./'§ - 0.7-4:.2.
1130 AnswNs ro S •lee d P,roblen,-.

p:w.ilTlt:Lric equu1ions of the langcnt lin~ are ,r = "· _


\' = ,1 - 411, ;: = -2 + 611 and 'the equation
of rhe normal plane i:: .r - 4y + 6;: Jt, 16 0. =

Chapter 6
S cnoN 6. 1
I. -./W: 2. - I ~ (x - ,\ ·)/(x y ~ I. or x ::: 0. _v _ 0. (.r • •\') #a (0. 0): 3. No:
4. Ye-.,;,, yes ; 6. h bisects 1hc econd and fourth x ·-quadrnnls. and the ::. axis lie.s in
the plane: 7. (:r - 1}2/ a 2 (y + 1! 2/1> 2 + :'1:/c2 = I: I J.. Complc1e the square to get
(x - 1) 2 + y 2 ( + 2) 2 = I). II is a ,q,hc:.•re of r.idiul> v"l3 c.en1c:n.."<.I al (I. 0. -2): 12. II is
an elliptic c:.·ylinder who e a:<i is 1hc :. a.\:i!i: 13. CClmplett: the square and divide by 4 10 get
(x + 1) 2/4 + _\' 2 = 2(;;; - 2). the cquu1.ion of nn elliptic panibuloid': 16. J7: 17. Bl!C:iuse
:: = ±c when .t = y = 0.
S1 ( /JIIN 6.2
7. (a) 4. (b) (n~ + b!)/ah; 8. (a) Does rnot c:.xis1. (b) 0: 9. (a) t•irnit = ,r /2 11:- x-), O+ und
-JT / 2 a-u ➔ 0- . Does no1 cxis1. (b) Docs 001 cxis1: 10. ta) 0. (b) Does not exis1: 11. (a) You
ger rwo diffrrcnt rcsulL~ (0 and I). Tht:r.:fore. the limit docs not c.xi I. (b) You gcr - :S/3 in each
ca..;c. Thcrcfon:. il nwy exi~r. and. in focL does exisr: 12. Both ,;equential limits gi\e 1. But the-
limit is equal 10 (I - 111 2) 2 /(1 + m 1 ) 2 . The two scquimti:il limitS corre-spond tom= 0 :JJ1d oo:
13. No. Jr is not defined at (0. 0): t-&. Ye...:. The limit of J (x. y) i, 7.ero. (See Prohlem 4. ):
15. No. The limi1 of j(x. y) = Ii i1f .randy upproach zero :ilong rhe curve x = 11 and y = 1:
16. (al. (bl.

SF.rnor-.i 6.)
I. (a) f x = l'J'. f,. == l + xeY. fu = 0. J~ = .rt•-\ f n = / 1.•, = eY. (b) / J = v cos x + lx.
f ., == si n .r. I,, ~ 2 - y sin x . f .., = 0. J. = fn = ·co, .r·: 2. (n ) f = - y/(x 2 + ,r 2).
f,. = ·/(. 2 y2), .fu = 2:cy/( 2 y2·)2, f ,, ,::: - lxy/( x~ + y 2)2, f n = f,,. = (y2 - X ~)/
(.~ 2 + y•)z . (b) f x = - - H.' - ( : ·', • f v == _ _.yc - (.-2 - 1>_ f .u. = (4.t2 . 2JI! ; ,-: • ,-11 . f ,,y =

(4y1 - l)e- (.-" •1• In- = Jy.r - 4.r_v;-( .~ 1: 11. .x + 1y - z = -l; 12. 1~ - y _ ,;·. = 0:
13. 4x + 2 - • = 3: 14. Lcr y = mx andl ~cc that the limit depend" upon m; 15. Tu . cc 1h:i1
/(x.y)is di. onrinuou~al(0.0).lcl_\'=m.r: l6.x 7 = 1'/P.a= 1/T: 11. U= 3NknT/2:
18. cau J,JV >r = O for :in ideal' gas JJ1d g / V·1 for ;i ,·a.11 di:r Waal. gas.

S ECTION 6.4
2. 61 sin 31 2 ; 3. 4t 3ll - tJ)e- 1 ': 4. (sin 2 t - cos 2 I) e- ,~,.., ,,r,, cos 21 e -hin 21 >12 : =-
1
5.. 2t 2,-1e' 3t 2e1\ 6. x e- J !.in(.u- ) - e- .r c: sf.rt•-·1 ) - e - Z.r j n(.re -·r);
7.2r 1- . inOcol, 0) ; 8.1; 9.(t e' cos )e' »oJ . I0.2(e - l); 16.U - T S PV=
G. the Gibbs energy: t7. A= U - TS. ,t he Helmholtz cncrg, .: 18. V = L j n 1V 1 where
V; = (il V /i,ln ;h. 1, .
SECTION 6,

I. ta 1 = e• tu+ c11, y t:i.y + · · ·. F--:_ = ~.!.l, ,•·


I \'
6x + l,r"t'
' ,, / "
) .:. t:i.y - I sin
;:;x . y ~y + · · · N01
unit1uc; 3. (;1) df = 2.r i-in y dx -1- x~ c:o~ y dy, (ib) ,/,~ --' + (,!~ 1 tJ) ~ / v:
Ju 1e ~ d11~
4. (a) tlf = (lxv + -\' 2 ~2) d:t + (.r 2 + 2.n. d\·.. 2.r- d · . (b) df = ..1.:::_
~ l + -2
,Jx + ~ d r+
I + ;:2 .
R
n·(I - - ~)
.· ; ·, d::.: S. dP == - - d T + ( -211 - RT
,) dV; 1. a:./ox =-2/3 and
(l +z )· V-h V· (V - b)·.
. ., 2z :. 6:.~ dx - I6\•tt/::
or./<1\'
·
= -1/3 at <I. 1, I): 8. (it) d.:: = - - ,Ix -
_
- d\', (b) d z =
5.:: + 12r:: 8v-
1· ·
-1 • ~

9. 5-1.5, 1.625<i!I; 10. :. u I x 1o: 1 N • m- 1: Ii Y ·~- II is the differential of


;rr~h. N{); U. No. Yc:s: B. Y<:1'. It is tbc differential of x 2 \ ' , .n· 1 + constant:
14. /(.r. y) = x 2 . in y + e + C'OllSlanl.
1
- •
Answers tn ~cl1·ctt•d Prohi1•ms 1131

$1:CTTON 6.6
I. (2.x, •! 3y 3) I+ (x 2 + 9x/) J: 2. (_1· 1:11, xy •I .\'•'·)I+ (x tos xy + :n··:) j + .1_,,,.: k:
v' 12: 8. The plane 8.r + 3y + 2: == 0:
1
3. I: 4. -1/./2: S. - I/ J3: 6. L 7.
9. The curves arc onhogonal: IO. 2x I+ ::j + y k: 11. u = -(6 l - 8j + IO kl/ J200:
12. -IOOt·- .lf.i(I + J): thl· rateofdcsccat is66.8: 16. (11 + j + k)/ ./6: 17. 4x •· ::!1· + 2;: =8:
18.u=-(1-j+k),J3: 19.70.5°: 20.y-4x+2=0: 21.x+y=I.

SECflON 6.7

I. (a) 0. (h) 2 .; 2x -I J.r 2• (c) y" + 4xy. (d) lxy co.~ ry: 2. (aJ Linear. (b) Lin-
ear. (c) Nonlinl·ar. (di Nonlinear, nonlin~·ar ,t' c 1 and c2 arc rnmplcx: 3. (n) d'1/dx~.
(b) d 1dx 2 + 2.r:d/dx - (I+ .r:'.): 4. -(u 2 • b1 + UJ~ {J,X l'OS b,· cos c.::: r)
s. {3) Com-
mule. tb) Do nor commutl'. (c) Commu1e: 6. (a) -2 cm .r. (b) 6(ix -• 18/ik1 v: 7. P
• l' ? (' • ')
and Q musl comrnu1e: 10. e e(.r - I) t' ( \' - I) + :.; !.. , - I)· + ::; l,y - I)· +
1
21•(.r - l)(y - I) • · · : 11. x y Oli.ryJ ]: 12. sin -I • co~ l(x .: I) ,
sin I s-in I l .
cos l(y - I) - -.,- Ix - I)· - - .,- (Y - [)· + (cus I - sin ll(x - l)(y - I)+···:
x-' X\'1° -
JJ. X - - - -·- + ·· ·
6 2

S1'.C:."flflN 6.S
I. It has ::a maximum .iLr = 0. /Lr) i~ not differentiable au= 0: 2. f'(O) J''(O) = .1 "'IO> O. = =
/ 1J 1(0) = - 24. f(.r) h.t, a loc1l (al'111;1lly !!IPhal) maJr:imum :u .r == O; 3. Thc-n: i, a minimum
:i1.r = 1/t·. Thcr~ is a ma.-..imum 31 x =
I in 10. I I, but I.here is no maximum in the open inicn·.i]
(0. I): 4. A ,ma'(i 111u111 :tu = 1and a minimum n1 .I" = I: 5. There it. an inflcc-1ion point al
.r = O: 7. (a) ,(O. OJ. (0,. 1/ J). (1/6. 1/ 6), (b) (0. 0). ( .J2. -Jl). (- J2. /2): 8. (al (0. 0).
(5/2, 5/4 ). {b) (- 1/ 2. l 1/ ~l. ( 1. I). (-2. 4): 9. Ia) (0. 01. D - 0, no wnclusion: (0. 1/3),
D < 0. ~,tddlc point: (I 6. 1/6) . f, 0. D > 0. a mj ni mum. \b) (0. 0). f u < 0. D > 0. u 1

maximum: (Ji., - ✓2.) . .f.u >· 0, D > O. a minimum: ( - ./i. J i) . f ,, 0. D 0. a minimum:


10. ('a ) (0. 0), fu < 0. V > 0, ;i max iJlllll_ll: (S./ 2. - S/4). D - 0 , a ~addle point. (b) Cl. I).
D -. 0. a ~addle point: (-2. 4). D ~~ 0. a ~addle point: (- i/2. II l/8). /,, < 0. D > 0. a
maximum; 11. ,(::a) A saddle poin t at (0. 0). (b) A minimum at (-4. 2): 12. a) addle
point at (·0. -2) and a minimum ut (-5. 3). (b) A minimum at {-2, I): 15. (50) 3: 16. A
cube whose_volume is (A,/6) 312. Nole that V = {3 if 1\ == 61!.

S :TION 6.9
I. ..fi.: 2. 2ab: 3. &ibc/ 3 !: 4. A square: 5. (a'.! + b1 + ,'.!) l/2 R: 6. A cube:
21 fl A -' 1
7. k\/ 2/ 3(6:r ) 1 ·: S. 1T l/2 : 9. 3(2/7) 12:1
10. l/(a 2 +fr+ ,) 112 : 11. 6(2/7) 1I~:
3714
12. 2: 13. 5/-./i 14. (2 1/ D. 2. 6Jt26) ; 15. 2: 16. L'l/2) 1' 2: 17. Clt1~l·,1.
(-1 / .,/26. -J; ,/26. -4/J26J: and fanhcs1. (l /..n6. 3/ ./26. 4/ ./26): 18. ({l/11) 11

SECTION 6.10
I. ,., 21>J /24: 2. j" 112 (., - I); J. M = l/~-1. Xc n, = Y.·rn = ~/:,: 4. M = 64/ I !:19. _..t m = 3 6/5:> .
. cm =- 1/ 5: S.M- J.:ra,1 / 16 . .rcm -1'6a jtl 5;r , ycrn- 64v/45rr: 6.4/3: 7.3n: 8.85/16:
9. xL-m = a/ 4.yc:m=b/4.- n = c/4 : 10.a~/10 + h 2/ IO: ll.,rn('/24: J2.;ra 6/24;
13. (a) ;r / 2. (b) 112; 20. It· - 2) /2; 21. i(.Ji + ln( I + ✓2) J: 22. rr /2: 23. ;r 0 6 /24.

Chapter 7
SH"TION 7.1

I. (- 4 1 - 2j+k)/,/2 I: 2. (- 1 - j · 2k)/J6: 3. divA=y~ rZ.


curl A = - 2.xy i + 2.n : J + (2_,·~ - 2.x_v) k: 4. div A= 3. curl A= .r(sin xy - in x.:) I
y(sin _·• - si_n xy) J +;:( ·in x.: - in _r .::) k: IO. (a) lnw mprcssiblc, {b) ·01 incomprcssibk.
(c I v = 2x I · y j - 2.:: k rcpreseo1s iJTOtutional flow: 14. (b) y3 + x 2 y; 19. (0. O. 0):
1133
clockwise dirci:1i,m looking down thl? NOJth Pole. Bec:11.1._--e ~~ ,im:re;1...c, inn countercl od ;wi:-c
direction. /:1 = 'l60 " - IJ>: 2. u. . c Equ.ition 2 with dr =di)= 0. r a. 0 ,r / 2 . .ind = =
=
0 ::: ,J> ::S 2rr: 4. 4,, -a 5 / .15; 5. jJf x ~ d V j[jy~ d V = Jff :. 1 dV by ,pbcrirnl s}mme lry:
,
6. jlf r·tlV=3 Jli''.I" , dV
. = 4rrir~ / 5: 7. ;rr/ 3: 8.8tr/
.. L'i; 12. co it, c, - sin tp
-.-c,~:
~Jn~
211. 4n(u 2 + h 2 + c 1 )" R,n / (2n + 1)(211 + J).

S1-n1 u l'J 8.5


I. The coordinarc ·uriuci:s ure right cyli ndcrs of rndit1<. r who..c axes arc 11:!c : axis (r-surfm,i:s):
planes Cllntaining th - a:ti-.. (0-surfnccs j . and pl a'f\C,l, pcrpcndii:ular ,10 the : ;ix i.~ <:: -~11rf\tl'c-, 1.
The cnordi ru11c rnrv are straight li n •s thro11gh the ongia 1111d parnllc:l 10 the xy-plane (r-
cu n'"i.:). circles <.: c1111:red 0 11 the -~ axj 0-cul'\it'J. :md str.ti,glht lint· · pcrpe-ndiculo.r 10 the x_1·-plunc
1

(.: -i:urves); 2. The coordinate sorfuces are ... phcrical surfaces of racliu ... r centcred .it 1hc origi n
(r-s111-fan·N); tit hr circular co1w1, of ;1nglc 6' who ,. nx ·s nrr the- c ;n:i., (11-surfoc:t~ ): and planes
c,.m~ining Ihc z. a.-;i (cp -surfau.·~). The coordimnc cur.'cs urc s1r:11ghr line, 1hrough the origin
(r-curv-dJ, great ci oc lc, lyi ng in pfan conraining the ;: .uis (/t-c ur,cs ) :mdi ,l· irclcs para llel to
rhe .fJ -plam: (qi-curvc,q: 5. r 1 cos2 0 sin H c, - r~ cos 11 ._in~ U c11 + r cus fJ c~; 6. Usc the
fae1 lf1::nj x k == I: 8. r; 9. n:in 0; IS. 4rr: 16. ,r R(R + SJ.

SECTION 8.6
2. 2:r I f/ (ch/e) sin - 1
,·J. IVhcrc , = Cc - b - J1l ~/ c: 9. 2n I b1 + (cb/ d ~i n- 1 c ]. where
t' = 1/i.0: 13. h ,1 = a (co.sh 2 11 - ~i_n1 f1) I/ . = hr., , I, = o cosh 1J .sin 0 : 14. 4JT b~c} J:
rrl} I+ e , , 1,
15. 2,,-/> 2 - - In - - . whl·rc t.' = (h· - t - J • ,i/J.
e I - e

Chapter 9
SECTION 9.1
I. 5; 2. -5: 3. 0, bccnu...c two columns arc cquaJ: 0. bccousc column I = 2 column 3:
4. 5: 6. -1: 7. .r~ -- l i: c.:a 0: 0. 0. ±JI 8 ..r· 1 - 4.r- == 0: 0. 0. ± 2: 10. (a) -1. (bl -1.
(c) + 1; 15. (9i5. 1/ 5): 16. ( I. 3. -~) ; 17. The detem,inanl of the foc rors of x and y is equ=il
to 1.cro. The I\\ () equations nre' incon.,i ~teni: 20. -19: 21. - 75.

$F:('TION 'J.2
I. <13/ 12. 7 / 12 . - 7 / l '1): 2. ! 1 I. -4. 3): 3. There i.~ 11() solution: 4. There is 110 ·o lution:
=
5. There is no uOlution: 6. ( I. 2. J. 4): 7. .r I "" r2 + 3x.i · 16/ 5. X3 - 3.t.i - "!. /5: =
8. x 1 = x . x ~ - 0, x - 0. .r~ = urbimiry; 9 . .r 1 - - J. + .~x5 , x! = 3 - x 3 + . r 5. X,1 = 1 - x 5 :
IO. 1 ± i : 11. None: 12. >.. f.,. 1: J3. 4 ± J j_

::idjA

12.
=

(ll)
(

8=(
-3

-2
l
l

1
0
3
0
-n
-~ ). BA
(b) A,o

t, 0,
~ ( =l -; _15 ) I , adj A=
( .,
-I
- 2
-I
I
l
( b) No . Thi:. rcsul1ing i:qualill'! , arc im:011~i~(l•Jll:
1136 Answers lo Sel d Pr blems

14. I\ and

eJJ -
e- '

t' - 1
I)• '

Chapter 11
SE IION I I.I
2. (a) y(..r) =c:/(.r - I). (b) x 2y =nsfa. {c} = .!2c1 _ ,,-u;,,: 3. y = c 1x or
.,'(ll
J(i
y = ci/,r: 4. T(I) = 20 + '11 80 - O.IJS~ 1
(1 in minut ). T(1) = 2.5° when! = 26.5 rninu!es.
or 20·.5 mlnu1es aficr T = IOCf'C: 5 . .x(I = A b, . when! A i.~ a C'()llst;1111.
I + -;,r!''
, \,_.l
.r-> r-,
.d, J -,. a/b as 1 -• oo: 6. (u) x -v. + =--
1 = r. (hJ nor e.".acr, (cJ - · - 4x = r.
(d)xy-co x- 2·iny = c: 7.(a) ,; xy .(bl 11 =y/. .(c) = x- _v.(d) v=y/x:
8.o(l)_mg(l - ,,- u, /ml: 9.x - x~ _\' = y: IO. x ~+ r 2 =c: lt.xl - y~= 1:
a
l2. c 1(1 sinO>: 13.y~= c • 15.201, 312 61, 512 = l4 - (l.26x 10- -')r,wherc
x 2:
Ii i.; in merer. : 3.06 hours; 16. x(t) =
JO e- 0 < 1• where x 1:- in gram. an<l I is in
min11ics: 7.4 x let gram : 17. x(1) = 200(.!5 , -H5 l.-o. )SO,). where i in gram · and
, i1- in minures: 7.54 x JO~ grums: 19. l'a) Lay - .r 1/ 3y 3 (b) In y + x'J. / 2y~ c:=. =
21. y = 2:r + + 3(y - 2) la(I +x - _,,.): 22. a= 0 . f1 - 3. =
S1::<71or,. l 1.2
1137

SECTION 11.3
I. Ye.s . yes: 2. No. 1'
1
= co1.h .r + sinh x: 3. Yes : 4. (a) y (x J = c 1ei.- c 1 - . (b) y(x) =
(c 1 + 11.r)e . (c) y(x ) =e- :?.•(c 1e'13 .r c~c-=c 1e:,_,_ + cze - ~'. (bJ y(x) =
3 ) ): 5. (a) _,·(x)
(.-,, (t- 1 ~o.s ./3 x + c2 sin J3 x). (c) \'" ( r) = r 1 <c:o~ 3.r + , 2 sfo J.r: 6. (a ) y(.r) = c 1 + r2 e- 6 '.

(b) y ,(x) = c 1e-1 + <::,,·u. (c) y(.ri = c 1 cos JJ .t -c~ sin JJ .r: 7. (..ii d I i = -.r (~ +; )·
(b) \'(xl

=,,' (I - 3x). (L:) \'(XJ
.
= ,.,- ~ [(l2 + Jj 1
) eJi-' (l _.Jj
2
1
) f'- '] :

- 11
8. (aJ y(xJ = (e- </3'11,· 1 ' - I). (b) y(.r) =XL' '. = r; (e ,fj · - e- "h):
(c) y(x )
2..,-J
9. (,1) _r(,r) = 2e2.t . (b) _v(.rl = eu ( -3 + 2e-' l. (c) y( x ) = 2t·2.c: 10. y(.r) =
c 1e .r + c2e-' + c e?.r : U . y(x) = e I . c 1x + ci x 2): 12. Yi(x) = .r.?(c / x -l ) = cif:r:. 2:
1
•( 1
13. y_ (.r) - c.x In .r.

S ECTION I 1.4
I. Because ~ is part o(
1he sollJtion 10 the homogeneous cqriation: 2. y(.r) =
c 11• - -' r2e- ~ 3. _d x) = ,· 1 + c: 2 ~in .x : .r (.r occur~ in ull three solution~ to the
..\ I:
inhomogcncous cqu, ti n Q): 4. (al .l'p{ , ) - xl?"' . (b) ."p(.r) -2 - x 2: 5. ta ) Yp (x) - · + ! c- •.=
(b) Jr(x ) =-11 - 2..,· ': 6. (a ) Yp( 1 =j , (b) = 1f + !} x ~x2 + ! 3:
_1·P(.1·J
=
7. (a.) Jp(.f> - ! .r c o$."!". (b) ."pfv) ~x ex: 8. = y .r) = c 1e1 ' + c2e- )• - 1, - g:
=
9. y(x) c 1 cos 2x + c :,. .in 2x - 1 cos Jr - 4 sin J.r: 11. Use 1.hc formula in Problem I 1.3.18:
20. It j~ the ,&Vern •c .11 e :ll which L·ncrgy is di),-1,ipatcd ; 26. y (x ) = <" 1 c2 r - ~ e-' S:in X:
=
27, ) '(.r) c 1 cos .r + t·~ sin ;r - ,r co, x + sin x · l'n sin x: 28. ·{:r) -= c 1e-·' + c;x
.n,- ·' + l! - .r .r In ;r ; 29. y(xl -= c.1t·--' ~ c~.H•- ..- - ~x 2e - .r · 2{,- ., In .r. 1x
S (~rru N 11.5
I. y( .r ) = ,·1.r + c!x2 ; 2. ·(x } =
1. + cdx : 3. _\·(x)
4
1.r~ + c~x:2 In x: =
6. yu.; = x- k 1 cos!2 In x ) c 2 sin(2 fn x)I: 7. _1•(x) :;; c 1.r + c~x ln .r: 8. y(.\)
1
=
x [ ,i- 1 1.·u,(JJ" In .rl + (· 2 :,,in(/3 Jn 1') 1: 9. y(.r) = ,·11 ·• c2 •2 + c. r 3 : IO. y (.r) =
c .r + 1· .r cos( In .r). + CJ-'" s:m(. I n .x): 11 . ytr,' = r /·.r - • .r - r J/ .,:
, 12. y(x)· = c .r c,
+ ~-
1 2 I 1 I
.t -

IS. y(x) = cons-ca.n1 and y(.r) = At,.:,.,: 17. x(y) = y In . · + c 1y + c~. or y = conilllilt.
S· "JON 11.6
2. TI1ey arc linearly indepcndcn1: 3. _\·1 (.r ) =t·1e' 4cieJ'. Yi( ) = - c1e' - cie4
4. y I (x) = 1t·- r + f- ~,) .,. Y:<x ) = - 2c 1e - r 2,'.!e.!.•: 5. y 1( ) - 4c 1 - 2c2e2·1 • =
y2 (x) = 3c 1 + c e2-': 6. y 1(.r) = c 1 2.t (si n .r - cos .r) + c2 2.x (cos .r + sin .r) .
y~(.r ) = ,· 1e co. x - c _e 'sin x: 7. ;v 1/.r ) 1
=
(c2 CO:<. 2.x - c 1 sin 2x ) , •2(;r) ==
t•-' (c 1 cos 2.r + c! . i n 2x ): 8. y 1(x) = 2£· 1£'"" + c_eeu . Y: l x) = -3r 1t'"' + cielu'.:

•- y(, l = <1 (:) ~ + c, uV (.(J- 1


•:

IO. y(x) = ,, ( j) ,u +c, (-n ,fu + '-' (-D 9


c ': II. nix)= ,a+

{J) cos 2x + ({J - o-) sin 2.r · ·' . y 2 x ):::: -2a cos :!x - 2/1 ,in 2.r - ,·3e•. _, ,(x) =
1138 Answers to -I ted Pro! lc rn s

-2" co, h - 2fi ,;, h; 13. y(,) ~ ~• ( -i) ,-" + '•;o ( _;) ,-, + Ao ( ~),
14. U!l -"1 = "1• .r1 = "~- x! = "J· i: = 11,4: ii I c:::i 11 -;i. ,,, 1'i 2 = - (k, ·. k!)ll1 + k~"-'· ,i.1 = 114.

m2u.1 =
k1u1 - (k2 + kJ)U3.

Chapler 12
Sr:rT lnN 12.l

SECTION 12.2
S. At ka~t 2: 6. At ka-.;1 (20) Ii!: 7. Ar, least ✓ 5/2: 8. At ka.-;\ I; 9. x = 0 is a rcg11l;1r
singulnr point: ((J • .1 = 0 and x = I are regular l'-ingular points: 11. x = - I is an irregular
singular p(1in1: 12. x = 0 i, an irregula.r ,ingutar pt1jn1 am1 x ±i arc n:-g.uf:lr ,inguhrr p(.)ililt.s: =
13. An irrc"ufnr singular point: 1.a. A regular singuh1r poin1: IS. A regular ~ingular poim:
16. AH poinB' except .l #- 0: 17. AH x: 18. All_,: 19. All points exc~•pt x :;;;i 0: 20. All
poinLS c:<cep1 x = ±I.

S£ ON l~.3
= -
2. /.J x ) ~ 30. r 2 + J5x·' . .f t ) = 15.r - 70.r·• 6J, ~ 4. P~(.t) =
l (3 - JOx 2 + 35x4 ). P5(x) = (
~ 15.~ ~ 70.r· + 63.1. ~ ): 9. Use Equarion 8 with j
1
= 0:
15• ~ '
d-f->,,(0 )
-!- COi 0. - ; ; ; -
dC➔n (O )
11(11 -f- l)B,,<0) = o.

SECTION 12.4
-.;; n
4. cosh(2.f) 1f-/. I - and ~inh (2x) 1,.~/x 1f2 : 7. \'/.t)
.
=Uo, If .? + 111xlf l ~ ~
L_., 2"11 I
: 8. r =I
and -1/J: 9. r = 1/2 and II:!: 10. r =0 and I: 11. r = 2. """°
SEITIOl',I 12.S
13. y (.r) = t·1x 11 2 111-1(X~) + r2.r 112 Y1 d(;r 2): 14. _l'(X) = I If ~l : r 1I rr'l-1) +
CJ. I/ Y11 ( ~ x V 4): 15. y(x) = ,· 1x - 2h (x ) c 1 x- 2 Y2 1 · ) : 16. v( r) = c 1s tn J 1 ( ~o x-l/ 2 ) +
1
1"_.\'
112 Y1 0 ( 2 ox.l 12 ): 22. /r + l)(r + 2) • • • (r + n) ( - - + _ l_ +... - 1- )
r + I r+2 r 11 •
Answers lu. Plt"CIPd P.rollf •11,s 1139

SECTIOl'l 12.6

I. Mult;ply by x' and use Equ,r;nn 12.5.42: 8. 1,;, = ( ~ )'" [ '11 - · ; ' + 0( , 91,,J.

l\1' = ( 2) •I- [ x-lf 1 - x 1I ':. O(x 11121]. l s/2 = ( ~) I/ ~ [x:5/ 1 - x9!- + O (x llf .)]:
. - ;r . JO . . ;r I5 21 0
I 0, Let r e' 11 in fa1ua1ion 22; 11. Use thl! fact thla'f l;"' c11:- 118 cos 111('
= = ,t,m,rr; 12. Use 1hc
=
fac:t that J;~. in,, .. in m H Sm,,,r: 16. Lei a= 0 and b - I.

Chapter 13
SECTIO:-. 13. l
6, (a) i- = y,
_i· = =
--yy - kx - fJxJ. (b) .i· y. _\' = E_1•( I !y·J - x. (l·) .i- c:: y .
= =
.-i· =fr I · .r: Jy - 1. (d) .r y . _,. -yy - w: sin x; 7. The 1rujl>clonc\ mo\c ,, ith a
con,tant s~cd in the pha~~- plane: 10. (u) (0. 0) and (4. 0). (b) (0. UJ ;ind (). 2J. (c) (0. 0) and
=
(2. ~ ). (d) A II lht: point:- on rhe line .r 2y; 11. (n) (0. 0). (0. 3). (5. 0 ). and C9. -3). ( b) (0. 0).

(0. ;), Cl. Ol. nnd ( I R. --20}; 12. ,•(I)=,.-, (a: {l) cu~ '21 + t,-, ( a-/) sin 2r:
13. v(r) =(a f.l '2/J) cos,+ ( -laa - fj) sin r: 14. (a) Circlel- about the origin. tb) Spirul

inro the ori_!!in: 20. ~ r ~ • IJ1c cif:envecror (I. w) T dominarcs. as r __,, - • the cigt:m·ccior
( I. - ,L,) 1 dominate~: 21. Wthcn Q > 0. 0 incrc . , · with rime; when n < 0, 0 dc:crcasc!> wit.h
ritnc-

Sn:-rm.-.: 113.2
3. A trajectory could go off in two diffcrenr <liJt-clion.s at ru1 interst:"i:tion; 4. (::i) ( I. I).
(b) (I. I). (-1. I), 1c) (,O. 0). (1. 2). (0. 4). (1/:l. 0). (d) <-1. 1). 0. I): S. (:1) (I. I),
=
1i -u - r: - u1•. i = 11 + 11 v. (b) (I. I). ,i = -1,. i• 2u - 2t• 11 2 - 2: (- 1, I}. =
i.,= -L•. l' =
- 2u - 2tt + 11 2 - w , (c) (0. 0). ,i Il l ="
u1•, i• = 4t1 - 112 2,n·:
(l,2),,i = - 311 + - 311 2 11 1.i = - -111 2r - 211v - L12 :(0.4).,i=511-Ju!+,w.
=
i• -4v - t, 2 - 211t·; 11 /3. 0). 1i -11 11 - J,,z uv, i• ~ 11 - v2 - 211t1. (d) (I. I).
= =
,i = 11 - 11, i.1 = 211 + ,, 2; (-1. -1). ,; = 11 - 11. i, = -211 + 2
11 : 6. (a) (I. I).(-: -,~).
l_b) (LI). ( ~ =~ ); (-1. I). ( -~ =D· (c) (0. 0). ( ~ ~} d. 2). ( =~ -D:
(0 . 4 ). ( 05 0) .; ( I/".l.
-4 0 ). (' - 01 IO/J
il /J) ' (d 1 ( I • I ) • ( 2I -1).
0 . ( - I . - 1). ( -2I - 0I) ..

13. A (:stuhle) pr.oprr node: 14. An (asymptotically ,t;1hkl improper node: 17. (a) Saddle
poinl. (h) Node. (c) Spiral ,pninl.

SLCi l UN I 3,.3
I. A ~table proper node: 2. An a,ymptoricaJly stable improper node: 3. /\ !-llddlc point
(un. tahlcl: 4. An :isymptoticall ~ srablc node: S. A ,_addle poim (unsrablc): 6. An
asymp101ically stable i.pirul poin!; 7. ( -2. -1). a cenrer (srabk): 8. ( ii. 1). an ,tsympmtically
srablc spir.il puim; 9. (2. - I}, .a s;1J'dk ~,inr (unstable): 10. (-1. -1). an a.sym ptotic,uly
srabl'c node ; 11. >.. .::: = a ± i. a c.:cnrcr if a = 0. a srablc s-pirj) point if n < 0. and an unsrablc
spiral point if a O: 12. ,;, _ = - I ± ia 112; An asympro1ic:1lly s1ablc node if" = 0. lln
asymptoticaJly rnble spiral point if a > 0. an a.s.ymptotically stable node if - I - a < 0. and
a .--addle point if a < -1: 13. (0 . ()), .1 cenre f 1..1..able): (- ,I 0. I 0). a ~addle point (un~tahle):
14. (0. OL a ,iJddk poinr fonsrnble}: (3, 21. :1 t:1;n1cr (Stuble); 15. (2. I). a ~addle poinr
(un,tahlcl; l - 2. - 1). uo ,1.~ymp101k,d ly ~table spiral poinl; 16. (0, 0). ,Ill unstable node:
( -1. I). a ...addle point (1rns1ablc): 17. ( I. I), an a,ymptorically stable spiral point.
1140 AnswCl'S lo c;,,.ll'I h•n Prnhl1•111~

SECTION 13.4
I. V(x) gocii truough a maxi.mum ar x = I: 6. The poinr (4, 0) is a critical point
, y! .'(:: r1 8 . . .
8. 1lle one w11h -
.,
. - - -
1
=-
12 1
= ,,
IO. x
.
= 0 along 1he x ax,s. l11erctore .
d,·
-=- = .i-7,. ;:::;;J -ix,'~ - -•
- - - , . oo Ul'l!ess
f "." . y) O w h'1ch II. doesn ' I unless (.r, y l .,s a cn11cn
=. .. I
dx v
poinr; · 11. (0. ·o) is a centcr: both (:i:3. 0) are saddle poinL-.: 12. The potential energy has
a ma.umum 1.11 x ·= ± J: 13. (0. 0) ·is a saddle point anti (±2. 0) are ccn1crs: 14. (0. 0)
is a, sadd le poin1 and (± 1. 0) nru ccnlcf:'i; 15. (0. 0) is a saddle poinr :1J1d (±1. 0) arc
cent.er~: 16. (0. 0) h, a ,addl'e point. (2. l'.\38. 0) is a cemer. wid (-0.332755. 0) is a ccnter:
18. {n) F(x) = l( {.t 3 - .r) and G (x) = {t~. There i~ a limit cycle. (bJ F(.t) - €( ~x.1 - .r) and
G(x) = fr Z + i·~J. There is a limit cydc. ·

SECTION 13.5
3. ,i = =
-brn/d and u ad11/b: 9. E3eh ~pccies goes almost 10 cxt..inction : 11. (50. 12.5)
is a critical point. The number of each ,pecic~ 0~1.:illatc~ ~lightly about iL, 1:rilicaJ value:
13. (0, 0) 11nd (4, 0) are !,.3ddle ,poiAt~ and (.~. 2) i~ an asymp101ically stable ~piraJ point..:
14. TI,c (criticul poinl ( 190. 2-") is an asympt(llil'ally stabk spiral point and (0, 5/4) i,
1

a s::iddle point ; is. y survi cs nndi .r become.~ cxlinc1: 16. y survive', and x b<;.,:(lme~
ci tinct.: 17. Both spcc-i c., , urvivc. They coexist: 18. The plrnse r,onrni1 is given by
O.~OO(X, Y) - 0. 100 In X - In Y =1.00; 20. When a < 0, the 1wo cri1ical poinL<: arc
,l),ymp101j1..~at,Jy ,1able. bul when 11 > O. they are asymptoti1:ally um,1able. The ,-,1lue {3 = 24.74 is.
the bonk.-rlinc lxtwL-cn 11 normal sy'.:Ucm and l.l chaotic sy~1crn .

Chapter 14
S ~CII OI'-' 14.1
S. P,1(.l') ='- lj x2 j x"': 7. If 1he Legendre polynomials arc orthogonal. then
only the n = m 1cnn1,. in the in1c-gral of tJ,e prcxlui.·t of G (x. , ) and G (x. 11) nre nonu:ro:
8. I= (111)-I/.? ln H I+ (ru) 112 V1I I - (lu,) 112 ]1. ; 9. G(L f) = (I - n- 1 = I+,+ 11 + · · · ;
IO. G(-1, t) = ,( I + 1)- 1 = I - t.;.. 1 2 - ; 3 + · · · : 14. Use tbc r~sult of Probl~m 2
and Equation 20: IS. U.,;e Equ.alions 9 and 20: 18. Tne S,Cquc:nce of p,u:tio.l sums
in Equal!ion 25 is mono1onically incre:.., ing and bounded 'by 1he left s'idc . Thus,
the ~cri~ converges. and w 12/(2.,, ·fa lll 11~a,, mLL~l go 10 zero :L'- n incrases :
, 3 7(rt 1 - 15) 11(945 - 105rt :! + ;r J)
19. ~mn.t=-P1(.r) + , P3(x)+ P~(x)+·· · :
;r rt · rr 1
=
20. I - .t = j - j /'1 (x) : 21. u I when x = {J :irnd u = - I wheo x = er:
2

22. /(x) =
j - j P2(.r/2) .

SECT101'1 14.2
=
I. tJ>i1 (.r) 11 - lJJ. 4> 1(x) = (4/rr) 1/J .r. ~ 1 (x) = (4.'1' ) - I/ J( lx i - I): 3. .1; 6.r J - 48.r 2 + 12:
1

IO. ,tJ.,i: 1he recur-.ion fomiula in Tllble 14.2: 11. U se tbe recun.ion formula in Table 14,2
succt.-s. ively 10 get x 2H,,(.r) in lenns of H,.+ 2(x). H,J.r). and H,._~(.r).

SECTION 14.3
1

7. - l)'!T./2/. y,,(x) = sin(l,1 - l)rrx/21. ,1 = I. 2 .... : 8. >.,. = (2n - ll/2 .


>.n = (211
.v,,(x)=co::;(2,, - 'l).r /2. 11 = l. 2, . .. : 9. .i.,, = nrr. y,,(x) = co, mr .r . 11 = I. 2 .... ;
IO. If tnnh .r =.r / al has a solution. or if al > I: 11. p(.r ) = r (x) = e-x ' and q (:r) = 0:
12. p(x) = ( I - .r 1 ) 1f!. 11(x) = 0. and r(x ) = (I - x 2 )- 1i2; JIJ. Use Equ:nion 12 wi1h
r(.r)= e - •·: l4.UseEqua1ionl2wi1hr(.r)=(I .r ;i} I.'~ 15.(n,)p(:r}= .r .::::. Owhen
x= 0. Singular. (b) p(x) = (I -.r 2 ) = 0 when x = ± I. Singulur. (c) p (.r) = (I .r;i),111 = 0
when .r =±I. Singulnr. 16. A= 0: Yol.r) = com1an1. i. i:- 0: 1. = ,m fa . 11 = I. 2 . ... .
Ynlx) =cosnrr.r:/a andsinmrx/a.

You might also like